masterclass book part 2

367
CONTENTS 1. CARDIOLOGY 1 2. RESPIRATORY 53 3. NEPHROLOGY 95 4. NEUROLOGY 120 5. GIT 156 6. RHEUMATOLOGY 202 7. HAEMATOLOGY 230 8. ONCOLOGY 246 9. ENDOCRINOLOGY 264 10. INFECTIOUS DIASEASES 287 11. EMERGENCY MEDICINE 301 12. DERMATOLOGY 310 13. PSYCHIATRY 324 14. OPTHALMOLOGY 333 15. PHARMA/TOXOCOLOGY 343 16. STATISTICS 361 17. PLATES 367

Upload: eng-kian-ng

Post on 20-Oct-2015

688 views

Category:

Documents


10 download

DESCRIPTION

MRCP Paer 2

TRANSCRIPT

Page 1: Masterclass Book Part 2

CONTENTS

1. CARDIOLOGY 1 2. RESPIRATORY 53 3. NEPHROLOGY 95 4. NEUROLOGY 120 5. GIT 156 6. RHEUMATOLOGY 202 7. HAEMATOLOGY 230 8. ONCOLOGY 246 9. ENDOCRINOLOGY 264 10. INFECTIOUS DIASEASES 287 11. EMERGENCY MEDICINE 301 12. DERMATOLOGY 310 13. PSYCHIATRY 324 14. OPTHALMOLOGY 333 15. PHARMA/TOXOCOLOGY 343 16. STATISTICS 361 17. PLATES 367

Page 2: Masterclass Book Part 2

MOHAMMED IS-HAG 1

1. A 46-year-old woman with permanent atrial fibrillation is experiencing rapid palpitations due to an elevated heart rate. Which from the following list of drugs are the three most appropriate for controlling ventricular rate response? A : Adenosine B : Amiodarone C : Aspirin D : Atenolol E : Digoxin F : Doxasocin G : Flecainide H : Isoprenaline I : Lidocaine J : Magnesium K : Mexilitine L : Moxonidine M : Propafenone N : Verapamil O : Warfarin. Comment : Beta-blockers, cardiac glycosides and calcium channel antagonists all have a negative chronotropic effect on the atrio ventricular (AV) node and are very useful in the acute and long-term control of ventricular rate in AF. Digoxin and verapamil however should be avoided in the rare patients who also have Wolf-Parkinson-White syndrome. While Amiodarone, Flecainide and Propafenone are commonly used to chemically cardiovert AF to sinus rhythm, they have only a small effect on AV nodal conduction and should not be chosen for rate control alone. Adenosine causes rapid but only very brief (a few seconds) blockade of AV nodal conduction. Lidocaine, Mexilitine and Magnesium are used only for the control of ventricular arrhythmias. Warfarin and Aspirin are commonly used in patients with AF for preventing thromboembolic complications. Isoprenaline is a beta agonist and would accelerate ventricular response in AF.Doxazosin and Moxonidine are antihypertensive agents. D : E : N: 2.

A 19-year-old man presents to accident and emergency with a fast pounding in his throat and presyncope. He is found to be tachycardic and his ECG is as shown. What is the most likely arrhythmia? A : Atrial fibrillation (AF) B : Ventricular tachycardia (VT) C : Atrial tachycardia D : Atrio-ventricular nodal re-entry tachycardia (AVNRT) E : Atrial flutter. Comment : The ECG shows a regular narrow complex tachycardia at a rate of 170. This excludes VT (broad complex)and AF(irregular).Although atrial flutter and atrial tachycardia are possible diagnoses there are no visible p waves,which would be expected. The most likely diagnosis by far is AVNRT,which is caused by a re-entry circuit formed within the AV node causing atria and ventricles to be activated simultaneously.This can cause reflux of jugular venous blood and a sensation of pounding in the throat. D:

Page 3: Masterclass Book Part 2

MOHAMMED IS-HAG 2

3. A 65-year-old lady is admitted with left sided pneumonia and pleural effusion. Pleural fluid is aspirated and sent for tests. Which of the following is an indication for inserting a chest drain? A : Pleural fluid pH <7.2 B : Serous pleural fluid C : Blood stained pleural fluid D : Pleural fluid glucose >2mmol/l E : Pleural fluid lactate dehydrogenase > 200IU/l. Comment : Infected pleural effusions should be drained. Infected pleural effusions should be drained if the pH<7.2, Gram stain shows organisms, the fluid is frankly purulent and clinical improvement is slow despite antibiotics. A : 4.

This is an angiogram from a 65-year-old man with limiting angina. He currently takes aspirin. The most appropriate treatment option is: A : long acting oral nitrate B : percutaneous coronary angioplasty with stenting C : coronary artery bypass grafting D : oral beta blocker E : intravenous GIIb/IIa receptor antagonist. Comment : The angiogram demonstrates significant left main stem (LMS) and left anterior descending artery (LAD) stenoses. Coronary artery bypass grafting has been shown to improve survival in patients with left main stem stenosis (> 60%), triple vessel disease and impaired left ventricular function and triple vessel disease with proximal left anterior descending artery disease, when compared to medical therapy. Angioplasty is exceedingly high risk and therefore is only contemplated when patients are deemed as non-operative candidates. C : 5. A 68-year-old female is referred to outpatients with a three-month history of dyspnoea and significant peripheral oedema. Clinical findings are solely of the oedema. Echocardiography demonstrates tickened myocardium with impaired relaxation and a bright speckled appearance. What is the likely diagnosis? A : Hypertrophic cardiomyopathy B : Cor pulmonale C : Sarcoidosis D : Amyloidosis E : Secondary neoplastic myocardial deposition. Comment : The commonest cause of death in amyloidosis is secondary to cardiac involvement. It may present with the insidious onset of vague symptoms, such as lethargy. Later peripheral oedema becomes a prominent feature.Amyloid depositis in the heart produce generalized thickening of the myocardium (as opposed to asymmetrical septal hypertrophy commonly seen in hypertrophic

Page 4: Masterclass Book Part 2

MOHAMMED IS-HAG 3

cardiomyopathy) and diastolic dysfunction. This produces a "stiffened" appearance to relaxation in the diastolic phase.A search for non-cardiac amyloid deposits is usually the most efficient way to coinfirm the diagnosis histologically. Cardiac biopsy may be useful. The treatment of cardiac amyloid is supportive and generally it carries a very poor prognosis. D : 6.

A 75-year-old woman presents with palpitations. Her ECG shows: A : Atrial fibrillation B : Atrial fibrillation with acute inferior myocardial infarction C : Atrial flutter D : Atrial flutter with acute inferior myocardial infarction E : Atrioventricular re-entrant tachycardia. Comment : The ECG shows atrial flutter with 4/1 block. There are no features to suggest myocardial infarction. C : 7. Which of the following statements regarding the natriuretic peptides ANP (atrial natriuretic peptide) and BNP (brain natriuretic peptide) is not correct? A : They are both vasodilators. B : BNP is produced by the ventricular myocardium. C : They inhibit sodium reabsorption in the collecting duct of the kidney. D : They stimulate aldosterone production in the adrenal glomerulosa. E : ANP is released in response to atrial stretch. Comment : ANP and BNP are members of the natriuretic peptide family. As its name suggests, ANP is primarily released from the atria in response to enhanced stretch. Although first discovered in porcine brain, BNP is found in large quantities in ventricular myocardium. Release of BNP is thought to occur in response to increased ventricular wall stress. ANP and BNP have similar biological properties, which include vasodilatation, natriuresis and diuresis. ANP and BNP have direct effects on renal function. These include increasing glomerular filtration rate as a consequence of efferent glomerular arteriolar vasoconstriction and afferent vasodilatation. In addition, they directly inhibit sodium reabsorption in the collecting tubules. Both ANP and BNP also inhibit renin and aldosterone release. Plasma ANP and BNP levels are elevated in patients with chronic heart failure and indeed levels may be used to aid in the diagnosis of heart failure. Furthermore, plasma ANP and BNP are helpful in providing prognostic assessment of patients with heart failure and post myocardial infarction. In view of the potential beneficial haemodynamic effects of ANP and BNP, there is current interest in the therapeutic manipulation of the natriuretic peptide system in patients with heart failure. This could be achieved by inhibiting their breakdown (neutral endopeptidase inhibitors) or infusing exogenous synthetic peptide. D :

Page 5: Masterclass Book Part 2

MOHAMMED IS-HAG 4

8. A 42-year-old man reviewed in outpatients has a 6-month history of increasing shortness of breath on exertion and feelings of lightheadedness when digging in his garden. His general practitioner organised an open access echocardiogram which showed a septal thickness of 26mm and a left ventricular outflow gradient of 85mmHg. Which of the following is NOT a risk factor for sudden death in patients with hypertrophic cardiomyopathy? A : Unexplained syncope B : Sudden death from HOCM in 2 or more first degree relatives <40years of age C : Family history of sudden death D : left ventricular wall thickness of >30mm E : Hypertension. Comment : Major risk factors for hypertrophic cardiomyopathy are: · Cardiac arrest (ventricular fibrillation) · Spontaneous sustained ventricular tachycardia · Family history of sudden death · Minor risk factors · Unexplained syncope · Left ventricualr wall thickness>30mm · Abnormal blood pressure on exercise(failure to rise from baseline by 25mmHg) · Non sustained Ventricular tachycardia · Left ventricular outflow obstruction · Microvascular obstruction · High risk genetic defect. E : 9. A 42-year-old man visits his GP, complaining of a recent onset of shortness of breath and dizziness upon exertion. His GP arranges for an echocardiogram, which showed hypertrophic obstructive cardiomyopathy (HOCM) with thickening of the septal wall and a left ventricular outflow gradient of 86mmHg. The man is referred to a cardiology outpatient clinic, where he volunteers that his father died suddenly of a heart attack aged 38 years of age and his younger brother died aged 17 years of age during a rugby scrum more than 20 years ago. He is not sure of the cause of death for his younger brother but does not think a post-mortem was ever carried out. He has two teenaged sons, both of whom are well, but his 14-year-old son fainted after scoring a goal for the school team last week. Which of the following statements are correct? A : The majority of patients with HOCM are symptomatic throughout life. B : The overall mortality in patients with HOCM is 50% per year. C : All first degree family members should undergo screening every 6 months. D : All patients should receive advice on avoiding dehydration and strenous exercise. E : Patients with HOCM do not need prophylaxis against infective endocarditis. Comment : · The majority of patients with HOCM are ASYMPTOMATIC. · The overall mortality in patients with HOCM is <1% per year. · All first degree family members should undergo periodic screening with echocardigraphy every 5 years for this autosomal dominant disorder since hypertrophy may not occur until 6th or 7th decade of life. Annual screening is recommended for adolescents 12 to 18 years of age. · All patients should be advised to avoid dehydration and strenuous exercise. · All patients with HOCM should be advised to use prophylaxis against infective endocarditis. D : 10. You are asked to speak to a 58 yr old man and his wife on the ward. He has been admitted for routine surgery (cholecystectomy) and when clerked says that he worked in shipyards for many years. The house surgeon enquires about asbestos exposure, and this leads to a great deal of anxiety. His wife is very keen to know what medical risks this poses for him. Which one of the following statements regarding asbestos exposure or asbestos related disease is true? A : pleural plaques are often associated with restrictive defect in respiratory function tests. B : the risk of mesothelioma increases with smoking. C : the risk of lung cancer increases by 50-fold in asbestos workers who smoked compared to

persons who neither smoked nor worked with asbestos. D : chemotherapy is the treatment of choice in patients with mesothelioma. E : adenocarcinoma is the predominant histological type in asbestos workers with lung cancer. Comment : Asbestos exposure is associetd with: · pleural plaques · diffuse pleural thickening · lung fibrosis (also known as asbestosis) · lung cancer · mesothelioma. The commonest abnormality seen in asbestos workers is pleural plaques, which are well-circumscribed areas of thickening affecting the parietal pleura. Pleural plaques are even seen in patients with mild

Page 6: Masterclass Book Part 2

MOHAMMED IS-HAG 5

asbestos exposure. In the majority of patients, these plaques are asymptomatic and do not cause any changes in respiratory function tests. Mesothelioma is a tumour of the mesothlial cells or the pro-mesothelial cells. It affects the pleura and, to a lesser extent, the pericardium and the peritoneum. The incidence of mesothelioma increases in patients with heavy asbestos exposure and in those who were exposed to asbestos at a young age. Cigarette smoking does not increase the incidence of mesothelioma. Mesothelioma is a fatal disease. The majority of patients die within 18 months of diagnosis. The disease is resistant to current treatment modalities such as chemotherapy and radiotherapy. Tumor resection is indicated in only a small proportion of patients. Lung cancer is associated with heavy asbestos exposure. The effects of asbestos exposure and smoking are multiplicative. The risk of lung cancer in asbestos workers who never smoke is increased by 5-fold. In smokers the risk is about 55-fold that in non-smoking persons who never worked with asbestos. C : 11.

What does this angiogram show? A : Left internal mammary artery graft B : Subclavian stenosis C : Normal coronary artery D : Patent ductus arteriosus E : Coronary artery vein graft to the left anterior descending artery. Comment : This angiogram demonstrates a left internal mammary arterial (LIMA) graft to the left anterior descending artery. Sternal wires can clearly be seen along with clips to occlude branches of the LIMA. The artery is cannulated via the left subclavian artery. In the majority of cases arterial grafts are preferred to saphenous vein grafts as their longevity is generally greater. Other arterial grafts commonly used are the right internal mammary artery (RIMA) and radial artery. A : 12. A 78-year-old man presents with an acute confusional state. He has postural hypotension and is dehydrated, with serum calcium 3.41 mmol/l. Which is the most appropriate initial treatment? A : Intravenous sodium pamidronate B : Intravenous 5% dextrose C : Oral prednisolone (20-60 mg) D : Intravenous frusemide (40 mg) E : Intravenous 0.9% sodium chloride. Comment : The initial priority is to restore intravascular volume by giving 0.9% sodium chloride intravenously. When this has been done, frusemide can be used to increase calcium excretion and sodium pamidronate can be used to effect rapid reduction in serum calcium concentration. The commonest causes of hypercalcaemia in an elderly man would be metastatic malignancy, myeloma and primary hyperparathyroidism. E :

Page 7: Masterclass Book Part 2

MOHAMMED IS-HAG 6

13.

A 45-year-old female is admitted with chest pain. She has previously been fit and well, apart from a road traffic accident 4 years ago. Her radiograph shows which of the following? A : It is normal B : Dextrocardia C : Tension pneumothorax D : Hiatus hernia E : None of the above. Comment : This lady had traumatic rupture of her diaphragm when she had her RTA, leading to massive herniation of her large bowel into her left hemidiaphragm. Her chest pain was related to gastrooesophageal reflux. E : 14. A 58-year-old female is admitted acutely with pulmonary oedema and hypotension. She had a mitral valve replacement four years previously for mitral regurgitation. Clinical examination reveals a diastolic murmur and pulmonary oedema. Her biochemistry and full blood count is normal. Her INR is 1.8. Echocardiography demonstrates thrombus around the valve. What is the best treatment? A : Intravenous heparin B : Thrombolysis C : Urgent surgical intervention D : Increased dose of warfarin E : Low molecular weight heparin. Comment : Mechanical mitral valves are more prone to thrombosis than aortic valves if the INR drops below the therapeutic level. This is a medical/surgical emergency. It is rapidly fatal without immediate surgical intervention. Thrombus in this context cannot be treated with anticoagulants/fibrinolytics. Anticoagulation is clearly an important consideration when considering what type of valve prosthesis should be considered for an individual. C : 15. A 66-year-old gentleman with known hypertension underwent coronary artery bypass grafting. Twelve hours later he becomes hypotensive despite inotropic support. What is the next most appropriate investigation? A : Urgent cardiac catheterization B : Re-exploration of surgery C : Transoesophageal echocardiogram D : Ventilation perfusion scan E : Serum potassium measurement. Comment : The most likely explanation of this gentleman’s deterioration is cardiac tamponade. Although trans-thoracic echocardiogram would be of help it does not exclude the diagnosis. Pericardial effusions, particularly in the early post-operative period, can be localized and difficult to identify by trans-thoracic echocardiography. An effusion may only occur posteriorly and therefore the best way to visualize these would be through transoesophageal echo. C : 16. A 60-year-old gentleman is being investigated for chest pain and undergoes exercise tolerance testing. Which feature is associated with a worse prognosis? A : Increased metabolic rate B : Ventricular tachycardia C : Increase in blood pressure with exercise D : Rapid resolution of heart rate in recovery E : Absence of symptoms during exercise.

Page 8: Masterclass Book Part 2

MOHAMMED IS-HAG 7

Comment : Exercise testing has long been an established method for identifying patients with underlying coronary disease. Apart from changes in the ST segments other features have been associated with underlying disease, including: · ventricular arrhythmias · inadequate blood pressure response · inadequate heart rate response · angina · poor MET response. B : 17. The following measurements were made from a cardiac catheterisation of a 28-year-old gentleman with palpitations and breathlessness: RA 6mmHg RV 22/6 PA 30/10 PAW 28 LV 128/10 What is the most appropriate next investigation? A : Exercise treadmill testing B : Trans-oesophageal echocardiography C : Cardiac MRI D : 24-hour tape E : Coronary angiography. Comment : The result of the catheterisation demonstrates a significant gradient across the mitral valve (PAW = pulmonary artery wedge, reflecting left atrial pressure)indicating mitral stenosis. The next investigation would be trans thoracic echocardiography to assess the suitability of the mitral valve for balloon valvuloplasty. General contra-indications to percutaneous balloon valvuloplasty are: significant mitral regurgitation, calcification of mitral valver leaflets, left atrial thrombus. All of these can be assessed by trans oesophageal echo. B : 18. A 63-year-old male with Marfan's syndrome presents with chest pain and is found to have an acute aortic dissection. Which two of the following cardiac conditions are associated with Marfan's? A : Atrial myxoma B : Mitral stenosis C : Pulmonary regurgitation D : Ventricular tachycardia E : Atrial septal defect F : Ventricular septal defect G : Cardiac amyloid H : Sarcoidosis I : Mitral valve prolapse J : Aortic stenosis. Comment : Cardiac complications of Marfan’s are relatively common. Usually they are related to aortic involvement. However, Marfan’s is associated with the following: · Mitral valve prolapse and regurgitation · Left ventricular dilatation and cardiac failure · Pulmonary artery dilatation · Regurgitation of the pulmonary valve. C : I: 19. A 45-year-old lady is seen in cardiac outpatients with a blood pressure of 160/80. Consistently it has been greater than 145>85. However ambulatory monitoring demonstrated a mean BP of 130/75. Which of the following are inappropriate indications for ambulatory blood pressure monitoring? A : Deciding diagnosis in borderline hypertension B : Making a diagnosis of left ventricular failure C : Identification of nocturnal hypertension D : Diagnosing Phaeochromocytoma E : Hypertensive patients resistant to treatment F : Hypertension of pregnancy G : Diagnosis of hypotension H : Monitoring hypertensive tretament I : Elderly patients J : Exclusion of white coat hypertension. Comment : A diagnosis of "white coat hypertension" was made in this lady and thus she did not require antihypertensive treatment. This is probably the strongest indication for ambulatory BP monitoring but the indications are broad. A diagnosis of left ventricular failure will not be made on BP alone. Equally a phaeochromocytoma may have transient rises in BP but as most ambulatory monitors will only measure once or twice an hour it is unlikely to be helpful in this situation B : D :

Page 9: Masterclass Book Part 2

MOHAMMED IS-HAG 8

20. A 40-year-old woman was referred for investigation of shortness of breath on effort over the preceding 1 year. Data from her cardiac catheterisation iare as follows: Pressure (mmHg) Oxygen saturation (%) SVC 15 64 IVC 13 65 RA 13 82 RV 51/7 85 PA 53/31 83 LV 134/3 99 Aorta 135/82 99 Which of the following are true concerning her diagnosis? A : This condition rarely presents in adulthood. B : Paradoxical splitting of the second sound is common. C : A rumbling mid diastolic murmur may occur. D : A systolic murmur is rare. E : The ECG often shows left bundle branch block. F : The chest radiograph is usually normal. G : Echocardiography shows left ventricular dilatation. H : Medical treatment is recommended in this case. I : Arrhythmias are rare with this condition. J : In children, this condition may be associated with skeletal abnormalities. Comment : This patient has a secundum atrial septal defect as shown by a step-up in oxygen saturations at atrial level. She also has pulmonary hypertension. Characteristic findings on auscultation include a wide fixed-split second sound, ejection systolic murmur and a mid diastolic murmur. The murmurs are caused by increased flow across the pulmonary and tricuspid valves respectively. The ECG often shows a right bundle branch block pattern and the chest radiograph shows right ventricular enlargement and prominent pulmonary arteries. Surgical or percutaneous closure is recommended in patients with symptoms, right ventricular dilatation, or a pulmonary: systemic flow ratio > 1.5. The Holt-Oram syndrome is the association of an atrial septal defect with bony abnormalities of the extremities. C : J: 21. A 57-year-old diabetic man with stable angina wishes to use Sildenafil (Viagra) to treat impotency. Concomitant use of which two of the following cardiac medications is contraindicated? A : Aspirin B : Clopidogrel C : Atenolol D : Amlodipine E : Digoxin F : Isosorbide mononitrate G : Lisinopril H : Nicorandil I : Simvastatin J : Warfarin. Comment : Sildenafil enhances Nitric Oxide (NO) mediated smooth muscle relaxation by blocking the degradation of cGMP. Although it is relatively selective to the corpus cavernosum it does still have some systemic effect. Both Isosorbide Mononitrate and Nicorandil release NO and their action is potentiated by Sildenafil, which can lead to profound hypotension. The other commonly used cardiac medications listed have no specific interaction with Sildenafil. F:H: 22. A 56-year-old man develops cardiogenic shock 8 hours after admission with an anterior myocardial infarction. He was thrombolysed with tPA on admission with rapid resolution of the ST elevation. He has a loud pansystolic murmur which is new. Which of the following are inappropriate in the management of this patient? A : Repeat thrombolysis B : Coronary angiography C : Intra aortic balloon pump D : Intravenous beta-blockers E : Inotropic support F : Urgent echocardiography G : Swann–Ganz pulmonary artery catheter H : Intravenous diamorphine I : Transfer to cardiothoracic surgical centre J : Consider positive pressure ventilation.

Page 10: Masterclass Book Part 2

MOHAMMED IS-HAG 9

Comment : Increasing revascularisation both with coronary artery bypass grafting (CABG) and percutaneous coronary intervention (PCI) has seen reductions in the number of deaths following acute myocardial infarction. However, shock remains the leading cause of death in hospital. It is clear that this patient has most likely developed a post MI ventricular septal defect or acute mitral regurgitation. In the absence of new ECG changes, thrombolysis is clearly contraindicated. The use of intravenous beta-blockers in the shocked patient is also relatively contraindicated. The mainstay of treatment is therefore supportive and surgical intervention, if appropriate. A : D : 23. A 65-year-old man presents with increasing breathlessness and swelling of his legs. These symptoms have occurred over a period of 2 months. He has otherwise been previously well, apart from a 2-year history of diarrhoea which his gastroenterologists have been investigating. On echo, he is found to have severe tricuspid regurgitation and a diagnosis of carcinoid syndrome is made. Which of the following statements about carcinoid syndrome is NOT true? A : Common sites of carcinoid tumours are the appendix and terminal ileum. B : After the tricuspid valve, the mitral valve is most commonly affected. C : Patients with cardiac carcinoid ususally die from valvular disease rather than carcinomatosis. D : Patients with cardiac involvement usually have more advanced disease. E : Patients with cardiac involvement usually have higher levels of 5-HIAA. Comment : Carcinoid heart disease is rare and usually affects the right-sided valves (i.e. tricuspid and pulmonary valves). Cardiac involvement is associated with more advanced disease and carries with it a poorer prognosis. It is associated with higher circulating levels of 5-HIAA. Most patients present with right-sided heart failure. Treatment of cardiac carcinoid involves management of the right-sided valve failure, pharmacotherapy to reduce secretion of tumour products and surgical intervention for the valvular pathology. B : 24. A 21-year-old university student complains of difficulty sleeping. She is in the middle of sitting her final exams and would like some medication for a few days to help her sleep. However, she is concerned about potential 'hang-over' effects and would prefer a drug which doesn't cause daytime drowsiness. Which agent would you prescribe? A : Diazepam B : Midazolam C : Promethazine D : Loprazolam E : Clomethiazole. Comment : Diazepam has a long half-life, principally because of its active metabolites. Midazolam is short-acting but is only used intravenously. Promethazine is an antihistamine with a 12-hour half-life and may cause daytime sedation. Clomethiazole is less safe in overdose, has dependence potential and is only licensed for sedation in the elderly. Loprazolam is short-acting (half-life 6–12 hours) and would be a reasonable choice. D : 25. A 68-year-old man with a past history of myocardial infarction, from which he made a good recovery, now presents with chest pain that might be due to cardiac ischaemia, but the history is not typical. He would not be able to perform a Bruce protocol treadmill test because of severe osteoarthritis of his knees and it is decided to perform radionucleotide myocardial perfusion imaging. Which two of the following statements about such imaging are correct? A : It should be avoided in asthmatic patients. B : It should be avoided in patients with aortic stenosis. C : It is useful in determining the functional severity of coronary lesions but does not predict

prognosis. D : It can be used to risk stratify patients undergoing surgical procedures. E : It is not helpful in patients unable to physically exercise to maximal capacity. F : It is readily possible to differentiate between reversible ischaemia and infarcted areas of

myocardium. G : It is of no value following coronary angiography. H : False positives are seen more commonly in middle aged women as compared to conventional

exercise testing I : It should not be used in patients with very poor left ventriuclar function. J : A normal image indicates risk of of coronary events of 2-3% per year. Comment : Radionucleotide myocardial perfusion imaging is very useful in assessing the degree of coronary obstruction in patients with suspected coronary artery disease. It provides diagnostic and prognostic data. Patients can be stressed using conventional treadmill or pharmacological stress with agents such as adenosine or dobutamine. Adenosine should be avoided in asthmatics. A normal image indicates risk of coronary events of <1% per year. The test can also be helpful in targeting intervention following angiography and in the detection of hibernating myocardium in patients with poor left ventricular function. D : F:

Page 11: Masterclass Book Part 2

MOHAMMED IS-HAG 10

26.

plate 1 A 57-year-old lady is being pre-assessed for a gynaecological procedure and is found to have a murmur. What does her echo image show? A : Aortic regurgitation B : Aortic stenosis C : Mitral regurgitation D : Mitral stenosis E : Infective endocarditis. Comment : The image demonstrates a posteriorly directed jet of mitral regurgitation in a lady with anterior leaflet prolapse. C : 27. A 40-year-old female presents with a 6-month history of progressive breathlessness. Cardiac catheter data are as follows:

Pressure (mmHg) Oxygen saturation (%)

IVC

RA a 16; v 38 68

RV 81/17 67

PA 78/52 63

LV 123/6 97

Aorta 128/70 98

A pulmonary angiogram showed no evidence of thromboembolic disease. Which of the following is true concerning her condition: A : The condition is more common in males. B : Presentation is usually in the first decade of life. C : An ejection systolic murmur is commonly heart. D : A pan systolic murmur is commonly heard. E : Pulmonary oedema is a common feature. F : The ECG often shows left axis deviation. G : Echo is usually unhelpful in the diagnosis. H : Warfarin is not recommended. I : Calcium antagonists are contraindicated. J : Atrial septostomy may provide temporary improvement. Comment : This lady has primary pulmonary hypertension with no evidence of right to left shunting. The high v wave pressure in the right atrium is due to tricuspid regurgitation. The condition is more common in females and usually presents in the fourth decade of life. Physical signs include parasternal heave, loud pulmonary second sound, pansystolic murmur (due to tricuspid regurgitation) and evidence of right heart failure. ECG typically shows right axis deviation and evidence of right ventricular hypertrophy. Warfarin is recommended to all patients and high dose calcium antagonists only to those with a significant drop in pulmonary pressures with vasodilator testing. Patients with evidence of vasoactivity may also respond to prostacycline or nitric oxide therapy. Lung or heart-lung transplantation is offered to those who fail to respond to medical treatment. D : J:

Page 12: Masterclass Book Part 2

MOHAMMED IS-HAG 11

28. A 25-year-old woman is referred because during a routine antenatal visit at 32 weeks of pregnancy the obstetrician hears a systolic murmur. She has no cardiac symptoms. Which are the two most likely causes of her murmur? A : Aortic stenosis B : Ventricular septal defect C : Aortic regurgitation with flow murmur D : Atrial septal defect E : Triscuspid regurgitation F : Peripartum cardiomyopathy G : Hypertrophic obstructive cardiomyopathy H : Aortic sclerosis I : Innocent systolic murmur J : Mitral valve prolapse. Comment : Given that the woman is well, it is almost certain that the murmur has an innocent cause, being due to the hyperdynamic circulation of pregnancy. It is also possible that the obstetrician has heard a venous hum or a mammary soufflé. Mitral valve prolapse would be much the commonest of the ‘cardiac lesions’ listed in a 25-year-old woman. The presence of symptoms should raise the suspicion of significant pathology, but it is important to remember that some degree of weakness, exertional dyspnoea, dizziness and peripheral oedema are common during pregnancy. It is therefore very important to gauge the severity of any symptoms in relation to the stage of pregnancy, and a great deal of unnecessary anxiety can be generated by doctors who fail to do so. The investigation of choice to exclude a significant cardiac lesion is echocardiography. I:J: 29.

PLATE 2 A 37-year-old woman is seen in outpatients with frequent episodes of palpitations. She tells you that she has had them for only 2 years and gets chest discomfort during the attacks. See image for her ECG. What is the correct diagnosis? A : Anxiety-producing sinus tachycardia B : Wolf-Parkinson-White syndrome C : Lown-Ganong-Levine syndrome D : Ventricular tachycardia E : Left bundle branch block. Comment : Wolf-Parkinson-White is the syndrome of pre-excitation secondary to antegrade conduction down an accessory pathway from atrium to ventricle. This is characterized on the 12-lead surface ECG by a short PR interval, a delta wave and broad complexes (as a result of the delta wave). The accessory pathway can be in any location around the atrioventricular valves, i.e. mitral or tricuspid valves. Using various algorithms it is possible to accurately locate the position of the accessory pathway. In this patient's case the negative deflection of the delta wave in lead V1 suggests the pathway is on the right side, i.e. around the tricuspid valve. If it had been positive it would have pointed to a left-sided pathway. During an electrophysiological study she was found to have a right-sided posteroseptal pathway which was successfully ablated. Lown-Ganon-Levine syndrome differs from Wolf-Parkinson-White syndrome in that it has a short PR interval without a delta wave. This is thought to be due to accessory conduction tissue from the atria to the atrioventricular node. Whilst the complexes are broad, this is clearly sinus rhythm and not ventriuclar tachcardia. B :

Page 13: Masterclass Book Part 2

MOHAMMED IS-HAG 12

30. A 78-year-old man presents with a 3-month history of syncopal episodes and chest pain, both occurring only on exertion. Select the two most likely diagnoses from the list below: A : Aortic stenosis B : Epilepsy C : Orthostatic hypotension D : Sick sinus syndrome E : Coronary artery disease F : Intermittent third degree heart block G : Supraventricular tachycardia H : Atrial fibrillation I : Vasovagal syncope J : Hypertrophic obstructive cardiomyopathy. Comment : A history of syncope that occurs only on exertion is suggestive of outflow tract obstruction, when it results from reflex bradycardia and vasodilatation. Aortic stenosis is therefore one of the diagnoses to consider, but it would be most unusual for hypertrophic obstructive cardiomyopathy to present at this age.Arrhythmias can occur at any time, but they may happen only on exercise if they are sensitive to increased sympathetic outflow or ischaemia, and coronary artery disease would clearly be a likely diagnosis in a man of 78 years with these symptoms.The key physical signs to establish a diagnosis of aortic stenosis would be a slow rising carotid upstroke, an undisplaced thrusting apex, and an ejection systolic murmur radiating to the carotids, perhaps with a palpable thrill. A : E : 31. A middle-aged man is brought by ambulance to the Medical Admissions Unit. He was fitting when picked up and is still having a grand mal convulsion. The most appropriate treatment is: A : Lorazepam 2 mg intravenously B : Fosphenytoin 15 mg/kg body weight phenytoin equivalent, intravenously at a rate of 100-150

mg phenytoin equivalent / min C : Phenytoin 15 mg/kg body weight, intravenously at a rate of 50 mg/min D : Diazepam 10 mg intravenously E : Phenobarbitone 10 mg/kg body weight, intravenously at a rate of 100 mg/min. Comment : All of these are recognized treatments for status epilepticus. First-line treatment should be with intravenous benzodiazepine, with lorazepam preferred to diazepam because of its longer duration of action. Fosphenytoin is the preferred second-line treatment (phenytoin if this is not available). Phenobarbitone is one of several agents that can be used as third-line treatment, but seek specialist advice if first and second-line treatments are ineffective. A : 32. An 83-year-old lady is admitted having been found on the floor by her home help. She had fallen 6 hours prior to admission and thought she may have blacked out, but was not able to give any more details. Which two of the following are correct? A : Each year in the community 2/3 of people over 65 years have at least one fall/year. B : Age-related changes affecting vision, vestibular system and proprioception have no overall effect

on balance. C : Among older people that fall serious injury is more likely in those with cognitive impairment. D : A prolonged lie time (time on the floor) is not associated with significant morbidity. E : In older patients presenting to A and E with recurrent falls, carotid hypersensitivity is found in

less than 5% of cases. F : Orthosatic postural hypotension can be diagnosed if there is a fall in systolic blood pressure of

10mmHg. G : If there is no history of joint pain, physical examination of the neck is unnecessary. H : There is no evidence that balance training reduces the incidence of falls in older people. I : The Get Up and Go test is useful for checking gait abnormalities. J : There is no evidence that reviewing medications in older patients prevents falls. Comment : A 1/3 of people aged 65 years and above have at least one fall/year. Age-related changes afffecting vision, proprioception, vestibular input, muscle strength and joint flexibilty can all compromise balance. In older patients that fall, serious injury is more likely in women and in those with cognitive impairment, a low body mass index, or with two or more chronic conditions e.g. asthma, cancer and diabetes mellitus.A prolonged lie time is associated with bronchopneumonia, pressure sores, hypothermia, dehydration or rhabdomyolysis. About 1/3 of patients 50 years and over presenting to A+E after a fall have evidence of carotid hypersensitivity. Orthostatic postural hypotension can be diagnosed if lying and standing blood pressures show a sustained postural drop: systolic BP by at least 20mmHg, or diastolic BP by at least 10mmHg, on standing for at least two minutes. Examination should include movement of neck to see if this causes symptoms such as dizziness or syncope. There is evidence from seven randomized controlled trials that balance exercise can reduce the number of falls in older people. The Get Up and Go test is very useful for assessing mobility and checking for gait abnormalities. Careful review of the type and number of medications is an essential element to preventing further falls. There is evidence that reducing medications reduces falls. C : I:

Page 14: Masterclass Book Part 2

MOHAMMED IS-HAG 13

33. A 68-year-old woman with ascites secondary to carcinoma of the ovary is complaining of early satiety and intermittent vomiting. The antiemetic of choice is: A : Cyclizine B : Haloperidol C : Ondanestron D : Metoclopramide E : Dexamethasone. Comment : Metoclopramide as a prokinetic agent will increase the rate of transit of food through the gastrointestinal tract and alleviate her symptoms. She should also be advised to eat 'little and often'. D : 34. A 25-year-old woman presents with central chest pain. Which two of the following factors would most support the diagnosis of pericarditis? A : Pain that came on suddenly B : Presence of a pleural rub C : Temperature 37.0 degrees C D : Pain that was tearing in nature E : Preceding viral illness F : Elevated jugular venous pressure (JVP) G : Pain that is variable with position H : Previous pneumonia I : Pain associated with reflux J : Pain that radiated to the back. Comment : The commonest cause of pericarditis is viral infection. Other causes include: Post myocardial infarction (MI) / post cardiac surgery · Rheumatoid arthritis and other autoimmune rheumatic disorders · Renal failure · Hypothyroidism · Rheumatic fever · Malignancy · Chest radiotherapy. Pericarditic pain can be indistinguishable from ischaemic cardiac pain, but is sometimes perceived ‘more superficially’ and eased by sitting forward (perhaps because this allows pericardial fluid to move to the anterior surface of the heart).Pain that is of sudden onset, tearing in nature, and radiates to the back would be suggestive of aortic dissection. An elevated JVP could be due to pericardial effusion / constriction, but would not be expected in acute pericarditis. E :G: 35. Which of the following signs would you expect to find in a 50-year-old man with carcinoma of the lung and a large pericardial effusion causing diastolic right ventricular collapse? A : Sinus tachycardia B : Ejection systolic murmur at left sternal edge radiating to the carotids C : Widely split second heart sound D : Double apical impulse E : Wide pulse pressure F : Tapping apex beat G : Early diastolic murmur H : Pulsus paradoxus I : Ejection click J : Loud aortic closure sound (A2). Comment : Invasion of the pericardium by mediastinal tumours can give rise to effusive constrictive pericarditis. Patients may complain of dyspnoea, fatigue and sometimes chest pain. On examination they may be tachycardic, have pulsus paradoxus and a raised venous pressure with prominent X and Y descents. A : H: 36. A 45-year-old man is referred by his general practitioner with palpitations. He has no other associated symptons and specifically he is not presyncopal. Holter monitoring has demonstrated short non sutained runs of a monomorphic broad complex tachycardia. Which two of the following are the most likely arrhythmias? A : Sinus tachycardia B : Atrial fibrillation with intermittent rate associated bundle branch block C : Right ventricular outflow tract tachycardia D : Atrial flutter with one to one conduction E : Wolff-Parkinson-White syndrome F : Ischaemic ventricular tachycardia G : Atrioventricular nodal reentry tachycardia H : Ventricular Fibrillation I : Torsades de pointes J : Atrioventricular reentry tachycardia.

Page 15: Masterclass Book Part 2

MOHAMMED IS-HAG 14

Comment : Whenever you approach a patient with a broad complex tachycardia it is always safest to presume they have ventricular tachycardia until proven otherwise. In fact, it is most likely that they will have a ventricular tachycardia rather than one of the other possibilities above, which may produce similar traces on a Holter monitor. Patients are not always syncopal with ventricular tachycardia. In fact, right ventricular outflow tract tachycardia usually presents with just palpitations. It occurs as a result of a triggered focus in the right ventricular outflow tract and generally carries an excellent prognosis. It is best treated with radiofrequency ablation. This is in contrast to ischaemic ventricular tachycardia which carries a very poor prognosis unless treated appropriately. In the context of impaired left ventricular function this invariably means with an implantable cardioverter defibrillator (ICD). C : F: 37.

PLATE 3 A 73-year-old man presents with a three month history of feeling non-specifically unwell, has lost 6 kg in weight and has recently noticed the rash shown in the enclosed image. The creatinine is 340 µmol/l. Urinalysis shows blood and protein. Renal ultrasound shows kidneys of 10.8 and 11.2 cm with increased cortical echogenicity. Which of the following statements is correct? A : Renal biopsy should be performed and is likely to show a focal necrotising glomerulonephritis B : The renal impairment is likely to be chronic (i.e. irreversible) C : Skin biopsy is likely to yield a specific diagnosis D : Renal biopsy should be performed and is most likely to show membranous glomerulonephritis E : Laboratory blood tests are unlikely to yield a specific diagnosis. Comment : The rash shown is consistent with a leucocytoclastic vasculitis, and the history would be entirely consistent with ANCA (antineutrophil cytoplasmic antibodies) positive systemic vasculitis. A skin biopsy will not yield a specific diagnosis, whereas a test for ANCA is likely to be positive.In view of the preserved renal size, active urine sediment and rash, it is likely that there is an acute (i.e. reversible) component to the renal failure and a biopsy should be performed. This is most likely to show a pauci-immune, focal segmental glomerulonephritis, but it is important to exclude other conditions. A : 38. A 32-year-old gentleman is currently being investigated for hypertrophic cardiomyopathy (HCM). Which of the following statements are associated with sudden cardiac death and HCM? A : Family history of sudden cardiac death (SCD) B : Chest pain C : Breathlessness D : NSVT on 24-hour tape E : Age>40 F : Mutation in myosin-binding protein C G : Increased VO2 max H : Male sex I : LV wall thickness of 15mm J : T wave inversion on ECG. Comment : Hypertrophic cardiomyopathy is associated with sudden cardiac death. The following features are particularly associated with an adverse prognosis (the presence of > 1 of these features increases an individual’s risk): · LV septal thickness >30mm · family history of sudden cardiac death · syncope · ventricular arrhythmias on 24-hour tape. A : D : 39. A cachectic 87-year-old man is admitted with acute urinary retention and is found to have a creatinine of 520 micro mol/l. He has been on a number of medications. Which of the following is least likely to be nephrotoxic in this situation? A : Ibuprofen B : Ramipril C : Allopurinol D : Lansoprazole E : Sulfasalazine.

Page 16: Masterclass Book Part 2

MOHAMMED IS-HAG 15

Comment : As he has muscle wasting, a creatinine this high probably indicates severe renal impairment. This should improve with catheterisation and a good fluid input, but it is essential to stop all potentially nephrotoxic drugs.Do not rely on your memory -always check with the BNF as continuing a nephrotoxic drug at this stage may lead to irreversible renal failure. Of this list, only lansoprazole should be continued until his renal function has improved. D : 40. A 17-year-old Afro-Caribbean girl presents herself as an emergency with a 7-day history of general arthralgia and a painful rash over her lower limbs. Chest radiograph shows bilateral hilar enlargement but routine full blood count and urea, electrolytes, liver function tests and calcium are normal. Clinical examination is normal, apart from a raised erythematous painful rash, and she is apyrexial. Which of the following is the most likely diagnosis? A : Lymphoma B : Primary pulmonary hypertension C : Sarcoidosis D : Systemic lupus erythematosis E : Tuberculosis. Comment : The clinical presentation is most likely to be sarcoidosis, based on her ethnic origins and the features of arthralgia, rash and bilateral hilar enlargement.Measurement of serum angiotensin-converting enzyme (ACE) may help to make the diagnosis, but fibreoptic bronchoscopy with bronchial/transbronchial biopsies may be needed to confirm it. C : 41. A 74-year-old lady presents with severe peripheral oedema and hypotension over a 6-month period. Echocardiography has identified a thickened, calcified pericardium. She has previously been well apart from a past history of breast cancer. Which of the following are the most likely causes of her underlying condition? A : Diabetes mellitus B : Coronary artery disease C : Tuberculosis D : Chronic renal failure E : Recurrence of her breast cancer F : Viral pericarditis G : Previous radiotherapy H : Bacterial sepsis I : Diabetes insipidus J : Drug induced. Comment : This lady has constrictive pericarditis which causes impaired filling of the ventricles and reduced ventricular function. The commonest causes are tuberculosis, mediastinal irradiation and previous cardiac surgery. Imaging such as echocardiography, CT and MRI will usually demonstrate thickened and calcified pericardium. C : G: 42.

PLATE 4 A 40-year-old man has a transoesophageal echo for investigation of a soft systolic heart murmur after inadequate images are obtained from transthoracic imaging. He has no past history of note and is asymptomatic. Apart from the feature shown (see image), his study was normal. What is the most appropriate management? A : Urgent referral for surgical closure

Page 17: Masterclass Book Part 2

MOHAMMED IS-HAG 16

B : Addition of aspirin C : Reassurance and regular follow-up with echo D : Reassurance and no follow-up E : Urgent referral for percutaneous closure Comment : The above image shows a patent foramen ovale which occurs in 10% of the population and is found in 40% of patients with a past history of stroke. If these defects are found in patients with a past history of a cerebrovascular accident, a right to left shunt must be looked for by injecting agitated saline and asking the patient to perform a Valsalva manoeuvre. If a significant right to left shunt is demonstrated, then either long-term anticoagulation or percutaneous closure is recommended, as there is a risk of paradoxical embolus. The results of ongoing studies should help determine whether percutaneous closure is superior to anticoagulation alone in this context. When seen as an incidental finding in patients without a history of stroke, no treatment is required. D : 43. A 48-year-old man is found to have a blood pressure of 176/112 when he attends his general practitioner for a ‘new patient check-up’. He takes occasional anxiolytics for anxiety, but his past medical history is otherwise unremarkable. Physical examination is normal, excepting for obesity (BMI 32). A ‘routinebiochemical screen is normal, excepting for potassium 3.3 mmol/l. The two most likely causes of his hypertension are: A : Renal hypertension B : Hypothyroidism C : Renovascular hypertension D : Cushing’s syndrome E : Primary hyperaldosteronism (Conn’s syndrome) F : Acromegaly G : Essential hypertension H : Isolated clinic (‘white coat’) hypertension I : Phaeochromocytoma J : Coarctation of the aorta. Comment : All of the conditions listed, excepting hypothyroidism, might explain hypertension, but all other than essential hypertension and ‘white coat’ hypertension are rare (together accounting for less than 5% of cases). Although a secondary cause of hypertension is very unlikely it would be important to look for clues in history and examination that might suggest renovascular disease (ischaemic heart disease, transient ischmaemic attack (TIA) / stroke, peripheral vascular disease), renal disease (previous nephritis, results of urine testing for e.g. insurance / employment medicals). Episodes of palpitations, sweating or headache may suggest phaeochromocytoma, but a less exotic cause such as anxiety would be a much more likely explanation. The serum potassium concentration is just below the lower limit of normal, but primary aldosteronism (Conn’s syndrome) remains exceedingly unlikely. In the case of an obese man it is also important to note that the blood pressure reading may be falsely elevated as a result of inadequate blood pressure cuff size, and it would be important to ensure that readings were taken with appropriate equipment. G:H: 44. A 48-year-old male with Marfans syndrome presents acutely with a Type A aortic dissection. Which two of the following cardiac conditions are also associated with Marfans? A : Hypertrophic cardiomyopathy B : Myocardial infarction C : Amyloidosis D : Sarcoidosis E : Ventricular septal defect F : Patent ductus arteriosus G : Ventricular tachycardia H : Mitral valve prolapse I : Wolff-Parkinson-White syndrome J : Restrictive cardiomyopathy. Comment : Cardiovascular complications of Marfans syndrome include the following: aortic dilatation and dissection (which may cause myocardial infarction), mitral valve prolapse and regurgitation, left ventricular dilatation, pulmonary arterial dilatation. Aortic regurgitation is common in the context of a dilated aortic root. Beta blockers should be considered in any patient with Marfans and a dilated aortic root. However, surgical intervention is the definitive treatment. When the aortic root is greater than 5 cms then the risk is significant. Marfans patients should generally be screened for cardiovascular complications on an annual basis. B : H:

Page 18: Masterclass Book Part 2

MOHAMMED IS-HAG 17

45. A 78-year-old man is admitted weak and unable to stand after vomiting for several days. His plasma sodium concentration is 123 mmol/l and his urinary sodium concentration is 8 mmol/l. What is the likely cause of his hyponatraemia? A : Syndrome of inappropriate antidiuresis (SIADH) B : Diuretic treatment C : Loss of sodium in vomit D : Hypovolaemic stimulation of ADH release E : Addison’s disease. Comment : Plasma ADH concentration is normally controlled by plasma osmolality, but pain, nausea, hypovolaemia and anaesthesia are all powerful stimulants of ADH release and can generate much higher plasma levels than are seen in response to tonicity. Nausea and hypovolaemia are likely to have stimulated very high ADH levels as the explanation for this man’s hyponatraemia. In recognition of these facts, SIADH can only be diagnosed when the following criteria are satisfied: 1. The patient is clinically euvolaemic (JVP seen, no postural hypotension) 2. Decreased plasma sodium concentration and osmolality 3. Inappropriately high urine sodium concentration (>20 mmol/l) and osmolality (> plasma) 4. There is normal adrenal, thyroid and renal function. D : 46. A 71-year male has a witnessed tonic-clonic seizure whilst standing at a bar, enjoying a pint. He has had a prior anterior myocardial infarction 2-years ago. Examination is normal. ECG shows sinus rhythm with anterior Q waves. Which two of the following are the most appropriate initial investigations? A : Alcohol provocation test B : Coronary angiography C : Carotid Dopplers D : CT head E : Echocardiogram F : Electrophysiological study G : Exercise tolerance test H : Tilt test I : 24-hour tape J : Urinary cathecholamine levels. Comment : A 24-hour tape is important to help to identify any brady or tachycardia (e.g. ventricular tachycardia) that might account for an arrhythmic cause of the seizure, as a consequence of cerebral hyoperfusion. A CT head scan is also mandatory in this age group to exclude a space-occupying lesion. Subsequent investigations will be dictated by the exact nature of the event – history from a witness is extremely valuable. D : I: 47. A 27-year-old female presented with a 6-month history of breathlessness. Her cardiac catheter data shows the following: Pressure

(mmHg) Oxygen saturation (%)

IVC 19 51 RA 22 52 RV 120/15 55 PA 121/70 56 LV 112/11 77 Aorta 108/67 78 Which two statements are true of her condition? A : The patient will appear pale. B : Haemopysis is a frequent symptom. C : The presence of clubbing should lead to an alternative diagnosis. D : A fixed split second sound will be heard. E : Pulmonary oedema is a common finding. F : The condition usually presents in later life. G : Embolic complications may occur. H : Pregnancy is safe. I : Prognosis is good with medical treatment. J : Surgical correction of the cardiac lesion is recommended. Comment : This patient has Eisenmenger's syndrome. An uncorrected right to left shunt has resulted in systemic pulmonary pressures with subsequent shunt reversal as evidenced by desaturation in left ventricular blood. From these data one cannot determine the level of the shunt.The clinical features of this condition are of right heart failure, systemic oxygen desaturation (causing central cyanosis), and polycythaemia which can lead to embolic complications. Clubbing is common. Prognosis is very poor and pregnancy results in maternal death. Closure of the shunt at this stage does not change the prognosis. B : G:

Page 19: Masterclass Book Part 2

MOHAMMED IS-HAG 18

48. An 87-year-old lady is admitted with left iliac fossa pain, tenderness and fever. Which of the following are true? A : Colonoscopy is likely to be helpful. B : Ultrasound or computerised tomography (CT) is indicated. C : Diverticulosis affects 10% of the population over 50 years of age. D : Bleeding is an unusual complication of diverticular disease. E : Colonoscopy is unhelpful in patients admitted with bright red rectal bleeding. F : Diverticulosis is associated with an increased risk of colorectal neoplasia. G : Recurrent urinary infections raise the possibility of a complication of diverticular disease. H : A diverticular stricture is readily distinguished from a malignant stricture. I : Following a diverticular abscess, the affected diverticulae fibrose and further problems are uncommon. J : Uncomplicated extensive diverticulosis may explain weight loss. Comment : Ultrasound and CT are useful in detecting bowel thickening, abscess formation and even fistulae. As diverticulitis is associated with multiple micro-perforations, colonoscopy is not advised. Diverticular disease is the commonest cause of colonic bleeding which may be detected and endoscopically treated at colonoscopy.Recurrent urinary infections, pneumaturia or faeces in the urine are features of a colo-vesical fistula.Diverticulosis affects 50% of the population over 50. Most patients have few if any symptoms, but if complications occur (abscess, stricture or bleeding), recurrence is common and surgery should be considered. B : G: 49. A 70-year-old man, who is known to have ischaemic heart disease and has had short-lived episodes of atrial fibrillation in the past, presents with 48 hours of fatigue and breathlessness. He is not very ill, but his pulse is 150 / min in atrial fibrillation. Which two drugs would be most appropriate to achieve ‘chemical cardioversion’? A : Digoxin B : Quinidine C : Procainamide D : Disopyramide E : Sotalol F : Atenolol G : Propanolol H : Verapamil I : Amiodarone J : Diltiazem. Comment : Restoration of sinus rhythm can be achieved pharmacologically or by DC cardioversion. However, DC cardioversion is not likely to lead to permanent restoration of sinus rhythm in a patient who has had previous episodes of AF, hence in this case an attempt at ‘chemical cardioversion’ is appropriate.Class III agents – potassium channel blockers that prolong myocyte repolarisation – are most appropriate: sotalol or amiodarone.Digoxin can be used for rate control but does not promote return of sinus rhythm, indeed it’s use may make this more unlikely.Class I agents – e.g. quinidine, procaineamide and disopyramide, which prolong the action potential – have been used to try to restore sinus rhythm, but NOT in patients with ischaemic heart disease (such as this man). Use of these agents in atrial fibrillation has been superseded by that of sotalol and amiodarone. E :I: 50. A 77-year old man presents with 5 hours of chest pain at rest. He had previously undergone stenting to his left anterior descending artery 4 years previously. ECG shows inferior T-wave inversion, with ventricular ectopics. His troponin T is elevated at 0.4. He is already taking aspirin. Which two of the following would be considered appropriate intial therapeutic interventions? A : Amiodarone B : Change aspirin to clopidogrel C : Coronary artery bypass grafting D : Digoxin E : Flecainide F : GIIb/IIa receptor blocker G : Percutaneous coronary intervention H : Dipyridamole (modified release) I : Prophylactic dose of low molecular weight heparin J : Thrombolysis. Comment : This patient has presented with a non-ST-segment elevation myocardial infarction (NSTEMI). This group of patients has a high risk of further adverse cardiac events. The mainstay of initial treatment is aggressive anti-platetlet therapy: aspirin with the addition of clopidogrel, and consideration of an infusion of GIIb/IIa receptor blocker (blocks the platelet receptor). Therapeutic dose of low molecular weight heparin should be commenced. Early angiography and percutaneous coronary intervention should also be considered. Thrombolysis has not been shown to benefit patients with NSTEMI. F:G:

Page 20: Masterclass Book Part 2

MOHAMMED IS-HAG 19

51.

A 45-year-old man presents to the coronary care unit with central chest pain, breathlessness and a normal ECG. Cardiac enzymes are normal. He has an exercise tolerance test which demonstrates no ECG changes but he gets chest pain at peak exercise. Angiography of his right coronary artery is normal. The angiogram of his left coronary artery is shown (see image). Which of the following would be the most appropriate management? A : Increased anti-anginal medication B : Coronary artery bypass grafting C : Percutaneous transluminal angionplasty (PTCA) and stent deployment D : Off-pump coronary artery bypass grafting E : PTCA alone. Comment : This man's coronary angiogram demonstrates a severe stenosis of the proximal left anterior descending artery. This is certainly amenable to PTCA. Where possible if the anatomy allows, stent deployment offers less restenosis/occlusion rates than PTCA alone. Coronary artery bypass grafting for single vessel coronary artery disease is generally only considered if PTCA is not possible or has failed. Off-pump coronary artery bypass grafting of the left anterior descending artery is an evolving concept utilizing equipment that holds the beating heart steadily enough to suture a graft into position. C : 52. A 58-year-old woman presents to casualty with back pain and has a blood pressure of 240/120. A CT scan of her chest and abdomen is organised and demonstrates an adrenal mass with type B aortic dissection, involving the left renal artery. Which of the following treatments are LEAST appropriate? A : Methyl dopa B : Endovascular aortic stenting C : Ramipril D : Surgical repair E : Intravenous GTN F : Sodium nitroprusside G : Atenolol H : Amlodipine I : Thiazides J : Phenoxybenzamine. Comment : A type B aortic dissection includes all aortic dissections not involving the ascending aorta. Their downward course variably involves the splanchnic and renal arteries. Most cases are managed conservatively without the need for surgical repair or endovascular stenting, although these treatments may be needed. The priority is blood pressure control. All the above blood pressure lowering drugs are useful apart from angiotensin-converting enzyme (ACE) -inhibitors which should be avoided (one of the renal arteries is involved and there is an appreciable risk of coexistent renal artery atherosclerosis in patients with aortic atherosclerosis). The adrenal mass and marked hypertension in a relatively young woman raise the possibility of a phaeochromocytoma. Unopposed betablockade (that is without alpha blockade) should be avoided in such patients. C : G:

Page 21: Masterclass Book Part 2

MOHAMMED IS-HAG 20

53. You see a 48-year-old Afro-Caribbean man in the outpatient clinic with uncomplicated essential hypertension. His blood pressure today is 154/102mmHg despite optimization of non-pharmacological therapy. Which one of the following drugs would you use as the first-line agent in this patient? A : Atenolol 50mg od B : Nifedipine 10mg tds C : Amlodipine 5mg od D : Ramipril 2.5mg od E : Enalapril 5mg bd. Comment : Non-pharmacological therapy should always be optimized prior to commencement of medication, whenever possible. Hypertension is particularly common in Afro-Caribbeans and associated with particularly higher risk of complications. Therefore effective long-term treatment, with a low threshold for multiple therapy where necessary, is particularly important. Studies indicate that drugs such as ACE (angiotensin-converting enzyme) inhibitors and Beta-receptor antagonists are less effective in Afro-Caribbeans. The reason appears to be related to the finding that the renin-angiotensin-aldosterone (RAA) system is commonly suppressed in the majority of Afro-Caribbeans. As such, drugs that suppress the RAA system are less likely to be effective. Calcium-channel blockers (CCBs) and diuretics appear to be more effective in this subgroup. However, diuretics may not be suitable in this case as they are commonly associated with impotence. Short-acting CCBs do not provide prolonged BP control, can cause reflex tachycardia and may be associated with higher mortality. Therefore, long-acting CCB should be the first-line drug of choice. Ideally, a once-daily agent with that provides a smooth 24-hour BP control (e.g. Nifedipine LA 30mg od or Amlodipine 5 mg od) to improve compliance would be preferable. C : 54. A 55-year-old lady has a 2-year history of exertional chest pain and has a positive exercise test with significant ST segement depression. She then has a coronary angiogram which demonstrates normal coronary arteries. She is told she has Cardiac Syndrome X. Which of the following statements about Syndrome X is incorrect? A : Patients have typical symptoms of occlusive coronary artery disease. B : Myocardial perfusion imaging is usually normal in patients with Sydrome X. C : The condition carries with it a normal life expectancy. D : Beta blockers, calcium antagonists and nitrates have all been shown to improve symptoms. E : It is caused by coronary microvascular dysfunction. Comment : Cardiac Sydrome X is characterised by typical exertional angina, ST changes on ECG, normal coronary arteries on angiography, absence of coronary artery spasm and absence of cardiac disease associated with microvascular abnormalities such as diabetes and hypertension. About 50% will have abnormal myocardial perfusion imaging. B : 55. A 53-year-old woman presents with a digoxin overdose. Which of the following statements is false? A : Tachyarrhythmias do not occur. B : Peak effects can be delayed after ingestion by 6-12 hours. C : Digoxin can precipitate severe hyperkalaemia (>6.5mmol/l). D : DC cardioversion can precipitate intractable ventricular fibrillation (VF) or asystole. E : Activated charcoal reduces absorption. Digoxin is slowly absorbed, hence peak effects can be delayed by up to 12 hours after overdose. Oral activated charcoal may reduce absorption, although giving multiple doses is controversial. Nausea and vomiting occur early after poisoning; other features include confusion, headache and visual disturbances. Any brady or tachyarrhthymia can occur. Treatment is with correction of hyperkalaemia and atropine for bradyarrhythmias. Digoxin-specific antibodies are useful in i) those with hyperkalaemia resistant to treatment, ii) bradyarrhythmias with hypotension non-responsive to atropine and iii) tachyarrhythmias with hypotension. Anti-arrhythmic drugs and DC cardioversion should be avoided as they can precipitate intractable asystole or VF. A : 56. A 35-year-old woman is admitted with severe pulmonary oedema 2 months after her second pregnancy. A diagnosis of peripartum cardiomyopathy is made. Which of the following statements about peripartum cardiomyopathy is false? A : It may occur at any stage of pregnancy. B : Maternal mortality is approximately 20%. C : If cardiac function returns to normal there is an increased risk of recurrence in subsequent pregnancies. D : Recovery of left ventricular function may take up to a year. E : Fifty per cent make a full recovery. Comment : Heart disease is the highest cause of maternal death in the UK. Peripartum cardiomyopathy is defined as cardiac failure occurring in the last month of pregnancy or within 5 months of delivery. It is rare and thought to represent a form of myocarditis. Adverse risk factors include being older than thirty, giving birth to twins and multiparity. A :

Page 22: Masterclass Book Part 2

MOHAMMED IS-HAG 21

57.

PLATE 5 What is the most likely diagnosis in this patient? A : Subendocardial anterolateral infarction B : Hypothermia C : Hypothyroidism D : Digoxin toxicity E : Hypokalaemia. Comment : A junctional bradycardia associated with “inverse ticks” of the ST segments is strongly suggestive of digoxin toxicity. D : 58. A 74-year-old woman presents with breathlessness. She is a small woman (55 kg) with a chest infection. She is not very unwell, but is in atrial fibrillation at a rate of 170/min. Her electrolytes are normal (K 4.2 mmol/l). As well as treating her pneumonia, you decide to digitalize by prescribing: A : Digoxin 0.25 mg orally once daily B : Digoxin 1.0 mg orally over 24 hours in divided doses C : Digoxin 1.0 mg intravenously over 20 min D : Digoxin 0.125 mg orally once daily E : Digoxin 0.25 mg orally three times daily for one week, then twice daily for one week, then once daily thereafter. Comment :The options for treatment of atrial fibrillation are: 1. DC cardioversion if the patient is compromised haemodynamically or has ischaemic cardiac pain. 2. Digoxin - 1.0-1.5 mg orally in divided doses over 24 hours, but can be given intavenously in emergency (0.25-0.5 mg over 10-20 min, repeated after four to eight hours to total intravenous loading dose of 0.5-1.0 mg) 3. 'Medical cardioversion' with amiodarone or flecainide In this clinical context it is likely that the atrial fibrillation (if new) will revert to sinus rhythm as the woman recovers from her pneumonia and most physicians would digitalize in preference to the other options described. B : 59. A 65-year-old gentleman attending the cardiology clinic complains of swelling and tenderness of his breasts. You diagnose probable gynaecomastia. Which of the following drugs is most likely to be the cause? A : Simvastatin B : Amiodarone C : Digoxin D : Aspirin E : Ramipril.

Page 23: Masterclass Book Part 2

MOHAMMED IS-HAG 22

Comment : Digoxin is the most likely cause of his gynaecomastia. This side effect is more common with longer-term use and may be unilateral or bilateral. Important differential diagnoses to consider include male breast cancer, liver disease, testicular tumours and hyperthyroidism. Other drugs that can cause gynaecomastia include oestrogens, spironolactone, cimetidine, verapamil and nifedipine. The gynaecomastia usually improves on stopping the drug or reducing the dose. C : 60. A 66-year-old man presents with a three-month history of syncopal episodes and chest pain on exertion. There are no abnormal physical findings on examination. The most likely diagnosis is: A : Aortic stenosis B : Hypertrophic obstructive cardiomyopathy C : Orthostatic hypotension D : Ischaemic heart disease E : Complete heart block. Comment : Aortic stenosis would certainly need to be considered in a man of this age with this history, but without any signs it is hard to sustain this diagnosis. The signs to look for would include a slow rising pulse, narrow pulse pressure, forceful apex, and aortic systolic murmur radiating to the neck (perhaps with carotid thrill). Hypertrophic cardiomyopathy would be an uncommon diagnosis in a man of 66 years but would require serious consideration in a younger patient not otherwise at risk of coronary disease. However, once again this diagnosis is hard to sustain with a normal physical examination.Complete heart block causes syncope, but there is rarely any relation of this to exercise and it does not cause pain. D : 61. An 85-year-old lady presents to the A&E department with an episode of syncope. An ECG shows a left bundle branch block pattern and her echocardiogram reveals a heavily calcified aortic valve with restricted movement. Which of the following statements is correct? A : A peak aortic valve gradient of <50mmHg indicates that the aortic stenosis will not require

surgical treatment. B : A peak aortic valve gradient of >50mmHg confirms that the aortic stenosis is the cause but her

old age means that medical therapy is the safest option. C : Syncope carries a poor prognosis in aortic stenosis and she requires urgent assessment for

aortic valve replacement. D : Ballon valvotomy may avoid the need for surgey and should be the initial approach. E : Her left bundle branch block on ECG suggests that transient bradycardia is more likely to be the

cause of her syncope and should first be investigated. Comment : In some patients with aortic stenosis the peak LV to Aortic gradient can be misleading because of reduced cardiac output due to impaired left ventricular function. In such cases the calculated aortic valve area is more useful as a guide to severity. In all cases of aortic stenosis the development of syncope signifies a poor prognosis and urgent surgery is usually needed to replace the valve. Balloon valvotomy has been used successfully in adolescent non-calcified valvular stenosis, but is rarely used in adults and not in heavily calcified valves. C : 62. An HGV (group 2 entitlement) driver should not drive post myocardial infarction (MI) until which of the following conditions has been met? A : Four weeks post infarction. B : Angiography has been undertaken. C : Completion of 3 stages of Bruce protocol exercise test 6 weeks post-MI. D : Completion of 3 stages of Modified Bruce protocol exercise test 4 weeks post-MI. E : Six months free of symptoms. Comment : Following MI, group 2 entitlement holders are disqualified from driving for at least 6 weeks. Re-licensing may then be permitted if the following exercise test requirements are met: the driver should be able to complete 3 stages of Bruce protocol exercise test (bicycle or treadmill), off anti-anginal medication for 48 hours. During the test they should remain symptom free, with no evidence of hypotension, ventricular tachycardia or significant ST segment shift (i.e. greater than 2mm horizontal or down-sloping ST segment depression).Coronary angiography is not required but, if it has been undertaken, re-licensing will not be permitted if left ventricular ejection fraction is < 40%, or if there is a significant, untreated left main stem stenosis equal to or > 50%, and/or proximal left anterior descending artery stenosis equal to or greater than 75%. C : 63. Which of the following statements is true of idiopathic dilated cardiomyopathy (IDC)? A : Over 50% of cases are caused by infection with Coxsackie B virus B : Approximately 25% of patients have evidence for familial disease C : Often responds to high dose steroids D : Should not be treated with ?-blockers E : Is common in patients with Noonan syndrome Comment : Dilated cardiomyopathy defined by the presence of a dilated and poorly functioning left ventricle in the absence of abnormal loading conditions or ischaemic heart disease is sufficient to cause

Page 24: Masterclass Book Part 2

MOHAMMED IS-HAG 23

systolic impairment. Although experimental and clinical data suggest that viruses contribute to the pathophysiology of IDC, most patients do not have evidence for viral infection at presentation. At least 25% of patients have evidence for familial disease with predominantly autosomal dominant inheritance. A further 20% of relatives have isolated left ventricular enlargement that can progress to IDC. Dilated cardiomyopathy also occurs in a number of X-linked diseases such as Becker’s and Duchenne’s muscular dystrophies and X-linked IDC.Noonan syndrome is a rare disorder characterized by features similar to Turner’s syndrome (short stature, neck webbing, and hypertelorism). Approximately 25% of patients have hypertrophic cardiomyopathy. Pulmonary stenosis (24%) and secundum atrioseptal defects (10%) are also frequent. B : 64. A 52-year-old man is on an ECG monitor following an acute myocardial infarct the previous day. Whilst you are on the ward round he suddenly loses consciousness and the nurse cannot find a pulse. The ECG monitor shows a broad complex tachycardia. After checking it is safe to approach, your first action should be to: A : start chest compressions. B : defibrillate at 360 joules. C : give a precordial thump. D : perform synchronised cardioversion at 200 joules. E : give lignocaine (lidocaine) 100 mg IV. Comment : Pulseless ventricular tachycardia is the likeliest diagnosis. This should be treated exactly the same as ventricular fibrillation. If the arrest is monitored, a precordial thump is indicated as it may terminate the VT. If this is unsuccessful, the patient should be defibrillated a soon as possible. C : 65. A 28-year-old woman is sent to the Medical Assessment Unit with pain in the left side of her chest. The pain is sharp, exacerbated by twisting, taking a deep breath, or coughing. She is tender over the left 5th and 6th ribs, but examination is otherwise entirely normal. Her oxygen saturation (breathing air) by pulse oximetry is 99%. She says that the pain came on shortly after she had to haul her misbehaving three-year-old daughter out of a shop. What is the most likely cause of her chest pain? A : Pulmonary embolism B : Pneumothorax C : Musculoskeletal D : Pneumonia E : Pericarditis. Comment : The history and physical findings mean that musculoskeletal pain is much the most likely cause for her presentation, but pneumothorax and pulmonary embolism would be reasonable considerations, although both would seem unlikely.Pneumothorax is best excluded by chest radiography.If blood D-dimer concentration is not elevated, then the likelihood of pulmonary embolism is negligible in a patient with low pretest probability of this diagnosis.Hence,if D-dimers are not elevated in this patient,she can be reassured and discharged(with analgesic medication if required)without recourse to investigation such as ventilation / perfusion scanning or CT pulmonary angiography.C: 66. A 70-year-old man presents with breathlessness. He has had a history of previous myocardial infarction and peripheral vascular disease. He has recently been started on captopril for hypertension. He has been anuric for 1 hour. His serum urea is 45 mmol/l and his potassium is 7.9 mmol/l. His ECG shows features compatible with hyperkalaemia. The first most appropriate step in management would be: A : ten units of soluble insulin with 50g glucose(50%) IV B : haemodialysis C : bicarbonate (100 mls of a 4.2% solution) by IVI D : calcium resonium 30g E : 10-30 ml calcium gluconate (10%) IVI. Comment : Giving calcium gluconate IV will instantly lower myocardial excitability and should produce very quick ECG improvement and hopefully prevent sudden cardiac arrest - it will not affect potassium levels. Other measures listed above can be used to promote potassium excretion but may take several hours to become effective. The patient may have renal failure precipitated by the use of angiotensin-converting enzyme (ACE) inhibitors with underlying renal artery stenosis. E : 67. An asymptomatic 45-year-old patient has an echocardiogram as part of a screening programme. A suspected left atrial myxoma is identified. What is the optimal initial treatment strategy? A : Aspirin and repeat echo in 1-2 months B : Chemotherapy C : Immediate surgical removal D : Repeat echo in 1-2 months E : Warfarinisation.

Page 25: Masterclass Book Part 2

MOHAMMED IS-HAG 24

Comment : Atrial myxomas are the commonest form of cardiac tumour. They far more commonly arise in the left atrium, most frequently attached to the inter-atrial septum. Systemic embolic complications are common and can be devastating. Hence, unless severe co-morbidity precludes anaesthesis, immediate surgical removal is advocated. Patients may present with dyspnoea, evidence of systemic embolisation or constitutional upset with fever and elevated inflammatory markers. C : 68.

A 50-year-old woman presents with shortness of breath and palpitations. Diagnostic cardiac catheterization is performed. From the information given in the table, what is the most likely complete diagnosis? A : ASD B : ASD and aortic stenosis C : Fallots tetralogy D : Sub-valvular aortic membrane E : VSD and aortic stenosis. Comment : The question asks for the complete diagnosis. The pressure data demonstrates a significant difference in pressure from LV to aorta (45 mmHg). This means that there is at least moderate aortic stenosis (from the data it is not possible to determine whether this is occurring at the level of the valve or from a sub-valvular membrane). The right heart pressures (RA, RV, PA) are slightly elevated, whereas the wedge pressure (effectively left atrial pressure) is normal. The aortic saturation is respectable at 95% and therefore excludes the diagnosis of Fallots. There is a step-up in saturation at the level of the RA, thereby confirming the presence of an ASD. Although there is a minor increase from RA to RV, this is not significant and relates to the mixing of blood in the RA and the exact point of sampling of RA blood. B : 69. You see a 60-year-old man who is on a diuretic, ACE inhibitor and beta-blocker because of heart failure following a recent myocardial infarction. However, he now complains of several episodes of rapid fluttering in his chest, which makes him breathless. You suspect paroxysmal atrial fibrillation and consider adding either of the following two medications: A : d-Sotalol B : Nifedipine C : Verapamil D : Lignocaine E : Digoxin F : Procainamide G : Adenosine H : Amiodarone I : Quinidine Comment : It can be very difficult to manage atrial fibrillation in patients who have a history of ischaemic heart disease and heart failure. Class I anti-arrhythmic agents such as procainamide and

Page 26: Masterclass Book Part 2

MOHAMMED IS-HAG 25

quinidine appear to increase the risk of sudden death in patients with LV dysfunction, probably through a pro-arrhythmic effect. This increased risk of sudden death was also found with d-sotalol when used in patients with heart failure. Nifedipine has no beneficial effect on atrial fibrillation, and verapmil may worsen heart failure if used in this patient who is already on a beta-blocker. Lignocaine and adenosine are only used intravenously, and do not work in paroxysmal atrial fibrillation. Digoxin (for rate control) or amiodarone (to reduce the frequency of attacks) may be potentially suitable in this case. E :H: 70.

PLATE 6 The following ECG was obtained from a 46-year-old man. What is the diagnosis? A : Hypokalaemia. B : Hypercalcaemia. C : Hypothermia. D : Hypocalcaemia. E : Hyperkalaemia. Comment : This tracing shows the classic appearances of severe hyperkalaemia. There is peaking of the T waves, reduction in P wave voltage and broadening of the QRS complex. The rate is slowed and an ominous sine wave pattern is apparent which usually preceeds asystole. Urgent treatment is required. E : 71. A patient presents with a suspected aortic dissection. Which of the following investigations is least helpful in confirming/refuting the diagnosis? A : Aortography B : CT scan C : Transthoracic echocardiogram D : Magnetic resonance imaging E : Transoesophageal echocardiogram. Comment : The choice of investigation will depend upon local availability and expertise in interpretation of the results. Magnetic resonance imaging and CT scanning are excellent methods for imaging the aorta, but require transfer of the patient to the radiology department. Transoesophageal echocardiography offers some advantages in particularly unstable patients, since it can be performed in the anaesthetic room, or on ITU. Whilst a Transthoracic echocardiogram may provide valuable information, such as detection of pericardial effusion or involvement of the aortic valve (regurgitation), it cannot exclude the diagnosis with any degree of certainty. C : 72. Which of the following is NOT normally associated with congenitally corrected transposition (atrioventricular and ventriculoarterial discordance)? A : Right (systemic) ventricular failure B : Arrhythmias C : Ventricular septal defect D : Tricuspid valve regurgitation E : Left (pulmonary) ventricular hypertrophy. Comment : Congenitally corrected transposition is a rare variant of congenital heart disease, which may not be detected in childhood and hence present in adult life. Morphologically, the right atrium connects to the left ventricle (LV), and the left atrium connects to the right ventricle (RV). Similarly the aorta arises from the RV and the pulmonary artery from the LV. The net effect of this ‘double discordance’ is to create a normal physiological circulation. However the RV and tricuspid valve (TV) are exposed to systemic pressure and with time tricuspid regurgitation and RV failure develop. As the LV is exposed to the low-pressure pulmonary circulation LVH does not develop under normal circumstances. Other congenital anomalies that are associated with congenitally corrected transposition and may force its presentation in infancy and are ventricular septal defect and coarctation. Arrhythmias are common and include the development of atrioventricular conduction block related to the abnormal position of the AV node within the heart, and tachyarrhythmias secondary to accessory atrioventricular connections or focal atrial tachycardias. E :

Page 27: Masterclass Book Part 2

MOHAMMED IS-HAG 26

73. A 60-year old female presents with recurrent syncope; ECG reveals complete heart block. She is scheduled for a pacemaker. Which of the following pacemakers would be most appropriate? A : AOO B : DDD C : Implantable cardioverter defibrillator (dual chamber) D : VVI E : VVIR. Comment : Pacemaker types are identified by a 3 or 4 letter code. The first letter = chamber paced (A=atrium; V=ventricle; D=both or dual). The second letter represents which chamber is sensed. The third is what response the pacemaker gives to a sensed beat (I=inhibit; T=trigger; D=both). The fourth, usually R (rate responsive) is for more fancy technologies! In this case of complete heart block, in order to maintain AV synchrony, a dual chamber pacemaker is required (DDD). Assuming the atrial (p wave) rate is normal the function will be generally sensing the p wave and then pacing the ventricle. B : 74. A 47-year-old male is admitted to the acute admitting ward with severe pulmonary oedema. His known to have HIV infection but has had no AIDS-defining illnesses. Which of the following is not a recognised cardiac manifestation of HIV infection? A : Myocardial neoplastic infiltration B : Pericardial Kaposi’s sarcoma C : Marantic mitral valve endocarditis D : Asymmetrical septal hypertrophy E : Antiretroviral drug induced cardiomyopathy. Comment : HIV infection can directly or indirectly affect all aspects of cardiac anatomy and physiology. Up to 20% of patients with HIV infection have a dilated cardiomyopathy. It account for approximately a third of deaths related to HIV infection. Neoplastic infiltration with Kaposi’s sarcoma or lymphoma can affect the myocardium or pericardium. Marantic (non-infective) endocarditis is a recognised complication of HIV infection. A number of antiretroviral drugs can cause a significant cardiomyopathy. Asymmetrical hypertrophy of the septum is associated with hypertrophic cardiac myopathy (HOCM), not HIV infection. D : 75. A 32-year-old woman has been referred to you by her doctor, after complaining of syncope and breathlessness. Her sister died suddenly in her 20’s. Clinically she has loud pulmonary second heart sound. What is the most likely diagnosis? A : Aortic stenosis B : Mitral stenosis C : Tricuspid regurgitation D : Primary pulmonary hypertension E : Aortic regurgitation. Comment : Given the lack of signs and family history of sudden death the most likely diagnosis is pulmonary hypertension. Further investigations would include transthoracic echocardiography and left and right cardiac catheterisation. D : 76. A 70-year old man is investigated for deteriorating anginal symptoms. Echo confirms a stenotic aortic valve with peak gradient 80 mmHg. Angiography demonstrates a discrete severe stenosis in his left anterior descending artery (LAD). Which would be the optimal treatment? A : Aortic valve replacement B : Aortic valve replacement and internal mammary artery graft to the LAD C : Beta-blocker, aspirin, statin and review D : Percutaneous coronary angioplasty to the LAD E : Percutaneous aortic valvotomy and coronary angioplasty to the LAD. Comment : This man has severe aortic stenosis and concomitant coronary artery disease. Whilst it is impossible to differentiate which lesion is causing his current symptoms, symptomatic aortic stenosis is associated with significantly impaired prognosis. Therefore surgical intervention is warranted. Percutaneous aortic valvotomy is relatively unsuccessful in adults, with rapid restenosis. B : 77. A 76-year-old lady is admitted to the coronary care unit with unstable angina. Her 12 lead electrocardiogram shows ST depression of more than 2mm in the anterior leads. Clinical examination shows evidence of acute heart failure. Despite medical treatment she has chest pain two hours after admission. Which of the following are correct? A : Morphine should not be given to relieve pain if s/l GTN has been ineffective as she is 76 years old. B : Verapamil should be given if she is unable to tolerate a beta blocker. C : Patients with impaired LV function (EF<50%) should not undergo coronary artery bypass grafting (CABG). D : Cardiac catheterisation is used to assess patients for CABG only. E : Patients with significant left main stem disease have increased survival with CABG.

Page 28: Masterclass Book Part 2

MOHAMMED IS-HAG 27

Comment : Morphine may be necessary to relieve anginal pain - age is not a contraindication, although aliquots of smaller doses e.g. 2.5mg should be used to achieve pain control. Calcium channel blockers should not be given to patients in pulmonary oedema or with impaired LV function.Impaired LV function (EF<50%) is not a contraindication to CABG. Cardiac catheterisation informs the choice between medical treatment, CABG and percutaneous transluminal coronary angioplasty.Patients with significant left stem main disease (>50% occlusion) or significant three vessel disease with depressed LV function have increase survival if they undergo CABG, although patients older than 75 yrs were not included in the trials that demonstrated such benefit. E : 78. A 79-year-old man is admitted via A&E with a blackout. He has had several in the past 6 months. He usually feels well within minutes of the event. There is no witness. His ECG is normal and examination is unremarkable. What is the most appropriate investigation? A : CT head B : EEG C : Echocardiogram D : Holter monitor E : Tilt table test. Comment : With blackouts having such a rapid recovery time, there is likely to be a cardiological aetiology and therefore CT head and EEG will have low yield. Holter monitors are well-known to have a low pick up rate. With a normal ECG, the heart structure, and therefore an echo is likely to be normal. The best investigation is a tilt-table test. E : 79. A 59-year-old man with moderate chronic obstructive lung disease is admitted breathless following an episode of syncope while shopping. There is no previous history of sycope. Past history includes multiple stab ligations for bilateral varicose veins four weeks previously and open cholecystectomy 10 years before. On examination he is breathless at rest, apyrexial, pulse 104, BP 110/60. Heart sounds normal. Chest examination reveals a few scattered wheezes. Neurological examination was normal ECG sinus tachycardia, inverted T waves in V1-V3. Chest X-ray normal. Oxygen saturations 92% on 40% oxygen. PEFR 290L/min. What is the most likely diagnosis? A : Acute pulmonary oedema B : Acute exacerbation of chronic obstructive pulmonary disease C : Pulmonary embolism D : Acute myocardial infarction E : Pneumocystis. Comment : The correct answer is pulmonary thromboembolism. This gentleman presents with breathlessness, which is the commonest symptom associated with pulmonary embolism, and syncope, which is a less common but often poorly recognised presenting symptom. He is hypoxic with oxygen saturations of only 92% on 40% oxygen and has non-specific but recognised ECG changes. His recent varicose vein surgery also gives him an additional risk factor. Acute myocardial infarction rarely presents with syncope with no ECG changes and ongoing hypoxia would not be explained with a 'normal' chest radiograph. Chronic obstructive pulmonary disease (COPD) is also unlikely given history of syncope and little evidence of severe airways obstruction. C : 80. A 68-year-old man is admitted to the coronary care unit and thrombolysed for an inferior myocardial infarction. He makes an uneventful recovery. His total serum cholesterol on the admission blood test is 4.8mmol/L. What action should be taken? A : He should be reassured that his cholesterol is normal. B : He should have a repeat fasting serum cholesterol measured before discharge from hospital. C : He should receive dietary advice and have his serum cholesterol measured in 3 month's time D : He should be started on an HMG Coenzyme A inhibitor. E : A full lipid profile should be obtained and lipid lowering drug treatment started if his LDL

fraction is>3.5mmol/L and his HDL is <1.0mmol/L. Comment : Drug treatment to lower serum cholesterol should be based on a person’s risk of an ischaemic heart disease (IHD) event rather than initial cholesterol level. Any person who has had a myocardial infarction has about a 10% chance (without treatment) of dying from ischaemic heart disease in the following year and about a 5% chance of IHD death in each year thereafter. All such people (in the absence of contraindications to the specific drugs) qualify for HMG coenzyme A reductase inhibitor therapy (Statins) regardless of their cholesterol level. There is a constant proportional relationship between serum cholesterol and disease risk, so any reduction in cholesterol level from any starting point leads to the same proportional reduction in IHD risk. Those people with the highest absolute starting risk (namely those with pre-existing IHD, such as this man who has had an AMI) stand to benefit the most. Non-pharmacological means of serum cholesterol reduction are far less effective, and whilst important are inadequate in this patient. D :

Page 29: Masterclass Book Part 2

MOHAMMED IS-HAG 28

81.

PLATE 7 The following data were obtained during diagnostic cardiac catheterization (see image). What is the underlying diagnosis? (Scale in the centre is 0-400 mmHg). A : Systemic hypertension B : Aortic stenosis C : Aortic incompetence D : Ventricular septal defect Correct Answer : B Comment : On the left (A), the image shows a typical left ventricular (LV) pressure recording. Peak LV systolic pressures are in excess of 240 mmHg (each major division on the vertical scale represents 40 mmHg). When the catheter is pulled back to the aorta, a typical trace is seen (B). In the aorta the peak systolic pressure is around 120 mmHg. As such a gradient in excess of 120 mmHg exists across the aortic valve, and this is compatable with severe aortic stenosis (>70 mmHg). Severe aortic stenosis can still be present with gradients < 70 mmHg, when left ventricular function is impaired. B 82. A 48-year-old female is referred to the clinic with palpitations. There are no other associated symptoms. She does however have a family history of sudden cardiac death. She has a normal ECG and echocardiogram. Her 24 hour ECG demonstrates frequent ventricular ectopic activity. Which of the following is the likely diagnosis? A : Brugada syndrome B : Long QT syndrome C : Hypertrophic obstructive cardiomyopathy D : Arrhythmogenic right ventricular dysplasia E : None of the above. Comment : Palpitations are a very common presenting feature and in the context of a normal ECG and echo, reassurance is generally all that is needed. In this lady the most concerning feature is a history of sudden cardiac death in the family. More information regarding this is therefore required to assess her risk. All of the others generally will have abnormal ECGs but in the absence of a specific family history of them it is likely that she will need no further investigation. E :

Page 30: Masterclass Book Part 2

MOHAMMED IS-HAG 29

83.

PLATE 8 A 75-year-old man had a pacemaker implanted 18 months earlier. He presents with pain over his pacemaker. This is what you find (see image). What is the appropriate management? A : Intravenous antibiotics and closure of the wound B : Reassurance C : Replace pacemaker generator and treat with antibiotics D : Sterile dressings and wait for healing via secondary intention E : None of the above. Comment : The generator has eroded through the chest wall skin. This denotes infection and therefore requires extraction of the whole pacemaker system, i.e. the generator and leads. It is inevitable that the leads will be infected as well. Replacement of the generator alone will be inadequate. Pacing system infection is rare (<1%) but a very serious complication. Endocarditis associated with infected pacing systems carries a mortality of a third. This system will need explanting using specialized lead extraction tools in an expert centre. The procedure carries with it a mortality of up to 5%. E : 84.

PLATE9 A 56-year-old lady had a pacemaker implanted 4 weeks earlier for complete heart block. This is the ECG recorded at her first pacemaker check. What is the appropriate management?

Page 31: Masterclass Book Part 2

MOHAMMED IS-HAG 30

A : Reassure that all is well B : Start amiodarone C : Consider DC cardioversion D : Admit for immediate revision of her pacemaker system E : None of the above. Comment : Her ECG is extremely abnormal! It is normal to see a pacing spike before each p wave and QRS complex with a dual chamber device. However, on this ECG the first spike triggers the QRS complex and the second spike is within the QRS complex. There are two possible causes of this ECG appearance. The device may have been connected incorrectly so that the atrial lead is connected to the ventricular channel. However, in this lady, the atrial lead had displaced and prolapsed into the right ventricle. Both need revision of the pacing system. D : 85. A 45-year-old man with dilated cardiomyopathy is being considered for cardiac transplantation. Which of the following is generally deemed to be a contraindication to cardiac transplantation? A : Any previous cancer B : Pulmonary artery wedge pressure <20mmHg C : Creatinine clearance <50 ml min-1 D : Previous alcoholism E : Hypertension. Comment : Patients with heart failure are considered for cardiac transplantation when significant symptoms persist despite maximal medical therapy. Due to the shortage of donors clinical guidelines have been established highlighting patients most suitable for transplantation. Significant impairment of renal function is generally considered to be a contraindication, assuming this is not reversible. Patients with prior history of cancer may be considered if there is no evidence recurrence (> 5 years on from diagnosis of cancer). C : 86. A 55-year old man presents with an acute ST-segment elevation myocardial infarction. He receives treatment with thrombolysis and aspirin. Four hours later he has ventricular fibrillation; sinus rhythm is restored with a single 200-joule shock. Which of the following is the appropriate next treatment? A : Amiodarone B : Beta-blocker C : Dobutamine D : Implantable cardioverter defibrillator (ICD) E : Temporary wire . Comment : Ventricular fibrillation is not uncommon within the first 24 hours following myocardial infarction, and is treated with DC-shock. Beta-blockers confer early mortality and morbidity benefit following acute myocardial infarction, and hence should be prescribed routinely as long as no contra-indications exist. Dobutamine is indicated for patients with cardiogenic shock. A : 87. A 70-year old man with prior anterior myocardial infarction presents with syncope and ventricular tachycardia (VT). Angiography reveals occluded LAD and poor left ventricular function. Thallium (nuclear) imaging reveals a fixed anterior defect with no evidence of reversible ischaemia. What would be the optimal therapeutic strategy? A : Amiodarone B : CABG and amiodarone C : Implantable cardioverter defibrillator (ICD) D : Implantable cardioverter defibrillator and beta-blocker E : Percutaneous angioplasty to the LAD. Comment : This man has ischaemic cardiomyopathy, but no evidence of reversible ischaemia on functional (thallium) assessment. He has had symptomatic VT and therefore is at high risk of sudden death. Current evidence suggests that these patients gain prognostic benefit from implantation of an ICD. Beta-blockers have also been shown to independently improve prognosis (and symptoms) in patients with impaired left ventricular function. D : 88. A 79-year-old woman is admitted via A&E having been found on her floor at home. Her neighbours had not seen her for at least 48 hours. Her core temperature was 30?C. Her ECG traces might show? A : Atrioventricular block B : Bradycardia C : ST segment changes D : Ventricular fibrillation E : All of the above. Comment : The 'classical' feature of the ECG in hypothermia is the 'J' wave, (although this is not pathognomic of hypothermia). As the cooling becomes more profound, the QRS complex broadens. There may be ST elevation or depression and T wave inversion, which worsens with acidaemia. Second

Page 32: Masterclass Book Part 2

MOHAMMED IS-HAG 31

degree and third degree atrioventricular block may evolve. Ventricular fibrillation becomes more common with lower core temperatures. E : 89. A 65-year-old male presents to the Emergency Department with palpitations but no syncope. He has a previous history of anterior myocardial infarction and a hospital admission with pulmonary oedema. Which of the following would NOT support considering him for an implantable cardioverter defibrillator? A : Ejection fraction on echocardiography of 40% B : Four beats of ventricular tachycardia on CCU monitor C : Left bundle branch block D : Previous treatment with amiodarone E : Atrial fibrillation. Comment : Ejection fraction is probably the most important stratification for considering patients for an ICD on a background of coronary artery disease. Most studies (e.g. MADIT) required patients to have an ejection fraction <30 or 35%. Non-sustained VT is also an important predictor of risk. Left bundle branch block (LBBB) on its own is not a specific indicator for an ICD, but patients with very broad complexes are at greater risk. These patients may also be suitable for cardiac resynchronisation pacing. A : 90. A 78-year-old man is admitted with fast atrial fibrillation and chest pain. He is given digoxin in A&E which reduces his rate to 90/min. His pain settles and his troponin I is negative. He has a previous history of myocardial infarction and heart failure. Which of the following would be the optimal management subsequently? A : Continue digoxin and add aspirin B : DC cardiovert immediately C : Start amiodarone, aspirin and cardiovert after 6 weeks D : Continue digoxin and warfarinise E : Start amiodarone, warfarin and cardiovert after 6 weeks. Comment : A number of recent studies have suggested that restoration of sinus rhythm is not necessarily better than just controlling heart rate. However, this patient has impaired left ventricular function and would benefit from restoration of sinus rhythm. This is best achieved with amiodarone. If this fails, DC cardioversion would be appropriate. Aspirin is insufficient in this situation to prevent thromboembolism, especially if DC cardioversion is being considered. E : 91. A patient with primary pulmonary hypertension is found to have a mean right atrial pressure of 20 mmHg and a cardiac output of 1.5 L per min. The mean pulmonary artery pressure is 80 mmHg. There is no vasoactive response with intravenous prostacyclin. He is already on warfarin. Which is the most appropriate first-line treatment: A : Ace-inhibitor B : Oral calcium antagonist C : Heart-Lung transplantation D : Prostacyclin treatment E : None of the above. Comment : This patient has severe pulmonary hypertension with poor prognosis untreated. Calcium antagonists should only be given to those who show reactivity to vasodilator challenge and have adequate cardiac output (>2.0 L/min).Even those patients with severe disease who fail to show vasoreactivity on provocation testing appear to benefit from prostacyclin treatment so this is offered to all such patients initially. Only those who do not benefit from prostacyclin or continue to deteriorate are considered for transplantation. D : 92. A 58-year-old man, a smoker for many years despite repeated advice that he should stop, has chronic obstructive pulmonary disease that is increasingly limiting his exercise capacity. You wish to conduct a trial of steroid therapy. Which of the following is the correct way to do this and interpret the outcome? A : Give oral prednisolone, 60 mg daily for 4 weeks, regarding a clear statement of subjective

improvement by the patient as a positive response. B : Give oral prednisolone, 10 mg daily for 2 weeks, regarding a clear statement of subjective

improvement by the patient as a positive response if accompanied by a rise in FEV1. C : Give oral prednisolone, 60 mg daily for 4 weeks, regarding a clear statement of subjective

improvement by the patient as a positive response if accompanied by a rise in FEV1 of >10%. D : Give oral prednisolone, 30 mg daily for 2 weeks, regarding an increase in FEV1 of >10% and

>200 ml as a positive response. E : Give oral prednisolone, 30 mg daily for 2 weeks, regarding a clear statement of subjective

improvement by the patient as a positive response. Comment : Steroid challenge is indicated in chronic obstructive pulmonary disease (COPD) of more than moderate severity. Standard practice would be to give prednisolone 30 mg daily for 2 weeks, regarding an increase in FEV1 of >10% and >200 ml as a positive response. Given the non-specific effects and many side effects of steroids, it is crucial to demonstrate functional improvement: many patients with COPD have sustained severe complications of steroid treatment, e.g. vertebral fracture, without any evidence that the steroids were beneficial for their chest. D :

Page 33: Masterclass Book Part 2

MOHAMMED IS-HAG 32

93. A 60-year-old man is given streptokinase for an anterior myocardial infarction (MI) six hours after the onset of pain. Ninety minutes post-thrombolysis he continues to have ischaemic sounding chest pain with persistent anterior ST elevation. Which of the following is the established treatment in this situation: A : Intravenous heparin. B : Further thrombolysis with streptokinase C : Further thrombolysis with tPA D : Emergency angiography and target PTCA or bypass surgery E : None of the above. Comment : The best way to manage patients with failed thrombolysis has yet to be determined. Continued ischaemic pain and failure of >50%resolution of STelevation are the most accurate predictors of failed reperfusionThe UK REACT study aims to resolve the most appropriate treatment strategy in this clinical setting. This multicentre, prospective, randomised trial compares intravenous heparin, repeat thrombolysis with rPA, and rescue percutaneous transluminal coronary angioplasty (PTCA) in patients with evidence of failed thrombolysis still within 12 hours of presentation. E : 94. A 34-year-old male known to have AIDS presents with dyspnoea and cardiomegaly. His echo demonstrates a dilated poorly functioning left ventricle. Which of the following are not common cardiac manifestations of HIV/AIDS? A : Aortic dissection B : Pericardial effusion C : Endocarditis D : Myocarditis E : Pulmonary hypertension. Comment : Cardiac complications of HIV infection are common with myocardial, pericardial and endocardial involvement. This is a result of not only direct involvement as a result of HIV infection, but also the cardiotoxicity of the drugs used in the management of HIVinfected patients.Aortic dissection is not a common recognized complication. A: 95. A 43-year-old male with myotonic dystrophy is admitted with ventricular tachycardia. This requires DC cardioversion as he is hypotensive. Ultimately he has an implantable cardioverter defibrillator to protect him. Which of the following is NOT a recognized cardiac complication of myotonic dystrophy? A : Mitral valve prolapse B : Pulmonary stenosis C : Atrial flutter D : Ischaemic heart disease E : Diastolic heart failure. Comment : Endomyocardial biopsies frequently demonstrate non-specific changes such as fibrofatty infiltration, hypertrophy and myocarditis. All of these may lead to all of the above apart, from pulmonary stenosis. B : 96. A 68-year-old woman presents with 6 weeks of increasing breathlessness. She is in atrial fibrillation at rate of 120/min and has signs of mild/moderate cardiac failure. She gives no history of previous arrhythmia or to suggest ischaemic heart disease, but has had mild hypertension for at least 5 years. There are no cardiac murmurs. There are no ischaemic changes on the ECG. The chest radioagraph shows a normal sized heart and mild pulmonary oedema. She is not thyrotoxic. Select the best of these management options: A : Perform DC cardioversion within next 24 hours. B : Give sotalol with aim of achieving ‘chemical cardioversion’. C : Give diuretic and digoxin; perform DC cardioversion within next 24 hours. D : Give diuretic and digoxin; initiate anticoagulation; perform cardioversion in 6 weeks. E : Give diuretic; start full-dose intravenous heparin; perform DC cardioversion within next 24 hours. Comment : In a patient with a probable first episode of AF an attempt should be made to restore sinus rhythm. However, if AF has been present for >48 hrs – as seems very likely in this case – it is necessary to anticoagulate before cardioversion, unless it can be documented by trans-oesophageal echocardiography (TOE) that the left atrium is free of thrombus. If this were the case, then cardioversion within the next 24 hours would be the preferred management option, with warfarinisation for a month afterwards (which is a high risk period for thromboembolism). D : 97. A 50-year old man presents 3 months post mitral valve replacement (metallic) (MVR) with increasing shortness of breath, fever and weight loss. Clinically he is in pulmonary oedema. Transoesophageal echo (TOE) confirms severe paravalvular mitral regurgitation. The first blood culture is positive for Staph epidermidis. Which is the optimal therapeutic approach? A : Intravenous antibiotics for 4 weeks and then repeat TOE B : Oral antibiotics and angiotensin-converting enzyme (ACE) -inhibitor C : Re-do mitral valve replacement - bioprosthetic D : Re-do mitral valve replacement - metallic E : Withold antibiotics and repeat 3 sets of blood cultures.

Page 34: Masterclass Book Part 2

MOHAMMED IS-HAG 33

Comment : Clinically this man has an infected MVR, with severe paravalvular leak. Following surgery, the commonest infecting organisms (up to around 9 months) are coagulase negative staphylococci. Antibiotics alone will not cure the infection; the valve must be replaced again. In this case this should be with a metallic valve since a bioprosthetic valve would be likely to need replacing after 10-15 years due to degeneration thereby subjecting him to a high-risk third operation. Bioprosthetic and metallic valves have similar risk for subsequent endocarditis. D : 98. A 28-year-old woman who abuses heroin intravenously presents with a high fever. She has aortic incompetence. Which of the following organisms is most likely to be the cause of her infective endocarditis? A : Staphylococcus aureus B : Streptococcus viridans C : An Enterococcus D : Staphylococcus albus E : A gram-negative bacillus Comment : In intravenous drug abusers Staph aureus accounts for 50% of cases of infective endocarditis, with other causes being enterococci (10%), Strep viridans (5%), gram negative bacteria (5%), other bacteria (5%), fungi (5%) and polymicrobial infection (5%). Coagulase-negative staphylococci account for less than 5%. By contrast, in patients who do not abuse drugs intravenously (and do not have prosthetic valves) the frequency of infection is as follows: Strep viridans 35%, enterococci 25%, Staph aureus 23%, culture negative 5-10%, other identified organisms 5-10%. A : 99. Which of the following is a strong indication for thrombolysis for a patient with a typical history for myocardial infarction presenting within 12 hours of onset of symptoms? A : S-T segment elevation of 1 mm in one lead only B : left bundle branch block C : elevated myoglobin without ECG changes D : second degree heart block E : S-T segment depression >2 mm in two adjacent chest leads Comment : The greatest benefit following thrombolytic therapy is seen in patients whose electrocardiogram (ECG) shows S-T segment elevation or bundle branch block. S-T segment elevation of >1 mm in two adjacent limb leads or >2 mm in two adjacent chest leads is significant. Although myoglobin concentration may be elevated within 2-3 hours of myocardial infarction there are no data on the benefits of thrombolysis in patients with elevated levels of this or other cardiac enzymes in the absence of ECG changes. Heart block may occur after myocardial infarction, but should not influence the decision on thrombolysis. If transvenous pacing is required following thrombolytic therapy, an internal jugular, femoral or brachial route of access is preferred to the subclavian route. B : 100.

PLATE 10

Which of the following does this ECG show? A : pacing for complete heart block B : temporary pacing in a patient with an anterior MI C : antitachycardia pacing D : VVI pacing E : DDD pacing. Comment : A temporary pacing system in the context of a myocardial infarction is usually for complete heart block. You would, therefore, see pacing spikes before each QRS complex rather than each P wave.

Page 35: Masterclass Book Part 2

MOHAMMED IS-HAG 34

Temporary dual chamber systems are becoming increasingly used, however, in patients with reduced left ventricular function. VVI pacing is single chamber ventricular pacing with no sensing in the atrium. Antitachycardia pacing is seen in Implantable Cardioverter Defibrillators (ICDs) for the treatment of ventricular tachycardia. This is clearly not the case here. If the patient's underlying rhythm was complete heart block a pacing spike would be seen before each QRS complex. This patient has a dual chamber pacemaker in DDD mode. No P waves are sensed so there is a pacing spike before each P wave. However, each QRS is sensed and hence no paced QRS complexes are seen. A patient with a single chamber (AAI) pacemaker for sick sinus syndrome may have a similar ECG. E : 101. A 28-year-old lady is referred by her general practitioner with a history of recurrent syncopal episodes dating back to her early teens. She is not on any regular medication. Her GP has faxed through a 12-lead Electrocardiogram taken in his surgery shortly after her last blackout which shows polymorphic ventricular tachycardia. Which of the following statements are correct ? A : It is unlikely that she has a congenital long QT interval. B : Hypokalaemia is not a risk factor for polymorphic ventricular tacycardia. C : Female sex is not a risk factor for polymorphic ventricular tachycardia. D : Sotalol may cause torsade de pointes. E : The risk of torsade de pointe is a linear function of the QT interval. Comment : Although torsade de pointe can occur in many settings it is usually seen in patients with one of the congenital long QT syndromes or in association with drug therapy. Given that she is not on medication a congenital long QT syndrome needs to be considered. Some further points to be considered are: · Hypokalaemia increases the risk of torsade de pointe. · Female sex is a powerful predictor of the risk of torsade de pointes in patients with congenital and acquired long QT intervals. · Sotalol can prolong the QT interval and may cause torsade de pointe. · The risk of torsade de pointe is NOT a linear function of the QT interval. D : 102. A 48-year-old man is admitted with worsening angina. Which two of the following drugs are most likely to have precipitated his condition? A : Sildenafil B : Nicorandil C : Methotrexate D : Piroxicam E : Prazosin F : Sumatriptan G : Nifedipine H : Carbimazole I : Carbamazepine. Comment : Sildenafil (or viagra), a phosphodiesterase type 5 inhibitor, is associated with angina pectoris. Catastrophic hypotension, myocardial infarction and death have occured in patients taking viagra in conjunction with nitrates so this drug combination is absolutely contraindicated. Sumatriptan is a 5HT1 agonist used in acute migraine which causes vasospasm. It is contraindicated in patients with ischaemic heart disease as it can cause coronary vasospasm. Drugs which increase heart rate such as adrenergic drugs (salbutamol, isoprenaline) or rapid thyroid replacement, and drugs which cause a drop in blood pressure (e.g. misoprostil) may also worsen existing angina. A : F: 103. A 59-year-old man has an inferior myocardial infarct and is treated with streptokinase. Twenty-four hours later he suddenly becomes hypotensive. Which of the following diagnoses does NOT need to be considered? A : Mitral regurgitation B : Pulmonary embolus C : Ventricular septal defect D : Right ventricular infarction E : Hypotensive effect of streptokinase. Comment : Streptokinase can cause hypotension, but not after a delay of 24 hours. Diagnoses that require serious consideration are: · Arrhythmia · Re-infarction (of left or right ventricle) · Acute mitral regurgitation (due to dysfunction or rupture of papillary muscle) · Ventricular septal defect · Cardiac rupture with tamponade. Pulmonary embolism cannot be excluded, but is much less likely than any of the other three possible diagnoses listed. E :

Page 36: Masterclass Book Part 2

MOHAMMED IS-HAG 35

104. A 60-year-old man presents to A&E with a 1.5 hour history of chest pain, a heart rate of 68 / min, blood pressure 140/85, and no signs of cardiac failure. An ECG shows ST elevation in leads V1-V3. Which of the following statements is correct? A : Treatment with a thrombolytic is preferable to primary coronary angioplasty. B : Streptokinase is the thrombolytic agent of choice. C : If the ST elevation resolves spontaneously aortic dissection should be considered. D : There is a role for intravenous beta blocker therapy. E : Intravenous heparin is mandatory whatever treatment strategy is used. Comment : In the early 1990s, controversy still existed about the use of angioplasty for the treatment of AMI. A meta-analysis in 1997 of 10 randomized trials comparing thrombolytic therapy with primary angioplasty (n=2606 patients) demonstrated that primary angioplasty was superior to thrombolytic therapy in terms of in-hospital mortality, non-fatal reinfarction, long-term survival and a lower incidence of stroke. The largest trial (PAMI-1) showed greatest benefit in high risk patients (cardiogenic shock, elderly patients, anterior wall myocardial infarctions). The under use of primary angioplasty in the UK is largely due to lack of resources, although most regard it as the optimal treatment strategy. GUSTO-1 demonstrated that Streptokinase was inferior to tPA when used as an accelerated 90-minute bolus infusion along with iv heparin. An analysis of pooled data from 28 trials showed an average 28% mortality reduction with use of beta blockers in acute myocardial infarction. The ISIS-1 trial first used intravenous atenolol (5mg) in AMI with a favourable mortality benefit, but this treatment is widely underused in current practice. D : 105. A 75-year-old gentleman with known peripheral vascular disease and intermittent claudication is unsuitable for surgical intervention. It has been decided to commence him on cilostazol. Which of the following is true concerning cilostazol? A : It is licensed for use in patients with rest pain. B : Its main action is to activate phosphodiesterases. C : It is safe in patients with previous haemorrhagic stroke. D : It is metabolised by cytochrome P450 3A4. E : It can be used as a treatment for other vascular diseases e.g. IHD, TIAs. Comment : Cilostazol is a potent inhibitor of platelet phosphodiesterases leading to inhibition of platelet aggregation. It also has vasodilating actions. It is currently licensed for the management of patients with intermittent claudication without rest pain and with no signs of tissue necrosis. Studies have shown that its principal hepatic metabolism is by the cytochrome P450 3A4 isoenzyme. Its use is contraindicated in patients with known bleeding tendencies (e.g. active peptic ulcer disease, previous haemorrhagic stroke in the last 6 months). Trials show an improvement in time to initial pain on walking and maximal walking distance when compared to placebo. D : 106.

PLATE11 A 57-year-old male had a pacemaker implanted 7 years ago following a number of syncopal episodes. He is otherwise well. This is his ECG recorded at a pacemaker check. What pacing mode might his device be programmed to? A : VVI B : DDDR C : OVO D : VOO E : AAIR F : VDO G : VVD H : IVI I : CDD J : VVIR.

Page 37: Masterclass Book Part 2

MOHAMMED IS-HAG 36

Comment : His ECG clearly shows a pacing spike before each p wave. Therefore, there has to be pacing and sensing in the atrium, so the only possible pacing modes are AAIR or DDR. The former mode could be used for someone with sinus node abnormalities but intact atrioventricular conduction. There may be a pacing lead in the ventricle that is sensing the QRS complexes. On the basis of the ECG it is not possible to determine this as there are no paced QRS complexes. The first letter in the configuration denotes the paced chamber (0=none, A=atrium, V=ventricle, D=dual). The second letter represents the sensed chamber (0=none, A=atrium, V=ventricle, D=dual). The third letter represents the response to sensing (0=none, T=triggered, I=inhibited, D=dual). The final letter denotes programming modes, e.g. R=rate response. B : E : 107. A 17-year-old boy is brought to the doctor because his mother has noticed that his exercise capacity is decreasing and he ‘looks a funny colour’. She says that ‘he had a heart murmur when he was a baby’ and he is cyanosed and clubbed. The most likely diagnosis is: A : Tetralogy of Fallot B : Transposition of the Great Arteries C : Atrial septal defect D : Coarctation of the aorta E : Eisenmenger’s syndrome. Comment : Presentation with cyanosis of cardiac cause in adolescence is typical of Eisenmenger’s syndrome, the pathophysiology of which is as follows: a large congenital left-to-right shunt causes increased pulmonary blood flow, resulting in a rise in pulmonary vascular resistance and pulmonary hypertension. When the pulmonary vascular resistance exceeds the systemic vascular resistance the shunt is reversed, causing cyanosis. Virtually any large left-to-right shunt can lead to Eisenmenger’s syndrome, including atrial septal defect, ventricular septal defect, patent ductus arteriosus and Ebstein’s anomaly. Tetralogy of Fallot and Transposition of the Great Arteries present at birth or in infancy. E : 108. A 56-year-old man is admitted to hospital with crushing central chest pain. He is known to have had a myocardial infarction (MI) 3 months ago. Despite treatment with simvastatin in addition to other usual cardiac medication, his ECG shows that he has had a second MI. Which of the following statements is LEAST accurate? A : The overall effect of a drug depends on the net effect on polymorphisms within its metabolic pathways and genetic variation in target cells. B : There is an interaction between apo E genotype and lipid response to statin therapy. C : When a genetic variation affects more than 1% of the population it is termed a polymorphism. D : Lack of concordance with treatment is a possible explanation. E : Data from RCTs show that all MI patients should receive a statin. Comment : A, B and C are correct statements about pharmacogenetics, and D is also true. E looks sensible, but remember that RCTs show what happens in a group of patients in comparison with a group of controls. In any trial showing the survival advantage on a drug, some individuals will have a greater than average response, some less than average and some will be harmed. At present we cannot easily distinguish between these groups, so all patients are given a statin. E : 109. A 79-year old female with known aortic stenosis (aortic valve gradient 60 mmHg 2 years previously), presents with uncontrolled atrial fibrillation (140 bpm). She is dyspnoiec on exertion and on examination has an elevated jugular venous pressure and basal crepitations. Which two of the following are indicated as part of the initial treatment regimen? A : Angiotensin-converting-enzyme inhibitors B : Amiodarone C : Beta-blockers D : Digoxin E : Flecainide F : Intravenous diuretics G : Intravenous nitrates H : Valvuloplasty I : Verapamil J : Warfarin. Comment : Negative inotropic drugs and arterial vasodilators should be avoided if possible in patients with significant aortic stenosis. The definitive treatment for symptomatic aortic stenosis is valve replacement. In adults, valvuloplasty has produced very disappointing results. If patients develop atrial fibrillation, attempts should be made to restore sinus rhythm and thereby maintain the atrial component to ventricular filling. Amiodarone has a much lesser negative inotropic effect than class I agents. Diuretics are required in patients with evidence of fluid overload, but caution must be exercised to avoid hypovolaemia and thereby reduce left ventricular diastolic filling pressures (and therefore cardiac output). Anticoagulation would be required as prophylaxis in view of the atrial fibrillation, but one should aim to obtain immediate therapeutic anticoagulation with heparin (or LMWH). B : F:

Page 38: Masterclass Book Part 2

MOHAMMED IS-HAG 37

110.

A 57-year-old man presents with dyspnoea and orthopnea. He has an early diastolic murmur. See image for an illustration of a dilated aorta. Which of the following does not cause an aortopathy? A : Marfan's syndrome B : Ehlers Danlos syndrome C : Syphilis infection D : Atherosclerosis E : Duchenne's muscular dystrophy. Comment : The image is an aortogram demonstrating a dilated ascending thoracic aorta. This most commonly arises from the process of cystic medial degeneration from conditions such as Ehlers Danlo’s syndrome or Marfan’s syndrome. These aneurysms often involve the aortic root. Dilatation of the root causes aortic regurgitation. The treatment of choice is usually aortic root and valve replacement. Atherosclerosis is a common cause whilst syphilitic infection is becoming considerably less common. Duchenne’s muscular dystrophy is associated with a cardiomyopathy rather than a vasculitic process. E : 111.

A 35-year-old man presents with a history of collapse and transient loss of consciousness for a few seconds after running upstairs. He has a 10-year history of chest pains on exertion and presyncope. On examination he has an ejection systolic murmur at the left sternal edge with a normal aortic closure sound and no radiation to the carotids. His ECG demonstrated left ventricular hypertrophy with inferolateral ST segment/T wave changes. His echocardiogram is shown (see image). What is the most likely cause of his collapse? A : Bicuspid aortic valve

Page 39: Masterclass Book Part 2

MOHAMMED IS-HAG 38

B : Atrioseptal defect C : Acute myocardial infarction D : Hypertrophic obstructive cardiomyopathy E : Coarctation of the aorta. Comment : 20-30% of patients with hypertrophic cardiomyopathy have dynamic left ventricular outflow tract obstruction caused by systolic contact between the anterior mitral valve leaflet and the interventricular septum. Clinically the presence of left ventricular hypertrophy and an ejection systolic murmur at the left sternal edge with a normal aortic closure and no radiation to the neck should suggest the diagnosis, particularly if associated with mitral regurgitation. The murmur is characteristically increased by manoeuvres that reduce afterload or preload. The M-mode echocardiogram demonstrates a parasternal long-axis view of the left ventricular with asymmetric septal hypertrophy and contact between the mitral leaflet and septum in systole. D : 112.

PLATE12 The first response to the cardiac rhythm shown should be: A : call the cardiac arrest team. B : defibrillate with 200J. C : confirm cardiac arrest. D : adrenaline (epinephrine) 1mg IV. E : start cardiopulmonary resuscitation. Comment : Look at the patient and apply the “lime jelly” test. If they are sitting up in bed eating their lime jelly, the chances are that the rhythm results from artefact (shivering, Parkinson’s) or that it comes from another patient. If they are face down in their lime jelly, it is likely that they have arrested! C : 113. A 77-year-old man presents with sudden onset of weakness of his right arm on a background of a 3-week history of thoracolumbar backpain, weight loss, fever and night sweats. Blood tests: full blood count (FBC) 9.8 g/dL, white cell count (WCC) 12.0, platelets 450, erythrocyte sedimentation rate (ESR) 110 mm/hr, creatinine 180 micromol/L and microscopic haematuria. What is the most likely diagnosis? A : Spinal osteomyelitis B : Myeloma C : Infective endocarditis D : Mycobacterium tuberculosis infection E : Tertiary syphilis Comment : Infective endocarditis is the only diagnosis that would explain all the symptoms and results. Vertebral osteomyelitis/discitis and stroke are both recognised complications of infective endocarditis and either can be the presenting feature of the disease. C : 114. A 48-year-old man of African origin presents with blood pressure 200/110mmHg. Urinalysis is negative and fundoscopy shows AV nipping. Which of the following treatment options is most appropriate? A : Oral atenolol 50mg od if hypertension confirmed over 1–2 weeks B : Oral enalapril 10mg bd if hypertension confirmed over 1–2 weeks C : Urgent admission for control of accelerated hypertension D : Oral nifedipine capsules 20mg tds if hypertension confirmed over 1–2 weeks E : Oral nifedipine LA 30mg od if hypertension confirmed over 1–2 weeks Comment :

Page 40: Masterclass Book Part 2

MOHAMMED IS-HAG 39

The diagnosis of accelerated hypertension requires the finding of fundal haemorrhages and exudates, with or without papilloedema, as manifestations of fibrinoid necrosis. This patient has very high blood pressure but no evidence of accelerated hypertension.BTS guidelines suggest that the finding of blood pressure 200–219/110–119mmHg should be confirmed with repeat measurements after 1–2 weeks, then treated if still elevated.Black patients have low renin hypertension, therefore ACE inhibitors and beta-blockers as single agents do not lower blood pressure in this group.Calcium channel blockers and diuretics are effective agents.Nifedipine should be used as a long-acting preparation,not a short acting one.E : 115.

PLATE 13 This 63-year-old man had a short history of heart failure and was commenced on lisinopril and frusemide by his GP. He had a fatal cardiac arrest three weeks later. The post mortem showed a restrictive cardiomyopathy. The first image shows cardiac histology stained with 'haematoxylin and eosin', the second stained with a specialized stain. Which of the following is the most likely diagnosis? A : Hypertension B : Hypertrophic obstructive cardiomyopathy C : Ischaemic heart disease D : Amyloidosis E : Haemochromatosis. Comment : The first image shows an amorphous eosinophilic substance within the specimen. The second image shows the classical apple green birefringence seen when the Congo red stain is viewed with polarised light. This is diagnostic of amyloid. Note the staining in the vessels as well as in the cardiac tissue. The man had no evidence of myeloma or lymphoma, but this was found to be an AL type of amyloid.The cardiac arrest was probably due to a dysrhythmia associated with amyloid rather than hypokalaemia. Coronary vessels were healthy for his age. D : 116. A 70-year-old man presents with shortness of breath that has developed over a few days. He has no other symptoms. One week ago he returned on a transatlantic flight from a holiday in Florida. Which two investigations are most likely to reveal the diagnosis? A : ECG B : Ventilation-Perfusion isotope scanning C : Echocardiogram D : Serum troponin I E : Chest radiograph F : Spiral computed tomography scanning G : Chest radiograph taken in expiration H : Spirometry I : Arterial blood gases J : Ultrasound scanning of the leg veins. Comment : The most likely diagnosis is pulmonary embolism, the two most specific tests for this being ventilation-perfusion isotope scanning and spiral computed tomography scanning.Ventilation scans are obtained using krypon-81m, technegas or xenon-133 and perfusion scans with intravenous 99m-Tc-labelled macroaggregates of albumin. Scans are reported as being normal or of low, medium or high probability, but it is very important to remember that reports must ALWAYS be interpreted in the light of the clinical context, and that a low probability scan does NOT mean that ‘pulmonary embolism is excluded’.Spiral computed tomography can detect intravascular clot from the pulmonary trunk down to the segmental arteries. It is the investigation of choice for patients with pre-existing lung disease, which renders the interpretation of ventilation-perfusion scans difficult or impossible. B : F:

Page 41: Masterclass Book Part 2

MOHAMMED IS-HAG 40

117. A 39-year-old lady who who has a past history of treated hypertension is in her 3rd trimester of pregnancy and requires on-going anti-hypertensive treatment. Which anti-hypertensive would you definitely not prescribe? A : Hydralazine B : Labetalol C : Lisinopril D : Methyl-dopa E : Nifedipine. Comment : Evidence underpinning the choice of anti-hypertensive therapy in pregnancy is inadequate to make firm recommendations. There are no reports of serious effects with methyl-dopa following long and extensive use. Calcium antagonists, labetalol and hydralazine are commonly used, particularly for resistant hypertension in the third trimester. However, angiotensin-converting enzyme (ACE)-inhibitors should be avoided because they may cause oligohydramnios, renal failure and intra-uterine death. C : 118.

A 28-year-old woman was referred for the investigation of hypertension. Routine examination demonstrated that she had absent pulses in the left arm. She was a non-smoker. An arch aortogram was performed (see image). Which of the following statements is correct? A : She has coarctation of the aorta which is the likely cause of her hypertension. B : The selective picture shows a severe left subclavian stenosis most probably due to atherosclerosis C : The study is normal and she most likely has essential hypertension D : The selective picture shows a severe left subclavian stenosis most probably due to Takayasu's arteritis E : She has hypertension due to renovascular disease. Comment : Hypertension in the young always needs investigation. The physical findings in this woman were highly suggestive of a large vessel disease process.The selective angio picture does not show coarctation but does reveal critical stenosis of the left subclavian artery,which is most likely to be due to Takayasu's arteritis in this case.Atherosclerosis would be unusual in a young non-smoker.Hypertension is probably driven by renal ischaemia due to either direct renal vessel involvement or indirectly by aortic narrowing (not seen on these pictures). Raised erythrocyte sedimentation rate (ESR) is typical of the active inflammatory phase of this illness; in Japan there have been reports of linkage with tuberculosis - steroids and other immunosuppressants can be effective. D : 119. A 20-year-old female student presents with central chest pain after four days of a ‘flu-like illness. The most likely diagnosis is: A : Acute viral pericarditis B : Gastro-oesophageal reflux C : Acute myocardial infarction D : Systemic lupus erythematosus E : Pulmonary embolism. Comment : The combination of a young patient and a ‘flu-like illness make acute viral pericarditis the most likely diagnosis in this case. The chest pain of pericarditis can be indistinguishable from that of myocardial infarction, excepting that sitting forward often eases it. The key physical sign to elicit would be a pericardial rub, and the key initial investigation would be the ECG, looking for widespread ST segment elevation, concave upwards. A : 120. Which of the following is not associated with cocaine abuse? A : Hypotension B : Contraction band necrosis C : Dilated cardiomyopathy D : Acceleration of atherosclerosis E : Aortic dissection.

Page 42: Masterclass Book Part 2

MOHAMMED IS-HAG 41

Comment : Cocaine is associated with both acute and long-term cardiovascular disorders. Most commonly it is associated with myocardial ischaemia secondary to coronary spasm. Coronary spasm usually responds to nitrates, calcium antagonists or alpha blockers. Beta-blockers should be used with caution. A : 121. A 76-year-old lady is referred to the outpatient clinic with a 3-week history of new onset angina. In clinic she complains of chest pain at rest and is found on examination to have bilateral basal crepitations, a third heart sound and a pansystolic murmur. A 12 lead electrocardiogram shows ST depression of > 2 mm in her anterior leads. Which of the following are correct? A : There is a higher risk of death with unstable angina than an acute myocardial infarction (MI) B : Short episodes of chest pain are markers of high risk unstable angina C : The finding of congestive cardiac failure has no prognostic signifigance D : Only ST depression of 2mm or more is of prognostic signifigance E : A new or worsening MR murmur is a high risk factor for death from unstable angina. Comment : In patients with unstable angina the risk of death is lower than with an acute MI. Prolonged episodes of severe chest pain are important risk factors of high-risk unstable angina Any evidence on physical examination of acute congestive heart failure or a new or worsening MR murmur place the patient at increased risk of death. Dynamic shifts in ST segment depression or elevation of >1mm or T wave inversions that resolve when symptoms are relieved are of prognostic significance. E : 122. A 70 year old woman presents with 8 hours of chest pain. Her pulse rate is 40/minute and blood pressure 105/85. The ECG shows complete heart block, ST segment elevation and Q waves in leads II, III and AVF. Which of the following statements is correct? A : Atropine should be given immediately. B : An isoprenaline infusion should be set up immediately. C : Thrombolysis should be given immediately. D : Thrombolysis should be avoided because she has completed her myocardial infarction. E : Thrombolysis should be avoided because she may require a temporary pacing wire. Comment : The top priority is to achieve myocardial reperfusion. The presence of chest pain and ST segment elevation, despite Q waves, on ECG indicate that thrombolysis is needed immediately. C : 123. A 37-year-old man presents with recurrent episode of atrial fibrillation. He is having approximately three episodes each week, lasting for up to 5 hours. He has previously been treated with flecainide, sotalol and amiodarone. Which of the following is appropriate in his management? A : Consider digoxin B : Consider AV node ablation and pacemaker implantation C : Consider him for an implantable cardioverter defibrillator (ICD) D : Consider radiofrequency ablation to isolate his pulmonary veins E : None of the above. Comment : He is spending a significant proportion of his time in atrial fibrillation (high atrial fibrillation burden). However, it is paroxysmal and therefore digoxin is inappropriate and may increase the number of episodes.AV node ablation should not be considered in a young patient with paroxysmal symptoms. Generally this is reserved for patients in chronic atrial fibrillation resistant to drug treatment. An atrial defibrillator may be appropriate but certainly not an ICD used to treat malignant ventricular arrhythmias.Pulmonary vein isolation for atrial fibrillation is an evolving technique and may be appropriate for this patient. D : 124. In a patient with resistant hypertension, which of the following would increase the degree of clinical suspicion for significant renovascular disease as aetiology? A : Arterial bruits B : Co-morbidity with diabetes C : Concentric left ventricular hypertrophy D : Grade III hypertensive change on fundoscopy E : Palpable kidney. Comment : The highest degree of clinical suspicion of renovascular disease will occur in patients with arterial bruits, discrepancy in renal size of >1.5 cm (on ultrasound) and co-morbid vascular disease. If a patient has none of these then renovascular disease is highly unlikely, Evidence of end-organ damage is not helpful in differentiating aetiology. A : 125. A 60-year-old woman develops hypotension and a new systolic murmur 36 hours after being successfully thrombolysed for an anterior myocardial infarction. Which of the following statements is correct? A : Acute mitral incompetence due to rupture of the posterior papillary muscle is the most likely diagnosis. B : Acute mitral incompetence due to rupture of the anterior papillary muscle is the most likely diagnosis.. C : A basal ventricular septal defect (VSD) is the most likely diagnosis. D : An apical ventricular septal defect is the most likely diagnosis. E : The systolic murmur is likely to be due to mitral valve prolapse.

Page 43: Masterclass Book Part 2

MOHAMMED IS-HAG 42

Comment : Anterior myocardial infarction is typically associated with apical VSDs whilst inferior myocardial infarctions are more commonly associated with basal VSD or posterior papillary muscle rupture. After confirmation of diagnosis by echocardiography or right heart catheter, which reveals a step up in oxygenation at ventricular level, urgent referral to a surgical centre is required, the outlook without surgical repair being extremely poor.Anterior myocardial infarction associated with apical VSD carries a better surgical outlook than inferior myocardial infarction associated with basal VSD. D : 126. A 72-year-old man on the coronary care unit has had a temporary transvenous pacing wire inserted. He suddenly becomes symptomatic with pre-syncope and a palpable pulse of 28 bpm. The ECG monitor shows pacing spikes that are not related to the QRS complexes. Which of the following actions is appropriate? A : Check that the pacemaker is switched on B : Check the pacemaker leads are connected C : Arrange for an urgent permanent pacemaker D : Start chest compressions E : Increase the pacing voltage to a maximum. Comment : If a patient with a temporary pacemaker becomes symptomatic it is important to check that the pacemaker is switched on and that the leads are still connected. The presence of pacemaker spikes on the ECG confirms that these are okay. It is likely that the tip of the pacing wire has migrated and that the threshold has increased. This may be overcome by increasing the voltage. If this is successful, the pacemaker wire will need to be repositioned. If inceasing the voltage is unsuccessful, external pacing should be substituted. E : 127. A 35-year-old lady with a long history of migraine presented with a mild right hemiparesis. She made a full recovery after 4 days but subsequent investigation revealed a patent foramen ovale (PFO). Paradoxical thromboembolism through a PFO is an important cause of stroke. Which of the following statements regarding PFO and closure of PFO is incorrect? A : Percutaneous closure of PFO is safe with a low incidence of recurrent neurological events. B : Divers with decompression sickness may warrant PFO closure. C : Spontaneous right to left shunting through a PFO indicates high risk. D : Closure of PFO in patients with recurrent migraine with aura should be considered. E : A three to five-fold higher prevalence of PFO is noted in patients with cryptogenic stroke. Comment : PFO occurs in approximately 20-30% of the population. Percutaneous closure of a PFO using a variety of closure devices is now an accepted practice in certain populations and in certain centres. All of the above are correct apart from D. A number of studies have suggested a correlation between migraine and PFO. However, there is currently insufficient evidence to support closure in these patients. D : 128. You find a middle aged man on a path in a park. He has no pulse and is not breathing. Which is the appropriate next step: A : Give two rescue breaths and initiate CPR at a ratio of 5 compressions to 2 breaths B : Give two rescue breaths and initiate CPR at a ratio of 15 compressions to 2 breaths C : Give a precordial thump D : Give two rescue breaths and go for help. E : Go to call 999 (emergency services) immediately. Comment : The guidelines on basic life support identify the importance of early access to defibrillation in cardiac arrest. They therefore suggest that no CPR is commenced until a call for emergency services has been made and the potential for early defibrillation is made possible. Once CPR begins the ratio of compressions to ventilations is 15 to 2. A precordial thump is not indicated in the unwitnessed collapse. E : 129. A 43-year-old woman with pulmonary hypertension attends clinic and asks to be prescribed bosentan. Which of the following is true? A : It is licensed for use in all patients with pulmonary hypertension. B : It inhibits the effects of endothelin-2 (ET-2). C : It binds to both endothelin A (ET-A) and endothelin B (ET-B) receptors. D : It is excreted primarily unchanged in the urine. E : It is not effective in patients with scleroderma. Comment : Bosentan is an antagonist of endothelin-1 binding to ET-A and ET-B receptors. It is licensed for the treatment of 'pulmonary arterial hypertension (PAH) to improve exercise capacity and symptoms in patients with grade III functional status'. It has been shown to be effective in patients with primary PAH and in those with PAH secondary to scleroderma. It is excreted in bile following metabolism by the cytochrome P450 enzymes and this is a potential source of interaction with drugs metabolised by the same isoenzyme (e.g. glibenclamide, ritonavir, ketoconazole, ciclosporin and itraconazole). C:

Page 44: Masterclass Book Part 2

MOHAMMED IS-HAG 43

130.

PLATE 14 A 62-year-old man presents with chest pain. What two features does his ECG show (see image)? A : Posterior myocardial infarction B : Atrial fibrillation C : Anterior myocardial infarction D : Right axis deviation E : Unstable angina F : Ventricular ectopic beats G : Inferior myocardial infarction H : Atrial ectopic beats I : Left axis deviation J : Left bundle branch block. Comment : This ECG shows the classical appearances of acute myocardial infarction with ST segment elevation. This is seen in leads V1-V5, indicating that the infarct is anterior. The rhythm strip shows a number of ectopics, of which there is a compensatory pause, indicating that they are ventricular in origin.C:F: 131.

This patient presented acutely unwell with the following ECG (see image). What is the correct diagnosis? A : Hyperkalaemia B : Ventricular tachycardia C : Acute pericarditis D : Acute anterolateral myocardial infarction E : Digoxin toxicity.

Page 45: Masterclass Book Part 2

MOHAMMED IS-HAG 44

Comment : The presence of tall, peaked T waves, flattened P waves, prolonged PR interval and wide QRS complexes are pathognomonic of hyperkalaemia. Give IV calcium immediately (10mls, 10% calcium gluconate) and call the renal team. A: 132.

A 30-year-old male with a heart murmur and breathlessness on effort is referred for transthoracic echocardiogram (see image). Which of the following is true of his condition? A : Sudden death is very common. B : Exercise testing is contraindicated. C : Atenolol may help his symptoms. D : Frusemide will help his shortness of breath. E : The ECG is always abnormal. F : It is very rare in elderly patients over 75 years of age. G : The condition can be confidently diagnosed in patients over 10 years pf age. H : Ischaemia may occur with a normal coronary arteriogram. I : Endocarditis prophylaxis is not required. J : Prophylactic amioderone is mandatory to prevent sudden death. Comment : The above patient has hypertrophic cardiomyopathy with marked asymmetrical septal hypertrophy and a resting outflow tract gradient of 55mmHg.The ECG may be normal in 25% cases and morphological expression of the disease may not be completed until the end of puberty.Twenty -five per cent of all cases occur in those over 75.Sudden death occurs in 1% cases and those considered high risk should be referred for an implantable defibrillator.Exercise testing is extremely useful at identifying those patients with dynamic outflow tract gradients who would benefit from more aggressive therapy.Surgical myectomy or percutaneous alcohol septal ablation is reserved for those with high (> 50mmHg) resting gradients with refractory symptoms despite optimal medical treatment. C:H: 133. A 72-year-old Caucasian woman in your outpatient clinic has uncomplicated essential hypertension. Her blood pressure is 162/102mmHg despite optimization of non-pharmacological therapy. Which one of the following treatments would you choose as the first-line agent for her? A : Atenolol 50 od B : Bendrofluazide 2.5mg od C : Bendrofluazide 5mg od D : Enalapril 5mg od E : Ramipril 2.5mg od. Comment : Hypertension is particularly common in those aged above 60 not least because of the steady rise in systolic blood pressure with age. These patients are at a high absolute risk of cardiovascular complications. Furthermore, anti-hypertensive treatment may also reduce incidence of heart failure and possibly dementia. Non-pharmacological therapy for hypertension should always be optimised prior to commencement of medication, whenever possible. Low-dose diuretics are accepted as the first-line treatment for hypertension in the elderly and appear to confer greater benefit than Beta-adrenergic receptor antagonists in this subgroup. 1. Treatment of isolated systolic hypertension in the elderly with the long-acting calcium channel blocker, nitrendipine, has been shown to reduce stroke and cardiovascular outcome. Therefore, calcium channel blockers may be suitable when diurectics are not tolerated, ineffective or contra-indicated.2 B:

Page 46: Masterclass Book Part 2

MOHAMMED IS-HAG 45

134.

A 35-year-old man is admitted with chest pain. His ECG and cardiac enzymes are normal. Because a murmur is heard he has a transesophageal echocardiography (TOE) (see image). Which of the following is true concerning the disorder shown? AoA is ascending aorta, AoD is descending aorta. A : Medical treatment is the best long term option. B : The most likely cause of the murmur is mitral regurgitation. C : Methyldopa is contraindicated in the initial management. D : The diagnosis can usually be made as easily with transthoracic echo. E : CT scan of the chest is usually superior to TOE in making the diagnosis. F : Pregnancy is protective from condition. G : Atherosclerosis is the most likely underlying cause of the condition in this case. H : Untreated overall prognosis is good. I : Paraplegia is a recognized complication. J : Persistent abdominal pain may be a worrying feature. Comment : The TOE images demonstrate a Type A aortic dissection with an intimal flap seen in the ascending and descending aorta. The most likely cause for his murmur is aortic regurgitation or co-existent bicuspid aortic valve. Untreated surgically, mortality is high so early referral to a cardiothoracic unit is crucial. Type B dissections involving the descending aorta only are usually treated medically in the first instance as the complications of surgery are high.TOE,MRI and CTchest have similar sensitivities and specificities in making the diagnosis.Methyldopa may be used in the initial management to control blood pressure. When the descending aorta is involved persistent abdominal pain may indicate involvement of the mesenteric arteries. Pregnancy is a rare but recognized cause. In young patients atherosclerotic disease is rarely the cause. I:J: 135.

PLATE 15 A 35-year-old man is referred for a transthoracic echocardiogram (see image) after a murmur is heard by his GP. Which of the following physical signs would be expected.

Page 47: Masterclass Book Part 2

MOHAMMED IS-HAG 46

A : An immediate diastolic murmur B : A machinery murmur C : An ejection click D : A loud first heart sound E : A late systolic murmur F : A fourth heart sound G : Pulsus alternans H : A wide pulse pressure I : Paradoxical splitting of the second heart sound J : Kussmaul's sign Comment : This patient has mitral valve prolapse (MVP) of the posterior leaflet with mitral regurgitation. Typical findings include a mid systolic click followed by a late apical systolic murmur. When the mitral regurgitation becomes severe the murmur can become pansystolic with a third heart sound and a displaced hyperdynamic apex. The first heart sound is usually soft. Atrial fibrillation may also occur.C:E: 136.

This is a M-Mode trace from a transthoracic echocardiogram (see image) taken on a 60-year-old man presenting with shortness of breath for 6 months. A subsequent angiogram showed normal coronary arteries. He is in sinus rhythm. Which of the following treatments will improve his prognosis long term? A : Atenolol B : Carvedilol C : Pindolol D : Digoxin E : Dobutamine F : Milrinone G : Frusemide H : Bendrofluazide I : Spironolactone J : Amlodipine. Comment : The M-mode trace demonstrates a dilated cardiomyopathy with significantly reduced ejection fraction. There is also a small pericardial effusion seen below the posterior wall of the left ventricle. Carvedilol, bisoprolol, bucindolol and metoprolol are the only beta-blockers thus far which improve long-term prognosis. Inotropes long term are associated with a worse prognosis.Digoxin has little impact on mortality but may improve quality of life and reduce hospital readmission rates in end stage disease.Amlodipine has no impact on mortality.B:I: 137. A 78-year-old man with known ischaemic heart disease and congestive cardiac failure is admitted as an emergency with severe pulmonary oedema. He is sitting with his legs over the side of the casualty trolley and gasping. He has smoked 20 cigarettes per day for many years. What is the first treatment he should receive? A : Frusemide 40 mg intravenously B : High flow oxygen via reservoir bag C : 35% oxygen D : Diamorphine 2.5 mg intravenously, with anti-emetic E : Isosorbide dinitrate by intravenous infusion at dose titrated against blood pressure.

Page 48: Masterclass Book Part 2

MOHAMMED IS-HAG 47

Comment : Unless it is definitely known that a patient has a tendency to retain carbon dioxide, all patients with severe breathlessness should be given high flow oxygen via a reservoir bag once it has been established that their airway is clear.After the patient has been started on high flow oxygen, give frusemide 40-80 mg IV and diamorphine 2.5-5 mg IV. If matters do not improve consider isosorbide dinitrate 2-10 mg/hr IV. If matters worsen, then call the ICU sooner rather than later (assuming that the man’s condition prior to this acute presentation means that this is appropriate).Concurrently try to establish a cause for the acute deterioration: has he had a myocardial infarction? If so, would he benefit from thrombolysis? B: 138. A 22-year-old man presents to A&E claiming to have taken a large quantity of paracetamol 24 hours previously. He washed the tablets down with vodka. Which of the following statements are correct? A : Measuring paracetamol levels at 24 hours is of no use. B : N-acetyl cysteine should be witheld until plasma paracetamol levels are known. C : The prognostic accuracy of the treatment nomogram is less certain at 24 hours post-ingestion. D : Methionine can be given if the patient is intolerant of N-acetyl cysteine (NAC). E : Clinical symptoms may occur > 24 hours post-ingestion. F : If the urea and electrolytes are normal, the patient can be sent home. G : Even at 24 hours, activated charcoal should be given as gastric transit time is prolonged by

paracetamol. H : The measurement of alanine transferase is of prognostic value. I : If he drank more than two units of alcohol with the overdose, he is at greater risk. J : If he is obese, he is at greater risk from the effects of the overdose. Comment : Levels should always be measured. If paracetamol is still present it may suggest that the overdose was taken over a period of time making interpretation of the nomogram very difficult. If a clinically significant quantity of paracetamol has been taken, start NAC whilst waiting for the results - it can always be stopped (in most hospitals it takes a significant amount of time for blood tests to be processed and the results retrieved). Most reactions to NAC are mild and can be overcome by giving it more slowly. For patients who are truly intolerant, methionine can be given up to 12 hours after ingestion. Clotting, liver function and U&E should all be checked before consideration is made as to whether the patient could go home. The best prognostic indicator is the INR. Patients who are anorexic for any reason, who drink significantly (>21 units/wk for men, >14 units/wk for women) or who are on enzyme-inducing drugs are at greater risk from paracetamol overdose. C:E: 139. A 72-year-old female patient has been admitted acutely to the ward via the Emergency Department. She was found collapsed at home, semiconscious. Her Glasgow coma score (GCS) is currently 10 and she is receiving oxygen via a venturi mask. You notice that she has stridor with marked inspiratory effort. Which of the following statements regarding basic airway management are correct? A : The length of an oropharyngeal airway should correspond to the length of the patient's little

finger. B : Oropharyngeal airways are contraindicated in patients with a base of skull fracture. C : Patients with preserved laryngeal reflexes will not tolerate a nasopharyngeal tube. D : Insertion of an oropharyngeal airway can trigger laryngospasm. E : A nasopharyngeal airway cannot be used if the nose is fractured. F : Both nasopharyngeal and oropharyngeal airways can provide a definitive airway for comatose

patients. G : Nasopharyngeal and oropharyngeal airways should be used when a jaw thrust is contraindicated. H : The diameter of a nasopharyngeal tube should be similar to the diameter of the patient's little

finger. I : A head-tilt chin lift produces less neck movement than a jaw thrust manoeuvre. J : Vomitus should be removed from the airway using a blind finger-sweep. Comment : Basic airway management involves initially opening the airway. This should be done with a head-tilt chin lift unless there is suggestion of a neck injury when a jaw-thrust manoeuvre is preferred. Material in the oropharynx should be removed under direct vision.Oropharyngeal (OPA) and nasopharyngeal (NPA) airways are useful adjuncts but do not provide a definitive airway for unconscious patients. A quick way to size a NPA is to choose one with an external diameter similar to the patient’s little finger. NPAs are contraindicated in base of skull fractures. If there is a nasal fracture, the most patent nostril should be chosen. NPAs are generally better tolerated than OPAs and can be used in patients where the laryngeal reflexes are preserved.OPAs should be sized with the length corresponding to the distance from the angle of the patient's mouth to the angle of the jaw. Inserting OPAs can trigger vomiting and laryngospasm.D:H:

Page 49: Masterclass Book Part 2

MOHAMMED IS-HAG 48

140. A 67-year-old man is on coronary care following an inferior myocardial infarct. He has had a temporary transvenous pacemaker inserted for Mobitz type II heart block. He suddenly becomes light-headed with chest pain. The cardiac monitor shows complete heart block with a rate of 20 bpm. What should your first two actions be? A : Call the cardiac arrest team B : Ask him to cough repeatedly C : Give a precordial thump D : Check the lead connections on the pacemaker E : Institute external pacing F : Obtain a 12 lead electrocardiogram G : Increase the pacemaker voltage to maximum H : Start an isoprenaline infusion I : Give streptokinase 1.5 MU intravenously J : Give morphine 5-10 mg intravenously. Comment : The patient's symptoms result from his bradycardia. The pacemaker is failing to capture. The first thing to do is to check the leads. The pacemaker voltage should then be increased to maximum which may enable it to capture. The threshold for pacing increases with time and should be checked at least daily. If capture occurs, arrangements should be made to reposition the pacemaker lead. If it does not, external pacing should be instituted.Cough and percussion pacing are emergency measures that occasionally work.D:G: 141. Isoprenaline is rarely found even on coronary care units nowadays. A 48-year-old man presents with 40 minutes of cardiac chest pain. Which two of the following ECG criteria are indications for thrombolysis? A : 2 mm or more elevation of the ST segment in any one standard ECG lead B : Atrial fibrillation C : 2 mm or more ST segment depression in two or more standard ECG leads D : 2 mm or more elevation of ST segments in two or more contiguous praecordial leads E : New right bundle branch block F : Ventricular tachycardia G : 1 mm or more elevation of ST segments in two or more standard ECG leads H : 2 mm or more elevation of ST segments in any two ECG leads I : 1 mm or more elevation of ST segments in any two ECG leads J : 1 mm or more elevation of ST segments in two or more contiguous praecordial leads. Comment : The other recognised ECG criterion for thrombolysis is new left bundle branch block. D:G: 142. A 46-year-old female has been referred by her GP with a two-year history of lethargy, an episode of atrial fibrillation and a murmur. On examination she is in sinus rhythm, normotensive with normal first heart sound. A transthoracic echo has demonstrated a significant left to right shunt. Which of the following is the most likely diagnosis? A : Patent ducutus arteriosus B : Patent foramen ovale C : Ostium secundum atrial septal defect D : Ostium primum atrial septal defect E : Ventricular septal defect. Comment : Secundum atrial septal defects are often not identified in childhood and can present with non specific symptoms such as lethargy. The first heart sound is often split but equally may be normal. The definitive investigation is transthoracic echocardiography. This will demonstrate first of all the anatomy and secondly using doppler indicate any significant shunting.Ostium secundum defects are more midseptal and by far more common than primum defects which are more inferiorly displaced towards the AV valve. Primum defects are often associated with anomalies of the mitral and/or tricuspid valve, and as such patients more commonly present in childhood.It is clear from the clinical picture that this is not a patent ductus arteriosus. Patent foramen ovale (PFO) is a common condition and generally benign. There may be some left to right flow on doppler imaging but this will not be significant. The harsh pansystolic murmur of a ventricular septal defect is usually obvious unless the defect is very large. In this case the patient would be likely to have more pronounced symptoms.c: 143. A patient with documented systolic dysfunction has permanent atrial fibrillation. His resting heart rate is 100 and systolic blood pressure 120 mmHg; there is no evidence of fluid retention. Creatinine is normal and he is already receiving appropriate doses of frusemide and angiotensin-converting-enzyme inhibitor. Which of the following would be the next therapy? A : Amiodarone B : Carvidolol C : DC cardioversion D : Digoxin E : Metoprolol.

Page 50: Masterclass Book Part 2

MOHAMMED IS-HAG 49

Comment : Beta-blockers have prognostic and symptomatic benefit in heart failure. In the UK only carvedilol and bisoprolol are licensed for this use.Digoxin can improve symptoms in severe heart failure. Whilst it may well slow resting heart rate in atrial fibrillation it has less benefit on exercise related increases in rate. B: 144. You see a patient with poorly-controlled atrial fibrillation and decide to start him on amiodarone. There is a potentially serious drug interaction whereby amiodarone increases the plasma concentrations of the following two drugs: A : Aspirin B : Verapamil C : Ramipril D : Nifedipine E : Ranitidine F : Bendrofluazide G : Digoxin H : Pravastatin I : Glibenclamide J : Spironolactone. Comment : Amiodarone inhibits CYP 3A4 and will increase the plasma concentrations of drugs such as ciclosporin, phenytoin, verapamil and warfarin. This may cause bradycardia, AV block and myocardial depression if verapamil and amiodarone are used together in treating atrial fibrillation, thus great care is needed in adjusting the dose. There is a similar interaction between digoxin and amiodarone, and it is recommended that the digoxin dose be halved in those taking both agents.B:G: 145. This patient has permanent atrial fibrillation and as such DC cardioversion will not, by definition, restore sinus rhythm. The following results were found at cardiac catheterization in a 45-year-old man who has been complaining of increasing breathlessness. What is the diagnosis?

Pressure(mmHg) Saturations (%)

RA 15 67

RV 40/7 67

PA 43/16 68

PCWP (mean) 26

<LV< td> 110/8 96

Ao 107/68 96

A : Aortic stenosis B : Ventricular septal defect (VSD) C : Aortic Regurgitation D : Mitral stenosis E : Pulmonary stenosis. Comment : There is no significant pressure gradient across the aortic or pulmonary valves thereby excluding aortic or pulmonary stenosis. There is no change in saturation from RA to RV, thereby excluding the diagnosis of a VSD with left to right shunting. We can, however, see the increase in mean capillary wedge pressure (providing a reflection of left atrial pressure) which is greater than left ventricular diastolic pressure (8 mmHg) in keeping with mitral stenosis. There is also an associated mild/moderate elevation in pulmonary pressures.D: 146. The degree of obstruction often progresses after birth and therefore children may present at a later date (although usually within the first two years). Which of the following statements about the unwanted effects of amiodarone is NOT true? A : Most patients develop corneal microdeposits within 1 month of commencing treatment. B : Thyroid dysfunction is more common in women, older patients and those who already have

thyroid problems. C : High doses of amiodarone are associated with tremor, ataxia and peripheral neuropathy. D : Patients should be monitored for early signs of pulmonary toxicity, as this is reversible if the

amiodarone is discontinued early enough. E : Amiodarone is often implicated in torsade de pointes. Comment : Amiodarone is useful in maintaining sinus rhythm in patients with paroxysmal atrial fibrillation, or who have remained in AF after cardioversion. It may also be used to prevent ventricular tachycardia. However, long-term use is associated with several unwanted effects. Some of these can be reduced by using a lower dose i.e. 100-200mg daily,for example the neurological effects such as tremor, ataxia and peripheral neuropathy are less common at lower doses. Others, such as early pulmonary toxicity and skin discoloration, are reversible if the drug is stopped, but remember that it has a very long half life. Amiodarone itself is not a frequent cause of torsade de pointes, unless it is coprescribed with other drugs which also prolong the QT interval, e.g. sotalol, quinidine and disopyramide. E:

Page 51: Masterclass Book Part 2

MOHAMMED IS-HAG 50

147. A 28-year-old woman is admitted with left sided pleuritic chest pain that began suddenly six hours ago. She is very anxious, with a respiratory rate of 18/min and a pulse rate of 90/min. The following would reassure you that she has not had a pulmonary embolus: A : Entirely normal physical examination (aside from RR 18/min and PR 90/min) B : None of the other answers is correct C : Oxygen saturation of 99% on air D : Slight tenderness elicited by pressing on the left side of the chest E : Her admission that she is anxious because a friend of hers had a pulmonary embolus recently. Comment : The common causes of pleuritic chest pain are pulmonary embolism, pneumonia and musculoskeletal pain. The safe and correct management is to assume pulmonary embolism unless the history or examination findings enable another positive diagnosis to be made, and this woman has had a pulmonary embolus until proven otherwise.B: 148. A history of local trauma or definite localised tenderness may enable a confident diagnosis of musculoskeletal pain to be made, but slight tenderness is not uncommon in pulmonary embolism, when examination can be normal, as can oxygen saturation. Which of the following statements is true concerning pregnancy and cardiovascular disease? A : Severe mitral stenosis is usually well tolerated. B : In valve disease antibiotic prophylaxis is recommended for a normal vaginal delivery. C : Caesarean section is recommended with hypertrophic cardiomyopathy. D : Epidural anaesthesia is recommended with hypertrophic cardiomyopathy. E : Peripartum cardiomyopathy is usually seen after delivery. Comment : Severe mitral stenosis is often poorly tolerated in pregnancy and in suitable symptomatic patients balloon valvuloplasty can be performed with minimal radiation. In those with valvular disease antibiotic prophylaxis is recommended for caesarean section or instrumented deliveries only. E: 149. Patients with hypertrophic cardiomyopathy can usually have a safe vaginal delivery and caesarean section should only be offered on obstetric grounds. Epidural anaesthesia should be avoided in these women due to poorly tolerated vasodilation. In a study of patients with myocardial infarction, the death rate of those given aspirin is 8%, compared with 10% in those not given aspirin. This means that: A : the relative risk of death after myocardial infarction is 1.25 in those given aspirin B : the relative risk reduction produced by aspirin is 2% C : the number needed to treat with aspirin to prevent one death is 8 D : the number needed to treat with aspirin to prevent one death is 10 E : the absolute risk reduction produced by aspirin is 2%. Comment : Absolute risk reduction (or increase) = (Risk in group 1) minus (Risk in group 2), which is 2% in this example.Relative risk is the difference of outcome in one group compared to another = (Risk in group 1) divided by (Risk in group 2). In this case aspirin reduced relative risk by 20%.E: 150. The Number Needed to Treat = 1 divided by (Absolute Risk Reduction), which is 1/0.02 or 50 in this example. When considering patients with Tetralogy of Fallot, which of the following is the most important determinant of the severity of clinical presentation? A : Size of the ventricular septal defect B : Degree of right ventricular outflow tract obstruction C : Size of the right atrium D : Severity of right ventricular hypertrophy E : The presence of an over-riding aorta. Comment : In Tetralogy of Fallot (right ventricular outflow tract obstruction, ventricular septal defect, right ventriuclar hypertrophy, over-riding aorta) the severity of clincial presentation is dependent upon the degree of obstruction to the right ventricular outflow tract. This is often dynamic (muscular bulge below the pulmonary valve) and will ultimately determine the degree of right to left shunting (and hence cyanosis etc.). B: 151. Which of the following is not true of diastolic heart failure? A : It affects a younger age group B : Reversal of E:A ratio is demonstrated on transthoracic echocardiography. C : Presence of fourth heart sound D : Ejection fraction of >40% E : It is associated with obesity. Comment : Diastolic impairment is estimated to occur in 20-50% of patients with heart failure. Symptoms of breathlessness are secondary to impaired filling of the LV, resulting in high filling pressures. This in turn results in pulmonary congestion, dyspnoea and oedema. Management of diastolic heart failure is aimed at reducing pulmonary congestion. There is much less evidence for the benefit of therapeutic interventions from clinical trials than for patients with left ventricular systolic dysfunction.A:

Page 52: Masterclass Book Part 2

MOHAMMED IS-HAG 51

152. Which of the following is commonly associated with cyanosis in the infant? A : Atrial septal defect B : Co-arctation C : Patent ductus arteriosus D : Transposition of the great arteries E : Ventricular septal defect. Comment : Integral to cyanotic congenital heart disease is right to left shunting of blood in the heart. Transposition of the great arteries is the commonest cardiac cause of cyanosis at birth.It is incompatible with sustained life unless there is an associated atrial or ventricular septal defect or patent ductus arteriosus. Atrial septal defects, patent ductus arteriosus and ventricular septal defects may all result in cyanosis in the form of the Eisenmenger syndrome; left to right shunting results in pulmonary hypertension with eventual reversal of shunt. D: 153. Which of the following conditions is not associated with a rise in serum cardiac troponins: A : myocardial infarction B : pulmonary embolus C : renal failure D : liver failure E : intracerebral bleed. Comment : Raised serum cardiac troponins are sensitive markers of cardiac necrosis secondary to any primary cause. Therefore, raised levels can be seen in acute pulmonary embolus (due to right ventricular infarction), myocarditis, and any intracerebral event (due to focal myocardial necrosis). Up to 40% of all patients with renal failure have raised serum troponins. The reasons and significance of this remain unclear. In acute coronary syndromes the presence of a raised troponin suggests a high risk group who would benefit from inpatient treatment. D: 154. A 50-year-old woman is seen in cardiology clinic with progressive breathlessness and ankle swelling. Auscultation of her heart reveals a fixed split second heart sound. Which of the following is true? A : The most likely diagnosis is an ostium primum atrial septal defect. B : The most likely diagnosis is a sinus venosus defect. C : The most likely diagnosis is mitral stenosis. D : She is likely to have pulmonary vascular disease. E : Surgery is indicated in the majority of cases. Comment : The most likely diagnosis is an ostium secundum atrial septal defect (ASD). Eighty-five per cent of ASDs are of the secundum type, 11% of the primum type and 4% sinus venosus defects involving the inferior vena cava (IVC) or superior vena cava (SVC). Clinical signs are rarely present. When people present in middle age with symptoms this usually relates to the development of progressive pulmonary vascular disease, pulmonary arterial hypertrophy and a change in the direction of flow across the shunt (left to right changes to right to left).Many ASDs are now being closed using percutaneous closure devices. D: 155. A 38-year-old woman who was previously fit and well presents with breathlessness that has been getting worse over 3 or 4 months. You suspect that she might have primary pulmonary hypertension. Which of the following physical signs would NOT be consistent with this diagnosis? A : Elevated jugular venous pressure B : Clubbing C : Palpable heave at left sternal edge D : Added heart sound over right ventricle E : Loud P2. Physical signs include elevated JVP (sometimes with a giant v wave of tricuspid regurgitation and pulsatile hepatomegaly), left parasternal heave, pansystolic murmur (tricuspid regurgitation) and right ventricular S4, peripheral oedema. Clubbing is not a feature and suggests chronic lung disease or cyanotic congenital heart disease in this context.An important differential diagnosis of primary pulmonary hypertension is chronic pulmonary thromboembolism. B: 156. Which of the following statements is correct regarding the neurohormonal response to stimulation of cardiovascular mechanoreceptors? A : non-osmotic release of vasopressin occurs as a result of increased atrial stretch B : a reduction in blood pressure leads to stimulation of high-pressure mechanoreceptors in the

aortic arch and subsequent inhibition of renal sympathetic activity C : a reduction in peripheral vascular resistance is sensed by the high-pressure mechanoreceptors

and results in stimulation of the renin-angiotensin-aldosterone system D : atrial natriuretic peptide (ANP) is released in response to stimulation of receptors in the carotid

sinus and results in natriuresis and diuresis E : endothelin-1 is a potent vasodilator, released in response to elevated blood pressure Comment : Two distinct categories of cardiovascular mechanoreceptors exist: high-pressure and low-pressure. High-pressure receptors are located in the left ventricle, carotid sinus, aortic arch and renal juxtaglomerular apparatus. These receptors respond to decreases in arterial pressure, peripheral vascular resistance or renal perfusion by stimulating reflexes that result in the activation of the

Page 53: Masterclass Book Part 2

MOHAMMED IS-HAG 52

sympathetic and the renin-angiotensin-aldosterone systems and the non-osmotic release of vasopressin. The major net effects are vasoconstriction and the retention of sodium and water. Low-pressure receptors are primarily found within the atria. An increase in atrial stretch results in enhanced secretion of ANP with subsequent vasodilatation and both natriuresis and diuresis. Endothelin-1 is a potent vasoconstrictor. The interactions between these systems are relatively complex in both health and diseased states. For example ANP, angiotensin II and the renal sympathetic nerves are able to modulate the renal juxtaglomerular release of renin.C: 157. You see a 48-year-old Afro-Caribbean man in the outpatient clinic with uncomplicated essential hypertension. His blood pressure today is 154/102mmHg despite optimization of non-pharmacological therapy. Which one of the following drugs would you use as the first-line agent in this patient? A : Atenolol 50mg od B : Nifedipine 10mg tds C : Amlodipine 5mg od D : Ramipril 2.5mg od E : Enalapril 5mg bd. Comment : Non-pharmacological therapy should always be optimized prior to commencement of medication, whenever possible. Hypertension is particularly common in Afro-Caribbeans and associated with particularly higher risk of complications. Therefore effective long-term treatment, with a low threshold for multiple therapy where necessary, is particularly important. Studies indicate that drugs such as ACE (angiotensin-converting enzyme) inhibitors and Beta-receptor antagonists are less effective in Afro-Caribbeans. The reason appears to be related to the finding that the renin-angiotensin-aldosterone (RAA) system is commonly suppressed in the majority of Afro-Caribbeans. As such, drugs that suppress the RAA system are less likely to be effective. Calcium-channel blockers (CCBs) and diuretics appear to be more effective in this subgroup. However, diuretics may not be suitable in this case as they are commonly associated with impotence. Short-acting CCBs do not provide prolonged BP control, can cause reflex tachycardia and may be associated with higher mortality. Therefore, long-acting CCB should be the first-line drug of choice.Ideally, a once-daily agent with that provides a smooth 24-hour BP control (e.g. Nifedipine LA 30mg od or Amlodipine 5 mg od) to improve compliance would be preferable. C: 158. In acute aortic dissection: A : surgery should only be considered for type A dissection when the aortic valve is involved B : opiates are best avoided due to the risk of bradycardia C : type B dissections commonly begin just prior to the origin of the left subclavian artery D : intravenous beta-blockers should be used for acute control of blood pressure E : calcium channel blockers should be avoided. Comment : The classification of aortic dissection depends upon the site of origin of the dissection flap. Type-A is more common and involves the ascending aorta, whereas type-B begins distal to the ascending aorta, usually just after the origin of the left subclavian artery. Type-B dissections are generally treated medically with strict blood pressure control. Early surgical intervention is indicated in the case of Type A dissection; tight blood pressure control is also required. Adequate analgesia, thereby decreasing sympathetic stimulation, is imperative in all cases and usually requires opiates.To achieve tight blood pressure control an intravenous agent with short half-life is generally used. The therapy of choice is generally considered to be a beta-blocker such as proanolol or labetolol (alpha and beta-receptor blocker). Beta-blockers have additional beneficial effects by reducing arterial wall stress (reducing rate of change of arterial wall pressure). D: 159. A previously fit young woman presents with pleuritic chest pain 48 hours after a transatlantic aeroplane flight. Her chest radiograph is clear. The correct course of action is: A : Order a lung ventilation / perfusion scan. Give low molecular weight heparin if this is positive B : Reassure. Give non-steroidal anti-inflammatory agent for pain control C : Give oral amoxicillin and clarithromycin D : Give low molecular weight heparin. Order a lung ventilation / perfusion scan E : Order a chest radiograph in expiration to exclude pneumothorax. Comment : This woman has had a pulmonary embolus until proven otherwise. It is correct to start treatment immediately when the index of suspicion is very high and the risks of treatment (anticoagulation) are low, as in this case. D: 160. Which of the following is true concerning Group 1 entitlement to drive with cardiovascular disease: A : Patients with angina cannot drive B : After myocardial infarction patients can drive after one week C : Patients with syncope cannot drive until the underlying cause has been found and treated D : After implantation of a pacemaker patients cannot drive for one month E : After angioplasty patients cannot drive for one month Comment : Group 1 entitlement refers to normal driving. In this situation patients can drive one week after angioplasty or permanent pacing. After myocardial infarction patients must not drive for one month. Those with stable angina may drive if their symptoms dont occur at rest or at the wheel. The laws concerning Group 2 entitlement are quite different (see module reference).C:

Page 54: Masterclass Book Part 2

MOHAMMED IS-HAG 53

1. A 26-year-old man is admitted to hospital with a history of dyspnoea of 4-6 weeks duration. He initially had flu-like symptoms and was treated by his doctor with a 10-day course of ciprofloxacin. However, he then started coughing up blood, leading to urgent referral. On examination he was dyspnoeic at rest, with bilateral crackles on auscultation of the lungs. Investigation revealed anaemia and impaired renal function (creatinine 220 micromol/l). Pulmonary function tests were normal apart from an abnormally high diffusion factor. Urine dipstick testing showed the presence of red blood cells. What is the most likely diagnosis? A : Lymphangiomyomatosis B : Goodpasture's syndrome C : Chronic eosinophilic pneumonia D : Bronchiolitis obliterans E : Invasive Aspergillosis. This patient has both lung and kidney involvement typical of a 'pulmonary renal syndrome'.Goodpasture described the association of pulmonary haemorrhage with renal failure(Goodpasture's syndrome)in1919, and the 'classic' cause of this, where the condition is due to the presence of circulating anti-glomerular basement membrane antibodies (anti-GBM antibodies) is termed Goodpasture's disease (although this wasn't the cause of the cases he described).Other causes of pulmonary haemorrhage and renal failure include Wegener's granulomatosis, microscopic polyangiitis and systemic lupus erythematous (SLE). B : 2. A 75-year-old man with a 30 pack year cigarette smoking history complains of continuous right shoulder pain, a persistent cough and weight loss. His chest radiograph shows a right apical shadow. Which of the following signs are consistent with a right Horner’s syndrome? A : Ipsilateral papillary constriction B : Contralateral papillary constriction C : Ipsilateral pupillary dilatation D : Contralateral pupillary dilatation E : Ipsilateral ptosis F : Contralateral ptosis G : Ipsilateral enophthalmos H : Contralateral enophthalmos I : Ipsilateral homonymous hemianopia J : Contralateral anhydrosis K : Ipsilateral deviation of the tongue on protrusion L : Inability for right lateral gaze M : Failure of the ipsilateral pupil to react to light N : Failure of the contralateral pupil to react to light O : Absent knee and ankle reflexes. Comment : Horner’s syndrome consists of papillary constriction, enophthalmos, ptosis and loss of sweating on the same side of the face due to interruption of the sympathetic supply to the pupil. This may occur in the brainstem, cervical part of the spinal cord, or in the thoracic outlet. Holmes–Adie syndrome is a benign condition. The affected pupil is larger than the other and appears not to react to light or accommodation, but in fact the reactions are slow and prolonged. The pupil is myotonic and in association with absent knee and ankle jerks forms Holmes-Adie syndrome. A : E : G: 3. A 45-year-old man is taking long-term theophylline for asthma. One evening, he is admitted to the Accident and Emergency department with convulsions. You suspect theophylline toxicity. Which one of the following statements is true? A : His convulsions should not be treated until a theophylline level is available. B : Theophylline toxicity may have been precipitated by the concomitant prescription of phenytoin. C : Theophylline toxicity may have been precipitated by the concomitant prescription of

erythromycin. D : Theophylline toxicity only occurs in the elderly. E : Theophylline is an example of a drug with a wide therapeutic range (therapeutic index). Convulsions should be treated immediately in the usual way, without waiting for confirmation of the theophylline level.Theophylline is metabolised by the CYP450 enzymes in the liver. Erythromycin inhibits CYP450 enzymes and increases the half-life of theophylline and hence plasma theophylline concentrations, which may lead to toxicity. By contrast, phenytoin induces CYP450 enzymes, which will decrease the half-life of theophylline and may lead to inadequate therapeutic levels.Theophylline toxicity is more likely in the elderly due to age-related reduction in the rate of its metabolism, but it can occur at any age.Theophylline is an example of a drug with aNARROW therapeutic range. It is recommended that plasma theophylline levels be maintained between 10 and 20 mg/l. C :

Page 55: Masterclass Book Part 2

MOHAMMED IS-HAG 54

4. Which one of the following statements regarding the lung and pleura is NOT true? A : The lung apex extends about 3cm above the medial part of the clavicle. B : The horizontal fissure separates the right middle lobe from the right lower lobe. C : The horizontal fissure may be visible on a plain radiograph of the chest. D : The lower margin of the pleura is about two ribs below the lower margin of the lung. E : The lower parts of the lung and pleura overlap the right surface of the liver. Comment : The apical pleura and lung project about 3–4 cms above the inner aspect of the clavicle, where they are related to the subclavian vessels and the brachial plexus: hence a chest radiograph to exclude pneumothorax is warranted following subclavian vein cannulation. The oblique fissure lies between the upper and lower lobes. The horizontal fissure, often seen on a plain radiograph, demarcates the middle lobe from the upper lobe on the right side. The lower border of the lung extends up to the tenth rib at the back and the pleura up to the twelfth, hence the lung and pleura overlaps the liver, kidney and spleen. B : 5. A 62-year-old woman is admitted with an infective exacerbation of chronic obstructive pulmonary disease (COPD). Admission blood gases taken while breathing 28% oxygen are as follows: pH 7.31, pO2 8.4 kPa and pCO2 7.4 kPa. Which of the following best describes the blood gas picture? A : Compensated Type I respiratory failure B : Compensated Type II respiratory failure C : Decompensated Type I respiratory failure D : Decompensated Type II respiratory failure E : Partially compensated respiratory alkalosis. Comment : These blood gases show the patient is hypoxic on oxygen and the elevated pCO2 combined with the low pH confirm a respiratory acidosis. Therefore, the blood gas picture is of decompensated Type II (respiratory) failure and the prognosis is poor unless treated appropriately, including consideration of non-invasive positive pressure ventilation (NIPPV). D : 6. An 86-year-old lady, generally fit and self-caring, is admitted with an acute exacerbation of chronic obstructive pulmonary disease (COPD). One hour after admission she remains distressed with a respiratory rate of 30/minute and is peripherally cyanosed. Repeated arterial blood gases show a severe respiratory acidosis with a pH of <7.3. She is becoming exhausted, oxygen saturations are falling, and the on call anaethetist says there are no intensive care beds available for mechanical ventilation. Which of the following treatments is correct? A : Repeat bronchodilator therapy and arrange repeat arterial gases in one hour. B : Give intravenous infusion of aminophylline. C : Intubate the patient at the bedside and manually ventilate until a bed is available. D : Arrange for noninvasive positive pressure ventilation. E : Give intravenous hydrocortisone. Comment : Enhancing ventilation by unloading fatigued ventilatory muscles is an important goal in treating exacerbations of acute COPD. Non invasive positive pressure ventilation has been shown to reduce rates of intubation, lower hospital mortality rates and lead to shorter hospital stays.Non invasive positive pressure ventilation should be considered when there is a need for ventilatory assistance as indicated by worsening dyspnoea, acute respiratory acidosis and worsening oxygenation. D : 7. You are treating a 15-year-old girl with asthma. She is currently on salbutamol inhaler as required and beclometasone 400 micrograms daily. She finds that she has to use her salbutamol inhaler two or three times a day, and that she sometimes has nocturnal cough. Which of the following two treatment options might you recommend? A : Continue current medication B : Eformeterol C : Nedocromil sodium D : Theophylline E : Montelukast F : Ipratropium G : Salbutamol sustained-release tablets H : Salmeterol I : Aminophylline J : Ketotifen. Comment : The presence of nocturnal cough and bronchodilator requirement of 2-3 times a day suggests that her asthma control is inadequate. The recommended options in this instance would be to either increase the inhaled steroid to 800 micrograms a day, or to add a long acting beta-2-agonist such as salmeterol or eformoterol. If both these options fail, we can then move on to consider alternative choices. B :H:

Page 56: Masterclass Book Part 2

MOHAMMED IS-HAG 55

8.

A 65-year-old smoker presents to A&E with a cough productive of green sputum and blood, fever, right-sided pleuritic pain and breathlessness. He is confused, has a temperature of 39.8C, BP 120/70, pulse 120 regular, respiratory rate 20. His cardiovascular examination is normal and he has bronchial breath sounds at the right apex. His full blood count, urea and electrolytes and liver function tests are normal, excepting a leukocytosis of 16. His chest radiograph is shown (see image). Which one of the following statements is true? A : Because of the lobar distribution, it can be assumed that the infecting organism is unlikely to be

an atypical one. B : He requires a bronchoscopy to rule out cancer causing an obstructive pneumonia and

haemoptysis. C : He has non-severe pneumonia. D : If his symptoms and signs resolve rapidly he need not have a follow-up chest radiograph. E : He should be given full dose anticoagulation to cover the possibility that this may be a

pulmonary embolus causing pulmonary infarction and haemoptysis. It is not possible to predict the organism from radiographic features.Pneumonia itself can cause haemoptysis and bronchoscopy should only be considered if pneumonia or haemoptysis fails to resolve at follow-up. Severity of pneumonia is defined according to the (new) Confusion (abbreviated mental test score <8), Urea >7 mmol, Respiratory rate (>30) and Blood Pressure (systolic < 90, diastolic <60) (CURB) criteria. More than two of these features suggests severe pneumonia and these patients should be assessed by a respiratory physician. All smokers, patients over 50 or those with non-resolving symptoms or signs should have a follow-up chest film at 6 weeks Although pulmonary infarction should be on the differential diagnosis, the clinical probability strongly favours pneumonia. There are no atypical features incompatible with pneumonia. There is lobar consolidation and he is coughing green sputum. There are no cardiovascular features to suggest PE. He should receive regular DVT prophylaxis. C : 9. A 40-year-old obese man is referred with excessive daytime sleepiness. Which of the following is NOT a feature of obstructive sleep apnoea? A : Intellectual deterioration B : Morning headache C : Snoring D : Awareness of sleep disturbance E : Nocturnal enuresis. Comment : Snoring occurs in all patients with obstructive sleep apnoea (OSA), and intellectual deterioration in 75%. Morning headache and nocturnal enuresis occurs in approximately a third of adult patients. Patients are not aware of their sleep disturbance and a witness description is necessary for the diagnosis to be made. D :

Page 57: Masterclass Book Part 2

MOHAMMED IS-HAG 56

10. A 26-year-old woman with epilepsy has been diagnosed with pulmonary tuberculosis and is about to be started on quadruple therapy. The following need to be discussed with her except: A : Method of contraception B : The need for screening for ocular complications with ethambutol C : Interactions with other medications D : Discoloration of urine due to isoniazid E : Compliance. Comment : Isoniazid inhibits carbamazepine and phenytoin and ethosuximide. Her anti-epileptic medication will need reviewing. She should also be reminded to check with a doctor or pharmacist before starting other prescribed or over the counter medications. Rifampicin turns the body fluids including urine orange, which can be alarming to a patient if they are not pre-warned. Methods of contraception will be important for any sexually active female. Rifampicin is a hepatic enzyme inducer. Accelerated metabolism leads to a reduced contraceptive effect of both the progesterone only and combined pill.Ethambutol has ocular toxicity and vision should be checked routinely at follow-up. D : 11. A 56-year-old male patient become unwell 2 days after returning from holiday in Spain. He is breathless, complains of pleuritic chest pain and has a temperature of 39 centigrade. A chest radiograph is performed which shows patchy consolidation in both lung fields. Which of the following treatments is indicated immediately? A : IV antibiotics after blood cultures are taken B : IV antibiotics after a sputum sample is obtained C : Subcutaneous low molecular weight heparin after clotting screen D : IV diuretic after 12 lead ECG E : Nebulized bronchodilators after peak expiratory flow rate (PEFR). Comment : The history and presentation are suggestive of Community Acquired Pneumonia, most probably related to his recent travel. This raises the possibility of penicillin resistance and Legionella as confounding factors. Treatment should be commenced immediately after blood cultures are obtained and not delayed to obtain sputum specimens. A starting combination of co-amoxiclav or cefotaxime/ ceftriaxone/ cefuroxime plus a macrolid to cover likely organisms is indicated. A : 12. A 58-year-old man, a smoker for many years despite repeated advice that he should stop, has chronic obstructive pulmonary disease that is increasingly limiting his exercise capacity. You wish to conduct a trial of steroid therapy. Which of the following is the correct way to do this and interpret the outcome? A : Give oral prednisolone, 60 mg daily for 4 weeks, regarding a clear statement of subjective

improvement by the patient as a positive response. B : Give oral prednisolone, 10 mg daily for 2 weeks, regarding a clear statement of subjective

improvement by the patient as a positive response if accompanied by a rise in FEV1. C : Give oral prednisolone, 60 mg daily for 4 weeks, regarding a clear statement of subjective

improvement by the patient as a positive response if accompanied by a rise in FEV1 of 10%. D : Give oral prednisolone, 30 mg daily for 2 weeks, regarding an increase in FEV1 of >10% and

>200 ml as a positive response. E : Give oral prednisolone, 30 mg daily for 2 weeks, regarding a clear statement of subjective

improvement by the patient as a positive response. Comment : Steroid challenge is indicated in chronic obstructive pulmonary disease (COPD) of more than moderate severity. Standard practice would be to give prednisolone 30 mg daily for 2 weeks, regarding an increase in FEV1 of >10% and >200 ml as a positive response. Given the non-specific effects and many side effects of steroids, it is crucial to demonstrate functional improvement: many patients with COPD have sustained severe complications of steroid treatment, e.g. vertebral fracture, without any evidence that the steroids were beneficial for their chest. D : 13. A 35-year-old woman is referred with a history of red, painful legs of 3 weeks duration that have not responded to a course of amoxycillin and flucloxacillin given for presumed cellulitis. She is afebrile, does not have any other symptoms, and has never smoked. Examination reveals tender purple / red nodules on her shins. Her full blood count, kidney and liver function tests are normal. A chest radiograph shows prominent hilae. What is the appropriate management? A : Arrange bronchoscopy and bronchoalveolar lavage to exclude malignancy B : Start azathioprine and prednisolone; follow up in clinic C : Start prednisolone; follow up in clinic D : Start simple analgesics; follow up in clinic E : Arrange CT scan of the lungs and lung biopsy Comment : The combination of bilateral hilar lymphadenopathy and erythema nodosum is diagnostic of sarcoidosis. This is usually self-limiting. She should however be followed up in clinic with full lung function tests including transfer factor and lung volumes. Serum angiotensin-converting enzyme (ACE) level and lung functions can be used to monitor disease. Worsening disease should be treated with prednisolone. D :

Page 58: Masterclass Book Part 2

MOHAMMED IS-HAG 57

14. A 63-year-old man attends the hospital with a history of proximal muscle weakness. He also gives a history of cough of 8 weeks duration and complains of pain of the small joints of the hands. He has small haemorrhages in the nail folds, but is not clubbed. In the chest he has bibasal crackles, and a chest radiograph reveals diffuse reticular infiltrates. Lung function tests confirm a restrictive pattern. What is the underlying cause of his interstitial lung disease? A : Cryptogenic fibrosing alveolitis B : Ankylosing spondylitis C : Polymyositis and dermatomyositis D : Rheumatoid arthritis E : Mixed connective tissue disorder. Comment : Polymyositis and dermatomyositis are inflammatory conditions involving the muscle and skin. Patients often complain of proximal muscle weakness and of pain in the small joints of the fingers. They may have ragged cuticles and haemorrhages at the finger nail folds. Interstitial lung disease can occur. Underlying malignancy (lungs, ovaries, breasts and stomach) is present in 5-8% of cases. C : 15. A 38-year-old asthmatic woman presents with an acute attack. Her arterial blood gases breathing air are as follows: pH 7.36, pO2 9.8 kPa, pCO2 5.2 kPa. These mean: A : the attack is not severe B : she should be given supplemental oxygen, but is unlikely to need a high FiO2 to achieve

normoxia C : cardiorespiratory arrest could be imminent D : her respiratory effort may be failing because she is getting tired E : she could have had a pneumothorax. Comment : A normal or elevated pCO2 in an asthmatic indicates failing respiratory effort, and although this woman's oxygen saturation is not severely depressed she is in danger of decompensation and - aside from high flow oxygen, nebulised salbutamol and ipatropium, and steroids - it would be prudent to inform the ICU of her existence. The gases are not bad enough, however, to suggest that cardiorespiratory arrest is imminent.Pneumothorax must be excluded in any asthmatic, but the presence or absence of pneumothorax can never be inferred from arterial blood gas analysis. D : 16. A 34-year-old previously fit homosexual man is admitted to the hospital with a one-week history of dyspnoea. His chest radiograph shows bilateral alveolar infiltrates and apart from a PaO2 of 6Kpa on breathing room air, his routine blood tests are normal. The admitting doctor makes a diagnosis of Pneumocystis carinii pneumonia (PCP). Which of the following is NOT true? A : Unilateral pleural effusions are commonly seen in this condition. B : The patient should be treated with intravenous co-trimoxazole. C : The patient should be treated with prednisolone. D : The patients lung function tests will show a reduced diffusing capacity for carbon monoxide (DLCO). E : PCP is associated with the syndrome of inappropriate ADH (SIADH) secretion. Comment : PCP is commonly seen in the immunosupressed. Patients usually complain of cough and dyspnoea. The cough is usually non-productive. Patients are usually hypoxic and chest radiograph characteristically shows bilateral alveolar infiltrates. Pleural effusions are very rare. The patient should receive intravenous co-trimoxazole in a dose of 120mg/kg for at least 3 weeks. Early administration of glucocorticoids to patients with moderate to severe hypoxia, decreases the risk of respiratory failure and death by over 50%. Pulmonary function tests demonstrate a decreased vital capacity, increased residual capacity, normal flow rates and a reduced DLCO. SIADH is known to occur with PCP. A : 17. A 27-year-old lady with asthma was admitted with an acute attack. Her symptoms started around 8.00am, but by 10.00am she was so unwell that she needed admission to the hospital. Shortly after arriving to the accident and emergency she was admitted to the intensive care unit. Similar incidents had occurred twice in the past. Which two of the following statements are correct for near fatal asthma? A : Previous severe asthma attacks are a risk factor. B : The short time lag between the start of symptoms and hospital admission is a risk factor. C : Female patients are especially at risk. D : The risk increases in obese patients. E : Family history of asthma is common. F : Allergy to peanut is commoner in patients with near fatal asthma. G : Near fatal asthma occurs more frequently during winter season. H : Long acting beta2 agonists are beneficial in preventing asthma attacks. I : Childhood eczema is a risk factor. J : Parental smoking is a risk factor. Comment : Near fatal asthma or brittle asthma is responsible for around 1000 deaths every year in the UK. It mainly occurs in young patients. Previous attacks and the short time between the start of symptoms and hospital admission are the two main risk factors. The treatment of choice is hospitalisation with early start of nebulised bronchodilators, oxygen and intravenous corticosteroids.

Page 59: Masterclass Book Part 2

MOHAMMED IS-HAG 58

The recovery rate is normally quick. Long acting beta2 agonists and leukotrine receptor antagonists are not successful in preventing near fatal asthma. There is no known relationship between near fatal asthma and personal history of allergy. The incidence of the disease does not differ according to gender and there are no seasonal variations for the rate of hospitalisation. A : B : 18. A 60-year-old man who smoked 20 cigarettes a day for 40 years enquired about recent advances in screening for lung cancer that he had read on the internet. Features of low-dose thoracic CT scans for lung cancer (LC) screening studies include: A : low sensitivity B : high specificity C : low cost D : no contrast media required E : the size of malignancies detected are no smaller than with chest radiograph F : less radiation than chest radiograph G : low false positive rate H : high false negative rate I : positive predictive value (PPV) over 50% J : low-dose CT detects more cases of lung cancer than chest radiograph. Comment : ADVANTAGES · Extremely sensitive and reliable: able to detect LC at early stages · No contrast media · Radiation dose of low-dose CT is 1/7th the radiation dose of a standard CT DISADVANTAGES · Expensive: $350 · Radiation exposure is about 10 times more than one for chest radiograph · False-positive results are very common. PPV is less than 10%. · Spiral CT scanning screening Two non-randomized studies from Japan: - almost 7000 people - smokers and non-smokers over 40 years of age -chest radiograph, sputum cytology and low-dose CT Early Lung Cancer Action Project, non-randomized trial: - 1000 people - high smokers over 60 years of age - chest radiograph and low-dose CT. RESULTS: Low-dose CT detects more cases of lung cancer than chest radiograph (27/1000 vs 9.1 to 7.6/1000) Low-dose CT compared with chest radiograph detected: - Non-calcified nodules: three times as commonly (23% vs 7%) - Malignancies: four times as commonly (2.7% vs 0.7%) - Stage I malignancies: six times as commonly (2.3% vs 0.4%) The size of malignancies are smaller with low-dose CT. However, the prevalence-screening rate for advanced disease on low-dose CT did not decrease. True clinical significance of small tumours founded by screening is unknown. CT founded at least one indeterminate nodule in 23% of patients. D : J: 19. A 28-year-old man is has recently been discharged from hospital after treatment for pneumococcal pneumonia. He has had repeated courses of antibiotics for sinus, ear and lower respiratory tract infections, and had sinus surgery the previous year. He is a life long non-smoker and is not on medication. His blood count prior to discharge was normal. In the absence of further clues in the history or examination, which single blood test is the most important? A : HIV antibody test B : Pneumococcal antibodies C : Immunoglobulin levels D : Liver function tests E : IgG subclass levels. Comment : Hypogammaglobulinaemia is associated with recurrent bacterial infections, most commonly of the respiratory tract. Delay of several years prior to diagnosis is usual, with associated morbidity. Patients with low immunoglobulin levels and recurrent infections should be treated with immunoglobulin replacement. More minor antibody defects, such as IgG subclass or specific antibody (to pneumococcus) defects can often be treated with appropriate vaccinations and/ or prophylactic antibiotics. C :

Page 60: Masterclass Book Part 2

MOHAMMED IS-HAG 59

20. A 74-year-old man was treated in hospital for exacerbation of Chronic Obstructive Pulmonary Disease (COPD). His condition has improved significantly and he is keen to go home. His repeated arterial blood gas (ABG) reading (on air) showed a pH of 7.35, pCO2 of 4.5kPa, pO2 of 7.1kPa, HCO3 of 26 mmo/L. What action would be most appropriate? A : Discharge the patient, prescribing an oxygen cylinder. B : Discharge the patient once his ABG returns to normal. C : Request an oxygen concentrator and discharge the patient once an oxygen concentrator is fitted. D : Discharge the patient and arrange a follow up in 6 weeks' time with a repeated ABG. E : Discharge the patient and advise a follow up by his GP. Comment : Long-term oxygen treatment is recommended if pO2 is less than 7.3kPa on two occasions in remission of COPD. This patient's p02 is likely to improve when he recovers from his current exacerbation and his ABG should be re-checked in about 6 weeks time. As he seems able to tolerate his mild hypoxia, he would not require any oxygen supplementation at home at this stage. D : 21. A 65-year-old lady is admitted with left sided pneumonia and pleural effusion. Pleural fluid is aspirated and sent for tests. Which of the following is an indication for inserting a chest drain? A : Pleural fluid pH <7.2 B : Serous pleural fluid C : Blood stained pleural fluid D : Pleural fluid glucose >2mmol/l E : Pleural fluid lactate dehydrogenase > 200IU/l. Comment : Infected pleural effusions should be drained. Infected pleural effusions should be drained if the pH<7.2, Gram stain shows organisms, the fluid is frankly purulent and clinical improvement is slow despite antibiotics. A : 22. A 48 yr old man presents on the medical take with right lower lobe pneumonia. In the acute assessment of an adult with community-acquired pneumonia which one of the following is NOT of prognostic importance? A : Urea >7mmol/l B : Confusional state C : PaO2 <10kPa D : Respiratory rate >30/min E : Diastolic BP <60 mmHg. Comment : The British Thoracic Society has recently published guidelines for assessment and management of adult community-acquired pneumonia (Thorax 56, supplement 4). Adverse prognostic features include: · Pre-existing factors: Age > 50 years Co-existing disease · Core clinical adverse prognostic features (CURB criteria): Confusional state · Urea >7mmol/l · Respiratory rate >30/min · Systolic BP <90mmHg and/or diastolic BP < 60 mmHg · Additional features: · Hypoxaemia, SaO2<92% or PaO2<8kPa · Bilateral/multi-lobe disease Patients with two or more core features are at high risk of death and should be managed as severe pneumonia in hospital. C : 23. A 65 year old man with a history of rheumatoid arthritis (RA) develops a left pleural effusion. Which of the following are typical of rheumatoid pleural effusions? A : Over 50% of patients with RA develop a pleural effusion at some stage. B : It is more commonly seen in female patients. C : The glucose level in the effusion fluid is less than 1.6mmol/l. D : The effusions mainly occur in patients without rheumatoid nodules. E : The effusions are usually transudates. F : Intrapleural corticosteroids is the treatment of choice. G : They are usually bilateral. H : They have high levels of cholesterol. I : The pH of pleural fluid in an effusion due to RA is usually > 7.30. J : They should be treated with a chest drain. Comment : Twenty per cent of patients with RA develop pleurisy and only 5% develop pleural effusions. It is commonly seen in older men with rheumatoid nodules and is usually an exudate. RA is

Page 61: Masterclass Book Part 2

MOHAMMED IS-HAG 60

unlikely to be the cause of an effusion if the glucose level is above 1.6. These effusions tend to have high cholesterol levels. C :H: 24. An 18-year-old woman has asthma for which she is using her salbutamol inhaler two or three times a day. What should be the next step in her management? A : Add regular long-acting inhaled beta agonist B : Advise her to use the salbutamol inhaler regularly three times a day C : Add regular inhaled steroid, e.g. beclometasone 100 microg twice daily D : Add regular inhaled steroid, e.g. beclometasone 1000 microg twice daily, plus regular long-acting inhaled beta agonist E : Add regular inhaled steroid, e.g. beclometasone 1000 microg twice daily. Comment : The British asthma guidelines for the stepwise management of chronic asthma in adults are as follows: Step 1: occasional use of inhaled short-acting beta agonists Step 2: regular inhaled anti-inflammatory agent, e.g. standard dose inhaled steroid Step 3: high-dose inhaled steroids, or low-dose inhaled steroids plus long-acting beta agonist Step 4: high-dose inhaled steroid and regular bronchodilators (sequential therapeutic trial of inhaled long-acting beta agonist, sustained release theophylline, inhaled ipatropium / oxitropium, oral long-acting beta agonist, high-dose inhaled bronchodilators, cromoglycate / nedocromil) Step 5: addition of regular steroid tablets This woman’s treatment needs to be escalated from Step 1 to Step 2. C : 25. A 19-year-old woman presents with fever and cough. Sputum samples are negative on microscopy for acid fast bacilli, but six weeks later M. tuberculosis is grown. She completed a course of chemotherapy for pulmonary TB two months previously. Her chest radiograph is unchanged from one taken at this time. Which of the following is the most likely explanation for these findings? A : She has HIV co-infection causing increased susceptibility to mycobacteria. B : The organism isolated is a contaminant. C : She has been re-infected with a different strain of TB. D : She has underlying IFNg receptor deficiency causing increased susceptibility to mycobacteria. E : She has been poorly adherent to therapy and her TB has recurred. Comment : Poor adherence is a common cause of treatment failure or early relapse; directly observed therapy (DOT) may improve adherence if this is the case. TB usually responds well to conventional treatment even if the patient is co-infected with HIV. In this situation, recurrent TB is most often caused by poor adherence (early) or reinfection (late). Underlying IFNg receptor/ IL12 deficiency is extremely rare and is associated with disseminated disease, usually with poorly pathogenic environmental mycobacteria.. Environmental mycobacteria may occasionally grow as contaminants in culture, but it is most unusual for MTB to do so. 26. A 56-year-old retired plumber had a chest radiograph that was arranged as a part of routine investigations before elective cholecystectomy. This showed bilateral pleural thickening. CT chest scan confirms calcified pleural plaques suggestive of previous asbestos exposure. He smokes 20 cigarettes daily, but denies any shortness of breath or chest pain, and full lung function tests are normal. Which two of the following are correct? A : Regular follow up with a repeated chest radiograph is required. B : Pleural biopsy is essential to make a final diagnosis. C : Bronchoscopy should be arranged to rule out lung cancer. D : No further investigations are required apart from annual spirometry. E : Asbestos related pleural plaques are precursors of malignant change. F : In asbestos related pleural plaques calcification of the pleural plaques is always seen on a chest

radiograph. G : The risk of lung cancer with combined exposure to asbestos and cigarette smoke appears to be

multiplicative. H : Asbestos-related pleural plaques entitle to Industrial Injuries Disablement benefit. I : Normal chest radiograph excludes asbestosis. J : Normal high-resolution computed tomography (HRCT) scan rules out asbestosis. Comment : Approximately 50% of persons exposed to asbestos develop pleural plaques. Up to 30% of asbestos-exposed individuals have an abnormal HRCT in spite of a normal chest radiograph. However, HRCT may still appear normal or near normal in cases of histopathologically-proven asbestosis. Calcification is identified by chest radiography in 20%, on CT scanning in 50%, and at morphologic examination in 80%. Benign pleural plaques do not require a regular follow up, but a patient should be advised to report any new symptoms as that may warrant further investigations.

Page 62: Masterclass Book Part 2

MOHAMMED IS-HAG 61

As this patient is a smoker, he should have spirometry annually as a screening for chronic obstructive pulmonary disease. D : H: 27. A 38-year-old homosexual man known to be HIV positive gives a 3-week history of progressive shortness of breath. No significant abnormality is found on physical examination and his chest radiograph is normal. Oxygen saturation at rest (on air) is 96%. Which one of the following would be of LEAST use in excluding the possibility of Pneumocystis carini pneumonia (PCP)? A : The finding of normal oxygenation on exercise. B : A normal high resolution CT (HRCT) scan of the lungs. C : A normal serum lactate dehydrogenase level (LDH). D : Normal peak expiratory flow rate. E : A blood CD4 count of 300 cells/mm3. Comment : In PCP a normal chest examination occurs in 50% cases and a chest radiograph is initially normal in up to 25%. The presence of PCP is highly unlikely if the response to exercise is normal or if measurement of the diffusing capacity for carbon monoxide (DLCO) is normal, defined by 70% of the predicted value or greater. HRCT has a high sensitivity for PCP among HIV-positive patients: a negative HRCT virtually excludes PCP. In HIV-infected patients the two blood tests that are most commonly abnormal in association with PCP are a CD4 count below 200 cells/mm3 and an elevated LDH, which is present in 90% of cases. A rising LDH level despite appropriate treatment portends a poor prognosis. D : 28. A 17-year-old Afro-Caribbean girl presents herself as an emergency with a 7-day history of general arthralgia and a painful rash over her lower limbs. Chest radiograph shows bilateral hilar enlargement but routine full blood count and urea, electrolytes, liver function tests and calcium are normal. Clinical examination is normal, apart from a raised erythematous painful rash, and she is apyrexial. Which of the following is the most likely diagnosis? A : Lymphoma B : Primary pulmonary hypertension C : Sarcoidosis D : Systemic lupus erythematosis E : Tuberculosis. Comment : The clinical presentation is most likely to be sarcoidosis, based on her ethnic origins and the features of arthralgia, rash and bilateral hilar enlargement. Measurement of serum angiotensin-converting enzyme (ACE) may help to make the diagnosis, but fibreoptic bronchoscopy with bronchial/transbronchial biopsies may be needed to confirm it. C : 29. A 73-year-old man with known chronic obstructive pulmonary disease (COPD) is admitted with symptoms of worsening breathlessness and confusion. His arterial blood gases are as follows: pH 7.20, PO2 6.0 kPa (45 mmHg), PCO2 9.1 kPa (68 mmHg). Which one of the following statements regarding his management is correct? A : Mechanical ventilation with tracheal intubation should be considered B : Patients with COPD are particularly difficult to wean from mechanical ventilation C : Non-invasive positive pressure ventilation (NIPPV) is not appropriate for patients with COPD D : Low arterial pH in COPD is associated with a high mortality rate E : Emphysema is a contraindication for nasal ventilation because of the risk of pneumothorax Comment : This patient should be considered for mechanical ventilation. Non-invasive intermittent positive pressure ventilation (NIPPV) is a useful method of treatment of acute or chronic respiratory failure. The results are best in patients with type II respiratory failure without acidosis. NIPPV should be either a first step of management in those who are suitable for mechanical ventilation or the management of choice in patients who are deemed to be unsuitable for intubation and ventilation. The rate of pneumothorax is small, but is higher in machines using volume-cycled mode rather than pressure cycled mode. The rate of successful weaning from mechanical ventilation of appropriately selected patients with COPD is similar to that for other diseases. A : 30. A 38-year-old woman is admitted with a 2-day history of fever, rigors and breathlessness. She looks extremely unwell and is confused, cyanosed, has a respiratory rate of 24/min and a systolic blood pressure of 80 mmHg. There is bronchial breathing at her right base, where a chest radiograph reveals consolidation. Which would be the most appropriate antibiotic regimen to use? A : Oral amoxicillin B : Oral amoxicillin and oral clarithromycin C : Intravenous amoxicillin and intravenous clarithromycin D : Intravenous cefotaxime and intravenous erythromycin E : Intravenous ceftazidime and intravenous vancomycin.

Page 63: Masterclass Book Part 2

MOHAMMED IS-HAG 62

Comment : This woman has severe pneumonia as defined by the British Thoracic Society guidelines, which require any two of the following features: confusion, urea >7mmol/l, respiratory rate >30/min, and hypotension (SBP <90mmHg, DBP <60mmHg).Appropriate treatment (as recommended by the British Thoracic Society) is with intravenous antimicrobials: co-amoxiclav 1.2g three times daily or cefuroxime 1.5g three times daily or cefotaxime 1g three times daily or ceftriaxone 2g once daily plus erythromycin 500 mg four times daily or clarithromycin 500mg twice daily. D : 31. A 40-year-old Afro-Caribbean lady presents with bilateral facial nerve palsy. You suspect the diagnosis of neurosarcoidosis. Which two responses are not recognised features of sarcoidosis? A : optic disc pallor B : diabetes insipidus C : parkinsonism D : matched oligoclonal bands in serum and CSF E : reduced transfer factor on respiratory function tests F : myopathy G : anterior uveitis H : peripheral neuropathy I : multiple white matter lesions on MRI J : amenorrhoea Comment : Sarcoidosis is a primary granulomatous disorder. Neurological complications occur in approximately 5% of patients. Cranial neuropathies from chronic basal meningitis are the main neurological complications of sarcoidosis; facial palsies, commonly bilateral, are most common. Other abnormalities include hypothalamic disturbances, optic nerve inflammation, peripheral neuropathy and myopathy. Meningeal enhancement on MRI with gadolinium is typical. Unmatched oligoclonal bands are typically seen in the CSF of patients with neurosarcoidosis. C : D : 32. A 55-year-old man with insulin-dependent diabetes mellitus and chronic obstructive pulmonary disease presents with five days of worsening breathlessness, fever and cough. On examination he looks very unwell, with a temperature of 38.7?C, a respiratory rate of 18/min and widespread coarse crackles and wheezes in his chest. A finger prick test reveals a glucose level of 18 mmol/l. His arterial blood gases (breathing air) are as follows: pH 7.30, pO2 7.8 kPa, pCO2 6.8 kPa, base excess –2 mmol/l. Which of the following is the most likely explanation for these findings? A : He has a metabolic acidosis with respiratory compensation. B : He has type 1 respiratory failure. C : He has type 2 respiratory failure. D : He has type 1 respiratory failure and significant metabolic acidosis. E : He has type 2 respiratory failure with compensatory metabolic alkalosis. Comment : Type 1 respiratory failure is defined as pO2 <8 kPa with pCO2 <6.5 kPa. Type 2 respiratory failure is defined as pO2 <8 kPa with pCO2 >6.5 kPa. The base excess is a calculated value that provides a measure of the degree of ‘metabolic’ as opposed to ‘respiratory’ disturbance of pH. A normal value is between –2 and +2. If this man had metabolic acidosis as a result of his diabetes, then a significantly negative ‘base excess’ would be expected. C : 33. A 72-year-old man has a recurrent pleural effusion of unknown cause. Which one of the following statements regarding chemical pleurodesis is true? A : Talc has a success rate of less than 50% B : Acute respiratory failure is a known complication with talc C : Talc causes fever in over 95% of cases D : Tetracycline has a success rate of over 90% E : It should be performed under general anaesthesia Comment : Chemical pleurodesis can be performed under sedation. Talc has a success rate of over 93% while tetracycline has a success rate of 67%. Chest pain and fever are the most common adverse effects seen with all chemical agents. The fever due to talc usually occurs 4-12 hours after instillation and does not last longer than 72 hours. A few cases of acute respiratory failure, empyema & arrythmia have been described with talc. B : 34. A 40-year-old woman, generally fit and well, is admitted with malaise and fever. Four weeks previously she had suffered a chest infection for which she was given a course of oral antibiotics. She felt better initially, but never recovered fully and is now getting worse. Her chest radiograph shows a right-sided pleural effusion. The most likely diagnosis is: A : Connective tissue disorder B : Lung cancer C : Tuberculosis D : Pulmonary embolism E : Pneumonia with effusion / empyema.

Page 64: Masterclass Book Part 2

MOHAMMED IS-HAG 63

Comment : Although all of the diagnoses listed could present in this way, pneumonia with effusion / empyema is much the most likely. Examination of the pleural fluid is critical: 1. If associated with pneumonia and the effusion is opaque / turbid and smells foul then it is clearly an empyema, which would also be suggested by pH<7.2 2. If not associated with pneumonia then check protein and LDH to distinguish exudate from transudate using Light’s criteria 3. Cytology 4. Microscopy and culture, both aerobic and anaerobic, and for TB. E : 35. A 51-year-old woman is admitted with sudden onset left sided pleuritic chest pain accompanied by shortness of breath. Her only past medical history is asthma, which has been well controlled on a low dose of inhaled corticosteroids, and varicose veins ligation 2 weeks ago. On auscultation her chest is clear. She is tachycardic (132/min), her peak expiratory flow (PEF) rate is 300 L/min (best 400 L/min). ECG shows sinus tachycardia. Arterial blood gases are as follows: pH of 7.34, p02 of 7.6kPa, and pCO2 of 3.5 kPa, so she is started on oxygen. Chest radiograph is normal. What would be the most appropriate immediate action? A : Start nebulised bronchodilators and monitor PEF rate B : Request D-dimer C : Start low molecular weight heparin and request a V/Q scan D : Start low molecular weight heparin and request CT pulmonary angiography. E : Request a chest radiograph in expiration. Comment : It is unlikely that this patient's symptoms are due to exacerbation of asthma as her PEF rate is only mildly reduced (75% best). A small pneumothorax, not apparent on the inspiratory chest radiograph, is also unlikely: a small pneumothorax would not caused marked hypoxia. The symptoms and findings clearly point out towards pulmonary embolism (PE). As the clinical probability of PE is high, measurement of D-dimer should not be performed, since the result would not alter the need for definitive investigation. Measurement of D-dimer should only be performed when the probability of PE is low, when a normal value would be taken as reassuring and further investigation would not be pursued. V/Q scan is likely to be unhelpful in view of her asthma; hence a CT pulmonary angiogram would be the imaging procedure of choice. D : 36. A 50-year-old man presents with breathlessness that has got gradually worse over three months. He has long-standing atrial fibrillation, for which he takes warfarin and amiodarone. On examination his pulse is 80/min in AF and he has fine bibasal crackles, but there are no other abnormal physical signs. The most likely diagnosis is: A : amiodarone-induced interstitial lung disease B : congestive cardiac failure C : pulmonary embolism D : idiopathic interstitial lung disease E : pulmonary haemorrhage. Comment : Congestive cardiac failure would clearly need to be considered in a man with known cardiac disease, evidenced in this case by atrial fibrillation, but this is not a sustainable diagnosis in the absence of signs other than basal crackles. However, you would obviously look carefully for raised JVP, displaced apex, LV or RV heaves, added heart sounds and peripheral oedema before excluding this diagnosis. Chronic pulmonary embolism could present with insidious breathlessness but is not likely in a man who is on warfarin, and there are no signs to support the diagnosis (raised JVP, RV heave, loud P2, RV gallop). The diagnosis of idiopathic interstitial lung disease cannot be made in the presence of a known secondary cause of interstitial lung disease. A : 37. In patients with cystic fibrosis (CF): A : there is a correlation between genotype and the severity of the disease B : neonatal screening results in a better survival rate and reduced decline in lung function C : allergic bronchopulmonary aspergillosis is a recognised complication, found in 15% of adult CF

patients D : the controlled intake of high calorie food (dietary control) is the first line of management of CF-

related diabetes E : reduced body mass index is an absolute contraindication for heart-lung transplantation. Comment : The commonest mutation in cystic fibrosis (CF) patients is the Delta-F508. Homozygotes of this mutation have CF. Other genes are also implicated. There is no correlation between genotype, the range of manifestations, the age of symptoms and the survival rate. Neonatal screening is advocated aiming at introducing prophylactic therapeutic measures. Prospective and retrospective studies have

Page 65: Masterclass Book Part 2

MOHAMMED IS-HAG 64

demonstrated that neonates detected by screening have better nutritional status than those who were discovered by symptoms. However, the age of acquisition of Pseudomonas aeruginosa and the decline of lung function were comparable in the two groups.The increased age of survival for CF patients is associated with increased complications such as diabetes mellitus, acquisition of multiuple bacteria in their respiratory flora, osteoporosis and liver disease. Diabetes is now seen in almost 10% of patients over the age of 20-years. The main manifestations are: · weight loss · repeated respiratory infections · decline of lung function. The treatment of choice is subcutaneous insulin. Calorie intake should not be restricted in CF patients who are prone to malnutrition due to their pancreatic insufficiency.Increasing incidence of allergic bronchopulmonary aspergillosis (ABPA) is a recognised phenomenon. ABPA is encountered in 1 in 6 adult CF patients. The manifestations include: · asthma symptoms · flitting opacities on the chest radiograph · increased eosinophil count · hyper-reactivity to the skin prick test · increased specific plasma IgE. The main treatment is high dose corticosteroids initially, with a smaller maintainance dose. The duration of treatment ranges from few months to few years. Anti-fungal agents are of no proven benefit. Heart-lung transplantation is offered to patients with a rapid decline in lung function despite optimal treatment, and to patients with respiratory failure. Moderately reduced body mass index (BMI) is an indication for transplantation. Severe reduction of BMI is a relative contraindication because it is associated with reduced survival. C : 38. You are called to A&E to review a 28-year-old woman with pleuritic chest pain. Which of the following statements concerning this patient are true? A : Low white count is consistent with systemic lupus erythematosus (SLE) B : Low white count is not consistent with tuberculosis. C : The presence of anticardiolipin antibodies IgG 17 GPLU/ml (NR <14), IgM 21 MPLU/ml (NR<10)

suggests that she may have a pulmonary embolus. D : The presence of antinuclear antibodies at a titre of 1/80 suggests that she may have a

connective tissue disease. E : The presence of antinuclear cytoplasmic antibodies (cANCA) at a titre of 1 in 20 suggests that

Wegener's granulomatosis is likely. Comment : Active SLE and tuberculosis are both associated with leukopenia. Low titre antinuclear antibodies (in the absence of DNA or ENA), antineutrophil cytoplasmic antibodies (ANCA) and anticardiolipin antibodies are non-specific and are commonly found in the presence of infection. A : 39. A 55-year-old gentleman is under investigation for breathlessness. A chest radiograph suggests a diffuse parenchymal lung disease. Which of the following statements are correct? A : Clubbing is seen in over 90% of patients with cryptogenic fibrosing alveolitis (CFA). B : Bilateral fine end inspiratory crackles occur in 40-50 % of patients with CFA. C : Pleurisy is a common feature in CFA. D : Lymphangioleiomymatosis is commonly seen in tall young men. E : Goodpasture's syndrome is commonly seen in smokers. F : Haemoptysis is commonly seen in lymphangitic carcinoma. G : A raised serum angiotensin-converting enzyme (ACE) level is diagnostic of sarcoidosis. H : Hypercalcaemia is seen in 11-18% 0f patients with sarcoidosis. I : Transbronchial lung biopsy (TBLB) can result in pneumothorax in over 25% of patients. J : The presence of traction bronchiectasis on high resolution CT scanning in a patient with CFA

suggests reversibility. Comment : Diffuse parenchymal lung disease (DPLD) is the name given to the group of diseases that affect the lung parenchyma in a diffuse manner. Clubbing is seen in 49-66% of patients with CFA and in up to 75% of patients with rheumatoid arthritis related DPLD. Bilateral fine end inspiratory crackles have been reported in up to 90% of patients with CFA. Pleurisy is common in DPLD due to rheumatoid arthritis and systemic sclerosis. It is rare in CFA. Lymphangioleiomyomatosis is a disease of young pre-menopausal women and is characterised by proliferation of smooth muscle in the walls of pulmonary lymphatics, which can lead to obstruction and formation of achylous pleural effusion. It can also lead to airflow limitation. Pneumothorax also occurs commonly.Goodpasture's syndrome characterised by alveolar haemorrhage and renal impairment occurs more commonly in smokers.Haemoptysis is commonly seen in pulmonary vasculitis and not in lymphangitic carcinoma. Raised serum ACE levels are very non-specific and not diagnostic of sarcoidosis. TBLB can result in pneumothorax in 0.7-2% of cases, this is more common if samples are taken from the right middle lobe or the lingula. Tranction bronchiectasis on HRCT suggests established fibrosis and irreversibility. E : H:

Page 66: Masterclass Book Part 2

MOHAMMED IS-HAG 65

40. You are called to the resuscitation room to see a 25-year-old man whose condition has suddenly deteriorated. He had arrived 30 minutes earlier with a 2-hour history of central pleuritic chest pain and breathlessness. He collapsed while awaiting radiograph and now is agitated and cyanosed with pulse 120/min and BP 80/40. Oxygen saturation is reading 79%, with the patient breathing high flow oxygen via a re-breathe mask. Respiratory examination reveals reduced breath sounds in the right lung field with deviation of the trachea towards the left. Percussion is resonant bilaterally. What immediate course of action should you take? A : Arrange for urgent portable chest radiograph B : Contact ITU to arrange for the patient to be ventilated C : Insert large bore needle into left hemithorax D : Insert large bore needle into right hemithorax E : Check arterial blood gases and commence Bipap if hypoxia is confirmed. Comment : Initially the history might suggest a number of diagnoses, including cardiac tamponade, massive pulmonary embolism, haemothorax or aortic dissection; however the respiratory examination findings indicate that he almost certainly has sustained a spontaneous pneumothorax, which has now developed into a tension pneumothorax.This is now a peri-arrest situation, and there is no time to arrange for portable chest radiograph, before attempting to reduce the pressure in the right hemithorax with the insertion of a large bore needle. If the diagnosis is correct, insertion may be accompanied by a loud ‘hiss’.Positive pressure ventilation is relatively contraindicated in this situation, and will probably not be required once the lung has re-inflated. D : 41. A 38-year-old woman presents with a two-year history of worsening breathlessness. Results of preliminary investigation are as follows: spirometry – FEV1 83% predicted, Vital Capacity (VC) 79% predicted, FEV1/VC ratio 78%; chest radiograph reported as being within normal limits; blood gases (breathing air) - PaO2 9.6 kPa, PaCO2 4.1 kPa. What are the two most likely diagnoses? A : Chronic obstructive pulmonary disease B : Cardiac failure C : Diffuse parenchymal lung disease D : Pneumocystis carinii pneumonia E : Anaemia F : Bronchiectasis G : Asthma H : Pulmonary vascular disease I : Diabetes mellitus J : Primary hyperventilation. Comment : The spirometry effectively rules out obstructive lung disease / asthma. Hypoxia excludes hyperventilation and anaemia as primary causes. The differential diagnosis lies between diffuse parenchymal lung disease and a problem with the pulmonary vasculature.In diffuse parenchymal lung disease the patient may have a dry cough, but there may be no specific features. It will be important to ask about systemic / iatrogenic disorders associated with lung disease, and specific employment history / recreational interests may also be relevant. Note that the chest radiograph can appear entirely normal in patients with diffuse parenchymal lung disease.Regarding diseases of the pulmonary vasculature: a small number of patients with pulmonary embolism present with breathlessness alone, but primary pulmonary hypertension needs careful consideration in this case. C : H: 42. You follow up a 55-year-old male smoker in your clinic with stable emphysema. His FEV1 is 0.6 (22% predicted) and FVC is 2.3 (63% predicted) giving an FEV1:FVC ratio of 26%. His arterial blood gases show a PO2 of 7.1 and 7.2 on two separate occasions. He is mildly breathless at rest, but severely breathless on exertion and his saturation drops from 91% to 88%. Which of the following statements is true? A : The major cause for his breathlessness is hyperinflation not hypoxia. B : He has severe emphysema because his FEV1:FVC ratio is less than 40%. C : There is good evidence to suggest that he will benefit from long-term oxygen therapy. D : He should not be considered for pulmonary rehabilitation because he is a current smoker. E : He should have a yearly chest radiograph to look for lung cancer. Comment : Although hypoxic at rest, this is mild and not causing significant dyspnoea. His breathlessness worsens considerably without much change in oxygen tension, suggesting that the cause of his dyspnoea is hyperinflation of his chest which worsens on exertion. Severity of emphysema is defined by the British Thoracic Society (BTS) in relation to FEV1, not FEV1:FVC ratio. Mild is 60-80% predicted; moderate 40-60% and severe <40%. Evidence for long-term oxygen therapy (Medical Research Council and Nocturnal Oxygen Therapy Trials) is only available in non-smokers. Therefore, first, he may not benefit, and second he is at risk of burns if he carries on smoking. He will benefit from pulmonary rehabilitation. Furthermore, some rehabilitation courses offer smoking cessation therapy. There is no evidence to suggest annual chest radiographs will benefit any group of patients in terms of primary prevention. A :

Page 67: Masterclass Book Part 2

MOHAMMED IS-HAG 66

43. A tall thin 18-year-old man with sickle cell disease presents to the Accident and Emergency department with a 36-hour history of cough, fever, breathlessness and pleuritic chest pain. The most likely diagnosis is: A : Pulmonary embolism B : Myocardial infarction C : Pneumococcal pneumonia D : Pneumothorax E : Acute chest syndrome. Comment : This is a typical presentation of acute chest syndrome, which is one of the commonest causes of death in adults with sickle cell disease. Pulmonary embolism and pneumonia cannot be excluded, but are less likely diagnoses Key aspects of management are: 1. Give high flow oxygen via reservoir bag 2. Rapid intravenous infusion of 1 litre of 0.9% saline 3. Start intravenous antibiotics, e.g. ampicillin 500 mg qds 4. Intravenous opioid for pain, e.g. diamorphine 5 mg, with antiemetic 5. Prophylaxis against venous thromboembolism, e.g. enoxaparin 20 mg SC od 6. Call for specialist advice if the patient deteriorates or does not improve rapidly. Exchange transfusion may be indicated if the patient becomes hypoxic. E : 44. A 28-year-old man with long-standing asthma is admitted with an acute exacerbation. Which one of the following would suggest that he has a life-threatening attack? A : His peak flow is 40% of predicted / personal best. B : He is confused. C : He is using accessory muscles of respiration. D : His pulse rate is 120 / min. E : His respiratory rate is 40 / min. Comment : An attack of asthma is life-threatening asthma if there is: li> peak flow <33% predicted / personal best; 1. silent chest; 2. cyanosis; 3. altered consciousness, confusion or coma; 4. exhaustion, inability to speak; 5. hypotension or bradycardia. An acute attack of asthma is severe if there is: 1. peak flow <50% predicted / personal best; 2. pulse rate >110/min; 3. respiratory rate >25/min; 4. cannot complete sentences in one breath; 5. use of accessory muscles of respiration / intercostal recession. B : 45. A 35-year-old woman known to suffer from rheumatoid arthritis gives a history of cough with copious phlegm with intermittent haemoptysis. She also gets recurrent chest infections. She is on weekly methotrexate along with a non-steroidal anti-inflammatory drug (NSAID). Her chest radiograph shows linear radioluciences at both bases. A high-resolution computed tomography scan (HRCT) of the chest confirms bronchiectasis. What is the most likely cause? A : Foreign body aspiration B : Kartagener's syndrome C : Old tuberculosis D : Rheumatoid arthritis E : Cystic fibrosis. Comment : Bronchiectasis may be due to many causes. Post infection (TB, measles, whooping cough etc), foreign body aspiration, adenomas, hypogammaglobulinaemia, Kartagener's syndrome, Young's syndrome, cystic fibrosis, allergic bronchopulmonary aspergillosis are the main causes. Up to 4 % of patients with rheumatoid arthritis develop bronchiectasis. D : 46. A 43-year-old woman has been diagnosed as suffering from bronchiectasis on a high-resolution computed tomography (HRCT) scan of the lung. Which one of the following statement is NOT true? A : Her immunoglobulin levels should be checked. B : She is at risk of developing a pneumothorax. C : Massive haemoptysis is the commonest cause of death in her age group. D : She is at risk of developing brain abscess . E : Recurrent chest infections are unlikely at this age.

Page 68: Masterclass Book Part 2

MOHAMMED IS-HAG 67

Comment : Congenital and acquired immunodeficiency are well known causes of bronchiectasis, hence serum immunoglobulins should be checked, particularly the IgG subclass (IgG1,2,3 & 4). Patients with immunoglobulin deficiency should be treated with regular immunoglobulin infusion. Pneumothorax, recurrent chest infections and brain abscess are well recognised complications. Haemoptysis is a common symptom but rarely causes death. C : 47. You are reviewing a 76-year-old man in clinic. He has been diagnosed as having chronic obstructive pulmonary disease (COPD) and results of arterial blood gases (according to the last Consultant letter in the case notes) suggest the patient might benefit from home oxygen. Which two of the following are correct in relation to domicillary long-term oxygen therapy (LTOT)? A : LTOT for 10 hours/day improves survival in patients with severe hypoxemia (pO2<8.0kPa). B : LTOT increases survival in patients with moderate hypoxemia. C : Nocturnal oxygen therapy improves survival in mild to moderate daytime hypoxemia. D : Patients should stop smoking after being prescribed LTOT. E : Nocturnal oxygen therapy improves survival in patients nocturnal sleep desaturation. F : LTOT should be prescribed for COPD patients who when stable have a resting PaO2<7.3kPa. G : LTOT is most economically provided by an oxygen concentrator which cannot be prescribed on the NHS. H : Prescriptions should specify both the amount of oxygen required and a flow aimed at keeping

PaO2 at rest above 8.0kPa. I : LTOT should be prescribed for patients with peripheral oedema and a PaO2 of<10kPa. J : In the UK, LTOT can only be prescribed by a respiratory physician. Comment : LTOT for at least 15 hours/day improves survival in patients with severe hypoxemia LTOT does not increase survival in patients with mild to moderate hypoxemia. Nocturnal oxygen therapy does not improve survival in patients with mild to moderate daytime hypoxemia Patients should stop smoking before being prescribed LTOT · Nocturnal oxygen therapy does not improve survial in patients with nocturnal sleep deprivation · LTOT should be prescribed for patients with COPD who, when stable, have a resting PaO2<7.3kPa or between 7.7 to 8.0kPa plus at least one of the following:-secondary polycythaemia, nocturnal hypoxemia, peripheral odema or pulmonary hypertension · LTOT is most economically provided via an oxygen concentrator, which can be prescribed on the NHS · Prescriptions should specify the amount of oxygen and the rate of flow aimed at keeping PaO2 at rest above 8.0kPa · LTOT should be considered if patients have PaO2 7.3-8.0kPa if they also have peripheral oedema · In Scotland LTOT can only be prescribed by a Respiratory Physician however in England, Wales and Northern Ireland LTOT can be prescribed by general practitioners F:H: 48. In which of the following emergency medical presentations is non-invasive ventilation an established treatment? A : Tension pneumothorax B : Acute asthma C : Community acquired pneumonia D : Acute exacerbation of chronic obstructive pulmonary disease E : Pulmonary oedema. Comment : Although non-invasive ventilation (NIV) is currently being evaluated in an number of emergency situations, the best evidence relates to exacerbations of chronic obstructive pulmonary disease (COPD), and in particular, patients with decompensated Type II respiratory failure. In these patients, physiological responses (heart and respiratory rate, and arterial blood gases) improve more quickly with NIV compared to standard treatment. Intubation is also less frequently required. D : 49. A 60-year-old man with chest pain is anxious of the effect of asbestos exposure that he might have incurred in his previous jobs. He worked as a thermal-insulating engineer for 10 years from the age of 16. He has smoked 20 cigarettes a day since he was 16. Which one of the following statements is correct? A : The fact that he worked with asbestos at young age is a risk factor for mesothelioma. B : Amosite (brown asbestos) and crocidolite (blue asbestos) are less potent carcinogens than

chrysotile (white asbestos). C : His smoking increases the possibility of mesothelioma. D : His asbestos exposure is a risk factor for bladder cancer. E : Chemotherapy is the main stay of treatment of patient with mesothelioma. Comment : Asbestos exposure is known to be a risk factor for mesothelioma and lung cancer. The relationship with other cancers is not certain. The rate of mesothelioma is increasing. This is mainly due to the long latent period between exposure and the clinical development of the disease. Amosite and crocidolite are more potent carcinogens than chrysotile. Exposure at an early age is another risk factor. Cigarette smoking does not increase the risk of mesothelioma.The majority of patients with mesothelioma present with advanced disease. Chemotherapy and radiotherapy alone are not helpful. Tri-modality treatment with surgery, chemotherapy and radiotherapy may be helpful in selected cases. A :

Page 69: Masterclass Book Part 2

MOHAMMED IS-HAG 68

50. A 38-year-old man presents with breathlessness and cough. He is unwell, with high fever, and has signs of consolidation in his right lower lobe. The most likely pathogen is: A : Legionella B : Neisseria meningitidis C : Streptococcus pneumoniae D : HIV E : Staphylococcus aureus. Comment : Pneumococcus remains the commonest cause of community acquired pneumonia. In giving empirical treatment you would want to cover for the possibility of Legionella and other atypicals (by giving clarithromycin or erythromycin) but these would be a less likely cause of this man’s illness. C : 51. A 77-year-old man is admitted as an emergency with increasing shortness of breath and a productive cough with green purulent sputum. His chest radiograph shows hyperinflation but no consolidation. Arterial blood gases shows an acute respiratory acidosis. Which of the following statements is true? A : Oxygen should not be given if he has significant hypercapnia. B : Bronchodilators in the acute exacerbation have no effect on forced vital capacity. C : Beta-adrenergic agonists are superior to anticholinergic agents. D : There are fewer side effects with ipratropium bromide than with beta adrenergic agonists. E : Metered dose inhalers are more expensive than nebulisers. Comment : Oxygen should be given for exacerbation of chronic obstructive pulmonary disease, aiming for oxygen saturation values of 90-92% with corresponding partial pressures of arterial oxygen of 60-65 mmHg. These targets ensure acceptable haemoglobin saturation whilst reducing the liklihood of the hypercapnia that can accompany the use of supplemental oxygen. Bronchodilators increase the FEV1 and forced vital capacity (FVC) by 15 to 29% over 60 to 120 minutes. Beta adrenergic agonists are often used as first-line treatment for acute exacerbations of chronic obstructive pulmonary disease, but have not been shown to be superior to anticholinergic agents and the two classes of agent are often used in combination. Side effects are less frequent and milder with ipratropium bromide than with beta agonists. Metered dose inhalers are cheaper than nebulisers. D : 52. A 60-year-old gentleman is referred to the hospital as a routine chest radiograph has shown bilateral apical scars with possibility of cavity formation. He is complaining of some cough for over 8 months and is known to suffer from mild chronic obstructive pulmonary disease (COPD). There is no history of immunosuppression and repeated sputum samples have grown Mycobacterium avium complex. Which of the following is true? A : Treatment should be the same as Mycobacterium tuberculosis but with an 8-month continuation phase using isoniazid and rifampicin. B : Patient should be notified within 1 week. C : Extrapulmonary disease is uncommon in immunocompetent patients. D : Surgery has no role in treatment. E : It commonly affects young fit men. Comment : M. avium complex (MAC) is a saprophyte and can be found in soil, water, dust etc. It typically affects middle age and old men with underlying lung diseases such as COPD, old TB, bronchiectasis. It may be contaminant, hence to make a diagnosis there should be radiographic evidence of disease (cavities etc), and it should be isolated in at least three sputum samples. Extrapulmonary disease is seen in immunosupressed patients. Recommended treatment is with rifampicin and ethambutol for 24 months. In fit patients, surgery resection of the diseased lung along with chemotherapy can cure. C : 53. You have been asked to see a 56-year-old gentleman in A&E. He was brought in with type I respiratory failure, has already been intubated and is waiting to be transferred to ITU. His wife says that he is quite a fit man and is not on any medication. He developed cough, fever, headaches and body aches 1 week ago. Since the last 24 hours he has developed dyspnoea on exertion. His wife found him unresponsive today. He is a car salesman by profession and keeps racing pigeons. His wife also mentions that two of his favourite pigeons died 2 weeks ago. Examination is consistent with a pneumonic illness. He has mild splenomegaly and a pale macular rash on the abdomen, anterior chest wall and legs. Which of the following is the likely causative organism? A : Staphyloccocus aureus B : Legionella pneumophila C : Mycoplasma pneumoniae D : Chlamydia psittaci E : Haemophilus influenzae. Comment : Psittacosis is a systemic illness with primarily pulmonary manifestations. The diagnosis is suggested by exposure to pigeons. The rash (Horder's spots) and splenomegaly are characteristic. Diagnosis can be made by a four-fold increase in the complement-fixing antibodies. Tetracycline for 2 weeks is the treatment of choice. D :

Page 70: Masterclass Book Part 2

MOHAMMED IS-HAG 69

54. A 20-year-old man gives a six-month history of worsening breathlessness and coughs up half a cupful of sputum daily. He was occasionally wheezy with viral illnesses as a young child, works in a ship yard and has smoked 20 cigarettes per day for the last four years. The most likely diagnosis is: A : Asthma B : Asbestosis C : Lung cancer D : Chronic obstructive pulmonary disease E : Bronchiectasis. Comment : Of the diagnoses listed, only bronchiectasis could explain the amount of sputum that this man coughs up each day. Possible causes to consider would include cystic fibrosis, alpha-1 antitrypsin deficiency and hypogammaglobulinaemia. E : 55. A 75-year-old man with a 30-pack year cigarette smoking history complains of continuous right shoulder pain, a persistent cough and weight loss. His chest radiograph shows a right apical shadow. On examination you note that he is clubbed, has a small right pupil and a right-sided ptosis. What is the most likely diagnosis? A : Small cell lung cancer B : Squamous cell carcinoma C : Bronchoalveolar carcinoma of the lung D : Adenocarcinoma of the left lung E : Bronchial carcinoid. It is likely that he has a right apical carcinoma of the lung causing a Horner’s syndrome (ipsilateral ptosis, meiosis and anhydrosis). Small cell lung cancer generally arises centrally as opposed to the more peripheral lesions of non-small cell lung cancer. Squamous cell carcinoma accounts for about 30% of all lung cancers and arises most frequently in proximal segmental bronchi. Most of adenocarcinomas of the lung are peripheral in origin. The tumour in this case is on the right and not the left. B : 56. A 43-year-old woman presents with breathlessness that has been getting gradually worse over a few weeks and now makes it difficult for her to walk upstairs. On physical examination she is found to have a large left sided pleural effusion but no other abnormalities. The presence of the effusion is confirmed by chest radiography. The most appropriate initial investigation would be: A : CT chest. B : Diagnostic aspiration of pleural fluid followed by drainage of effusion to dryness C : Diagnostic aspiration of pleural fluid with pleural biopsy D : Sputum cytology E : Diagnostic aspiration of pleural fluid. Comment : The first investigation should be diagnostic aspiration of pleural fluid for biochemical, microbiological and cytological analysis.Light’s criteria can be used to distinguish transudates from exudates: in exudates at least one of the following three criteria are met – pleural fluid protein concentration greater than 50% of that in plasma; pleural fluid LDH greater than 60% of that in plasma; pleural fluid LDH more that two thirds the upper limit of normal in plasma.Transudative pleural effusions are most commonly due to congestive cardiac failure but are sometimes associated with hypoproteinaemic states such as cirrhosis or nephrotic syndrome. Most other causes of pleural effusion are exudative. E : 57. A 60-year-old-man complained of pain in both wrist evolving over 8 weeks. He noted swelling around that area but denied stiffness. On examination there was swelling and tenderness just proximal to the wrist joints without limitation of movement. There was also prominent finger clubbing. Radiographs revealed periosteal reaction over the lower end of the radius and ulnar. Each of the following disorders could be the cause behind this patient complain EXCEPT? A : Mesothelioma B : Bronchiectesis C : Diabetes mellitus D : Crohn's disease E : Whipple's disease. Comment : Hypertrophic osteoarthropathy (HOA) is a syndrome characterized by proliferative changes in the skin and skeleton. Proliferative periostitis of the radius and fibula and digital clubbing are commonly seen. Two forms of the syndrome are seen, a rare idiopathic from called pachydermoperiostitis (3-5%) (familial autosomal dominant), and more common secondary form (95-97%). Secondary HOA was initially described in association with chronic suppurative infection and malignancy of the lung and pleura. Therefore it used to be called hypertrophic pulmonary osteoarthropathy (HPOA). Pleural causes include pleural fibroma and mesothelioma. Pulmonary causes include bronchogenic carcinoma, pulmonary tuberculosis; pulmonary abscesses, bronchiectasis, emphysema; and Pneumocystis carinii infection in patients with AIDS, Hodgkin’s disease, metastases, or cystic fibrosis. Cyanotic heart disease with a right-to-left shunt is the only cardiac cause described.

Page 71: Masterclass Book Part 2

MOHAMMED IS-HAG 70

Abdominal causes include liver cirrhosis, ulcerative colitis, Crohn disease, Whipple disease, and biliary atresia. In long standing insulin dependent diabetes mellitus, contracture of the flexor tendons of the fingers with tight, waxy skin appearance leads to diabetic cheirarthropathy. There is no pain and minor functional impairment. Diabetes mellitus is not associated with HOA. C : 58. A 56-year-old women, a smoker of 20 cigarettes daily, presents with a 6-month history of progressive shortness of breath. Her past medical history is unremarkable apart from Raynaud’s syndrome for which she takes a calcium channel blocker. On examination no significant abnormality is found apart from telangiectasia. Her chest radiograph shows clear lung fields, prominent pulmonary arteries and mildly enlarged heart. Spirometry is normal, but gas transfer is reduced to 50% predicted. What is the most likely diagnosis? A : Cor pulmonale secondary to chronic obstructive pulmonary disease B : Multiple pulmonary emboli C : Pulmonary arterial hypertension D : Sarcoidosis E : Congestive cardiac failure . Comment : Normal spirometry excludes chronic obstructive pulmonary disease. Raynaud’s syndrome with telangiectasia and radiological appearances suggestive of pulmonary hypertension with impaired gas transfer are most likely due to a vasculitic process in pulmonary circulation associated with an autoimmune rheumatic disorder. C : 59. You are asked to speak to a 58 yr old man and his wife on the ward. He has been admitted for routine surgery (cholecystectomy) and when clerked says that he worked in shipyards for many years. The house surgeon enquires about asbestos exposure, and this leads to a great deal of anxiety. His wife is very keen to know what medical risks this poses for him. Which one of the following statements regarding asbestos exposure or asbestos related disease is true? A : pleural plaques are often associated with restrictive defect in respiratory function tests. B : the risk of mesothelioma increases with smoking. C : the risk of lung cancer increases by 50-fold in asbestos workers who smoked compared to

persons who neither smoked nor worked with asbestos. D : chemotherapy is the treatment of choice in patients with mesothelioma. E : adenocarcinoma is the predominant histological type in asbestos workers with lung cancer. Comment : Asbestos exposure is associetd with: · pleural plaques · diffuse pleural thickening · lung fibrosis (also known as asbestosis) · lung cancer · mesothelioma. The commonest abnormality seen in asbestos workers is pleural plaques, which are well-circumscribed areas of thickening affecting the parietal pleura. Pleural plaques are even seen in patients with mild asbestos exposure. In the majority of patients, these plaques are asymptomatic and do not cause any changes in respiratory function tests.Mesothelioma is a tumour of the mesothlial cells or the pro-mesothelial cells. It affects the pleura and, to a lesser extent, the pericardium and the peritoneum. The incidence of mesothelioma increases in patients with heavy asbestos exposure and in those who were exposed to asbestos at a young age. Cigarette smoking does not increase the incidence of mesothelioma.Mesothelioma is a fatal disease. The majority of patients die within 18 months of diagnosis. The disease is resistant to current treatment modalities such as chemotherapy and radiotherapy. Tumor resection is indicated in only a small proportion of patients.Lung cancer is associated with heavy asbestos exposure. The effects of asbestos exposure and smoking are multiplicative. The risk of lung cancer in asbestos workers who never smoke is increased by 5-fold. In smokers the risk is about 55-fold that in non-smoking persons who never worked with asbestos. C : 60. A 28-year-old man with end stage renal failure due to IgA nephropathy received a renal transplant 9 months previously. Progress was complicated by several episodes of rejection that required high-dose steroid treatment. He now presents with malaise, fever and breathlessness. He is hypoxic (arterial oxygen saturation 88% on air) and his chest radiograph shows patchy interstitial shadowing. Which one of the following statements regarding Pneumocystis carini pneumonia (PCP) in non-HIV infected patients is NOT correct? A : Patients who are receiving more than two weeks of high dose corticosteroids with or without

other immunosuppressive agents are at risk of PCP. B : Non-AIDS patients generally have a more fulminant onset of symptoms than HIV-infected

patients. C : In contrast to AIDS patients, induced sputum and broncho-alveolar lavage specimens are less

often diagnostic. D : The duration of therapy in non-AIDS patients is 14 days compared to 21 days for AIDS patients. E : The outcome of PCP in non-AIDS patients is generally better than in those with AIDS.

Page 72: Masterclass Book Part 2

MOHAMMED IS-HAG 71

Comment : In contrast to AIDS patients who can have a high burden of organisms, induced sputum or even specimens obtained by broncho-alveolar lavage (BAL) are less often diagnostic in other immunocompromised patients who have a lesser burden of organisms. Hence when clinical suspicion for PCP is high and sputum or BAL negative, tissue should be obtained.The duration of treatment recommended for patients with AIDS is 21 days, based upon the burden of organisms, slower response time and observed early relapses after standard treatment for 14 days. Non-AIDS patients with PCP generally respond to therapy within four to five days. E : 61. A breathless 70-year-old smoker presents with the following lung function tests. What is the most likely diagnosis? FEV1 1.5L (60%)/ FVC 1.8 L (55%)/ FEV1/FVC ratio = 84%/ TLC = 66% predicted/ RV = 57% predicted/ TLCO = 55% predicted/ KCO = 60% predicted A : Emphysema B : Fibrosing alveolitis C : Anaemia D : Asthma E : Obesity Comment : The lung function tests show a significant restrictive defect. Only fibrosing alveolitis or obesity may fit this picture but given the decrease in KCO (i.e. after correcting for alveolar volumes), the most likely answer is fibrosis, as pure obesity would not affect the KCO as the gas exchange after correcting for the alveolar volume would in fact be high. B : 62. A 26-year-old patient with known cystic fibrosis (CF) presented with weight loss, urinary frequency and frequent infective exacerbation of his chest problem. The most likely complication of his CF is: A : Liver cirrhosis B : Diverticulitis C : Secondary amyloidosis D : Diabetes mellitus E : Renal tubular acidosis. Comment : Approximately 20% of adult patients with cystic fibrosis develop diabetes mellitus (DM). The reason for this is direct damage to the insulin-producing islets. The commonest manifestations of DM in CF patients are weight loss, increased frequency of infective exacerbation, as well as decline of lung function tests. Increased urinary frequency is not specific for DM in adult CF patients. For unknown reasons, renal tubular acidosis and secondary amyloidosis are quite rare in CF patients, despite the highly damaged lung which is frequently colonized with bacteria causing repeated clinical exacerbations.Liver cirrhosis occur in 5-10% of adult CF patients. The manifestations are similar to those in non-CF patients.Diverticulits is very rare in treated CF patients. D : 63. A 29-year-old woman presents with a 3-month history of malaise, joint pain and arthralgia. Three weeks ago, she noticed a painful rash on her shins. Her chest radiograph reveals large lymph nodes in the hilar and paratracheal areas. Which two of the following features would NOT be expected in a case of sarcoidosis? A : Facial skin rash B : Arthritis in the small feet joint C : Anterior uveitis D : Superior vena cava obstruction E : Lung nodules F : Splenomegaly G : Renal stones H : Ulcerated cervical lymph nodes I : Complete heart block J : Cardiomyopathy. Comment : Sarcoidosis is characterised by formation of granulomas, often in many organs. Lymphadenopathy is the most common feature of sarcoidosis, in which the glands are rubbery, soft and tend not to compress adjacent organs. Features such as superior vena cava obstruction, laryngeal nerve palsy and phrenic nerve palsy due to mediastinal lymph gland enlargement are very rare in sarcoidosis, and if present they suggest another diagnosis such as lymphoma or lung cancer. Biochemical features of sarcoidosis include hypercalcaemia and hypercalciuria which are due to secretion of vitamin D-like material from the epithelioid cells in granulomata. Hypercalciurea may lead to renal stone formation. The heart can be involved in sarcoidosis. The conduction system may be infiltrated with granulomas leading to branch blocks or to complete heart block. Cardiomayopathy is also seen. Hepatomagaly or splenomagaly are reported in a quarter of patients with sarcoidosis. The spleen is directly involved with the disease in most cases, with enlargement rarely due to portal hypertension. D : H:

Page 73: Masterclass Book Part 2

MOHAMMED IS-HAG 72

64. A 61-year-old woman with rheumatoid arthritis is referred with a history of recurrent chest infections, intermittent wheeze and production of half a teacupful of phlegm daily, on occasions with a streak of blood. She is a retired secretary and has never smoked. What is the most likely diagnosis? A : Chronic bronchitis and emphysema B : Lung cancer C : Diffuse interstitial lung fibrosis D : Bronchiectasis E : Tuberculosis. Comment : Bronchiectasis is associated with rheumatoid arthritis, occurring in 3-4% of patients with this condition. As with all other causes of bronchiectasis, it presents with recurrent chest infections and excessive phlegm. Recurrent haemoptysis is a common feature.Chronic obstructive pulmonary disease (COPD) and lung cancer are unlikely in a non-smoker. Tuberculosis should be excluded as it can cause bronchiectasis and haemoptysis.Interstitial fibrosis occurs in up to 20% of patients with rheumatoid arthritis but is not associated with sputum production or haemoptysis (unless there is an associated cancer).The woman should undergo pulmonary function tests to assess the presence of airway obstruction, which is associated with bronchiectasis. A HRCT scan of the lungs is the investigation of choice to confirm presence of bronchiectasis. D : 65. A 48 yr old woman presents with a pleural effusion 3 weeks after receiving antibiotics from her general practitioner for a chest infection. You suspect a parapneumonic effusion. Which one of the following statements regarding parapneumonic effusions is true? A : is any effusion associated with bacterial pneumonia, lung abscess or bronchiectasis. B : always need to be drained. C : is commonly due to mycobacteria. D : should be treated surgically. E : should never be aspirated. Comment : Any pleural effusion associated with bacterial pneumonia, lung abscess or bronchiectasis is a parapneumonic effusion. Up to 40% of patients with pneumonia have an accompanying pleural effusion and the majority resolve without any specific therapy directed towards the pleural fluid. The evolution of these effusions can be divided into 3 stages: · exudative · fibropurulent · organistaion. During these stages the pH and glucose level of the pleural fluid becomes progressively lower. These effusions can be monitored by the pH: · pH<7 - absolute indication to insert an intercostal drain · pH>7.20 - usually do not need drainage · pH 7-7.20 - should be monitored and intercostal drain inserted if the pH swings towards 7. Prior to the antibiotic era Streptococcus pneumoniae or haemolytic streptococci were the commonest organisms. Anaerobic organisms are commonly seen nowadays. A : 66. A 65-year-old man presents with a history of worsening breathlessness and cough. His arterial blood gases show the following recordings when taken at room air: pH 7.26 /pO2 6.5kPa/pCO2 9.5 kPa /Bicarbonate 32mmol/L. Which of the following is the most likely diagnosis? A : Obstructive sleep apnoea (OSA) B : Acute asthma C : Pulmonary embolus D : Acute exacerbation chronic obstructive pulmonary disease (COPD) E : Pulmonary oedema. Comment : Although OSA can cause chronic respiratory failure, it is unusual to have chronic type 2 respiratory failure except in combination with some other cardiopulmonary illness. This blood gas would be most compatible with a patient with severe COPD and chronic type 2 respiratory failure with an acute exacerbation. D : 67. In pulmonary function tests, the FEF 25%-75% (Forced Expiratory Flow rate between 25% and 75% of the Forced Vital Capacity): A : is effort dependent B : reflects the status of the large airways C : is useful to identify tracheal obstruction D : is not affected in smokers E : is impaired in bronchiolitis obliterans. Comment : The FEF 25%-75% primarily reflects the status of the small airways and is more sensitive than the FEV1 for identifying early airway obstruction. It is effort independent and is impaired in bronchiolitis obliterans, smokers, rejection reactions in bone marrow transplant, lung and heart transplant. E :

Page 74: Masterclass Book Part 2

MOHAMMED IS-HAG 73

68. Which of the following is true regarding sarcoidosis? A : Erythema nodosum occurs on the lower limb only. B : The adrenal glands are commonly involved. C : It is known to cause pleural effusions. D : Hypercalcaemia occurs in over half of the patients. E : A raised serum ACE is diagnostic. Comment : Erythema nodosum is a painful nodular panniculitis that may occur on the shins, calves and outer aspects of the arms. Sarcoidosis may involve any organ of the body. The adrenal glands are not commonly involved. Exudative pleural effusions are known to occur in sarcoidosis. It may also cause chylothorax. Hypercalcemia occurs in 2-10% of patients. A raised serum angiotensin-converting enzyme (ACE) level is not diagnostic as it may be raised in diabetes mellitus, hyperthyroidism, alcohol liver disease or silicosis asbestosis. C : 69. Rheumatoid arthritis can affect the lungs in all of the following ways, except: A : Pleural effusion B : Bronchiectasis C : Bronchiolitis obliterans D : Chylothorax E : Interstitial pulmonary fibrosis. Comment : Rheumatoid arthritis can affect the lungs in numerous ways. Rheumatoid nodules can mimic a lung mass. Chylothorax is not recognised to occur with rheumatoid arthritis. D : 70. Occupational asthma: A : Represents one in a thousand of all asthma cases. B : Specific IgE to the suspected material is increased up to three years after cessation of exposure. C : Specific provocation test with the suspected material is legally required before the worker

becomes eligible for compensation. D : Two-hourly recordings of peak flow during two weeks at work and two weeks off work is the

investigation of choice in diagnosing occupational asthma. E : Avoidance of exposure is not always necessary in patients with occupational asthma. Comment : Occupational asthma is defined as asthma induced or aggravated by work. 3-6% of all asthma cases are suspected to be occupational in nature. The highest groups at risk include: · spray painters (isocyanate) · plastic makers (epoxy resins) · farmers (grain dust and wood dust) · bakers (flour). The diagosis of occupational asthma hinges on a careful history of work-related symptoms and periods of improved symptoms during weekends and holidays. Full blood count may show eosinophilia. Specific IgE may be raised but it tends to disappear within six months of leaving work. Static respiratory function tests are often normal, but non-specific provocation tests with histamine and methacholine may indicate hyper-reactivity. The main objective test in occupational asthma is the two-hourly serial measurement of peak flow for at least two weeks while at work and two weeks while away from work. A specific provocation test is required when there is an unrecognised material which might cause asthma, or when there is more than one material implicated in the symptoms. It is not required legally to prove the diagnosis. In all cases of occupational asthma avoidance of exposure is the mainstay of management.D : 71. Regarding Legionnaires disease, which of the following statements is correct? A : The causative organism, Legionella pneumophila, does not spread by contaminated aerosols. B : The incubation period is 2-10 days. C : Pleural effusions do not occur. D : Rifampicin with penicillin is the treatment of choice. E : It is not associated with cavitation. Comment : Legionnaires disease was first recognised in the late 1970’s following an outbreak of disease among the Pennsylvania state American Legion conventioneers in Philadelphia. Legionella pneumophila is an obligate aerobic gram negative bacilli. It is associated with both sporadic and epidemic infections. The natural habitat of the bacterium is water and it is distributed widely throughout natural and man-made reservoirs. It spreads through contaminated aerosols. The incubation period is 2-10 days and the spectrum of the disease varies from asymptomatic seroconversion to severe life-threatening pneumonia. Constitutional symptoms such as headache, fever, rigors, myalgias and lethargy are frequent. Cough which is non-productive initially becomes more pronounced and productive. Haemoptysis occurs in one third of the patients. Extrapulmonary manifestations such as diarrhoea, haematuria, arthralgias, renal failure, hepatic function abnormality, hyponatremia may occur. Small pleural effusions are found in 50% of the patients. Cavitation occurs in some patients, particularly those receiving corticosteroids. Erythromycin or doxycycline are the drugs of choice and treatment should be continued for at least 3 weeks to prevent relapse. Rifampicin in a dose of 600mg should be added to the treatment in patients who are severely ill or who have cavitation. B :

Page 75: Masterclass Book Part 2

MOHAMMED IS-HAG 74

72. A 55-year-old gentleman with a smoking history of 50 pack years has been diagnosed to be suffering from a lung mass based on a chest radiograph. He is presently waiting a bronchoscopy. You predict that this is a non-small cell lung cancer. Which one of the following does not support your prediction? A : Hypertrophic pulmonary osteoarthropathy (HPOA) B : Hypercalcaemia with a normal radioisotope bone scan C : Syndrome of inappropriate antidiuretic hormone secretion (SIADH) D : Disseminated intravascular coagulation (DIC) E : Thrombocytosis. Comment : Paraneoplastic syndromes are commonly seen with lung cancer. SIADH is most comonly seen with small cell carcinoma. HPOA, hypercalcaemia without bone metastasis is more common in squamous cell carcinoma. DIC and thrombocytosis are more common with adenocarcinoma. C : 73. A 37-year old white south African man presents with a 3-week history of progressive shortness of breath that has not improved following two courses of antibiotics (amoxicillin and erythromycin). He does not keep any pets and is on no other medication. On auscultation his chest is clear, but he is profoundly hypoxic. What is the most likely diagnosis? A : Extrinsic allergic alveolitis B : Sarcoidosis C : Pneumocystis carini pneumonia D : Pulmonary tuberculosis E : Wegener's granulomatosis. Comment : Severe hypoxia with relatively minimal radiological changes makes Pneumocystis carinii the most likely diagnosis. C : 74. Malignant mesothelioma: A : is a pulmonary malignancy due to asbestos. B : is a pulmonary malignancy due to coal dust. C : is treated with radiotherapy. D : has a median survival of 3 years. E : is usually diagnosed with pleural fluid cytology. Comment : Malignant mesothelioma is a pleural malignancy in which there is almost always a history of asbestos contact. No specific treatment has been found to be of benefit, except radiotherapy, which reduces seeding and invasion through percutaneous biopsy sites. Median survival is approximately 12 months. Pleural fluid provides a diagnosis less than 50% of the time, and even multiple pleural biopsies can be negative in some cases due to a florid fibrotic reaction. C : 75. In eosinophilic bronchitis: A : haemoptysis is a common symptom B : cough is not a symptom C : absence of variable airflow obstruction in the presence of sputum eosinophilia is diagnostic D : inhaled corticosteroids are not helpful E : peripheral blood eosinophilia is commonly seen. Comment : Eosinophilic bronchitis (EB) presents with chronic cough and sputum eosinophilia without the abnormalities of airway function seen in asthma. It responds to inhaled corticosteroids. Haemoptysis is not a symptom of EB. C : 76. A 19-year-old Indian man presents with a 6-week history of cough, fever and weight loss. Acid fast bacilli are seen on sputum smear and he is commenced on antituberculous therapy. His condition deteriorates and his culture subsequently grows M. tuberculosis resistant to rifampicin, isoniazid and ethambutol. Which of the following is incorrect regarding subsequent management? A : He should be admitted to a negative pressure isolation cubicle B : He should be isolated until he has had 2 weeks therapy with at least three active drugs C : During isolation, all visitors should wear appropriate masks to filter out M. tuberculosis D : All close contacts should be screened for tuberculosis E : On discharge he should receive directly observed therapy. Comment: Patients with multidrug resistant tuberculosis should be isolated in a negative pressure isolation room. These patients should be treated with five active drugs including an injectable preparation such as amikacion or streptomycin. He should remain isolated until his sputum smears have become negative or culture of a smear-positive sputum is negative - this often means isolation for 3-4 months. Approximately 1% of UK isolates of MTB are multidrug resistant and about 6% are isoniazid-resistant. Resistance is more likely in patients who have been previously treated (but not complied) and those who acquired disease abroad. B :

Page 76: Masterclass Book Part 2

MOHAMMED IS-HAG 75

77.

A 42-year-old West Indian woman presents with a painful rash on her legs, weight loss, night sweats and the following chest radiograph (see image). What is the most likely diagnosis? A : Carcinoma bronchus B : Tuberculosis C : Sarcoidosis D : Lymphoma E : Streptococcal infection. Comment : The history is most compatible with erythema nodosum and therefore in combination with night sweats one should consider either TB or sarcoidosis. Given the chest radiograph of bilateral hilar lymphadenopathy, the likeliest diagnosis is sarcoidosis. C : 78. A breathless 23-year-old woman has the following lung function tests: FEV1 1.1L (60%)/ FVC 1.3 L (55%)/ FEV1/FVC ratio = 84%/ TLC = 66% predicted/ RV = 57% predicted/ TLCO = 55% predicted/ KCO = 110% predicted What is the most likely diagnosis? A : Acute sickle crisis B : systemic lupus erythematosus (SLE) pneumonitis C : Scoliosis D : Asthma E : Cystic fibrosis. Comment : The lung function tests show a significant restrictive defect. Only kyphoscoliosis or a pneumonitis may fit this picture but given the normal/high KCO (i.e. after correcting for alveolar volumes), the most likely answer is kyphoscoliosis as the gas exchange after correcting for the alveolar volume would in fact be high. C : 79. A 21-year-old patient with cystic fibrosis (CF) presented with weight loss and repeated infective exacerbations. Her fasting blood glucose was found to be 18 mmol/litre (324 mg/dl). Which of the following statements is correct? A : She will be prone to repeated infected exacerbations of her lung disease. B : She must start a glucose-reduced diet and oral hypoglycaemic agents as a first-line treatment. C : She is likely to develop retinopathy within 2 years if her disease is untreated. D : Gaining weight should be avoided to reduce insulin resistance. E : CF-Related diabetes is a contraindication for heart lung transplant. Comment : CF-related diabetes is seen in up to 20% of adult patients with CF. The manifestations of the disease are with weight loss and rapid decline in lung function.Glucose levels fluctuate for the first few years, but are consistently raised during exacerbations. Diet is not a suitable management for CF-related diabetes. The use of oral hypoglycaemic agents is controversial. The treatment of choice is injectable insulin. Due to the short life latency of CF patients and that this may be due to the different natural history of the disease, microvascular complications of CF-related diabetes are not common. The condition is not a contraindication for organ transplantation when it is well-controlled. A :

Page 77: Masterclass Book Part 2

MOHAMMED IS-HAG 76

80. Regarding narcolepsy: A : cataplexy is not the most specific symptom B : polysomnography is diagnostic C : it is associated with high levels of hypocretin 1 in the cerebrospinal fluid D : patients are known to have an unusually high rate of HLA-DQB1*0602 E : it is best treated with nasal continuous positive airway pressure (CPAP). Comment : Narcolepsy was first described in 1880 by Jean Baptiste E. Gelineau. It is characterized by excessive day time sleepiness and abnormal rapid eye movement (REM) sleep regulation. Recently it has been reported that the neuropeptide, hypocretin 1 levels in the CSF are at a lower level than in normal subjects. REM sleep represents 20-25% of an adults sleep and occurs in about 5 periods during the night, each following a cycle of NREM sleep. Cataplexy is the most specific symptom and is characterised by partial or complete muscle weakness that can involve the face, neck, legs or total body. The patients are aware of their surroundings but are not able to move. This may present as falls, clumsiness, dropping articles, etc. Cataplexy can be triggered by strong emotions such as laughter, anger, surprise etc. Sleep paralysis and hypnagogic hallucinations are the other symptoms seen with narcolepsy. Polysomnography is the gold standard for diagnosing obstructive sleep apnoea (OSA). Multiple sleep latency test (MSLT) is used to diagnose narcolepsy. MSLT measures the time required for a subject to fall asleep during 4-5 nap opportunities. The presence of REM sleep during at least 2 such naps is diagnostic of narcolepsy. Prior to performing MSLT, OSA should be excluded by polysomnography. Patients with narcolepsy are known to have an unsually high rate of HLA-DQB1*0602. Narcolepsy is treated using stimulant medication. Methylphenidate, amphetamine, modafinil & pemoline have all been tried. D : 81. Regarding pneumothorax, which statement is true: A : Traumatic pneumothorax can usually be safely observed without intervention. B : Patients with spontaneous primary pneumothorax can fly within 6 weeks as long as they have

access to oxygen. C : Patients with asymptomatic secondary pneumothorax can be discharged from A&E to be

reviewed by their GP the following day. D : Patients may only dive after bilateral surgical pleurectomy even if the pneumothorax was

unilateral. E : Patients with emphysema should have pleurodesis or surgical intervention after their first

pneumothorax because of higher risk of recurrence. Comment : Traumatic pneumothorax usually requires intervention. Patients are only allowed to fly once resolution of pneumothorax has been demonstrated radiographically and 6 weeks after the event. Diving is high risk in those with a risk of recurrent pneumothorax and can only be undertaken after bilateral pleurectomy. All patients with secondary pneumothorax should be observed in hospital even those without symptoms. According to the statistical and perceived risk of recurrence, accepted indications for operative intervention are as follows: · Second ipsilateral pneumothorax · First contralateral pneumothorax · Bilateral spontaneous pneumothorax · Persistent air leak (>5–7 days of tube drainage; air leak or failure to completely · re-expand) · Spontaneous haemothorax · Professions at risk (e.g. pilots, divers) D : 82. A 55-year-old man known to suffer from rheumatoid arthritis (RA) is admitted with a left pleural effusion. He has never smoked and apart from dyspnea, he is asymptomatic. Which of the following is true?? A : Rheumatoid arthritis is unlikely to be the cause of the effusion if pleural fluid glucose is above

1.6mmol/L. B : In pleural effusion secondary to RA, pleural pH is usually >7.35. C : Rheumatoid pleural effusions have high C4 complement levels. D : Up to 40% of patients with rheumatoid arthritis develop pleural effusions. E : The majority of patients with rheumatoid pleural effusions are women. Comment : Pleural involvement occurs in 5% of patients with rheumatoid arthritis. It usually affects men and it can be serous, turbid, yellow, milky or haemorrhagic. RA is unlikely to be the cause of a pleural effusion if the pleural fluid glucose is more than 1.6mmol/L. Eighty percent of rheumatoid pleural effusions have a pleural fluid pH of less than 7.30. In rheumatoid pleural effusions, C4 complement level is below 0.04 g/L. A : ·

Page 78: Masterclass Book Part 2

MOHAMMED IS-HAG 77

83. A 78-year-old man is referred to the chest clinic with history of dyspnoea for over 3 months. He is a retired teacher and has never smoked. He has a past medical history of atrial fibrillation, which is well controlled on digoxin. He also takes warfarin and the occasional paracetamol. He is clubbed and hypoxic on air with a SaO2 of 89%. He has bilateral crackles and a chest radiograph confirms fibrosing alveolitis. Which of the following combination of lung function is typical of fibrosing alveolitis? A : Reduced FEV1 & FVC, FEV1/FVC<70%, raised TLC & RV, reduced TLCO B : Reduced FEV1 & FVC, FEV1/FVC >70%, raised TLC & RV, reduced TLCO C : Normal FEV1 & FVC, FEV1/FVC >70%, raised TLC & RV, reduced TLCO D : Reduced FEV1 & FVC, FEV1/FVC>70%, reduced TLC & RV, reduced TLCO E : Reduced FEV1 & FVC, FEV1/FVC >70%, raised TLC & RV, normal TLCO. Comment : Restrictive lung disorders are characterised by reduced FEV1 & FVC, FEV1/FVC >70%, reduced TLC & RV and reduced TLCO. Obstructive disorders are characterised by reduced FEV1 & FVC, FEV1/FVC < 70%, raised TLC & RV (gas trapping) and reduced TLCO (emphysema) or normal or raised TLCO (non-smoking asthmatics). Mixed disorders may have reduced FEV1 & FVC, FEV1/FVC < 70% and raised TLC & RV, reduced, normal or raised TLCO depending on whether the obstructive disorder is due to emphysema or asthma. D : 84. Allergic broncho-pulmonary aspergillosis (ABPA): A : only occurs in the immunosuppressed B : requires treatment with amphotericin C : can cause permanent lung damage D : is usually diagnosed by sputum culture E : causes a raised IgA. Comment : ABPA usually occurs in association with asthma, but can occur in cystic fibrosis. It is treated with either oral or inhaled steroids, depending on severity. There is limited evidence supporting anti-fungal agents like itraconazole, but amphotericin is not used. Specific IgG and IgE are produced, and these can be used to diagnose the condition. In addition skin-testing or examination of sputum with fungal stains can be used. Sputum culture is not often helpful. Untreated, ABPA leads to proximal bronchiectasis particularly affecting the upper lobes. C : 85. Regarding aspergillomas, which of the following is not true? A : Aspergillomas should not be treated with amphotericin. B : They commonly present with haemoptysis. C : They should be treated with oral steroids. D : They usually develop in preexisting lung cavities E : They do not cause bronchiectasis. Comment : Aspergillomas are masses of fungal mycelia that grow in preexisting lung cavities. The commonest symptom is haemoptysis, which may range from trivial to massive. They do not require treatment with either anti-fungals or steroids and are not associated with bronchiectasis, unlike allergic bronchopulmonary aspergillosis. C : 86. A 24-year-old woman complains of excessive daytime sleepiness, and also mentions that she has fallen asleep at work. Her partner says that he has sometimes found her asleep during supper. A diagnosis of narcolepsy is made. Which of the following is true? A : Nocturnal apnoea spells are characteristic of this condition. B : Nasal Continuous Positive Airway Pressure (CPAP) is the treatment of choice. C : Patients with narcolepsy have raised hypocretin levels in the cerebrospinal fluid (CSF). D : Cataplexy is pathognomic. E : Narcoleptic patients have normal nocturnal sleep pattern. Comment : Narcolepsy is characterised by cataplexy, sleep paralysis, hypnagogic hallucinations and excessive daytime sleepiness. Over 24 hours, patients with narcolepsy do not sleep more than normal controls, but they are prone to fall asleep throughout the day, often at inappropriate times. Nocturnal apnoea spells are not a feature. Such patients lack hypocretin in the CSF. In patients with narcolepsy, night sleep is often interrupted by repeated awakenings and terrifying dreams. D : 87. Which of the following statements regarding extrinsic allergic alveolitis is correct? A : It is an IgE-mediated lung disease. B : The chronic form characteristically affects the upper lung zones. C : Toluene- iso- cyanate is a common cause. D : Farmers are a high-risk group only if they are exposed to hay in enclosed cellars. E : The treatment of choice is with bronchodilators and inhaled corticosteroids. Comment : Extrinsic allergic alveolitis is an incorrect name. The condition is not an allergic reaction but an inflammation of types III and IV hypersensitivity reaction. The commonest risk groups are farmers, pigeon breeders and mushroom pickers. Isocyanate is associated with occupational asthma and not with extrinsic allergic alveolitis. B :

Page 79: Masterclass Book Part 2

MOHAMMED IS-HAG 78

88. Which statement is true regarding small cell lung cancer: A : It has the best prognosis of all lung cancer. B : It is usually arises in the proximal airways. C : 30% can be treated surgically. D : It is associated with hypercalcaemia of paraneoplastic origin. E : It can be treated by radical radiotherapy if surgery is unsuitable. Comment : Small cell lung cancer has the worst prognosis and is rarely suitable for surgical resection. Even when caught early the 2-year survival is in the order of 20-25%. The mainstay of therapy is chemotherapy. Squamous cell cancer is radiosensitive and suitable for radical radiotherapy if surgery is not possible. Small cell cancer is associated with syndrome of inappropriate antidiuretic hormone (ADH) and squamous cell cancer is associated with paraneoplastic hypercalcaemia through increased parathyroid-related hormone. Seventy-five percent of small cell cancers arise in the proximal airways. B : 89. In cryptogenic fibrosing alveolitis (CFA): A : the chest radiograph shows upper zone shadowing B : most patients respond to oral steroids C : a lung biopsy is usually performed D : lung transplantation is not possible E : spirometry shows a restrictive pattern. Comment : In CFA there is predominantly basal disease. The spirometry is restrictive. Lung biopsy is rarely performed, only if the clinical or radiological features are atypical. Steroids have an effect in less than 50% of patients. Lung transplantation may be the only possible treatment for patients with progressive resistant disease. E : 90. A 60-year-old man with chest pain is anxious of the effect of asbestos exposure that he might have incurred in his previous jobs. He worked as a thermal-insulating engineer for 10 years from the age of 16. He has smoked 20 cigarettes a day since he was 16. Which one of the following statements is correct? A : The fact that he worked with asbestos at young age is a risk factor for mesothelioma. B : Amosite (brown asbestos) and crocidolite (blue asbestos) are less potent carcinogens than

chrysotile (white asbestos). C : His smoking increases the possibility of mesothelioma. D : His asbestos exposure is a risk factor for bladder cancer. E : Chemotherapy is the main stay of treatment of patient with mesothelioma. Comment : Asbestos exposure is known to be a risk factor for mesothelioma and lung cancer. The relationship with other cancers is not certain. The rate of mesothelioma is increasing. This is mainly due to the long latent period between exposure and the clinical development of the disease. Amosite and crocidolite are more potent carcinogens than chrysotile. Exposure at an early age is another risk factor. Cigarette smoking does not increase the risk of mesothelioma. The majority of patients with mesothelioma present with advanced disease. Chemotherapy and radiotherapy alone are not helpful. Tri-modality treatment with surgery, chemotherapy and radiotherapy may be helpful in selected cases. A : 91. A middle-aged meat factory worker is found to have Q fever pneumonia. Which of the following statements is correct? A : His occupation is important. B : He requires high dose penicillin. C : There are no long-term sequelae. D : The organism responsible is Chlamydia pneumoniae. E : Q fever is a notifiable disease. Comment : Q fever is due to Coxiella burnetii and is acquired through contact with animals. It is not notifiable, but can occur in outbreaks in farming communities and in abbatoirs. Treatment is with prolonged courses of tetracyclines. Rarely infection can be persistent leading to chronic symptoms including fatigue, malaise and sweats. Lengthy antibiotic courses can be curative. A : 92. Continuous positive airway pressure (CPAP) is useful in the management of respiratory failure secondary to: A : infective exacerbation of chronic obstructive pulmonary disease (COPD) B : acute pulmonary oedema C : tension pneumothorax D : large right pleural effusion E : pulmonary embolism. Comment : CPAP has been tried in a variety of conditions. Patients with pulmonary oedema have been shown to improve clinically in terms of gas exchange, pulse and respiratory rate in the trials which have been performed, although none have as yet shown improved mortality. This is a useful technique for emergency departments to utilize. B :

Page 80: Masterclass Book Part 2

MOHAMMED IS-HAG 79

93. A 48-year-old man is found to have a blood pressure of 176/112 when he attends his general practitioner for a ‘new patient check-up’. He takes occasional anxiolytics for anxiety, but his past medical history is otherwise unremarkable. Physical examination is normal, excepting for obesity (BMI 32). A ‘routine’ biochemical screen is normal, excepting for potassium 3.3 mmol/l. The two most likely causes of his hypertension are: A : Renal hypertension B : Hypothyroidism C : Renovascular hypertension D : Cushing’s syndrome E : Primary hyperaldosteronism (Conn’s syndrome) F : Acromegaly G : Essential hypertension H : Isolated clinic (‘white coat’) hypertension I : Phaeochromocytoma J : Coarctation of the aorta. Comment : All of the conditions listed, excepting hypothyroidism, might explain hypertension, but all other than essential hypertension and ‘white coat’ hypertension are rare (together accounting for less than 5% of cases).Although a secondary cause of hypertension is very unlikely it would be important to look for clues in history and examination that might suggest renovascular disease (ischaemic heart disease, transient ischmaemic attack (TIA) / stroke, peripheral vascular disease), renal disease (previous nephritis, results of urine testing for e.g. insurance / employment medicals).Episodes of palpitations, sweating or headache may suggest phaeochromocytoma, but a less exotic cause such as anxiety would be a much more likely explanation. The serum potassium concentration is just below the lower limit of normal, but primary aldosteronism (Conn’s syndrome) remains exceedingly unlikely.In the case of an obese man it is also important to note that the blood pressure reading may be falsely elevated as a result of inadequate blood pressure cuff size, and it would be important to ensure that readings were taken with appropriate equipment. G:H: 94. A 37-year-old man with asthma comes to your clinic. His current medication consists of a low dose of inhaled corticosteroids and inhaled short-acting beta 2 agonist which he takes, on average, three to four times a day. What would you do? A : Advise him to continue with his current medication. B : Commence a high dose of inhaled corticosteroids. C : Add an inhaled long-acting beta 2 agonist. D : Add a leucotriene receptor antagonist. E : Add a long-acting anticholinergic. Comment : This patient is in Step Two of asthma treatment. His asthma is not well-controlled, as he needs to take his rescue medication more than twice a day. Therefore, his treatment should be stepped up. In Step Three of asthma treatment, the current British Thoracic Society guideline advises first adding inhaled long-acting beta 2 agonist (LABA) and then assessing the situation.If there is a good response to LABA, this medication should be continued. If there is benefit from LABA but control is still inadequate, LABA should be continued, and the inhaled corticosteroids should be increased to a high dose. If there is no response to LABA, then that treatment should be stopped and inhaled corticosteroids should be increased to a high dose. Anticholinergics have very little role in the management of stable asthma. C : 95. In small cell lung cancer: A : hypercalcemia is commonly seen B : Cushing's syndrome is characterized by buffalo hump, striae and central obesity C : radiotherapy is the treatment of choice D : hypertrophic pulmonary osteoarthropathy is rarely seen E : syndrome of inappropriate antidiuretic hormone secretion (SIADH) is rarely seen. Comment : Small cell carcinoma is rarely associated with hypercalcemia and SIADH. Cushing's syndrome in small cell carcinoma does not manifest classically by buffalo hump, striae or central obesity due to the short natural history. Its presence is suspected by arterial hypertension, hyperglycemia, hypokalemia, alkalosis and muscle weakness. Chemotherapy is the treatement of choice. Hypertrophic pulmonary osteoarthropathy is very rare in small cell carcinoma of the lung. D : 96. Which of the following is true regarding hypersensitivity pneumonitis? A : Typically the patient complains of chest discomfort 24 hours after exposure to the causative

antigen. B : The major abnormal findings are diffuse rales. C : Pleural effusion is commonly seen. D : It responds dramatically to azathioprine. E : It never leads to cor pulmonale.

Page 81: Masterclass Book Part 2

MOHAMMED IS-HAG 80

Comment :Hypersensitivity pneumonitis (HP), also called extrinsic allergic alveolitis, is a group of immunologically-induced diseases associated with repeated exposure to finely dispersed organic dust. The patient presents 4-6 hours after exposure to the antigen with chills, fever, dyspnoea and general malaise. This lasts for 18-24 hours after exposure ceases. Diffuse rales are the major physical finding. Pleural effusion is not typically seen. Acute and subacute forms are characterized by interstitial and alveolar nodular infiltrates usually bilaterally. Mild episodes clear spontaneously. Severe attacks require treatment with prednisolone, oxygen and rest. Bronchodilators may help if there is severe bronchospasm. In its chronic form it results in irreversible damage to the lungs leading to cor pulmonale. B : 97.

A 72-year-old gentleman with a 10-year history of progressive breathlessness presents. He has had a marked deterioration over the past six months and is now breathless after walking 50 yards on the flat. He is a smoker with an equivalent exposure to 40 pack years. See image for chest radiograph. The most likely diagnosis is: A : Pulmonary embolism B : Right hilar mass with malignancy C : Emphysema D : Cardiac failure E : Asthma. Comment : The chest radiograph demonstrates hyperinflation with low flat diaphragms and fewer lung markings at the bases. Lung function would help confirm the diagnosis. C : 98.

A 55-year-old woman is referred with a 10-day history of productive cough, fever and increasing shortness of breath. Her chest radiograph shows: A : widened mediastinum B : right upper zone consolidation C : right upper lobe collapse D : superior mediastinal mass E : anomalous superior vena cava.

Page 82: Masterclass Book Part 2

MOHAMMED IS-HAG 81

Comment : Note also the tenting of the right hemidiaphragm and the raised right hilum. Depending on the clinical context the commonest causes to consider would be infection, carcinoma of the lung and foreign body. Other tumours, e.g. carcinoid, benign adenoma, and pulmonary infarct would be much less likely. C : 99.

A 52-year-old man has this chest radiograph taken as part of a health screening program. The two most likely diagnoses are: A : Sarcoidosis B : Lung cancer C : Rheumatoid nodule D : Tuberculosis E : Bronchial adenoma F : Pneumonia G : Lung abscess H : Arteriovenous malformation I : Wegener’s granulomatosis J : Secondary metastasis. Comment : The most important differential is clearly lung cancer, with a secondary metastasis next on the list.Appropriate investigation would involve: 1. CT scan of chest, liver and adrenals 2. Bronchoscopy 3. Sputum for cytology and microbiology (including Acid-fast bacillus (AFB)) 4. Blood tests for routine haematology, coagulation screen, electrolytes (including calcium) and liver function tests.B: J: 100.

A 72-year-old man presents with sudden onset epigastric pain (10/10), not relieved by vomiting, or by a GTN spray. His past medical history includes ischaemic heart disease and ulcerative colitis. On examination his abdomen is soft with slight epigastric tenderness, but there is no rebound and bowel sounds are present. ECG shows sinus tachycardia and right bundle branch block (RBBB) . Chest radiograph shows widening of the mediastinum and bilateral atelectasis. Oxygen saturation is 86% on air: ABG reveal pH of 7.42, pCO2 of 6.5 kPa, p02 of 6.8kPa. What is the diagnosis? A : Dissection of the aorta B : Ruptured oesophagus C : Bilateral pulmonary emboli D : Mediastinal emphysema E : Megacolon.

Page 83: Masterclass Book Part 2

MOHAMMED IS-HAG 82

Comment :Severe hypoxia accompanied by tachycardia, RBBB and bilateral atelectasis strongly suggest pulmonary embolism, so CT pulmonary angiography was carried out, confirming moderate-sized thrombi in both main pulmonary arteries. C : 101.

A 62-year-old man presents as a medical emergency with a few week history of dry cough and progressive shortness of breath. He lives with his male partner and has been in a stable relationship for the past 5 years. On examination he is pyrexial and has bilateral basal crackles. His ABG showed pH of 7.35, pCO2 of 4.5kPa, pO2 of 6.2kPa. His chest radiograph is shown (see image). Which three of the following investigations would be most useful at the time of his presentation? A : Bronchoscopy B : Induced sputum C : Transbronchial biopsy D : Blood cultures E : Sputum culture F : Heaf test G : Allergic bronchopulmonary aspergillosis screen H : C reactive protein I : Autoantibody screen J : antineutrophil cytoplasmic antibodies (ANCA) test K : erythrocyte sedimentation rate (ESR) L : Eosinophil count M : Anti-GBM antibody N : Cytomegalovirus (CMV) titres O : HIV test. Widespread alveolar infiltration seen on a chest radiograph and severe hypoxia suggest Pneumocystis carini pneumonia (PCP) in this context.An HIV test should be arranged (after consent has been obtained) and empirical anti-PCP therapy should be started.In view of severe hypoxia neither bronchoscopy nor induced sputum are clinically appropriate investigations at the time of presentation, but induced sputum should be obtained for examination for PCP as soon as his oxygenation improves. D : N:O: 102. Alpha 1 antitrypsin: A : is produced by Type II pneumocytes B : stimulates neutrophil elastase C : is a glycoprotein D : deficiency is autosomal recessive E : can cause hepatitis. Comment : Alpha 1 antitrypsin is a glycoprotein synthesized in the liver and comprises 90% of the serum alpha 1 globulin seen on electrophoresis. It is composed of 394 amino acids and it is coded for by a single gene located on chromosome 14. Its deficiency is autosomal dominant and can cause emphysema and cirrhosis. It is an anti protease and inhibits neutrophil elastase. C :

Page 84: Masterclass Book Part 2

MOHAMMED IS-HAG 83

103.

A 60-year-old lady has been diagnosed as suffering from asthma by her doctor for over 12 months. Her initial symptoms were of non-productive cough, worse at night, intermittent wheeze and dyspnoea on exertion. She is on high dose inhaled steroids, a long acting beta agonist and a short acting beta agonist. She still complains of nocturnal symptoms and has been referred to the chest clinic. Her chest radiograph is shown (see image). She has never smoked and has never kept any pets. What is the next appropriate step? A : Add an anticholinergic B : Add oral steroids with a biphosphonate C : Add a proton pump inhibitor D : Add a leukotriene antagonist E : Add theophylline. Comment : The chest radiograph shows a hiatus hernia. The underlying cause of her nocturnal symptoms may be gastrooesophageal reflux which can cause cough as well as intermittent wheeze. She should be started on a proton pump inhibitor and at the same time she should maintain a peak expiratory flow rate (PEFR) record. If her PEFR are acceptable, she should be gradually weaned off some of her extensive asthma treatment. C : 104.

A 77-year-old man is admitted as a medical emergency with a history of progressive shortness of breath. His chest radiograph is shown (see image). Apart from scoliosis, what other radiological abnormality is seen? A : Left upper lobe consolidation and collapse B : Localised left upper pneumothorax C : Left upper lobectomy D : Left thoracoplasty E : Impossible to comment as the film is rotated. Comment : Thoracoplasty is a surgical procedure in which, to decrease thoracic volume, resected ribs are either removed or are replaced with their convexity inward. The only current indication for thoracoplasty is to close a persistent pleural space, but before 1952 thoracoplasty was frequently used as a form of collapse therapy for cavitary pulmonary tuberculosis. Scoliosis can develop after thoracoplasty in pre-pubertal patients (its severity is related to the number of ribs removed), eventually leading to nocturnal hypoventilation, respiratory failure and a requirement for oxygen therapy / non-invasive ventilation at night. D :

Page 85: Masterclass Book Part 2

MOHAMMED IS-HAG 84

105.

A 65-year-old male presents to A&E with acute onset shortness of breath. Arterial blood gases show PO2 7.3, PCO2 3.9 and pH 7.45 breathing air. Blood pressure is 130/70 and pulse is 120, regular. JVP is elevated 3 cm and there is no ankle oedema. Chest is clear. No past history of thromboembolic disease, but has a positive family history. He has no other known risk factors for venous thromboembolic disease. His CT pulmonary angiogram is shown (see image). Which of the following statements is true? A : Providing there are no contraindications, he should be thrombolysed in view of the size of the

clot. B : He should be anticoagulated for life in view of his family history (provided no contraindications). C : He should receive prophylactic heparin for long-haul flights in future. D : He should be screened for common cancers. E : It is better to use unfractionated heparin acutely if he has lupus anticoagulant to achieve higher

levels of anticoagulation. Comment : He has suffered a non-massive pulmonary embolus (PE). Massive pulmonary embolus is defined as pulmonary embolus associated with hypotension (systolic BP <90mmHg) or other adverse features such as cardiac arrest. Thrombolysis is only currently indicated for massive PE. Life-long prophylaxis after first idiopathic PE should only be considered with the results of thrombolphilia testing and must be balanced against the risk of bleeding. Life-long prophylaxis remains debatable after first PE, except in the context of lupus anticoagulant. He should receive low molecular weight heparin and venous compression stockings for future long-haul flights. There is an increased risk of cancer being detected within 6–12 months of a first episode of venous thromboembolism (VTE), particularly in those with no other risk factors and/or recurrent episodes. Previously unrecognised cancer, present in 7–12% of those with idiopathic VTE, can usually be detected by a combination of careful clinical assessment, routine blood tests, and chest radiography and, if these are satisfactory, the current consensus is that it is not appropriate to proceed to tests such as ultrasound, CT scanning or endoscopy. He should not be anticoagulated with unfractionated heparin if he has lupus anticoagulant since this abnormality interferes with activated partial thromboplastin time (APTT) monitoring. C : 106. The wife of this 63-year-old man complained that he was sleepy and increasingly aggressive. Which of the following is LEAST likely to be helpful in his management? A : Non-invasive ventilation B : Salbutamol nebules C : Loop diuretics D : Long-term oxygen therapy (LTOT) E : 'Flu vaccination. Comment : This man with severe kyphoscoliosis has probably developed chronic respiratory failure. All patients with significant respiratory disease should be offered annual 'flu vaccination. He should be assessed for non-invasive ventilation and long-term oxygen therapy and is likely to have cor pulmonale with pitting oedema, which will be improved symptomatically with careful use of diuretics. He might have reversible airways obstruction (as this is a common condition,) but this would be an additional problem. B :

Page 86: Masterclass Book Part 2

MOHAMMED IS-HAG 85

107.

A 74-year-old patient previously treated for pulmonary tuberculosis presents with cough and a 250 ml haemoptysis. The chest radiograph is shown (see image). Which of the following diagnoses is correct? A : Active tuberculosis B : Carcinoma of the bronchus C : Pulmonary infarction D : Aspergillus infection E : Bacterial pneumonia. Comment : The patient has scaring and cavitation from old tuberculosis. The lesion in the right apex is a fungal ball within and existing pulmonary cavity – an aspergilloma. The diagnosis can be confirmed by CT scanning and growth of aspergillus from respiratory specimens. Massive haemoptysis in a person with known cavitary lung disease is highly suggestive on the development of an aspergilloma. Aspergillomata respond poorly to antifungal therapy. Definitive therapy is surgical and bronchial artery embolization may be used to stop haemorrhage. D : 108.

A 60-year-old Somalian man presents with a 2-month history of weight loss, night sweats and back pain. A chest radiograph shows bilateral fine nodular shadowing but no lymphadenopathy. A high resolution CT is performed (see image). What is the likeliest diagnosis? A : Testicular carcinoma with pulmonary metastases B : Sarcoidosis C : Disseminated adenocarcinoma D : Miliary tuberculosis E : Lymphoma. Comment : This patient has classical miliary tuberculosis, given his chest radiograph and CT findings of miliary nodular shadowing with possible evidence of tuberculous epididymitis. Sarcoidosis is a possibility but is less likely given the lack of lymphadenopathy. It is unlikely to be testicular carcinoma given his age and that this causes larger pulmonary nodules. The other conditions do not normally present with testicular pain. Pulmonary lymphoma causes hilar lymphadenopathy. D :

Page 87: Masterclass Book Part 2

MOHAMMED IS-HAG 86

109.

A 55-year-old woman with long-standing rheumatoid arthritis attends the chest clinic with a history of non-productive cough and dyspnoea of 8 weeks duration. There is no wheeze, weight loss or haemoptysis. She has never smoked and has not been exposed to any organic dust. On examination she had hand deformities typical of rheumatoid, with nodules at the elbows. Her lung fields were clear on auscultation. A chest radiograph is shown (see image). What are the two possible diagnoses? A : Asthma B : Emphysema C : Lung fibrosis secondary to rheumatoid arthritis D : Bronchiolitis obliterans E : Bronchiectasis F : Aspergillosis G : Atypical pneumonia H : Pulmonary eosinophilia I : Occult lung cancer J : Bilateral hilar lymphadenopathy due to sarcoidosis Comment : The chest radiograph is normal. Asthma is possible, but emphysema is unlikely in a non-smoker, and the history of respiratory symptoms is much too short to entertain this diagnosis. Bronchiolitis obliterans, first described in 1835, can be due to toxic fume inhalation, viral infections, connective tissue disorders, bone-marrow and lung or heart-lung transplantation, or drug toxicity (penicillamine). It is characterised by cough and dyspnoea without wheeze. Diagnosis can be confirmed by lung biopsy. A : D : 110. A 48-year-old woman presents with a pleural effusion 3 weeks after receiving antibiotics from her general practitioner for a chest infection. You suspect a parapneumonic effusion. Which one of the following statements regarding parapneumonic effusions is true? A : is any effusion associated with bacterial pneumonia, lung abscess or bronchiectasis. B : always need to be drained. C : is commonly due to mycobacteria. D : should be treated surgically. E : should never be aspirated. Comment : Any pleural effusion associated with bacterial pneumonia, lung abscess or bronchiectasis is a parapneumonic effusion. Up to 40% of patients with pneumonia have an accompanying pleural effusion and the majority resolve without any specific therapy directed towards the pleural fluid. The evolution of these effusions can be divided into 3 stages: · exudative · fibropurulent · organistaion. During these stages the pH and glucose level of the pleural fluid becomes progressively lower. These effusions can be monitored by the pH: · pH<7 - absolute indication to insert an intercostal drain · pH>7.20 - usually do not need drainage · pH 7-7.20 - should be monitored and intercostal drain inserted if the pH swings towards 7. Prior to the antibiotic era Streptococcus pneumoniae or haemolytic streptococci were the commonest organisms. Anaerobic organisms are commonly seen nowadays. A :

Page 88: Masterclass Book Part 2

MOHAMMED IS-HAG 87

111.

A 25-year-old gentleman with a history of asthma attends the casualty department with a history of acute dyspnoea. He is hypoxic with a PaO2 of 7.8 Kpa. The chest radiograph is shown (see image). What is the appropriate management? A : Anti-coagulation and urgent spiral CT thorax B : Admission to ITU and ventilatory support C : Continuous positive airway pressure (CPAP) D : High flow oxygen, nebulised bronchodilators and oral steroids E : None of the above. Comment : The chest radiograph shows a right pneumothorax. He needs a needle aspiration and may need an intercostal chest drain if needle aspiration is not successful. E : 112.

A 50-year-old Indian male accountant gives a history of 3 months cough, weight loss and haemoptysis. He was found to have some nasal crusting but otherwise examination was normal. His CRP and ESR are elevated but he has normal biochemistry except for an elevated creatinine of 400. His chest radiograph showed a few nodules. A CT thorax is shown (see image). What is the most likely diagnosis? A : Tuberculosis B : Bronchial carcinoma C : Staphylococcal pneumonia D : Wegener’s granulomatosis E : Systemic lupus erythematosus (SLE)

Page 89: Masterclass Book Part 2

MOHAMMED IS-HAG 88

Comment : The two most appropriate diagnoses were Wegener’s and tuberculosis. However, given the renal dysfunction and nasal disease, a vasculitis is most likely. The CT shows multiple and occasionally cavitating nodules. Although staphylococcal infection cavitates, the history is too long for this. Appropriate investigations would involve bronchoscopy and serum ANCA. D : 113.

The chest CT is shown from a woman of 50 who underwent lumpectomy, followed by radiotherapy to her right breast and axilla for breast carcinoma. She presents with cough and breathlessness. What is the likely diagnosis A : Allergic bronchopulmonary aspergillosis (ABPA) B : Eosinophilic pneumonia C : Recurrent breast cancer D : Radiation pneumonitis E : Bacterial pneumonia. Comment : The CT shows sharply demarcated consolidation in the right upper lobe in a region within the likely field of radiotherapy that would encompass the breast and axilla. The sharp border to the area of abnormality is characteristic of radiation pneumonitis. Radiation can produce organising pneumonia or interstitial fibrosis within the lung. ABPA is characterized by proximal bronchiectasis. Eosinophilic pneumonia produces patchy peripheral pulmonary infiltrates and is characteristically very responsive to steroid treatment. Recurrent breast metastases are unlikely in view of the air bronchograms within the abnormal area. Segmental pneumonia is a possible diagnosis but less likely than radiation pneumonitis. D : 114. A 70-year-old smoker presents with a long history of breathlessness with exertion and has the following lung function: · FEV1 1.1L (60%) · FVC 1.3L (55%) · FEV1/FVC ratio = 84% · TLC = 65% predicted · RV = 58% predicted · TLCO = 55% predicted · KCO = 60% predicted What is the most likely diagnosis? A : Chronic Obstructive Pulmonary Disease B : Fibrosing alveolitis C : Severe kyphoscoliosis D : Asthma E : Pleural plaques. Comment : The lung function tests show a significant restrictive defect. Only kyphoscoliosis or a fibrosing disease may fit this picture, but given the low KCO (i.e. after correcting for alveolar volumes), the most likely answer is fibrosing alveolitis, as the gas exchange after correcting for the alveolar volume would be higher in scoliosis. Plaques do not tend to cause any restriction. B :

Page 90: Masterclass Book Part 2

MOHAMMED IS-HAG 89

115.

PLATE 16

A 32-year-old male presents with fever, dry cough and breathlessness. Resting oxygen saturation on air is 97% and his chest radiograph was reported as normal. This appearance of his tongue was noted (see image). What is the most likely diagnosis? A : Pneumocytsis carinii pneumonia B : Q fever C : Legionnaire’s disease D : Infective endocarditis E : Pulmonary candidiasis. Comment : There is a white patch on the side of the tongue indicating oral hairy leukoplakia (OHL). OHL is almost pathognomonic of underlying immunodeficiency due to HIV infection. OHL is generally seen when the CD4 count is less than 350 and is due to local Ebstein Barr virus infection of the skin. Pneumocytsis carinii pneumonia (PCP) is one of the commonest causes of breathlessness in patients with a CD4 count of less than 200. The presentation described is typical of early PCP when the patient may still have a normal chest radiograph and only becomes hypoxic on exercise. A : 116. A 55-year-old lorry driver has been referred because of daytime somnolence. His wife reported that he snores and is very concerned that he stops breathing for a few seconds while he sleeps. He has no significant past medical history. Apart from being obese (BMI 31) his examination is normal. Obstructive sleep apnoea is suspected. Which of the following is most likely to be true? A : An Epworth Sleepiness Score will be diagnostic. B : A normal overnight pulse oximetry excludes the diagnosis. C : Polysomnography will be diagnostic. D : Multiple sleep latency test is indicated. E : Fibreoptic nasoendoscopy will be diagnostic. Comment : Obstructive sleep apnoea (OSA) is a common sleep disorder. The risk of developing OSA increases with age and obesity. It is more common in males and is characterised by snoring, apnoea spells, choking and gurgling sounds during sleep and excessive daytime somnolence. The Epworth Sleepiness Score evaluates general levels of sleepiness and is not diagnostic. A normal overnight pulse oximetry does not exclude OSA. The gold standard for diagnosis is polysomnography. Multiple sleep latency test measures sleepiness objectively. Fibreoptic nasoendoscopy is performed in selected patients to locate the site of obstruction. C :

Page 91: Masterclass Book Part 2

MOHAMMED IS-HAG 90

117.

PLATE 17 A 34-year-old intravenous drug user presents with a persistent fever and shortness of breath. His radiograph shows bilateral discrete lesions. Blood cultures taken on admission flag positive after 24 hours. The Gram stain is shown (see image). What is the organism and what is the most likely underlying diagnosis? A : Staphylococcus aureus B : Streptococcus bovis C : Candida albicans D : Pneumocystis carinii E : Mycobacterium tuberculosis F : HIV infection G : Left-sided endocarditis H : Right-sided endocarditis I : Miliary tuberculosis J : Pneumonia. Comment : The clinical picture can be explained by a diagnosis of right-sided endocarditis with secondary septic pulmonary emboli. The Gram stain shows a candidal species. Candidal endocarditis is seen in the intravenous drug using population where organism is inadvertently introduced as a contaminant of the intravenous cocktail. C : H: 118. A businessman of 60 presents with several months of increasing dyspnoea. He is a non-smoker and has always felt fit. His past medical history includes only a broken leg playing rugby in 1980, complicated by a deep vein thrombosis (DVT). His chest radiograph is shown. What is the likely diagnosis? A : Left ventricular failure secondary to silent myocardial infarction B : Primary pulmonary hypertension C : Atrial septal defect D : Chronic thromboembolic pulmonary hypertension E : Systemic sclerosis with interstitial lung disease. Comment : The chest radiograph shows changes of severe pulmonary hypertension, with marked enlargement of the proximal pulmonary arteries and attenuation of the smaller pulmonary vessels. There is no interstitial pulmonary oedema to suggest left ventricular failure. With a prior history of DVT, the most likely diagnosis is chronic thromboembolic pulmonary hypertension (CTEPH). Primary pulmonary hypertension can occur at any age, but the mean age of diagnosis is 36 years and there is a female preponderance of 1.7:1. Chronic thromboembolic pulmonary hypertension has an unknown incidence, but probably occurs in no more than 0.1% of survivors of pulmonary embolus. The pathophysiology is not understood. Antiphospholipid antibodies are found in 10-20% of patients referred for surgical treatment. Progression to pulmonary hypertension and death occurs not because of repeated emboli, as was originally thought – serial perfusion scans on patients in follow-up do not demonstrate new perfusion defects. Rather, it appears that there develop progressive pulmonary hypertensive changes in the small vessels of the pulmonary vascular bed. The clinical course of CTEPH is usually one of an asymptomatic (or “honeymoon”) period, during which right ventricular hypertrophy develops, followed by progressively increasing dyspnoea and exercise intolerance, during which the right ventricle dilates and fails.The only effective treatment is surgical.Pulmonary thromboendarterectomy is a difficult and specialised operation to remove the organized thrombus, which is not endoluminal but incorporated into the walls of the pulmonary arterial tree. The surgeon has to dissect down to the level of the original arterial intima, performing a true endarterectomy, rather than a thrombectomy. In San Diego, where the number of cases performed now exceeds 1200, the operative mortality is about 7% and the results following surgery excellent. The only UK centre offering this procedure is Papworth Hospital. D :

Page 92: Masterclass Book Part 2

MOHAMMED IS-HAG 91

119. Traumatic rupture of the diaphragm - A : mainly affects the right hemi diaphragm. B : can be diagnosed definitely by chest radiograph. C : is usually obvious at the time of injury. D : may present with nausea and vomiting. E : mainly affects young females. Comment : Traumatic rupture of the diaphragm usually results from blunt trauma caused by motor vehicle accidents and penetrating trauma from gunshot and stab wounds. It predominantly affects males in the third decade. The left side is affected in 68% of the patients and the right side in 24% of the patients. Bilateral rupture occurs in 2% of patients. Diagnosis based on chest roentgenogram may be made in 41% of the patients. The diagnosis is not obvious at the time of injury and may be missed in up to 66% of the patients. Nausea and vomiting may occur in the late stages. D : 120. A 62-year-old man presented with increasing shortness of breath. He was a current smoker with over 20 packs per year exposure. He was found to have a right-sided pleural disease. A final diagnosis of mesothelioma was made. Which of the following statements is correct? A : Radiotherapy is the primary treatment. B : Patients should be advised about state compensation and entitlement to a disability pension. C : Chemotherapy is the most appropriate treatment. D : Pleural aspiration is usually adequate for a diagnosis. E : Sarcomatous mesothelioma has a good prognosis. Comment : The correct answer is B. As asbestos is associated with the development of mesothelioma, all patients should be advised that they may be eligible for sate compensation and disability benefits. These claims must be made within 3 years of initial diagnosis. B : 121. A 24-year-old lady gives a history of cough and dyspnoea for many years. She also gets frequent chest infections treated with prolonged courses of antibiotics and coughs up an eggcupful of phlegm daily. Three weeks ago she had an episode of haemoptysis. She has never smoked and works in an office. Which of the following investigations will establish the diagnosis? A : V/Q scan B : Bronchoscopy C : Spirometry D : High resolution CT chest scan E : Pek expiratory flow recording Comment : The history is suggestive of bronchiectasis and high resolution CT of the chest will establish the correct diagnosis. D : 122. A 67-year-old man has been diagnosed with chronic obstructive pulmonary disease (COPD). Spirometry confirms severe COPD with a FEV1 of less than 30% predicted. In the last 12 months he has been admitted to hospital on four occasions with COPD exacerbation. Which one of the following statements is true? A : Exacerbations can be reduced by daily oral steroids. B : Nebulisers help to reduce exacerbations. C : Long-acting beta 2 agonists reduce exacerbations. D : Oral theophyllines can reduce rate of exacerbations. E : Long-acting anticholinergics can reduce exacerbation rate. Comment : Severe COPD is diagnosed if the FEV1 is less or equal to 30% predicted. Studies have shown that patients treated with long acting anticholinergic (tiotropium) had fewer exacerbations per year. Use of rescue medication was often less in patients on tiotropium. E : 123. Regarding byssinosis, which of the following statements is correct? A : Chronic long-term exposure to cotton dusts results in lung fibrosis. B : Patients usually present with haemoptysis. C : The commonest presentation is chest tightness and shortness of breath on the first day of the

work week. D : Patients should be treated with immunosuppressants. E : It is best diagnosed by lung biopsy. Comment : Byssinosis is due to inhalation of dusts produced by the handling of flax and hemp. The characteristic presentation is chest tightness and dyspnoea on the first day of the work week. Haemoptysis is not a feature. Long-term exposure may lead to chronic productive cough with faster annual decline in the FEV1. Non-smokers with byssinosis have enlarged submucosal glands and goblet cell hyperplasia resembling chronic bronchitis. Immunosuppressants have no role the treatment. C :

Page 93: Masterclass Book Part 2

MOHAMMED IS-HAG 92

124.

PLATE 18 A 43-year-old Portugese man presents with a 2 week history of feeling short of breath, fever and a rash. He is found to be hypoxic with a PO2 of 6 on air but with a few sparse crackles. He has a rash over his back and an abnormal chest radiograph (see images). What is the most likely pathogen? A : Mycobacterium tuberculosis B : Pneumocystis carinii C : cytomegalovirus D : Streptococcus E : Cryptococcus Comment : The skin lesions are characteristic of Kaposi’s sarcoma and the chest radiograph (CXR) suggestive of a perihilar infiltrate/alveolar shadowing. This patient is likely to have a human immunodeficiency virus related disease.The likeliest cause for this patient's LRTI is pneumocystis given the clinical history, examination and CXR. B : 125. An obese 62-year-old man presents with breathlessness. His chest radiograph shows: A : bilateral pleural effusions B : cardiomegally with bilateral pleural effusions C : cardiomegally D : cardiomegally with right pleural effusion E : right pleural effusion. Comment : The transverse cardiac diameter is more than 50% of the width of the rib cage, indicating the presence of cardiomegally. The right costo-phrenic angle is obscured by fluid, but the left costo-phrenic angle is clear. D : 126. A 38-year-old woman who was previously fit and well presents with breathlessness that has been getting worse over 3 or 4 months. You suspect that she might have primary pulmonary hypertension. Which of the following physical signs would NOT be consistent with this diagnosis? A : Elevated jugular venous pressure B : Clubbing C : Palpable heave at left sternal edge D : Added heart sound over right ventricle E : Loud P2. Primary pulmonary hypertension can present with breathlessness, fatigue, angina (due to right ventricular ischaemia) or presyncope / syncope.Physical signs include elevated JVP (sometimes with a giant v wave of tricuspid regurgitation and pulsatile hepatomegaly), left parasternal heave, pansystolic murmur (tricuspid regurgitation) and right ventricular S4, peripheral oedema. Clubbing is not a feature and suggests chronic lung disease or cyanotic congenital heart disease in this context.An important differential diagnosis of primary pulmonary hypertension is chronic pulmonary thromboembolism. B :

Page 94: Masterclass Book Part 2

MOHAMMED IS-HAG 93

127. A 64-year-old lady is reviewed in outpatients with the histopathology results from a biopsy taken during bronchoscopy. The report shows cells consistent with a small cell carcinoma. She has been referred to the local multi-disciplinary team but her case as yet has not been discussed. Both the patient and her son want to know what future treatments she might be offered. Which of the following statements are correct? A : Small cell carcinoma does not metastasise early. B : Small cell carcinoma has a slow doubling time. C : Limited small cell carcinoma is defined as tumour confined to the mediastinum. D : Bone marrow evaluation is essential for staging. E : Patients with small cell should undergo a chest CT, bone scanning and a brain CT. Comment : Small cell lung cancer is characterised by its propensity for early metastases and rapid doubling time. Limited disease is defined as tumour that can be encompassed within a single tolerable radiation port. Staging should include a chest CT, a brain CT and bone scanning. Bone marrow evaluation adds little information and is not required. E : 128. A 56-year-old man with emphysema presented with increasing dyspnoea and left-sided pleuritic chest pain. A chest radiograph revealed a left-sided pneumothorax and that the lung edge measured 4 cm away from the chest wall. The most appropriate management of his condition is: A : Refer to a respiratory outpatient clinic B : Admit and monitor with daily chest radiographs C : Intercostal tube drainage D : Simple aspiration E : Review with daily chest radiographs Comment : The correct answer is C. The lung edge measurement suggests a greater than 50% pneumothorax.In the context of chronic underlying respiratory disease such as chronic obstructive pulmonary disorder, patients are best managed by admission and pleural drainage C :. 129. A 43-year-old woman with long-standing asthma is admitted with an exacerbation. She is cyanosed and unable to speak more than three words at a time. She is using her accessory muscles, chest expansion is reduced but the same on both sides, and a wheeze can be heard bilaterally. Which one of the following is the best initial treatment? A : Maximum inspired oxygen by face mask; nebulised salbutamol (5 mg) driven by oxygen B : Nebulised salbutamol (5 mg) driven by air C : Nebulised salbutamol (50 mg) driven by oxygen D : Oxygen 35% by face mask; nebulised salbutamol (10 mg) driven by 35% oxygen E : Maximum inspired oxygen by face mask; decompress both sides of the chest by inserting venflons into the second intercostal spaces in the mid-clavicular line bilaterally. Comment : Resuscitation is the first priority. Maximum inspired oxygen should be given by facemask: this is best achieved using a reservoir bag at a flow rate of 15 l/min, which can generate an FiO2 of about 85%. Nebulised salbutamol (5-10 mg) driven by oxygen should be given, and many would add ipatropium bromide (Atrovent, 500 microg) to the nebuliser chamber at the same time as the salbutamol. If the woman does not improve, then call for assistance from ICU sooner rather than later. Although it is always important to consider pneumothorax in any breathless patient, there is no evidence at all to suggest that this woman has bilateral pneumothoraces and she would not be well served by chest decompression. A : 130. A 24-year-old homosexual man presents with a 2-day history of progressive breathlessness, dry cough and slight fever. On examination he was tachypnoeic, but there were no abnormalities on auscultation. See image for chest radiograph. The most likely diagnosis is: A : Miliary tuberculosis B : Pneumocystis carnii pneumonia C : Congestive cardiac failure D : Atypical pneumonia E : Intersitial pneumonitis. Comment : There is bilateral perihilar interstitial shadowing and paucity of clinical signs which is classical for pneumocystis carnii pneumonia. Atypical pneumonia does not cause this degree of breathlessness without clinical signs. B : 131. A 72-year-old woman with known COPD on maximal medication is referred with a 4-month history of morning headaches. ABG showed pH of 7.34 pCO2 of 5.8 kPa, pO2 of 7.4kPa and HCO3 27 mmHg. What would you recommend? A : Long-term oxygen therapy. B : Overnight SaO2 and CO2 monitoring. C : Repeated ABG in 3-weeks time. D : CT brain scan. E : Ambulatory oxygen therapy.

Page 95: Masterclass Book Part 2

MOHAMMED IS-HAG 94

Comment : This patient is likely to have nocturnal hypoxia with overnight or early morning hypercapnia and may require nocturnal oxygen supplementation. Overnight SaO2 with a pulse oximeter and transcutaneous CO2 measurement should be carried out. B : 132. A 24-year-old man with a history of depressive illness is brought to the accident and emergency department after being found unconscious in a garage. Carbon monoxide poisoning is considered. Which one of the following statements regarding carbon monoxide and carbon monoxide poisoning is true? A : It is produced by the incomplete combustion of carbon containing fuel. B : Alteration in consciousness is the most frequently seen symptom in carbon monoxide poisoning. C : The presence of a cherry red skin colour is found in about 50% of cases presenting to hospital. D : Pulse oximetry classically shows low oxygen saturation. E : Carbon monoxide cannot be measured in expired air. Comment : Carbon monoxide poisoning results in the death of about 50 people in the UK each year. It is produced by the incomplete combustion of carbon containing fuel: gas, coal, oil, wood and coke. Inadequate maintenance leading to poor combustion of fuel and inadequate removal of waste products as a result of blocked chimney are the main causes of poisoning.Headache is the most common symptom (90%) followed by nausea & vomitting (50%), vertigo (50%), alteration in consciousness (30%) and weakness (20%).The cherry red skin colour occurs when COHb concentration exceeds 20% but it is rarely seen in life. Pulse oximetry gives false high oxygen saturation and it is not recommended.CO levels can be measured in expired air and this must be done soon after the exposure. A : 133. A 24-year-old man presents with fever, breathlessness, cough and sputum production. His only medical history of note is long-standing heavy alcohol consumption, but he had no respiratory complaint whatever until 6 weeks ago when he developed breathing difficulty with high fever and rigors. He was given antibiotics by his general practitioner and began to improve, but this improvement has not been sustained. The most likely diagnosis is: A : Lung abscess B : Bronchiectasis C : Chronic obstructive pulmonary disease (COPD) D : Asthma precipitated by chest infection E : Empyema. Comment : Bronchiectasis typically causes chronic breathlessness and sputum production, often with febrile infective exacerbations, but will not be the diagnosis in a patient with a short respiratory history.The history strongly suggests pneumonia 6 weeks previously that has failed to clear. Empyema could cause persistent fever, malaise and breathlessness, but would not explain continued sputum production. Alcoholism is a risk factor for aspiration and cavitating pneumonia. A : 134. A 42-year-old asthmatic treated with high dose of inhaled corticosteroids developed a skin rash. His routine blood tests showed mild renal dysfunction and blood eosinophilia. His chest radiograph is normal. Which blood test would you arrange next? A : Aspergillus fumigates precipitins B : Total Ig E level C : Antineutrophil cytoplasmic antibodies(c-ANCA) D : Anti-nuclear antibody (ANA) E : Anti GB antibody. Comment : Skin rash and renal dysfunction in an asthmatic suggest vasulitis or Churg–Strauss syndrome. C :

Page 96: Masterclass Book Part 2

MOHAMMED IS-HAG 95

1. A 45-year-old man is found to have a positive urine dipstick for blood at a routine medical. He has no significant past medical history, has no symptoms, and is fit and well. What are the two most likely diagnoses? A : Renal carcinoma B : Alport’s syndrome C : Urinary stones D : Adult polycystic kidney disease E : Bladder cancer F : Minimal change glomerulonephritis G : Loin pain haematuria syndrome H : Diabetes mellitus I : IgA nephropathy J : Thin membrane nephropathy. Comment : Everyone has some red blood cells in their urine. Various studies have shown that 2.5-13% of men have positive urine dipstick tests for blood, the vast majority of whom do not have serious urinary tract or renal pathology.The younger the patient, the more likely that haematuria is glomerular in origin, with IgA nephropathy and thin membrane nephropathy the two commonest causes. Urinary tract tumours become more likely in older patients.20-30% of patients with IgA nephropathy progress to end-stage renal failure in 20 years. There is no effective specific treatment, although some advocate the use of dietary fish oil supplements. Management involves the use of angiotensin-converting enzyme (ACE) inhibitors, which retard the rate of progression of all types of proteinuric renal disease. I:J: 2.

The image shows an electronmicrograph of part of a glomerulus. This renal biopsy was from a patient with microscopic haematuria and proteinuria (1 g/24 hours). The plasma creatinine was in the normal range. ‘A’ marks the capillary lumen. Select one of the following statements: A : The glomerular basement membrane appears normal. B : The podocytes show extensive foot process fusion and effacement. C : The arrow indicates subendothelial deposits which are abnormal. D : These appearances effectively rule out systemic lupus erythematosus (SLE), subacute bacterial

endocarditis or mixed essential cryoglobulinaemia as causes of the nephritis. E : The most likely diagnosis is post streptococcal glomerulonephritis.

Page 97: Masterclass Book Part 2

MOHAMMED IS-HAG 96

Comment : The arrowhead indicates deposits that are located under the endothelium - i.e. subendothelial, resulting in an abnormally widened, irregular, glomerular basement membrane. The podocyte foot processes are essentially intact. In poststreptococcal nephritis the deposits are subepithelial, in contrast to those seen here. Subendothelial deposits reflect deposition of circulating immune complexes, and are seen in SLE nephritis (usually accompanied by deposits in other sites) with less common causes being mixed essential cryoglobulinemia, subacute bacterial endocarditis (SBE), and primary type I mesangiocapillary glomerulonephritis. C : 3. A 53-year-old man presents with haematuria and is suspected of having bladder cancer. In taking the history, which three of the following are not implicated in the pathogenesis of bladder cancer? A : Smoking B : Radiation exposure C : Schistosomiasis mansonii D : Benzene E : Analgesic abuse F : Naphthylamiones G : Cyclophosphamide H : Lithium I : Schistosomiasis haemotibium J : Azo dyes K : Bladder stones L : Aflatoxin M : Indwelling catheters N : Family history of bladder cancer O : Radioiodine for hyperthyroidism. Comment : Transitional cell carcinomas of the genitourinary tract are strongly linked to smoking and industrial exposure in the rubber/dye industries. Squamous carcinoma of the bladder is linked to chronic infection (S. haemotibium) or irritation. C :H:L: 4. A patient with lymphoma excretes 1.5g urinary protein but has a negative dipstick test. The reason for this seeming inconsistency is: A : dipsticks preferentially detect albumin rather than immunoglobulin because albumin is

negatively charged B : the molecular weight of the excreted protein is too low to be detected by dipsticks C : Tamm-Horsfall proteins block the reaction of the secreted protein and the dipstick D : the urine is not sufficiently concentrated E : dipsticks only recognise heavy chain sequences. Comment : A normal person excretes up to 150 mg of protein per day which is chiefly Tamm-Horsfall mucoprotein. Dipsticks can detect as little as 50 mg protein per litre. However, a false negative result occurs with immunoglobulins which are positively charged. A : 5. In a patient with resistant hypertension, which of the following would increase the degree of clinical suspicion for significant renovascular disease as aetiology? A : Arterial bruits B : Co-morbidity with diabetes C : Concentric left ventricular hypertrophy D : Grade III hypertensive change on fundoscopy E : Palpable kidney. Comment : The highest degree of clinical suspicion of renovascular disease will occur in patients with arterial bruits, discrepancy in renal size of >1.5 cm (on ultrasound) and co-morbid vascular disease. If a patient has none of these then renovascular disease is highly unlikely, Evidence of end-organ damage is not helpful in differentiating aetiology. A : 6. A 23-year-old African woman presents with seizures, hypertension, a rash, a raised ESR, a normal CRP and a creatinine of 373 micromoles/l. What is the most likely diagnosis? A : Myeloma B : Hashimoto’s disease C : Systemic lupus erythematosus D : Staphylococcal septicaemia E : Sickle cell disease. Comment : Systemic lupus erythematosus is a common condition and the prevalence is higher in women than men and is higher in black people than white. Typical features include skin rashes, neurological or psychiatric abnormalities and renal disease. It is often found that although the ESR is elevated, the CRP is normal. C :

Page 98: Masterclass Book Part 2

MOHAMMED IS-HAG 97

7. A pregnant 35-year-old presents with right upper quadrant pain at 28 weeks gestation. On examination, her blood pressure is 180/105. She has proteinuria, a creatinine of 120micromoles/litre and abnormal liver function tests. Her blood film shows a microangiopathic haemolytic anaemia and her platelet count is 30. A few weeks previously her blood pressure was 110/80 and her platelet count was 120. Which two of the following statements are true? A : As a pregnant woman her blood pressure would be expected to be higher than before the

pregnancy. B : The low platelets suggest that she has systemic lupus erythematosus. C : She certainly has essential hypertension. D : The development of proteinuria suggests pre-eclampsia. E : Blood pressure is usually higher in the first half than the second half of pregnancy. F : The problem is unlikely to recurr in a subsequent pregnancy. G : Alpha methyl dopa cannot safely be used to lower her blood pressure. H : Grand mal seizures may occur if the condition persists. I : She requires urgent dialysis. J : Acute liver failure is inevitable. Comment : Blood pressure falls during the first half of pregnancy and then rises again during the second half of pregnancy to levels approaching those before the pregnancy. Generally, blood pressure is a little higher in the second half than the first half of pregnancy. These changes probably reflect changes in peripheral resistance. Pregnancy induced hypertension can be severe and can be associated with endothelial and organ dysfunction. The clinical consequences of such dysfunction can be varied and certainly include proteinuria, renal impairment, abnormal liver function, microangiopathic haemolytic anaemia, platelet dysfunction and seizures. The priority is to reduce the blood pressure to prevent these changes. Aspirin is of prophylactic benefit in subsequent pregnancies, which carry a significant risk of recurrence. Alpha methyl dopa is a standard well tested drug used during pregnancy. Labetolol is also used. Other drugs, such as calcium channel blockers, have been used, but have a less well-established track record of safety during pregnancy D : H: 8. A 68-year-old lady presents with a 4-day history of anuria. She has a past history of intermittant claudication and is a long-term smoker. Which of the following statements are correct? A : If kidneys are of normal size renal artery occlusion can be excluded. B : If there is evidence of renal asymmetry and first investigation should be a biospy of the smaller

kidney. C : An IVU would be the investigation of choice. D : One should consider urgent arteriography and revascularisation. E : Weakly positive pANCA serology would establish a diagnosis of microscopic polyangitis. Comment : Absolute anuria would be a rare presentation of a rapidly progressive glomerulonephritis. A weakly positive pANCA is a relatively common finding and in many instances is a false positive result. One would need histology to confirm the diagnosis in these circumstances. Renal artery occlusion can present in this way and the presence of two normal size kidneys does to exclude the diagnosis. An IVU is not helpful in these instances, as non-opacification of the kidneys would occur in the presence of poor renal function irrespective of the cause. When there is renal asymmetry the larger of the two kidneys should be biopsied as the diagnostic yield is likely to be greater. Arteriography and revascularisation should be considered as this may well salvage the situation is carried out early enough. D : 9. An anxious 52-year-old woman has adult polycystic kidney disease. She shows you a long list of things that she says are associated with this condition. Which of the following is NOT a recognized association? A : Subarachnoid haemorrhage B : Cerebellar cysts C : Liver cysts D : Colonic diverticuli E : Mitral valve prolapse. Comment : Adult polycystic kidney disease (APKD) is the most common inherited renal disease leading to end-stage renal failure, accounting for 5-10% of those receiving renal replacement therapy. There are two known loci: PKD1 on chromosome 16 accounts for 85% of cases and PKD2 on chromosome 4 for 10%. Presentation is with abdominal pain, haematuria, hypertension, urinary tract infection, incidental discovery of an abdominal mass, or as a result of family screening (or serendipitous imaging of the kidneys, e.g. by ultrasound examination ordered for another purpose). B :

Page 99: Masterclass Book Part 2

MOHAMMED IS-HAG 98

10.

PLATE 19 The enclosed image shows a portion of a percutaneous renal biopsy taken from a 48-year-old woman with nephrotic syndrome. This includes medulla and transitional cell epithelium. Which one of the following statements is correct? A : The appearances would be consistent with cholesterol embolisation. B : The appearances are secondary to proteinuria. C : A portion of a normal glomerulus is seen in the biopsy. D : The procedure was probably complicated by haemorrhage. E : There is evidence of interstitial fibrosis. Comment : Percutaneous renal biopsy aims to obtain samples of the renal cortex. Samples of renal medulla are not only less likely to yield diagnostic information, but also carry a substantially increased risk of haemmorhage, particularly if the renal pelvis is breached by the biopsy needle - as has clearly occurred in this case since the sample includes transitional epithelium. D : 11. A 25-year-old man has been feeling unwell for a few days and has noticed his urine is darker than usual. His GP finds that he is hypertensive (BP 160/100), his urine is positive for blood and protein, and his creatinine is 940 µmol/l. The patient remembers having a sore throat about a week before becoming ill. You consider post-streptococcal glomerulonephritis to be a likely diagnosis. Select the two answers that you agree with. A : A renal biopsy showing a paucimmune focal segmental glomerulonephritis with crescent

formation would support your diagnosis. B : If your diagnosis is correct the patient has an approximately 85% likelihood of needing longterm

renal replacement therapy. C : The patient could have chronic renal failure with small kidneys on renal ultrasound. D : Red cell casts in the urine would make post-infectious nephritis unlikely. E : It would be reasonable to offer the patient an outpatient appointment within 1 week and ask the

GP to check the creatinine in 24 hours. F : Serological tests are generally unhelpful in diagnosing post-streptococcal nephritis. G : If your diagnosis is correct, plasma exchange should be offered as soon as possible. H : The patient may well need dialysis within 24 hours. I : The presence of antibodies to double stranded DNA would support your diagnosis. J : Typically in post-streptococcal glomerulonephritis circulating complement levels are normal. Comment : The patient certainly has renal failure. Whether this is acute or chronic, he is very likely to need dialysis within 24 hours. Indeed, emergency dialysis may be indicated depending on knowledge of the potassium, pH, the clinical status of the patient and whether there is a pericardial rub or a flap. Sore throats are common, and it may be that the patient has chronic renal failure, and the sore throat is a coincidence. Post-streptococcal nephritis generally resolves spontaneously and the likelihood of requiring long term renal replacement therapy if this is the underlying diagnosis is probably about 1%.

Page 100: Masterclass Book Part 2

MOHAMMED IS-HAG 99

Post-streptococcal nephritis has a diffuse proliferative appearance on light microscopy, and is associated with the presence of antibodies to streptococcal antigens and hypocomplementaemia, both of which are important in making the diagnosis. C :H: 12. A 50-year-old man attends the Accident and Emergency department and reports that on passing urine he also passed a small lump. He retrieved this and has brought it along in a jam jar of water. He is otherwise well apart from severe osteoarthritis of the left hip. His serum creatinine is 160 micromoles/l and his albumin is 40 g/l. A frozen section is performed by a helpful pathologist who reports that the lump is a renal papilla. Which of the following could account for this in his case? A : Membranous nephropathy B : Non-steroidal anti-inflammatory drug use for his osteoarthritis C : Nephrotic syndrome D : HIV nephropathy E : End stage renal disease. Comment : Analgesics, including non-steroidal anti-inflammatory drugs can cause papillary necrosis. This is likely to be caused by vasoconstriction of blood vessels within the kidney. Prostaglandins cause tonic vasodilatation of renal blood vessels. Non-steroidal anti-inflammatory drugs block prostaglandin synthesis and so block this vasodilation, resulting in relative vasoconstriction. B : . 13. A 36-year-old renal transplant receipient of 18 months is admitted with a history of a rising serum creatinine, pyrexia, weight loss, anaemia, abnormal liver enzymes and cervical lymphadenopathy. His immediate post transplant period was complicated by an episode of cellular rejection and one of vascular rejection treated with methylprednisolone and anti thymoglobulin (ATG) respectively. Which of the following statements are true regarding his condition? A : Cytomegalovirus (CMV) disease is the most likely underlying problem. B : His anaemia is best treated with erythropoeitin. C : A transplant biopsy is unlikely to be helpful in establishing a diagnosis. D : If post transplant lymphoma is diagnosed, graded reduction in immunosuppression is likely to be

of benefit. E : The features described are most consistant with chronic rejection. Comment : Post transplant lymphoma, or more correctly post transplant lymphoproliferative disease (PTLD), is associated with the degree of previous immunosuppression. In the case of this patient, he was exposed to baseline immunosuppressive drugs and also potent anti-rejection therapy in the form of ATG. It is often driven by Epstein–Barr virus and can be treated by graded reduction in immunosuppression. In a significant number of cases PTLD is present in the transplanted organ. Treatment of the underlying PTLD will often improve blood counts. It would be unusual for CMV disease to present at this time, usually occurring much earlier in the post transplant period. Chronic rejection rarely causes systemic symptoms such as those describe here. D : 14. A 36-year-old man presents with microscopic haematuria and hypertension. Ultrasound scan shows several cysts in the kidneys, and two solid lesions, the larger of which is 5 cm diameter. The man is not in contact with his family, but he knows that his father died undergoing surgery for a brain tumour aged 30 and also that one of his father's two sisters had some kind of kidney problem, and was deaf in one ear. Which of the following statements is true? A : Kidney surgery is unlikely to be indicated. B : Should the patient ever require renal replacement therapy, transplantation would not be an

option. C : The most likely diagnosis is Alport's disease. D : The patient may have von Hippel Lindau disease. E : It is reasonable not to investigate the hypertension further. Comment : von Hippel Lindau disease is transmitted as an autosomal dominant condition. Affected individuals may have any of the following: · Renal cysts · Clear cell renal cell carcinoma (CCRCC) · Retinal angiomas · Central nervous system haemangioblastoma · Phaeochromocytoma. Another manifestation is endolymphatic sac tumours, causing deafness. Below 3 cm, solid lesions can be monitored, but above this size they need removal. Hypertension suggests the patient may have a phaeochromocytoma, and this should be actively sought as it would be important to remove it. Although surgeons try to preserve renal tissue, ultimately operations to remove CCRCC may lead to end stage renal failure. As long as there is no evidence of metastasis, transplantation can be considered. D :

Page 101: Masterclass Book Part 2

MOHAMMED IS-HAG 100

15.

PLATE 20 A 52-year-old accountant, with a 6-year history of Raynaud’s phenomenon presents to her G.P. with a 4-month history of worsening dysphagia. Incubation of her serum on Hep 2 cells reveals the following immunofluorescent pattern (see image). A : Primary Raynauds disease B : systemic lupus erythematosus (SLE) C : Limited systemic sclerosis D : Rheumatoid arthritis E : Sjogren's disease. Comment : Raynaud’s phenomenon is common, occurring in 4-15% of the general population and in the majority of cases is not associated with connective tissue disease. However, the detection of an anti-centromere nuclear staining pattern has a high specificity for limited systemic sclerosis, especially in the context of dysphagia which is a feature of the disease. C : 16. You see a 12-year-old with sickle cell (SC) disease who describes a recent episode of macroscopic haematuria. The parents are worried that the child has serious kidney disease. Concerning sickle cell disease, select the two statements which you agree with: A : Renal involvement is more likely in SC disease than in Hb S homozygotes. B : Hypertension is common in patients with sickle cell disease. C : Renal transplantation is contraindicated in patients with sickle cell disease and end stage renal

ailure. D : Macroscopic haematuria is usually due to papillary necrosis. E : Survival of patients with sickle cell nephropathy on dialysis programmes is worse than patients

with diabetes of the same age. F : Red cells in the vasa recta of the renal medulla are protected from sickling by the hypertonic

environment. G : Loss of urinary concentrating ability is common in patients with sickle cell disease. H : Glomerular filtration rate is typically 30% of normal by the age of 10 years. I : Administration of recobinant human erythropoietin reduces the number of sickling crises by

approximately 50%. J : Renal medullary carcinoma is a relatively indolent tumour in patients with sickle cell disease. Comment :In sickle cell disease, sickling is a particular problem in the medullary vasa recta due to hypertonicity and hypoxia. Reducing medullary hypertonicity may reduce renal damage. Diminished concentrating ability is an early and universal finding in sickle cell disease. Survival in patients with sickle cell nephropathy is similar to non-diabetic dialysis patients. Interestingly early in the course of sickle cell disease GFR is actually increased, and only falls below normal by the age of about 30. Hypertension is significantly less common in sickle cell disease than in normal black individuals. Renal medullary carcinoma is a highly aggressive tumour which occurs in patients with sickle trait or sickle cell disease. D : G: 17. A 30-year-old man, a haemodilaysis patient with diabetic nephropathy, has the following blood results: calcium 2.9 mmol/l, phosphate 2.5 mmol/l. Alkaline phosphatase and albumen are in the normal range and PTH is at the lower end of the normal laboratory range. The patient's current medication includes alfacalcidol each day and calcium acetate as a phosphate binder before each meal. Select the most appropriate statement. A : Phosphate restriction in the diet should be reinforced, a change of phosphate binder may be

necessary and alfacalcidol should be reduced or stopped. B : The dialysate calcium should be lowered to return the calcium to the normal range. C : The calcium acetate dose should be reduced to return the calcium to the normal range. D : The patient has autonomous hyperparathyroidism. E : These values are satisfactory for a haemodialysis patient. Comment :

Page 102: Masterclass Book Part 2

MOHAMMED IS-HAG 101

The calcium x phosphate product is high putting the patient at risk of metastatic calcification. Lowering the phosphate (rather than the calcium) is the most important measure, as the elevated phosphate stimulates parathyroid proliferation and PTH secretion. This is best done by reinforcing dietary restriction and ensuring that the patient is recieving adequate dialysis. The vitamin D analogue will be increasing absorption of calcium and phosphate so this should be reduced or stopped; ideally the patient will remain on a small dose as this helps to stop parthyroid proliferation.If these changes do not return the calcium and phosphate to satisfactory values, the patient should be changed to a phosphate binder which does not contain calcium. Aluminium based compounds are best avoided because of the risk of toxicity so sevelamer would be an appropriate (but expensive) choice. A : 18. Which of the following would you regard as the clearest contraindication to choosing continuous ambulatory peritoneal dialysis as treatment modality for a patient with endstage renal failure? A : Presence of large (22 cm) bilateral polycystic kidneys B : Known impairment of left ventricular function C : Severe visual impairment D : Previous succesful repair of an inguinal herinia E : Presence of a defunctioning colostomy and mucus fistula. Comment : Impaired left ventricular function is a more significant concern for haemodialysis than continuous ambulatory peritoneal dialysis (CAPD). Visual impairment raises some challenges for CAPD, but these can be overcome in most instances. Moderate size polycystic kidneys do not usually present a problem for CAPD. E : 19. Which of the following options do you think would be the most appropriate in supporting a diagnosis of reflux nephropathy in a 40-year-old man with plasma creatinine 540 micromole/litre, 2.5g proteinuria per 24 hours and a childhood history of repeated urinary tract infections? A : Micturating cystogram B : Renal ultrasound C : Intravenous urography D : Computerized tomography with intravenous contrast E : Isotopic imaging with 99mTc-DTPA. Comment : In the majority of patients with reflux nephropathy, ureteric reflux does not persist beyond childhood so a micturating cystogram is not usually helpful. Ultrasound is a good method of detecting renal scars, although it is less sensitive than intravenous urography or DMSA scanning in detecting small scars in those with normal renal function. Intravenous urography is often relatively unhelpful in patients with this degree of renal impairment due to poor concentration of contrast. DMSA, which is taken up by tubular cells after filtration by the glomerulus, can be used to detect scars (as opposed to DTPA which is filtered by the glomerulus and not taken up by tubular cells). B : 20. A 19-year-old army recruit presents with acute renal failure. Which of the following features is not consistent with a diagnosis of rhabdomyolysis? A : elevated plasma creatinine kinase level B : anuria C : elevated plasma creatinine D : blood and red cell casts in the urine E : hyperkalaemia. Comment : Rhabdomyolysis results from muscle injury and the release of myoglobin from muscle cells. Myoglobin is toxic to the renal tubules and oliguria or even anuria can occur. Myoglobinuria usually occurs. Muscle cells also release creatinine kinase and potassium when they are injured. Red cell casts are not a feature of rhabdomyolysis and would suggest a glomerular disease. D : 21. A 46-year-old man presents with a 33-year history of lithium carbonate therapy for bipolar affective disorder. He is polyuric (11 litres) and has 2 grams / 24 hours of proteinuria. His GFR is 47-mls/min/1.73 m2. BP is raised at 150/100 mm Hg. He has a renal biopsy. What would be typical findings? A : IgA glomerulonephritis B : Crescentic glomerulonephritis C : Renal Vasculitis D : Focal glomelurosclerosis and interstitial nephritis E : Granulomata. Comment : Lithium toxicity is much more common than appreciated by most psychiatrists. It is rarely detected early enough for reversibility. It is typically a chronic interstitial nephritis but a secondary FSGS lesion due to hyperfiltration of remnant nephrons can be seen in 30% of cases. Options include running lithium at very low levels, or a switch to valproate or carbemazepine. D :

Page 103: Masterclass Book Part 2

MOHAMMED IS-HAG 102

22.

PLATE 21 A 28-year-old woman received a kidney transplant from her brother. Six weeks post transplantation she was admitted with epigastric pain and vomiting. She was febrile. Simple blood tests demonstrated markedly abnormal LFTs with a hepatocellular pattern. An oesophago-gastro-duodenoscopy (OGD) was performed and the gastric biopsy is shown below. Which of the following medications had been given at the time of transplantation to try to prevent this complication? A : Aspirin B : Ranitidine C : Nifedipine D : Septrin E : Acyclovir. Comment : The gastric biopsy shows a number of very abnormal large cells. These cells contain both nuclear and cytoplasmic inclusion bodies. They are characteristic of cytomegalovirus infection . The donor in this case was CMV positive and the recipient was CMV negative. This is a very high risk combination for CMV disease and antiviral prophylaxis was given in the form of acyclovir. This may not be the most effective prophylaxis and some units will use either oral gancicolvir or valacyclovir. The diagnosis in this case was further confirmed by polymerase chain reaction (PCR) analysis of blood to detect CMV DNA and antingenaemia testing to show the presence of replicating CMV in the patients WBCs. She responded well to intravenous ganciclovir.E: 23. A 36-year-old renal transplant receipient of 18 months is admitted with a history of a rising serum creatinine,pyrexia,weight loss,anaemia,abnormal liver enzymes and cervical lymphadenopathy. His immediate post transplant period was complicated by an episode of cellular rejection and one of vascular rejection treated with methylprednisolone and anti thymoglobulin (ATG) respectively. Which of the following statements are true regarding his condition? A : Cytomegalovirus (CMV) disease is the most likely underlying problem. B : His anaemia is best treated with erythropoeitin. C : A transplant biopsy is unlikely to be helpful in establishing a diagnosis. D : If post transplant lymphoma is diagnosed, graded reduction in immunosuppression is likely to be

of benefit. E : The features described are most consistant with chronic rejection. Post transplant lymphoma, or more correctly post transplant lymphoproliferative disease (PTLD), is associated with the degree of previous immunosuppression. In the case of this patient, he was exposed to baseline immunosuppressive drugs and also potent antirejection therapy in the form of ATG .It is often driven by EpsteinBarr virus and can be treated by graded reduction in immunosuppression. In a significant number of cases PTLD is present in the transplanted organ. Treatment of the underlying PTLD will often improve blood counts. It would be unusual for CMV disease to present at this time, usually occurring much earlier in the post transplant period. Chronic rejection rarely causes systemic symptoms such as those describe here. D :

Page 104: Masterclass Book Part 2

MOHAMMED IS-HAG 103

24. A 30-year-old man, a haemodilaysis patient with diabetic nephropathy, has the following blood results: calcium 2.9 mmol/l, phosphate 2.5 mmol/l. Alkaline phosphatase and albumen are in the normal range and PTH is at the lower end of the normal laboratory range. The patient's current medication includes alfacalcidol each day and calcium acetate as a phosphate binder before each meal. Select the most appropriate statement. A : Phosphate restriction in the diet should be reinforced, a change of phosphate binder may be

necessary and alfacalcidol should be reduced or stopped. B : The dialysate calcium should be lowered to return the calcium to the normal range. C : The calcium acetate dose should be reduced to return the calcium to the normal range. D : The patient has autonomous hyperparathyroidism. E : These values are satisfactory for a haemodialysis patient. Comment : The calcium x phosphate product is high putting the patient at risk of metastatic calcification. Lowering the phosphate (rather than the calcium) is the most important measure, as the elevated phosphate stimulates parathyroid proliferation and PTH secretion. This is best done by reinforcing dietary restriction and ensuring that the patient is recieving adequate dialysis. The vitamin D analogue will be increasing absorption of calcium and phosphate so this should be reduced or stopped; ideally the patient will remain on a small dose as this helps to stop parthyroid proliferation. If these changes do not return the calcium and phosphate to satisfactory values, the patient should be changed to a phosphate binder which does not contain calcium. Aluminium based compounds are best avoided because of the risk of toxicity so sevelamer would be an appropriate (but expensive) choice. A : 25. A 28-year-old man has had several attacks of renal colic. He passes stones consisting mainly of calcium oxalate. His 24-hour urinary calcium excretion is twice the upper limit of normal. Which two alterations to his diet are most likely to be helpful in reducing the risk of stone recurrence? A : Reduced salt intake B : Increased fluid intake C : Avoidance of alcohol D : Avoidance of refined sugar E : Avoidance of foods high in oxalate F : Instigation of a calcium-free diet G : Avoidance of dairy products H : Increased salt intake I : Reduced fluid intake J : Avoidance of dietary meat. Comment : Whatever the cause of urinary stones, increasing fluid intake always reduces the concentration of stone-forming substances in the urine and is always the most important modification to diet. Patients should be instructed to drink enough to obtain a urinary volume of over 3 litres / day, and those who continue to be troubled by stones may need to drink even more. Hypercalciuria, usually idiopathic, is found in 65% of patients with urinary stones and is explained by intestinal hyperabsorption of calcium. However, reducing dietary calcium intake is NOT the answer, since this leads to increased intestinal absorption of oxalate and can thereby increase the risk of stone formation. Avoidance of non-staple foods high in oxalate (e.g. rhubarb, spinach) is sensible. There is also a (lesser) role for reduction in intake of salt and refined sugar. B : E : 26. A 45-year-old man with polycystic kidney disease and plasma creatinine 300µmol/l is found to have potassium 5.1 mmol/l, calcium 2.25 mmol/l, albumin 40 g/l, phosphate 1.65 mmol/l, and PTH 20 pmol/l (NR 1.1-6.8). He has already seen a specialist dietician for advice concerning potassium and phosphate intake. He is currently taking an angiotensin-converting enzyme (ACE)-inhibitor for hypertension and a proton pump inhibitor for reflux oesophagitis. Which of the following statements is true? A : He should be commenced on alfacalcidol 0.5 mcg od, to correct deficiency of active vitamin D and suppress parathyroid hormone (PTH). B : An appropriate treatment would be calcium acetate 1g bd to be taken with meals. C : Aluminium hydroxide should be the first choice if a phosphate binder is prescribed. D : Parathyroidectomy should be considered, especially if he is keen on having a renal transplant in

the future. E : Sevelamer (Renagel) should definitely be used here rather than other phosphate binders. Comment : Hydroxylated vitamin D does suppress PTH expression, and alfacalcidol should be considered for prophylaxis against renal bone disease and progressive hyperparathyroidism. However, the patient's phosphate level is already elevated, and vitamin D supplementation will tend to increase this further. Even if the phosphate level were not elevated this would be an inappropriately high starting dose. Parathyroidectomy is not indicated here - the patient only has a mildly elevated level of PTH which is likely to improve with better phosphate control. There would also be a risk of adynamic bone disease.

Page 105: Masterclass Book Part 2

MOHAMMED IS-HAG 104

Aluminium-containing phosphate binders carry the risk of aluminium accumulation and CNS effects. Sevelamer is only clearly indicated if calcium-containing binders cannot be used because the calcium level (or calcium-phosphate product) are undesirably high. Calcium acetate is a very reasonable choice here. It should be taken with (or just before) meals and may offer advantages over calcium carbonate, especially in patients with reduced gastric acidity. B : 27. A 37-year-old man, a type I diabetic for six years, presents with a rising creatinine over two years. There is mild background diabetic retinopathy and no neuropathy. Proteinuria was also noted two years ago, and is now 12 grams / day and he is nephrotic. He has no microscopic haematuria. BP is mildly elevated at 145/95 mm Hg. This is not typical of diabetic nephropathy because: A : Onset of renal problems too soon after diagnosis of type I DM B : Patients under 40 do not get renal diabetic complications C : Significant hypertension is invariable in diabetic nephropathy D : The absence of microscopic haematuria excludes a glomerular lesion E : Nephrotic syndrome is not due to diabetic nephropathy. Comment : Typically it is 10 - 20 years after the diagnosis of type I DM that patients first experience micro-albuminuria, then heavier proteinuria, and renal functional decline (as hyperfitration and raised GFR finishes). Thus this is too soon, and also the proteinuria did not precede the renal impairment. Hypertension is seen in about 90% of patients with diabetic nephropathy (remember elevated BP in this setting is anything > 130/80 mm Hg). Few patients with diabetic nephropathy have microscopic haematuria, but some do). About one third of diabetic nephropathy patients go through a nephrotic phase. This patient requires a renal biopsy (and had membranous nephropathy which responded to immunosuppression). A : 28. In a patient presenting with the nephrotic syndrome, which one of the following features is not a recognised association? A : Diabetes mellitus B : Myeloma C : Cigarette smoking D : HIV infection E : Systemic lupus erythematosus. Comment : Diabetic nephropathy can cause heavy proteinuria, sufficient to cause the nephrotic syndrome. Myeloma can cause AL amyloidosis which is associated with proteinuria that can be severe enough to cause the nephrotic syndrome. HIV infection has been associated with a form of focal segmental glomeruosclerosis which is a cause of the nephrotic syndrome. Systemic lupus erythematosus can cause various patterns of glomerulonephritis, including a membranous nephropathy which is a cause of the nephrotic syndrome. C : 29. A 45-year-old man with polycystic kidney disease and plasma creatinine 300µmol/l is found to have potassium 5.1 mmol/l, calcium 2.25 mmol/l, albumin 40 g/l, phosphate 1.65 mmol/l, and PTH 20 pmol/l (NR 1.1-6.8). He has already seen a specialist dietician for advice concerning potassium and phosphate intake. He is currently taking an angiotensin-converting enzyme (ACE)-inhibitor for hypertension and a proton pump inhibitor for reflux oesophagitis. Which of the following statements is true? A : He should be commenced on alfacalcidol 0.5 mcg od, to correct deficiency of active vitamin D

and suppress parathyroid hormone (PTH). B : An appropriate treatment would be calcium acetate 1g bd to be taken with meals. C : Aluminium hydroxide should be the first choice if a phosphate binder is prescribed. D : Parathyroidectomy should be considered, especially if he is keen on having a renal transplant in

the future. E : Sevelamer (Renagel) should definitely be used here rather than other phosphate binders. Comment : Hydroxylated vitamin D does suppress PTH expression, and alfacalcidol should be considered for prophylaxis against renal bone disease and progressive hyperparathyroidism. However, the patient's phosphate level is already elevated, and vitamin D supplementation will tend to increase this further. Even if the phosphate level were not elevated this would be an inappropriately high starting dose. Parathyroidectomy is not indicated here - the patient only has a mildly elevated level of PTH which is likely to improve with better phosphate control. There would also be a risk of adynamic bone disease. Aluminium-containing phosphate binders carry the risk of aluminium accumulation and CNS effects. Sevelamer is only clearly indicated if calcium-containing binders cannot be used because the calcium level (or calcium-phosphate product) are undesirably high. Calcium acetate is a very reasonable choice here. It should be taken with (or just before) meals and may offer advantages over calcium carbonate, especially in patients with reduced gastric acidity. B :

Page 106: Masterclass Book Part 2

MOHAMMED IS-HAG 105

30.

PLATE 3

A 73-year-old man presents with a three month history of feeling non-specifically unwell, has lost 6 kg in weight and has recently noticed the rash shown in the enclosed image. The creatinine is 340 µmol/l. Urinalysis shows blood and protein. Renal ultrasound shows kidneys of 10.8 and 11.2 cm with increased cortical echogenicity. Which of the following statements is correct? A : Renal biopsy should be performed and is likely to show a focal necrotising glomerulonephritis B : The renal impairment is likely to be chronic (i.e. irreversible) C : Skin biopsy is likely to yield a specific diagnosis D : Renal biopsy should be performed and is most likely to show membranous glomerulonephritis E : Laboratory blood tests are unlikely to yield a specific diagnosis. Comment : The rash shown is consistent with a leucocytoclastic vasculitis, and the history would be entirely consistent with ANCA (antineutrophil cytoplasmic antibodies) positive systemic vasculitis. A skin biopsy will not yield a specific diagnosis, whereas a test for ANCA is likely to be positive.In view of the preserved renal size, active urine sediment and rash, it is likely that there is an acute (i.e. reversible) component to the renal failure and a biopsy should be performed. This is most likely to show a pauci-immune, focal segmental glomerulonephritis, but it is important to exclude other conditions. A : 31. In a patient with diabetes mellitus and a serum creatinine of 210 micromoles per litre, which one of the following findings would lead you to suspect a diagnosis other than diabetic nephropathy? A : Normal fundoscopy B : HbA1C of 8.1% C : Normal-sized kidneys on ultrasound examination D : Proteinuria of 1.2g per 24 hours E : Knowledge that the creatinine had been 110 micromoles per litre 18 months earlier. Comment : Diabetic nephropathy is a microvascular complication and rarely occurs without other evidence of microvascular disease, such as retinopathy. It is often associated with suboptimal glycaemic control and proteinuria. Renal size is preserved but following the development of microalbuminuria the disease may progress with alarming rapidity. A : 32. In renal amyloidosis, which one of the following statements is correct? A : nephrotic syndrome is a rare presentation B : amyloid deposits can affect the glomeruli, tubules and blood vessels C : there are nodular glomerulosclerotic lesions that mimic diabetic nephropathy D : renal vein thrombosis is not seen E : regression of deposits, and resolution of proteinuria, cannot be achieved.

Page 107: Masterclass Book Part 2

MOHAMMED IS-HAG 106

Comment : In renal amyloidosis there is amorphous deposition of amyloid proteins in the tissues. Clinical presentation is with proteinuria, nephrotic syndrome or chronic renal failure. Renal vein thrombosis may complicate, as it can any cause of the nephrotic syndrome. AL amyloid may respond to cytotoxic therapy, but many cases do not, and in patients who are elderly and frail a conservative approach is usually adopted. B : 33. A 17-year-old male presents with a short history of sudden onset of severely swollen ankles. He is otherwise well, and urinalysis shows +++ protein. A urine collection shows that he is excreting 6g protein per 24 hours, the plasma albumen is reduced, and the creatinine is in the normal range. What is the most likely underlying diagnosis? A : Thin membrane nephropathy B : Membranous nephropathy C : Minimal change nephrotic syndrome D : IgA nephropathy E : Mesangiocapillary glomerulonephritis. Comment : The patient has nephrotic syndrome and at this age the commonest underlying diagnosis on renal biopsy is minimal change nephrotic syndrome. This would also be consistent with the lack of haematuria, the sudden onset and the normal excretory renal function. Thin membrane nephropathy presents with asymptomatic haematuria. IgA nephropathy can present with nephrotic syndrome but this is relatively unusual and microscopic haematuria is usually present.Mesangiocapillary glomerulonephrits or membranous nephropathy can present with nephrotic syndrome but are less common. C : 34. A 25-year-old woman presents to A&E 1 hour after consuming 28 x 500mg paracetamol tablets. Which of the following statements is true? A : If the INR is normal on a sample taken four hours from the time of ingestion, liver damage is

unlikely to occur. B : Alcohol ingestion at the time of consumption of paracetamol is an indication for N-acetyl-

cysteine treatment if paracetamol level at 4 hours exceeds the ‘high-risk’ line. C : Activated charcoal may be beneficial if given immediately. D : Onset of tinnitus may be an early symptom of liver failure. E : Deterioration in conscious level within the first 24 hours usually suggests hepatic

encephalopathy. Comment : Abnormal blood clotting following paracetamol overdose results from loss of production of clotting factors by hepatocytes, and is therefore a good early marker of synthetic liver function. The INR rises first because Factor VII has the shortest half-life and is therefore the first to disappear from the blood. However it is unusual to see any abnormality in blood clotting less than 18 hours from ingestion, so normal INR at 4 hours is unhelpful. Abnormal INR at the time of admission may indicate prior chronic liver disease, warfrin ingestion or suggest that ingestion of the drug occurred earlier than the patient reports. Because acute alcohol intoxication inhibits liver enzyme function, less paracetamol will be metabolised to the toxic metabolite, so that liver damage is, if anything, less likely. Chronic alcohol abuse causes liver enzyme induction and is an indication for treatment with N-acetyl cysteine if paracetamol level exceeds the ‘high risk’ line. Activated charcoal is only likely to be beneficial if given within 1 hour of ingestion of paracetamol. If the patient complains of tinnitus, this suggests concurrent salicylate consumption, which requires specific treatment according to the plasma level. Hepatic encephalopathy rarely occurs less than 48 hours from consumption; reduction in level of consciousness in the first 24-48 hours is usually a result of concurrent consumption of sedative drugs (particularly opiates in combination drugs such as co-dydramol of co-proxamol) or hypoglycaemia. Regular monitoring of blood glucose is recommended for patients with elevated paracetamol level requiring treatment with N-acetyl cysteine, or following any change in conscious level. C : 35. A 55-year-old man with long-standing type II diabetes and intermittent claudication has recently started on continuous ambulatory peritoneal dialysis (CAPD) for end stage renal failure (ESRF) which is presumed to be due to diabetic nephropathy. He is blood group A, his wife is blood group O, and they have two children. He is interested in the possibility of transplantation. Which of the following is true? A : Risk benefit analysis shows that diabetics with end stage renal failure are better treated with

dialysis than transplantation. B : In the UK, wife to husband transplants are not permitted under current legislation. C : An exercise ECG should be performed to determine if he needs coronary angiography prior to

transplantation. D : Recent evidence suggests that this patient would benefit from combined kidney and pancreas

transplantation. E : If a transplant is performed the CAPD catheter would usually be left in situ at the time of the

operation. Comment : Although diabetics with end stage renal disease have a relatively poor survival rate following transplantation, their survival on dialysis is dismal; risk benefit analysis shows that this group

Page 108: Masterclass Book Part 2

MOHAMMED IS-HAG 107

may benefit most from transplantation compared to dialysis. Wife-to-husband transplants (and vice-versa) are legal in the UK and the results are generally excellent. An exercise ECG is not sufficient evaluation here, especially as the intermittent claudication may limit the test. This patient would need coronary angiography, and iliac angiography, prior to transplantation. Pancreatic transplantation is desirable in type I diabetics. The PD catheter is usually left in situ - in the event that the graft does not work for a period the patient can then be dialysed easily. The catheter is usually removed 3 months after a successful transplant. E : 36. A 35 year-old women presented 2 hours after collapsing at home with severe headache. On examination she was drowsy and had neck stiffness. Her temperature was 37.5. She had a mild right hemiparesis. Which of the following is the most appropriate first diagnostic investigation? A : CT brain scan B : Lumbar puncture and examination of the cerebrospinal fluid C : Four-vessel angiography D : MR angiogram E : MR brain scan. Comment : The history is suggestive of a subarachnoid haemorrhage. Urgent CT brain scan will identify more than 95% of patients with suspected subarachnoid haemorrhage if performed within 1-2 days after headache onset (van Gijn 1982). Lumbar puncture is potentially dangerous and will add no extra information if brain CT shows definite evidence of extravasated blood. If CT is negative and there are no contraindications, LP should be performed.Four vessel angiography will be needed later once the patient is stabilised to identify the source of the bleeding and surgical or endovascular treatment. In the acute stages of SAH (because of the speed of investigation and availability) CT imaging is superior to MRI. MRI imaging becomes much more useful if presentation is delayed after the first few days (>4) when CT sensitivity for subarachnoid blood rapidly declines.MR angiography is without risks and reasonably sensitive (90%) – useful for screening people at risk of intracranial aneurysms but less suitable for patients with subarachnoid haemorrhage. A : 37. In atherosclerotic renal artery stenosis (ARAS), which of the following is a clear-cut reason for arterial intervention (i.e. angioplasty/stenting)? A : Presence of raised blood pressure B : Heavy proteinuria C : Flash pulmonary oedema D : > 75% stenosis but normal contralateral kidney E : Abdominal bruit. Comment : The number of compelling reasons for arterial intervention in this setting is modest - perhaps only flash pulmonary oedema, and severe stenosis in a single functional kidney. In ARAS the effect on BP of successful angioplasty or stenting is modest and disappointing - though hidden within the unimpressive "group" response to intervention, there can be individual striking responses (here, it is the "Resistive Index" concept recently reported by Rademacher et al (N Engl J Med 346(6):410-417) which has proved potentially useful in determining which patients may respond well). Proteinuria can be trivial or nephrotic-range in ARAS depending on whether there is secondary FSGS. An abdominal bruit is not a useful sign except to indicate abdominal atherosclerosis (more accurately, turbulent blood flow). The presence of a stenosis alone (except to those with an uninhibited oculo-stenotic reflex, often remuneration-driven) is not enough to warrant intervention on present evidence - this is what the mainly UK-based ASTRAL study is investigating currently (contact me / John Scoble for details). C : 38. You are providing a seminar to patients approaching the need for renal replacement. A group of patients contemplating starting continuous ambulatory peritoneal dialysis (CAPD) ask you about medical contraindications to this modality. Which of the following is not a relative contraindication to CAPD treatment? A : Previous perforated diverticular disease and sigmoid colectomy B : Bilateral inguinal herniae C : Diabetes mellitus D : Severe chronic obstructive pulmonary disease E : Arthritis mutilans. Comment : Previous pelvic surgery reduces the likelihood of successful peritoneal dialysis, adhesions often making catheter placement difficult and reducing the peritoneal surface available for dialysis. Inguinal herniae rapidly fill with peritoneal dialysate causing patient discomfort and inefficient dialysis. Peritoneal dialysis is a good treatment for patients with diabetes mellitus although the glucose load in the dialysate will necessitate a change in insulin dosage, which can be administered intraperitoneally. Splinting of the diaphragm by a large volume of intraperitoneal fluid often exacerbates chronic obstructive pulmonary disease (COPD), and this should be considered a relative, but not absolute contraindication. Effective peritoneal dialysis requires a degree of manual dexterity and would not be easy for a patient with arthritis mutilans. C :

Page 109: Masterclass Book Part 2

MOHAMMED IS-HAG 108

39.

PLATE22 A 30-year-old woman presents with arthralgia and tiredness. She is found to be hypertensive (160/100 mmHg). The creatinine is 300 micromol/l, there is 0.9 g proteinuria per 24 hours and the kidneys are normal size on ultrasound. Full blood count shows a slightly reduced platelet count, mild anaemia and evidence of microangiopathy. The patient's hand is shown (see image). Select one of the following answers: A : High-dose oral or intravenous steroids are the most effective treatment. B : Angiotensin-converting enzyme (ACE) inhibitors should be avoided in this setting. C : The most likely diagnosis is haemolytic–uraemic syndrome. D : Autoantibodies to glomerular basement membrane are likely to be present in the serum. E : Renal biopsy is likely to show arteriolar changes, fibrin thrombi and fibrinoid necrosis. Comment : The cutaneous appearances and renal presentation are typical of scleroderma with a renal crisis. In patients who develop a renal crisis this often occurs early in the course of the disease. There is usually relatively minor proteinuria, hypertension and evidence of microangiopathy. Typically the renal biopsy will show arteriolar changes and fibrinoid necrosis, and the best current treatment is with ACE-inhibitors. E : 40. During a routine medical check, a 27-year-old man who is taking no medication is found to have a blood pressure of 180/97 and is found to have a low serum potassium. His urine contains only a trace of protein on dipstick analysis. Which of the following diagnoses is likely? A : Minimal change nephropathy B : Primary hyperaldosteronism C : Mesangiocapillary glomerulonephritis D : Primary hyperparathyroidism E : Pseudohypoaldosteronism. Comment : Aldosterone enhances distal nephron sodium reabsorption and potassium secretion in the collecting tubules. The result is that if aldosterone levels are raised, potassium depletion is commonly seen. High levels of aldosterone can also cause sodium retention and hypertension. A high blood pressure can itself cause damage to the glomeruli and a trace of proteinuria is a common finding in hypertensive patients. B : 41. Which of the following is a feature of X-linked hypophosphataemic vitamin D -resistant rickets? A : Elevated parathyroid hormone (PTH) B : Hypercalacaemia C : Bone pain D : Hyperphospataemia E : Low bicarbonate.

Page 110: Masterclass Book Part 2

MOHAMMED IS-HAG 109

Comment : This is the commonest hereditary form of renal phospate wasting. Serum phospate is low and urine phosphate is inappropriately high. This in combination with abnormal osteoblast activity leads to abnormal mineralisation of growing bone and Rickets. The defect is in the PHEX gene coding for a Zn metallopeptidase, although the mechanism of pathogenesis is unclear. Treatment is with oral phosphate (difficult to tolerate) and high dose activated Vitamin D. Renal damage may ensue due to treatment-associated hypercalcaemia. C : 42. In cholesterol embolisation syndrome: A : Fever is a recognised feature. B : Hypocomplementaemia is not seen. C : Inflammatory markers are not raised. D : Anti-coagulation is recommended. E : Statin-based lipid-lowering therapy is not helpful. Comment : In cholesterol embolisation syndrome: · Acute presentations can be associated with fever. · Low complement is seen as part of the pseudo-vasculitis. · C-reactive protein (CRP) and erythrocyte sedimentation rate (ESR) are raised and there may be an eosinophilia. · Anti-coagulation can cause the problem and is contra-indicated. · Statins help by stabilising atherosclerotic plaques. A : 43. Which of the following is NOT a recognised cause of renal disease in sarcoidosis? A : Membranous nephropathy B : Amyloidosis C : Granulomatous interstitial nephritis D : Nephrolithiasis E : Hypercalaemia. Comment : Renal disease in patients with sarcoidosis can be identified in up to 20% of cases at post mortem although clinically significant disease is less common. Up to 14% develop renal stone and hypercalcaemia may lead to tubular toxicity and dehydration. The classic lesion of granulomatous interstitial nephritis is often found on renal biopsy but again is not usually clinically significant. A variety of glomerular diseases including membranous nephropathy have been described although causal relationship is not certain. Amyloidosis is not a feature of sarcoid. B : 44. An 18-year-old woman had macroscopic haematuria at the time that she had a sore throat. Following this she is found to have persistent microscopic haematuria, 0.1g/24-hour proteinuria and a normal plasma creatinine. Blood pressure is 115/64. A renal biopsy shows mesangial deposition of IgA on immunofluorescence. The tubules and interstitium are normal. Which of the following statements is true? A : An angiotensin-converting enzyme (ACE) inhibitor should be given since it is known to reduce the risk of progressive renal failure in this setting. B : There is approximately a 30% chance of end stage renal failure over the next 5 years. C : It is important that the patient's siblings are screened by testing their urine for blood. D : The prognosis is relatively good compared to other patients with IgA nephropathy. E : Circulating levels of C3 are likely to be reduced. Comment : Episodes of macroscopic haematuria, normal renal function, female sex, normal blood pressure, minimal proteinuria and normal tubulointerstitial morphology all place this patient in a good prognostic category. Although there is probably a genetic component to IgA nephropathy it is very unusual for relatives to be affected. D : 45. A 32-year-old man presents to his GP with a non-specific history of malaise. He has no history of diarrhoea. Initial investigations identify a significant anaemia with fragmentation on the blood film and low platelets. A clotting screen is normal. His biochemistry identifies significant renal dysfunction with a serum creatinine of 260umol/l. BP was recorded at 170/100. What is the most likely diagnosis? A : Thrombotic thrombocytopenic purpura (TTP) B : Haemolytic uraemic syndrome C : Malignant hypertension D : Disseminated intravascular coagulation E : Systemic vasculitis. Comment : This is a classic presentation of non-diarrhoeal haemolytic uraemic syndrome (D- HUS) or atypical HUS. This may be sporadic or familial. In sporadic cases association with HIV, malignancy, systemic lupus erythematous (SLE) and some drugs (e.g. cyclosporin) has been reported. In some familial cases mutations have been detected in Factor H which regulates alternative pathway activation of compliment.TTP produces a similar picture but usually also has evidence of neurological involvement. There is also evidence that the mechanism underlying the disease is different with reduced activity of a von Willebrand factor cleaving protease ADAMTS13. Treatment of sporadic HUS is with plasma exchange and fresh frozen plasma. The renal outlook is relatively poor and mortality around 10%. B :

Page 111: Masterclass Book Part 2

MOHAMMED IS-HAG 110

46.

This patient presented acutely unwell with the following ECG (see image). What is the correct diagnosis? A : Hyperkalaemia B : Ventricular tachycardia C : Acute pericarditis D : Acute anterolateral myocardial infarction E : Digoxin toxicity. Comment : The presence of tall, peaked T waves, flattened P waves, prolonged PR interval and wide QRS complexes are pathognomonic of hyperkalaemia. Give IV calcium immediately (10mls, 10% calcium gluconate) and call the renal team. A : 47. The following are NOT used for the estimation of renal function: A : Serum creatinine B : Serum cystatin C C : Creatinine clearance D : Ethylenediaminetetraacetic acid (EDTA) clearance E : Urinary albumin/creatinine ratio. Comment : Serum creatinine is the most commonly used screening test for renal function. Cystatin C is currently being evaluated and may prove to be more accurate in mild renal impairment. Creatinine clearance is used as a surrogate test for measuring the GFR. The clearance of the radioisotope EDTA however is the most accurate marker of GFR in routine clinical practice. The albumin / creatinine ratio is a method of quantifying urinary protein losses and not an estimate of renal function. E : 48. Which of these statements concerning the kidney is correct? A : The main site of action of thiazide diuretics is the thick ascending limb of the loop of Henle. B : Renin is produced primarily in the collecting duct. C : Parathyroid hormone increases phosphate reabsorption by the kidney. D : The main site of action of frusemide is in the thick descending limb of the loop of Henle. E : Calcidiol is hydroxylated by 1-alpha hydroxylase in the tubular epithelial cells. Comment : Frusemide acts on the Na+K+2Cl- cotransporter in the thick ascending limb of Henle's loop; thiazides act mainly on the Na+Cl- cotransporter in the distal convoluted tubule. Parathyroid hormone decreases phosphate reabsorption by the kidney. Renin is produced by the juxta glomerular apparatus. E : 49. In systemic vasculitis: A : the kidney is rarely affected B : macroscopic haematuria is typical C : focal necrotising glomerulonephritis and crescent formation are typical D : large deposits of immunoglobulin are seen in glomeruli on immunofixation E : interstitial inflammatory change on a renal biopsy is not recognised.

Page 112: Masterclass Book Part 2

MOHAMMED IS-HAG 111

Comment : Renal involvement is common, often with a very active urinary sediment (microscopic haematuria, red cell casts and proteinuria). Many cases are associated with a positive antineutrophil cytoplasmic antibody (ANCA), although this is neither diagnostic or essential. Immunostains are usually negative and the renal lesion is often termed pauci immune. C : 50. In the investigation of significant proteinuria (> 1 gram / 24 hours), the following are relevant factors: A : Albumin/creatinine ratio B : Urinary sodium concentration C : Urinary albumin to IgG ratio D : Tamm–Horsfall protein concentration E : Urinary complement concentration. Comment : Urinary albumin/creatinine ratio is another way to quantitate proteinuria. Urinary sodium has little bearing on proteinuria. Tamm–Horsfall protein is a normal constituent of urine. Urine complement levels are not of clinical use. The ratio of albumin leak to large MW protein leak is a marker of protein selectivity - in minimal change disease typically there is preferential albuminuria, whereas in many other renal protein leaking conditions in adults there is non-selective proteinuria. C : 51. A 60-year-old man has end-stage renal failure and is about to start renal replacement therapy. He has read a booklet about peritoneal dialysis, but seems to have got himself muddled. Which one of the following statements is true about peritoneal dialysis? A : it is inferior to haemodialysis B : it is associated with long-term changes in peritoneal membrane structure and function C : infection rates are typically more than 1 episode of peritonitis per 6 months of treatment D : it is contra-indicated in diabetic patients E : it means patients are unable to go to work because of dialysis exchanges. Comment : Peritoneal dialysis (PD) was the bedrock of dialysis in the UK in the 1970s, and has become so in other countries with inadequate health resources. In wealthier countries it has been slower to become established and accounts for a far smaller proportion of treated patients. Actuarial survival is the same, or better, for PD in the early stages of dialysis, but once residual renal function is lost, PD increasingly struggles to dialyse many patients well. Some motivated patients can work with co-operation from employers. Travel on PD is also much easier. Diabetic patients can fare well, though the glucose load from PD changes insulin requirements. Peritonitis rates of one infection per 18 - 24 patient months are now the norm with modern systems. Automated peritoneal dialysis (shorter duration high volume overnight exchanges while the patient is asleep) is more convenient for many patients, and can extend the useful life of PD in 'fast transporters', i.e. patients in whom the osmotic gradient collapses quickly. B : 52. A 23-year-old man presents with a rash on his legs. Stick testing of his urine reveals proteinuria + and haematuria +++. What is the most likely diagnosis? A : Henoch Schonlein purpura B : Mixed essential cryoglobulinaemia C : Minimal change nephropathy D : IgA nephropathy E : Membranous glomerulonephritis. Comment : Mixed essential cryoglobulinaemia will often present with palpable purpura on the legs and nephritis, but is an uncommon disease of older patients. If the man did not have a rash, then IgA nephropathy would the most probable cause of his urinary findings. A : 53. In a patient with hypertension, which one of the following features is consistent with a clinical diagnosis of Liddle's syndrome (a mutation affecting the sodium channels in the distal tubules)? A : raised catecholamine levels B : high aldosterone levels C : skin nodules D : high renin levels E : hypokalaemia. Comment : Liddle's syndrome is caused by a mutation in the sodium channel (ENaC) in the distal nephron. The mutation keeps the channel open, which has a similar effect to a raised aldosterone level. However, the aldosterone levels are not raised and may be lowered as a result of feedback from the sodium and water retention. There is increased sodium reabsorption and potassium loss. The condition is sometimes called pseudohyperaldosteronism. The hypertension is caused by sodium retention. This is a difficult question, but if you know that sodium handling is abnormal and there is hypertension, it is likely that this is because of sodium retention. In the distal nephron sodium reabsorption is mechanistically linked to potassium secretion, so increased sodium retention will lead to increased potassium secretion and hypokalaemia. E :

Page 113: Masterclass Book Part 2

MOHAMMED IS-HAG 112

54. A 30-year-old man with no antecedent illness presents with severely raised BP (150/110 mm Hg), frothy urine, peripheral oedema and lethargy. There is +++ blood and ++++ protein on urinalysis. 24-hour protein loss is 6.6 grams. Plasma albumin is 28 g/L. Plasma C3 is 0.10 (low). Plasma creatinine is 145 umol/l. Which of the following renal lesions is most likely to be found on renal biopsy? A : IgA nephropathy B : Lupus C : Focal sclerosing glomerulosclerosis D : Post infectious glomerulonephritis E : Mesangiocapillary glomerulonephritis. 55. Comment : This man has nephritic syndrome (hypertension, oedema, and proteinuria). Lupus is unusual in young men. FSGS is typically nephrotic syndrome, and commoner in older people. Post infectious Gn is possible but the infection should be clinically apparent, or in the history. MCGN presenting in a young man with nephrotic syndrome and hypertension and hypocomplementaemia would fit best. E : 56. A 31-year-old woman presents to her GP with marked leg oedema. Urine dipstick analysis shows 4+ Protein. Blood tests identify normal renal excretory function (creat. 68) and marked hypoalbuminaemia (albumin 19). A renal biopsy is reported as normal other than foot process fusion on the electron microscopy. Which of the following agents would be considered inappropriate as initial therapy? A : Cyclosporin B : Frusemide C : Simvastatin D : Warfarin E : Prednisolone. Comment : The diagnosis is minimal change glomerulonephritis, also known as minimal change nephropathy, minimal change disease, lipoid nephrosis and idiopathic nephrotic syndrome. Standard initial therapy is with corticosteroids, typically prednisolone at a dose of about 1mg/kg/day, which introduces remission in around 80 % of patients. Supportive therapies are also given, diuretics to clear oedema will clearly be appropriate in this case, and some nephrologists would prescribe warfarin to reduce risk of thromboembolism and a statin to reduce hypercholesterolaemia, although many would elect to see whether or not the patient went into a rapid remission that would render these agents unnecessary. Ciclosporin is useful as a steroid-sparing agent in patients who frequently relapse but would not be used as initial treatment. A : 57. A 53-year-old man presents having felt unwell for several weeks with general malaise and fatigue. He has had recurrent sinusitis for over a year and occasionally noticed a rash on his chest. Routine blood tests show that he has a serum creatinine of 180micromoles/l. Which of the following is the most likely diagnosis? A : Chronic renal failure B : Systemic vasculitis C : Minimal change nephropathy D : Urinary tract infection E : Focal segmental glomerulosclerosis. Comment : In any patient with a history of chronic progressive worsening of symptoms and renal impairment, it is important to consider systemic vasculitis as a possible cause. In this case, the rash and sinusitis are further pointers. Sinusitis can arise from involvement of the nasal tract and sinuses in the Wegener's pattern of the disease. Often a patient with systemic vasculitis will have a long history of indolent disease, but will then present late with severe aggressive disease. Useful immunological tests include assays for anti-neutrophil cytoplasmic antibodies (ANCA) and direct assays for antibodies against the ANCA antigens myeloperoxidase and proteinase 3. B : 58. In Hepatitis-C associated renal disease: A : patients can never safely receive a renal transplant once on dialysis B : membranous nephropathy is the commonest histological lesion C : cryoglobulinaemia and mesangiocapillary glomerulonephritis are typical D : treatment with interferon is ineffective E : immunosuppression is contra-indicated. Comment : Mesangiocapillary glomerulonephritis (GN) is commonest type of GN associated with Hepatitis C, though membranous GN can be seen.Interferon and other anti-viral agents can be very effective at reducing disease activity and cryocrit. Immunosuppression is usually given.The issue of patients on dialysis with Hepatitis C (commonest in Italy and Spain) who are being worked up for renal transplantation is important. Whether or not liver function tests are abnormal, these patients require estimation of viral load, viral genotyping, and a liver biopsy. If the disease is active, or the liver badly damaged, then pre-emptive anti-viral treatment is often employed before transplantation. Transplantation of a liver and a kidney can also be considered. C :

Page 114: Masterclass Book Part 2

MOHAMMED IS-HAG 113

59. A 40-year-old man with a 25-year history of diabetes treated with insulin is found to have 2g proteinuria per 24 hours, haemoglobin 10.5 g/dl, albumen 30g/l, calcium 2.0 mmol/l, phosphate 1.8 mmol/l, creatine 250 µmol/l, cholesterol 7 mmol/l. He has significant peripheral oedema. Blood pressure is 135/85 mm Hg. HbA1C is 9.5%. In order to preserve native renal function, which of the following do you consider most important? A : Strict restriction of dietary phosphate B : Commence a loop diuretic C : Commence an HMG CoA reductase inhibitor D : Commence an angiotensin converting enzyme inhibitor E : Greatly improved diabetic control aiming for HbA1c < 7.2 %. Comment : There is clear evidence that ACE inhibition in diabetics delays progression of renal failure even if they are normotensive. A loop diuretic will improve the peripheral oedema. Cholesterol lowering is appropriate given this patient's vascular risk factors, but there is little evidence that this alters the progression of renal disease. D : 60. A 28-year-old man presents with the nephrotic syndrome. Renal biopsy reveals membranous nephropathy. Which one of the following statements is true regarding this condition? A : prognosis for renal survival is universally poor B : the commonest cause of nephrotic syndrome in children C : immunostaining on the renal biopsy is usually negative D : most patients present with the nephrotic syndrome E : renal vein thrombosis is a very rare complication. Comment : Membranous nephropathy is the commonest cause of the nephrotic syndrome in adults, whereas most children have minimal change disease. It has a very variable outcome with spontanous remission occuring in up to 1/3 of patients. Renal failure can develop in about 20% of patients. All the complications of the nephrotic syndrome are seen including renal vein thrombosis. The renal biopsy shows a thickened glomerular basement membrane on light microscopy, granular IgG and C3 on immunostaining and subepithelial deposits on electron microscopy. D : 61. A 22-year-old woman presents with recurrent symptoms of urinary frequency and dysuria associated with cloudy urine. These settle rapidly with courses of antibiotics. Which one of the following would NOT be a recognised strategy for trying to prevent recurrence? A : Long-term prophylaxis with trimethoprim 100 mg at night B : Regular drinking of cranberry juice C : Practice of double micturition D : Advice to void the bladder before and after sexual activity E : Use a shower rather than a bath. Comment : Cranberry juice has antiseptic properties and has been shown in a controlled trial to be effective at reducing risk of urinary tract infection. The practice of double micturition endeavours to make sure that the bladder is completely empty after voiding, making it more difficult for infection to take hold. The woman should be advised to empty her bladder, wait for a further 60 seconds on the toilet and then try to void again. Some find that pressing on their suprapubic region will help them to express more urine. E : 62. A 53-year-old man is found to have a blood pressure of 185/95 at a routine medical check. His general practitioner checks his electrolytes and renal function, which are normal, and then starts him on an angiotensin-converting enzyme inhibitor. A week later, the general practitioner repeats these blood tests and arranges to see the patient again. Which one of the following is NOT a possible direct consequence of starting an angiotensin-converting enzyme (ACE) inhibitor ? A : A rise in plasma potassium B : A rise in plasma creatinine C : A rise in renal sodium reabsorption D : A fall in glomerular filtration rate E : A fall in aldosterone production. Comment : Angiotensin II has a number of actions on the kidney. It is a vasoconstrictor, but its effect on the efferent arterioles is greater than that on the afferent arterioles, resulting in an increase in glomerular filtration rate. It also promotes sodium reabsorption in the proximal tubule by an action on sodium/hydrogen exchange at this site. Angiotensin II stimulates aldosterone release from the adrenal gland. Aldosterone promotes distal tubular sodium reabsorption and potassium secretion. ACE inhibitors block the effects of angiotensin II by inhibiting its formation. Therefore, glomerular filtration rate often falls slightly and aldosterone levels fall. The fall in aldosterone reduces sodium reabsorption in the distal tubule and the fall in angiotensin II reduces sodium reabsorption in the proximal tubule. Conversely, the fall in aldosterone reduces distal tubular potassium secretion, so potassium levels can rise. C :

Page 115: Masterclass Book Part 2

MOHAMMED IS-HAG 114

63.

A 28-year-old woman was referred for the investigation of hypertension. Routine examination demonstrated that she had absent pulses in the left arm. She was a non-smoker. An arch aortogram was performed (see image). Which of the following statements is correct? A : She has coarctation of the aorta which is the likely cause of her hypertension. B : The selective picture shows a severe left subclavian stenosis most probably due to

atherosclerosis C : The study is normal and she most likely has essential hypertension D : The selective picture shows a severe left subclavian stenosis most probably due to Takayasu's

arteritis E : She has hypertension due to renovascular disease. Comment : Hypertension in the young always needs investigation. The physical findings in this woman were highly suggestive of a large vessel disease process. The selective angio picture does not show coarctation but does reveal critical stenosis of the left subclavian artery, which is most likely to be due to Takayasu's arteritis in this case. Atherosclerosis would be unusual in a young non-smoker.Hypertension is probably driven by renal ischaemia due to either direct renal vessel involvement or indirectly by aortic narrowing (not seen on these pictures). Raised erythrocyte sedimentation rate (ESR) is typical of the active inflammatory phase of this illness; in Japan there have been reports of linkage with tuberculosis - steroids and other immunosuppressants can be effective. D : 64. In Fibromuscular Dysplasia: A : A single lesion is typical B : Onset is rare in patients younger than 50 C : Males are more commonly affected than females D : Cure of hypertension is not unusual after angioplasty-stenting E : No other arteries are affected.

Page 116: Masterclass Book Part 2

MOHAMMED IS-HAG 115

Comment : Multiple lesions, typical in younger female patients. Cure is not guaranteed but can be seen frequently (as opposed to atherosclerotic disease). Iliacs, carotids and coronaries can be affected. D : 65. Which one of the following is not a recognized feature of renal disease due to myeloma? A : Renal amyloidosis B : Pre-renal acute renal failure C : Minimal change disease D : Cast nephropathy E : Nephrotic syndrome Comment : Myeloma can cause hypercalcaemia which acts on the tubules to cause sodium and water loss which can in turn result in pre-renal acute renal failure. AL amyloidosis can occur in the kidney and can cause nephrotic syndrome. Renal disease can also arise from the formation in the tubules of casts which contain immunoglobulin light chains. Alternatively, light chain deposition disease can occur with deposits of light chains visible on electron microscopy in glomerular and tubular structures. C : 66. Which one of the following statements about haemolytic uraemic syndrome is correct? A : It is usually associated with an abnormal clotting profile (PT and APTT). B : It can complicate treatment with cyclosporin. C : When associated with E Coli O157, it responds to antibiotic therapy. D : It responds to treatment with steroids. E : Immunosuppression is commonly used to treat this condition. Comment : Haemolytic uraemic syndrome (HUS) is characterized by microangiopathic haemolytic anaemia, normal clotting profile, and low platelets. The kidneys are often involved and presentation with acute renal failure is not uncommon. The most common form is seen following infection with E.Coli 0157. Immunosuppresive therapy plays no part in the treatment of this condition; however infusions of fresh frozen plasma (FFP) in combination with plasma exchange is routinely used. There is little evidence that antibiotics play a major role in the treatment of HUS associated with E.Coli 0157. Cyclosporin can cause HUS in renal transplant recipients, especially in those also receiving rapamycin which increases tissue concentrations of cyclosporin. B : 67.

The following ECG was obtained from a 46-year-old man. What is the diagnosis? A : Hypokalaemia. B : Hypercalcaemia. C : Hypothermia. D : Hypocalcaemia. E : Hyperkalaemia. Comment : This tracing shows the classic appearances of severe hyperkalaemia. There is peaking of the T waves, reduction in P wave voltage and broadening of the QRS complex. The rate is slowed and an ominous sine wave pattern is apparent which usually preceeds asystole.Urgent treatment is required. E :

Page 117: Masterclass Book Part 2

MOHAMMED IS-HAG 116

68. Haemodialysis (HD) patients: A : have better short-term (patient) survival than peritoneal dialysis (PD) patients B : typically exhibit first signs of beta-2-microglobulin amyloid in their knees C : have better blood pressure control than peritoneal dialysis patients D : with diabetes have > 50% 10 years survival E : have much better blood pressure control if their dialysis treatment times are extended to eight hours each session Comment : It is largely a futile exercise to claim that PD or HD is better. They are different. In many series, the short-term survival on PD can be as good or better than on HD. Diabetics do badly on all types of dialysis, with greatly increased death rates due to cardiovascular disease (myocardial infarction, LVF, sudden death). Beta-2-microglobulin amyloid eventually affects any patient on dialysis long enough, for example 10 years. The wrists (carpal tunnel syndrome), shoulders and hips are the typical places to have symptoms and to see bone cysts. Very prolonged treatment times on HD, for example 8 hours thrice weekly, or daily HD, are associated with greatly improved blood pressure control because a negative salt balance can be achieved and maintained. E : 69.

PLATE 23 The image shows the fundus of a patient who had a grand mal seizure 2 hours previously resulting in admission to the accident and emergency department (see image). The blood pressure was high following the seizure (200/130 mmHg). The creatinine was found to be 670 µmol/l and the urea 28 mmol/l. The kidneys are of normal size and not obstructed on ultrasonography. Select the most appropriate statement. A : The likely cause of the seizure and renal failure is an overdose of aspirin B : The seizure was most probably due to insulin-induced hypoglycaemia C : The blood film is likely to show evidence of microangiopathic haemolysis D : The renal failure is likely to be acute, and due to rhabdomyolysis E : The seizure was probably due to uraemic encephalopathy. Comment : The fundus shows papilloedema and haemorrhages. Taken together with the blood pressure the most likely cause of the renal failure is malignant phase hypertension, which is usually associated with microangiopathy. This degree of biochemical disturbance would be unlikely to give rise uraemic encephalopathy and seizures without another factor. Two hours following a seizure is too early for rhabdomyolysis to induce this degree of acute renal failure. C :

Page 118: Masterclass Book Part 2

MOHAMMED IS-HAG 117

70. You are talking to a 42-year-old man with end-stage renal failure and his wife regarding the possibility of renal transplantation. Which one of the following statements regarding renal transplantation is true? A : it is not ethical to allow donation except for donor-recipient pairs who are proven relations B : acute rejection rates of > 50% are typical C : the rate of lymphoma is increased compared to dialysis patients D : a rise in plasma creatinine in the first 2 weeks is invariably due to rejection E : delayed graft function beyond four weeks will always result in graft loss. Comment : Because of the increasing mismatch between cadaveric donors and needy recipients, many approaches have been devised towards expanding the donor pool. These include non-heart beating donation, and increasing the use of live-related and live-unrelated donors. In the situation of living donation, there is typically less good tissue-type matching than in cadaveric donation, but any disadvantage is heavily outweighed by the excellent quality of the donated kidneys (by design). With modern anti-rejection drugs (rapamycin, mycophenylate mofetil, tacrolimus), acute rejection rates are around 20%. Lymphoma is greatly increased, especially Epstein-Barr virus driven tumours. Early rises in plasma creatinine can be due to acute tubular necrosis (ATN), calcineurin toxicity, infection or rejection. C : 71. A 58-year-old man is reasonably fit and weighs 70 kg with known chronic renal failure due to reflux nephropathy. His plasma creatinine reading is 320 µmol/L; the value one year ago was 280 µmol/L. His potassium level is 5.6 mmol/L, urea 19.0 mmol/L, phosphate 1.8 mmol/L, calcium 2.25 mmol/L, haemoglobin 10.1 g/dL, ferritin 120 µmol/L. His appetite is good, but he feels tired. Blood pressure is 120/72 mmHg on an ACE inhibitor and he has 1.8g proteinuria per 24 hours. What treatment would you recommend? A : He needs to commence dialysis within the next few days. B : He should start oral alfacalcidol at 0.25 mcg per day in order to prevent renal bone disease. C : Treatment with recombinant human erythropoietin would probably be appropriate. D : This level of blood pressure is too low for a patient with chronic renal impairment and his ACE

inhibitor should be reduced or stopped. E : His dietary protein should be restricted to in order to delay progression of his renal failure. Comment : There is no immediate indication for dialysis, and a good appetite is very reassuring in this context. He is likely to have significant hyperparathyroidism, due to hyperphosphataemia and reduced active vitamin D. However, commencing alfacalcidol will further increase his phosphate level, which is undesirable. A more appropriate course is to ensure that dietary phosphate (and potassium) intake is restricted, and to prescribe a phosphate binder (such as calcium acetate).Blood pressure control, preferably with an ACE (angiotensin converting enzyme) inhibitor or ARB (angiotensin receptor blocker) is the most important measure in delaying progression of renal disease. A reading of 120/72 is satisfactory and not too low. Restriction of dietary protein intake has not proved to be an effective or practical strategy in humans to prevent progression of renal disease.This patient would almost certainly benefit from rhEPO to correct his anaemia. The ferritin above 100 µmol/L makes iron deficiency unlikely. Other causes of anaemia (eg B12 deficiency, haemolysis, myeloma) should be considered, but at this level of renal function he will have relative EPO deficiency. C : 72. Which of the following is true with end-stage renal disease treated by haemodialysis? A : indigestion is best treated with aluminium containing antacids B : dialysis restores normal life expectancy C : parathyroid homorne (PTH) levels are often high D : sexual function and libido are usually normal E : sodium intake is not important. Comment : Aluminium can accumulate in end-stage renal disease, even with dialysis and it has been associated with dialysis-related dementia. For this reason, it is usual to avoid aluminium containing compounds when possible. Sexual function and libido are often impaired - women are usually infertile and man can have lowered testosterone levels with end stage renal disease. Sodium intake is important and is usually restricted. A high sodium intake can make patients thirsty, but if they drink to satisfy the thirst then they will become hypervolaemic. This is because they have no way of excreting this fluid between dialysis sessions. transplantation is necessary to maintain life in patients with end stage renal disease, but morbidity and mortality still remain higher than for the rest of the population. The prognosis is better with transplantation than with dialysis. Parathyroid hormone levels tend to rise in end stage renal disease for several reasons, including high phosphate levels which may be associated with low calcium levels and a deficiency of vitamin D, which is normally synthesised by the kidney. C :

Page 119: Masterclass Book Part 2

MOHAMMED IS-HAG 118

73.

A patient on dialysis presents with a large swollen tender shoulder. Aspiration yields a milky slurry. Plain films show this appearance (see image). What is the most likely underlying cause of this problem? A : Poor control of calcium and phosphate B : Primary hyperparathyroidism C : Osteolytic metastasis D : Myeloma E : Septic arthritis. Comment : This is an example of tumoural calcinosis. Excess calcium and phosphate (which are at best only metastable in plasma) precipitate in either crystalline or amorphous forms in the skin, joints, blood vessels and other places. This ectopic calcification can cause local effects as here, including pain (though often these are painless) and deformity. Treatment is by engendering a negative calcium and phosphate balance, e.g. increased dialysis clearance, stopping positive calcium balance, and recourse to renal transplantation. A : 74. Which of the following conditions does not characteristically cause chronic interstitial nephritis when renal involvement is present? A : Chronic lead intoxication B : Primary Sjogren's syndrome C : Sarcoidosis D : Balkan nephropathy E : Scleroderma renal crisis. Comment : Renal involvement in scleroderma is a serious complication and the leading cause of death in this condition. Renal crisis is characterised by significant hypertension and rapidly deterioration renal function. The pathological findings are similar to those seen in accelerated hypertension often with an associated microangiopathy. All the other conditions are characterised by a chronic interstitial nephritis. Balkan nephropathy is a renal disease found in populations around the tributaries of the river Danube. The toxic agent is most likely a fungal toxin. E : 75. A 42-year-old woman presents with a plasma creatinine of 240micromoles/l and 3g of proteinuria per 24 hours. She weighs 200 kg. A renal biopsy is not possible because of her size. What is the most likely diagnosis? A : minimal change nephropathy B : focal segmental glomerulosclerosis C : membranous nephropathy D : Bartter's syndrome E : myeloma. Comment : There is a recognised association between severe obesity and focal segmental glomerulosclerosis. B :

Page 120: Masterclass Book Part 2

MOHAMMED IS-HAG 119

76. Which of the following is not a well-recognised cause of renal disease due to multiple myeloma? A : Ig A nephropathy B : Cryoglobulinaemia C : Amyloidosis D : Hypercalaemia E : Light chain deposition disease. Comment : Renal disease is a common feature of multiple myeloma and many different pathologies have been identified. The most common reason is usually seen in the context of dehydration, sepsis and hypercalcaemia. High calcium can lead to enhanced light chain toxicity and tubular damage as well as inducing renal vasoconstriction. Excess light chains are responsible for several different pathologies including classic myeloma kidney (Cast nephropathy), renal amyloidosis (usually lambda light chains) and light chain deposition disease (usually kappa light chains). Cryoglobulinaemia is uncommon but may cause a membranoproliferative glomerulonephritis. IgA nephropathy is unrelated and is characterised by mesangial deposition of non-clonal IgA which is abnormally glycosylated. A : 77. Which of the following statements concerning patients with Anderson–Fabry disease is untrue? A : Angiokeratomas particularly affect the face B : It is inherited in an X linked fashion C : Females can be affected D : The defect is in lysosomal alpha-galactosidase A E : Painful peripheral neuropathy is a common feature. Comment : The incidence is estimated at 1:40 000. The angiokeratomas typically affect the ‘bathing trunk’ area, rather than the face. Although regarded as X linked recessive, it is now clear that female carriers can be severely affected. A : 78. In acute interstitial nephritis: A : proteinuria is typically in the nephrotic range B : eosinophils in the renal interstitium suggests a drug-allergy C : steroid therapy is unhelpful D : complete recovery of renal function is unusual E : secondary glomerular pathology is frequently seen. Comment : Acute interstitial nephritis usually presents with renal impairment. Systemic manifestations include fever, arthralgia and skin rash. Some cases are idiopathic, but the commonest cause is allergic reaction to drugs, typically penicillins or NSAIDs, but many other agents have been incriminated, including frusemide, cimetidine and allopurinol. B :

Page 121: Masterclass Book Part 2

MOHAMMED IS-HAG 120

1. A 25-year-old woman attends A&E with a 48-hour history of unilateral headache of gradual onset associated with visual blurring and nausea. The headache is similar to her previous migraine, but examination reveals that her right pupil is larger than the left. Eye movements are normal and there is no diplopia. Which of the following statements is correct? A : If the left pupil fails to respond to light it is probably a Holmes–Adie pupil. B : Maintenance of the degree of inequality in different light intensities is suggestive of physiological

anisocoria. C : She may have a right Horner's syndrome. D : A left Argyll-Robertson pupil is a likely explanation. E : The features are strongly suggestive of 3rd cranial nerve palsy caused by posterior

communicating artery aneurysm. Comment : Minor discrepancies in pupillary diameters are common and usually represent physiological anisocoria, confirmed by the maintenance of the degree of inequality in different light intensities.A Holmes–Adie pupil is usually larger, and fails to react to light, with sluggish reaction to accommodation.Argyll–Robertson pupils are small, fail to react to light or accommodation, and may be unilateral as in this case; however since the cause of this condition is neurosyphilis, a condition now almost unheard of in the UK, this would be extremely unlikely in a woman of this age. When pupillary abnormalities are identified, always consider the possibility of a Horner's syndrome on the side of the smaller pupil - there will be ipsilateral partial ptosis and may be anhydrosis of the same side of the face. Third cranial nerve palsy due to compression by a posterior communicating artery aneurysm may cause dilation of the ipsilateral pupil; however it is uncommon for eye movements to be spared. B 2.

A 50-year-old heavy smoker is referred to the clinic with a six-month history of paraesthesiae in his fingertips, progressive weakening of grip strength in the upper limbs and general stiffness in his arms and legs. He is a manual labourer and often involved in heavy lifting. On examination, he has increased tone in the upper and lower limbs, power of 4/5 for finger flexion, finger extension and finger abduction and pathologically brisk reflexes with inverted C5/6 jerks and extensor plantar responses. An MRI of his cervical spine is shown (see image). What is the diagnosis? A : Cervical spondylosis B : Syringomyelia C : Epidural metastases from a primary neoplasm D : Transverse myelitis E : Acute disc herniation.

Page 122: Masterclass Book Part 2

MOHAMMED IS-HAG 121

Comment : The MRI scan shows multilevel degenerative change in the cervical region. There is severe cord compression with myelopathic change due to osteophytic indentations at the C3/4 and C4/5 levels. Cervical spondylotic myelopathy is the most frequent cause of myelopathy in those over the age of 50. It usually develops insidiously, but may be associated with sudden acceleration, especially following a slight head and neck injury. Clinical findings include bilateral upper motor neurone weakness of the legs (paraparesis, paraplegia) or legs and arms (quadriparesis, quadriplegia), bilateral impairment of sensation with a 'level' separating a region of normal sensation from a region of impaired sensation (either spinothalamic-mediated or posterior column-mediated sensory modalities may be impaired), and bowel or bladder sphincter dysfunction. Sphincter dysfunction is usually preceded by motor or sensory findings. Less commonly, neck pain and Lhermitte's sign (electric shock-like sensation down the spine with neck flexion) may be present.Cervical spondylotic myelopathy is more common in congenitally narrow spinal canals where disc protrusion, osteophytes, ligamentum flavum hypertrophy, posterior longitudinal ligament ossification and vertebral body subluxations may all play a role in compressing the cord. Decompressive surgery is usually considered to be an effective treatment in severe cases or rapid progression, but treatment is less clear-cut in mild and moderate cases without rapid progression. Some patients improve with conservative treatment such as intermittent cervical immobilization with a soft collar, use of anti-inflammatory medications and intermittent bed rest in patients with pain. Many variables play a role in decompressive surgery, including the risk of operating on an irrelevant stenotic process (e.g. in patients with motor neurone disease) and surgery-related morbidity and mortality. In slowly evolving cases, the best timing for surgery is also difficult to determine. In a recently reported two-year prospective randomized trial of patients with slowly progressive mild and moderate cervical spondylotic myelopathy, surgical treatment did not show better results than conservative treatment over the follow-up period. In general, best surgical and non-surgical treatment results are seen in cases with mild neurological impairment that is present for less than six months, and when the age of the patient is below 70. A 3. A 24-year-old female with known type 1 diabetes mellitus is admitted via the accident and emergency department in a semi-concious state (Glasgow Coma Score [GCS] = 10). The admitting SHO documents her to be agitated and tachypnoeic with a respiratory rate of 36 breaths/minute. The staff nurse reports that the blood sugar checked with a bed-side stick test is ‘high’. In these circumstances: A : intravenous fluids must be given with caution until a chest radiograph has been undertaken to

exclude pulmonary oedema B : a ventilation/perfusion (V/Q) scan should be arranged once the patient has been stabilized and

adequately resuscitated C : broad-spectrum antibiotics should be commenced as soon as blood cultures have been taken D : intravenous fluids must not be supplemented with potassium chloride until the result of repeat

electrolytes performed at 4 hours post-admission are available E : oral fluids should be with withheld. Comment : The picture here is one of moderate to severe diabetic ketoacidosis (DKA) – a condition that is still associated with significant morbidity and mortality. The tachypnoea almost certainly reflects the presence of a significant metabolic acidosis (with an attempt to compensate through ‘blowing off’ carbon dioxide - Kussmaul’s respiration), rather than primary lung pathology (although the latter may of course coexist and could have precipitated the acute episode). Patients with DKA are usually severely dehydrated (5-10L) and aggressive fluid resuscitation is a cornerstone of management – aim to give 5-6L within the first 24 hours, but bear in mind the clinical setting, for example a fit 24-year-old is likely to tolerate more aggressive fluid replacement than an elderly patient with a history of cardiac disease – consider central venous pressure (CVP) monitoring in the latter group. In either case, do not delay/withhold fluids whilst awaiting a chest radiograph (which may take several hours to obtain!). Unless there are specific clinical indications, a V/Q scan should not be routinely requested and broad-spectrum antibiotics should not be routinely given – remember that the white cell count may be raised in DKA even in the absence of infection/sepsis. Although potassium levels are often normal or even slightly elevated at the point of admission, life-threatening hypokalaemia may develop rapidly, especially in response to insulin (which ‘pushes’ potassium inside cells). Therefore, whilst it is reasonable (and usually advisable) not to add potassium to the first bag of fluid, thereafter supplementation should be instituted unless there are strong clinical indications to the contrary. In a semi-conscious patient the risk of aspiration is high (especially in the presence of delayed gastric emptying as is typically the case with DKA), and accordingly the patient should be made nil by mouth and a nasogastric tube sited. E

Page 123: Masterclass Book Part 2

MOHAMMED IS-HAG 122

4.

PLATE 24 The wife of this 63-year-old man complained that he was sleepy and increasingly aggressive. Which of the following is LEAST likely to be helpful in his management? A : Non-invasive ventilation B : Salbutamol nebules C : Loop diuretics D : Long-term oxygen therapy (LTOT) E : 'Flu vaccination. Comment : This man with severe kyphoscoliosis has probably developed chronic respiratory failure. All patients with significant respiratory disease should be offered annual 'flu vaccination. He should be assessed for non-invasive ventilation and long-term oxygen therapy and is likely to have cor pulmonale with pitting oedema, which will be improved symptomatically with careful use of diuretics. He might have reversible airways obstruction (as this is a common condition,) but this would be an additional problem. B 5. A 48-year-old man is found to have a blood pressure of 176/112 when he attends his general practitioner for a ‘new patient check-up’. He takes occasional anxiolytics for anxiety, but his past medical history is otherwise unremarkable. Physical examination is normal, excepting for obesity (BMI 32). A ‘routine’ biochemical screen is normal, excepting for potassium 3.3 mmol/l. The two most likely causes of his hypertension are: A : Renal hypertension B : Hypothyroidism C : Renovascular hypertension D : Cushing’s syndrome E : Primary hyperaldosteronism (Conn’s syndrome) F : Acromegaly G : Essential hypertension H : Isolated clinic (‘white coat’) hypertension I : Phaeochromocytoma J : Coarctation of the aorta. Comment : All of the conditions listed, excepting hypothyroidism, might explain hypertension, but all other than essential hypertension and ‘white coat’ hypertension are rare (together accounting for less than 5% of cases). Although a secondary cause of hypertension is very unlikely it would be important to look for clues in history and examination that might suggest renovascular disease (ischaemic heart disease, transient ischmaemic attack (TIA) / stroke, peripheral vascular disease), renal disease (previous nephritis, results of urine testing for e.g. insurance / employment medicals). Episodes of palpitations, sweating or headache may suggest phaeochromocytoma, but a less exotic cause such as anxiety would be a much more likely explanation. The serum potassium concentration is just below the lower limit of normal, but primary aldosteronism (Conn’s syndrome) remains exceedingly unlikely. In the case of an obese man it is also important to note that the blood pressure reading may be falsely elevated as a result of inadequate blood pressure cuff size, and it would be important to ensure that readings were taken with appropriate equipment. GH

Page 124: Masterclass Book Part 2

MOHAMMED IS-HAG 123

6. A 65-year-old male patient presents with a 3-month history of progressive difficulty in swallowing, and dysarthria. Examination reveals a weak, fasciculating, spastic tongue and a brisk gag reflex and jaw jerk. The likely diagnosis is: A : Basilar artery thrombosis B : Multiple sclerosis C : Miller–Fisher syndrome D : Amyotrophic lateral sclerosis E : Wernicke's encephalopathy Comment : Multiple sclerosis typically presents in the younger age group (<50 years of age) and produces upper motor neurone signs only (no wasting or fasciculation). Miller–Fisher is a variant of Guillain–Barré syndrome and presents with ataxia, areflexia and ophthalmoplegia. Basilar artery thrombosis is an acute event, often with obtundation, mixed cranial nerve dysfunction and long tract signs. Wernicke's encephalopathy presents with gaze palsies, and cognitive impairment. Amyotrophic lateral sclerosis is characterised by a combination of upper and lower motor neurone abnormalities. 20% of patients present with bulbar onset. D 7. You are called to the resuscitation room to see a 25-year-old man whose condition has suddenly deteriorated. He had arrived 30 minutes earlier with a 2-hour history of central pleuritic chest pain and breathlessness. He collapsed while awaiting radiograph and now is agitated and cyanosed with pulse 120/min and BP 80/40. Oxygen saturation is reading 79%, with the patient breathing high flow oxygen via a re-breathe mask. Respiratory examination reveals reduced breath sounds in the right lung field with deviation of the trachea towards the left. Percussion is resonant bilaterally. What immediate course of action should you take? A : Arrange for urgent portable chest radiograph B : Contact ITU to arrange for the patient to be ventilated C : Insert large bore needle into left hemithorax D : Insert large bore needle into right hemithorax E : Check arterial blood gases and commence Bipap if hypoxia is confirmed. Comment : Initially the history might suggest a number of diagnoses, including cardiac tamponade, massive pulmonary embolism, haemothorax or aortic dissection; however the respiratory examination findings indicate that he almost certainly has sustained a spontaneous pneumothorax, which has now developed into a tension pneumothorax. This is now a peri-arrest situation, and there is no time to arrange for portable chest radiograph, before attempting to reduce the pressure in the right hemithorax with the insertion of a large bore needle. If the diagnosis is correct, insertion may be accompanied by a loud ‘hiss’. Positive pressure ventilation is relatively contraindicated in this situation, and will probably not be required once the lung has re-inflated. D 8. A 65-year-old man presents with pain and weakness in his left arm. The following features are consistent with a C5/C6 radiculopathy (root problem) except: A : weakness of deltoid muscle B : reduced biceps reflex C : sensory loss over the ring and index finger D : weakness of supraspinatus muscle E : winging of the scapula. Comment : C5/C6 supplies the following muscles: deltoid, biceps, brachioradialis, supra and infra spinati, pectoralis major (clavicular head), serratus anterior (causes winging of scapula if weak), extensor carpi radialis longus. Both biceps and brachioradialis reflexes may be affected in a C5/C6 lesion Sensory loss can be variable but typically involves the lateral (radial) aspect of the upper and lower arm including the thumb. C8 typically supplies the ring and little finger. C 9. A 28-year-old man presents with the nephrotic syndrome. Renal biopsy reveals membranous nephropathy. Which one of the following statements is true regarding this condition? A : prognosis for renal survival is universally poor B : it is the commonest cause of nephrotic syndrome in children C : immunostaining on the renal biopsy is usually negative D : most patients present with the nephrotic syndrome E : renal vein thrombosis is a very rare complication. Comment : Membranous nephropathy is the commonest cause of the nephrotic syndrome in adults, whereas most children have minimal change disease. It has a very variable outcome with spontanous remission occuring in up to 1/3 of patients. Renal failure can develop in about 20% of patients. All the complications of the nephrotic syndrome are seen including renal vein thrombosis. The renal biopsy shows a thickened glomerular basement membrane on light microscopy, granular IgG and C3 on immunostaining and subepithelial deposits on electron microscopy. D

Page 125: Masterclass Book Part 2

MOHAMMED IS-HAG 124

10. You request an MRI brain scan on a patient with a neurodegenerative condition. Her husband asks for additional information on the safety of MRI scanning. Which of the following statements is NOT true? A : Modern metallic hip prostheses are MRI-compatible. B : Prosthetic heart valves are MRI-compatible. C : Earrings should be removed before brain scanning as they cause considerable image artefact. D : Cerebral aneurysm clips are an absolute contra-indication to MRI scanning. E : A maximum of three MRI scans should be performed on any one patient per year. Comment : Absolute contra-indications to MRI include: · aneurysm clips · cardiac pacemakers which can not be safely switched off · the presence of shrapnel or an intraocular metallic foreign object. All modern metallic joint prostheses, and prosthetic heart valves (including metallic) are MRI-compatible. Jewellery is MRI-compatible but causes significant image artefact if close to the area being scanned. MRI does not use ionising radiation and numerous scans are therefore safe. E 11. A 48-year-old with recurrent carcinoma of the colon admits to poor sleeping, anhedonia and feelings of hopelessness. Which of the following would you do? A : Reassure him that this is a normal response to his predicament B : Start anti-anxiolytic drugs C : Suggest counselling D : Start an anti-depressant E : Suggest relaxation. Comment : Depression is the most likely diagnosis. This needs to be treated along conventional lines. Other modalities may be of additional benefit. D 12. A 28-year-old man with asthma presents with an acute attack. He is very breathless and cannot complete sentences. Which of the following is the best immediate management? A : Nebulised salbutamol (5 mg) driven with air B : Organise chest radiograph to exclude pneumothorax C : Nebulised salbutamol (5 mg) driven with high flow oxygen via reservoir bag D : Nebulised salbutamol (50 mg) driven with 35% oxygen E : Nebulised salbutamol (5 mg) driven with 35% oxygen. Comment : Beta2-agonists are pulmonary vasodilators as well as bronchodilators. Their administration can rapidly worsen the V/Q mismatch which is the cause of hypoxia in asthma. They can therefore cause reduction in arterial oxygen tension unless supplemental oxygen is given, and this man should be given the highest inspired oxygen concentration that can be obtained.It would also be reasonable to mix ipatropium bromide (500 microg) with the salbutamol given in option C. C 13. A previously asymptomatic 75-year-old lady with myelodysplastic syndrome (subtype chronic myelomonocytic leukaemia) presents with easy bruising and unprovoked soft tissue haematomas. The following results are obtained at presentation: Hb 10.6g/dl; Wbc 29.9 x 10^9/l; plts 67 x 10^9/l; PT ratio 0.9; APTT ratio 1.1; fibrinogen 0.8g/l D-dimers 2467ng/ml ( 0-300). Which of the following is NOT a possibility? A : Her disease has transformed. B : She has developed a partially compensated consumptive coagulopathy. C : A 'wait and see' approach should continue to be the mainstay of treatment. D : Treatment of her underlying condition may alleviate the coagulopathy. E : Replacement therapy with cryoprecipitate, fresh frozen plasma and platelets should be instituted. Comment : The 'high risk' myelodysplastic syndromes have a tendency to transform into acute myeloid leukaemia. The lady has a compensated consumptive coagulopathy shown by a low fibrinogen, normal activated partial prothrombin time (APTT) and a markedly elevated D-dimer. She has become overtly symptomatic and this requires treatment as the risk of intracranial haemorrhage is a real one. Treatment of her underlying disease may alleviate her coagulopathy by eradicating the malignant clone secreting this procoagulant activity. Replacement therapy with cryoprecipitate, fresh frozen plasma and platelets should be instituted. C 14. A patient presents with sensorineural deafness. Which of the following is not associated with sensorineural hearing loss? A : Basilar meningitis B : Benign positional vertigo C : Meniere’s disease D : Otosclerosis E : Paget’s disease. Comment : Benign positional vertigo is not associated with hearing loss. Although otosclerosis commonly causes conductive hearing loss, sensorineural deafness is also seen. B

Page 126: Masterclass Book Part 2

MOHAMMED IS-HAG 125

15. A 76-year-old woman is admitted with sudden onset left sided weakness. CT brain scan shows a right parietal infarct. Nursing staff notice she coughs and appears to choke when drinking fluids. Which of the following statements is correct? A : A gastroscopy should be organized as soon as possible. B : Swallowing difficulties are likely to persist long term. C : The speech and language therapist may recommmend fluid thickeners. D : Nasogastric (NG) feeding will protect against aspiration. E : Percutaneous endoscopic gastrostomy (PEG) feeding should be considered if dysphagia persists

for greater than one month. Comment : Dysphagia associated with stroke is generally due to abnormalities of the oropharyngeal phase of swallowing. Unlike patients with dysphagia due to oesophageal tumours or strictures, patients with dysphagia due to neurological disorders find fluids more difficult to swallow than solids. Bedside assessment and video fluoroscopy are the basis of evaluation.Swallowing difficulties associated with stroke are usually transient and recovery occurs within two weeks in the majority of cases. Adequate intake can often be safely maintained using fluid thickeners to increase the viscocity of ingested fluids. Although there is no firm evidence as yet to support a particular approach, most physicians would consider PEG feeding if there were no signs of recovery after one to two weeks. C 16. A 25-year-old man presents to A&E with severe headache. This headache is unusual and he is investigated with a CT of his brain (see image). Which of the following statements is true? A : The patient has a subdural haematoma. B : The patient has an extradural haematoma. C : The patient has meningitis. D : The patient has a subarachnoid haemorrhage (SAH). E : The patient has brain tumour. The scan show hyperdensity throughout the subarachnoid space which is most consistent with a SAH. D 17.

PLATE 25 A patient who has a scan as shown is most likely to have which one of the following signs or symptoms? A : Pain on the outer aspect of the feet. B : Pain radiating down both thighs C : Altered sensation in the pelvic area D : Loss of the right ankle jerk E : Loss of anal sensation. Comment : Outer aspect of the feet is S1 dermatome. The ankle jerk is transmitted through S1. Anal sensation is S3 dermatome. B

Page 127: Masterclass Book Part 2

MOHAMMED IS-HAG 126

18. The daughter of a 62-year-old man takes him to A&E where you are asked to see him to give a medical opinion. Early that morning he developed a clumsy right hand and difficulty speaking, the problem with his hand having now persisted for 12 hours. He is known to have hypertension, asthma and rheumatoid arthritis. In addition he suffered from migraines as a young man. He smokes a pipe. On examination, he has some weakness of the intrinsic hand muscles on the right only. Visual fields, speech and sensation are normal. Blood pressure is 190/90 mmHg. Which of the following is the most likely diagnosis? A : Left hemisphere lacunar stroke B : Migraine equivalent C : Left middle cerebral artery territory cardioembolic stroke D : Left pontine microhaemorrhage E : Neck-tongue syndrome. Comment : The history suggests the dysarthria-clumsy hand syndrome, one of the classic lacunar syndromes that are strokes in the subcortical regions (or brain stem) secondary to small vessel disease. The usual site of damage in the dysarthria-clumsy hand syndrome is the internal capsule or pons, infarction being more common than haemorrhage although both are caused by disease of small perforating arterioles. Contributory risk factors include smoking, hypertension and hypercholesterolaemia.Migraine equivalents (aura-like symptoms without headache) can provide diagnostic difficulties and should always be considered in those with a previous history of migraine, but there is no suggestion of the characteristic slow spread of symptoms in this patient.The neck tongue syndrome consists of pain and parasthesias in one half of the tongue precipitated by neck movement, often associated with occipital pain and ipsilateral hand parasthesias. A 19. An 84-year-old man presents with a 6-month history of increasing confusion, visual hallucinations, reduced mobility and falls. Which type of dementia fits this history best? A : Alzheimer's disease B : Pick's disease C : Dementia with Lewy bodies D : Parkinson's disease E : Vascular dementia Comment : There is some overlap between the different types of dementia, but in this case there are clues that this is dementia with Lewy bodies (DLB) with the early development of instability, falls and hallucinations. Other features include fluctuating cognition, depression and delusions.Treatment is usually symptomatic, but remember that neuroleptic drugs such as haloperidol will worsen Parkinsonian features, so consider using atypical antipsychotic agents such as quetiapine. There is some evidence that the dementia may respond to anticholinesterase inhibitors.Dementia associated with Parkinson's disease tends to occur much later in the course of the disease. C 20. A 77-year-old man presents with sudden onset of weakness of his right arm on a background of a 3-week history of thoracolumbar backpain, weight loss, fever and night sweats. Blood tests: full blood count (FBC) 9.8 g/dL, white cell count (WCC) 12.0, platelets 450, erythrocyte sedimentation rate (ESR) 110 mm/hr, creatinine 180 micromol/L and microscopic haematuria. What is the most likely diagnosis? A : Spinal osteomyelitis B : Myeloma C : Infective endocarditis D : Mycobacterium tuberculosis infection E : Tertiary syphilis Comment : Infective endocarditis is the only diagnosis that would explain all the symptoms and results. Vertebral osteomyelitis/discitis and stroke are both recognised complications of infective endocarditis and either can be the presenting feature of the disease. C 21. You see a 70-year-old lady who has severe Parkinson’s disease and is on co-careldopa and apomorphine. She complains of nausea and vomiting due to her medication. Which one of the following drugs would be prescribe for these symptoms? A : Domperidone B : Metoclopramide C : Prochlorperazine D : Entacapone E : Betahistine. Comment : Drugs such as apomorphine and bromocriptine cause vomiting through peripheral stimulation of the chemoreceptor trigger zone. Worsening of Parkinson’s disease may result from the use of dopamine antagonists; however, domperidone is much less likely to cross the blood–brain barrier and is therefore the preferred agent in this case. Entacapone is a catechol-O-methyltransferase (COMT) inhibitor which increases levodopa levels, thus worsening nausea and vomiting. Betahistine is used in vertigo. A

Page 128: Masterclass Book Part 2

MOHAMMED IS-HAG 127

22. A 28-year-old male presents following an overdose and anticholinergic syndrome is suspected. Which one of the following is true? A : Tricyclic antidepressants are not a cause B : Bradycardia is common C : Physostigmine is the treatment of choice D : Mydriasis occurs E : Urinary incontinence is common. Comment : Anticholinergic syndrome occurs following overdose with drugs that have prominent anticholinergic activity including tricyclic antidepressants, antihistamines and atropine. Features include dry, warm, flushed skin, urinary retention, tachycardia, mydriasis (dilated pupils) and agitation. Although physostigmine, a reversible inhibitor of acteylcholinesterase, is effective in treating symptoms, there is a significant risk of cardiac toxicity (bradycardia, AV conduction defects and asystole). Treatment therefore consists of withdrawal of the precipitating drug and supportive care. D 23. A 56-year-old woman presents with progressive leg weakness over 3 days. She has noted minor urinary incontinence in addition. She has a past medical history of breast cancer treated 10 year ago with lumpectomy and radiotherapy. Which is the investigation of choice? A : Myelography B : Plain spinal radiographs C : Lumbar puncture D : Computed tomography (CT) E : Magnetic resonance imaging (MRI) Comment : She is likely to have spinal cord compression from metastases. Plain radiographs and CT may give some information but the imaging modality of choice is MRI. Lumbar puncture and myelography carry the risk of clinical deterioration. E 24. A 16-year-old girl presents with a history of episodes of bizarre behaviour following her parents' recent divorce. Her mother describes these attacks to you. Which two of the following features would suggest a true epileptic seizure rather than a non-epileptic attack? A : Limb flailing B : Opisthotonus C : Carpet burns D : Tongue biting E : Pelvic thrusting F : Duration of several hours G : Cyanosis H : Rapid recovery I : Directed violent behaviour J : Repetitive chewing movements Comment : Non-epileptic attacks can be distinguished fairly reliably on clinical grounds; however difficulty often arises with frontal lobe seizures which can be bizarre and occur without EEG change. Cyanosis and repetitive chewing movements are more suggestive of epileptic seizures. Recovery usually occurs over a few minutes rather than immediately. Carpet burns, opisthotonus and pelvic thrusting are classically associated with non-epileptic attacks. Tongue-biting is seen with both types of attack and is non-discriminotory. G . J 25. A 43-year-old man is referred after an episode of limb paralysis occurring suddenly at nighttime. He describes waking shortly after falling asleep and being unable to move his limbs or to shout out for help. In addition he describes feeling as if he could not breathe properly. Symptoms lasted for about a minute. His father recently died from a stroke during sleep. He has no other symptoms apart from daytime sleepiness, which he puts down to working long hours. Blood pressure is 140/90 mmHg. There are no other abnormalities on examination. What is the most likely diagnosis? A : Cervical disc prolapse B : Depression and anxiety C : Brainstem transient ischaemic attack D : Nocturnal seizures E : Narcolepsy. Comment : This man is describing sleep paralysis. Sleep paralysis may be isolated, or occur in the context of narcolepsy, in which it affects 15-45% of patients. The symptoms represent the atonia of REM sleep. Awareness is preserved during the attack. Although the respiratory muscles are only ever mildly affected in comparison to the limbs, patients may describe a feeling of suffocation that can be particularly frightening. E

Page 129: Masterclass Book Part 2

MOHAMMED IS-HAG 128

26. A 29-year-old man presents with a 9-month history of depression, 4-month history of painful sensory disturbance in both legs and most recently cognitive impairment with myoclonus. His MRI scan reveals thalamic hyperintensity on T2-weighted images. His EEG is normal. The most likely diagnosis is: A : corticobasal degeneration B : progressive multifocal leucoencephalopathy C : juvenile myoclonic epilepsy D : new variant Creutzfeldt-Jakob disease E : Wilson's disease. Comment : New variant CJD commonly presents in young adult with psychiatric symptoms, followed by non-specific painful sensory symptoms, most often in the lower limbs. Cognitive impairment, pyramidal signs, myoclonus and primitive reflexes then develop. Mean disease duration is approximately 14 months. MRI commonly shows high signal on T2-weighted images in the pulvinar (posterior aspect of thalamus). EEG is often normal, compared to sporadic CJD in which triphasic waves are observed. Diagnosis is made ante-mortem by brain biopsy or more commonly tonsillar biopsy. CSF contains 14-3-3 protein. A clear link with BSE has now been established. D 27.

PLATE 26 You see a patient in clinic with the abnormality marked ‘x’ in the image. A lesion affecting which of the following nerve roots could account for the patient’s condition? A : C4 B : C5 C : C6 D : C7 E : C8. E : 28. A 30-year-old After 12 hours he has weakness of his arms but remains afebrile and is not confused. Over the next 12 hours he develops respiratory failure and requires artificial ventilation. What is the likely diagnosis? A : Tetanus B : Diphtheria C : Botulism D : Strychnine poisoning E : Rabies. Comment : Tetanus and strychnine poisoning both produce muscle spasm which may lead to respiratory failure but not muscle weakness.Rabies produces a uniformly fatal encephalitis characterized by pharyngeal spasm triggered by water. Diphtheria presents with a pharyngitis and a membrane over the tonsils. The toxin may cause myocarditis and neurotoxicity with palatal paralysis and cranial nerve palsies may occur. Botulism typically produces a descending paralysis which starts with diplopia or blurred vision due to difficulty with accommodation and progresses to weakness of the neck, arms and respiratory muscles. C

Page 130: Masterclass Book Part 2

MOHAMMED IS-HAG 129

29. A 40-year-old patient presents with progressive leg weakness. Which features would be against a diagnosis of Guillain–Barré syndrome? A : Severe back pain B : Recent chest infection C : Urinary incontinence at the beginning of the illness D : Fluctuating blood pressure E : Sluggish pupillary reactions F : Normal arm and leg relexes G : Marked fatiguability of leg movement H : Subjective sensory disturbance I : Dysarthria J : Shortness of breath on exertion. Comment : Guillain–Barré classically presents with an ascending paralysis beginning in the legs. Sensory involvement is variable, but usually less than motor features. Reflexes are lost early in the illness. Demyelinating neuropathies characteristically cause fatigue with repeated use. The cranial nerves are commonly affected, most commonly the facial nerves. The autonomic nerves can be involved leading to pupillary abnormalities and haemodynamic instability. The bladder and bowel can be affected, but early involvement would be more suggestive of a conus lesion. Guillain–Barré is very painful, and can present with back pain. C.F 30. A 30-year-old woman presents with a 3-day history of left-sided retro-orbital pain and progressive visual loss. She has been previously fit and well. The following are recognised features of optic neuritis, except: A : central scotoma B : relative afferent pupillary defect C : colour desaturation D : Ptosis E : sheathed retinal vasculature F : peripheral visual field loss G : pupil unreactive to light and accommodation H : swollen optic disc on fundoscopy I : normal optic disc on fundoscopy J : delayed visual evoked potential. Comment : Optic neuritis presents with rapid, progressive visual loss, often associated with orbital or retro-orbital pain. Examination reveals reduced visual acuity, impaired colour vision, a central scotoma, relative afferent pupillary defect and retinal vein sheathing. It may be associated with either a normal appearing optic disc (retrobulbar neuritis) or a swollen optic disk in typical optic neuritis. Visual evoked potentials are commonly delayed. There are a number of causes including: sarcoidosis, Wegener's granulomatosis, tuberculosis, syphilis, meningitis or most commonly idiopathic (either in isolation or in association with multiple sclerosis). D.F 31. A 60-year-old alcoholic presents with a 4-week history of increasing lethargy, headache and confusion. On examination, he was confused, had signs of mild neck stiffness and photophobia. Cerebrospinal fluid analysis revealed the following results: Opening pressure: 35 cm H2O White cells: 250 per mm3 (80% lymphocytes, 20% polymorphs) Protein: 1.5 g/l Glucose: 2.0 mmol/l (serum glucose 8.0 mmol/l) Gram stain: Negative Which of the following are the two most likely diagnoses? A : Cryptococcal meningitis B : Herpes simplex encephalitis C : Herpes simplex meningitis D : Japanese B encephalitis E : Listeria monocytogenes meningitis F : Meningococcal meningitis G : Mumps meningitis H : Ruptured mycotic aneurysm I : Subarachnoid haemorrhage J : Tuberculous meningitis. Comment : The clinical features of insidious onset, meningeal irritation and cerebrospinal fluid (CSF) findings of a lymphocytic pleocytosis, high protein and low glucose make fungal and tuberculous difficult to distinguish. Therefore Ziehl-Neelsen stain, polymerase chain reaction for Mycobacterium tuberculosis, cryptococcal antigen and India ink stain should be carried out on the CSF. A chest radiograph to look for pulmonary tuberculosis will also be helpful. A.J

Page 131: Masterclass Book Part 2

MOHAMMED IS-HAG 130

32. You are referred a 60-year-old man complaining of increasing difficulty with speech. Which of the following two features would make you characterize it as a posterior (receptive) type of dysphasia? A : Agrammatic speech B : Defective voice production C : Fluent speech D : Impaired articulation E : Impaired naming of objects F : Non-fluent speech G : Poor comprehension H : Poor writing I : Preserved repetition J : Silent reading unaffected. Comment : Defective voice production results in dysphonia. Impaired articulation results in dysarthria. Anterior dysphasia is characterized by non-fluent hesitant speech with an agrammatic or telepathic element, the substitution of words or syllables, poor writing with errors similar to speech, impaired naming of things and preserved comprehension, repetition and reading. Posterior dysphasia is characterized by fluent speech with normal rhythm, neologisms, paraphrasias, substitutions and poor comprehension, repetition and reading. C.G 33. A 77-year-old woman presents with her husband who describes her dementing illness. You diagnose dementia with Lewy bodies. Which two features are the most helpful in making this diagnosis? A : Loss of short-term memory B : Markedly fluctuating conscious level C : Hallucinations D : Stepwise deterioration E : Brisk reflexes F : Early impairment of language G : A tendency to fall on stairs H : Weight loss I : Nocturnal wandering J : Concomitant diabetes. Comment : Dementia with Lewy bodies is probably the second commonest dementia after Alzheimer's disease. The patients show features of dementia, with Parkinsonism, psychiatric symptoms and sometimes striking variation in performance or even conscious level. All dementias tend to produce loss of short-term memory and nocturnal wandering, and may be associated with weight loss if the patient is supposedly self-caring. Stepwise deterioration, brisk reflexes and diabetes may be associated with vascular dementia. Early impairment of language is most marked in Primary Progressive Aphasia. Patients with progressive supranuclear palsy (PSP) are particularly prone to fall down stairs because of their impaired eye movements. B . C 34. A 22-year-old man presents with a four-day history of worsening diplopia. In the affected eye, which two of the following features would support the diagnosis of a third nerve palsy? A : Eye is directed downwards and medially at rest B : Loss of consensual light reflex when light is shone into affected eye C : Inward rotation of the eye on attempted down-gaze D : Pain in the eye E : Eye is directed upwards and laterally at rest F : Dilated pupil G : Eye is directed upwards and medially at rest H : Nystagmus on attempted lateral gaze I : Constricted pupil J : Lack of ptosis. Comment : In a third nerve palsy there is ptosis, the eye points ‘down and out’ and the pupil is dilated. On attempted downgaze the eye rotates inwards (best seen by looking at a conjunctival vessel) due to action of the superior oblique muscle (innervated by the IVth cranial nerve) on an eye that is abducted. A fourth nerve palsy inactivates the superior oblique muscle, causing double vision with vertical separation of images that is most marked when the patient attempts to look down, e.g. when reading or walking down stairs. A sixth nerve palsy inactivates the lateral rectus muscle, causing double vision with horizontal separation of images that is most apparent when the patient attempts to look to the affected side. C.F

Page 132: Masterclass Book Part 2

MOHAMMED IS-HAG 131

35. A 40-year-old man presents with a sensory peripheral neuropathy. Which two of the following are unlikely to be the cause of this man’s symptoms? A : Alcohol B : Amyloid C : Human immunodeficiency syndrome D : Lead E : Leprosy F : Paraneoplastic G : Paraproteinaemic H : Porphyria I : Sjogren’s syndrome J : Vitamin B12 deficiency. Comment : Lead toxicity and porphyria typically cause motor peripheral neuropathy. D.H 36. A 48 yr old man is referred because he has a tremor. Which one of the following features would NOT support the diagnosis of essential tremor? A : It is an action tremor. B : It is ameliorated by alcohol. C : It is associated with a positive family history. D : It has a higher frequency (Hz) than Parkinsonian tremor. E : It is helped by anticholinergics. Comment : Essential tremor (ET) is a common movement disorder of unknown pathogenesis. Its prevalence has been quoted to range from one to 22%, probably due to variations in definitions of ET. It is more common than Parkinson's disease (PD). It has a bimodal distribution, with peaks in the second and sixth decades of life. A family history has been reported in 17 to 100% of cases, the reason for the discrepancy being that unless all symptomatic and asymptomatic members of a family are examined, the number of affected relatives will be underestimated. ET has been mapped to several gene loci and more are expected to be identified in the future. Recently, core criteria have been suggested for the diagnosis of ET: (1) bilateral action tremor of the hands and forearms, (2) absence of other neurological signs, except the cogwheel phenomenon, and (3) may have isolated head tremor with no signs of dystonia. Secondary criteria include a long disease duration (more than three years), a positive family history and beneficial response to alcohol. Features which are against the diagnosis of ET, although not exclusively so, include unilateral, focal or leg tremor, gait disturbance, rest tremor, rigidity, bradykinesia, sudden or rapid onset of the tremor, current drug treatment causing or exacerbating the tremor and isolated head tremor with abnormal posture (suggesting dystonia). However, there are suggestions that dystonia, parkinsonism and deafness are more common in ET. The frequency of the tremor is 4-10 Hz (compared to 4-6 Hz in PD). Propranolol and primidone are effective in the treatment of ET. If one drug does not control the tremor, a combination of both drugs may be beneficial. In severe cases refractory to medical treatment, surgical options including thalamic stimulation and thalamotomy may be needed. E 37. A 54-year-old woman presents with asymmetric hand muscle bulk. The associated clinical signs that would make you suspect a T1 root lesion include: A : combined wasting of 1st dorsal interosseus and abductor pollicis brevis muscles B : sensory loss over the whole palm C : diminished finger reflexes D : loss of pain sensation over the shoulder and across the upper chest and back E : muscle wasting in shoulder girdle F : ipsilateral Horners syndrome G : brisk triceps reflexes H : inverted biceps reflex I : contralateral Horners Syndrome J : extensor plantars. Comment : A T1 root lesion will lead to wasting of all intrinsic hand muscles, diminished sensation along the medial aspect of the arm, and will be associated with a Horner’s syndrome. In all such patients a chest radiograph is mandatory, whether they are smokers or not, although this would heighten your suspicion that an apical lung tumour has caused the root compression. Sensory loss over the whole palm in this situation would suggest combined ulnar and median nerve involvement, and the dissociated -cape distribution sensory loss is seen in intrinsic cord lesions at around T1, e.g. syringomyelia. The muscle wasting in this diagnosis is caused by compression of the anterior horn cells by the syrinx. Finger reflexes are a C8 reflex, but are most useful when brisk, rather than absent, which is difficult to distinguish from normal. An inverted biceps reflex is a feature of a cord lesion at C5,6. A.F

Page 133: Masterclass Book Part 2

MOHAMMED IS-HAG 132

38. A 60-year-old man is found to have a parietal lobe tumour. Which of the following would you expect to find in this patient? A : Ataxia B : Bitemporal hemianopia C : Contralateral motor weakness D : Contralateral sensory disturbance E : Dysphasia F : Memory disturbance G : Neglect H : Nystagmus I : Personality change J : Spastic paraparesis. Comment : Common presentations of a parietal lobe tumour include contralateral sensory or cortical sensory loss, neglect, apraxia and contralateral homonymous field defect (that sometimes consists solely of lower quadrantanopia). D.G 39. You are asked to see a 36-year-old woman with headaches. Which two of the following features would suggest a diagnosis of tension-type headache? A : Aggravated by physical activity B : Associated with ptosis C : Attacks commonly occur at a frequency of 20-40 times a day D : Aura prior to headache E : Bifronto-temporal pain F : Commonly associated with vomiting G : Headache relieved by alcohol H : Lancinating pain I : Regular aspirin is beneficial for pain relief J : Throbbing pulsatile pain. Comment :Tension-type headache is characterized by a constant, non-pulsatile, band-like, bi-fronto-temporal pressure on top of the head. The headache is not worsened by physical activity. Associated mild nausea, but not vomiting, is common. Chronic forms are worsened by anxiety and stress. Alcohol may relieve the headache. Regular simple analgesia may be the commonest cause of chronic daily headache. There are no abnormal features on examination. Amitryptyline is the drug treatment of choice. E .G 40. A 72-year-old man is referred with suspected Parkinson’s disease. He complains of shaking of his hands which is worse when he tries to do things such as drinking a cup of tea, which has got progressively worse over the last 5 years. He comments that his father had a similar condition. He is otherwise well and has a normal posture and gait. You diagnose benign essential tremor. Which two statements about this condition are correct? A : The tremor is an intention tremor. B : Drugs such as sodium valproate and lithium may produce a similar tremor. C : In young people, Wilson's disease, indicated by a high serum caeruloplasmin, should be

excluded. D : Many cases are familial with autosomal recessive inheritance. E : This is a clinical diagnosis and no baseline investigation is required. F : Medication should always be offered since it may halt progression of the condition. G : Primidone is well tolerated initially but has a high rate of long-term complications. H : Gabapentin, a GABA-antagonist, is a useful second-line agent. I : Over 90% of patients will have marked improvement with pharmacotherapy therapy. J : Thalamic stimulation may be recommended in those with disabling tremor unresponsive to

medication. Comment : Benign (a misnomer since it can be disabling ) essential tremor (BET) predominantly affects those over the age of 70. The tremor is pathological, unlike the physiological tremor we all have which may become noticeable in stressful situations. It is an ACTION tremor which occurs during voluntary motion and when the arms are outstretched. An intention tremor is coarse and occurs as a limb approaches a target. It is a sign of cerebellar disease. It is important to take a drug history since several medications may enhance physiological tremor and lead to confusion. Wilson’s disease is rare. There is a rest and action tremor. There are usually other neurological abnormalities. The serum caeruloplasmin is LOW. The familial form (30-50% cases) of BET has an autosomal DOMINANT form of inheritance. The diagnosis is indeed a clinical one and there are no validated laboratory markers, but at least thyroid function tests should be done to exclude hyperthyroidism which may present with a sustained enhanced physiological tremor. Medication should only be used for patients who are disabled and /or embarrassed by their tremor. Primidone is often effective, but there is a high incidence of nausea and vomiting when initiating treatment, requiring discontinuation in up to one fifth of patients.

Page 134: Masterclass Book Part 2

MOHAMMED IS-HAG 133

Propranol has similar efficacy, but although better tolerated initially may be less well tolerated long term. Gabapentin is structurally similar to GABA (i.e. not an inhibitor ). There is some evidence of disturbance of the GABA – ergic system in essential tremor and it can be an effective alternative treatment. Unfortunately a large proportion of patients do not respond to pharmacotherapy. In such patients, if warranted, thalamotomy or deep brain stimulation by an electrode placed in the thalamus may be tried. B.J 41. A GP refers to you a 30-year-old woman with a 3-month history of diplopia and fatiguable weakness. Which of the following are not features of myasthenia gravis: A : difficulty swallowing B : ptosis C : nasal speech D : worsening of symptoms with angiotensin-converting enzyme (ACE) inhibitors E : positive Tensilon test in 90% of patients F : abnormal single-fibre electromyography G : pupillary abnormalities H : worsening of symptoms on initiating high-dose steroids I : thymoma J : worsening of symptoms with beta-blockers. Comment : Myasthenia gravis (MG) is caused by an antibody-mediated autoimmune attack against acetylcholine receptor receptors at neuromuscular junctions. Antiacetylcholine receptor antibodies are present in approximately 80% of patients with generalised MG and approximately 50% of patients with ocular MG. Seventy-five percent of patients have thymic abnormalities, either hyperplasia or less commonly, thymoma. Patients usually present with ptosis, diplopia, bulbar symptoms or limb weakness, but not fatigue per se. Exacerbations are seen with intercurrent infections, pregnancy, and certain drugs, such as aminoglycosides, beta-blockers, calcium antagonists etc. Steroids are an effective treatment but should be started at low doses and increased slowly to avoid worsening of symptoms. The Tensilon test is positive in 90% of patients but has been largely superseded by single-fibre EMG and testing for the presence of antiacetylcholine receptor antibodies. Tendon reflexes are normal, and there are no pupillary abnormalities. D.G 42. A 66-year-old man presents with a twelve-month history of dysarthria and dysphagia. Which two of the following statements are true? A : A tensilon test is unlikely to be helpful. B : The most likely diagnosis is a brain stem cerebrovascular accident (CVA). C : Normal muscle strength in the arms excludes motor neurone disease (MND). D : Fasiculations in the tongue strongly suggest myaesthenia gravis. E : Assessment of respiratory function is important. F : NCS and EMG could be diagnostic. G : MRI of the cervical spine is mandatory. H : LP is mandatory. I : The diagnosis could be Guillian Barre syndrome. J : Clinical tests of sensation are probably not relevant in this case. Comment : This patient turned out to have bulbar onset MND. The differential does include MG, arm strength can be retained, and tongue fasciculations are highly suggestive of MND. The patient could have marked respiratory muscle weakness which would not be obvious clinically; this will influence the prognosis markedly. NCS/EMG after a tensilon test is the most useful test. GBS is defined by a 4-week history. MRI and LP are more use in patients with signs below the neck. Testing sensation is vital as it radically alters the differential diagnosis. A.F 43. A patient present to clinic with intention tremor of the left arm and a wide-based unsteady gait. Where is the lesion likely to be? A : Left pons B : Right globus pallidus C : Right primary cortex D : Right subthalamic nucleus E : Left cerebellar hemisphere. Comment : Lesions of the cerebellum lead to motor disabilities ipsilateral to the side of the lesion. The tremor of Parkinson's disease is most prominent at rest, although postural tremors may be present. The gait is shuffling with the centre of gravity shifted forwards beyond the base of the feet. Patients with extensive ischemic small vessel disease may present with a similar gait, often termed 'marche-a-petit-pas.' The steps are short and, because patients are unsteady, the feet may be a little more widely separated than normal. However, the centre of gravity is over the base of the feet. These patients do not usually have tremors, but may have rigidity similar to that seen in idiopathic Parkinson's disease. E

Page 135: Masterclass Book Part 2

MOHAMMED IS-HAG 134

44. A 75-year-old man with no recent history of head trauma presents with a one-week history of intermittent dysarthria. This progresses during the day and is usually relieved following a period of rest. He gives a past history of cerebrovascular disease and has recently started bezafibrate for the treatment of hyperlipidaemia, in addition to regular aspirin. He has been a lifelong smoker. Which two of the following investigations would be most helpful in his management? A : Carotid doppler studies B : CT brain scan C : Acetyl choline receptor antibodies D : Chest radiograph E : MRI scan of the brain F : Blood glucose G : Erythrocyte sedimentation rate (ESR) H : Temporal artery biopsy I : Random cholesterol measurement J : Cervical spine radiograph. Comment : The patient’s symptoms suggest myasthenia, and bezafibrate is a recognised cause of this, indeed the timing of the onset of his symptoms suggests this may be the most likely cause. The presence of acetylcholine receptor antibodies would be suggestive of myasthenia gravis, and a chest radiograph is necessary to rule out an underlying primary lung tumour. A tensilon test, whereby a small dose of edrophonium is injected and produces rapid resolution of symptoms caused by myasthenia,may be helpful. It is unlikely that the other investigations listed would affect his initial management, although imaging of the brain, preferably with MRI, should be considered if the symptoms progress despite discontinuation of the bezafibrate with negative antibodies and tensilon test. Random measurement of lipids will not influence initial management,although fasting lipids after discontinuation of the bezafibrate may help in the decision of whether treatment with a statin is justified. C.D 45. A 28-year-old man presents with pain radiating to the front of his right thigh. Which two nerve roots could be involved? A : S1 B : L2 C : S3 D : L3 E : L5 F : S4 G : L1 H : L6 I : L4 J : S2 Comment : L2,3 radiates to the anterior thigh. L4 radiates through the knee and down the medial side of the calf to the medial malleolus. L5 radiates through the buttock, down the posteriolateral aspect of the thigh, lateral aspect of calf and across the dorsum of the foot to the big toe.S1 radiates through the inner buttock to the posterior aspect of the thigh, posteriolateral aspect of the calf to the lateral border of the foot. B.D 46. A 45-year-old man is referred to you with an assortment of neurological symptoms and a family history of Huntingdon's disease (HD). Which two of the following are not typically seen in HD? A : paranoia B : emotional lability C : upper motor neurone / pyramidal weakness D : athetosis E : homonymous hemianopia with macular sparing F : self-neglect G : dysarthria H : memory impairment I : bradykinesia J : psychosis Comment : Huntington's disease is an autosomal dominant condition characterized by neurological and psychiatric disturbances. Age of onset is usually in the fourth or fifth decade. Personality change, depression, and psychosis with progressive dementia in association with clumsiness, twitching, chorea and gait disturbance are typically seen. Parkinsonian features become more prominent as the disease progresses. Genetic abnormality (CAG trinucleotide repeat) resides on chromosome 4. Brisk reflexes with clonus may be seen but pyramidal / upper motor neurone weakness is rare. Ocular motor dysfunction is common but field defects are not seen in HD. C. E

Page 136: Masterclass Book Part 2

MOHAMMED IS-HAG 135

47. A 57-year-old teacher presents with a 6-month history of increasing forgetfulness. The two most likely diagnoses are: A : Alcohol-related dementia B : Multi-infarct dementia C : Parkinson’s disease D : Cerebral tumour E : Huntington’s disease F : Hypothyroidism G : AIDS dementia complex H : Chronic subdural haematoma I : Hydrocephalus J : Alzheimer’s disease. Comment : Alzheimer’s disease, multi-infarct dementia and dementia with Lewy bodies are the three commonest causes of dementia, although all of the conditions listed can cause dementia. Gradual progression of symptoms would be typical of Alzheimer’s, as would an absence of so-called ‘subcortical’ features (where the patient is slow with disturbed attention and motivation). There may be a family history of dementia in some cases. The presence of history, symptoms or signs of vascular disease (or its risk factors) would all increase the likelihood of a multi-infarct dementia, as would step-wise progression. Dementia with Lewy bodies often presents with fluctuating cognitive impairment affecting both memory and higher cortical function. The presence of delusions, visual hallucinations, abnormal sleep pattern and motor features of parkinsonism would all support this diagnosis. B.J 48. A 58-year-old man presents complaining of unsteadiness. Which of the following symptoms or signs is usually inconsistent with a cerebellar lesion? A : Symptoms worse in dark B : Symptoms have got worse over several years C : Symptoms are very brief and associated with head movement D : Sustained horizontal nystagmus that doses not fatigue E : A wide-based gait F : Bilateral upgoing plantars G : Papilloedema H : Headache I : Diplopia J : Pain in the neck and wasting of hand muscles. Comment : Cerebellar lesion can be acute or chronic, symmetrical or asymmetric, isolated or part of a more widespread degenerative condition. The symptoms are characteristically worse in the dark and can come on slowly in degenerative conditions. Headache and double vision are common in pontine-cerebellar lesion. On examination the classical findings are horizontal nystagmus, with gait and limb ataxia. The findings of papilloedema suggest a posterior fossa space-occupying lesion (SOL) and the upgoing plantar may be indicative of a brain stem lesion, or MSA. Very brief symptoms associated with head movement are more suggestive of benign paroxysmal positional vertigo (BPPV); neck pain and wasting of hand muscles should raise the possibility of a cervical myelopathy. C.J 49. You are called to see a 65-year-old patient with hypertension and elevated urea and creatinine who is undergoing dialysis. Which two responses are not recognized neurological sequelae of chronic renal disease? A : altered mental status B : seizures C : muscle cramps D : restless legs E : autonomic dysfunction F : muscle weakness G : pure motor neuropathy H : foot drop I : impotence J : optic atrophy Comment : Chronic renal failure is associated with encephalopathy, the mechanism of which remains unclear. Sensorimotor peripheral neuropathy is a common complication leading to dysaesthesias, muscle cramps, restless legs and distal limb weakness. Autonomic dysfunction leads to postural hypotension, impotence and gastointestinal disturbances. Dialysis may cause rapid shifts of water into the brain resulting in headache, seizures, muscle cramps and nausea. Patients undergoing dialysis for more than 1 year may develop a fatal encephalopathy characterised by speech problems, hallucinations, gait disturbances etc., possibly due to aluminium toxicity. G.J

Page 137: Masterclass Book Part 2

MOHAMMED IS-HAG 136

50. A 49-year-old Afro-Caribbean woman sees you in outpatients because three weeks previously she noticed that the right side of her face was numb. She felt otherwise well, with no other symptoms. On examination she had diminished perception of light touch and pinprick in the lower 2/3 of her face, and in the intaoral cavity. In addition, the right corneal reflex was slow. There were no other abnormalities on examination. Possible anatomical sites of trigeminal system involvement include: A : cortical lesion B : trigeminal nuclear lesion C : preganglionic trigeminal nerve root involvement D : gasserion ganglion lesions E : cavernous sinus syndrome F : superior orbital fissure syndrome G : foramen magnum H : foramen ovale I : foramen rotundum J : peripheral branches of the trigeminal nerve. Comment : This woman probably has dysfunction of the mandibular and maxillary division of the trigeminal nerve. The loss of the corneal reflex does not necessarily imply involvement of the ophthalmic division, as the sensory innervation to the cornea is shared by the ophthalmic and maxillary branches. There is no motor involvement. Having determined this, one can begin to determine the possible site of the lesion. Lesions of the trigeminal system are said to be diagnosed by ‘the company they keep’, i.e. associated signs. Loss in such a discrete sensory distribution would not be caused by a cortical lesion, although parietal lesions can cause isolated depression of the contralateral corneal reflex, but hemifacial anaesthesia could result from a small thalamic lesion. Involvement of the rostral (upper) portion of the spinal trigeminal nucleus could result in sensory loss in an onion-skin distribution (upper portion / peri- and intaoral; lower portion / lateral face), that could mimic this patient’s facial sensory loss. The spinal trigeminal nucleus is often involved as part of a lateral medullary syndrome, and either complete, upper, or lower facial hemianaesthesia may be associated, but intra-oral sensation is spared, as this is supplied by the uppermost portion of the spinal trigeminal nucleus, which extends into the pons. Lesions of the preganglionic trigeminal nerve roots may be damaged by tumour, infection, trauma or aneurysm. Lesions in this location are suggested by associated involvement of other cranial nerves (VI, VII, VIII). The trigeminal ganglion may be involved in middle cranial fossa lesions. Pain is often the presenting feature, but sometimes numbness. The distribution is variable and might affect only the lower part of the face. These symptoms may precede several connective tissue diseases, e.g. Sjogren's syndrome, rheumatoid arthritis, systemic sclerosis. In these cases, facial sensory loss is the first symptom in about 10% of cases. The cavernous sinus could not be the site of the lesion, as the ophthalmic and maxillary branches, but not the mandibular branches run through it. Similarly, the superior orbital fissure contains the ophthalmic branch of the trigeminal nerve, together with CN III, IV, VI. A small single lesion could not affect the peripheral branches of the trigeminal nerve. Meningeal involvement at the skull base would produce more diffuse signs, and carcinomatous deposits compressing individual braches (e.g. infraorbital branch of the maxillary division) is to occur singly, and so this combination of the two lower divisions of the trigeminal nerve being affected is unlikely. Anatomical diagnosis of lesions involving the trigeminal nerve is difficult, and relies on being absolutely clear about the signs in the face, and associated signs. Isolated trigeminal sensory neuropathy is the commonest diagnosis in this situation, but is a diagnosis of exclusion. B. D 51. A 60-year-old woman presents with difficulty climbing stairs and standing from a sitting position. You examine her and find she has evidence of painless proximal myopathy. The two most likely causes are: A : Myasthenia gravis B : Motor neuron disease C : Inclusion-body myositis D : Polymyositis E : B12 deficiency F : Thyroid disorder G : Spinal stenosis H : Myotonic dystrophy I : Chronic inflammatory demyelinating polyneuropathy J : Polymyalgia rheumatica.

Page 138: Masterclass Book Part 2

MOHAMMED IS-HAG 137

Comment : The causes of proximal myopathy can be divided into inherited (e.g. Duchenne's, Becker's, facio-scapular-humeral and myotonic dystrophies), inflammatory (e.g. polymyositis, dermatomyositis, polymyalgia rheumatica, inclusion body myositis), endocrine (e.g. steroid-induced, thyroid disorders), metabolic (e.g. glycogen sotrage disorders), toxic (e.g. alcohol, chloroquine, clofibrate). Myasthenia gravis causes fatiguability and commonly occurs in women in the second or third decade of life. B12 deficiency, and CIDP are causes of neuropathy. Polymyalgia rheumatica presents with pain. Inclusion-body myositis presents in this age group with a myopathy principally affecting the thigh and forearm muscles, but is less common than polymyositis and thyroid disorders. D.F 52. A 37-year-old man presents with foot drop. In the assessment of this patient: A : assessment of foot inversion and eversion is critical B : assessment of hip flexion tests the L5 nerve root C : brisk reflexes are an incidential finding D : compression of the obturator nerve should be considered E : pain to the anterior thigh is suggestive of L4 compression F : lumbar plexopathy is a common cause G : common peroneal palsy is associated with numbness over the lateral aspect of the leg and

dorsum of the foot H : ankle jerk should be absent I : marked weakness is more suggestive of a lumber radiculopathy J : sciatica nerve lesions most commonly lead to equal weakness of muscle supplied by the peroneal

and tibial nerve. Comment : Assessment of foot eversion and inversion helps distinguish a common peroneal palsy and multi-level radiculopathy. Hip flexion is test of the L1/2 root. Brisk reflexes suggest upper motor neurones (UMN) which can cause foot drop. Obturator nerve supplies the hip adductors.L4 compression is associated with pain in the medial aspect of the lower leg. Brachial and lumbar/sacral plexopathy are rare. The ankle jerk will be present in L5 lesions.Lumbar radiculopathy can present with little weakness because of the dual innervation of most muscles in the lower leg. A.G 53. A 30-year-old patient presents with episodes of muscle pain after several minutes of exercise and poor exercise tolerance. However, he describes a ‘second wind’ phenomenon if he exercises through the initial barrier. Which two of the following are the most likely diagnoses? A : Acid maltase deficiency B : Becker’s muscular dystrophy C : Carnitine palmitoyl transferase deficiency D : Inclusion body myositis E : Lambert-Eaton syndrome F : Motor neurone disease G : Myasthenia gravis H : Myophosphorylase deficiency I : Myotonic dystrophy J : Phosphorylase b kinase deficiency. Comment : Myophosphorylase deficiency (McArdle’s disease) typically presents in the second and third decade. The ‘second wind’ phenomena is characteristic and is the result of diversion of blood flow to muscle and the onset of fatty acid metabolism. The same clinical picture is seen with phosphorylase b kinase deficiency. H.J 54. A 68-year-old woman is referred having had a ‘funny turn’, the question asked by the general practitioner being ‘was this a transient ischaemic attack (TIA)?’ Which two features listed below would be acceptable to sustain the diagnosis of TIA? A : Vertigo B : Diplopia C : Dysphagia D : Dysphasia E : Dysphonia F : Loss of balance G : Tinnitus H : Impairment of the function of one hand I : Amnesia J : Sensory symptoms confined to one part of limb or face. Comment : A transient ischaemic attack (TIA) is defined as an episode of acute loss of focal cerebral or monocular function lasting less than 24 hours.Since any signs will almost certainly have disappeared before the patient is assessed, the diagnosis depends entirely upon the history. Non-focal symptoms such as loss of consciousness, dizziness, mental confusion, generalised weakness and incontinence are not acceptable as evidence of TIA. In addition, some focal signs occurring in isolation should not be interpreted as TIA because they can so often mislead. These include:

Page 139: Masterclass Book Part 2

MOHAMMED IS-HAG 138

· vertigo · diplopia · dysphagia · dysphonia · loss of balance · tinnitus · amnesia · sensory symptoms with restricted distribution. D .H 55. A 70-year-old man complains of episodic severe throbbing headaches, worse on movement and associated with nausea. He has had one or two episodes per month for the last year, lasting between 4 to 24 hours. You diagnose migraine. Which two statements are true about this condition? A : Migraine may start at any age, but incidence peaks in old age. B : Some forms of migraine are familial. C : The presence of aura, e.g. visual symptoms, is required for the diagnosis. D : Lifestyle modification is not helpful. E : The pain does not respond to non-steroidal anti-inflammatory drugs (NSAIDs). F : Ergot derivatives have a favourable side effect profile. G : The 'Triptans', e.g. sumatriptan, are 5-HT receptor antagonists. H : Triptans should be avoided in ischaemic heart disease. I : Pizotifen is used to treat acute attacks. J : There are no effective preventative treatments. Comment :Migraine can start at any age, but incidence peaks in adolescence.In familial hemiplegic migraine there is dysfunction of ion channels due to mutations. Classic migraine is preceded or accompanied by aura. However, most patients have migraine without aura (previously known as common migraine). Migraine is characterized by the presence of at least some of a number of associated features, e.g. nausea or vomiting, aggravation on movement, sensitivity to light or sound. Non-pharmacological measures such as avoidance of precipitants and regular sleep and meals might help. Dry therapy can be divided into those used to treat acute attacks and those taken daily to reduce the frequency and severity of attacks. Aspirin, paracetamol and NSAIDs may be useful with an anti-emetic for acute attacks. Ergot derivatives have a long history of use for acute attacks; however, there is a significant risk of adverse vascular events. The 'Triptans', e.g. sumatriptan, are 5-HT receptor AGONISTS also used to treat acute attacks. They have a much better safety record then ergot derivatives. However, they may cause vasoconstriction and are therefore not recommended in patients with ischaemic heart disease and cerebrovascular disease. In those who have very frequent attacks, very disabling attacks, or little response to acute medications, preventative therapy should be considered. Pizotifen is an example of preventative therapy which can be effective for some people. B.H 56. A 50-year-old woman presents with a history of progressive distal weakness of her upper and lower limbs. Examination also reveals reduced tone and impaired sensation to all modalities in a glove and stocking distribution to the metacarpophalangeal joint (MCPJ) and knees bilaterally. Ankle and knee reflexes are absent. Possible underlying diagnoses include the following, except: A : Diabetes mellitus B : Alcoholism C : Porphyria D : Chronic inflammatory demyelinating neuropathy E : Hereditary motor and sensory neuropathy F : Myeloma G : Small cell carcinoma of the lung H : Guillain-Barre syndrome I : HIV infection J : Motor neurone disease. Comment : Motor neurone disease results in only motor deficit. Muscle weakness is the commonest presenting symptom and may be limb or bulbar or both at onset. Clinical features include wasting, fasciculation and normal sensation. Reflexes may be pathologically brisk. Porphyria and lead toxicity cause a purely motor peripheral neuropathy. C.J 57. A 60-year-old man is brought into A&E after being found unconscious. Examination reveals bilateral pinpoint pupils. What would this suggest? A : Bilateral oculomotor lesion B : Bilateral uncal herniation C : Midbrain lesion D : Pontine lesion E : Tectal lesion. Comment : A pontine lesion will cause bilateral miosis. D

Page 140: Masterclass Book Part 2

MOHAMMED IS-HAG 139

58. Which of the following are not considered to be neurotransmitters: A : Dopamine B : 5-HT C : GABA D : Glutamate E : Enkephalin F : Noradrenaline G : Alpha-synuclein H : Acetylcholine I : Endorphin J : Aopmorphine. Comment : Apha-synuclein is a genetic locus, mutations of which are associated with Parkinson's disease, multisystem atrophy and dementia with Lewy bodies. Aomorphine is a dopamine agonist used in advanced Parkinson's disease. G.J 59. A 30-year-old woman, with a long history of intravenous drug abuse, is admitted with an abscess in her right groin related to the use of non-sterile needles. This is drained, and she is admitted to a medical ward for intravenous antibiotic treatment. She ‘goes missing’ from the ward at 14.00h, but is found outside a back entrance of the hospital smoking a cigarette at 15.30h and then returns to the ward. At 21.30h you are asked to see her by the ward nurses because she says she feels very unwell. She is nauseated, has had some diarrhoea, and yawns as you talk to her. Her temperature is 37.6 C and her pupils are dilated. The most likely diagnosis is: A : Opioid toxicity B : Alcohol withdrawal C : Clostridium difficile infection D : Opioid withdrawal reaction E : Toxic shock syndrome. Comment : The symptoms and signs of opioid withdrawal include dysphoric mood, yawning, insomnia, nausea, vomiting, diarrhoea, muscle aches, lacrimation / rhinorrhoea, pupillary dilatation, piloerection, sweating and fever. Initially give 10 mg of methadone syrup and wait about 60 min to determine its effect. Continue administering in 10 mg doses until symptoms are under control. It is rare to exceed a total dose of 40 mg over 24 hours. D 60. A 37-year-old man is admitted one hour after the sudden onset of severe headache, which he described as like 'being hit on the head with a hammer'. You suspect subarachnoid haemorrhage, but a CT scan of the brain is reported as normal. To pursue the diagnosis you should: A : wait for 24 hours after the onset of headache, then perform a lumbar puncture to look for

xanthochromia B : repeat the CT scan C : perform a lumbar puncture to look for xanthochromia as soon as possible D : perform a lumbar puncture to look for red blood cells in the CSF as soon as possible E : refer to neuroradiologist for cerebral angiography. Comment :The investigation of choice in suspected subarachnoid haemorrhage (SAH) is immediate CT scan without contrast, taking very thin cuts through the base of the brain to optimise the chances of seeing small collections of blood. Imaging within 12 hours using modern scanners has a 98-100% sensitivity for SAH.Lumbar puncture should be performed in suspected SAH if the CT scan is not diagnostic. The CSF specimen should be centrifuged without delay and examined by spectrophotometry for the presence of xanthochromia due to the presence of oxyhaemoglobin and bilirubin. Note, however, that xanthochromia may not be present if the CSF is examined within 12 hours of haemorrhage occurring, so lumbar puncture should be delayed for 24 hours. A 61. You are referred a 74-year-old woman who had multiple episodes of unilateral transient loss of vision in the right eye described as ‘a curtain coming down’ lasting for several minutes. She is a heavy smoker and has hypertension. She is on aspirin, perindopril, indapamide and simvastatin. She is found to have atrial fibrillation with an apical rate of 78 beats per minute. Investigations reveal a 90% ipsilateral carotid stenosis and 35% contralateral disease and a normal transthoracic echocardiogram. What is the most appropriate course of action? A : Add clopidogrel to aspirin B : Anticoagulation with heparin, then warfarin C : Continue present medication without change D : Referral for consideration of carotid endarterectomy, with delayed post-operative warfarin E : Start prednisolone 60mg and arrange temporal artery biopsy. Comment : If carotid stenosis is greater than 70%, caratid endartectomy confers a beneficial effect and should be considered. Emboli from fibrillating hearts tends to cause large strokes, therefore it is very likely that the carotid stenosis is the source of the emboli. D

Page 141: Masterclass Book Part 2

MOHAMMED IS-HAG 140

62.

An 81-year-old man is brought to A&E by blue light ambulance having collapsed in the supermarket. Urgent CT scan of the brain shows: A : a crescent-shaped infarct B : lesion suggestive of a brain tumour C : haemorrhage in the putamen D : primary brain haemorrhage and chronic subdural haematoma E : a crescent-shaped infarct and signs of cerebral atrophy. Comment : Remember blood appears white on a CT scan. The putamen is the commonest site for hypertensive intracerebral haemorrhage. There is some space around the front of the brain (especially contralateral to the bleed as the latter is causing some mass effect) but the sulcal pattern is clear and the space CSF density, suggesting that this is due to atrophy rather than the presence of a subdural. C 63. A recently married 26-year-old woman presents with a 2-month history of recurrent episodes of a rising epigastric sensation followed by lip-smacking and chewing movements with loss of awareness. The most likely diagnosis and the most appropriate investigation are: A : frontal lobe epilepsy and EEG B : migraine and anti-cardiolipin antibodies C : transient ischaemic attacks and CT scan D : multiple sclerosis and MRI brain scan E : temporal lobe epilepsy and MRI brain scan. Comment : The diagnosis is temporal lobe epilepsy, most likely of mesial temporal lobe origin. The commonest cause is hippocampal sclerosis which is readily identified on MRI. An EEG would also be useful, however the clinical seizure semiology is not suggestive of frontal lobe epilepsy. E 64. Which of the following is associated with axonal neuropathies? A : Reduced conduction velocity B : Reduced compound muscle action potential amplitude C : Abnormally slow F-response D : Delayed P100 latency E : Conduction block. Comment : A is associated with demyelinating neuropathies. C is associated with very proximal disease, i.e. radiculopthies. D is a feature of performing visual evoked potentials in those with optic nerve disease. E is usually associated with certain types of demyelinating neuropathy. B 65. In Alzheimer's disease: A : an Abbreviated Mental Test (AMT) score of less than 5/10 is diagnostic B : it is easy to manage the patient in the community C : the condition almost always presents with gradual deterioration D : most carers appreciate being told that the person they care for has Alzheimer's disease E : the pathology shows characteristic neurofibrillary tangles in the glia and amyloid deposits in the

neurones. Comment : A low AMT score may indicate dementia, acute confusion, severe depression, deafness, dysphasia or the inability to understand English - it does indicate that further cognitive evaluation is needed. Dementia of all types is a common reason for admission to institutional care as it is difficult to maintain confused people in the community. Although Alzheimer's disease is a progressive disorder, the presentation is not infrequently ‘acute’ and may be precipitated by a variety of factors from moving house to the death of the spouse who had been covering up the patient’s deficits, to an acute illness resulting in acute on chronic confusion. The pathology is intra neuronal tangles and extra neuronal amyloid. In a survey carried out by the Alzheimer's Disease Society there was a clear message that getting a diagnosis was really frustrating. By the time the patient or more usually the carer sought help, the problem was already severe and a reluctance to make the diagnosis just extended the time in which the family received no support. D

Page 142: Masterclass Book Part 2

MOHAMMED IS-HAG 141

66.

PLATE 27 The 45-year-old man shown (see image) is admitted with ptosis, diplopia and dysphagia. On examination he is found to have fatigable muscle weakness and normal deep tendon reflexes. Which of the following blood tests is most likely to confirm the underlying diagnosis? A : Acetylcholine receptor antibodies B : Anti GM1 antibodies C : Anti GQ1b antibodies D : Anti Hu antibodies E : Anti Purkinje cell antibodies. Comment : Acetylcholine receptor antibodies are present in about 80% of patients with generalized myasthenia gravis. A 67.

The image shows a T1-weighted axial MRI slice of a 32-year-old male patient with band heterotopia, a malformation of cortical development. Which of the following statements is incorrect? A : Malformations of cortical development (MCDs) frequently cause epilepsy. B : MCDs can be found incidentally on MRI in otherwise apparently normal subjects. C : MCDs are almost invariably associated with skin lesions. D : MCDs do not become malignant. E : MCDs can be genetically determined. Comment : MCDs are a common cause of refractory focal epilepsy, although they may be found incidentally on MRI scans performed for other reasons. Patients may be cognitively impaired or completely normal. The abnormalities are non-progressive developmental lesions, and as such are not malignant or invasive. Skin lesions are rare. MCDs may be acquired in utero during cerebral development or genetically determined. Overall, surgical treatment is associated with seizure freedom in only approximately 40%. The image shows band heterotopia, also called double cortex syndrome because of its appearance on MRI. It is thought to represent arrest of migrating neurons during cortical development as they move from the periventricular region to the cortex. C

Page 143: Masterclass Book Part 2

MOHAMMED IS-HAG 142

68. A 30-year-old female patient presents with a 5-day history of ascending sensorimotor deficit of all four limbs. You suspect the diagnosis of Guillain–Barré syndrome (GBS). Which one of the following responses is true: A : In 90% of individuals there is a history of a preceding upper respiratory or gastrointestinal

infection B : Hyperreflexia is typical C : Autonomic dysfunction is rare D : The cerebrospinal fluid is typically normal throughout the course of the illness E : Treatment with intravenous immunoglobulin is as effective as plasma exchange Comment : GBS affects people of all ages. Approximately 60–70% of patients have a preceding infective illness approximately 1–4 weeks before onset of neurological deficit. These include: CMV, EBV, viral hepatitis, HIV, mycoplasma and most commonly, Campylobacter jejuni. Initial features may be sensory or motor deficit or weakness alone. Hyporeflexia is an invariable feature. Assisted ventilation is required in up to 20% of patients. Autonomic dysfunction is common, therefore patients should be placed on a cardiac monitor. Steroids are ineffective. E 69. A 67-year-old man presents with ptosis of the left eye. You explain that this could be due to Horner’s syndrome. A medical student asks you how the sympathetic fibres reach the pupil. You reply that they travel in a branch of the: A : third nerve B : sixth nerve C : seventh nerve D : fifth nerve E : fourth nerve. Comment : Sympathetic fibres reach the pupil as the long ciliary nerves from the naso-ciliary branch of the fifth nerve. Sympathetic fibres to the eyelid travel in the branch of the third nerve to levator palpebrae superioris. D 70. A 78-year-old man presents with a stroke. When conducting your physical examination of him you are trying to localize this. Which one of the following features would NOT be consistent with the diagnosis of a subcortical lacunar stroke? A : Pure motor stroke B : Pure sensory stroke C : Ataxic hemiparesis D : Dysarthria and a clumsy hand E : Dysphasia. Comment : Evidence of higher cortical involvement, e.g. dysphasia, dyscalculia or visuospatial disorder, or disturbance of consciousness, excludes a lacunar syndrome. All of the other features are consistent with this diagnosis. E 71.

A 60-year-old man presents with a sudden onset left hemiplegia. A CT scan of the head is performed (see image). Which of the following statements is true? A : The patient has a cerebellar infarct. B : The patient has a cerebellar haemorrhage. C : The patient has cerebellar tumour. D : No specific treatments are available if the patient deteriorates. E : The patient should be commenced on aspirin. Comment : The CT shows a high attenuation lesion most consistent with a primary haemorrhage. An infarct would be hypodense. A tumour could underlie the bleed but is less likely. The patient is at high risk of hydrocephalus. Neurosurgery would be life-saving. All anti-platelets and anti-coagulates should be stopped. B

Page 144: Masterclass Book Part 2

MOHAMMED IS-HAG 143

72. A 30-year-old man presents with a tremor. He has had this for many years, but it has become worse recently such that he now finds it socially embarassing. His father had a similar problem. His gait is normal. The most likely diagnosis is: A : Wilson's disease B : Familial cerebellar degeneration C : Parkinson's disease D : Huntington's disease E : Benign essential tremor. Comment : Benign essential tremor is a fine tremor that often starts in childhood or adolesence, but only presents later when it becomes functionally debilitating. The tremor is typically worse on using the affected limb, e.g. using a knife and fork or holding a cup. It can be sporadic or inherited (autosomal dominant with variable penetrance). The tremor is improved by alcohol. Tremor is rarely the only symptom of cerebellar disease, and gait disorder would certainly be present if tremor was caused by cerebellar disorder. E 73. A 67-year-old man is seen in A&E after falling down his stairs. He is noted to have some Parkinsonian features, but the astute SHO considers the possibility of progressive supranuclear palsy (PSP). Which of the following findings DO NOT support her diagnosis? A : Conjunctivitis B : Reduction of upward gaze C : Poor short term memory D : Startled expression E : Symmetrical akinesia. Comment : The primary diagnostic indicators include the early onset of postural instability and vertical gaze palsy. However, reduction in upgaze is non-specific as it may be present in a number of other conditions or even normal ageing. Paralysis of downgaze is very highly suggestive of PSP although even this is not pathognomonic, as it has been reported in frontotemporal dementia. Other features of PSP include very reduced blinking, which may lead to dry eye and conjunctivitis, and retrocollis and frontalis over activity, which leads to a startled expression. The akinesia is usually symmetrical from its onset, contrasting with the asymmetrical findings in early Parkinson's disease. B 74. A 30-year-old man describes recurrent daily attacks of severe constant unilateral orbital pain over the last 2 months. He tends to get bouts of this type of attacks once to twice a year, typically lasting six to eight weeks in duration. Which of the following would be appropriate prophylactic treatment for this patient? A : Amitryptyline B : Oxygen 100% C : Pizotifen D : Propranolol E : Verapamil. Comment : The patient has cluster headache. Prophylactic treatment includes verapamil, prednisolone, lithium carbonate, ergotamine and methysergide. E 75. You are asked to give a medical opinion on a 67-year-old man who attends A&E after experiencing sudden onset left-sided weakness lasting for 20 minutes. Three days previously he had suffered an episode of blurred vision in his right eye that lasted for 5 minutes and was associated with a headache lasting for an hour. Previously he had been seeing his GP for elevated blood pressure and for help in stopping smoking. Neurological examination was normal. His blood pressure was 150/90 mmHg, heart rate 84 and regular. There were no audible carotid bruits. Which of the following is the most likely diagnosis? A : Transient ischaemic attack secondary to giant cell arteritis B : Migraine equivalent C : Transient ischaemic attack secondary to carotid artery disease D : Transient ischaemic attack secondary to small vessel disease E : Transient ischaemic attack secondary to cardioembolism. Comment : The left-sided weakness is due to right cerebral hemisphere dysfunction, and with the sudden onset is likely to represent a transient ischaemic attack. The combination of these two symptoms strongly suggests a right internal carotid artery stenosis. Cardioembolism is unlikely in the absence of either atrial fibrillation or previous cardiac symptoms of either cardiac ischaemia or cardiac failure. Transient occlusion of a small penetrating vessel i.e. a lacunar syndrome could cause the hemiparesis but not the amaurosis fugax. Giant cell arteritis should always be considered in patients over 6o years old, but there are no specific features to support the diagnosis, although it is always worthwhile checking the erythrocyte sedimentation rate (ESR). Lastly, migraine equivalents (aura-like symptoms without headache) can provide diagnostic difficulties but there is no suggestion of the characteristic slow spread of symptoms in this patient. C

Page 145: Masterclass Book Part 2

MOHAMMED IS-HAG 144

76. A 60-year-old man is referred to you with stiffness and fluctuating confusion. His symptoms began about three years ago when he noticed stiffness in his legs. The stiffness gradually progressed and now affects all limbs. He also has an associated mild tremor affecting all limbs. He has noted that his writing has become illegible. Over the last six months, he has developed hallucinations and reports seeing insects on the walls of his house. He is also increasingly forgetful at home. At night, his wife says that he is often restless and agitated. On examination, his mini-mental score was 15/30. Blood pressure fluctuated between 140-160/80-100 with no consistent postural drop. He is symmetrically rigid and slow with a tremor in all limbs. Cranial nerve examination reveals mild restriction of conjugate upgaze eye movement. His gait was shuffling with a tendency to fall backwards. What is the most likely diagnosis? A : Benign essential tremor B : Idiopathic Parkinson’s disease C : Lewy body dementia D : Multiple system atrophy E : Progressive supranuclear palsy. Comment : The combination of progressive cognitive decline, fluctuating symptoms, visual hallucinations, extrapyramidal signs (rigidity and bradykinesia more prominent than tremor) and sleep pattern reversal suggest a diagnosis of Lewy body dementia. C 77. The following are NOT features of cerebellar tremor: A : Head tremor (titubation) B : 3Hz tremor in the horizontal plane C : Absent during movement initiation D : Legs are rarely affected E : Rapid alternating movements are impaired. Comment : A - titubation may be a featre of both this and essential tremor.B - 3Hz is the typical rate, compared to 5-8Hz for essential tremor, and 8-12Hz for physiological tremorC - absent at rest and early stages of movement, getting progressively worse (intention tremor).D - legs commonly affected. The legs are rarely affected in essential tremor. E - common feature. D 78. From the following list, the most efficacious treatment for restless leg syndrome is most likely to be: A : Gabapentin B : Clonazepam C : Codeine phosphate D : Cabergoline E : Ferrous sulphate. Comment : Although head to head trials are not available, it is accepted that dopamine agonists have the greatest chance of therapeutic success in restless leg syndrome. All the other agents have been, and still are, used. D 79.

PLATE 29 A 60-year-old woman is asked to dorsiflex both her feet (see image). On examination, her reflexes in the lower limbs are all normal. An abnormality affecting which of the following root lesions would be expected to give the above clinical picture? A : L2 B : L3 C : L4 D : L5 E : S1. Comment : A lesion of L5 causes weakness of ankle dorsiflexion, inversion and eversion and dorsiflexion of the great toe. A lesion of S1 causes weakness of plantar flexion, eversion and knee flexion, and an absent ankle jerk. Therefore a foot drop in association with the loss of the ankle jerk will only occur with involvement of both L5 and S1. D

Page 146: Masterclass Book Part 2

MOHAMMED IS-HAG 145

80. A 58-year-old man presents with back pain that radiates through the knee and down the medial side of the calf to the medial malleolus. The nerve root involved is: A : L2 B : L3 C : L4 D : L5 E : S1. Comment : L2,3 radiates to the anterior thigh.L5 radiates through the buttock, down the posteriolateral aspect of the thigh, lateral aspect of calf and across the dorsum of the foot to the big toe.S1 radiates through the inner buttock to the posterior aspect of the thigh, posteriolateral aspect of the calf to the lateral border of the foot. C 81. A 44-year-old man presents with double vision that is worst when trying to read a book or walk down stairs. The most likely cause is: A : oculomotor (3rd nerve) paralysis B : trochlear (4th nerve) paralysis C: abducens (6th nerve) paralysis D: supranuclear gaze palsy E: internuclear ophthalmoplegia. Comment : Diplopia is most marked when looking in the direction in which the weak muscle has its purest action. The superior oblique muscle, innervated by the fourth nerve, acts to depress the eye and is maximally effective when the eye is looking medially, hence diplopia on reading or going downstairs is typical of fourth nerve paralysis. B 82. A teenager is diagnosed with juvenile myoclonic epilepsy. What first line treatment would you prescribe? A : Carbamazepine B : Ethosuximide C : Phenytoin D : Sodium valproate E : Topiramate. Comment : Approximately 90% of patients with juvenile myoclonic epilepsy become seizure free on sodium valproate. D 83.

PLATE28 This image of the substantia nigra shows the presence of alpha synuclein-positive Lewy bodies (pigmented nerve cells) and Lewy neurites (pigmented 'streak-like' bodies). Which one of the following statements regarding alpha synuclein diseases is NOT true? A : Idiopathic Parkinson's disease (PD) is associated with Lewy bodies and Lewy neurites B : Dementia with Lewy bodies (DLB) is characterized by large numbers of Lewy bodies and Lewy

neurites in cortical brain areas and in the substantia nigra C : Lewy body pathology is commonly seen in multiple system atrophy (MSA) D : The formation of filamentous inclusions in alpha synucleinopathies is associated with decreased

solubility of the alpha synuclein protein E : Missense mutations in the gene encoding for alpha synuclein cause familial variants of PD. Comment : Synucleins are soluble proteins that are abundant in the brain but whose functions are poorly understood. The synuclein family currently consists of four members, of which alpha synuclein is the most studied.Parkinson's disease (PD) is the most common neurodegenerative movement disorder. The main neuropathological feature of the disease consists of eosinophilic, round cytoplasmic inclusions in the form of Lewy bodies and Lewy neurites. These inclusions are particularly numerous in the substantia nigra. Lewy bodies are confined largely to nerve cells, with glial cells being only rarely affected. The alpha synuclein gene has been mapped to chromosome 4. Missense mutations in the gene have been shown to cause familial variants of PD. In addition to PD, Lewy bodies and Lewy neurites also constitute the defining neuropathological characteristics of dementia with Lewy bodies (DLB). Unlike PD, DLB is associated with large numbers of Lewy bodies and Lewy neurites in cortical brain areas. But like

Page 147: Masterclass Book Part 2

MOHAMMED IS-HAG 146

PD, DLB is also characterized by Lewy body pathology in the substantia nigra. Clinically, multiple system atrophy (MSA) is often mistaken for PD. However, unlike PD, Lewy body pathology is not seen in MSA. Instead, glial cytoplasmic inclusions, which consist of filamentous alpha synuclein aggregates, are the defining neuropathological feature of MSA. The principal brain regions affected are the substantia nigra, striatum, locus coeruleus, pontine nuclei, inferior olives, cerebellum and spinal cord. The formation of the filamentous inclusions in nerve cells for PD and DLB, and in glial cells in MSA is associated with decreased solubility of alpha synuclein. C 84. A 38-year-old woman has weakness of her right foot. You are trying to decide whether she has a common peroneal nerve lesion or an L5 root lesion. Which of the following statements is true? A : An L5 root lesion will cause sensory and motor symptoms, but a common peroneal nerve lesion

will only cause motor symptoms. B : A common peroneal nerve lesion will cause weakness of ankle dorsiflexion, eversion and

inversion, but an L5 root lesion not affect inversion. C : An L5 root lesion will cause loss of the ankle jerk, but a common peroneal nerve lesion will not. D : A common peroneal nerve lesion will cause weakness of ankle dorsiflexion and eversion but will

not affect inversion, which will be affected in an L5 root lesion. E : The two conditions cannot be distinguished clinically, but can by nerve conduction studies. Comment : A common peroneal nerve lesion leads to foot drop with loss of ankle and toe dorsiflexion and ankle eversion. Numbness occurs over the lateral aspect of the lower leg and dorsum of the foot. It is usually due to pressure over the fibular head. An L5 root lesion causes weakness of ankle dorsiflexion, inversion and eversion and dorsiflexion of the great toe. Numbness occurs down the posteriolateral aspect of the thigh, lateral aspect of the calf and across the dorsum of the foot to the great toe. Loss of the ankle jerk occurs in lesions of S1. D 85. A 40-year-old obese man is referred with excessive daytime sleepiness. Which of the following is NOT a feature of obstructive sleep apnoea? A : Intellectual deterioration B : Morning headache C : Snoring D : Awareness of sleep disturbance E : Nocturnal enuresis. Comment : Snoring occurs in all patients with obstructive sleep apnoea (OSA), and intellectual deterioration in 75%. Morning headache and nocturnal enuresis occurs in approximately a third of adult patients. Patients are not aware of their sleep disturbance and a witness description is necessary for the diagnosis to be made. D 86. A 38-year-old woman presents with severe headache. Your house physician tells you that her ‘right eye is not normal’, and you suggest that the patient might have a third nerve palsy. The houseman asks you what signs he should look for to confirm this diagnosis. You reply: A : there is ptosis, a small pupil, and the eye is positioned ‘down and out’ B : there is a small pupil and the eye cannot abduct C : there is ptosis, a dilated pupil, and the eye is positioned ‘down and out’, but will rotate inwards if

the patient attempts to look down D : there is a small pupil and the eye is positioned ‘down and out’ E : there is ptosis, a dilated pupil, and the eye is positioned ‘down and in’, and will not move in any

direction. Comment : The third (oculomotor) nerve innervates all the muscles controlling movement of the eyeball excepting the superior oblique (fourth nerve) and lateral rectus (sixth nerve). The superior oblique acts to depress the eyeball and the lateral rectus to abduct it, hence the unopposed action of these muscles leads to the eye being ‘down and out’. Action of the superior oblique on the abducted eye leads to inward rotation of the eye if the patient attempts to look down.The third nerve carries parasympathetic fibres to the pupil, paralysis of which leads to papillary dilatation. This is not an invariable feature of third nerve palsy, being absent in some medical (e.g. diabetes) causes, but is seen with compressive damage – and an aneurismal cause is likely in this woman. C 87. Which of the following signs are not suggestive of a carotid transient ischaemic attack (TIA)? A : Aphasia B : Bilateral visual loss C : Contralateral hemiparesis D : Contralateral homonymous hemianopia E : Dysarthria. Comment : Bilateral visual loss is a sign of vertebrobasilar TIA. Unilateral monocular visual loss is a sign of carotid TIA. B

Page 148: Masterclass Book Part 2

MOHAMMED IS-HAG 147

88. Which of the following is not typical of the Miller-Fisher syndrome? A : An upper respiratory tract infection is the most common antecedent illness B : Diplopia is the most common initial symptom C : The prognosis for a full recovery is good D : It is associated with elevated anti-GQ1b IgG antibody E : Profound limb weakness occurs in half of all cases. Comment : Miller-Fisher syndrome is characterized by the triad of external ophthalmoplegia, ataxia and loss of tendon reflexes. It is considered to be a variant of the Guillain- Barre syndrome and accounts for 5% of its cases. Median age of onset is approximately 40 years and it is slightly more common in men than women. Its incidence is higher in spring, with an upper respiratory tract infection being the most common antecedent illness. The most common initial symptom is diplopia. Other early symptoms include: · ataxia · dysaesthesia in the limbs · dysphagia · blepharoptosis · photophopbia The median interval between the onset of diplopia and ataxia is one day. Neurological signs that may develop during the course of the illness include external and internal ophthalmoplegia, nystagmus, facial palsy, bulbar palsy, hyporeflexia or areflexia and, uncommonly, sensory loss (proprioception and joint position sense). Pure Miller-Fisher syndrome is typically not associated with profound limb weakness. Cerebrospinal fluid examination reveals albuminocytological dissociation. Serum IgG antibodies to the ganglioside GQ1b are found in the acute sera of most patients with Miller-Fisher syndrome and Guillain-Barre syndrome with ophthalmoplegia, suggesting that the antibodies are related to the pathogenesis of the disease. There is also evidence that molecular mimicry may be involved between ganglioside GQ1b and a Camphylobacter jejuni lipopolysaccharide, in which case infection by C jejuni bearing GQ1b-like lipopolysaccharide may trigger the production of anti-GQ1b IgG antibody. The antibody then binds to ocular motor nerves and deep cerebellar nuclei causing ophthalmoplegia and cerebellar ataxia. Miller Fisher syndrome has a benign prognosis and nearly all patients have no or little disability six months after disease onset. E 89.

The image shows a T1-weighted coronal MRI image of a patient with Rasmussen's encephalitis (RE). The following statements are true of Rasmussen's encephalitis except: A : It may present with epilepsia partialis continua. B : It usually presents in the first decade of life. C : It is associated with progressive hemiplegia. D : Hemispherectomy can be curative. E : Secondarily generalized seizures are rare. Comment : RE is a disorder of previously healthy children usually younger than 10 years of age. Seizures are limited on one hemisphere but later may become bilateral. The initial feature is often epilepsia partialis continua of one limb. Episodes of secondary generalization are common. Progressive hemiplegia is the rule. Seizures soon become refractory to anti-epileptic medication. Most children require hemispherectomy for seizure control. The cause of RE is unknown but may be auto-immune mediated. Anti-glutamate receptor antibodies have been found in some patients. E

Page 149: Masterclass Book Part 2

MOHAMMED IS-HAG 148

90. Which clinical features are more suggestive of Lambert–Eaton syndrome than myaesthenia? A : Ocular muscles are involved first B : No change in reflexes after hard muscular contraction C : Presence of a thymoma D : A dry month E : Normal electromyographic (EMG). Comment : Compared to myasthenia gravis (MG), Lambert-Eaton myasthenic syndrome (LEMS) affects distal muscles initially, is associated with autonomic signs including a dry month and in approximately 40% of patients, reflexes return after muscle contraction. EMG is usually diagnostic. D 91. A 48 yr old man has attacks of facial / head pain and you are considering the diagnosis of cluster headache. Which one of the following statements is NOT true of this condition? A : Effective acute treatment includes 100% oxygen, inhaled for 15-20 minutes B : The pain commonly follows the distribution of the trigeminal nerve C : The acute attack is frequently alleviated by alcohol D : Often occurs in the same month each year E : Cluster headache is five times more prevalent in males than females. Comment : Cluster headache is a strictly unilateral, excruciating headache with an abrupt onset and cessation. The pain commonly follows the distribution of the trigeminal nerve, particularly the orbital or temporal divisions. Prevalence is one per 1000 with a male to female ratio of 5:1, and patients typically present in their 30's or 40's.Pain is associated with cranial autonomic dysfunction. The International Headache Society classification diagnostic criteria require the cluster attacks to be accompanied by at least one of the following, which have to be present on the pain side: · Conjunctival injection · Lacrimation · Miosis · Ptosis · Eyelid oedema · Rhinorrhoea · Nasal blockage · Forehead or facial sweating. Although these are typically transient symptoms and last only as long as the attack, ptosis or miosis may occasionally persist. Cluster headache has a striking circadian and circannual periodicity. Each attack may occur at the same time of day or night and each cluster may occur in the same month of each year. The attacks within a cluster usually last a few minutes to several hours, and each cluster may last a few weeks to several months. Attack precipitants include alcohol, nitroglycerine, exercise, and an elevated environmental temperature. Symptomatic cluster headache occurs in infectious, vascular and neoplastic intracranial lesions. Differential diagnoses of cluster headache include migraine and paroxsysmal hemicrania, which is a syndrome that is clinically similar except more common in females, attacks are briefer and more frequent, and it completely responds to indomethacin. Treatment of cluster headache includes: · General measures: alcohol avoidance during cluster bout · Abortive agents: triptans (e.g. subcutaneous or nasal sumatriptan), oxygen (100% at 7-12 litres per minute for at least 15-20 minutes), topical lignocaine (nasal drops or spray can also be effective) · Prevention: to prevent further attacks during a cluster, steroids, methysergide (short-term only) or ergotamine can be used · Long-term prevention: to prevent clusters from occurring, verapamil and lithium can be used, or surgery can be performed to interrupt the trigeminal sensory or autonomic pathways. C 92. An 82-year-old man with a history of hypertension and atrial fibrillation is admitted after two episodes of loss of conciousness associated with generalized shaking. After a period of drowsiness and confusion he is now alert and orientated. There are no neurological signs. There is no evidence of intercurrent illness. Which of the following statements is correct? A : It is unusual for seizures to start for the first time at this age. B : A brain tumour is the most likely underlying cause. C : The most likely aetiology is occult cerebrovascular disease. D : An EEG can reliably differentiate seizures from other causes of syncope. E : Chances of complete seizure control with antiepileptic drugs are small. Comment : Studies have shown that the age-specific incidence and prevalence of treated epilepsy is higher in elderly people than in any other age group. Cerebrovascular disease (overt and occult) is by far the commonsest cause in this age group. Cerebral tumours account for only a minority. A reliable history from the patient and witness are more valuable than an EEG. Up to 38% of healthy old people may have abnormal EEGs whilst few old people with seizures have abnormal inter-ictal EEGs. An EEG should therefore be interpreted with caution. Appropriate treatment will control seizures in more than 70% of affected patients. C

Page 150: Masterclass Book Part 2

MOHAMMED IS-HAG 149

93. An 72-year-old female is admitted by her GP with intermittent confusion over the last 48 hours. She is on no regular medication. There is no past medical history of note apart from a simple fall complicated by a Colles fracture and minor head injury 3 weeks before. Clinical examination is normal, including mental state assessment. The night following admission she is noted to be slightly confused. Routine bloods, resting ECG and chest radiograph are normal. Which of the following conditions must be excluded? A : Alzheimer's disease B : Intercurrent urinary tract infection C : Sub-arachnoid haemorrhage D : Sub-dural E : Transient ischaemic attack. D: 94. A 43 -year-old man complains of sensory loss in his left arm and hand. On examination he has subjectively diminished light touch and pinprick sensation in the left hand extending to above the elbow. Joint position sense appears intact. He has difficulty distinguishing his lighter from a pen using the left hand (with eyes closed), and his two-point discrimination is 11 mm. What will your first investigation be? A : CT head B : Electromyogram (EMG) and nerve conduction studies C : MRI cervical spine D : Chest radiograph E : Vitamin B12 levels. Comment : The sensory signs described here are cortical sensory signs, and are indicative of a lesion of the anterior parietal cortex, particularly mid postcentral gyrus. A 95. A 70-year-old man presents with left-sided hemiparesis associated with a gaze paralysis to the right. Which part of the brain is damaged? A : Left frontal lobe B : Left pons C : Right frontal lobe D : Right medulla oblongata E : Right pons. Comment : The lateral gaze centre is situated in the pons. A lesion in this area will cause impaired conjugate gaze to the side of the lesion, with consequent deviation away from the side of the lesion. E 96. A 36-year-old female patient presented with acute onset of double vision and was found to have normal eye movements vertically and looking to the right. On left lateral gaze however, there was absence of adduction of the right eye, and nystagmus in the abducting left eye. This eye movement disorder can be explained by a lesion in the: A : right medial longitudinal fasciculus B : sixth cranial nerve on the left C : left medial longitudinal fasciculus D : left cerebellopontine angle E : third cranial nerve on the right. Comment : The diagnosis is right internuclear ophthalmoplegia due to a lesion in the right medial longitudinal fasciculus. The likely underlying pathology is multiple sclerosis, although other possibilities include a pontine glioma or vascular lesion. A left 6th nerve palsy affects abduction in the left eye only. A third nerve palsy results in ptosis, and an eye fixed looking "down and out" due to the unopposed actions of lateral rectus and superior oblique muscles. A 97. The clinical features of restless leg syndrome DO NOT include which of the following? A : Motor restlessness - an urge to get up and move around B : Relief of symptoms on moving C : Association with tremor in 30-40% D : Symptoms worse in the evening or at night E : Uncomfortable sensory symptoms in arms, trunk and legs. Comment : A, B, D and E are typical features and form part of the diagnostic criteria. Symptoms are most commonly experienced in the legs, but other parts of the body can be less commonly affected. The condition has been associated with idiopathic Parkinson's disease, peripheral neuropathy, but the link is not clear. It is seen in 20% of pregnancies. It is associated with iron deficiency anaemia, and may respond to iron supplements. It is also associated with diabetes mellitus and hypothyroidism, so worth checking the blood sugar and thyroid function. There is a family history in 92% (linked to chromosome 12q and 14q) of cases. C

Page 151: Masterclass Book Part 2

MOHAMMED IS-HAG 150

98. A 35-year-old man presents to the A&E department complaining of a sudden onset of occipital headache, associated with vomiting, which started within the previous two hours and is continuing. The headache is much more severe than his usual migraine, which has not previously started suddenly. Examination, including fundoscopy is unremarkable, with no neck stiffness or photophobia. Following prescription of analgesia and an antiemetic, which of the following management options would be the most appropriate? A : Reassurance that migraine is the most likely and discharge from the A&E department after a

short period of observation B : Immediate lumbar puncture, to exclude meningitis and subarachnoid haemorrhage C : CT brain scan, followed by discharge if normal D : CT brain scan followed by lumbar puncture if CT normal E : Immediate referral for cerebral angiography. Comment : Despite the previous history of migraine, the patient displays two concerning features: sudden onset and greater severity, which requires that subarachnoid haemorrhage (SAH) be excluded. CT brain scan is normal in up to 30% of patients with SAH, so that a negative scan cannot be used to exclude the diagnosis if suspected clinically. However, a positive CT brain scan enables the diagnosis to be made non-invasivally. Absence of red blood cells, or the absence of xanthochromia is sensitive for the exclusion of SAH. Where lumbar puncture (LP) is traumatic, the presence of large numbers of red cells may complicate interpretation. In this situation, spectophotometric analysis of the supernatant for red cell pigments may be helpful. Bilirubin and oxyhaemoglobin take 12 hours to appear in the cerebrospinal fluid (CSF) following a bleed, and for this reason LP should usually be delayed until more than 12 hours from the onset of symptoms. Because of the invasive nature of cerebral angiography, and relatively high complication rate, it is usually reserved for patients whose other investigations are positive. D 99. Which of the following is not a characteristic of episodic memory : A : Childhood memories B : What you had for breakfast C : Describing how to use a kettle D : Recollecting a memorable address E : The dates of WWII. Comment : C relates to semantic memory - the working knowledge of the world the others relate to episodic memory - recollection of events. C 100. Which one of the following features is NOT true of pure hereditary spastic paraparesis? A : autosomal dominant inheritance is the commonest mode of inheritance B : spasticity is more common in the lower, than upper, limbs C : paresis is greater than spasticity D : cranial nerve function is normal E : life expectancy is normal. Comment : Hereditary spastic paraparesis (HSP) is a heterogenous group of disorders characterised by progressive spasticity in the lower limbs. Upper limbs are less commonly and severely affected. Spasticity is typically more severe than weakness. Pure and complicated forms of HSP have been described. The following are common in pure HSP: · sphincter disturbances · mild dorsal column involvement · pes cavus. Relevant negative findings in pure HSP include: · normal cranial nerves · no involvement of the corticobulbar tract. Complicated HSP may be associated with a variety of clinical features: · amyotrophy · epilepsy · extrapyramidal signs · sensory neuropathy · retinal changes · dementia. Autosomal dominant inheritance is the commonest mode of inheritance. The molecular genetics of HSP are being rapidly uncovered and it is thought that mutations in the spastin gene account for up to 50% of autosomal dominant cases. At present, genetic testing is not widely available and HSP remains largely a diagnosis of exclusion. The severity of cases vary between and within families, but life expectancy is normal. Early onset (<35 years) is associated with a slower disease progression. C

Page 152: Masterclass Book Part 2

MOHAMMED IS-HAG 151

101. An 83-year-old man is admitted complaining of acute onset weakness in both legs and low back pain for the previous 2 months for which he had been taking codeine phosphate. On examination, he was cachectic and pale. His cranial nerves and upper limbs were normal. His hips and knees were weak. He had a sensory level just below the umbilicus with extensor plantar reflexes. Rectal tone was normal, but he had a large, craggy prostate. Which is the most important investigation to perform? A : Bone scintigram B : Prostate specific antigen C : Transrectal prostatic biopsy D : MRI thoracic spine E : Radiograph thoracolumbar spine. Comment : This man has cord compression and therefore needs urgent imaging of his spine to facilitate early treatment with radiotherapy. The other tests are all useful, and may help with later management. D 102. A 19-year-old woman presents with optic neuritis and the T2-weighted brain MRI demonstrates multiple high signal lesions in the white matter. The risk of a further episode of neurogical dysfunction is: A : 75% B : 10% C : 50% D : 25% E : 95%. Comment : The risk of a further episode of neurogical dysfunction, i.e. developing multiple sclerosis, is moderately high after the occurrence of optic neuritis in the context of multiple white matter lesions. A 103. You are examining the pupils of a 48-year-old woman in whom the diagnosis of multiple sclerosis is suspected. Which of the following observations would make you conclude that she has a left relative afferent pupillary defect? A : You move a light from her left eye to her right eye, and the left pupil constricts. B : You move a light from her right eye to her left eye, and the left pupil dilates. C : You flash a light on and off in her right eye, and the left pupil constricts, but it does not when

you flash it on and off in her left eye. D : You move a light from her right eye to her left eye, and the left pupil constricts. E : You move a light from her left eye to her right eye, and the left pupil dilates. Comment : A relative afferent pupillary defect (RAPD) is detected using the 'swinging light test'. It allows comparison of pupillary constriction generated through the afferent pathway (within the retina or optic nerve anterior to the chiasm) of each eye with that produced consensually. After checking for direct and consensual pupillary responses in each eye, proceed as follows. Shine a bright light into the right eye for five seconds. Look at the pupil of the left eye. Move the light swiftly to shine in the left eye. If the left pupil dilates, there is a left RAPD. Now observe the pupil of the right eye. Move the light back to the right eye. If the right pupil dilates there is a right RAPD. B 104. A 56-year-old alcoholic patients presents with a 1-week history of progressive confusion and unsteadiness of his gait. On examination he has no stigmata of chronic liver disease and no evidence of portal hypertension. Eye movements and cerebellar function appear normal. Which of the following statements is correct? A : Wernicke's encephalopathy is unlikely in the absence of ophthalmoplegia. B : Treatment with high dose oral thiamine is likely to reverse the symptoms. C : If the patient is hypoglycaemic, intravenous thiamine should be administered before

administration of glucose. D : Hepatic encephalopathy is the most likely explanation for his symptoms. E : A single dose of intravenous thiamine is usually sufficient for treatment of acute wernicke's

encephalopathy. Comment :The classical triad of confusion, ataxia and ophthalmoplegia is present in only 10% of patients with Wernicke's encephalopathy; confusion is the commonest symptom, present in over 80% of cases, and the condition may mimic and complicate 'simple' alcohol withdrawal. Treatment requires high doses of intravenous thiamine to be given repeatedly, in order for passive diffusion to occur across the blood brain barrier into the brain cells depleted of B-vitamins. Oral absorption is unreliable, particularly in patients who are nutritionally deficient and those who are continuing to drink alcohol. Administration of glucose increases the metabolic activity in brain cells and may exacerbate or precipitate Wernicke’s in patients with chronic thiamine deficiency. Hepatic encephalopathy is unlikely in the absence of clinical evidence of liver disease. C

Page 153: Masterclass Book Part 2

MOHAMMED IS-HAG 152

105. In a patient with dementing illness, which condition can respond to steroids? A : Alzheimer's B : Lewy body demetia (LBD) C : Hashimotos's encephalopathy D : Vascular dementia E : Normal pressure hydrocephalus (NPH). Comment : Hashimotos's encephalopathy is a rare but potentially treatable cause of a dementing illness. Alzheimers, LBD, and VaD can now be symptomatically helped with medication or NPH might improve with surgery. C 106. A 19-year-old male presents with a history of a single tonic-clonic seizure, the morning after a party. On direct questioning, he also reports the occurrence of occasional blank spells and brief jerking of his upper limbs several times a month. Which of the following drugs is most suitable? A : Phenytoin B : Carbamazepine C : Sodium Valproate D : Gabapentin E : Clonazepam. Comment : The diagnosis here is juvenile myoclonic epilepsy (JME), and valproate is the treatment of choice. The chance of seizure recurrence with this type of epilepsy is high so treatment is warranted. Phenytoin and carbamazepine are useful only for partial seizures or generalised tonic-clonic seizures. They are ineffective, or may in fact worsen, myoclonic jerks and absence seizures. Gabapentin is licensed only for use in partial epilepsy. Clonazepam is effective treatment for myoclonic jerks, but less effective and more sedating than valproate in absences and generalised tonic-clonic seizures. An alternative to valproate in JME, particularly in women of child-bearing age, is lamotrigine. C 107.

In the clinical assessment of a 55-year-old man presenting with a tremor, which features would suggest idiopathic Parkinson’s disease (PD)? A : Symmetrical onset B : Positive family history C : Improvement with alclohol D : Worse when reaching for objects E : Sudden onset of tremor F : Increased tone with distraction G : Progressive difficulty with handwriting H : Tremor restricted to arms I : Dizziness when standing J : Nystagmus on horizontal gaze. Comment : PD is typically asymmetric at onset, and may remain so for many years. Rarely PD is familial, but benigh essential tremor (BET) is typically familial. Improvement with alcohol is more suggestive of BET. Intention tremor suggests cerebellar tremor. Sudden onset tremor suggests cerebrovascular disease. Increased tone (rigidity) is typical of PD. Micrographia is often the presenting complaint in PD. Tremor restricted to arms is more suggestive of BET, and tremor involving the legs is highly suggestive of PD .Dizziness when standing suggests autonomic involvement which would be more suggestive of multi-system atrophy. Nystagmus is not a feature of PD. F.G

Page 154: Masterclass Book Part 2

MOHAMMED IS-HAG 153

108. A 48-year-old man with a long history of alcohol abuse is admitted after being found collapsed in the street. He is agitated and tremulous. You make a diagnosis of alcohol withdrawal. Which one of the following is appropriate management for this condition? A : Nurse in slightly darkened room B : Monitor for hyperglycaemia and control blood glucose using an insulin infusion if necessary C : Give antipsychotics to reduce agitation and likelihood of seizures D : Avoid benzodiazepines to reduce likelihood of respiratory depression E : High-calorie, high-carbohydrate diet. Comment :Important aspects of immediate management of alcohol withdrawal include benzodiazepines (reducing schedule), nursing in well-lit quiet environment, thiamine supplementation, monitoring for and treating hypoglycaemia, rehydration, high-calorie high-carbohydrate diet, and avoiding antipsychotics (which lower seizure threshold). E 109. A 34-year-old woman is brought into A&E after taking an uncertain quantity of paracetamol two hours previously and trying to hang herself. She becomes agitated and insists that she wants to go home immediately. You judge that she is at high risk of suicide. How should you proceed? A : Call the duty psychiatrist, and with other staff in the A&E department attempt to restrain her

under Common Law until they arrive B : Ask her to sign a ‘discharge against medical advice’ form and let her go C : Call the duty psychiatrist, but let the patient go if she insists and the duty psychiatrist does not

arrive in time to see her D : Detain her under section 5(2) of the Mental Health Act E : Call the hospital security services, restrain her and sedate her. Comment : In an A&E department the suicidal patient who declines to be admitted for observation and treatment should be managed as follows: · Ensure that a member of staff stays with them at all times · Call the duty psychiatrist · If they attempt to abscond before or during psychiatric assessment, the staff of the A&E department have a duty under Common Law to restrain the patient If a patient who is already being nursed on medical, surgical or obstetric ward, or in a high dependency or intensive care unit, develops a mental illness (or has an exacerbation of a pre-existing disorder), their physician or surgeon can authorise their compulsory detention for up to 72 hours under section 5(2) of the Mental Health Act. A 110. A middle-aged man is brought by ambulance to the Medical Admissions Unit. He was fitting when picked up and is still having a grand mal convulsion. The most appropriate treatment is: A : Lorazepam 2 mg intravenously B : Fosphenytoin 15 mg/kg body weight phenytoin equivalent, intravenously at a rate of 100-150

mg phenytoin equivalent / min C : Phenytoin 15 mg/kg body weight, intravenously at a rate of 50 mg/min D : Diazepam 10 mg intravenously E : Phenobarbitone 10 mg/kg body weight, intravenously at a rate of 100 mg/min. Comment : All of these are recognized treatments for status epilepticus. First-line treatment should be with intravenous benzodiazepine, with lorazepam preferred to diazepam because of its longer duration of action. Fosphenytoin is the preferred second-line treatment (phenytoin if this is not available). Phenobarbitone is one of several agents that can be used as third-line treatment, but seek specialist advice if first and second-line treatments are ineffective. A 111. A 72-year-woman is admitted after a fall. She lives alone and works in the hospital benevolent society shop. You are called to see her because she is agitated, accusing staff of stealing her jewellery and wanting to leave. Which of the following would be your first intervention? A : Prescribe haloperidol 0,5mg to calm her down and call the psychiatrist on call to assess whether

she should be detained under the Mental Health Act. B : Order an urgent CT head scan. C : Do a physical examination and review the blood and urine test results D : Interview the patient to establish her concerns, take a history, assess her mental state and try to

reassure her. E : Ask the nurses to move her to a quiet, well lit room and try to get a family member to come and

sit with her. Comment : A sudden change in the mental state or sudden onset of behaviour that is out of character in an older person is most likely to be due to an acute confusional state (delirium). Even if a patient is confused it is paramount to take a history, which may give clues to the cause of the delirium. It also gives you the opportunity to assess the severity of confusion. The patient will benefit from reassurance. Of course, don’t dismiss the possibility that the patient is not confused and her jewellery is indeed missing! A physical review and review of investigations must be done as soon as is possible as delirium is a serious condition with a high morbidity and mortality. Sedation should only be used as a last resort and preferably only once the cause of the delirium has been established. D

Page 155: Masterclass Book Part 2

MOHAMMED IS-HAG 154

112. A 43-year-old man is referred after an episode of limb paralysis occurring suddenly at nighttime. He describes waking shortly after falling asleep and being unable to move his limbs or to shout out for help. In addition he describes feeling as if he could not breathe properly. Symptoms lasted for about a minute. His father recently died from a stroke during sleep. He has no other symptoms apart from daytime sleepiness, which he puts down to working long hours. Blood pressure is 140/90 mmHg. There are no other abnormalities on examination. What is the most likely diagnosis? A : Cervical disc prolapse B : Depression and anxiety C : Brainstem transient ischaemic attack D : Nocturnal seizures E : Narcolepsy. Comment : This man is describing sleep paralysis. Sleep paralysis may be isolated, or occur in the context of narcolepsy, in which it affects 15-45% of patients. The symptoms represent the atonia of REM sleep. Awareness is preserved during the attack. Although the respiratory muscles are only ever mildly affected in comparison to the limbs, patients may describe a feeling of suffocation that can be particularly frightening.E: 113.

PLATE 30 A 39-year-old keen camper presents with a two-week history of headaches, a 'droopy face', joint pains and a low grade temperature. He had returned from a holiday in the USA 3 months earlier. The only abnormality on neurological examination is shown (see image). A lumbar puncture was carried out and the results of the CSF analysis are as follows: 200 cells/µl, over 95% lymphocytes, protein 1.4 g/l, glucose 2.0 mmol/l (serum glucose 8.0 mmol/l). What treatment would you institute in this patient? A : Chemotherapy agents B : Intravenous acyclovir C : Intravenous ceftriaxone D : Intravenous gentamicin E : Oral rifampicin, isoniazid, pyrazinamide and ethambutol. The treatment of choice for meningitis of Lyme disease is intravenous penicillin or ceftriaxone. C A GP refers you a 72-year-old man with a history of cardiac failure following detection of a bruit on the right side of the neck. He is a heavy smoker and is on aspirin, frusemide and simvastatin. Investigation with Doppler ultrasound and MRA reveals a 90% stenosis of the right external carotid artery and a 60% stenosis of the left internal carotid artery. His blood pressure is 190/100mmHg. What is the most appropriate management option? A : Right carotid endarterectomy B : Left carotid endarterectomy C : Right carotid stent D : Optimise medical treatment E : Formal carotid angiogram Comment : The 90% stenosis is in the right external carotid artery (ECA) and is probably the cause of the bruit. ECA stenosis does not require surgical intervention. The left ICA disease is asymptomatic, is less than 70% stenosis and should not be considered for intervention. Formal angiography carries approximately a 1% risk of causing a stroke and should only be performed where ultrasound and MRA results are non-concordant, or when MRA is unavailable. Carotid stenting may provide an alternative to endarterectomy in symptomatic, severe ICA stenosis, especially in medically unfit patients such as those with cardiac failure. However, it has not yet been shown in randomised trials to be as safe or effective as

Page 156: Masterclass Book Part 2

MOHAMMED IS-HAG 155

endarterectomy therefore its use should be restricted to ongoing trials. This man should be advised to stop smoking and his blood pressure should be gently lowered to around 140/80 mmHg by the addition of an ACE inhibitor to the diuretic in the first instance. D : 114. A 30-year-old man presents with a three-day history of spontaneous, deep, boring, right shoulder/neck pain, which is followed a few days later by weakness and then wasting of the right deltoid, spinati and triceps muscles. The likely diagnosis is: A : C5/C6 radiculopathy B : Mononeuritis multiplex C : Rotator cuff tendonitis D : Brachial neuritis E : Syringomyelia Idiopathic brachial neuritis/plexopathy is an immune-mediated disorder. Half of the cases are preceded by an upper respiratory tract infection, a flu-like illness, or immunisation. The initial feature is the abrupt onset of unilateral arm pain, usually located in the shoulder. Lessening of pain is associated with evolving weakness, which peaks 2-3 weeks after the onset of pain. Cervical radiculopathy is the main differential diagnosis, however, it is unusual for pain to subside as pain is increasing. Approximately one third recover after 12 months, 75% by 2 years and 89% by the end of 3 years. D : 115. A patient presents with high stepping gait. Which of the following is the most likely diagnosis? A : Cerebellar lesion B : Diffuse cerebrovascular disease C : Parkinson’s disease D : Peripheral neuropathy E : Proximal myopathy. Comment : The high stepping gait reflects bilateral foot drop. D : 116. A 70-year-old man is referred for increasing forgetfulness. On closer questioning, he admits to some urinary incontinence and unsteadiness on walking. He smokes 40 cigarettes a day and has been a heavy drinker in the past. What is most likely diagnosis? A : Alcoholic cerebellar degeneration B : Alzheimer’s disease C : Frontotemporal dementia D : Multi-infarct dementia E : Normal pressure hydrocephalus. Comment : The triad of dementia, urinary incontinence and gait disturbance is classically associated with normal pressure hydrocephalus. E : 117. You are asked to see a 53-year-old publican who was admitted via A&E after being found slumped in her chair. She reports that she is drowsy because she has been assaulted. Her GCS is 13/15. Her pupils and eye movements are normal. She has no focal neurological deficits. There is no evidence of trauma to her face or head. She has had an emergency CT scan that has been verbally reported as normal. She has a raised MCV, ALP and low platelets. The most likely diagnosis is: A : Wernike's encephalopathy B : Subdural haematoma C : Korsakoff's psychosis D : Alcohol withdrawal E : Postictal drowsiness. Comment : The diagnosis is likely to be alcohol related as she has a high-risk profession and abnormal blood tests. Subdural haematoma is usually easily diagnosed by CT scan. She has no evidence of nystagmus or ataxia suggestive of Wernicke's encephalopathy. D and E are possible diagnoses (although withdrawal usually occurs 48-72 hours after the last alcoholic drink and we do not have any timings here), but her confabulation makes Korsakoff's the best diagnosis. Do not forget to exclude hypoglycaemia, hyponatraemia and hypomagnesaemia as causes of fits in alcoholics. C : 118. A patient presents to clinic with intention tremor of the left arm and a wide-based unsteady gait. Where is the lesion likely to be? A : Left pons B : Right globus pallidus C : Right primary cortex D : Right subthalamic nucleus E : Left cerebellar hemisphere. Lesions of the cerebellum lead to motor disabilities ipsilateral to the side of the lesion. The tremor of Parkinson's disease is most prominent at rest, although postural tremors may be present. The gait is shuffling with the centre of gravity shifted forwards beyond the base of the feet.Patients with extensive ischemic small vessel disease may present with a similar gait, often termed 'marche-a-petit-pas.' The steps are short and, because patients are unsteady, the feet may be a little more widely separated than normal. However, the centre of gravity is over the base of the feet. These patients do not usually have tremors, but may have rigidity similar to that seen in idiopathic Parkinson's disease. E :

Page 157: Masterclass Book Part 2

MOHAMMED IS-HAG 156

1. In addition to ceftriaxone, sometimes ampicillin is added to 'blind' antibiotic regimens in cases of meningitis. What organism is this intended to cover in such a case? A : Penicillin-resistant pneumococci B : Meningococcus group B C : Meningococcus group C D : Listeria E : Enterococci. Comment :Listeria can cause disease in the immunosuppressed (including pregnant women). Ampicillin is the drug of choice. A-C are covered well by ceftriaxone and E is not an important cause of meningitis except in special cases. D: A 61-year-old woman presents with tiredness. Her full blood count is as follows: Hb 9.1 g/dl, MCV 84 fl, MCH 28 pg, WBC 10.2 x 10^9/l, platelets 387 x 10^9/l. Which two descriptive terms are correct? A : Hypochromic B : Leucopenic C : Megaloblastic D : Microcytic E : Normochromic F : Normocytic G : Leucocytosis H : Thrombocytopenic I : Macrocytic J : Thrombocythaemic. Comment : The normal ranges are as follows: Hb 12-16 g/dl (women), 13-17 g/dl (men); MCV 82-102 fl, MCH 27-32 pg, WBC 4-11 x 10^9/l, platelets 130-400 x 10^9/l. This patient has a normochromic normocytic anaemia. A common cause for this would be the anaemia of chronic disease, e.g. secondary to rheumatoid arthritis or polymyalgia rheumatica, but it would be wrong to jump immediately to this diagnosis. A blood film should be examined because an MCV and MCH that fall within the normal range could result from two populations of cells – one microcytic and one macrocytic – which would be described as a dimorphic picture. This might result from, for example, combined iron and folate deficiency. E :F: 2. A 78-year-old woman presents with progressive dysphagia to solids, anaemia and 10kg weight loss. Which of the following statements are true? A : The patient is unlikely to be able to perform a barium swallow. B : The patient is most likely to have a foreign body in the oesophagus. C : The anaemia is unlikely to be related to the oesophageal diagnosis. D : Histology is not required to make a diagnosis. E : Surgery is the treatment of choice in the majority of patients. F : Dilatation at initial endoscopy may be required. G : Endoscopic perforation rate is higher in malignant strictures than in benign peptic strictures. H : There is no role for localised radiotherapy. I : A misplaced wall stent can easily be removed and repositioned. J : Overall 5-year survival rate is approximately 20%. Comment : Barium swallow should be the investigation of choice and should be arranged as a matter of urgency in this scenario. Perforation rate is higher with malignant strictures than with benign strictures, and the swallow can guide the endoscopist to the level at which the tumour is likely to be found. Many patients will be anaemic due to slow but constant blood loss from the friable surface of the tumour. Endoscopic biopsies are essential to confirm the diagnosis and differentiate between squamous and adenocarcinomas. Less than a third of patients are amenable to surgery due to advanced disease or comorbidity in this elderly group of patients. In patients with advanced dysphagia, it may be necessary to dilate at the initial biopsy though this carries a risk of perforation. Other palliative measures include the placement of expanding oesophageal wall-stents, laser photocoagulation or radiotherapy (either external beam, or implantation of radioactive wires into the tumour). Once sited, wall stents cannot be removed. Tumour ingrowth may require further stents to be placed within existing ones. The diagnosis of oesophageal carcinoma has a universally poop prognosis with median survival of less than a year, and 5-year survival rates less than 5%.F:G:

Page 158: Masterclass Book Part 2

MOHAMMED IS-HAG 157

3. A 21-year-old chef is admitted with a five day history of bloody diarrhoea, fever and confusion. He is febrile, normotensive, has a pulse rate of 110, and has a soft diffusely tender abdomen. Blood results: haemoglobin 7.2 g/dL, white cell count 22, platelets 20, creatinine 380 mmol/L. Which one of the following statements is NOT true: A : A blood film may show fragmented red blood cells. B : This syndrome could be caused by Escherichia coli or shigella infection. C : The diagnosis has implications for the patient’s return to work. D : There is an animal reservoir for the infection. E : Most cases seen in the UK occur in association with outbreaks. Comment : The chef has hemolytic uraemic syndrome(HUS),most likely as a consequence of E.coli 0157:H7 infection. HUS is a triad of renal failure, thrombocytopaenia and microangiopathic haemolytic anaemia +/- central nervous system symptoms (TTP). E. coli O157:H7 and Shigella dysenteriae type I are the organisms firmly associated with the syndrome, though E. coli accounts for 85-95% of infective cases. The overall rate of HUS with a case of E. coli O157:H7 infection with bloody diarrhoea is only 5-10%.Cattle have been shown to be a reservoir of infection, and people whose occupation brings them into contact with farm animals are at increased risk. Most cases in the UK are sporadic in nature but outbreaks have been associated with contaminated meat and dairy products.The infection has public health implications, particularly in this situation when the patient is a food handler. He should be excluded from work until two consecutive negative faecal specimens taken after recovery have been obtained. E : 4. A 38-year-old smoker presents with 4 month's diarrhoea, abdominal pain and weight loss. She has noticed uncomfortable red marks on her legs. Which of the following statements is true? A : Crohn’s disease is more likely than ulcerative colitis. B : Nicotine patches may be of therapeutic benefit. C : A stool culture is not necessary in the absence of recent foreign travel. D : Fistulae require surgical intervention in Crohn’s disease. E : Dietary restriction an important part of the treatment Comment : The relative risk of Crohn’s disease is three-fold greater in smokers. A similar association exists for ulcerative colitis and non-smoking such that nicotine patches have been used in mild to moderately active disease. Infectious colitis can present with identical symptoms and macroscopic appearance as idiopathic inflammatory bowel disease. Whilst surgery may have an important role, anti-tumour necrosis factor antibody is licensed for use in both refractory and fistulating Crohn’s disease.Malnutrition is a predictor of poor outcome in many conditions, including Crohn’s disease. Even if an elemental diet is not the chosen therapeutic modality, nutritional support should form an important part of management. A : 5. A 23-year-old woman is referred to clinic because her GP thinks she has developed acute hepatitis B. Which of the following investigations done when she became jaundiced is consistent with this diagnosis? A : HB core antibody positive and surface antibody positive B : HbsAb positive and core antibody negative C : HbsAg and HBeAg positive D : An alkaline phosphatase of 800 IU/l and an ALT of 50 IU/l E : HBs Ag positive and e antibody positive. Comment : At the time of clinical acute hepatitis B with onset of jaundice, there is active viral replication with elevated HBV DNA levels and presence of e antigen and surface antigen. High serum transaminase levels are common although a cholestatic phase with high alkaline phosphatase levels can occur following the transaminitis in 5%. Subsequently e antibodies develop followed by surface antibodies in the majority of individuals who clear the virus. Evidence of previous infection is indicated by the presence of antibodies to core protein and surface protein. Those individuals who have been immunised will have antibodies to the surface protein only. C : 6. A 45-year-old man with known cryptogenic cirrhosis has developed tense ascites despite diuretics and you are asked to perform a paracentesis. Which two of the following statements are true? A : The risk of large bowel perforation is 1%. B : No more than 10 litres of fluid should be drained per session. C : 6-8g of albumin should be infused for every 1 litre of fluid drained. D : Saline is an alternative replacement fluid to albumin. E : Ultrasound guidance is usually needed. F : Leakage from the site of the paracentesis is uncommon. G : Repeat paracentesis is not as good as a Levine shunt in controlling tense ascites. H : The drain should generally be removed within 6 hours of insertion. I : The patient requires overnight stay in hospital. J : Portal vein thrombosis is not a recognised cause.

Page 159: Masterclass Book Part 2

MOHAMMED IS-HAG 158

Comment : Paracentesis is an effective treatment for tense ascites. There is no limit to the amount of fluid that can be drained providing that albumin is infused at the same time as this prevents the development of renal failure. Albumin is more effective than crystalloids in preventing renal impairment following paracentesis. Ultrasound guidance is usually only needed after repeated paracentesis when the fluid can become loculated. Prompt removal of the drain prevents secondary peritonitis and the procedure can be safely performed as a day case. Both hepatocellular carcinoma and portal vein thrombosis should be excluded by ultrasound in a cirrhotic who develops worsening ascites. C:H: 7. A 28-year-old man 1 year post stem cell transplant for acute leukaemia presents with increasing confusion, jaundice and abdominal pain. Pertinent results include an ALT of 4300IU/l, AST of 3500IU/l and GGT of 870IU/l. Imaging is unremarkable and synthetic function was significantly impaired. The patient was treated for liver failure but, which cause is most likely out of the following? A : Graft versus host disease B : Alcoholic hepatitis C : Primary sclerosing cholangitis D : Herpes simplex hepatitis E : Rhabdomyolysis. Comment : Levels of ALT or AST as high as this generally indicate the presence of marked hepatocellular injury from medications (e.g. paracetamol) or toxins (e.g. herbal remedies or mushroom toxicity), viruses, or ischaemia. In an immunocompromised patient such as this, one must consider unusual causes of hepatitis. In particular cytomegalovirus, Epstein–Barr virus, and herpes simplex viruses (HSV) 1, 2, and 6 have all been known to cause severe acute hepatitis. Imaging in such patients must specifically include Doppler studies of the portal and hepatic veins in order not to miss portal vein thrombosis or a Budd–Chiari syndrome (hepatic vein thrombosis). D : 8. A sprightly 79-year-old woman is referred to your clinic with a 2-month history of diarrhoea. Which of the following statements is correct? A : An infective cause is likely. B : Negative anti endomysial antibodies rule out coeliac disease. C : Nocturnal diarrhoea suggests irritable bowel syndrome. D : Her cholecystectomy 3 months earlier may be the cause. E : Positive culture (>10 6 organisms per ml) of a small bowel aspirate is diagnostic of the syndrome

of bacterial overgrowth. Comment : Although it is not understood why, a surprising 10% of patients develop diarrhoea after a cholecystectomy. Nocturnal diarrhoea suggests an organic cause.It is rare to identify an infective cause for diarrhoea when the symptoms have lasted for more than 4 weeks. Antiendomysial antibodies (to tissue transglutaminase) are the single most reliable antibody test for coeliac disease, but there are false negatives. Although a positive culture of a small bowel aspirate is almost diagnostic, this can occur in healthy individuals. D : 9.

PLATE 31 An elderly patient is admitted with 5 days diarrhoea. Sigmoidoscopy appearance is shown (see image). Which of the following statements is true? A : This is more typical of ulcerative colitis than Campylobacter infection. B : Relapse after antibiotic treatment of Clostridium difficile infection is rare. C : If this patient is known to have ulcerative colitis, no stool culture is necessary. D : The presence of erythema nodosum means that the diagnosis is ulcerative colitis. E : Enterohaemorrhagic E. coli is associated with haemorrhagic colitis and haemolytic uraemic

syndrome.

Page 160: Masterclass Book Part 2

MOHAMMED IS-HAG 159

Comment : Sigmoidoscopic appearance of infectious colitis and chronic idiopathic inflammatory bowel disease may be indistinguishable. The mucosal appearance of Clostridium difficile infection is also variable, although the adherent yellow-white plaques, or ‘pseudomembrane’, is characteristic. Relapse after treatment of Clostridium difficile is common, occurring in 30% of cases. Erythema nodosum may occur with Campylobacter infection. E : 10. A 45-year-old man with a past history of chronic alcohol abuse presents is brought to A&E after vomiting one litre of fresh red blood. Blood pressure on arrival is 90/40 with pulse 110 /min. He is noted to be jaundiced, with multiple spider naevi and to have marked ascites. Which of the following drugs is most likely to be beneficial in his initial treatment? A : Intravenous ranitidine B : Oral propranolol C : Intravenous terlipressin D : Intravenous proton pump inhibitor E : Helicobacter eradication therapy. Comment : This patient has evidence of chronic liver disease with portal hypertension, so that a diagnosis of bleeing oesophageal varices should be strongly considered. Intravenous terlipressin has been shown to reduce variceal pressure following bleeding, which may influence prognosis. Propranolol is beneficial in primary and secondary prevention of variceal haemorrhage in the long term, but has no role in the acute setting. Intravenous proton pump inhibitors may be beneficial in patients with actively bleeding peptic ulceration, but have no role in variceal bleeding. Intravenous ranitidine has never been shown to affect outcome following GI bleeding of any type, while eradication of helicobacter will only be important in the short term if he is shown at endoscopy to have peptic ulceration as a cause for his bleeding. C : 11. In a patient with a bullous rash and altered bowel habit, which of the following statements regarding tissue transglutaminase (tTG) are relevant? A : Coeliac disease is associated with mutations in the promoter region of the tTG gene. B : Overexpression of the tTG gene in epithelial cells leads to activation of gluten peptides. C : IgA antibodies react with a form of this enzyme in the skin, causing dermatitis herpetiformis. D : An ELISA test for IgA antibodies against tTG is easier to perform than immunofluorescence,

although it has a lower sensitivity and specificity. E : Antibodies to tTG are a marker for coeliac disease, although the protein probably has no

pathogenic role in the disease. F : Antibody titres against tTG decline as the disease is controlled. G : Dapsone inhibits activity of the tTG enzyme. H : Antibodies to tTG are probably a false-positive as the patient has Crohn’s disease. I : Antibodies to tTG are likely to be suppressed initially, and will appear in the chronic phase of the

illness. J : Antibodies to tTG are irrelevant as this is a classical presentation of glucagonoma. Comment : tTG antibodies are a reliable marker for active coeliac disease, and modern enzyme-linked immunosorbent assay (ELISA) tests have sensitivities and specifities approaching 100%. tTG may play a pathogenic role in coeliac disease, although so far no inherited mutations associated with the disease have been discovered. Dermatitis herpetiformis, a bullous rash that is associated with coeliac disease, may be caused by antibody reaction against a form of transglutaminase expressed in the skin. Dapsone is effective against dermatitis herpetiformis, but not against coeliac disease. Gluten peptides are modified by tTG, creating immunodominant epitopes that stimulate intestinal epithelial T cell activation and proliferation. C : F: 12. A 60-year-old man with known ulcerative colitis and diverticular disease comes to clinic complaining of passing faeces per urethra. Cystoscopy confirms a fistula between his bladder and bowel. Which treatment is most likely to be effective? A : total parenteral nutrition (TPN) B : steroids C : antibiotics D : surgery E : elemental diet. Comment : These cases are difficult to manage. Ulcerative colitis is less likely to cause enterovesical fistulae than is Crohn’s disease, so the fistula in this case is probably due to diverticular disease. Malignancy and ischaemia need also to be borne in mind. Treatment may be medical, surgical or a combination, but definitive surgery is usually favoured where possible. Diagnosis is best made in liaison with the urologists as well as gastrointestinal surgeons and often involves cystoscopy, CT scanning as well as careful lower GI investigations. D :

Page 161: Masterclass Book Part 2

MOHAMMED IS-HAG 160

13. A 30-year-old male recently returned from trekking in Nepal presents with a 6-day history of bloody diarrhoea with abdominal cramps but no fevers. He has taken some antibiotics, obtained in Nepal, with little effect. Which of the following would be an unlikely cause? A : Entamoeba histolytica B : Trichuris C : Acute schistosomiasis D : Ulcerative colitis E : Clostridium difficile colitis. Comment : Schistosomiasis is not endemic in the Indian subcontinent, though would be a cause of the above elsewhere in the tropics. Heavy worm loads with Trichuris can cause bloody diarrhoea. Cosmopolitan causes of bloody diarrhoea should always always be considered as a cause despite the travel history and C. difficile disease excluded when antibiotics have been taken. C : 14. A 50-year-old man with alcoholic cirrhosis is admitted with diuretic resistant ascites. Initial investigations show Na 127 mmol/l, creatinine 90micromol/l, bilirubin 110 micromol/l, prothrombin time 18 seconds, albumin 27g/l. Which one of the following drugs should be avoided? A : Flucloxacillin B : Non-steroidal anti-inflammatory drugs (NSAIDs) C : Paracetamol D : Rifampicin E : Chlordiazeopoxide. Comment :Once diuretic resistant ascites has occurred, blood flow to the kidney is reduced and it is easy to precipitate hepatorenal failure. NSAIDs do this by further reducing renal blood flow and should be avoided. Paracetamol is safe in a dose not exceeding 4 grams/day.Rifampicin can cause cholestasis when used alone. However it is a useful drug for treating pruritus in biliary diseases when cholestyramine does not work and can be used in patients with end-stage liver disease providing liver biochemistry is monitored. Although chlordiazepoxide may accumulate causing encephalopathy, alcoholic withdrawal must be treated/prevented in a cirrhotic who is actively drinking on admission. Doses must be titrated on a day-to-day basis. B : 15. A previously healthy patient of 24 years of age presents to hospital complaining of 12 hours of epigastric discomfort and vomiting. Blood tests reveal an Alanine Transaminase of 8,400 (normal range <45), Bilirubin 24 (normal <15) and INR 2.8. What is the most likely diagnosis? A : Gilbert's syndrome B : Acute Hepatitis A C : Epstein Barr Virus (EBV) Hepatitis D : Alcoholic Hepatitis E : Undisclosed paracetamol overdose. Comment : The marked elevation of the INR in the context of the raised ALT and relatively normal bilirubin indicated that this patient is likely to be developing fulminant hepatic failure. Idiosyncratic drug reactions (including recreational drugs such as 'ecstasy') and paracetamol overdose are the commonest cause for this presentation. Fulminant hepatic failure rarely occurs with Hepatitis A and EBV hepatitis, although the short history and the relatively normal bilirubin make this very unlikely in this case. Alcoholic hepatitis may produce an elevated INR, usually as a result of underlying chronic liver disease or cholestasis, but the ALT is rarely more than 200. Gilbert's syndrome causes isolated elevation of bilirubin and is usually asymptomatic. E : 16. You are asked to review a 45-year-old man recovering from a road traffic accident, just off intensive care. Although able to eat and drink small amounts he remains generally weak and has lost 20% of his pre-morbid weight. His bowels are working normally. The orthopaedic team ask your advice regarding his nutrition. Do you suggest: A : Regular monitoring of his weights only taking action if he fails to put weight on B : Insertion of a tunnelled central venous line for parenteral nutrition C : Booking him for percutaneous gastrostomy (PEG) at the earliest available opportunity D : Oral dietary supplements E : Insertion of a naso-gastric tube and commencement of supplemental enteral nutrition as soon as

possible Comment : Malnutrition is common in hospital and impairs recovery from illness. This gentleman is malnourished and recovering from major trauma and some form of nutrition is necessary. He has evidence that his GI tract is working and therefore short-term enteral nutrition is appropriate and whilst supplements may be sufficient it is more likely that intensive nutrition via a naso-gastric tube is going to be needed to meet his increased nutritional demands. It is inappropriate to simply monitor him and, in view of a functioning GI tract, not appropriate to choose parenteral nutrition. The nutrition is needed short-term and therefore a PEG is not required. E :

Page 162: Masterclass Book Part 2

MOHAMMED IS-HAG 161

17. A previously fit 35-year-old man presents with pyrexia and tachycardia, severe bloody diarrhoea and mucus per rectum. His symptoms have been present for three weeks. The most likely diagnosis is: A : ischaemic colitis B : bacterial colitis C : ulcerative colitis D : protozoal colitis E : non-steroidal anti-inflammatory drug (NSAID) induced colitis. Comment : Ulcerative colitis is most likely. An infectious colitis is less likely if the symptoms have been present for more than one week, but patients with acute severe colitis should be treated with oral ciprofloxacin to cover bacterial colitis until the diagnosis is clear. Ischaemic colitis would be very unusual at this age and as he was previously fit significant NSAID consumption is not likely. C : 18. A 74-year-old man with a past history of myocardial infarction and intermittent claudication is admitted with severe central abdominal pain. He looks unwell, with a tachycardia, but the abdomen is soft, without peritonism. His WBC is elevated at 15.8 and he is acidotic (pH 7.22). The most likely diagnosis is: A : Posterior perforation of peptic ulcer B : Appendicitis C : Myocardial infarction D : Pancreatitis E : Mesenteric ischaemia. Comment : Mesenteric infarction manifests as an acute abdomen in the later stages, but earlier on there is often severe abdominal pain with little in the way of abdominal signs. In a patient with abdominal pain but few signs an elevated WBC count and the presence of acidosis should always raise the possibility of mesenteric ischaemia / bowel infarction, especially if the patient is an arteriopath, as in this case. E : 19. A 43-year-old man presents with an acute colitic illness and is passing between five and 10 bloody stools per day. Which one of the following statements is true? A : Steroids are contraindicated until negative stool culture results have been obtained. B : Approximately 30% of cases of acute severe ulcerative colitis (UC) occur in newly diagnosed

cases. C : Clostridium difficile always produces characteristic endoscopic and histological changes. D : The finding of cytomegalovirus (CMV) inclusion bodies in biopsy samples is conclusive evidence

for a pathogenic role and mandates intravenous ganciclovir instead of intravenous corticosteroid. E : Amoebic dysentery can be safely discounted as a possible cause five years after last travel in the

tropics Comment : Of patients experiencing their first attack of UC, 20% fulfil the criteria for severe disease. At first presentation or at the time of a subsequent flare of acute colitis it is always important to check stool cultures for standard bacterial pathogens including Salmonella, Campylobacter, and Shigella as well as Clostridium difficile. However, in the context of severe disease, it is not always possible to withhold steroids until negative cultures have been obtained, and steroid therapy does not appear to adversely affect outcome even when stool cultures come out positive. Indeed, infective colitis can trigger an episode of inflammatory bowel disease. While Clostridium difficile may produce the characteristic pseudomembranes on macroscopic appearance, it does not always do so. The histopathologist can usually differentiate idiopathic IBD from C. diff but the gold standard remains detection of C. diff toxin on stool assay.Differentiating CMV colitis from ulcerative colitis in which CMV inclusion bodies occur as “bystanders” is virtually impossible. For patients with pre-existing IBD therapeutic immunosuppression may increase the risk of CMV infection. Where CMV inclusion bodies are found or special stains demonstrate the presence of CMV antigen in the gut mucosa, it is usually considered prudent to give intravenous ganciclovir. However, in patients with known ulcerative colitis it is also necessary to treat concomitantly with intravenous steroids. Amoebic dysentery usually occurs in people living in or recently returned from the tropics. However, asymptomatic individuals can continue to excrete cysts for many years, and this is the case for a small proportion of people living in Europe. Diagnosis is usually by microscopy of a hot stool to identify amoebae which have phagocytosed red blood cells. These may also be seen on biopsy specimens from the colon. Treatment is metronidazole 800 mg three times a day for 5-10 days, followed by diloxanide. B : 20. A 74-year-old woman presents with a history of pruritus and lethargy. On investigation she is found to have a mildly elevated alkaline phosphatase and a positive anti-mitochondrial antibody. A : The level of the alkaline phosphatase correlates well with the extent of hepatic fibrosis B : IgA levels are likely to be elevated C : Liver transplantation has a poor outcome for this condition D : Azathioprine has been shown to be an effective treatment E : The patient should undergo bone mineral densitometry.

Page 163: Masterclass Book Part 2

MOHAMMED IS-HAG 162

Comment : Liver functon tests correlate poorly with histology in PBC – the disease may progress insidiously with normal or near-normal LFTs. Classically IgM levels are elevated. Liver transplantation has a good prognosis (90–95% survival) in this condition, though many patients are too old for consideration at the time of presentation. No benefit has been demonstrated in clinical trials of steroids, azathioprine or methotrexate. Osteoporosis is a common complication, possibly due to vitamin D malabsorption and/or premature ovarian failure. All patients with PBC should be screened for the condition. E : 21. A middle-aged man presents with progressive difficulty in swallowing. There is no preceding history of heartburn. He has lost weight, and is worried about having cancer. Which of the following statements are true? A : You share his worry about cancer, although you maintain that the symptoms may be due to non-malignant . disease B : Barrett’s oesophagus and adenocarcinoma are particularly unlikely given the lack of preceding

gastroesophageal reflux. C : To expedite management, a one-stop clinic offering immediate endoscopy as the next step, and

subsequently appropriate initial treatment, would be ideal. D : Neurological disorders such as motor neurone disease may also cause difficulty with swallowing,

although they are not associated with weight loss. E : As the patient does not volunteer heartburn or odynophagia, peptic stricture is unlikely. F : Oesophageal dysmotility syndromes can cause dysphagia, and present exactly like this. G : A high resolution CT scan of the thorax is now probably the best initial investigational modality. H : Sadly, at this stage the outlook is hopeless. I : Globus, a functional GI disorder, may also present in this way. J : Although oesophageal cancer is a possible diagnosis, this is unlikely if the patient is a native of

the United Kingdom, as the incidence of this cancer, which is rare in any case, is falling. Comment : The most common causes of dysphagia, which is a serious symptom that should prompt a rapid diagnostic and therapeutic response, are peptic strictures, carcinoma of the oesophagus or cardia, and oesophageal dysmotility. Neurological disease may also cause difficulty in swallowing, and consequent weight loss, often associated with choking, coughing, and regurgitation. Globus, which describes the sensation of a lump in the throat, is not associated with weight loss. In middle-aged men, Barrett’s oesophagus and adenocarcinoma seem to be increasing in incidence, and the conditions are often not preceded by a history of heartburn. This makes it harder to rationally select patients for screening programmes. Endoscopy performed before barium swallow or some other non-invasive means of determining the oesophageal anatomy can be disastrous, as a friable tumour may be perforated on endoscopy. Although CT or MRI scans provide important information on the potential spread of disease beyond the oesophagus, or involvement of the oesophagus by thoracic disease, they are not first-line investigations. Barium swallow, endoscopy, and oesophageal manometry are the most helpful initial investigations, as they will provide a diagnosis. Dysmotility syndromes are amenable to endoscopic therapy, and some early neoplasms can be treated successfully, or palliated remarkably by a combination of surgery, endoscopic therapy, and supportive care. Unfortunately, while squamous cell carcinoma of the oesophagus is rare in the UK, adenocarcinoma of the oesophagus, associated with Barrett’s oesophagus, seems to be increasing in incidence among men in the Western world. A :F: 22. A 26-year-old woman with Crohn’s disease is 8 weeks pregnant. She asks several questions about her medications and the effect of the disease on her unborn child. Which of the following are true? A : She should stop her azathioprine. B : She should stop her methotrexate. C : She should avoid prednisolone if the disease becomes active. D : Daily folic acid is recommended. E : She should stop her mesalazine. F : Flexible sigmoidoscopy is contraindicated. G : Premature births are less common. H : Smoking has an adverse effect on the baby but not Crohn's disease. I : She should avoid breast feeding if taking prednisolone. J : She should have a caesarian section at term. Comment : There is a slight increase in preterm and low birth weight of babies born to mothers with inflammatory bowel disease. Standard doses (up to 3g/day) of 5-aminosalicylic acid drugs appear safe in pregnancy. There appears to be no increased risk to mother and baby of continuing azathioprine during pregnancy, although studies in inflammatory bowel disease are small. If the patient has been well for some years, the physician may discuss withdrawal of the drug before conception. Steroids used for exacerbations of disease are also probably safe. Prednisolone is largely metabolised by the placenta and whilst it is present in breast milk, the concentration is low. The risks to mother and baby of active inflammatory bowel disease during pregnancy outweigh the risks of these treatments. Methotrexate is teratogenic and should not be used in pregnancy. Smoking is associated with Crohn's disease and continuing smoking adversely affects prognosis. B : D :

Page 164: Masterclass Book Part 2

MOHAMMED IS-HAG 163

23. A 65-year-old man complains of epigastric pain and difficulty swallowing. At endoscopy Grade I reflux oesophagitis is seen. Which one of the following statements is NOT true? A : Acid suppression with a proton pump inhibitor should relieve symptoms. B : Repeat endoscopy is required to ascertain response to treatment. C : Helicobacter eradication is not needed unless there is associated duodenitis. D : Raising the head of the bed and avoiding eating/drinking 3 hours prior to going to bed can aid

symptoms. E : The lowest dose of proton pump inhibitor should be used in the long term to treat his

symptoms. Comment : Reflux oesophagitis is very common and easily treated with simple postural measures and acid suppression.Most patients respond to treatment and there is no need to repeat endoscopy in every case. Severe disease, presence of oesophageal ulceration and follow up of Barrett’s oesophagitis are reasonable indications for repeat endoscopy.There is no convincing evidence for routine Helicobacter eradication in this clinical situation and the lowest dose of acid suppression should be given when symptoms have settled, if any is required at all. B : 24. A 40-year-old man is found to be Hepatitis B surface antigen and e antigen positive. He says he was told he had acute hepatitis B infection 10 years before when he became jaundiced. Liver biopsy reveals moderate hepatic inflammation and fibrosis. Which two of the following drugs could be used singly in his treatment? A : Zidovudine B : Ritonavir C : Ribavirin D : Interferon gamma E : Interferon alpha F : Efavirenz G : Lamivudine H : Acyclovir I : Interleukin-2 J : Azithromycin. Comment : Interferon alpha and lamivudine are both individually licensed for treatment of chronic hepatitis B. Two other drugs not currently licensed that have potent activity against chronic hepatitis B include adefovir and tenofovir. E :G: 25. A 70-year-old lady presents with watery diarrhoea. Flexible sigmoidoscopy is reported as normal but histology shows a lymphocytic infiltration. Which of the following best describes optimum management and complications? A : The patient should be warned about an increased risk of colorectal carcinoma. B : Non-steroidal anti-inflammatory drugs (NSAIDs) are unlikely to have caused her symptoms and

can safely be continued. C : Toxic megacolon is a common complication. D : Budesonide is effective treatment. E : Collagenous colitis has a similar microscopic appearance. Comment : All patients who have diarrhoea and undergo flexible sigmoidoscopy or colonoscopy should have colon biopsies taken to exclude microscopic colitis, which is characterised by a normal appearance to the mucosa but a lymphocytic infiltration histologically. It is associated with NSAID use and unlike classical ulcerative colitis is rarely associated with toxic megacolon. Although cholesytramine and sulphasalazine have been used effectively as treatment, budesonide is the only drug shown to be effective in a controlled clinical trial. Collagenous colitis overlaps with this condition in that the clinical symptoms are similar but microscopically there is a layer of collagen beneath the submucosa rather than lymphocytic infiltration. D : 26. A 42-year-old man presents with abdominal discomfort, bloating and a change in bowel habit over the last 8 months. He may pass one formed stool or up to six loose stools per day. The discomfort tends to occur when he has the diarrhoea and is eased following defaecation. Which of the following statements is true? A : He fulfills the Rome II criteria for the diagnosis of irritable bowel syndrome (IBS). B : No further investigation is needed. C : Coeliac disease is unlikely in the absence of weight loss. D : His father had colon cancer aged 40, giving the patient a lifetime risk of 1:40. E : He is very likely to improve with a high fibre diet. Comment : Investigation should be tailored to the individual. Genetic inheritance has a role in coeliac disease, colon cancer (in which the lifetime risk is 1:10 if a first degree family member presents before the age of 45) and inflammatory bowel disease, all of which can occasionally present with similar symptoms. About half of IBS patients worsen with a high fibre diet. A :

Page 165: Masterclass Book Part 2

MOHAMMED IS-HAG 164

27. A 28-year-old man complains of a 3-year history of difficulty swallowing both solids and liquids. A barium meal shows a slightly dilated oesophagus and subsequent endoscopy is normal apart from oesophageal candidiasis. The next best investigation is: A : anti-Ro and anti-La antibodies B : fasting glucose C : oesophageal manometry D : CT thorax E : the edrophonium chloride (Tensilon) test. Comment : A history of dysphagia to both liquids and solids is fairly typical for achalasia. The barium meal findings are consistent, though not diagnostic, and endoscopy has ruled out a peptic oesophageal stricture. Confirmation of the diagnosis is by oeosphageal manometry, which shows a high resting lower oesophageal sphincter pressure that fails to relax on swallowing. Anti-Ro/La antibodies are positive in scleroderma, which may affect the oesophagus causing dysmotility. Oesophageal candidiasis may occur in achalasia or diabetes but the latter should not cause a dilated oesophagus. The Tensilon test is a diagnostic test for myasthenia gravis, which may present with difficulty swallowing. C : 28. An 82-year-old lady is admitted from her residential home with an ischaemic stroke. Initial examination showed her to be underweight. Her swallowing was impaired and she was put on IV fluids for 2 weeks before feeding was introduced via a nasogastric tube. Subsequent examination revealed further weight loss and a body index of less than 18. Which of the following statements is correct? A : Protein energy malnutrition (PEM) does not increase mortality in patients with stroke. B : The rate of weight loss is independent of the catabolic state of the patient. C : Lean body mass in older people is usually increased with age. D : Appetite is reduced with age. E : Stroke volume is increased in patients with PEM. Comment : PEM increases morbidity and mortality in older patients but especially those with congestive cardiac failure and stroke. The rate of weight loss is increased in patients who are ill with an increased catabolic state.Lean body mass in older people is usually decreased with age. Appetite is reduced with age Heart rate and stroke volume are decreased with PEM causing an increased circulatory time. D : 29. A 34-year-old HIV positive man, poorly compliant with treatment, presents to the gastroenterologists with diarrhoea. Sigmoidoscopy revealed mucosal inflammation with focal haemorrhage, oedematous folds and polypoid lesions. Inclusion bodies were visible on examination of a biopsy. What is the most likely cause of his symptoms and signs? A : Cytomegalovirus (CMV) colitis B : Cryptosporidiosis C : Amoebiasis D : Disseminated mycobacterium avium intracellulare (MAI) E : Ulcerative colitis. Comment : The broad differential diagnosis for HIV associated diarrhoea includes: 1. Bacterial infections (Salmonella, Shigella, Campylobacter) 2. Parasitic infections (Cryptosporidium, Isospora, Giardia, Microsporidia, Entamoeba histolytica) 3. Mycobacterial infections (Mycobacterium avium complex (MAC), Mycobacterium tuberculosis) 4. Viral infections (Cytomegalovirus) 5. Drug-associated diarrhoea (e.g. nelfinavir) 6. Idiopathic diarrhoea (HIV enteropathy). CMV colitis is uncommon in patients who are not severely immunocompromized. GI tract involvement may occur alone or in the setting of disseminated disease. CMV has three major patterns of infection: primary infection (in the immunocompetent host causing few or no symptoms; after initial infection, latent viral infection occurs), secondary reactivation (patients seropositive with latent infection who reactivate because of immunodeficiency) and superinfection. Within the colon, ulcerative changes may be seen and watery diarrhoea can develop. As ulcers increase in depth, erosion into blood vessels may cause profuse bloody diarrhoea. Inflammatory polyps do develop, which, rarely, obstruct the colon. Severe inflammation and vasculitis sometimes leads to ischaemia and transmural necrosis, with subsequent perforation and peritonitis. Classically findings on microscopy include giant cells with large ovoid or pleomorphic nuclei containing basophilic inclusions (owl's eyes). A : 30. Which of the following conditions are unlikely to present with a 3-week history of loose bloody stool in a traveller? A : Ulcerative colitis B : Colonic malignancy C : Intestinal amoebiasis D : Giardiasis E : Schistosoma mansoni.

Page 166: Masterclass Book Part 2

MOHAMMED IS-HAG 165

Comment :Giardiasis presents with a small-bowel diarrhoea, foul flatus and burping, bloating and nausea. It is not associated with bloody diarrhoea. It is important to remember that the majority of problems a returning traveller will present with reflect underlying disease processes rather than exotic conditions acquired abroad; this is becoming increasingly important with an ageing population travelling overseas.D: 31. A 58-year-old man presents with diarrhoea and weight loss. Amongst many investigations he has a lactulose breath test. The reason for the test is to detect: A : diarrhoea due to laxative consumption B : bacterial overgrowth in the small bowel C : hypolactasia D : infection with H pylori E : malabsorption due to small intestinal disease. Comment : Bacterial fermentation of an oral dose of the carbohydrate lactulose releases hydrogen. If there is an increase in bacteria in the upper small bowel, or if there is rapid intestinal transit, then an early peak in hydrogen can be detected in expired air. An oral dose of conjugated bile acid, often glycine-glycocholate, with radiolabelled carbon in the amino portion, can be given as an alternative to lactulose. Bacterial action releases radiolabelled glycine, detected following metabolism as labelled carbon dioxide in the breath. What functional tests would you use to detect hypolactasia, H pylori infection or malabsorption due to small intestinal disease? B : 32. A 50-year-old man presents to you with jaundice, anorexia, cachexia and mild abdominal discomfort. He is a heavy smoker and has spent much of his youth in East Asia. In the past he had a total colectomy for ulcerative colitis. Ultrasound of the abdomen demonstrates a normal pancreas, intrahepatic duct dilatation and a perihilar mass. Which of these statements are true regarding cholangiocarcinoma? A : The best diagnostic investigation is an endoscopic retrograde chlangiopancreatography (ERCP) or percutaneous transhepatic chlangiography (PTC) and biopsy. B : Heavy smoking is a recognised risk factor. C : Ulcerative colitis is a recognised risk factor. D : His travel history is irrelevant. E : It is associated with an elevated alpha-fetoprotein. F : Portal vein chemotherapy is the treatment of choice. G : Systemic chemotherapy is the treatment of choice. H : Only a minority of the tumours are adenocarcinoma. I : The average age at presentation is 50-55 years. J : Perihilar tumours are the least common. Comment : Cholangiocarcinoma occurs anywhere in the biliary tree; 95% are adenocarcinomas. They may be divided into three anatomical groups: perihilar (~70%), distal, intrahepatic tumours. The management of intrahepatic tumours is surgical resection, distal tumours are managed with a pancreaticoduodenectomy and perihilar tumours may be managed with both operative and non-operative interventions. The average age at presentation is between 60-65 years, with a slight preponderance in men, 1.3:1. Risk factors include: ulcerative colitis, sclerosing cholangitis, nitrosamines, Thorotrast, isoniazid, methyldopa, oral contraceptives, liver flukes (e.g. Clonorchis sinensis & Opisthorchis viverrini) in East Asia and Thailand. Cigarette-smoking is a risk factor which hasn’t, as yet, been implicated. More than 90% present with jaundice; pruritus, abdominal pain and cachexia are less common. Serum CEA and AFP are usually normal, Ca19-9 may be elevated. Ultrasound examination or CT scan help to delineate the tumour extent, including involvement of hilar vessels, liver and distant metastases. Documentation of bile duct dilatation and anatomy is defined through either percutaneous transhepatic (PTC) or endoscopic retrograde cholangiography (ERC). Tissue diagnosis can be established via percutaneous needle aspiration, brush or scrape biopsy or cytology of the bile. The type of surgery is dependent on the site and extent of the tumour. Trials of chemotherapy have been limited and small, and not proven to enhance survival in patients. Chemo-radiotherapy has been used in selected patients although the patient groups have included those with resected as well as unresected tumours. A : C : 33. A 21-year-old student develops abdominal pain, fever and diarrhoea 12 hours after eating a chicken sandwich that had been in his fridge for a week. The most likely cause is: A : Campylobacter B : Norwalk virus C : Salmonella D : E coli O157 E : Giardia.

Page 167: Masterclass Book Part 2

MOHAMMED IS-HAG 166

Comment :Campylobacter is the commonest cause of bacterial gastroenteritis in the UK, but its incubation period is three to five days. By contrast, the incubation period of salmonella> is eight to 48 hours.Treatment of salmonella gastroenteritis is symptomatic. If the patient is very ill, then ciprofloxacin is the antibiotic of choice, but antibiotics prolong the duration of stool carriage and should not be given routinely. C : 34. A 24-year-old man presents with a 6-month history of watery diarrhoea with blood, weight loss and arthralgia. The two most likely diagnoses would be: A : Bacterial overgrowth B : Crohn's disease C : Pancreatic carcinoma D : Antibiotic-induced diarrhoea E : Ulcerative colitis F : Caecal carcinoma G : Irritable bowel syndrome H : Giardiasis I : Tropical sprue J : Laxative abuse. Comment : While any of the conditions in the list can present as diarrhoea, the presence of blood in the stool and arthralgia is suggestive of inflammatory bowel disease. Bowel cancer can also present with diarrhoea and rectal bleeding but is much more rare in this age group. B : E : 35. Wilson's disease is: A : acommon inherited disorder of copper metabolism that responds to regular venesection B : arare disorder of copper metabolism that presents with cardiac and endocrine dysfunction C : associated with a Coomb’s negative haemolytic anaemia D : associated with a characteristic single amino acid substitution in the ATP binding cassette region

of a the ATPB7 gene that is present in over 95% of caucasian patients E : caused by a relative deficiency of caeruloplasmin, which is the major serum copper binding protein. Comment : Wilson's disease is a rare autosomally recessively inherited disorder of copper metabolism associated with mutations in the ATPB7 gene,which appears to encode a membrane bound copper transporting ATPase.Many different mutations are found in various kindreds, and there is corresponding heterogeneity in the disease, which may present with progressive neurological dysfunction, particularly of the basal ganglia, insidious onset of hepatic cirrhosis,or with fulminant liver failure in younger patients.In cases of fulminant liver failure associated with Wilson's disease, Coomb's negative haemolytic anaemia is a characteristic clinical feature and may be the only clue to the aetiology of the disorder.Rare and minor cardiac abnormalities may also be present.Raised circulating levels of copper,increased excretion of copper in the urine,and reduced circulating caeruloplasmin levels are thought to be secondary features.Fulminant liver failure usually necessitates emergency liver transplantation, while chronic neurological damage may be limited by the use of copper chelating treatment with penicillamine or trientene. Oral zinc appears to limit the uptake of copper from the intestine, and may be beneficial.C: 36. Which of the following forms of acute viral hepatitis has a much higher mortality in pregnant than non-pregnant females? A : Hepatitis A B : Hepatitis B C : Hepatitis C D : Hepatitis E E : Hepatitis G. Comment : The mortality of hepatitis E in pregnant women, particularly in the third trimester, are of the order of 20-30%. It is transmitted by the faecal oral route like hepatitis A and epidemics have been associated with faecal contamination of water. There is no chronic phase of infection. Hepatitis G virus is currently not known to cause any human disease, not even hepatitis, despite its name. D : 37. A 56-year-old man is admitted with melaena, tachycardia and hypotension. Which of the following are true? A : Consider an urgent abdominal CT with contrast if he has previously had an abdominal aortic

aneurysm repair. B : Intravenous omeprazole is indicated. C : If the ulcer base is clean, there is a 20% risk of re-bleeding. D : A normal haemaglobin level signifies a minor bleed. E : The overall mortality rate in the UK for upper gastrointestinal bleeding is about 5%. Comment : A suspected aorto-enteric fistula requires urgent investigation. If endoscopy reveals a clean ulcer base, the re-bleed rate is less than 5%. Stigmata of recent haemorrhage on endoscopy (fresh clot, visible vessel or active bleeding) are indications for intravenous omeprazole. These findings are also indications for endoscopic therapy. The overall mortality rate for upper gastrointestinal bleeding in the UK is 14%. A :

Page 168: Masterclass Book Part 2

MOHAMMED IS-HAG 167

38. A 44-year-old man with a family history of diabetes is referred by his GP because of mildly elevated LFT's. He weighs 108Kg, and is 1.78m tall. His waist measures 168cm and his hips 140cm. Which of the following statements are FALSE? A : He is clinically obese. B : Obesity has a genetic component. C : He is at increased risk of developing diabetes. D : He most likely has an underlying endocrine disorder. E : He is at increased risk of developing cerebrovascular disease. F : He should be put on a strict 1000kcal/day diet. G : Hypothyroidism should be excluded. H : Calorie controlled diet should be combined with long-term low-intensity exercise. I : His abnormal LFTs may be due to fatty liver. J : Gastroplication should be reserved for extreme cases where other methods have failed. Comment : This man has a BMI of 34 with a WHR>1; by definition obese. Patients are at increased risk of a wide range of illnesses including diabetes and cardiovascular disease. The condition usually occurs due to a simple imbalance of energy input and expenditure and rarely has an underlying primary endocrine disorder. Management should aim for gradual weight loss with a 600kcal/day deficit and gentle long-term exercise program. Orlistat may be beneficial if added in to these treatment modalities. Surgery remains controversial and should be reserved for severe and resistant cases. D : F: 39. A 34-year-old lady, born in Hong Kong, is referred to clinic because at a hospital employment medical she was found to have hepatitis B. Her serology is as follows: HbsAg positive, HBe Ab positive. Liver function tests, full blood count and clotting profile were all normal. Her partner has the following serology: HBs Ab positive, HBc Ab positive. There is no family history of liver disease. Which of the following statements is correct? A : Pregnancy is contraindicated. B : She cannot perform exposure prone procedures without her viral load being measured. C : Her risk of developing hepatocellular carcinoma is high. D : She needs interferon treatment. E : She is at high risk of infecting her partner sexually with hepatitis B. Comment : She is likely to be a ‘healthy carrier’ of hepatitis B. Her risk of clearing hepatitis B spontaneously is 1% a year. This is not increased by interferon. Interferon is used to treat eAg positive individuals with high serum transaminases and in this setting causes seroconvertion to e Ab, a fall in viral load and normalisation of liver biochemistry in about 30% cases. As a young woman, with a normal liver and e Ab (normally corresponds to a low viral load of <105 copies/ml) she is at low risk of hepatocellular carcinoma. Her partner is immune to hepatitis B from previous infection. Until relatively recently if an individual was e Ab positive it was possible to perform invasive exposure prone medical procedures (i.e. non-laparoscopic invasive surgery). This is now only possible if a viral load is consistently less than 103 copies/ml. Any child she has in the UK will be given hepatitis B immunoglobulins and vaccinated at birth to prevent transmission of hepatitis B. B : 40. A 78-year-old woman admitted 1 week earlier with a stroke develops diarrhoea. Which of the following statements are true? A : Campylobacter is the most likely cause. B : Clostridium difficile rarely relapses after a full course (7 days) of metronidazole. C : If an abdominal radiograph shows a toxic megacolon, ulcerative colitis is the most likely diagnosis. D : There is no indication for sigmoidoscopy and biopsy. E : Cephalosporins are the commonest antibiotic associated with Clostridium difficile infection. Comment : A relapse rate of up to 30% occurs after treatment of C. difficile, which (along with Campylobacter) can cause a toxic megacolon. The characteristic pseudomembrane seen on sigmoidoscopy and histological appearance are important diagnostic clues. E : 41. A 70-year-old man is admitted for investigation of anorexia and weight loss. He is 1.7m tall, was 100kg 6 months earlier and now weighs 85kg. Which of the following statements is true? A : His body mass index is 29.4. B : Serum albumin correlates well with the severity of malnutrition. C : Malnutrition affects up to 20% of hospital in-patients. D : He is underweight (according to the World Health Organization classification). E : Parenteral nutrition should be commenced. Comment : Malnutrition affects up to half of hospital patients. Body mass index is calculated from weight/height squared. The World Health Organization classifies a BMI of <18.5 as underweight, 18.5-24.9 as normal, 25-30 as overweight and >30 as obese. Low albumin levels are more commonly due to inflammation, malignancy, fluid overload, and renal or gastrointestinal losses than malnutrition. He has lost 15% of his body weight, so nutritional support should be instituted. However, it is likely that enteral feeding is more appropriate as it likely to be more safe, simple and cheap. A :

Page 169: Masterclass Book Part 2

MOHAMMED IS-HAG 168

42. A 31-year-old man with known selective IgA deficiency and abdominal pain is tested for anti-endomysial antibodies (EMA). His results are as follows: IgA endomysial antibody negative, IgG endomysial antibody positive. Which of the following statements is true? A : Coeliac disease can be confidently excluded on the basis of these results. B : His IgA deficiency is irrelevant to his negative IgA EMA status. C : The negative IgA EMA is of no diagnostic value in the presence of IgA deficiency. D : Referral for small bowel biopsy is not warranted on the strength of these results. E : For diagnostic purposes, IgA and IgG EMA are equally important Comment : Since there is a recognized association between coeliac disease and IgA deficiency, it is important to recognize that IgA EMA may be spuriously negative in patients with coeliac disease and total IgA deficiency, as in this case. For this reason, testing for coeliac disease antibodies always includes measurement of both IgA and IgG antibody isotypes or measurement of serum IgA. Such a testing strategy will alert both the laboratory and the requesting clinician of 'false-negative' results as in the case here. In keeping with the mucosal route of antigen entry, IgA endomysial antibodies are more important than IgG antibodies as a diagnostic marker of coeliac disease. C : 43. A 23-year-old man, previously fit and well, presented with jaundice. His mother has systemic lupus erythematosus (SLE). Investigations show: Bilirubin 175micromol/l, ALP normal, ALT 1228 IU/l, albumin 39g/l, Prothrombin time normal, smooth muscle antibody positive 1 in 320. A liver biopsy shows evidence of a hepatitis with perivenular collapse and a predominance of portal tract plasma cells. Which one of the following best describes this condition? A : It is not steroid responsive. B : Progression to liver transplantation is common. C : It is characterised by high IgA levels. D : Hepatic fibrosis is often present at presentation in adults. E : An ERCP is necessary to confirm the diagnosis. Comment : Autoimmune liver disease presents as an acute hepatitis and jaundice may be a feature at presentation. Many patients have histological evidence of fibrosis and even cirrhosis at presentation despite being relatively asymptomatic. The prognosis with long-term immunosuppression is excellent even in the presence of cirrhosis and few patients subsequently develop liver failure. The condition is steroid responsive and patients are often maintained on 7.5-10mg prednisolone or azathioprine monotherapy. High IgA levels are seen in alcoholic liver disease. A high IgG level i.e. 30-40g/l is very suggestive of autoimmune hepatitis in the clinical setting described here. D : 44. A 76-year-old lady is admitted to the coronary care unit with unstable angina. Her 12 lead electrocardiogram shows ST depression of more than 2mm in the anterior leads. Clinical examination shows evidence of acute heart failure. Despite medical treatment she has chest pain two hours after admission. Which of the following are correct? A : Morphine should not be given to relieve pain if s/l GTN has been ineffective as she is 76 years old. B : Verapamil should be given if she is unable to tolerate a beta blocker. C : Patients with impaired LV function (EF<50%) should not undergo coronary artery bypass grafting (CABG). D : Cardiac catheterisation is used to assess patients for CABG only. E : Patients with significant left main stem disease have increased survival with CABG. Comment : Morphine may be necessary to relieve anginal pain - age is not a contraindication, although aliquots of smaller doses e.g. 2.5mg should be used to achieve pain control.Calcium channel blockers should not be given to patients in pulmonary oedema or with impaired LV function.Impaired LV function (EF<50%) is not a contraindication to CABG.Cardiac catheterisation informs the choice between medical treatment,CABGand percutaneous transluminal coronary angioplasty .Patients with significant left stem main disease (>50% occlusion) or significant three vessel disease with depressed LV function have increase survival if they undergo CABG, although patients older than 75 yrs were not included in the trials that demonstrated such benefit. E : 45. A 70-year-old man with dysphagia is seen in clinic. He is unable to swallow any solids or fluids. What is the best course of action? A : Admit the patient from clinic for further investigations B : Arrange a CT abdomen and chest to look for metastatic cancer C : Arrange an appointment in the combined oncology-surgery clinic D : Presribe high dose proton pump inhibitors (PPIs) to reduce acid reflux E : Book the patient for percutaneous endoscopic gastrostomy (PEG) insertion. Comment : This man has absolute dysphagia and needs to be in hospital. He may have a malignancy, a peptic stricture or a food bolus. Neurological or muscle disorders are less likely. Most would arrange early cautious upper GI endoscopy or failing that a gastrograffin swallow (risk of aspirating barium). If a malignant lesion is likely then further investigations are needed to determine the best treatment strategy. Options include stenting, chemo-radiotherapy, PEG feeding (inserted either endoscopically or surgically) or palliation. Investigations such as CT, PET and endoscopic ultrasound allow staging of malignant disease and a multi-disciplinary approach to treatment is probably best. A :

Page 170: Masterclass Book Part 2

MOHAMMED IS-HAG 169

46. A 54-year-old restaurant manager presents with chronic abdominal pain, epigastric in origin and radiating through to the back, precipitated by alcohol. Plain abdominal radiograph shows linear calcification in the area. He also gives a history of weight loss and offensive floating diarrhoea. Which of the following statements are true? A : His condition is likely to be hereditary. B : His condition affects at least 1% of the population in the UK. C : Erythema ab igne is an unrelated clinical finding. D : Serum amylase is likely to be markedly elevated. E : His pancreolauryl test is likely to be abnormal. F : He should be screened for diabetes. G : While pancreatic enzyme supplements may help the diarrhoea, they are unlikely to have any

effect on his pain. H : Opiate analgesia should be escalated until pain control is achieved. I : Moderation of alcohol intake will have no effect at this late stage J : He is at no increased risk of pancreatic cancer. Comment : Chronic alcohol abuse is by far the commonest cause of chronic pancreatitis in this country. It is a surprisingly rare condition, with an estimated incidence of <4 per 100 000. Pain control can be difficult. Patients often apply heat pads (hence erythema ab igne) and opiate abuse and dependence can complicate the issue. Pain specialists and techniques such as coeliac blocks may be useful. Amylase is normal or mildly elevated in chronic pancreatitis (unlike acute pancreatitis). Exocrine function can be assessed by pancreolauryl test, and enzyme supplements may help both malabsorption and pain. Mild diabetes is common and should be treated. Alcoholic abstinence should be strongly encouraged. There is an increased incidence of pancreatic cancer in this population. E :F: 47. A 48-year-old man who had coronary artery bypass grafts 4 weeks ago is admitted with melaena. He is pale and shocked, with cold hands, P110 / min and systolic BP 90 mmHg. You cannot obtain intravenous access in his arms. To obtain vascular access you should: A : insert a femoral venous catheter B : insert an internal jugular venous catheter C : insert a subclavain venous catheter D : insert an internal jugular or subclavian venous catheter (depending on your experience and

confidence with the two techniques) E : place one of his arms in a bowl of warm water to dilate the veins so that you can obtain

peripheral venous access Comment : All veins will be constricted in this man, making their cannulation difficult. Never attempt subclavian or internal jugular venous cannulation in someone who is obviously hypovolaemic: the risks are high. Approaches to the subclavian vein may be complicated by pneumothorax or haemothorax, both of which could easily be fatal in this clinical situation. Some approaches to the internal jugular vein can also be complicated by pneumothorax, and all can be complicated by puncture of the carotid artery. The femoral vein lies medial to the artery in the groin: remember the acronym ‘NAVY’ – Nerve, Artery, Vein, Y-fronts. Inadvertent puncture of the femoral artery is much less likely to have dire consequences than the complications of central vein cannulation listed above. A : 48. A 64-year-old woman presents with a six-month history of progressive dysphagia for solids. She has a long-standing history of rheumatoid arthritis and retrosternal burning pain, regularly using over-the-counter preparations for indigestion. The two most likely diagnoses are: A : Pharyngeal pouch B : Duodenal ulceration C : Acute gastritis D : Scleroderma E : Benign oesophageal stricture F : Gastric carcinoma G : Motor neurone disease H : Aneurysm of the thoracic aorta I : Pseudobulbar palsy J : Oesophageal carcinoma. Comment : The history of dysphagia for solids suggests a mechanical (rather than neurological) cause, and the relatively long history indicates that a benign diagnosis is most likely. However, carcinoma of the oesophagus cannot be excluded and barium swallow followed by endoscopy is required to rule out malignancy. Gastro-oesophageal reflux disease is the likeliest cause of benign stricture in this case, but other benign causes include drugs (non-steroidal anti-inflammatory agents, potassium salts, bisphosphonates, tetracyclines), radiation (e.g. for bronchial carcinoma) and candidiasis. E :J:

Page 171: Masterclass Book Part 2

MOHAMMED IS-HAG 170

49. A 39-year-old presents with bilirubin 75 micromol/l, ALT 1850 IU/l, alkaline phosphatase 150 IU/l. She has recently returned from a beach holiday in West Africa where she suffered from gastroenteritis. Which of the following statements is correct? A : A positive HAVIgG antibody suggests acute hepatitis A infection. B : Acute hepatitis A has a mortality of 2%. C : A live-attenuated virus vaccine is available to protect travellers to high-risk areas for hepatitis A. D : Pooled human immunoglobulin can be given to close contacts within 2 weeks of exposure. E : Hospital admission is mandatory. Comment : Acute hepatitis A is diagnosed on the basis of a positive IgM antibody (IgG is a marker of previous exposure, which may well have been asymptomatic). Hepatitis A has a mortality of less than 0.05%. The vaccine is a formalin-inactivated vaccine. Pooled immunoglobulin prophylaxis is available if less than two weeks from exposure. Hospital admission is rarely required unless patient has vomiting and/or dehydration. However, they should be monitored closely as an out-patient. The acute hepatitic phase is often followed by a prolonged cholestatic phase, and while relapsing disease is rare, patients should be followed until LFTs normalize. D : 50. A 35 yr old woman presents with abdominal bloating, change in the form of her stool with nocturnal diarrhoea, abdominal pain relieved by defaecation, and marked gastrocolic reflex. Which one of these features is NOT a feature of Irritable Bowel Syndrome: A : Bloating B : Change in the form of the stool C : Nocturnal diarrhoea D : Abdominal pain relieved with defecation E : Marked gastrocolic reflex. Comment : Nocturnal diarrhoea should not be attributed to IBS unless alternative causes have been very carefully and thoroughly investigated. The remaining features listed may be caused by other conditions, but are all recognized as clinical presentations of IBS. C : 51. A 32-year-old woman is referred to the gastroenterology clinic with lethargy and mild elevations in her serum alanine aminotransferase (ALT 85 iu/L; normal <40). Other investigations include: aspartate aminotransferase 80iu/L, GGT 60 iu/L (normal <40). The bilirubin, alkaline phosphatase, albumin and prothrombin time are normal. Her full blood count is normal and the MCV is 85fL. What is the most likely diagnosis? A : Chronic hepatitis C B : Alcoholic hepatitis C : Primary biliary cirrhosis D : Haemochromatosis E : Wilson’s disease Comment : Chronic hepatitis C is a very common cause of minor elevations in serum transaminases. Other liver function tests can be entirely normal and assessment of viral status by PCR along with histological assessment may be needed, particularly if treatment is being considered. Isolated elevations in AST (aspartate aminotransferase) can occur with muscle disease and the AST:ALT ratio can be useful in diagnosing alcoholic liver disease, because more than two-thirds of patients will have a ratio greater than 2. Menstruating women are generally protected against haemochromatosis.Wilson’s disease although rare must always be considered in a young person with chronic hepatitis. However, cirrhosis is often present by the age of 31,co-existent haemolysis (elevated bilirubin) is common and transaminases are often higher. A : 52. A 33-year-old injecting drug user with known chronic hepatitis B presents to casualty with jaundice, pruritus and confusion. Ultrasound examination was unremarkable and liver function tests suggested a hepatitic picture. Which of the following is the most likely explanation? A : Acute hepatitis C B : Development of hepatocellular carcinoma C : Biliary stones D : Superinfection with hepatitis D virus E : Acute hepatitis E. Comment : Hepatitis D is a RNA virus, structurally unrelated to hepatitis A, B, or C, that causes an infection requiring the assistance of hepatitis B virus particles in order to replicate. Disease manifestations include a self-limited acute infection, acute fulminant liver failure and end stage liver disease from chronic infection. Since hepatitis D virus is transmitted by blood and blood products, the risk factors for infection are similar to hepatitis B virus and infection may occur at the same time as hepatitis B or represent super-infection; the latter scenario should be suspected in a patient with chronic hepatitis B whose condition suddenly deteriorates. Diagnosis is usually serological (IgM antibodies indicate acute infection). Clinical manifestations of acute hepatitis C are extremely rare (usually within 2 months of exposure to HCV). Acute hepatitis E is most common in the developing world (tropical climates, inadequate sanitation, and poor personal hygiene). D :

Page 172: Masterclass Book Part 2

MOHAMMED IS-HAG 171

53. A 50-year-old man presents with flushing worse after drinking alcohol and diarrhoea. Examination reveals a thin individual with hepatomegaly. The most likely diagnosis is: A : glucagonoma B : mastocytosis C : hyperthyroidism D : carcinoid syndrome E : phaeochromocytoma. Comment : Carcinoid syndrome is usually associated with functioning malignant tumours that produce serotonin and arise from enteroendocrine cells in the ileum. It may occur with comparable tumours elsewhere in the GI tract, pancreas, gonads, or bronchi and rarely, highly malignant tumours such as oat cell carcinomas of the lung, pancreatic islet cell carcinomas, and medullary carcinomas of the thyroid gland can be responsible. Intestinal tumours do not usually produce the syndrome unless metastatic spread to the liver has occurred, because the metabolic products released by the tumour are rapidly degraded by blood and liver enzymes in the portal circulation (e.g. hepatic monoamine oxidase and serotonin). Hepatic secondaries release metabolic products directly into the systemic circulation via the hepatic veins. Primary pulmonary and ovarian carcinoid products bypass the portal route, as do rare intestinal carcinoids with only intra-abdominal spread that drain directly into the systemic circulation or the lymphatics. A common and early sign is uncomfortable flushing, typically of the head and neck, often precipitated by emotion or eating hot food and drinks or following alcohol consumption. Notable skin colour changes may occur, ranging from pallor or erythema/cyanosis. Abdominal cramps with recurrent diarrhoea often occur. Right-sided endocardial fibrosis, leading to pulmonary stenosis and tricuspid regurgitation may also develop, whilst left heart lesions, reported with bronchial carcinoids, are rare because serotonin is destroyed during passage through the lung. A few patients have asthmatic wheezing, and some have decreased libido and impotence; pellagra develops rarely. D : 54. An 83-year-old man is about to be discharged home, having spent 3 weeks in hospital being treated for C. difficile diarrhoea associated with pseudomembranous colitis. Unfortunately, the nursing staff report that he has had a recurrence of profuse, malodorous, greenish diarrhoea. He has already had one course of oral metronidazole therapy. Which of the following is the most appropriate treatment? A : Intravenous metronidazole B : Oral metronidazole C : Intravenous teicoplanin D : Intravenous vancomycin E : Oral ciprofloxacin Comment : Approximately 20% of patients with pseudomembranous colitis relapse, usually 2 weeks to 2 months after treatment. This is related to the persistence of C.difficile spores that are not killed by antibiotic therapy, rather than to metronidazole resistance, hence recurrence of disease should be treated with a second course of oral metronidazole.If he fails to respond to a second course of oral metronidazole the second line agent would be oral (not intravenous) vancomycin. B : 55. At upper GI endoscopy antral gastritis is noted in a patient with duodenal ulceration. However, a urease test for Helicobacter pylori is negative. The patient should be: A : investigated for other causes of duodenal ulceration with a fasting gastrin level B : managed with a proton pump inhibitor (PPI) alone C : treated with eradication therapy nevertheless D : offered surgery as a definitive cure E : have repeat endoscopy in 3 months. Comment : Helicobacter pylori is associated with 95% of duodenal ulcers and 80% of gastric ulcers. Tests for H pylori include serology, histological biopsy, urease testing, urea breath testing and faecal antigen assays. False negative urease testing can occur if patients have been treated with antibiotics, bismuth or proton pump inhibitors in the prior 1-2 weeks. Thus in this case where a H pylori- associated gastritis and duodenal ulcer is most likely, eradication should be prescribed regardless of the urease test. C : 56. Which of the following is true in the investigation of a 30-year-old man with a 2-month history of diarrhoea and weight loss? A : Stool examination is most likely to identify a cause B : Hydrogen breath testing for small-bowel bacterial overgrowth is a first-line investigation C : IgA anti-endomysial antibody determination is mandatory D : A normal C-reactive protein (CRP) excludes inflammatory bowel disease E : The absence of a travel history precludes an infectious aetiology Comment : In the investigation of diarrhoea, history is very important. Organic disease is more likely if the diarrhoea is of less than 3 months duration and nocturnal. It is unusual to find an infective cause if symptoms have persisted for over 4 months, but untrue to suggest that the absence of overseas travel precludes an infectious aetiology. Coeliac disease is extremely common with >0.2% sero-prevalence in

Page 173: Masterclass Book Part 2

MOHAMMED IS-HAG 172

Western populations but can present in subtle ways. Mild diarrhoea associated with a mild anaemia is an important presentation in adults and serology (for IgA anti-endomysial antibody against tissue transglutaminase) has a >95% accuracy in diagnosis. Florid ulcerative colitis can be present without evidence of an on-going acute phase response as measured by CRP. Small-bowel bacterial overgrowth would be unusual in a young person in the absence of a structural or functional intestinal disorder (i.e. blind loop from surgery or diabetes mellitis) but would need to be considered in the elderly. C : 57. A 30-year-old woman is seen with weight loss and diarrhoea. Three years ago she spent 6 months in Egypt. Previous investigations have included negative endomysial antibodies, normal hydrogen breath test and normal barium follow through. Which method is the most specific and simple way of diagnosing chronic giardiasis? A : Urine culture B : Serology C : Histology from the second part of the duodenum D : Colonoscopic appearances E : Small bowel enteroscopy. Comment : Giardia lamblia is a very common intestinal parasite and a frequent cause of diarrhoeal illness throughout the world. Although water remains the most common mode of transmission, there has been an increase in the number of person-to-person cases, especially related to children, as well as an increase in food-borne cases. Chronic diarrhoea and malabsorption are seen with persistent infection and since treatment is simple and effective, diagnosis by duodenal biopsy is important. New antigen detection tests for stool may ultimately replace histology in some situations. The greatest clinical experience in treatment is with the nitroimidazole drugs, i.e., metronidazole, tinidazole, and ornidazole, which are highly effective. A 5- to 7-day course of metronidazole can be expected to cure over 90% of individuals, and a single dose of tinidazole or ornidazole will cure a similar number. C : 58. You are called to review a 67-year-old man admitted 4 days ago with central abdominal pain. A diagnosis of acute pancreatitis was made clinically and biochemically, and the surgeons have managed him conservatively. A few small gallbladder stones were noted on ultrasonography but the common bile duct was undilated and there were no intraduct stones. A CT scan has been requested. He is a strict Muslim whose only other past medical history is ulcerative colitis, for which he is on treatment. The two most likely causes of his pancreatitis are: A : Alcohol-induced pancreatitis B : Hereditary pancreatitis C : Drug-induced pancreatitis D : Gallstone pancreatitis E : Idiopathic pancreatitis F : Hyperparathyroidism G : Mumps infection H : Pancreas divisum I : Traumatic pancreatitis J : Scorpion envenomation Comment : Acute pancreatitis is an inflammatory process affecting the pancreas with variable involvement of other regional tissues or remote organ systems. There are many causes of acute pancreatitis but gallstones (30–50%) and alcohol (10–40%) account for the majority of cases. Less common causes include: (i) drugs (e.g. azathioprine, sulphasalazine, frusemide), (ii) toxins, (iii) trauma (blunt trauma to the abdomen as well as iatrogenic trauma e.g. postoperative and ERCP), (iv) hypertriglyceridemia, (v) hypercalcaemia, (vi) infections (eg mumps, Coxsackie, cytomegalovirus), (vii) congenital anomalies (pancreas divisum, choledochocele), (viii) ampullary or pancreatic tumors, (ix) vascular abnormalities (atherosclerotic emboli, hypoperfusion, vasculitis), (x) hereditary pancreatitis (mutations in trypsinogen gene), and (xi) idiopathic causes (10–25% of patients). In this man alcohol can be excluded since he is a strict Muslim. Of the other causes gallstone pancreatitis remains the most likely. Although gallstone pancreatitis can occur at any age, it is most common between the ages of 50 and 70 years and, the diagnosis of gallstone-induced pancreatitis can often be missed. Ultrasound is only 70% to 80% sensitive in detecting gallstones during the acute phase of pancreatitis. His history of ulcerative colitis is important since azathioprine has become a common treatment for this chronic condition and pancreatitis is a well recognised and important complication of azathioprine. Other side effects of azathioprine include dose-related bone-marrow depression (leucopaenia, thrombocytopenia, anaemia), cholestatic liver damage and rarely veno-occlusive liver disease. Azathioprine is also associated with gastro-intestinal disturbances, reversible alopecia, rashes, muscle and joint pains, fever, rigors, pneumonitis, meningitis, arrhythmias, renal dysfunction and hypotension, some or all of which may represent hypersensitivity reactions. The other causes of pancreatitis whilst recognised are rare and unlikely. Idiopathic pancreatitis is a diagnosis of exclusion. C : D :

Page 174: Masterclass Book Part 2

MOHAMMED IS-HAG 173

59. A 62-year-old woman is referred to the outpatient clinic for investigation of abnormal liver function tests (LFTs). Which of the following statements are true? A : An ALT/AST ratio greater than one is suggestive of alcoholic liver disease. B : Copper and caeruloplasmin should be measured in order to exclude Wilson's disease. C : An alpha-1-antitrypsin level 1.5 times the upper limit of normal may explain the abnormal LFT's. D : Liver biopsy is contraindicated in patients over the age of 60 years. E : Low albumin levels are likely to reflect malnutrition. F : A negative HBeAg excludes active infective hepatitis B. G : An elevated HbA1c may be significant to the diagnosis. H : A positive anti-mitochondrial antibody is highly specific for primary biliary cirrhosis. I : A positive anti-nuclear antibody is highly specific for auto-immune hepatitis. J : An elevated alpha-feto protein is pathognomonic for hepatocellular carcinoma. Comment : An AST/ALT ratio is suggestive (though not pathognomonic) of alcoholic liver disease. It would be exceptionally rare for Wilson's disease to present for the first time in someone of this age, and tests for this condition should not be performed in patients over the age of 50 unless there are genuine reasons to believe that the diagnosis possible. Alpha-1-antitrypsin deficiency, rather than excess, leads to disease. There is no upper age-limit to liver biopsy, though the risk benefit ratio and likelihood of influencing management should be weighed up in all patients. Patients with pre-core mutant hepatitis B will not have detectable HBeAg but are still viraemic and hence infectious. Fatty liver is believed to be the commonest reason for abnormal LFT's in this country, and a significant proportion of people with fatty liver have type II diabetes. Antimitichondrial antibodies are highly indicative of primary biliary cirrhosis, as opposed to anti-nuclear antibodies, which may be raised in autoimmune liver disease but are highly non-specific. Alpha-fetoprotein may be raised in hepatocyte regeneration in the absence of HCC, and by contrast may be negative in the presence of hepatoma. G:H: 60. A 20-year-old man complains of intermittent epigastric pain for 6 months. Which of the following statements are true regarding his investigation and treatment? A : Gastroscopy is the investigation of choice. B : 50% of 20-year-olds are infected with Helicobacter pylori in the UK. C : A 7-day course of a proton pump inhibitor plus amoxycillin would be appropriate. D : A 13C urea breath test may be falsely negative if the patient is also taking non-steroidal anti-

inflammatory drugs (NSAIDs). E : H pylori serology is only effective in confirming eradication success if performed at least 4 weeks

after treatment. Comment : Gastric carcinoma is rare before the age of 40 and in the absence of alarm symptoms, other non-invasive methods of investigation should be considered.Twenty per cent of 20-year-olds and 50% of 50-year-olds are H pylori positive and the lifetime risk of duodenal ulcer is up to 10% in these individuals. H pylori produces a urease which release 13C from orally administered 13C urea which can be measured in exhaled breath 13CO2.Proton pump inhibitors reduce H pylori load and may result in false negative tests. Suitable H pylori eradication regimens include proton pump inhibitor based triple therapies, usually including amoxycillin (or metronidazole) and clarithromycin. H pylori antibody levels take many months to fall, so ensuring eradication success is more appropriate with a urea breath test which can be done 4 weeks after treatment. A : 61. A 35-year-old woman, otherwise fit and well, presents with isolated transaminitis. She denies excess alcohol use and ultrasound of her liver reveals only fatty change. Her standard liver screen is negative. Which statement best reflects this clinical scenario? A : She most likely has Non A-E Hepatitis and should be screened for Hepatitis G. B : Silent alcohol abuse should be assumed. C : Non-alcoholic fatty liver disease is most likely. D : Wilson’s disease needs to be diagnosed by liver biopsy. E : She should be advised that she can’t be a blood donor. Comment : Non-alcoholic fatty liver disease (NAFLD) is a spectrum of diseases that includes simple steatosis, steatohepatitis, advanced fibrosis and cirrhosis. The diagnosis should be suspected in patients with persistently elevated alanine aminotransferase (ALT) values with: · negative screening for viral hepatitis · autoimmune hepatitis and metabolic liver disease · no high risk alcohol or medication use · fatty infiltration on ultrasound. NAFLD is associated with the metabolic syndrome (obesity, insulin resistance, hyperlipidaemia and hypertension) and is therefore often seen in obese patients, who may have impaired glucose intolerance. Treatment and prognosis is unclear, but attention to modifiable risk factors is important including weight loss, diabetic management and treatment of hyperlipidaemia. C :

Page 175: Masterclass Book Part 2

MOHAMMED IS-HAG 174

62. An 84-year-old lady presents to A&E with confusion and epigastric discomfort. She is not hypoxic. Liver function tests are abnormal: bilirubin 120micromol/l, alkaline phosphatase 750Iu/l and AST 110 IU/l. Her CRP is raised at 120mg/l. An ultrasound shows gallstones in the gallbladder and a mildly dilated common bile duct but no intraduct stones. The most likely diagnosis is: A : Carcinoma of pancreas B : Cholecystitis C : Choledocholithiasis D : Ampullary carcinoma E : Primary sclerosing cholangitis. Comment : This lady has cholangitis which complicates common bile duct (CBD) stones but not biliary malignancy, unless a biliary stent has previously been inserted and has blocked. Primary sclerosing cholangitis can be complicated by cholangitis but this is unusual in a patient who has not had previous instrumentation of the biliary tree (i.e. ERCP). Do not forget that elderly patients rarely complain of pain and that common bile duct stones may be present several years after a cholecystectomy and present in this way. About 75% of CBD stones can be delineated by ultrasound although detection will be limited when bile duct dilatation is absent. Other useful non-invasive imaging modalities include CT and MRI of the biliary tree. Cholelithiasis represents the presence of gallstones within the gallbladder; choledocholithiasis is the presence of gallstones within the common bile (10% of patients with gallstones have choledocholithiasis, which is sometimes called common duct stones). Cholangitis refers to inflammation of the common bile duct, usually as a result of secondary infection, while cholecystitis is a disorder marked by inflammation of the gallbladder. It is usually caused by the passage of a stone from the gallbladder into the cystic duct (connects the gallbladder to the common bile duct). In 5-10% of cases, cholecystitis develops in the absence of gallstones (acalculous cholecystitis). C : 63. With regard to the cancer risk in inflammatory bowel disease (IBD): A : Colonic surveillance in long-standing pan ulcerative colitis (UC) has been shown to reduce

mortality. B : The risk in long-standing Crohn's colitis is equivalent to risk in long-standing UC. C : A tubular adenoma with moderate dysplasia detected in the course of surveillance in long-

standing UC mandates colectomy. D : Cumulative risk of colorectal carcinoma (CRC) 30 years after diagnosis of pan UC is

approximately 5%. E : The risk of CRC in pan UC is not affected by 5-aminosalicylic acid. Comment : Estimates of cumulative risk of CRC 25-30 years after diagnosis of pan UC vary between 10 and 20%, with the risk approaching 40% in those diagnosed in childhood. The data in Crohn's disease is more limited but indicates a risk at least equivalent to UC and possibly greater - possibly related to the more widespread use of immunosuppressive agents. Many units undertake two to three-yearly surveillance colonoscopy with biopsies beginning eight to ten years after a diagnosis of extensive UC or Crohn's colitis, with the objective being to detect a field change of dysplasia. CRC in the context of long-standing IBD does not tend to follow a classic adenoma carcinoma sequence, but instead pre-cancerous changes can be detected as a field change with dysplasia observed on random biopsies from around the colon. The finding of such dysplasia, particularly when high-grade or multifocal, mandates colectomy. However, dysplasia detected in an adenomatous polyp can probably be safely managed in the normal way by polypectomy and increased frequency of surveillance.Although most studies show that cancers detected by surveillance are at an earlier stage, the studies are somewhat biased, and there is no prospective randomized data demonstrating reduced mortality from a CRC in a population undergoing surveillance. However, risk of CRC in IBD does appear to be reduced by maintenance treatment with 5-aminosalicylic acid such as sulphasalazine or mesalazine. B : 64. Regarding cholangiocarcinoma: A : Risk factors include primary sclerosing cholangitis (PSC) and chronic parasitic infection. B : Early surgery, particularly liver transplantation, is potentially curative. C : Metal stents, which can be easily removed should be used to relieve obstruction while planning

surgery. D : Patients with PSC should have regular measurements of the CA19-9 antigen, as this is an early

marker of the disease. E : The rate is declining in the west as a result of aggressive management of ulcerative colitis, which

is the major associated cause of PSC and cholangiocarcinoma. Comment : Cholangiocarcinoma is rare, but increasing in incidence in the UK, for unknown reasons. Risk factors include PSC and chronic parasitic infections of the biliary tract, particularly in the Far East. Liver transplantation is contraindicated as the disease invariably recurs.Metal stents are used for palliation, and cannot be removed. The CA19-9 antigen is raised in many cases, but the sensitivity and specificity of the test is woefully inadequate for screening. A :

Page 176: Masterclass Book Part 2

MOHAMMED IS-HAG 175

65. Hepatitis C is what kind of virus? A : A retrovirus B : A DNA virus C : A variable RNA virus D : A close relative of hepatitis B virus E : A defective helper virus. Comment : Hepatitis C virus is highly variable, as are many RNA viruses, and exists in multiple different genotypes. C : 66. A 36-year-old man presents with a 2-day history of severe pain in the left knee. He has recently returned from a holiday in Spain. Examination shows low grade fever (37.5?C) and marked synovitis of the left knee with a tense effusion. Which are the two most likely diagnoses? A : Lyme disease B : Rheumatoid arthritis C : Pseudogout D : Gout E : Septic arthritis F : Osteosarcoma G : Osteoarthritis H : Reactive arthritis I : Torn medial meniscus J : Spontaneous haemarthrosis. Comment : Reactive arthritis, consequent upon either genital chlamydia trachomatis or enteric infection, is the most likely diagnosis. He is a little younger than many patients presenting with gout, but this is not a rare diagnosis in this age group, particularly if alcohol intake is high. The important diagnosis to exclude is septic arthritis, but this is a rare cause of monoarthritis in this age group. Aspiration of synovial fluid is the key diagnostic tool to differentiate between these diagnoses. D : H: 67. As part of a routine health check, a young women is found to have the following liver function tests: Bilirubin 37µmol/L (0-18), AST 14 IU/L (5-40), ALT 26 IU/L (5-45), alkaline phosphatase 125 IU/L, Albumin 40 g/L (36-45), Hb 13.7 g/dL (11.5-15.5), reticulocyte count 1.2%. What is the most likely diagnosis? A : Chronic alcoholic liver disease B : Autoimmune haemolytic anaemia C : Common bile duct stone D : Gilbert's syndrome E : Primary biliary cirrhosis. Comment : An isolated hyperbilirubinaemia is invariably due to Gilbert's syndrome (4% of the population). It should be regarded as a variation of normal. The most important condition in adulthood which could give similar biochemical results would be haemolytic anaemia. A reticulocyte count (raised) and serum haptoglobin (mops up the haem. therefore low) would differentiate. D : 68. In a patient who is jaundiced and HBsAg and HBeAg positive with evidence of cirrhosis (ascites) and an albumin of 28g/L and a prothrombin time of 21 sec, the best treatment would be: A : Ribavirin and alpha-interferon B : Ribavirin C : Lamivudine D : Beta-interferon E : Alpha-interferon. Comment : Ribavirin has no effect on hepatitis B replication, but used in combination with interferon is more effective than ribavirin alone in eradicating chronic infection with hepatitis C. Interferon was until recently first line treatment for chronic hepatitis B infection associated with elevated serum transaminases. However, there is often a flare in ALT (alamine aminotransferase) on starting treatment, which in the presence of end stage liver disease (elevated PT, bilirubin and ascites here) can lead to liver failure so, in these circumstances, it is contra-indicated. Lamivudine suppresses HBV replication and is safe to use in decompensated end-stage cirrhosis. C : 69. A 76-year-old woman was admitted with poor mobility. Clinical examination showed a body mass index of < 18. Her husband had died 8 months previously and she had been anorexic since. Her body temperature was noted by nursing staff to be low, confirmed by rectal temperature. A : Protein energy malnutrition (PEM) is rare in older hospitalised patients B : A body mass index of < 18 in older patients is not associated with increased mortality C : There is no association between PEM and hypothermia D : There is no association between PEM and heat intolerance E : PEM results in smaller diameter muscle fibres as well as loss of muscle fibres.

Page 177: Masterclass Book Part 2

MOHAMMED IS-HAG 176

Comment : PEM is common in older patients affecting upto 40%.Severe PEM with BMI <18 is associated with increased mortality in older patients.There is an association with heat intolerance and hypothermia in patients with PEM. PEM does result in loss of muscle fibres and smaller diameter muscle fibres. E : 70. A 52-year-old Asian woman is admitted comatose with acute liver failure. Auto-antibodies, viral serology, ferritin, copper and alpha-1-antitrypsin are all negative or normal. Which of the following statements concerning hepatic drug toxicity is true? A : Acute liver failure can rarely be ascribed to anti-tuberculous therapy. B : A history of herbal remedies may well be relevent. C : A course of clavulanic acid-amoxicillin completed 2 weeks ago can safely be excluded as the

cause. D : Prognosis is better than acute liver failure secondary to viral hepatitis. E : Liver transplantation is contraindicated. Comment : Always consider drug toxicity in the presence of jaundice or abnormal liver biochemistry. This often means obtaining a detailed drug history from the GP and asking family members about herbal remedies and over-the-counter medication. Acute liver failure has frequently been reported following anti-tuberculous therapy. Although there is usually a very clear temporal association between taking the drug and the onset of hepatotoxicity, halothane and co-amoxiclav may present up to three weeks after taking the drug. Prognosis is worse than with liver failure secondary to acute viral hepatitis. While the mainstay of treatment is supportive, the need for transplanataion should be borne in mind, and early discussions should take place with a transplant unit. B : 71. A 62-year-old man has a gastroscopy because of reflux symptoms. Which of the following statements are true? A : Barrett’s oesophagus is found in over 5% of the asymptomatic population. B : Yearly gastroscopy and multiple biopsies reduce mortality in patients with long segment Barrett’s

oesophagus. C : The severity of reflux symptoms correlate with the degree of inflammation and presence of

Barrett’s oesophagus. D : Short segment Barrett’s oesophagus does not have an increased risk of malignancy. E : Barrett’s oesophagus carries a 40-fold increased risk of squamous cell carcinoma. Comment : No treatment has been shown to reduce the risk of malignancy or mortality rate in patients with Barrett’s oesophagus, which carries a 40-fold increased risk of adenocarcinoma. The majority of patients die from unrelated causes. A : 72. A 48-year-old man with alcoholic cirrhosis of the liver is admitted with haematemesis and melaena. The two most likely sources of bleeding are: A : Mallory Weiss tear B : Oesophageal varices C : Oesophageal malignancy D : Gastric varices E : Gastric malignancy F : Peptic ulcer G : Portal hypertensive gastropathy H : Aorto-enteric fistula I : Reflux oesophagitis J : Duodenal malignancy. Comment : The causes of haematemesis in patients with liver cirrhosis are as follows: bleeding oesophageal varices (60%), bleeding peptic ulcer (20%), portal hypertensive gastropathy (5%), bleeding gastric varices (5%), other causes / undiagnosed (10%). Significant gastrointestinal haemorrhage from oesophageal varices can present with melaena in the absence of haematemesis. Variceal haemorrhage can be the presenting feature of previously undiagnosed liver disease. B : F: 73. A 45-year-old man with no previous medical history is found to have oesophageal varices when endoscoped to investigate dyspepsia. Which of the following are true: A : Variceal sclerotherapy is indicated. B : Variceal banding is indicated. C : Beta-blockers do not affect the long term outcome. D : If admitted with variceal bleeding, terlipressin may reduce bleeding. E : Variceal bleeding has a 10% mortality rate. Comment : Beta-blockers are of overall benefit in primary prophylaxis, i.e. in reducing the chance of a first bleed. Variceal bleeding has a mortality rate of about 30%, and terlipressin may reduce bleeding when administered acutely. D :

Page 178: Masterclass Book Part 2

MOHAMMED IS-HAG 177

74. A 43-year-old woman with a long history of non-insulin dependent diabetes mellitus is noted to have an alanine aminotransferase (ALT) that is twice the upper limit of normal on routine screeing. She says that she does not drink alcohol and never has done. She is obese (BMI 28). Which of the following is the most likely cause for her elevated ALT? A : Haemochromatosis B : Gall stones C : Alcohol consumption D : Hepatic steatosis E : Drug-induced hepatitis. Comment : Hepatic steatosis is associated with diabetes mellitus, hypercholesterolaemia, hypertriglyceridaemia and obesity.Diagnosis may require liver biopsy, although resolution of abnormal liver blood test results on correction of hyperglycaemia, hyperlipidaemia or obesity may make this unnecessary. D : 75. A 78-year-old woman is admitted on take because she is unable to cope at home. She says that she feels ‘a bit weak’ but admits to no other symptoms. On examination she looks as though she has lost a lot of weight and is jaundiced, but there are no other abnormal physical signs. The most likely diagnosis is: A : Gall stones B : Carcinoma of the stomach with hilar lymph nodes C : Chronic pancreatitis D : Carcinoma of the pancreas E : Cholangiocarcinoma. Comment : Pancreatic carcinoma classically causes painless jaundice, but the absence of pain does not exclude the diagnosis of gallstones. The other diagnoses listed can all cause jaundice but are much less likely to be the explanation in this case. If pancreatic cancer is the diagnosis, then liver blood tests are likely to reveal elevation of alkaline phosphatase (ALP) and gamma-glutamyl transpeptidase (GGT). Elevation of aspartate transaminase (AST) and alanine transaminase (ALT) will be less marked, and they may be normal. The key investigation is ultrasound, which is likely to reveal dilatation of intra- and extra-hepatic bile ducts. Endoscopic retrograde cholangiopancreatography (ERCP) or CT scanning should confirm the diagnosis. D : 76. after starting a gluten-free diet for coeliac disease. Which of the following is the commonest cause of failure to respond to treatment? A : Pancreatic insufficiency B : Dietary non-compliance C : Small bowel adenocarcinoma D : Enteropathy associated B cell lymphoma of the small intestine E : Ulcerative jejunitis. Comment : Poor dietary compliance accounts for most cases where there is a failure to improve on treatment. Ulcerative jejunitis, small bowel adenocarcinoma and an enteropathy-associated T cell lymphoma are recognised but rare complications and there is an association with pancreatic sufficiency. B : 77. An 87-year-old lady is admitted with left iliac fossa pain, tenderness and fever. Which of the following are true? A : Colonoscopy is likely to be helpful. B : Ultrasound or computerised tomography (CT) is indicated. C : Diverticulosis affects 10% of the population over 50 years of age. D : Bleeding is an unusual complication of diverticular disease. E : Colonoscopy is unhelpful in patients admitted with bright red rectal bleeding. F : Diverticulosis is associated with an increased risk of colorectal neoplasia. G : Recurrent urinary infections raise the possibility of a complication of diverticular disease. H : A diverticular stricture is readily distinguished from a malignant stricture. I : Following a diverticular abscess, the affected diverticulae fibrose and further problems are

uncommon. J : Uncomplicated extensive diverticulosis may explain weight loss. Comment : Ultrasound and CT are useful in detecting bowel thickening, abscess formation and even fistulae.As diverticulitis is associated with multiple micro-perforations, colonoscopy is not advised. Diverticular disease is the commonest cause of colonic bleeding which may be detected and endoscopically treated at colonoscopy. Recurrent urinary infections, pneumaturia or faeces in the urine are features of a colo-vesical fistula. Diverticulosis affects 50% of the population over 50. Most patients have few if any symptoms, but if complications occur (abscess, stricture or bleeding), recurrence is common and surgery should be considered. B : G:

Page 179: Masterclass Book Part 2

MOHAMMED IS-HAG 178

78. A patient with chronic Hepatitis C is referred to your clinic for anitiviral treatment. However, interferon-alfa and ribavirin may not be suitable for those patients who: A : have moderate-severe disease B : were previously intravenous drug users C : are heavy alcohol drinkers D : are haemophiliacs and have not had a liver biopsy E : have relapsed following initial response to interferon-alfa monotherapy. Comment : Combination therapy with interferon-alfa and ribavirin is generally recommended for those with moderate-sever disease (histological diagnosis of significant scarring and/or significant necrotic inflammation).While NICE guidance suggests that problems with drug interactions,safety,and compliance may arise in existing intravenous drug users, those who have given up the habit should not be excluded from therapy. However, treatment is not generally recommended in those patients who consume large quantities of alcohol, given the increased risk of liver damage. In cases where a liver biopsy carries a high risk (e.g. haemophilia), treatment can be initiated without histological confirmation .Both treatment - naïve patients and those who have relapsed following initial response to interferon-alfa should be considered for 6 months of combination therapy. C : 79. A 58-year-old female is admitted acutely with pulmonary oedema and hypotension. She had a mitral valve replacement four years previously for mitral regurgitation. Clinical examination reveals a diastolic murmur and pulmonary oedema. Her biochemistry and full blood count is normal. Her INR is 1.8. Echocardiography demonstrates thrombus around the valve. What is the best treatment? A : Intravenous heparin B : Thrombolysis C : Urgent surgical intervention D : Increased dose of warfarin E : Low molecular weight heparin. Comment : Mechanical mitral valves are more prone to thrombosis than aortic valves if the INR drops below the therapeutic level. This is a medical/surgical emergency. It is rapidly fatal without immediate surgical intervention.Thrombus in this context cannot be treated withanticoagulants / fibrinolytics . Anticoagulation is clearly an important consideration when considering what type of valve prosthesis should be considered for an individual. C : 80. A 45-year-old man with chronic alcoholic liver disease was admitted earlier in the day following a large haematemesis.He was treated with IV terlipressin and urgent endoscopy was arranged after initial fluid resuscitation and correction of a mild coagulopathy with fresh frozen plasma. Endoscopy revealed bleeding oesophageal varices, which were injected with sclerosant with apparently good effect. He has been stable on the ward for the past 6 hours, but you are called to see him in the early hours of the following morning because he has had a further 500ml haematemesis, and his blood pressure has dropped from 130/90 to 90/50. Which of the following measures would be most appropriate in addition to fluid resuscitation? A : Urgent repeat endoscopy and sclerotherapy B : Change terlipressin to octreotide C : Administer 10mg vitamin K intravenously D : Insert sengstaken tube, inflate gastric balloon and apply traction E : Urgent surgical intervention. Comment : Balloon tamponade is the most effective treatment for control of variceal bleeding if endoscopic therapy has failed. Even though the varices were bleeding from within the oesophagus, inflation of the gastric balloon with application of traction usually stops the bleeding by compressing the vessels as they cross the gastrooesophageal junction. Rarely, inflation of the oesophageal balloon may be required. It is unlikely that repeat endoscopy would prevent on-going bleeding if he has already received successful sclerotherapy, unless there was doubt about the source of bleeding previously. Octreotide is less effective than terlipressin in controlling portal pressure, and therefore switching therapy is unlikely to be helpful. Although the effects of the fresh frozen plasma administered earlier will probably have worn off, vitamin K will probably not help as the coagulopathy is probably due to synthetic dysfunction rather than vitamin K deficiency. Surgical transection of the oesophagus is rarely indicated, and has largely been superseded by radiological shunt insertion (e.g. TIPSS). D : 81. An 81-year-old lady is admitted with a 2-week history of diarrhoea. Six weeks prior to admission she had been treated with a course of antibiotics. Clinical examination reveals dehydration. Her full blood count shows a neutrophilia and C-reactive protein is elevated at 120 mg/l (normal range <6 mg/l). A stool sample is positive for Clostiridium difficile. Which one of the following statements is correct? A : C. difficile infection is always newly acquired. B : The antibiotics given 6 weeks ago is not responsible for her present diarrhoea. C : Older people are at higher risk of developing C. difficile infection than younger people. D : Older people have similar rates of relapse after treatment as younger people. E : C. difficile does not disrupt intercellular tight junctions in bowel mucosa.

Page 180: Masterclass Book Part 2

MOHAMMED IS-HAG 179

C. difficile diarrhoea may be newly acquired or may be due to endogenous overgrowth following disruption of the normal flora, usually by antibiotic therapy.Symptoms may develop as early as one day after exposure to antibiotics, although typically they develop 5 to 10 days after initiation of antibiotic therapy.They can also arise as late as10weeks after cessation of antibiotic treatment.Older patients are at higher risk of developing C. difficile diarrhoea and having recurrent disease than younger people.C. difficile Toxin A causes disruption of the intercellular tight junctions causing fluid secretion. C : 82. A 36-year-old woman is admitted with a history of cough and shortness of breath of 8 weeks' duration. Her chest radiograph shows bilateral mid-zone infiltrates and blood count shows a raised eosinophil count. Which two of the following drugs might be responsible for her condition? A : Bleomycin B : Amiodarone C : Calcium tablets D : Prednisolone E : Heroin F : Insulin G : Cyclophosphamide H : Busulfan I :Sulfasalazine J : Paracetamol. Comment : Drug-induced pulmonary eosinophilia is the most common type of pulmonary eosinophilia seen in the western world. Drugs that can be responsible include ampicillin, aspirin, captopril, bleomycin, carbamazepine, dapsone, ethambutol, gold, methotrexate, penicillin, penicillamine, sulphonamides (including sulfasalazine), tamoxifen and tetracycline. A:I: 83. A 45-year-old man with a past history of chronic alcohol abuse presents is brought to A&E after vomiting one litre of fresh red blood. Blood pressure on arrival is 90/40 with pulse 110 /min. He is noted to be jaundiced, with multiple spider naevi and to have marked ascites. Which of the following drugs is most likely to be beneficial in his initial treatment? A : Intravenous ranitidine B : Oral propranolol C : Intravenous terlipressin D : Intravenous proton pump inhibitor E : Helicobacter eradication therapy. This patient has evidence of chronic liver disease with portal hypertension, so that a diagnosis of bleeing oesophageal varices should be strongly considered.Intravenous terlipressin has been shown to reduce variceal pressure following bleeding, which may influence prognosis. Propranolol is beneficial in primary and secondary prevention of variceal haemorrhage in the long term, but has no role in the acute setting.Intravenous proton pump inhibitors may be beneficial in patients with actively bleeding peptic ulceration, but have no role in variceal bleeding. Intravenous ranitidine has never been shown to affect outcome following GI bleeding of any type, while eradication of helicobacter will only be important in the short term if he is shown at endoscopy to have peptic ulceration as a cause for his bleeding. C : 84. A 23-year-old woman presents with a 6-week history of bloody diarrhoea opening her bowels ten times daily, and three times at night. She is tachycardic and has abdominal pain. She has no other previous medical history. She is a non-smoker. Which one of the following statements is true? A : Stool culture is not necessary as the diarrhoea has been present for 6 weeks. B : Sigmoidoscopy is mandatory. C : Histology reliably differentiates infection from inflammatory bowel disease. D : Plain abdominal radiograph should not be performed as she is of child-bearing age. E : Colectomy is the treatment of choice. Comment The likelihood is that this lady has inflammatory bowel disease. Bloody diarrhoea suggests colonic involvement and the fact that she is a non-smoker would make ulcerative colitis more likely. It is essential to take stool cultures, as this may be a primary infective colitis. Alternatively infection may co-present with a flare-up or initial presentation of ulcerative colitis and needs treating in its own right.Endoscopic examination of the colon is mandatory. It allows visualisation of the colonic mucosal and biopsy for histological examination. It may determine the extent of the disease. Although histology may be diagnostic, it is sometimes very difficult to exclude infection as a cause. Bowel frequency of thirteen times daily with tachycardia and abdominal pain would indicate severe colitis. Such a patient should be admitted for intravenous steroid therapy. Approximately 70% of patients will respond to steroids and thus avoid colectomy.A plain abdominal radiograph is essential to rule out the complication of toxic megacolon, although clinical judgement may be used if pregnancy is confirmed. B :

Page 181: Masterclass Book Part 2

MOHAMMED IS-HAG 180

85. A 34-year-old women presents with a 2-year history of intermittent non-bloody diarrhoea. She is currently undergoing IVF treatment. Last year she suffered a Colles’ fracture of the right arm, and radiographs at the time commented on the presence of osteopenia. She is known to be IgA deficient. Which one of the following statements is correct? A : The most likely diagnosis is Crohn’s disease. B : Her anti-endomysial antibody test is highly likely to be positive. C : The clinical picture is consistent with lactase deficiency. D : Gastroscopy with duodenal biopsy is indicated. E : A barium enema is likely to produce helpful information. Comment : The clinical picture fits best with coeliac disease and duodenal biopsies are the gold standard for diagnosis. Anti-endomysial antibodies are sensitive and specific, but miss the disease in about 5% of the population who are IgA deficient. A barium enema is unlikely to be helpful given the non-bloody nature of the diarrhoea; a small bowel meal may be more helpful to eliminate the differential diagnosis of Crohn’s disease. D : 86. A 38-year-old woman weighs 103 kg and is 1.65m tall (BMI=37.8). Which of the following statements is true? A : She should aim to consume a diet that produces a daily negative energy balance of about

500kcal. B : She should be encouraged to continue to smoke in order to supress her appetite. C : Orlistat (pancreatic lipase inhibitor) is the easiest way for her to lose weight. D : There is no restriction on alcohol consumption so long as it is in the form of wines or spirits. E : Her life expectancy is normal. Comment : Up to 20% of the UK population are clinically obese (i.e. BMI>30). This has serious consequences both on mortality and morbidity. Management is based around a negative energy balance as a consequence of long-term low-intensity exercise and diet (500–600 kcal/day defecit). Crash and fad diets are unlikely to be sustained. All alcohol should be minimised as it contains a significant number of calories though is devoid of nutrition. Drug treatments should only be considered where behaviour modification has failed, and are only likely to work alongside exercise and diet. A : 87. A 65 year old man with a history of rheumatoid arthritis (RA) develops a left pleural effusion. Which of the following are typical of rheumatoid pleural effusions? A : Over 50% of patients with RA develop a pleural effusion at some stage. B : It is more commonly seen in female patients. C : The glucose level in the effusion fluid is less than 1.6mmol/l. D : The effusions mainly occur in patients without rheumatoid nodules. E : The effusions are usually transudates. F : Intrapleural corticosteroids is the treatment of choice. G : They are usually bilateral. H : They have high levels of cholesterol. I : The pH of pleural fluid in an effusion due to RA is usually > 7.30. J : They should be treated with a chest drain. Comment : Twenty per cent of patients with RA develop pleurisy and only 5% develop pleural effusions. It is commonly seen in older men with rheumatoid nodules and is usually an exudate. RA is unlikely to be the cause of an effusion if the glucose level is above 1.6. These effusions tend to have high cholesterol levels. C : H: 88. A 28-year-old woman has fistulizing Crohn's disease. Which one of the following statements is true? A : Fistulogram is indicated in peri-anal disease to assess communication with the bowel. B : Medical or surgical therapy is always indicted. C : TPN with bowel rest is a recognised treatment. D : Peri-anal sepsis requires aggressive surgical intervention. E : Repeated courses of infliximab are safe and effective for severe disease. Comment : Azathioprine and 6 mercaptopurine have been demonstrated to be effective in closing fistulae in trials as well as a meta-analysis, and these should be considered standard therapies in patients in whom fistulae are troublesome (Pearson, May et al, 1995). Asymptomatic perianal fistulae in the context of Crohn's disease, with or without minor discharge, do not necessarily require specific treatment. If there is large volume discharge or recurrent sepsis, then investigation in the form of pelvic MRI scan, endoanal ultrasound and examination under anaesthesia (EUA) is warranted. EUA also allows the laying open of fistulae and the placement of Seton sutures through the fistulous tracts. The Setons prevent the fistulous tract from closing down, and thus prevent the formation of abscess cavities along the tracts. This is particularly important to prevent destruction of the anal sphincters, and timely conservative surgical management is of key importance. Aggressive surgical management of fistulising Crohn's disease is usually inappropriate. Defunctioning ileostomy, with diversion of the faecal stream, may allow resolution of perineal sepsis, but there is a high incidence of relapse when bowel continuity is restored. In clinical trials infliximab has been shown to be effective in closing fistulae in 46% when given

Page 182: Masterclass Book Part 2

MOHAMMED IS-HAG 181

at Weeks 0, 2 and 6 at a dose of 5 mg/kg (Present, Rutgeerts et al, 1999). However, if possible, it should be used in concert with an immunosuppressant treatment such as azathioprine or methotrextate, both to reduce the likelihood of provoking an immune reaction to infliximab (which is a chimeric antibody derived from both human and mouse antibodies) and to hold disease in remission induced by infliximab. Repeated infusions of infliximab can be used, but efficacy tends to diminish over time and the long term biological consequences of this approach are uncertain. TPN may be indicated particularly for high fistulae in the post-surgical setting. Active Crohn's disease must be treated, the patient often kept nil by mouth and proton pump inhibitors and octreotide used to help reduce volume of fistula output. Fistulogram is rarely helpful in delineating fistulous tracts. MRI and EUA are generally much better. C : 89. A 45-year-old man with a long history of reflux symptoms is referred for open access endoscopy. The endoscopist notes a red velvety lining in the distal 10cm of the oesophagus. Biopsies from this area are reported as showing columnar epithelium with intestinal metaplasia. Which one of the following statements is true? A : This is a rare condition occurring in approximately 0.1% of patients endoscoped for reflux

symptoms. B : There is approximately a 1% risk per annum of the patient developing oesophageal carcinoma. C : Proton pump inhibitors may cure the condition over time. D : It is an adaptive response to chronic reflux and is resistant to ulceration. E : It may be precipitated by chronic non-steroidal anti-inflammatory (NSAID) ingestion. Comment :Barrett’s oesophagus describes a replacement of the normal oesophageal squamous epithelium by a metaplastic columnar epithelium as a response to chronic reflux. It is relatively common, with a prevalence of around 1% in autopsy studies and around 10% in patients undergoing gastroscopy for reflux symptoms. The intestinal type of metaplasia has premalignant potential and adenocarcinoma develops in approximately 1% per year. Proton pump inhibitors may relieve symptoms but do not effect cure. B : 90. A 65-year-old man complains of epigastric pain and difficulty swallowing. At endoscopy Grade I reflux oesophagitis is seen. Which one of the following statements is NOT true? A : Acid suppression with a proton pump inhibitor should relieve symptoms. B : Repeat endoscopy is required to ascertain response to treatment. C : Helicobacter eradication is not needed unless there is associated duodenitis. D : Raising the head of the bed and avoiding eating/drinking 3 hours prior to going to bed can aid symptoms. E : The lowest dose of proton pump inhibitor should be used in the long term to treat his symptoms. Comment : Reflux oesophagitis is very common and easily treated with simple postural measures and acid suppression.Most patients respond to treatment and there is no need to repeat endoscopy in every case. Severe disease, presence of oesophageal ulceration and follow up of Barrett’s oesophagitis are reasonable indications for repeat endoscopy.There is no convincing evidence for routine Helicobacter eradication in this clinical situation and the lowest dose of acid suppression should be given when symptoms have settled, if any is required at all. B : 91. A 45-year-old woman known to have alcoholic liver disease presents in a hypotensive, tachycardiac and confused state. She shows evidence of haematemesis and melaena. Emergency endoscopy shows fresh blood in the oesophagus with bleeding varices. Sclerotherapy and banding are attempted but the patient remains unstable. Which of the following statements is NOT true? A : Intubation should be considered if a safe airway cannot be maintained. B : Oral prednisolone should be prescribed for acute alcoholic hepatitis. C : Intravenous antibiotics (e.g. Cefotaxime) should routinely be prescribed to patients with variceal

haemorrhage. D : If further bleeding occurs a Sengstaken tube may be required. E : Glypressin (a synthetic analogue of vasopressin) should be prescribed to reduce splanchnic and

portal blood flow. Comment : In patients with cirrhosis and portal hypertension, gastrointestinal haemorrhage is a major complication and cause of death. Oesophageal varices are present in approximately 50% of patients with cirrhosis. Among patients with varices, the risk of gastrointestinal haemorrhage ranges from 30% to 50% and half of these patients die within 6 weeks after bleeding.Normal care includes (in addition to resuscitation) airway management, routine antibiotics, early endoscopy along with pharmacotherapy with either vasopressin or octreotide (or related compounds). Glypressin is now the widely used drug. Vasopressin and glypressin reduce portal pressure by splanchnic arteriolar vasoconstriction, whereas somatostatin and octreotide lower portal pressure by decreasing hepatic blood flow.Temporary tamponade can be achieved with a Sengstaken-Blackmore tube. It should be considered as a salvage procedure. Tamponade is 90% successful at stopping haemorrhage. Unfortunately 50% patients re-bleed within 24 hours of removal of tamponade. Prednisolone has a role, after careful consideration, in patients with alcoholic hepatitis but not in the acute management of variceal haemorrhage. B :

Page 183: Masterclass Book Part 2

MOHAMMED IS-HAG 182

92. A 68-year-old woman presents with a six-month history of diarrhoea and weight loss. Extensive investigation eventually culminates in the finding of multiple diverticula on a small bowel enema. The first-line treatment should be: A : Tetracycline or augmentin for 10 days B : Laparotomy and resection of the affected bowel C : Metronidazole for 10 days D : Vancomycin for 10 days E : Trial of lactose-free diet. Comment : The presence of bacterial overgrowth in the small bowel is usually suspected on the result of a breath test. Bacterial fermentation of an oral dose of carbohydrate releases hydrogen, and of an oral dose of conjugated bile acid (with radiolabelled carbon in the amino portion) releases radiolabelled carbon dioxide. If there is an increase in bacteria in the upper small bowel, then an early peak in hydrogen or carbon dioxide can be detected in expired air.The diagnosis of bacterial overgrowth is confirmed if there are >100,000 mixed bacteria / ml in a duodenal aspirate. Small bowel enema will detect diverticula or strictures as a cause for overgrowth.Initial treatment is with 10 days of augmentin or tetracycline. Prolonged courses may be needed, and cyclical antibiotics for one week every four weeks may be used. Laparotomy / resection would only be contemplated if these manoeuvres failed, and if anatomical abnormality was confined to a relatively small segment of bowel. A : 93. A patient with progressive bulbar palsy is referred for placement of a percutaneous enterogastrostomy tube for feeding. Which of the following is correct? A : There is a significant risk of bleeding, infection, and pain following the procedure, and a

substantial risk of procedure-related mortality. B : The procedure requires a general anaesthetic and is most efficiently performed by means of a

small laparotomy. C : The procedure is ideally performed under radiological guidance, which obviates the need for

endoscopy and sedation. D : The main advantage for this patient would be the reduced risk of aspiration of gastric contents. E : In view of the likelihood that the patient has a neurological condition such as motor-neurone disease, blood

gas measurement should be obtained before the procedure is undertaken. Comment : PEG tube placement is often requested from the gastroenterology and endoscopy service because endoscopic placement of the tube obviates the need for general anaesthesia and surgery, and because the durability of endoscopically-placed tubes is greater than that of radiologically-placed tubes. However, there is a significant risk of complications, including life-threatening bleeding and infection. This should be explained to the patient and their carers as part of the informed consent process. Although bypassing the need to swallow may reduce aspiration caused by disordered swallowing, a PEG tube does not prevent regurgitation of gastric contents, or their aspiration into the larynx, trachea, and bronchial tree. It is quite correct to be wary of respiratory difficulties occurring in patients with neurological disorders who undergo endoscopy. However, normal blood gas measurements could provide false reassurance – the critical measurement is the patient’s ventilatory reserve, best determined by measuring their vital capacity. A : 94. A 30-year-old man presents with progressive central abdominal pain and vomiting associated with significant weight loss (five stone in 3 months).He gives a history of binge drinking and depression, and smokes twenty cigarettes per day. Because eating provokes abdominal pain and vomiting, he has eaten virtually nothing for a month. CT scanning of his abdomen showed a normal pancreas but dilated loops of small bowel with a possible terminal ileal stricture. His albumin level was 20 and C-reactive protein level was 50.Which statement is NOT true? A : His symptoms are consistent with a Crohn’s stricture. B : Chronic pancreatitis is very unlikely in view of the normal CT. C : Infliximab should be prescribed as soon as possible. D : Regular monitoring of his electrolytes is indicated in view of a risk of refeeding syndrome. E : He should be informed of a risk of a stoma if surgery is indicated. Comment : This gentleman had two strictures at operation (right hemicolectomy) which were due to Crohn’s disease.The best treatment for such a patient is surgical and not medical. There is no clear evidence to suggest that post-operative medical therapy reduces the risk of recurrent Crohn’s disease necessitating surgery, although typically Pentasa and/or immunomodulators such as Azathioprine are prescribed. However, abstinence from smoking should be encouraged as the rate of recurrence is reduced by approximately 50% in non-smokers.In view of his recent near-total starvation and significant weight loss, refeeding syndrome is a possibility; careful attention to his electrolyte levels, gradual re-introduction of feeding, and replenishment of B vitamins is required. All patients undergoing colonic surgery need to be aware of the possibility of stoma formation, even if, as in this case, it is unlikely. C :

Page 184: Masterclass Book Part 2

MOHAMMED IS-HAG 183

95. A 46-year-old man presents with jaundice. Dipstick testing of his urine reveals the presence of bilirubin but no urobilinogen. This means that: A : jaundice is likely to be pre-hepatic B : bile must be flowing freely into the gut C : jaundice cannot be obstructive D : renal function is normal E : there must be complete obstruction to bile flow. Comment : When haemoglobin is broken down the porphyrin ring is converted into biliverdin and thence to bilirubin.Unconjugated bilirubin is relatively insoluble and is transported in the blood as a complex with albumin. It is not excreted in the urine.Hepatocytes take up unconjugated bilirubin and conjugate it to form the soluble diglucuronide, which is excreted into the bile.Further metabolism by gut bacteria forms the soluble colourless compound urobilinogen. Some of this enters the blood stream and is excreted in the urine. Urobilinogen remaining in the gut is converted to the brown pigment, urobilin, and is excreted. If a patient with jaundice has no urobilinogen in their urine, then they must have complete obstruction to bile flow. E : 96. A 55-year-old man is still anaemic, has some loose stools and has failed to regain weight several months after starting a gluten-free diet for coeliac disease. Which of the following is the commonest cause of failure to respond to treatment? A : Pancreatic insufficiency B : Dietary non-compliance C : Small bowel adenocarcinoma D : Enteropathy associated B cell lymphoma of the small intestine E : Ulcerative jejunitis. Comment : Poor dietary compliance accounts for most cases where there is a failure to improve on treatment. Ulcerative jejunitis, small bowel adenocarcinoma and an enteropathy-associated T cell lymphoma are recognised but rare complications and there is an association with pancreatic sufficiency. B : 97. A 40-year-old female nursing home resident recently in hospital with Salmonella gastroenteritis is readmitted with further diarrhoea. In her management plan, which statement is LEAST appropriate? A : Rehydrate orally or intravenously depending on clinical and biochemical assessment. B : Nurse her in isolation. C : X-ray her abdomen and measure inflammatory markers such as ESR/CRP. D : Send stool on multiple occasions for culture and sensitivity, including C. difficile toxin. E : Request a colonoscopy as an inpatient. Comment : Not all episodes of infectious diarrhoea need treating with antibiotics, and often, resistance to certain antibiotics (e.g. ciprofloxacin) leads to sub-optimal treatment. Any patient who has been in hospital recently is at risk of C. difficile infection - especially if they may have had antibiotics. This needs to be actively tested for and treated appropriately (metronidazole as first-line treatment, then oral vancomycin). Hence, it is imperative in managing patients with diarrhoea to ensure adequate and suitable stool samples are sent early in the illness. Colonic imaging is usually not indicated in infective colitis (other than a simple X-ray to exclude toxic megacolon) and is only necessary if symptoms persist and stool cultures are negative. E : 98. Which of the following is true of anorectal abscess: A : It is more common in males B : It is most commonly caused by Crohn’s disease C : It is typically caused by infection of minute cracks in the peri-anal skin D : It is more common in those who practice receptive anal intercourse E : It usually responds to antibiotic treatment. Comment : Anorectal abscesses generally develop from infections of the anal glands, which lie in the intersphinteric space between the internal and external anal fissure, and drain into the anal canal via a duct that opens onto the dentate line. The condition is not associated with any sexual practices, although other peri-anal conditions, such as ulcers and warts are, but for unknown reasons, it is more common in men. Crohn’s has a prevalence of about seven per 100 000, and about a third of cases may involve the anorectal region with abscesses and fistulae. However anorectal abscess are more common. Most abscesses require incision and drainage, with or without antibiotic cover. There is a risk of persistent fistula formation after the abscess has drained. A : 99. Regarding Crohn’s disease: A : 5 aminosalicylic acid (5-ASA) compounds reduce the rate of relapse by about 30%. B : Finger clubbing is a recognised feature. C : Unlike ulcerative colitis, there is only a small genetic risk of developing the condition. D : Peri-anal inflammation is almost universally present. E : Patients should be encouraged to stop smoking, although this may increase the risk of relapse.

Page 185: Masterclass Book Part 2

MOHAMMED IS-HAG 184

Comment : Crohn’s disease can affect any part of the intestinal tract, and three forms predominate: terminal ileal disease, colitis, and peri-anal Crohn’s. Peri-anal disease is the most infrequent (about 25%). Crohn’s is frequently associated with a positive family history (in up to 15% of patients), and the genetic component is probably greater than for ulcerative colitis. Smoking increases the risk of relapse, and 5-ASA compounds only have a modest impact on acute or chronic disease. B : 100. A 34-year-old man is referred because of abnormal liver function tests (LFTs). Which of the following are true? A : An ALT:AST ratio of over 2.0 is suggestive of alcohol as a cause. B : Raised IgA is not uncommon in alcoholic liver disease. C : An AST of over 500 IU/l is typical of acute alcoholic hepatitis. D : Hepatis steatosis would rarely be the cause. E : Non-alcoholic steatohepatitis (NASH) invariably has a benign course. F : Obesity and type 2 diabetes are factors commonly associated with NASH. G : Mallory’s hyaline does not occur in NASH. H : Steroids are indicated if liver histology shows active inflammation in NASH. I : A raised ferritin almost always indicates genetic haemachromatosis (GH). J : Over 90% cases of GH in the United Kingdom are C282Y homozygotes. K : A low iron diet is mandatory in GH. L : GH has a low risk of complicating hepatoma compared to other causes of cirhosis. M : Negative anti-hepatitis C antibody assay excludes this as a cause. N : Primary biliary cirrhosis is not the cause in a male patient. O : Primary sclerosing cholangitis should be suspected if the patient has coeliac disease. Comment : An AST:ALT ratio of over 2.0 is often found in alcoholic hepatitis, with an AST rarely above 400IU/l. Hepatic steatosis (or fatty liver) is a common cause of mild LFT abnormality. When active inflammation is present (non-alcoholic steatohepatitis), as many as 40% may progress histologically and some may develop cirrhosis, particularly those with evidence of balloon degeneration, Mallory’s hyaline or fibrosis. Some improvement in LFT can be seen following efforts to lose weight and exercise. Ferritin is an acute phase protein, so is often raised in alcoholic hepatitis. All suspected genetic haemochromatosis (GH) patients should be genotyped: 93-95% are C282Y homozygotes. Venesection of one unit of blood removes 250mg iron and maximum absorption is 1mg per day, so dietary intervention is limited to avoiding excess iron. Hepatoma most commonly complicates cirrhosis due to GH; hepatitis B and C. Polymerase chain reaction may detect hepatitis C RNA in the presence of negative hepatitis C serology in acute infection or in the presence of immunodeficiency/immunosuppression. Primary sclerosing cholangitis is associated with ulcerative colitis. B : F:J: 101. A 47-year-old woman is referred with iron deficient anaemia. She has no gastrointestinal symptoms. Which of the following are true? A : Coeliac disease is very unlikely in the absence of symptoms. B : Raised serum anti-gliadin antibody levels are 95% sensitive for the diagnosis of coeliac disease. C : The prevalence of coeliac disease in the UK approaches 1 in 200. D : Howell Jolly bodies in the blood film suggest haemolysis. E : Antibody profiles take at least 1 year on a strict gluten-free diet to become negative. F : Osteoporosis is a recognized complication of coeliac disease. G : Coeliac disease patients with ulcerative jejunitis tend to be at lower risk of malignancy. H : Barium follow-through invariably shows increased flocculation of barium and coarser mucosal

folds. I : A single biopsy is adequate for the diagnosis of coeliac disease provided it is obtained distal to the

duodenal bulb. J : Pancreatic insufficiency is the commonest cause of failure to improve on a gluten-free diet. Comment : Anaemia is a common presentation of coeliac disease. Raised anti-endomysial antibodies have a 95% sensitivity for the diagnosis of coeliac disease, which affects up to 1 in 200 of the UK population. Howell Jolly bodies are a feature of hyposplenism which is associated with coeliac disease. Barium studies may be normal and as the histological changes are patchy, four biopsies are advised. Endomysial antibody levels usually fall within 6 weeks of dietary treatment and non-compliance is the commonest reason for an absence of clinical or serological response. Osteoporosis is a common complicating factor and improves following institution of the diet. Ulcerative jejunitis is associated with an increased risk of enteropathy associated T cell lymphoma. C:F: 102. Which of the following statements concerning liver transplantation is correct? A : Donor and recipient should have three matches on human leukocyte antigen (HLA) tissue-typing B : Is most commonly performed for cases of acute liver failure C : Overall one-year survival rates are approximately 60% D : HIV infection is not necessarily a contra-indication to transplantation E : Xenotransplantation is widely used in Japan. Comment :Donor and recipients are matched on the basis of blood group and body size - unlike renal transplantation, HLA matching is not required. Approximately 15% of liver transplants are performed for

Page 186: Masterclass Book Part 2

MOHAMMED IS-HAG 185

acute liver failure, the remainder are performed in patients with chronic liver disease.Overall one-year survival rates currently run between 80 and 95%. One-year survival rates are lower in those patients transplanted with acute hepatic failure, but these only form the minority of transplants.Improvement in prognosis for HIV-infected patients as a result of new anti-retroviral therapies mean that under exceptional circumstances HCV co-infected patients may be considered for transplantation. Xenotransplantation (the use of genetically modified animal donors) remains a theoretical advance in transplantation and at the present time, no human trials have taken place. Religeous objection to organ donation in Japan means that the majority of liver transplant grafts are obtained from living donors. D : 103. A 35-year-old woman presents with abdominal pain and vomiting. She has been unwell for about six weeks with anorexia, nausea and weight loss. Examination reveals a tender palpable mass in the right iliac fossa. The two most likely diagnoses are: A : Psoas abscess B : Tuberculosis C : Abdominal secondaries D : Crohn’s disease E : Campylobacter F : Caecal carcinoma G : Hypernephroma H : Appendix mass I : Ovarian carcinoma J : Lymphoma. Comment : The presentation with gastrointestinal symptoms occurs on a background suggesting a systemic component to the illness, hence the most likely diagnoses are an inflammatory mass associated with Crohn’s disease or an appendix abscess. In an older patient caecal carcinoma would be a more likely explanation.Aside from assessment of cardiovascular status (is resuscitation required?) and for the presence of peritonism (which would suggest a perforated viscus and the need for laparotomy), examine carefully for signs that would support a diagnosis of Crohn’s disease. These include clubbing, aphthous ulceration, perianal skin tags / ulceration / fistulae and (less commonly) seronegative arthritis, sacroiliitis, iritis and skin rashes (erythema nodosum, pyoderma gangrenosum). D : H: 104. A 37-year-old building labourer presents with heartburn and acid regurgitation. Which of the following statements are true? A : Oesophagogastroduodenoscopy (OGD) is required in order to diagnose gastro-oesophageal reflux disease

(GORD). B : First-line management should be a proton-pump inhibitor. C : He should be encouraged to drink milk at bedtime. D : It is essential to eradicate Helicobacter pylori for optimal symptom control. E : The patient should be encouraged to stop smoking. F : A surgical corset may improve posture and hence symptoms. G : Oesophageal pH monitoring may be useful in managing drug-resistant cases. H : All patients with Barrett’s oesophagus should be entered into a surveillance programme. I : There is an increased incidence of oesophageal cancer in patients with a long history of gastro-oesophageal

reflux disease. J : Surgical fundoplication optimises symptom control and is the treatment of choice in the majority of cases. Comment :GORD is a clinical diagnosis based on history. Only a minority of patients with GORD will have endoscopic oesophagitis, and a normal OGD does not exclude the diagnosis. First-line management should be lifestyle advice, followed by simple antacids. A proportion of patients will not require a proton-pump inhibitor.Although milk has a short-term alkali effect, its fat content and presence in the stomach will stimulate gastrin and acid production. Patients should be advised to eat or drink nothing for three or four hours before retiring. Belts and corsets increase intra-abdominal pressure and will exacerbate symptoms, so they should be discouraged. Not only does nicotine relax the lower oesophageal sphincter, but also smokers produce less saliva, one of the clearing mechanisms for transient lower oesophageal sphincter relaxations.H. pylori has no role in GORD, and indeed some studies suggest eradication may worsen symptoms. Oesophageal pH monitoring is not sufficiently available for routine diagnosis, but it may be useful in confirming the diagnosis in apparently resistant cases. The debate over Barrett’s surveillance continues, though none would argue for it in short-segment disease or the elderly.While in patients with oesophageal cancer there is an increased history of reflux symptoms, the reverse is not true. Any association has yet to be proven. While surgical (or endoscopic) fundoplication may be beneficial in a small number of drug-resistant cases, there is significant morbidity (gas bloat, flatulence etc) and mortality. Symptoms also frequently recur over time requiring a return to medical therapy. E :G:

Page 187: Masterclass Book Part 2

MOHAMMED IS-HAG 186

105. A 43-year-old woman has been diagnosed as suffering from bronchiectasis on a high-resolution computed tomography (HRCT) scan of the lung. Which one of the following statement is NOT true? A : Her immunoglobulin levels should be checked. B : She is at risk of developing a pneumothorax. C : Massive haemoptysis is the commonest cause of death in her age group. D : She is at risk of developing brain abscess . E : Recurrent chest infections are unlikely at this age. Comment : Congenital and acquired immunodeficiency are well known causes of bronchiectasis, hence serum immunoglobulins should be checked, particularly the IgG subclass (IgG1,2,3 & 4). Patients with immunoglobulin deficiency should be treated with regular immunoglobulin infusion.Pneumothorax, recurrent chest infections and brain abscess are well recognised complications.Haemoptysis is a common symptom but rarely causes death. C : 106. A 54-year-old Caucasian gentleman is referred with mildly abnormal liver function tests (ALT 74 U/l [5-45], AST 63 U/l [5-40]). He has been complaining of tiredness and aches and pains in his knees and his GP therefore tested his full blood count (FBC), ferritin and sugar which show: Hb 15.5g/dl [12.5-15.5], ferritin 3452 micrograms/litre [12-200], fasting glucose 13.2 mmol/l [<11]. Which two tests are most useful in establishing a diagnosis? A : Autoimmune profile B : Skeletal survey C : HbA1C D : Liver biopsy E : HFE gene test F : Hepatitis A IgM serology G : Duodenal biopsies H : Haemoglobin electrophoresis I : Radiograph of knees J : Serum uric acid. Comment : The massively raised ferritin, diabetes and arthralgia (chondrocalcinosis) strongly suggest that this man has genetic haemochromatosis. Liver biopsy with estimation of hepatic iron stores is the diagnostic test. Over 90% of affected individuals are homozygous for the HFE Cy282 mutation, and if found screening should be offered to relatives. Coeliac disease may present with mildly abnormal liver function tests, but is associated with iron (and folate) deficiency. D : E : 107. Regarding aspirin and colorectal cancer (CRC), which of the following statements is true? A : Regular aspirin reduces the risk of CRC by approximately 50%. B : Regular aspirin reduces the risk of adenoma formation by approximately 50%. C : Regular aspirin reduces the risk of mortality from CRC by approximately 50%. D : Regular aspirin reduces the risk of mortality in patients with CRC by approximately 50%. E : Regular aspirin reduces the risk CRC in ulcerative colitis by approximately 50%. Comment :Regular aspirin, and other non-steroidal anti-inflammatories (NSAIDs) such as sulindac, consistently show a strong protective effect against the formation of adenomatous polyps in patients with familial polyposis, as well as in the general population. The protective effect is also shown in people at high risk of adenoma formation, for example following resection of CRC, where the benefit has been measured in clinical trials. However, there are no data on actual rates of CRC, or on mortality from CRC, as they would require much longer-term follow-up. B : 108. A 56-year-old man presents with iron deficient anaemia. Which of the following are true? A : Gastritis identified at gastroscopy is an acceptable cause and avoids the need for further investigation. B : Coeliac disease is an unlikely cause in the absence of other symptoms. C : Investigations of both upper and lower gut are recommended. D : Ferrous sulphate can be stopped when the haemaglobin is within normal range. E : Three negative faecal occult blood tests exclude a gastrointestinal cause. F : In the absence of symptoms, a sustained response to a three-month course of iron supplements

means that gastrointestinal investigation is unnecessary G : Basophilic stippling on the blood film suggests haemolysis. H : Howell Jolly bodies on the blood film suggests portal hypertension and liver disease. I : Iron deficiency may occur in association with pernicious anaemia. J : Colorectal polyps do not cause iron deficient anaemia. Comment : Both men and post-menopausal women are recommended to have both upper and lower gastrointestinal investigations for iron deficient anaemia as dual pathology occurs in 5-10% of cases. Gastritis of varying severity is a common endoscopic finding and would not readily be accepted as a cause for anaemia without further investigation. Large colorectal polyps may bleed sufficiently to cause anaemia. Coeliac disease commonly presents with iron deficient anaemia without overt gastrointestinal symptoms and the blood film may show Howell Jolly bodies and target cells, features of hyposplenism.

Page 188: Masterclass Book Part 2

MOHAMMED IS-HAG 187

Basophilic stippling classically occurs in lead poisoning. In an achlorhydric stomach, there is impaired conversion of ferric iron to the better-absorbed ferrous form, and iron deficiency may occur. Although haemoglobin level may improve by over 0.5g/dl per week, 3-4 months iron supplements are needed to replenish iron stores. C : I: 109. A 54-year-old man with a long history of ulcerative colitis and primary sclerosing cholangitis presents with obstructive jaundice, weight loss, fever and abdominal pain. Endoscopic retrograde cholangiopancreatography (ERCP) reveals a tight stricture of the common bile duct. Which are the two most appropriate responses? A : CA19-9 is highly specific and sensitive for cholangiocarcinoma. B : Cytology brushings at ERCP have a 95% pick-up rate. C : Drainage of the biliary system may be achieved percutaneously. D : There is no need for cross sectional imaging as the diagnosis is clear from the ERCP. E : He should be commenced on broad-spectrum antibiotics. F : Cholangiocarcinomas never metastasise. G : Patients with cholangiocarcinoma should be referred urgently for consideration of liver

transplant. H : A metal stent should be inserted while awaiting definitive surgery. I : There is no place for radical surgery in the management of cholangiocarcinoma. J : Five-year survival rate for cholangiocarcinoma is approximately 30%. Comment : CA19-9 is neither highly sensitive nor specific, and cytology brushings are positive in only 50-70% of cases. Cross-sectional imaging is required to stage the disease, identify metastases and plan management. Ascending cholangitis is common and should be treated appropriately. Metal stents, while remaining patent for longer than plastic stents, are difficult to remove and should only be placed once surgery has been excluded. Where drainage cannot be obtained at ERCP, percutaneous transhepatic cholangiography (PTC) may be necessary. Cholangiocarcinoma is an absolute contraindication to transplantation as outcome is universally poor. Two-year survival is under 20%. C : E : 110. A 17-year-old girl with diarrhoea has evidence of villous atrophy on her duodenal biopsies. Which two of the following are NOT associated with villous atrophy? A : Coeliac disease B : Whipples disease C : Chronic pancreatitis D : Lymphoma E : Carcinoid syndrome F : Starvation G : Tropical sprue H : Giardiasis I : Hypogammaglobulinaemia J : Bacterial overgrowth. Comment :The classic cause of duodenal villous atrophy is coeliac disease, in which case it is due to intolerance of gluten, a group of proteins found in wheat, barley and rye. However, there are many other causes of villous atrophy, as listed above. C : E : 111. A 30-year-old asks for advice regarding his family history of colorectal cancer, which affected his father and brother. Which of the following are true? A : Hereditary non-polyposis colon cancer (HNPCC) gives a lifetime risk of colon cancer of 1:6. B : Barium enema is the investigation of choice. C : Colonoscopic surveillance should be offered D : Three negative faecal occult blood tests exclude a diagnosis of colon cancer. E : Colonoscopy causes perforation in 1:10 000 cases. F : Cancers in HNPCC predominantly affect the left colon. G : A carcinoembryonic antigen (CEA) level should be measured. H : He should have colonoscopy if his iron deficiency recurs after 4 months iron supplements. I : Endoscopic polypectomy may be followed by bleeding for up to 48 hours. J : His lifetime risk of colon cancer is about 1:6. Comment : There is a lifetime risk of 1 in 6 to an individual with two first-degree relatives with colon cancer. Colonoscopic surveillance is offered to those with a risk of 1 in 12 or greater (which includes those with one first degree and one second-degree relative or one first-degree relative under 45 years of age).HNPCC is an autosomal dominant condition with a lifetime risk of colon cancer of 1 in 2.Barium enema misses 10-20% of cancers and polyps over 1 cm in size, does not offer the opportunity of removing pathology identified and is not suitable for surveillance because of radiation exposure.Faecal occult blood is not detected in 30-50% of significant polyps and cancers. Colonoscopic perforation occurs in about 1 in 1000 examinations and post-polypectomy bleeding may occur up to 14 days later. Male patients should have upper and lower gastrointestinal investigations if anaemic, irrespective of response to iron. C : J:

Page 189: Masterclass Book Part 2

MOHAMMED IS-HAG 188

112. An 87-year-old lady is admitted after a fall. Physical examination shows her to be cachexic with a body mass index (BMI) of less than 18. Which of the following statements are true? A : Malnutrition is readily recognised in hospital. B : There is no clinical benefit to correcting under nutrition in older patients. C : Malnutrition develops when there is reduced nutritional intake only. D : Malnutrition does not increase length of stay in hospital. E : Malnutrition has no effect on pyschological well-being. F : Nutritional support is needed when patients have lost 25% of their body weight. G : Malnutrition impairs the cell mediated immune response. H : BMI can distinguish fat mass from lean mass. I : BMI may be misleading if there is muscle wasting due to motor neurone disease. J : The majority of hospital trusts ensure all patients are weighed at least once-weekly. Comment : Malnutrition is poorly recognised in hospital. There is clinical benefit to correcting under nutrition in older patients. Malnutrition may develop when intake is reduced, there is increased nutrient requirement or altered ability to utilise or absorb nutrients.Malnutrition increases length of stay in hospital and adversely affects psychological well-being (depression anxiety etc). Nutritional support is required when a patient has lost 5 to 10% of their body weight and further weight loss is likely.The cell-mediated immune response is impaired in malnutrition. BMI does not distinguish between fat mass and lean mass. BMI may be misleading if there is muscle wasting due to motor neurone disease. In 2001 less than 50 % of trusts weighed their patients on a weekly basis. G:I: 113. Two weeks after returning from a beach holiday in the Eastern Mediterranean a 28-year-old woman presents with jaundice, anorexia and nausea. She has an ALT of 1507IU/L, and no biliary dilatation on ultrasound. Which of the following comments are correct? A : The presence of HBsAb and absence of HBcAb implies acute hepatitis B infection. B : Irrespective of the history, she should be referred to an alcohol counsellor. C : Patients with acute hepatitis A should be admitted directly to a Liver Unit. D : She should be 'signed-off' from her job at a sandwich bar for the next three months. E : She should commence treatment with alfa-interferon as soon as possible. F : HAV IgG should be requested to confirm the diagnosis of acute hepatitis A. G : The cholestatic phase of hepatitis A may last for several months. H : She should be advised to be vaccinated before her return trip next year in order to prevent a

repeat occurrence. I : She will not be able to donate blood in the future. J : Mortality from acute hepatitis A is less than 1 in 2000. Comment : HBsAb+ve, HBcAb-ve implies hepatitis B vaccination. The findings in this woman are consistent with acute hepatitis A and the diagnosis should be confirmed with a positive HAV IgM. Patients do not routinely need admission, unless there is evidence of liver failure (rising PT, encephalopathy). Occasionally, transplantation is required though mortality is <0.04%. The majority of patients experience a variable cholestatic phase (as transaminases fall) before complete resolution and subsequent immunity. Faecal shedding of the virus has normally ceased before clinical disease is apparent. Scrupulous hand hygiene is essential to prevent transmission of the virus while shedding. Blood donations are screened for.previous exposure to hepatitis B and C but not A. G:J: 114. You follow up a 55-year-old male smoker in your clinic with stable emphysema. His FEV1 is 0.6 (22% predicted) and FVC is 2.3 (63% predicted) giving an FEV1:FVC ratio of 26%. His arterial blood gases show a PO2 of 7.1 and 7.2 on two separate occasions. He is mildly breathless at rest, but severely breathless on exertion and his saturation drops from 91% to 88%. Which of the following statements is true? A : The major cause for his breathlessness is hyperinflation not hypoxia. B : He has severe emphysema because his FEV1:FVC ratio is less than 40%. C : There is good evidence to suggest that he will benefit from long-term oxygen therapy. D : He should not be considered for pulmonary rehabilitation because he is a current smoker. E : He should have a yearly chest radiograph to look for lung cancer. Comment : Although hypoxic at rest, this is mild and not causing significant dyspnoea. His breathlessness worsens considerably without much change in oxygen tension, suggesting that the cause of his dyspnoea is hyperinflation of his chest which worsens on exertion.Severity of emphysema is defined by the British Thoracic Society (BTS) in relation to FEV1, not FEV1:FVC ratio. Mild is 60-80% predicted; moderate 40-60% and severe <40%. Evidence for long-term oxygen therapy (Medical Research Council and Nocturnal Oxygen Therapy Trials) is only available in non-smokers. Therefore, first, he may not benefit, and second he is at risk of burns if he carries on smoking. He will benefit from pulmonary rehabilitation. Furthermore, some rehabilitation courses offer smoking cessation therapy. There is no evidence to suggest annual chest radiographs will benefit any group of patients in terms of primary prevention. A :

Page 190: Masterclass Book Part 2

MOHAMMED IS-HAG 189

115. A young man presents with a history of weight loss, diarrhoea, intermittent rectal bleeding and abdominal pain. Which of the following diagnoses could possibly account for all of the symptoms? A : Ulcerative colitis B : Microscopic colitis C : Radiation enteritis D : Intestinal tuberculosis E : HIV infection F : Giardiasis G : Amoebiasis H : Tropical sprue I : Whipple’s disease J : Clostridium difficile-associated diarrhoea K : Chronic pancreatitis L : Thyrotoxicosis M : Secretory neuroendocrine tumour N : Bacterial overgrowth O : Laxative abuse. Comment : Diarrhoea-associated with weight loss usually indicates a chronic or severe pathology, and if there is also rectal bleeding, this narrows the differential diagnosis further. Abdominal pain is relatively non-specific, and unless it is a major feature of the presentation, rarely helps to distinguish one cause from another, although intestinal ischaemia causing blood diarrhoea may be particularly painful (mesenteric angina).Inflammatory bowel disease,radiation enteritis, and amoebiasis typically cause rectal bleeding, although it may be microscopic. Intestinal tuberculosis may also cause blood loss, although this is rarely macroscopic. Microscopic colitis, HIV-associated diarrhoea,giardiasis, tropical sprue, Whipple’s disease, and C. difficile-associated diarrhoea typically do not cause rectal bleeding, and other causes should be considered first. Chronic pancreatitis typically causes steatorrhoea, while thyrotoxicosis, secretory neuroendocrine tumours and laxative abuse cause watery diarrhoea. Bacterial overgrowth may cause steatorrhoea.A: C:G: 116. A 56-year-old man with hepatitis B cirrhosis is offered 6-monthly ultrasound screening to detect hepatocellular carcinoma (HCC). What two pieces of advice are correct? A : The risk of hepatocellular carcinoma is 1-3%/year. B : Chemoembolisation of the tumour is curative. C : Liver transplantation can cure single tumours less than 10cm. D : Small tumours <3cm can be ablated with ethanol. E : An elevated AFP is diagnostic of HCC. F : The risk of HCC is less in men than women. G : Tumours occur less often in hepatitis B cirrhosis than in alcoholic cirrhosis. H : Most tumours are hypovascular. I : Six-monthly CT scans are an alternative screening tool. J : Screening has been shown to be effective in controlled clinical trials. Comment :There are no controlled clinical trials of screening for hepatocellular carcinoma. However most clinicians screen cirrhotic patients. Those with viral hepatitis and haemochromatosis are most at risk. AFP levels also rise with viral replication and so it is no longer felt to be a good screening tool although it is still a useful diagnostic test. Transplantation is curative for single tumours less than 5cm. Small tumours can be ablated with ethanol or radiofrequency ablation but the risk of new tumours remains in the cirrhotic liver. Chemoembolisation is effective in increasing life expectancy but is not curative. Interval tumours do occur and ultrasound will underestimate size and tumour multicentricity but the radiation risk of CT and cost precludes this as a screening tool. A : D : 117. On medical take you admit a 90-year-old woman with falls. No precipitating cause is found but she is noted to be constipated on PR examination, and this is felt to be contributory to her admission. She is on multiple medications. Which two medications are most likely to be contributing to her constipation? A : Paracetamol B : Senna C : Codeine phosphate D : Isosorbide mononitrate E : Aspirin F : Nifedipine G : Atenolol H : Topical eyedrops I : Ranitidine J : Quinine sulphate

Page 191: Masterclass Book Part 2

MOHAMMED IS-HAG 190

Comment : Falls are a common reason for admission in the elderly. Many causes for falls can be found, and two often-overlooked gastroenterological causes include occult GI bleeding and constipation. Hence PR examination is an essential part of the clinical examination. Constipation can be considered primary (e.g. slow transit syndromes, irritable bowel syndrome) or secondary. Secondary causes include metabolic, gastrointestinal, neurological and psychiatric. As with many things in the elderly the complications of polypharmacy are common. With regards to the association between drugs and constipation different classes of medication are more commonly associated. Thus analgesics (opioid and NSAID) cause constipation as do many anti-depressants. Cardiac medications, in particular ACE-inhibitors and calcium channel blockers,are known to cause constipation.Other well recognised associations exist for bisphosphonates and proton pump inhibitors. In this case the two most likely drugs contributing to constipation are codeine phosphate and nifedipine. Paracetamol and aspirin are not usually a cause of constipation, senna is a laxative and the others are not noted for their association with constipation. C : F: 118. A 35-year-old woman is referred with a history of red, painful legs of 3 weeks duration that have not responded to a course of amoxycillin and flucloxacillin given for presumed cellulitis. She is afebrile, does not have any other symptoms, and has never smoked. Examination reveals tender purple / red nodules on her shins. Her full blood count, kidney and liver function tests are normal. A chest radiograph shows prominent hilae. What is the appropriate management? A : Arrange bronchoscopy and bronchoalveolar lavage to exclude malignancy B : Start azathioprine and prednisolone; follow up in clinic C : Start prednisolone; follow up in clinic D : Start simple analgesics; follow up in clinic E : Arrange CT scan of the lungs and lung biopsy Comment : The combination of bilateral hilar lymphadenopathy and erythema nodosum is diagnostic of sarcoidosis. This is usually self-limiting. She should however be followed up in clinic with full lung function tests including transfer factor and lung volumes. Serum angiotensin-converting enzyme (ACE) level and lung functions can be used to monitor disease. Worsening disease should be treated with prednisolone. D : 119. A 47-year-old man presents with a 1-day history of increasing confusion, drowsiness and jaundice. He has been suffering from depression for the last 6months, but has no other significant past medical history. The two most likely causes of his acute liver failure are: A : Hepatitis A B : Autoimmune chronic active hepatitis C : Primary biliary cirrhosis D : Hepatitis B E : Budd Chiari syndrome F : Paracetamol overdose G : Hepatitis C H : Leptospirosis I : Drug induced (not paracetamol) J : Wilson’s disease. Comment : The three commonest causes of acute liver failure are paracetamol overdose, non-A non-B non-C viral hepatitis (cryptogenic) and drug-induced. Has he taken an overdose, and has he been exposed to any new drug in the last two weeks, particularly a non-steroidal anti-inflammatory agent? Is he at risk of exposure to hepatitis B? In cases of paracetamol overdose remember that N-acetylcysteine improves prognosis, even in patients who present more than 16 hr afterwards. On day 2 (24-48 hours) after a paracetamol overdose the indications for referral to a specialist centre for consideration of transplantation are arterial pH<7.3, INR>3, encephalopathy, creatinine >200 micromol/l or hypoglycaemia. F:I: 120. A 32-year-old man is referred from a neurology clinic having been found to have a parkinsonian-like syndrome and mildly abnormal liver function tests. He has a psychiatric history and a reliable witness denies the previous use of alcohol use or psychotropic medication. Which two of the following statements are true? A : The condition is due to an excess of the copper carrying enzyme, ferroxidase. B : Requesting a slit lamp examination of his eyes may be helpful. C : A low caeruloplasmin level is diagnostic. D : The patient is too old for a diagnosis of Wilson’s disease. E : A haemolytic anaemia suggests chronic autoimmune hepatitis. F : Treatment of this condition is ineffective. G : Screening of first degree family relatives is appropriate. H : Regular venesection is indicated. I : Systemic steroids are indicated. J : Kayser-Fleischer rings are a diagnostic finding.

Page 192: Masterclass Book Part 2

MOHAMMED IS-HAG 191

Comment :Wilson’s disease is an autosomal recessive disease in which excess copper is deposited in the liver, basal ganglia, cornea and other organs. It is due to a reduced production of the copper carrying enzyme, ferroxidase. Most, but not all, patients present between the ages of 5 and 30. Five to ten per cent of normal individuals have a low serum caeruloplasmin. Most patients with Wilson’s disease have low serum caeruloplasmin, low serum copper and high urinary copper concentrations. A slit lamp examination is helpful in detecting Kayser-Fleishcer rings, which can occasionally be found in other cholestatic liver diseases.Haemolysis is a recognized feature and family screening should be performed as copper chelation with penicillamine results in an excellent prognosis in the absence of irreversible damage. B : G: 121. A 42-year-old man with long-standing ulcerative colitis presents with fatigue, pruritus and an elevated alkaline phosphatase. Endoscopic retrograde cholangiopancreatography (ERCP) shows irregular stricturing and beading of the intra and extra-hepatic bile ducts. Which of the following statements are correct? A : The presence of pANCA antibodies are diagnostic of primary sclerosing cholangitis (PSC). B : Primary sclerosing cholangitis is more common in women than men. C : Liver biopsy may be normal. D : Methotrexate has been shown to improve outcome. E : The use of ursodeoxycholic acid will significantly improve outcome. F : The presence of a cholangiocarcinoma should prompt urgent referral for liver transplantation. G : Fever and jaundice should be treated with metronidazole. H : Biliary brushings at ERCP have a low pick-up for cholangiocarcinoma. I : Dominant, non-malignant strictures may benefit from stenting. Comment : Antibodies to pANCA are present in 60-80% of individuals with PSC but are not specific. The condition is more common in men (70:30 ratio). Liver biopsy may be normal, and ERCP is the definitive test. Neither steroids, azathioprine, methotrexate nor cyclosporin have been shown to benefit outcome. Urso may improve liver function tests LFT's) and pruritus but does not alter prognosis, although early data suggest a reduction in the risk of neoplasia. Cholangitis should be treated with an antibiotic with good biliary penetration such as ciprofloxacin. Dominant strictures should be brushed, which while not infallible has a sensitivity for cholangiocarcinoma of about 70%. Dominant strictures can be stented, but patients with end-stage liver disease should be considered for transplantation. Cholangiocarcinoma is however an absolute contraindication to transplantation, as they do universally badly. C : I: 122. A 74-year-old gentleman presents with a 3-month history of change in bowel habit. He has a mild microcytic anaemia. His faecal occult blood (FOB) test is negative and his carcinoembryonic antigen (CEA) is normal at 3. Which three of the following statements are correct? A : Examination of the large bowel is not necessary given the negative FOB test and normal CEA. B : CEA is highly specific for colorectal carcinoma. C : Initial treatment is with iron sulphate and investigation if anaemia returns. D : Colonoscopy is a more accurate test compared to a barium enema. E : If upper and lower gastrointestinal tract examinations were normal, a small bowel meal would often show

an abnormality. F : A urinary laxative screen should be requested. G : CEA levels have a sensitivity of almost 100% in metastatic (Dukes D) colorectal carcinoma. H : A flexible sigmoidoscopy to 60cm is adequate. I : He should have both upper and lower gastrointestinal tract examinations. J : If a colorectal carcinoma were discovered, an isolated liver secondary would render him incurable. K : If he is found to have grade 3 reflux oesophagitis at gastroscopy, lower gastrointestinal tract

examination is unnecessary. L : Ulcerative proctitis is a significant risk factor for the development of colorectal carcinoma. M : A low ferritin level would be highly predictive of an iron deficiency anaemia. N : The risk of perforation from colonoscopy is approximately 1:10 000. O : He needs no further investigation as a barium enema 12 years ago was normal. Comment : A 3-month history of change in bowel habit in an elderly person necessitates lower GI tract examination with either a colonoscopy or a barium enema. Colonoscopy is the most accurate test; perforation risk is about 1:1500. FOB is neither highly sensitive nor specific for colorectal carcinoma, and decisions to investigate a change in bowel habit should not be based on the result. It may have a role in population screening to identify a target group. CEA levels suffer from similar problems. CEA levels are only elevated in about 75% of patients with disseminated colorectal carcinoma, and elevated levels may occur in stomach and breast cancer. Investigation of iron deficiency anaemia should involve both upper and lower GI tract examinations, since important dual pathology occurs in about 5%. In those in whom these tests are negative, small bowel meals may identify a cause, but the yield is low. Family history and extensive ulcerative colitis are risk factors for colorectal carcinoma. Second to bone marrow examination, low ferritin levels are highly predictive of iron deficiency. The risk of colorectal cancer in males and postmenopausal women with iron deficiency anaemia is up to 10% therefore prompt investigation is required. Solitary (less than 3) liver metastases are potentially resectable and do not necessarily indicate incurability. D:I:M:

Page 193: Masterclass Book Part 2

MOHAMMED IS-HAG 192

123. A 26-year-old woman is found to be anaemic and has a raised anti-gliadin IgG antibody titre. Which of the following statements are true? A : A blood film has no role in contributing to the diagnosis. B : The diagnosis is unlikely to be anything other than coeliac disease. C : The presence of Howell Jolly bodies would support a diagnosis of coeliac disease. D : Endomysial antibodies are less sensitive than anti-gliadin antibodies in the diagnosis of coeliac

disease. E : IgA deficiency is rare in coeliac disease. F : Histological changes may take over a year to resolve completely on a gluten-free diet. G : Obesity virtually excludes a diagnosis of coeliac disease. H : Small bowel radiology is the appropriate next investigation. I : There is no increased risk of coeliac disease in family members. J : Iron and folate deficiency, but not B12 deficiency, occur in coeliac disease. Comment : Up to 10% of menstruating woman may develop anaemia and isolated anti-gliadin IgG antibodies are present in 10% of the population. Anti-endomysial antibody is the most sensitive (80-100%) and specific (98-100%) antibody in the diagnosis of coeliac disease. It may be negative in IgA deficiency, which affects up to 4% of patients. A blood film may show Howell Jolly bodies, a feature of hyposplenism which may occur in coeliac disease. Iron, folate and B12 deficiency may occur. Obesity may affect up to a quarter of newly diagnosed patients. Small bowel radiology is not a sensitive test for coeliac disease, which affects between 5-20% of first-degree relatives. C : F: 124. A 65-year-old man was found to have mild iron deficiency anaemia at a well man clinic. Which of the following statements are true? A : Duodenal biopsies should be taken if gastroscopy is normal. B : A history of pernicious anaemia might explain the findings. C : If asymptomatic, investigation could be reserved for those not responding to iron supplements. D : He could be safely booked for same day flexible sigmoidoscopy, biopsy and barium enema. E : The perforation rate for barium enema is greater than that for colonoscopy. F : Gastritis identified at gastroscopy would be an acceptable cause. G : Iron supplements can be stopped once the haemoglobin is within normal range. H : Haemoglobin electrophoresis should be requested. I : Colon cancer is likely only in the presence of a change in bowel habit. J : Barium enema can detect angiodysplasia. Comment : In the absence of a clear cause of iron deficient anaemia at gastroscopy, duodenal biopsies should always be taken to exclude coeliac disease. Gastritis, oesophagitis and duodenitis are not acceptable causes of anaemia. Pernicious anaemia might be implicated as iron absorption is less effective in the achlorhydric stomach and there is a two to three-fold increase risk of gastric cancer. Barium enema should not be done within 24 hours of rectal or colonic biopsy, but otherwise has a perforation rate of 1:30 000, compared to that of diagnostic colonoscopy at 1:3000. Barium enema is ineffective at diagnosing mucosal lesions. Iron supplements should be given for 3-6 months to replenish iron stores. A : B : 125. A 70-year-old woman presents with malaise and loose stool, and a mild iron-deficiency anaemia is noted. She has had vague limb pains for a number of years. A colonoscopy is normal. A gastroscopy is macroscopically normal but a duodenal biopsy taken shows blunting of villi and inflammatory cells in the lamina propria. Which of the following are true of coeliac disease in older people? A : It is more common in Asian countries. B : Less than 5% are diagnosed after the age of 60. C : The toxicity of oats protein is greater than wheat or rye. D : Minor gastrointestinal and constitutional symptoms are more common than overt malabsorption. E : Bone pains are due to poor absorption of vitamin E. F : Dementia is the most commonly associated neurological condition. G : Dermatitis herpetiformis is an associated disease which has a predilection for the trunk. H : Polyvalent pneumococcal vaccine may be required. I : Anti-gliadin antibodies are a more specific indicator of coeliac disease than anti-endomysial antibodies. J : There is an increased risk of colonic carcinoma. Comment : Coeliac disease predominantly affects Europeans and those with European heritage, with prevalence rates up to 1 in 500 in Ireland. It affects all ages with a mean age of presentation in the 40s and at least 20% diagnosed after age 60. Genetic predisposition (most commonly DQ2) and environmental factors lead to cell-mediated damage of the small intestine and the characteristic histological features. Classic symptoms of the condition are steatorrhoea, osteomalacia and anaemia. However, more subtle and vague symptoms are more common in older people and the diagnosis may not be considered. Gluten in wheat, rye and barley has an established causal relationship. The toxicity of oats is unclear. The wide range of symptoms include: · lethargy · anorexia · weight loss · diarrhoea · aphthous ulcers

Page 194: Masterclass Book Part 2

MOHAMMED IS-HAG 193

· bone pain (due to vitamin D malabsorption and osteomalacia). Gate ataxia and peripheral neuropathy are the most common neurological problems. Dermatitis herpetiformis tends to affect the extensor surfaces of the arms, knees and buttocks. Folate and iron deficiencies may lead to macrocytosis or a dimorphic blood film. Howell-Jolly bodies characteristic of hyposplenism may also be seen in the film. Such patients require pneumococcal vaccination.Antigliadin antibodies are a sensitive but not specific serological test for screening. Antireticulin and antiendomysial antibodies are more specific. None are 100% sensitive however and if suspicious, intestinal biopsy should still be arranged. Treatment is a gluten-free diet, for symptomatic relief but also to reduce the associated risk of small bowel lymphoma and adenocarcinoma, and squamous cell carcinoma of the pharynx and oesophagus. D :H: 126. A 72-year-old man has a percutaneous endoscopic gastrostomy (PEG) inserted for swallowing difficulties due to his advanced Parkinson’s disease. He becomes confused and has a tachycardia 48 hours after commencing his PEG feed. Which of the following statements are true? A : Aspiration pneumonia is a possible cause. B : Serum phosphate should be measured. C : The absence of cranial nerve signs excludes Wernicke’s encephalopathy. D : Air under the diaphragm on an erect chest radiograph is an indication for laparotomy. E : Anticoagulation is contraindicated. F : PEG feeding avoids the risks of aspiration in patients with swallowing difficulties. G : 30-day mortality rate after PEG insertion is less than 5%. H : PEG insertion is contraindicated in Crohn’s disease. I : Antibiotic prophylaxis prior to PEG insertion is of no proven benefit. J : PEG feeding improves outcome in patients with advanced dementia. Comment : PEG site infection and aspiration pneumonia are common complications. Aspiration may occur during endoscopy but may occur later due to aspiration of feed in a recumbent patient. Air under the diaphragm may occur even after an uneventful PEG placement. Reported 30-day mortality rates after PEG insertion vary between10-30%.It is likely that many die as a consequence of their underlying disease,and those with advanced dementia have a particularly poor prognosis. The re-feeding syndrome describes a range of metabolic disturbances (commonly affecting phosphate, potassium and magnesium) which occur during the re-feeding of a starved patient. Clinical manifestations are many and varied and include neurological, cardiac, respiratory, haematological and immunological features. Wernicke’s encephalopathy may occur without all the classical features of confusion, ataxia and ophthalmoplegia. A : B : 127. A 44-year-old woman presents with 3 months vague abdominal pain, diarrhoea and weight loss. Duodenal biopsies, small bowel studies, colonoscopy and biopsies taken from colon and terminal ileum are all normal. Which of the following statements are true? A : Facial flushing and a systolic murmur suggest the need to measure urinary hydroxyindole-acetic

acid. B : The symptoms are consistent with irritable bowel syndrome. C : The commonest site of origin of a gastrointestinal carcinoid is the stomach. D : A serum k+ level of 2.4 is a typical finding in a patient with a VIPoma. E : Previous duodenal ulcer disease is unlikely to be of relevance. F : Carcinoid syndrome has a poor prognosis as the tumour grows rapidly. G : An eating disorder can be discounted. H : Tryptophan is responsible for the symptoms in carcinoid syndome. I : Diarrhoea due to a neuroendocrine tumour usually reduces with fasting. J : Raised levels of gastrin in a fasted patient are always due to a gastrinoma. Comment : The commonest site of origin of intestinal carcinoids is the terminal ileum, although they may arise from the foregut, midgut or hindgut. The classical symptoms of the carcinoid syndrome are flushing, wheeze and diarrhoea. Fibrosis of tricuspid and pulmonary valves may occur. Metastatic carcinoids take up tryptophan and metabolize it to 5-hydroxytryptamine which appears to have a role in the pathogenesis of the diarrhoea and bronchoconstriction. Many carcinoids are slow growing with a good prognosis. Raised gastrin levels may occur in achlorhydria due to pernicious anaemia or proton pump inhibition. The diarrhoea of endocrine tumours is secretory and tends not to decrease with fasting. Weight loss cannot be ascribed to irritable bowel syndrome and diarrhoea may be due to laxatives in eating disorders. A : D : 128. A young surgeon is tested for Hepatitis B virus. His serology result shows him to be HBsAg negative, anti-HBc antibody positive. What is his status? A : He is an asymptomatic carrier. B : as been successfully vaccinated against the virus. C : He has been infected with the virus in the past and is now immune. D : He has the virus and is highly infectious. E : He has been vaccinated, but has not mounted an immune response to the vaccine. Comment : As he is HBsAg-ve he does not have the virus. Anti-HBc antibody positive indicates that he has met the wild type virus (i.e. had a past infection) but has successfully cleared it. A person who has been vaccinated (with

Page 195: Masterclass Book Part 2

MOHAMMED IS-HAG 194

HBsAg) will be anti-HBs antibody positive, but will be anti-HBc antibody negative as they have never seen the core part of the virus. C: 129. A 27-year-old mother of three presents in the outpatient clinic with a 2-year history of alternating constipation and diarrhoea associated with abdominal bloating. Which of the following statements are true? A : Nocturnal diarrhoea is highly suggestive of irritable bowel syndrome. B : A past history of sexual abuse may be relevant. C : Recent weight gain is a sinister feature. D : Barium follow-through is the investigation of choice. E : Allergy testing should be performed in order to exclude dietary precipitants. F : Full colonoscopy is mandatory in order to exclude Crohn’s disease. G : Microcytic anaemia can be safely attributed to her history of menorrhagia. H : Complimentary therapies should be discouraged as they are likely to interfere with standard

therapy. I : Systemic yeast infection should be excluded. J : Biofeedback techniques may be beneficial in symptom control. Comment : Nocturnal diarrhoea is not a feature of IBS and should raise suspicion of inflammatory bowel disease. Some studies have identified an increased incidence of past sexual abuse in patients with IBS as compared to controls. Documented, unexplained weight loss is a cause for concern, but weight gain is rarely significant.Barium follow-through involves considerable radiation exposure to a woman of childbearing age and should be reserved for those individuals who have objective evidence or histories highly suggestive of small bowel Crohn's disease. While some patients will have food intolerances, these are not classical allergies and will not be identified by skin-testing. Elimination diets may be useful, but are time-consuming and may be disruptive to the patient's lifestyle.In many patients, flexible sigmoidoscopy and random rectal biopsies may be sufficient to exclude significant colonnic disease. Microcytic anaemia cannot be ignored, and while gynaecological causes are more likely in a woman of this age, GI investigation should take place. Complementary therapies may help lower stress levels in patients and can bring about improvement in symptoms. There is little evidence to support the effect of currently available pharmacological treatments much beyond a placebo effect. While the concept of systemic fungal infection has gained popularity with some alternative practitioners, there is little scientific evidence to support this. Biofeedback techniques have been demonstrated to improve patient symptoms, but are time-consuming to teach and not available in all centres. B : J: 130. A 61-year-old lady presents with a 4-year history of hypertension and an incidental finding of massive hepatomegaly. There is a family history of renal failure. A CT scan confirms multiple large thin walled hepatic cysts and multiple cysts in the kidney. Which of the two pieces of information have been correctly given to the patient? A : It is an autosomal recessive condition. B : There is a risk of subarachnoid haemorrhage. C : She is likely to develop liver failure with jaundice and coagulopathy. D : Liver transplantation is contraindicated. E : Abdominal discomfort from the cysts can be treated surgically. F : Aspiration of liver cysts successfully relieves abdominal discomfort. G : The condition precludes renal transplantation. H : Cirrhosis occurs in the non cystic liver. I : Angiotensin-converting enzyme (ACE)-inhibitors should not be used to treat the hypertension. J : Pancreatic cysts do not occur. Comment : The patient is likely to have adult polycystic kidney disease. Cysts can affect the liver and pancreas. About 50% of individuals with multiple large liver cysts will have this condition (i.e. also polycystic kidneys). Patients are at risk of subarachnoid haemorrhage and it is important to control hypertension. No treatment is needed for the liver cysts unless they cause severe discomfort or bleeding has occurred into a large cyst causing pain. Cysts reaccumulate after aspiration but surgical fenestration of large cysts is successful in selected cases. Liver failure in adult polycystic liver disease is very uncommon but portal hypertension due to sinusoidal compression is well described. Although successful liver-kidney transplantation has been reported it is rarely indicated. However in cases needing renal transplantation it is often necessary to remove the diseased kidneys to make room for the transplant kidney. B : E :

Page 196: Masterclass Book Part 2

MOHAMMED IS-HAG 195

131. A 22-year-old patient who is otherwise in good health complains that certain foods cause her to suffer severe abdominal pain and develop diarrhoea shortly after eating. She is convinced that she has a yeast infestation. Which of the following could be recommended initially? A : A short course of treatment with fluconazole, which should eliminate candidal infection, if it were

present, is a reasonable first step. B : The symptoms are typical of irritable bowel syndrome, and should be treated symptomatically,

without further investigation. C : A careful dietary history would be helpful D : The symptoms are compatible with lactose intolerance E : Coeliac disease is highly likely particularly if the patient has a family history F : A therapeutic trial of treatment with metronidazole, for presumed giardiasis is a reasonable first step G : Amitriptyline, used at a relatively low dose, particularly at night, may be helpful for the abdominal pain H : Peptic ulcer disease or Helicobacter pylori associated gastritis are unlikely because they do not

cause diarrhoea. I : The most likely diagnosis is diarrhoea predominant IBS (IBS-D), and the advent of new treatments

for this condition will probably modify our approach to patients such as this J : The clinical scenario strongly suggests dumping syndrome and intestinal hurry, making diabetes

mellitus a strong possibility Comment : A patient complaining of abdominal pain and altered bowel habit could certainly have a form of irritable bowel syndrome (IBS). However, the diagnosis cannot be established without some consideration of alternative explanations, as exclusion of organic pathology remains a diagnostic criterion. Although there is currently a great deal of interest in the use of specific 5-HT3 receptor antagonists in the treatment of IBS with diarrhoea, the recent experience of toxicity associated with Alosetron (which was withdrawn from use) suggests that physicians should be cautious about introducing related agents as the first line of therapy. Amitriptyline, used at a low dose, offers some analgesic effect in IBS. Nonetheless, it seems unsuitable as an initial treatment without a clear diagnosis, and would have no effect on the diarrhoea. The patient’s symptoms are not typical of coeliac disease, where the diarrhoea is not usually related so closely to eating, or to giardiasis, where cramping abdominal pains, wind, and diarrhoea are usually unrelated to eating. Abdominal pain, faintness, nausea, and possibly diarrhoea may occur in the dumping syndrome, however, in this context it seems a distant possibility. Abdominal pain is a typical feature of peptic ulceration and Helicobacter pylori associated intestinal disease, and depending on the demographics of one’s practice, it may be a frequent cause of symptoms such as this. Peptic ulceration associated with the Zollinger-Ellison syndrome may certainly cause diarrhoea, and the diagnosis cannot be ruled out entirely, but it is a very rare disease. As the patient reports that eating makes her unwell, a careful dietary history is indicated. Her symptoms are consistent with lactose intolerance, and suspicion would be increased if the yeast she was wary of was that found in cheese. A positive lactose intolerance test (e.g. breath test or duodenal biopsy for lactase activity) would clinch the diagnosis. C : D : 132. A 66-year-old woman is admitted with an acute hepatitis (ALT 670 IU/l). She is taking many prescribed medicines, some of which have recently been changed. Of note she has had a painful right hip following a fall seven days ago. In the previous month she has taken the following drugs, which two are most likely to be the cause of her abnormal liver biochemistry? A : Diclofenac B : Lisinopril C : Warfarin D : Digoxin E : Hormone replacement therapy F : Lanzoprazole G : Amoxicillin H : Ciprofloxacin I : Paracetamol J : Temazepam Nonsteroidal anti-inflammatory agents are a common cause of abnormal liver function tests. Cholestatic liver function tests can occur as an idiosyncratic reaction to chlorpromazine, the dextropropoxyphene component of co-proxamol, and fluxcloxacillin. Although any drug can cause abnormal liver function tests this has only rarely been reported with the other drugs listed, eg. warfarin, lanzoprazole. Do not forget that inadvertent paracetamol poisoning is not uncommon. With respect to drug induced liver disease, some drugs cause direct toxicity and the injury is generally predictable, dose-related, and characteristic for the drug. Other drugs produce damage in an apparently idiosyncratic way - the injury is usually unpredictable and not dose-related. Most acute cases can be divided into hepatocellular, cholestatic (with or without inflammation), and miscellaneous reactions (e.g. mixed forms of hepatic dysfunction, or granulomatous reactions). Some drugs can produce chronic damage, including chronic hepatitis and tumours. Mild jaundice may also occur as a result of drug-induced haemolysis (unconjugated hyperbilirubinaemia), without true hepatic damage or abnormal liver function tests. A:I:

Page 197: Masterclass Book Part 2

MOHAMMED IS-HAG 196

133. Virtual colonoscopy, which uses radiological techniques and computer algorithms to reconstruct an image of the colon’s interior, is gaining acceptance. Which of the following statements regarding this technique are correct: A : The technique is greatly superior to conventional colonoscopy in its ability to detect mucosal

lesions B : A very small risk of perforating the colon remains, despite the lack of intra-luminal

instrumentation C : The main insuperable problem is that the presence of faecal material introduces diagnostic

uncertainty, because the Hounsfield number of faeces is close to that of normal tissue D : Hypersensitivity to iodine is a major difficulty because of the necessity of administering intravenous

contrast agent E : A major difference with barium enema is the lack of intra-luminal contrast F : The main disadvantage of virtual colonoscopy is the inability to obtain mucosal biopsies and to

perform polypectomy G : The technique has gained greater acceptance because modern techniques allow it to be

performed with a very ow dose of radiation to the subject H : There is no radiation dose as magnetic resonance imaging is used I : In addition to not having to have instrumentation of the bowel, the main advantage to patients is

that they do not have to use powerful laxatives to evacuate the bowel beforehand J : Widespread use of this technique has meant that barium enemas are no longer indicated in

routine clinical practice. Comment : Virtual colonoscopy involves the use of high-resolution CT scanning, with rapid, modern scanning, and computer-assisted analysis providing the operator with 2-dimensional and 3-dimensional reconstructions of the intestine’s lumen. Patients have extensive bowel cleansing and also ingest a small amount of barium, which infiltrates any residual faecal matter so that the faecal matter is relatively radio-opaque and distinguishable from tissue. The bowel is also insufflated with air. This procedure introduces a small risk of colonic perforation. Intravenous contrast is not always used and reactions are rare nonetheless. Eventually it is conceivable that MRI could be adapted for virtual colonoscopy, although this is still not a reality. A number of studies comparing virtual colonoscopy with conventional colonoscopy demonstrate that the efficacy of the techniques is comparable, although the inability to take mucosal biopsies and perform polypectomy is limiting, particularly as virtual colonoscopy has a greater tendency to detect small mucosal lesions, whose histological character and clinical significance remain undetermined. Although an exciting advance radiologically, extensive studies along with training of more personnel are needed before this technique will replace barium enemas.B:F: 134. A 67-year-old man presents with anaemia Hb 8.3g/dl (12.5-16.5) and ferritin 4 micrograms/litre (12-200). He denies any upper or lower gastrointestinal symptoms. His urine dipstick is negative. He takes low-dose aspirin as he has read in the newspapers that it is good for the circulation. Which are the best two tests in terms of further investigation? A : Faecal occult bloods x 3 B : Colonoscopy C : Gastroscopy D : Small bowel enteroscopy E : Small bowel meal F : Gastroscopy and duodenal biopsies G : Intrinsic factor antibodies H : Bone marrow examination I : Abdominal CT scan J : Labelled red cell scan. Comment : All men and post menopausal women with iron deficiency anaemia should have both their upper and lower gastrointestinal (GI) tract examined. Upper GI tract pathology is seen in about 50% and lower GI tract pathology in around 20%. No cause is found in about 25%. Up to 10% may harbour colorectal neoplasia. Important dual pathology is seen in about 5% hence the need for bi-directional endoscopy. Coeliac disease is seen in 2-5% so duodenal biopsies should always be taken. Small bowel meal (and enteroscopy) may be required if iron deficiency anaemia is recurrent despite iron replacement therapy though its yield is low. B : F: 135. A 65-year-old man, retired publican, known to the diabetic team for long-standing diabetes and the cardiologists for long-standing paroxysmal AF, was admitted with haematemesis. An endoscopy revealed a gastric varix whilst abdominal imaging showed a cirrhotic looking liver with a suspicious lesion in the right lobe; splenomegaly and ascites were also demonstrated. His ferritin was >1000 ng/ml and transferrin saturation 65%. Alpha-fetoprotein was 990 ng/ml. What is the working diagnosis? A : Cryptogenic cirrhosis secondary to non-alcoholic steatohepatitis B : Haemochromatosis with hepatocellular carcinoma C : Alcoholic liver disease D : Alpha-1-antitrypsin deficiency E : Hepatitis C cirrhosis with hepatocellular carcinoma

Page 198: Masterclass Book Part 2

MOHAMMED IS-HAG 197

Comment : This man has haemachromatosis. He is cirrhotic with a ferritin >1000 ng/mL and a transferrin saturation >65%. This indicates he has iron overload and haemochromatosis is the abnormal accumulation of iron in parenchymal organs, leading to organ toxicity. Genetic haemachromatosis is the most common inherited liver disease in Caucasians and the most common autosomal recessive genetic disorder. The organs involved are the liver, heart, pancreas, pituitary, joints, and skin, with the clinical features of haemochromatosis including cirrhosis of the liver, diabetes, hypermelanotic pigmentation of the skin, and heart failure. Primary hepatocellular carcinoma, complicating cirrhosis, is responsible for about one-third of deaths in affected homozygotes. Since haemochromatosis is a relatively easily treated disorder if diagnosed, this is a form of preventable cancer. Despite advances in the diagnosis of haemachromatosis (principally genetic screening for the C282Y mutation responsible for most, but not all, forms of disease), treatment remains simple, inexpensive, and safe.Patients have weekly therapeutic phlebotomy of 500 mL of whole blood (equivalent to approximately 200-250 mg of iron). Some patients can tolerate twice-weekly phlebotomyTherapeutic phlebotomy is performed until iron-limited erythropoiesis develops (taken as failure of the haemoglobin level to recover before the next phlebotomy; aiming for a transferrin saturation less than 50% and serum ferritin levels less than 50 g/mL, preferably 20 ng/mL).Most patients then require maintenance phlebotomy-1unit of blood every 2-3months.Therapeutic phlebotomy may improve or even cure some of the manifestations and complications of the disease,such as fatigue,elevated liver enzymes,hepatomegaly,abdominal pain, arthralgia,and hyperpigmentation.Other complications usually show little or no change after phlebotomy. B : 136. An 81-year-old man was admitted with a 3-week history of diarrhoea following a course of antibiotics 4 weeks previously for a chest infection. His GP had treated him with Imodium, but this had produced little improvement. On examination he was tender all over his abdomen, but without guarding. Which one of the following investigations would be most useful in excluding a diagnosis of pseudomembranous colitis? A : Abdominal radiograph B : Barium enema C : Positive stool sample for C. difficile toxin D : Flexible sigmoidoscopy E : Abdominal ultrasound. Comment : The diagnosis of pseudomembranous colitis requires direct visualisation of pseudomembranes on bowel mucosa or on microscopic examination of a biopsy sample. 90% of cases can be detected by flexible sigmoidoscopy, although in mild cases pseudomembranes may not be evident macroscopically. The classic sigmoidoscopic appearance of 2 to 10 mm raised yellow nodules are pathognomonic.An abdominal radiograph may be normal in pseudomembranous colitis, although in severe cases it may show a complicating toxic megacolon. Barium enema and abdominal ultrasound are of no use in confirming or excluding the diagnosis of pseudomembranous colitis, and a positive stool culture for C. difficile toxin would make (rather than exclude) the diagnosis. D : 137. A 40-year-old woman with crohn’s disease who has recently been treated with chimeric mouse-human antibody Infliximab (which binds to and blocks the activity of tumour necrosis factor-? (TNF?)), has recurrent disease, and it is apparent that the clinical response to Infliximab infusions is waning. Which of the following statements is MOST correct? A : The declining response to treatment may be associated with antibodies against the Infliximab

(human anti-chimera antibodies or HACA), and a period of immunosuppression may suppress this untoward reaction

B : The declining response to treatment is unlikely to be associated with antibodies against the Infliximab (human anti-chimera antibodies or HACA), as the patient is not reported to have had an anaphylactic reaction

C : The declining response to treatment may be associated with antibodies against the Infliximab (human anti-chimera antibodies or HACA), and the patient may now be effectively treated with a different antibody against TNF?, such as Adalimumab

Comment : An emerging problem with the use of “biologicals” (usually recombinant proteins) is the emergence of neutralizing antibodies that block the efficacy of treatment, without necessarily causing anaphylaxis or other infusion reactions. Structurally different protein therapeutics usually remain effective, and Adalimumab has been shown to be effective in patients who no longer respond to Infliximab infusions due to the development of HACA. The soluble TNF receptor Etanercept also blocks the activity of TNF?; however, for unknown reasons, it is not as effective as the antibodies in treating crohn’s disease. Natalizumab, the antibody against ?4?7 integrins, which was potentially effective in some patients with crohn’s disease, has been withdrawn from clinical use following the discovery that a number of patients treated with the antibody had developed the rare and highly aggressive central nervous system disorder, progressive multifocal leukoencephalopathy (PML), which is associated with JC virus infection. C :

Page 199: Masterclass Book Part 2

MOHAMMED IS-HAG 198

138. At a routine follow-up, a 65-year-old man with long standing ulcerative colitis is noted to have a sterile pyuria with eosinophils and dysmorphic red cells present on urine microscopy. He denies any urinary symptoms but his creatinine is mildly elevated. What is the most likely diagnosis? A : Urinary tract infection B : TB of the urinary tract C : Interstitial nephritis secondary to sulphasalazine D : Polycystic kidney disease E : Brucellosis. Comment : Sulphasalazine is metabolised by colonic flora, releasing sulphapyridine, an absorbable antibiotic, and 5-aminosalicylic acid (5-ASA), the active ingredient. The sulphapyridine acts only to carry the 5-ASA to the colon and, when released by bacterial metabolism, is absorbed and is responsible for dose-related side effects. Common dose-related symptoms include anorexia, nausea, dyspepsia and diarrhoea, whilst haematological complications include impairment of folate absorption and Heinz-body haemolytic anaemia. Hypospermia may occur but is reversible upon drug withdrawal. Hypersensitivity reactions are rarer but include fever, rash, bone marrow suppression, infiltrative lung disease, a lupus-like syndrome, pancreatitis, interstitial nephritis and hepatic toxicity. Aside from urinary tract infection, the other conditions listed are all possible causes of sterile pyuria, but none except interstitial nephritis due to sulphasalazine are suggested by this clinical scenario. C : 139. A 30-year-old woman with long-standing medically-treated Crohn’s disease presents to clinic with increasing abdominal pain associated with intermittent vomiting and bloating. Her inflammatory markers are normal. Which is the most likely cause of her symptoms? A : Exacerbation of colonic colitis B : Stricture formation secondary to long standing disease C : Colo-vesical fistula D : Adhesions E : Colorectal cancer. Comment : The most common symptoms of Crohn's disease are abdominal pain, often in the lower right quadrant, and diarrhoea. Rectal bleeding, weight loss and fever may also occur. Bleeding may be serious and persistent, leading to anaemia. Children with Crohn's disease may suffer delayed development and stunted growth. Long standing inflammation can result in stricture formation which may present as intermittent bowel obstruction. Markers of inflammation such as CRP or ESR are not necessarily significantly raised if the obstruction is due to fibrosis. Management of Crohn’s disease involves a combination of nutritional, medical and surgical treatment. Stricture formation, in particular, is usually treated surgically, although endoscopic treatments are possible. Patients with small bowel strictures are also advised to eat a low fibre diet, in order to reduce the risk of food bolus obstruction. B : 140. An 83-year-old woman is admitted from A&E unable to swallow solids and with increasing difficulty for liquids. She is otherwise well, and blood tests are normal. Barium swallow and upper GI endoscopy reveals a tight, smooth stricture at 34cm and brushings and biopsies show no malignant cells. Oesophageal dilatation is planned. In the consent discussed with the patient, which is least likely to occur? A : Pain B : Oesophageal tear requiring stent placement C : Aspiration pneumonia D : Oesophageal rupture necessitating oesophagectomy E : Bleeding Comment : Therapeutic endoscopy carries significant risks and patients must be informed appropriately before consenting to any procedure. The principal significant complications of oesophageal dilatation, apart from mild pain/discomfort are perforation, pulmonary aspiration, and bleeding. A UK audit reported an overall perforation rate of 2.6% with a mortality of 1%. Perforation was less common following dilatation of benign strictures (1.1% with a mortality of 0.5%) than following dilatation and/or intubation of malignant strictures (6.4% with a mortality of 2.3%). Elderly patients appeared at greater risk. The risk of perforation in achalasia is reported to be higher, at up to 7% (mostly 3–4%) with a mortality of <1%. To diagnose perforation requires a high index of suspicion and typical symptoms include chest pain, and dysphagia. Signs include pyrexia, tachycardia, hypotension, tachypnoea. Subcutaneous emphysema may be present. Undiagnosed systemic sepsis rapidly develops and death often occurs with 48 hours. Chest x-ray may show pleural air / fluid levels and mediastinal emphysema. A water-soluble contrast swallow can confirm diagnosis. Conservative management may be appropriate for small perforations without systemic upset and small-contained thoracic leaks. Treatment requires nil by mouth, antibiotics and intravenous hydration. Failure of conservative management will need surgery. If operated on within 24 hours mortality is 5-10% and if delayed more than 48 hours mortality is more than 50%. D :

Page 200: Masterclass Book Part 2

MOHAMMED IS-HAG 199

141. A 50-year-old man had an abdomino-perineal resection for a Dukes A rectal carcinoma 9 years ago. His brother died from metastatic colorectal cancer. Interval colonoscopy at 5 years post surgery was normal. He now presents to clinic with an iron deficiency, anaemia and jaundice. In his investigations, which statement is LEAST appropriate? A : Further lower GI investigation with flexible sigmoidoscopy should be performed as a matter of

urgency B : Abdominal ultrasound is an appropriate first line investigation for his jaundice. C : Post-operative screening by colonoscopy at 5-year intervals is reasonable practice. D : Measurement of CEA is of limited value in monitoring colorectal cancer patients. E : Family screening for the man's children is appropriate. Comment : Up to 50% of colorectal cancer patients will develop a recurrence of their tumour and most of these will die from their disease. Eighty per-cent of recurrences after resection of colorectal cancer occur within the first two years after surgery, but even an intensive investigation follow up programme will fail to detect approximately 50% of asymptomatic recurrences.Patients who did not undergo complete colonoscopy before surgery should be offered colonoscopy within six months of discharge. This can be regarded as completion of the initial diagnostic work-up to identify individuals who have polyps or a tumour elsewhere in the colon. Colonoscopy may then be offered at five-yearly intervals to check for new polyps or tumours. It should not normally be carried out more frequently when the patient has a ‘clean’ (polyp-free) colon, but patients with five or more adenomas should be offered more frequent checks. Colonoscopy may not be beneficial for patients with clean colons and life-expectancy of less than 15 years, since they are very unlikely to develop a new colorectal tumour during this period. The role of carcinoembryonic antigen (CEA) monitoring is uncertain. There is still no evidence that the lead time provided by CEA monitoring confers any survival benefit.Meta-analysis shows that for people with at least one affected first-degree relative (parent, child or sibling), the risk of having a diagnosis of colorectal cancer is more than double that for the general population. People with two first-degree relatives with colorectal cancer, or one first-degree relative whose colorectal cancer is diagnosed before the age of 45, have a lifetime risk of death from colorectal cancer of one in six or one in 10 respectively. The BSG/ACPGBI guidelines suggest that people who meet these criteria should be referred for colonoscopy at 35-40 years of age, or as soon thereafter as the risk is recognised. Stated more generally, screening in relatives should commence at an age 10 years younger than the age at which the affected index case was diagnosed with colon cancer.Polyps can be seen and removed during endoscopic examination of the bowel; 90% of adenomas can be detected by colonoscopy and about 70% by flexible sigmoidoscopy, which typically only examines the colon distal to the splenic flexure. However, in an individual at high risk of synchronous, metachronous, or recurrent cancer (and where iron deficiency suggests the possibility of gastrointestinal bleeding) a full colonoscopy, rather than a limited examination, is indicated. A : 142. A 35-year-old female teacher is referred for investigation of abnormal liver tests, which comprise 2-3 fold elevations in the aminotransferase. There is no history of liver disease, and a screen for viral hepatitis is negative. In the trawl of autoantibody tests, a positive anti-tissue transglutaminase antibody test is highlighted. Which of the following statements is incorrect? A : Coeliac disease is recognised as a cause of abnormal liver tests, and may be associated with

inflammatory infiltration of the hepatic portal tracts, and progressive liver disease B : There is an association of coeliac disease and Primary Biliary cirrhosis (PBC) C : There is an association of coeliac disease and primary sclerosing cholangitis (PSC) D : The most likely diagnosis is autoimmune hepatitis, which is frequently associated with a raised serum

immunoglobulin level, and a variety of autoantibodies including anti-tissue transglutaminase E : The presence of a positive tissue transglutaminase antibody is highly suggestive of a diagnosis

of coeliacdisease in this context Comment :A positive tissue transglutaminase antibody is highly suggestive of a diagnosis of coeliac disease in any context. It is well recognised that liver chemistry may be abnormal in coeliac disease, and pathologically this may be associated with inflammatory cell infiltrates. There are also recognised associations between coeliac disease and PBC and PSC, although there are some discrepancies between different studies. There is no satisfactory explanation for a link between these conditions. Autoimmune hepatitis typically is associated with a raised serum immunoglobulin level, and a variety of autoantibodies against hepatic antigens, as well as a positive antinuclear antibody and frequently positive smooth muscle antibody. E :

Page 201: Masterclass Book Part 2

MOHAMMED IS-HAG 200

143. A 76-year-old man with a long history of rheumatoid arthritis is admitted after a haematemesis. His full blood count is as follows: Hb 10.8 g/dl, MCV 86 fl, WBC 7.4, platelets 243. Which of the following statements is true? A : The normal platelet count makes it unlikely that he has had a significant gastrointestinal bleed. B : The normal blood count is reassuring: he is unlikely to have had a significant gastrointestinal

bleed. C : The blood count does not tell you whether or not he has had a significant gastrointestinal bleed. D : The blood count suggests iron deficiency anaemia. E : The high platelet count is consistent with recent gastrointestinal haemorrhage, but could also

reflect activity of his arthritis. Comment :The blood count shows a normocytic anaemia, consistent with the anaemia of chronic disorders and not suggestive of iron deficiency.Acute loss of blood does not lead to any immediate change in the full blood count: haemodilution takes some time to occur, hence the full blood count can never be used to decide whether or not someone has suffered a significant acute haemorrhage. Physical examination will tell you this, the two most reliable signs of intravascular volume depletion being postural hypotension (lying and sitting) and a reduced jugular venous pressure. C : 144.

PLATE 32 In the case of a patient presenting with iron deficiency anaemia and the accompanying endoscopic appearances of a part of the small intestine (see image), which of the following statements are true: A : The photograph shows an ulcerated terminal ileum with evidence of lymphoid nodular

hyperplasia. B : The prominent mucosal folds and atrophic villi are consistent with a diagnosis of coeliac disease. C : Shallow ulcers with punctate fresh haemorrhages are the likely cause of the anaemia. The

appearances are typical of NSAID induced enteropathy D : Fresh haemorrhage from beyond the field of view indicates a bleeding point in the distal small

intestine. These findings are highly suggestive of a bleeding ulcer within a Meckel’s diverticulum. E : The appearances are entirely normal. Comment : The image shows the second and third part of the duodenum, which is smooth with an absence of well-defined ridges and the normally velvety appearance imparted by thousands of villi. The red colour of the mucosa is typical and does not signify fresh or recent haemorrhage. Ulceration usually produces a superficial slough that is typically green or yellow-brown, and there is no evidence of ulceration in this picture. The terminal ileum may appear very similar to this image, but usually bile-stained fluid is present, and in any case there is no ulceration present. B :

Page 202: Masterclass Book Part 2

MOHAMMED IS-HAG 201

145. A 38-year-man is referred to clinic having been found to be hepatitis C antibody positive and hepatitis C RNA positive. Which of the following is a contraindication to treating him with combination therapy with interferon alpha and ribavirin? A : Previous intravenous drug misuse B : The presence of Hepatitis B core antibody C : Cirrhosis D : Untreated depression E : Hepatitis C genotype 1. Comment : The major side effects of alpha interferon are neuropsychiatric effects such as depression. Untreated depression is likely to worsen during treatment and should be treated before commencing therapy for hepatitis C. Hepatitis B core antibody indicates previous exposure to hepatitis B infection and previous intravenous drug users (commonest cause of HCV acquisition in the UK) are likely to have been exposed to hepatitis B in the past. Cirrhosis is not a contra-indication to therapy providing liver function is preserved (e.g. no ascites/encephalopathy) and although patients with hepatitis C genotype 1 respond less well to treatment than genotype 2 and 3 this is not a contra-indication to treatment. Factors that may predispose to a higher risk of adverse events in addition to major psychiatric disorders include: · cardiovascular disease (significant arrhythmias · congestive heart failure · uncontrolled hypertension or ischaemic heart disease) · active autoimmune diseases · poorly controlled epilepsy · diabetic retinopathy (interferon can exacerbate diabetic retinopathy) · thyroid disease (relative contraindication, as interferon can cause an autoimmune thyroiditis). Other factors increasing the risk of adverse events include myelosuppression (e.g. thrombocytopaenia and neutropaenia). Therapy should not be instituted if the platelet count is less than 80x109/l or the neutrophil count is less than 1.0x109/l. Renal failure and anaemia increase the risk of adverse effects from ribavirin. Finally, ribavirin is teratogenic, hence patients (and their partners) must use adequate contraception. Absolute contra-indications to therapy with interferon and ribavirin are decompensated liver disease, active alcohol abuse, pregnancy or lack of appropriate contraception and expected non-compliance. D : 146. An 84-year-old lady is referred to the medical admission unit by her general practitioner with an upper gastrointestinal haemorrhage. She was recently started on aspirin for a transient ischaemic attack. Her haemoglobin on admission is 9.4g/dl. Which of the following is correct? A : Mortality from peptic ulcer haemorrhage is still high and lies between 25-30%. B : Approximately 50% of patients will stop bleeding spontaneously. C : The finding of a clean ulcer base at endoscopy is associated with a 50% risk of rebleed. D : Age is not a major risk factor for uncontrolled bleeding. E : The presence of a non-bleeding vessel is associated with a 50% risk of rebleed. Comment : Mortality from a peptic ulcer haemorrhage lies between 2 and 6%. 80% of patients will stop bleeding spontaneously. A clean ulcer base is not associated with further bleeding. People 65 years and over have a higher risk of uncontrolled bleeding. The presence of a non-bleeding visible vessel on endoscopy is associated with a 50% risk of rebleeding. E :

Page 203: Masterclass Book Part 2

MOHAMMED IS-HAG 202

PLATE 33 1. A 78-year-old man presents with a 2-day history of severe headache localising to his right temple. He also feels tired and lethargic. He denies any eye problems and pain on chewing. The biopsy of his right temporal artery is shown (see image). Which of the following statements fit best? A : This is a normal temporal artery biopsy. B : This man should be treated with 60 mg prednisolone. C : There is no need to consider bone protection. D : 50% of patients with this condition develop permanent visual loss. E : An ESR of less than 40mm/hr excludes Giant Cell Arteritis (GCA). Comment : This biopsy shows thickening and lymphocytic infiltration highly suggestive of GCA. He should indeed be treated with high dose steroid initially. It is essential to consider bone protection as he is likely to be on steroids for up to 2 years. (He may also need gastric protection.) 15-20% of patients develop permanent visual loss. The ESR is only elevated in 80% of cases, so if the history is good, the diagnosis should still be considered. B : 2. A 25-year-old African man with known HIV infection presents with fever, cervical lymphadenopathy, splenomegaly and renal impairment. Significant findings on laboratory testing include a strongly positive anti-neutrophil cytoplasmic antibody (ANCA) of proteinase-3 specificity (PR3-ANCA). The positive predictive value of his PR3-ANCA for a diagnosis of small vessel vasculitis is approximately: A : <10% B : 10-20% C : 20-40% D : 40-60% E : >60%. Comment : False-positive ANCA antibodies are a well-recognised pitfall in this situation. A variety of infections are known to be associated with ANCA including bacterial endocarditis and a range of tropical infections including Amoebiasis and Malaria. It underlines the need to obtain histological evidence of small vessel vasculitis before proceeding to immunosuppressive therapy. A : 3. A 30-year-old woman presents with a 3-month history of recurrent episodes of urticaria. There are no clues in the history to suggest a possible trigger. During each episode she is tired but otherwise well and clinical examination is unremarkable. What is the most likely diagnosis? A : C1 inhibitor deficiency B : Urticarial vasculitis C : Food allergy D : Systemic mastocytosis E : Chronic idiopathic urticaria. Comment : The cardinal presenting feature of C1 inhibitor deficiency is angioedema. Urticarial vasculitis is characterized by painful wheals often lasting more than 24 hours and accompanied by systemic symptoms. In the absence of any clues in the history, this patient's urticaria is unlikely to be due to food allergy. Mastocytosis is very rare; the urticaria associated with mastocytosis is invariably accompanied by Darier's sign and GI involvement. E :

Page 204: Masterclass Book Part 2

MOHAMMED IS-HAG 203

4.

An 82-year-old woman is admitted with a history of poor mobility. There is no history of trauma and her only complaint is pain in her left knee. Examination of the knee shows that it is painful to touch but there are no signs of inflammation. An abdominal radiograph incidentally shows a left hip fracture (see image). Why is she having knee pain? A : Pain in the knee is referred to the hip by the superior gluteal nerve. B : Pain in the knee is referred to the hip by the inferior gluteal nerve. C : Pain in the knee is referred to the hip by the femoral, sciatic and obturator nerves. D : Pain in the knee is referred to the hip by the musculocutaneous nerve. E : Pain in the knee is an incidental finding. Comment : The knee pain was referred from the hip to the knee. Branches of the femoral, sciatic and obturator nerves all give twigs to both joints. The geniculate branch of the obturator is the main conveyer of pain referred from hip to knee. Referred pain from the hip to the knee is well known to occur in osteoarthritis of the hip but that it can occur in hip fractures maybe less well appreciated. Some older patients with severe osteoporosis may sustain a hip fracture simply by turning with no history of trauma. C : 5. A 68-year-old woman with longstanding congestive cardiac failure (ejection fraction 20%) presents with a hot, swollen right knee. The following results are obtained: FBC normal, Urea 11mM, Creatinine 196ul. Synovial fluid: many monosodium urate crystals seen on microscopy, culture sterile. What is the best treatment for her acute arthritis? A : Allopurinol B : Colchicine 0.5mg every 2-4 hours C : Indomethacin 50mg tds D : Co-codamol 30/500 every 6 hours E : Intra-articular corticosteroids. Comment : This patient has acute gout, probably diuretic-induced. Intra-articular corticosteroids are safe and highly efficacious in this situation, once sepsis is excluded. Allopurinol has no role in the acute treatment of gout. Colchicine at these doses is very poorly tolerated due to GI toxicity. Non-steroidals are very likely to precipitate a severe deterioration in renal function and may also exacerbate heart failure. E :

Page 205: Masterclass Book Part 2

MOHAMMED IS-HAG 204

6. A 66-year-old woman has a 15-year history of deforming rheumatoid arthritis (RA). She is maintained on D-Penicillamine. Two weeks ago she noticed increased difficulty in climbing stairs and three days before admission she was unable to comb her hair or feed herself. Neurological assessment reveals grade 3/5 weakness. Babiniski sign positive bilaterally. Which of the following test is MOST appropriate at this stage? A : Plain radiograph of the cervical spine B : Electromyography (EMG) C : Nerve conduction study (NCS) D : Isotope bone scan E : Magnetic resonance imaging (MRI) of the cervical spine. Comment : The recent limb weakness and the presence of pyramidal signs in the legs in patients with RA are highly suggestive of spinal cord lesion at the cervical or thoracic region. RA primarily affects the cervical spine. Affliction of the thoracic or lumbar spine is rare. The anatomic abnormalities occur as a consequence of the destruction of synovial joints, ligaments, and bone. Atlantoaxial subluxation (AAS) is the most common. Patients may experience weakness, decreased endurance, gait difficulty, paresthesias of the hands, and loss of fine dexterity. Multiple neurologic signs may be elicited on physical examination, including diffuse hyperreflexia, lower extremity spasticity, a spastic gait, and Babinski's sign. Although plain radiograph of the cervical spine is regarded as the initial imaging assessment tool for neck pain in patient with RA, those with symptoms or signs of cord compression should undergo immediate MRI and be sent for surgical consultation. MRI is considered the most sensitive imaging modality for diagnosing spinal cord compression, evaluating its extent, and assessing soft tissues (panus) as well as bone destruction. D-Penicillamine causes various neuromuscular disorders including myshtenia gravis. There is no clinical evidence for muscle disease or peripheral neuropathy in this patient. Therefore, EMG or NCS are not necessary. Isotope bone scan is a sensitive test and will identify an increased activity at the cervical region but it is not specific and has little role in investigating spinal cord compression cases. E : 7. A 68-year-old woman with longstanding congestive with a 3-month history of fatigue. She has read about lupus on the world wide web and asks her GP to carry out appropriate blood test(s) to rule out lupus. Which single blood test is best suited to screen for lupus in this situation? A : Antinuclear antibody B : Serum Immunoglobulin levels C : Anti-cardiolipin antibodies D : Anti-DNA antibodies E : Anti-ENA antibodies. Comment : ANA testing carried out on Hep-2 cells offers the highest degree of sensitivity for lupus. Conversely, a negative ANA in this situation effectively rules out systemic lupus erythematosus. The other tests although useful do not, if normal or negative, enable you to rule out lupus. A : 8. A 25-year-old woman with a history of 3-second trimester fetal losses is planning a fourth pregnancy. She has evidence of the primary anti-phospholipid syndrome (strongly positive cardiolipin antibody, positive lupus anticoagulant but no evidence of lupus). Which of the following treatment regimens offer her the best chance of having a successful pregnancy? A : Steroids alone B : Steroids combined with low-dose aspirin C : Low-dose aspirin alone D : Low-dose aspirin combined with low molecular weight heparin E : Intravenous immunoglobulin. Comment : Of the above treatment options, low-dose aspirin combined with low molecular weight heparin offers her the best chance of having a successful pregnancy. D : 9. You are called to A&E to review a 28-year-old woman with pleuritic chest pain. Which of the following statements concerning this patient are true? A : Low white count is consistent with systemic lupus erythematosus (SLE) B : Low white count is not consistent with tuberculosis. C : The presence of anticardiolipin antibodies IgG 17 GPLU/ml (NR <14), IgM 21 MPLU/ml (NR<10)

suggests that she may have a pulmonary embolus. D : The presence of antinuclear antibodies at a titre of 1/80 suggests that she may have a

connective tissue disease. E : The presence of antinuclear cytoplasmic antibodies (cANCA) at a titre of 1 in 20 suggests that

Wegener's granulomatosis is likely. Comment : Active SLE and tuberculosis are both associated with leukopenia. Low titre antinuclear antibodies (in the absence of DNA or ENA), antineutrophil cytoplasmic antibodies (ANCA) and anticardiolipin antibodies are non-specific and are commonly found in the presence of infection. A :

Page 206: Masterclass Book Part 2

MOHAMMED IS-HAG 205

10.

PLATE34 A 35-year-old nurse developed Raynaud's phenomenon six months ago (see image). The antinuclear antibody (ANA) test was positive at 1:1000 with a speckled staining pattern. She came to the out-patient department complaining of excessive tiredness and lethargy. The creatine phosphokinase (CPK) was three times higher than the upper normal limits, and the hand joints radiographs showed no erosions. Which one of the following tests would you request to establish the diagnosis? A : Anti-U1 RNP B : Anti-ds DNA C : Anti-Scl 70 D : Anti-centromere antibodies E : Anti-PM-Scl Comment : The picture shows puffy hands, and fingers with swan neck and boutonniere deformities. The fact that she had gross deformities of the hands without joint damage or erosions, might suggest that she has Jaccoud's arthropathy, which is more often seen in patients with connective tissue disease such as systemic lupus erythematosus (SLE). This patients also exhibits features probably suggestive of an overlap between SLE, myositis and scleroderma which constitute the diagnosis of mixed connective tissue disease (MCTD).MCTD is an overlap syndrome characterised by combinations of clinical features of SLE, systemic scleroderma and polymyositis. The presenting symptoms of MCTD are most often: · Raynaud's phenomenon · puffy hands · arthralgias · myalgias · fatigue. The various features of the connective tissue disorders making up MCTD develop over months and years. A defining feature of MCTD is the presence of antibodies against the U1 ribonucleoprotein (U1 RNP) complex, and hence the presence of high titre anti-U1 RNP will confirm the clinical diagnosis of MCTD. Although the other tests might also be positive, they will not be helpful in establishing the underlying cause of the whole clinical spectrum. A : 11. A 70-year-old female has a 17-year history of rheumatoid arthritis. She presents with recurrent attacks of red congested eyes with a sensation of grittiness. The most likely cause of her red eyes is likely to be: A : scleritis B : episcleritis C : keratitis D : keratoconjunctivitis sicca E : choroiditis.

Page 207: Masterclass Book Part 2

MOHAMMED IS-HAG 206

Comment : Approximately 25 % of patients with rheumatoid arthritis (RA) will have ocular manifestations. keratoconjunctivitis sicca, scleritis, episcleritis, keratitis, peripheral corneal ulceration, and other less common entities such as choroiditis, retinal vasculitis, episcleral nodules, retinal detachments, and macular edema. Keratoconjunctivitis sicca, or dry eye syndrome, is the most common ocular manifestation of RA and has a reported prevalence of 15 to 25 %. The patient reports a gritty sensation in the eyes . Scleritis and episcleritis as causes of recurrent red eyes are distinguished on the basis of anatomy and appearance. Symptoms may be similar, but the pain in scleritis is more evident and severe. Tenderness to palpation of the globe can help differentiate the two. After asking the patient to look down with eyelids closed, the physician gently presses the globe. Patients with scleritis have tenderness on palpation, while those with episcleritis do not.. Unlike scleritis, patients with episcleritis do not complain of blurred vision or photophobia. The importance of correctly diagnosing and distinguishing between scleritis and episcleritis is based on the potential ocular and systemic complications associated with scleritis. Studies have shown that patients with RA-associated scleritis have more widespread systemic disease and a higher mortality rate than those episcleritis. D : 12. A 72-year-old man with longstanding rheumatoid arthritis (RA) presents with a 3-day history of numbness and clumsiness of both hands and difficulty walking more than a few yards because of weakness and unsteadiness of his legs. Which of the following causes of his deterioration seems most likely? A : Carpal tunnel syndrome B : Peripheral neuropathy C : Secondary osteoarthritis D : Systemic vasculitis E : Cervical myelopathy. Comment : All the options could occur in RA but the combination of recent onset sensory symptoms in the hands and weakness of the legs is highly suggestive of cervical cord compression. An urgent MRI scan is indicated. Options B and D could produce a similar clinical picture but are less common than myelopathy in the context of RA. E : 13. A 16-year-old girl has been referred from her GP. She states that she has taken thirty paracetamol tablets (500mg) over the last 12 hours. She is currently seeing the psychiatrists for outpatient management of anorexia. Which of the following is the most appropriate immediate management? A : Perform gastric lavage and check paracetamol levels four hours after the last tablet was taken B : Give activated charcoal and check paracetamol levels four hours after the last tablet was taken C : Give activated charcoal and check paracetamol levels immediately D : Give activated charcoal and start n-acetyl cysteine E : Wait for the results of paracetamol levels before instituting treatment. Comment : Paracetamol levels are very difficult to interpret if the patient has taken the tablets over a period of time. This girl is at high risk because of the alleged number she has taken and the coexistant anorexia. Gastric lavage is only ever indicated if the patient has taken a large number in the hour or so before presentation. Charcoal will help to prevent absorption. It would be safest to start n-actyl cysteine and check her clotting over the next 24 hours. D : 14. A 65-year-old with a mitral valve replacement presents to A&E with pyrexia and fainting. She is unwell, hypotensive, anaemic and pyrexial. She has a vague history of suffering from a reaction to penicillin in her childhood. After taking blood cultures she is started on broad-spectrum antibiotics. Cardiac valvular vegetations are seen on echocardiography and her blood grows methicillin-sensitive Staphylococci – the microbiologist suggests naficillin as the most appropriate antibiotic, but is concerned that she may have allergy to beta lactam-based antibiotics. Which of the following is most appropriate to investigate this lady’s history of possible penicillin allergy? A : serum tryptase B : skin prick test to penicillin C : serum penicillin specific IgE D : patch test to penicillin E : serum IgE. Comment : A history of penicillin allergy is relatively common in hospital patients, and is in most not due to type I hypersensitivity reactions. A diagnosis of penicillin allergy crucially requires a detailed history of the drug reaction, and can be confirmed by a positive skin prick test to the major and minor determinants of penicillin. Skin prick testing is carried out if there is a clinical need for penicillin treatment e.g. treatment of infective endocarditis. A patient is unlikely to develop anaphylaxis with a negative penicillin skin prick test. The detection of penicillin specific IgE in the serum is unreliable. B :

Page 208: Masterclass Book Part 2

MOHAMMED IS-HAG 207

15.

A 70-year-old diabetic had problems with decreased mobility of his back. The problem had progressively worsened over the last seven years. His lumbar spine was remarkable for loss of the normal lordosis and decreased range of motion in all planes. He had no pain on percussion over the spine or sacroiliac joints. He was otherwise in good health, and the erythrocyte sedimentation rate (ESR) was normal. A radiograph of the lumbar spine is shown. The most probable diagnosis is: A : Ankylosing spondylitis (AS) B : Alcaptonuria C : Osteoporosis D : Spondylolisthesis E : Diffuse idiopathic skeletal hyperostosis (DISH) Comment : Radiographs of the lumbar spine showed only minimal degenerative disc disease, but large, coarse osteophytes that bridged the lower vertebral bodies. These findings are classic for DISH. Criteria for DISH require that new bone formation bridge four contiguous vertebral bodies in the absence of significant degenerative disk disease, and in the absence of inflammatory sacroiliac or facet changes. There is increased prevalence of DISH in diabetes mellitus and obesity. The lack of sacroiliac tenderness in this man, with longstanding symptoms, normal sedimentation rate and advanced age of onset, rendered AS an unlikely diagnosis. Alcaptonuria is a hereditary disease in which homogentisc acid oxidase activity is missing. Homogenitisc acid accumulates and cause pigmentation of cartilage and other connective tissues (ochronosis). It usually starts in the second or third decade. The vertebral bodies of the lumbar spine show degeneration of the intervertebral discs, with narrowing of the space and dense calcification of remaining disc material. There is variable fusion of vertebral bodies, but little osteophyte formation and minimal calcification of intervertebral ligaments. Osteoporosis is associated with reduced bone density and wedge fractures of the vertebrae. Spodylolisthesis is forward movement of one vertebra in relation to an adjacent vertebra. E : 16. A 30-year-old female nurse presents with a 3-month history of Raynaud's phenomenon. Clinical examination reveals cold hands but no other evidence of connective tissue disease. Which of the following tests is most helpful in determining future progression to systemic connective tissue disease? A : Positive anti-mitochondrial antibody B : Positive anti-gastric parietal cell antibody C : Positive smooth muscle antibody D : Positive anti-nuclear antibody E : Positive rheumatoid factor.

Page 209: Masterclass Book Part 2

MOHAMMED IS-HAG 208

Comment : Although 5% of the general population have Raynaud's phenomenon, only a minority go on to develop systemic connective tissue disease. A positive ANA is the single best predictor of existing or future progression to connective tissue disease in this situation. D : 17. A 30-year-old teacher presents with a 6-month history of swelling and pain involving the distal interphalangeal joints of the hands. The ESR is 65 mm in the first hour. What is the most likely diagnosis? A : Generalised osteoarthritis B : Rheumatoid arthritis C : Psoriatic arthritis D : Systemic lupus erythematosus (SLE) E : Gout. Comment : Arthritis with predominant involvement of the distal interphalangeal joint occurs most often in generalised osteoarthritis and psoriatic arthritis. The fact that this patient is relatively young and has a raised ESR indicates an underlying inflammatory disease is the most likely cause of her symptoms. Therefore psoriatic arthritis is the most likely diagnosis in this case. Rheumatoid arthritis and SLE are known to affect the proximal interphalangeal (PIPs) and the metacarpophalangeal (MCPs) joints. Chronic gouty arthritis might involve the DIPs, but more often it involves the MCPs and PIPs in asymmetrical fashion with or without tophus formation.Examination of the skin and nail for psoriasis is very important in confirming the diagnosis. Hairlines in the scalp, the naval and the palms are areas often involved in psoriasis but easily missed. C : 18. A 28-year-old woman presented with fatigue and extreme tiredness. Physical examination revealed facial skin rash and tenderness across the small joints of the hands. She was concerned that she might have systemic lupus erythematosus (SLE). Which of the following tests when NEGATIVE will virtually exclude the diagnosis of SLE? A : Antinuclear antibody (ANA) B : Anti-double stranded DNA (Ads-DNA) C : Anti-Sm antibodies D : Anti-histone antibodies E : Anti-Ro/SSA antibodies. Comment : Almost all patients with SLE have positive ANA test results. The ANA test is sensitive but not specific for SLE. A negative result argues strongly against a diagnosis of active SLE, but does not exclude the possibility of other autoimmune diseases. Antibodies to Sm antigen are highly specific for a diagnosis of SLE (>99%). However, only about 25% of patients with SLE have anti-Sm antibodies. Anti-DNA antibodies are diagnostic of SLE (specificity >99%). However, only 60% of patients with SLE will have these antibodies. Therefore, absence of anti-DNA or anti-Sm antibodies should not exclude SLE as a diagnosis. Anti-Ro/SS-A antibodies are found in 30% of patients with SLE. Anti-histone antibodies are identified in small proportion of SLE patients. They are more often seen with drug-induced lupus. A : 19. An 8-year-old has been admitted for a third time with haemolytic-uraemic syndrome and is found to have a low C3. Which of the following is the most likely diagnosis? A : Lupus B : Gram-negative septicaemia C : Factor H deficiency D : Factor I deficiency E : C3 deficiency. Comment : Factor H deficiency accounts for 5-10% of all cases of haemolytic-uraemic syndrome. Typically it has a higher mortality and over half of patients will suffer relapses. Over-activation of the alternative complement pathway results in a low serum C3. Although able to cause a low C3, lupus, factor I and C3 deficiency are not associated with recurrent haemolytic-uraemic syndrome. C : 20. The parents of a 10-year-old asthmatic boy with peanut allergy are concerned about the risk of future anaphylaxis if he were to inadvertently ingest peanuts. Which of the following features is the single most important predictor of anaphylaxis in this situation? A : Level of peanut-specific IgE in his serum B : Strength of positive skin test response to peanut C : Poorly controlled asthma D : Previous steroid therapy E : Family history of nut allergy. Comment : Poorly controlled asthma is an important risk factor for fatal anaphylaxis in this situation. Children such as this should have their asthma well controlled and in addition, have ready access to self-injectable adrenaline. C :

Page 210: Masterclass Book Part 2

MOHAMMED IS-HAG 209

21.

PLATE 35 A 78-year-old woman found it much easier to manage after the occupational therapist provides several aids and appliances for use in the kitchen. From their appearance, the problem affecting her hands (see image) is: A : rheumatoid arthritis B : nodal osteoarthritis C : psoriatic arthropathy D : tophaceous gout E : Charcot joints. Comment : This is the classic appearance of arthritis mutilans in psoriatic arthropathy, with a deforming destructive arthritis of the small joints. Osteolysis results in shortening of some of the digits. The pattern is not that of rheumatoid arthritis and the wrists appear to be spared. There is too much destruction for osteoarthritis. The appearance of gout may be similar, though the tophi would be easier to see in life (as opposed to a photo). Charcot joints (neuropathic arthropathy) usually affect the foot and ankle. C : 22. A 35-year-old man is referred to you for investigation of recurrent infection. He has had frequent respiratory tract infections for the past 5 years, requiring 4-5 courses of antibiotics each winter. A month previously he was admitted with pneumococcal pneumonia. Two months prior to that he had sinus surgery. However, this did not improve his recurrent sinusitis. Which of the following is not in the differential diagnosis? A : Antibody deficiency B : HIV infection C : Bronchiectasis secondary to recurrent infection D : Complement C6 deficiency E : Smoking 5 cigarettes per day. Comment : Antibody deficiency is typically associated with respiratory tract infections. Ask about diarrhoea and bacterial skin infections which are also common. Take a careful drug history and bear in mind the possibility of lymphoproliferative disease. HIV infection, although primarily associated with CD4 loss, also results in antibody dysfunction, resulting in recurrent respiratory tract infections in some patients. Ask about features of cellular immune deficiency (oral candida, herpes simplex and zoster, warts). Ask about risk factors. Recurrent bacterial chest infections, whatever their cause, will eventually result in bronchiectasis, hence the importance of early diagnosis and treatment. Terminal complement deficiencies (C5-9) are extremely rare. Patients are well but have increased susceptibility to neisserial infection. Smoking causes ciliary paralysis. The resultant mucociliary dysfunction is a common (and reversible) cause of recurrent respiratory tract infection. D :

Page 211: Masterclass Book Part 2

MOHAMMED IS-HAG 210

23. A 15-year-old boy presented with arthralgia, skin rash and haematuria. Renal biopsy showed focal necrotising glomerulonephritis with diffuse mesangial IgA deposits. What is the most likely diagnosis? A : Systemic lupus erythematosus (SLE) B : Henoch–Scholein purpura C : Juvenile rheumatoid arthritis D : Post-streptococcal glomerulonephritis E : Goodpasture's syndrome. Comment : Henoch–Sch?nlein purpura (HSP) is characterised by the tissue deposition of IgA-containing immune complexes. The pathogenesis of this disorder may be similar to that of IgA nephropathy, which is associated with identical histologic findings in the kidney. HSP occurs more often in children than in adults, and many cases follow an upper respiratory tract infection, suggesting that the precipitating antigen may be infectious The clinical manifestations include a classic tetrad that can occur in any order and at any time over a period of several days to several weeks: rash, arthralgias, abdominal pain, and renal disease typically purpuric (with normal clotting studies) and distributed symmetrically over the lower legs and arms. B : 24. A 64-year-old man presents to A&E with a 2-day history of increasing pain and swelling of his left knee. He denies a history of trauma. On examination, the knee is hot, red, swollen and extremely tender. Which of the following investigations is most important? A : Plain radiograph of the knee B : Blood cultures C : C-reactive protein (CRP) D : Joint aspiration E : Plasma uric acid level. Comment : Aspiration of the knee with microscopy to look for pus cells in the case of septic arthritis and crystals in the case of gout or pseudogout is the most useful investigation. Blood cultures may yield an organism. The radiographs are really as a baseline and are unlikely to show acute changes. Uric acid levels may not be elevated in an acute episode of gout. A raised CRP is non-specific, but can be used to monitor the effectiveness of treatment. D : 25.

PLATE 20 A 52-year-old accountant, with a 6-year history of Raynaud’s phenomenon presents to her G.P. with a 4-month history of worsening dysphagia. Incubation of her serum on Hep 2 cells reveals the following immunofluorescent pattern (see image). A : Primary Raynauds disease B : systemic lupus erythematosus (SLE) C : Limited systemic sclerosis D : Rheumatoid arthritis E : Sjogren's disease. Comment : Raynaud’s phenomenon is common, occurring in 4-15% of the general population and in the majority of cases is not associated with connective tissue disease. However, the detection of an anti-centromere nuclear staining pattern has a high specificity for limited systemic sclerosis, especially in the context of dysphagia which is a feature of the disease. C : 26. A 31-year-old man with known selective IgA deficiency and abdominal pain is tested for anti-endomysial antibodies (EMA). His results are as follows: IgA endomysial antibody negative, IgG endomysial antibody positive. Which of the following statements is true? A : Coeliac disease can be confidently excluded on the basis of these results. B : His IgA deficiency is irrelevant to his negative IgA EMA status. C : The negative IgA EMA is of no diagnostic value in the presence of IgA deficiency. D : Referral for small bowel biopsy is not warranted on the strength of these results. E : For diagnostic purposes, IgA and IgG EMA are equally important Comment : Since there is a recognized association between coeliac disease and IgA deficiency, it is important to recognize that IgA EMA may be spuriously negative in patients with coeliac disease and

Page 212: Masterclass Book Part 2

MOHAMMED IS-HAG 211

total IgA deficiency, as in this case. For this reason, testing for coeliac disease antibodies always includes measurement of both IgA and IgG antibody isotypes or measurement of serum IgA. Such a testing strategy will alert both the laboratory and the requesting clinician of 'false-negative' results as in the case here. In keeping with the mucosal route of antigen entry, IgA endomysial antibodies are more important than IgG antibodies as a diagnostic marker of coeliac disease. C : 27.

PLATE 36 A 13-year-old boy was hospitalised because of a purpuric rash involving the legs (see image) associated with abdominal pain and fever (38°C). The patient also complained of arthralgia involving the knees and ankles. Urinalysis showed proteinuria with microscopic haematuria. A biopsy of the purpuric lesion revealed leucocytoclastic vasculitis in the small vessels. Which of the following statements is true about this boy's illness? A : The urine abnormality and fever is most probably related to a recent urinary tract infection B : In such cases blood cultures are often positive for Pseudomonas aerogenosa C : Renal biopsy typically shows mesangial IgA deposition D : Bilateral small kidneys are a constant feature on ultrasound of the kidneys E : Untreated, up to 50% will develop chronic renal failure. Comment : Henoch-Schonlein Purpura (HSP) is recognized as a systemic small vessel vasculitis involving mainly the blood vessels of the skin, GI tract, the kidneys and the joints.HSP affects mainly children between the ages of three and 10 years. Males are affected more often than females (1.5:1) and in about two thirds of children an upper respiratory tract infection precedes the onset of HSP by one to three weeks. Kidney involvement in HSP is secondary to vasculitis which is not associated with previous or concurrent urinary tract infection. HSP nephritis becomes clinically manifest in only 20-30%. It usually presents as macroscopic haematuria and proteinuria lasting from a matter of days to several weeks. The kidneys are usually of normal size. Of those patients with renal involvement, as many as 10% may develop chronic renal failure and end-stage renal disease. However, fewer than 1% of all patients with HSP suffer this poor prognosis. C : 28. A 67-year-old woman was referred to a specialist rheumatology clinic. She had poorly controlled rheumatoid arthritis. For the last fifteen years she had been treated with various disease-modifying drugs including sulphasalazine and methotrexate. Blocking the biologic function of which of the following molecules will provide significant anti-inflammatory effect in RA? A : Interleukin-10 (IL-10) B : Interleukin-4 (IL-4) C : Interleukin-1 (IL-1) D : Soluble TNF receptors E : Transforming growth factor-beta (TGF-beta). Comment : Advances in research have identified specific cells and cell products (primarily cytokines) that may play distinct roles in RA activity. It has been well documented that in RA, proinflammatory

Page 213: Masterclass Book Part 2

MOHAMMED IS-HAG 212

cytokines mediate synovial proliferation and articular tissue destruction. For these reasons, cytokines have become a target for therapy. Cytokine-based therapies include a) blocking of proinflammatory cytokines (IL1, IL2, IL6, IL8, TNFa, G-CSF, GM-CSF) or, b) augmentation of anti-inflammatory cytokines (IL4, IL10, IL11, IL13, TGFb, soluble TNF receptors, soluble IL2 receptors). Two of the major cytokines thought to incite inflammation in RA patients are TNF and IL-1. This has resulted in the introduction of two inhibitors of TNF, the soluble TNF receptor construct Etanercept and the anti-TNF monoclonal antibody (mAb) Infliximab. More recently anti-IL-1 Kineret has been licensed for use in active RA. On the other hand IL10, IL-4 and TGF-beta may be considered a natural ‘dampeners’ of the immune response and can be used as anti-cytokines. Therefore augmentation of theses cytokines, rather than blocking them, hold promise for new biologic therapy in RA. C : 29. Which one of the following enzyme defects is not known to cause hyperuricaemia? A : Increased posphoribosylpyrophosphate (PRPP) synthetase activity B : Hypoxanthine-guanine phosphoribosyltransferase (HGPRT) deficiency C : Glucose-6-phosphatase (G-6-P) deficiency D : Xanthine oxidase deficiency E : Fructose-1-phosphate aldolase deficiency Comment : Enzyme defects that increase the rate of purine biosynthesis may cause gout by overproduction of uric acid. Four specific defects appear to be important as causes of uric acid overproduction: · HGPRT deficiency · PRPP synthetase overactivity · G-6-P deficiency · fructose-1-phosphate aldolase deficiency. Patients with the Lesch-Nyhan syndrome, which is caused by a virtually complete deficiency of HGPRT, are characterized by: · choreoathetosis · striking growth and mental retardation · spasticity · self-mutilation · marked hyperuricemia, with excessive uric acid production and uric acid crystalluria. In patients with G-6-P deficiency (von Gierke's disease, GSD type I) and hereditary fructose intolerance caused by fructose-1-phosphate aldolase deficiency, hyperuricemia develops, in part, because of accelerated purine nucleotide catabolism. In patients with liver xanthine oxidase deficiency, purine breakdown stops at the xanthine-hypoxanthine stage. Plasma and urinary urate are very low. Allopurinol is a xanthine oxidase inhibitor. D : 30. A 32-year-old man presents with a 4-month history of back pain. The pain is worse in the morning and after sitting watching TV. Plain radiograph of the spine/pelvis shows evidence of sacroiliitis. Each of the following clinical features might be identified on clinical examination of this patient EXCEPT? A : Keratoderma blenorhegica B : Rheumatoid nodule C : Onycholysis D : Uveitis E : Urethritis. Comment : The long (> 1 hr) early morning and rest stiffness are highly suggestive of an underlying inflammatory condition. The pelvic radiograph confirms the presence of sacroiliitis. Bilateral sacroiliitis are typical for ankylosing spondylitis. The frequency of asymmetric sacroiliitis may be higher in other spondyloarthropathies, e.g., reactive arthritis, Reiter’s syndrome, spondylitis associated with psoriasis, or inflammatory bowel disease.Reiter's syndrome is characterised by a triad of arthritis, urethritis, and conjunctivitis. Reiter's syndrome develops in the setting of postdysenteric or postvenereal illness. The characteristic rashes of keratoderma blennorrhagicum and circinate balanitis may be present. Psoriatic spondyloarthropathy is characterised by psoriatic plaques. The skin involvement may be subtle and should be searched for carefully. The cleft of the buttock, scalp hairline, and penis are site often involved but may be missed easily if thorough examination of these areas was not actively conducted. Psoriatic nail changes include onycholysis, yellow nails and nail pitting. Inflammatory bowel disease consists of ulcerative colitis and Crohn's disease. A diagnosis of ankylosing spondylitis may be made when specific features of Reiter's syndrome, psoriasis, or inflammatory bowel disease are absent. Sacroiliitis is also encountered in tuberculosis, sarcoidosis and brucellosis. Rheumatoid arthritis is not associated with lumber or sacroiliac joint disease. However cervical spondylitis and atlanto-axial subluxation is not an uncommon feature of RA. B :

Page 214: Masterclass Book Part 2

MOHAMMED IS-HAG 213

31. A 36-year-old woman presents with deteriorating nocturnal paraesthesia affecting both hands. It improves during the morning. Which of the following tests is least likely to be helpful in establishing the cause? A : Thyroid function test B : Rheumatoid factor C : Tinels test D : MRI scan hands E : Urinary hCG. Comment : Carpal tunnel syndrome classically presents with nocturnal paraesthesia in the palms of the hands which improves by shaking / using the hands. It is caused by compression of the median nerve within the carpal tunnel in the wrist. MRI hands would not be indicated. Causes of Carpal tunnel syndrome include hypothyroidism, synovitis including rheumatoid arthritis and pregnancy. Tinel’s test is a provocation test to reproduce symptoms by percussion over the median nerve. D : 32.

PLATE 37 This patient was trying to stand up. On examination he has periorbital oedema with a faint purple hue. The serum creatinine kinase (CK) levels were 10-times the upper limit of normal. Which of the following manifestations is least likely to be associated with this disorder? A : blue and white color change in the fingers on exposure to cold B : difficulty in swallowing liquids C : double vision on looking to the sides D : radial pulse of 40 beats per minute E : persistent cough, haemoptysis and weight loss of 12 kg in the last three months Comment : This patient is not able to stand up from a chair unaided. This indicates proximal muscle weakness. The skin rash and the raised levels of muscle enzyme point towards an inflammatory myopathy, namely dermatomyosytis (DM).DM is easily recognized and diagnosed by a characteristic rash: · a heliotrope rash (blue-purple discoloration) on the upper eyelids, with oedema · a flat red rash on the face and upper trunk. Erythema of the knuckles accompanied by a raised, violaceous scaly eruption (Gottron's sign) is also characteristic, and may precede or follow muscle weakness. Raynaud's phenomenon with characteristic color change (pallor, cyanosis and rubor) on exposure to cold is encountered in patient with DM, more often when there is overlap with other connective tissue disease such as scleroderma or mixed connective tissue disease. Inflammation of the skeletal muscles of the oropharynx and upper oesophagus lead to dysphagia especially for liquids in the initial stages of the disease which progresses to difficulty in swallowing solids as the disease progresses. Electrocardiographic evidence of conduction defects and arrhythmias occur frequently in childhood and adult DM, although overt clinical symptoms are uncommon.There is an increased incidence of underlying malignancy in adult DM, ranging from 5% to 15%. Carcinoma of the bronchus, ovaries and breast are the most common.Ocular muscles remain normal, even in advanced, untreated cases, and if these muscles are affected, the diagnosis of inflammatory myopathy should be in doubt. C :

Page 215: Masterclass Book Part 2

MOHAMMED IS-HAG 214

33. A 30-year-old woman develops a systemic reaction characterized by hypotension, bronchospasm and widespread urticaria soon after induction of anaesthesia for cholecystectomy. Which of the following blood test results would suggest that her reaction was associated with mast cell degranulation? A : Elevated plasma tryptase B : Hypernatraemia C : Hypokalaemia D : Hypocomplementaemia E : Hypergammaglobulinaemia. Comment : The constellation of acute symptoms in this case is highly suggestive of a systemic allergic reaction, either anaphylaxis (if IgE mediated) or an anaphylactoid reaction (if non-IgE mediated). Both of these reactions are due to extensive mast cell degranulation leading to release of large amounts of tryptase in to the circulation. Elevated tryptase levels in the context of this lady's reaction are highly suggestive of an anaphylactic/anaphylactoid reaction. This lady will need to be investigated during convalescence in an allergy clinic to determine the cause of her reaction. A : 34. A 75-year-old woman presents with acute pain and swelling in the right knee. On examination the right knee is hot and tender with moderate synovial fluid collection. Radiograph of the right knee shows calcification of the meniscus. Synovial fluid was sterile on culture and rhomboid shaped crystals were identified on microscopy of the fluid. Which of the following statements about this condition is NOT true? A : Narrowing of the joint space is often present B : The symphysis pubis is one of many other joints that can be affected by this disorder C : Serum calcium levels may be elevated in some patients D : Negatively birefringent crystals may be identified on synovial fluid analysis E : There may be large subchondral cysts Comment : This patient presents with acute monoarthritis of the right knee. The clinical picture and the radiological findings are suggestive of pseudogout and calcium pyrophosphate deposition (CPPD), which is characterised by the presence of positively birefringent crystals on synovial fluid analysis (negatively birefringent ones are found in gout). The knee is the most common site affected by this disorder. Other locations include: · the triangular fibrocartilage of the wrist · sacroiliac joints · symphysis pubis · the glenoid of the shoulder · the labrum of the hip · the elbow · the ankle · acromioclavicular joint. CPPD is associated with hemochromatosis, primary hyperparathyroidism, hypothyroidism, and hypomagnesemia. Other findings include joint space narrowing, large subchondral cysts and many intra-articular bodies from subchondral bone fragmentation. D : 35. A 60-year-old-man complained of pain in both wrist evolving over 8 weeks. He noted swelling around that area but denied stiffness. On examination there was swelling and tenderness just proximal to the wrist joints without limitation of movement. There was also prominent finger clubbing. Radiographs revealed periosteal reaction over the lower end of the radius and ulnar. Each of the following disorders could be the cause behind this patient complain EXCEPT? A : Mesothelioma B : Bronchiectesis C : Diabetes mellitus D : Crohn's disease E : Whipple's disease. Comment : Hypertrophic osteoarthropathy (HOA) is a syndrome characterized by proliferative changes in the skin and skeleton. Proliferative periostitis of the radius and fibula and digital clubbing are commonly seen. Two forms of the syndrome are seen, a rare idiopathic from called pachydermoperiostitis (3-5%) (familial autosomal dominant), and more common secondary form (95-97%). Secondary HOA was initially described in association with chronic suppurative infection and malignancy of the lung and pleura. Therefore it used to be called hypertrophic pulmonary osteoarthropathy (HPOA). Pleural causes include pleural fibroma and mesothelioma. Pulmonary causes include bronchogenic carcinoma, pulmonary tuberculosis; pulmonary abscesses, bronchiectasis emphysema; and Pneumocystis carinii infection in patients with AIDS, Hodgkin’s disease, metastases, or cystic fibrosis. Cyanotic heart disease with a right-to-left shunt is the only cardiac cause described.

Page 216: Masterclass Book Part 2

MOHAMMED IS-HAG 215

Abdominal causes include liver cirrhosis, ulcerative colitis, Crohn disease, Whipple disease, and biliary atresia. In long standing insulin dependent diabetes mellitus, contracture of the flexor tendons of the fingers with tight, waxy skin appearance leads to diabetic cheirarthropathy. There is no pain and minor functional impairment. Diabetes mellitus is not associated with HOA. C : 36. A 35-year-old man is referred to you for investigation of recurrent infection. He has had frequent respiratory tract infections for the past 5 years, requiring 4-5 courses of antibiotics each winter. A month previously he was admitted with pneumococcal pneumonia. Two months prior to that he had sinus surgery. However, this did not improve his recurrent sinusitis. Which of the following is not in the differential diagnosis? A : Antibody deficiency B : HIV infection C : Bronchiectasis secondary to recurrent infection D : Complement C6 deficiency E : Smoking 5 cigarettes per day. Comment : Antibody deficiency is typically associated with respiratory tract infections. Ask about diarrhoea and bacterial skin infections which are also common. Take a careful drug history and bear in mind the possibility of lymphoproliferative disease. HIV infection, although primarily associated with CD4 loss, also results in antibody dysfunction, resulting in recurrent respiratory tract infections in some patients. Ask about features of cellular immune deficiency (oral candida, herpes simplex and zoster, warts). Ask about risk factors. Recurrent bacterial chest infections, whatever their cause, will eventually result in bronchiectasis, hence the importance of early diagnosis and treatment. Terminal complement deficiencies (C5-9) are extremely rare. Patients are well but have increased susceptibility to neisserial infection. Smoking causes ciliary paralysis. The resultant mucociliary dysfunction is a common (and reversible) cause of recurrent respiratory tract infection. D : 37. A 36-year old woman is referred with a 1-year history of muscle pain, tiredness and sleep disturbance. She denies fever, weight loss and arthralgia. Examination reveals tenderness over her occiput, trapezius and lumbar area. Her blood results show a normal ESR, CRP, FBC, a weakly positive ANA 1:80 and normal complement. Which is the most likely diagnosis? A : Polymyositis B : System lupus erythematous (SLE) C : Sjogren’s syndrome D : Polymyalgia rheumatica E : Fibromyalgia. Comment : The lack of constitutional symptoms, normal inflammatory markers and normal examination, apart from evidence of tender points, make an inflammatory rheumatological disease unlikely. The presence of tender points, history of muscle pain and sleep disturbance are suggestive of fibromyalgia – a non-inflammatory pain disorder. E : 38. A 40-year-old woman presents with a 6-month history of a purpuric rash on her legs, non-specific joint pains and vague ill health. Initial investigations reveal a positive rheumatoid factor. The GP makes a tentative diagnosis of rheumatoid arthritis and refers her to hospital. Results of further investigations are as follows: Urinalysis: red cell casts, protein+, ANA 1/80, anti-DNA negative, anti-ENA negative, serum C3 1.02 g/l (NR 0.75-1.65), C4 <0.02 g/l (0.20-0.65), CRP <5mg/l (NR <5), creatinine 145 (NR 50-140). Which is the most likely diagnosis? A : Systemic lupus erythematosus (SLE) B : Rheumatoid arthritis C : Mixed cryoglobulinaemia D : Primary complement deficiency E : C1 inhibitor deficiency. Comment : This lady has a hypocomplementaemic glomerulonephritis (note active urine sediment) and a purpuric rash accompanied by a positive rheumatoid factor and an isolated positive ANA. The combination of renal and skin involvement in this setting with a markedly low C4 suggests mixed cryoglobulinaemia. Her serology does not support SLE. The key investigations which would help confirm the diagnosis would be a warm clotted sample for cryoglobulins and a renal biopsy. C : 39. A 60-year-old accountant complains of recurrent attacks of exquisite pain and swelling in the left big toe. Which of the following conditions is NOT likely to be associated with this disorder? A : Chronic alcoholism B : Obesity C : Rheumatoid arthritis D : Diabetes mellitus E : Diuretic therapy.

Page 217: Masterclass Book Part 2

MOHAMMED IS-HAG 216

Comment : Acute gout is intensely inflammatory, and is therefore characterised by severe pain, redness, swelling and disability. At least 80% of initial attacks involve a single joint, typically in the lower extremity, most often at the base of the great toe (first metatarsophalangeal joint, known as podagra), or in the knee. Trauma, surgery, starvation, alcohol ingestion, dietary overindulgence, and ingestion of drugs -diuretics (cyclosporin and low dose aspirin) affecting serum urate concentrations may all promote gouty attacks. Similarly chronic disorders such as diabetes mellitus, obesity, hyperparathyroidism and hypothyroidism are associated with increased incidence of acute gouty attacks. Rheumatoid arthritis is not associated with increased incidence of hyperuricaemia or gout. C : 40. A 34-year-old man presents with severe low back pain, which has forced him to stop work as a bus driver. He has had back pain on and off for many years, on occasion with right-sided sciatica. The pain used to be helped by rest, but is now present more or less all the time and is stopping him from sleeping properly. The most likely diagnosis is: A : mechanical back pain B : ankylosing spondylitis C : myeloma D : osteoporosis E : osteoarthritis. Comment : The history is typical of a patient with mechanical back pain, best treated by encouraging mobilization, simple analgaesia, a graded rehabilitative exercise programme and treatment of depression (if present). ‘Red flag’ symptoms, requiring urgent investigation to exclude sinister pathology, include: · age >55 or <18 years · progressive pain · night pain · systemic symptoms · progressive neurological deficit · past history of malignancy or immunosuppression · recent trauma. A : 41. A 50-year-old man gives an 8-month history of episodic, painful soft tissue swellings involving his hands, eyes and lips. There is no temporal relationship to food. Which of the following tests is the most useful? A : Skin prick tests to various food allergens B : Complement C : Full blood count (FBC) D : Glucose tolerance test E : Urinary 5-hydroxyindoleacetic acid. Comment : The most likely diagnosis in this patient is chronic idiopathic angioedema, However, the rarer C1-inh deficiency has to be considered and would be excluded by a normal C4 level. The lack of any temporal relationship of his symptoms to the ingestion of any food excludes an allergic cause, which only rarely presents as chronic angioedema. B : 42. An 80-year-old woman is admitted having tripped over a mat. Physical examination is apparently unremarkable. She is unable to weight bear and the Orthopaedic team have discharged her with a normal pelvic radiograph. She has been transferred to a medical ward as she is still not able to walk. What investigation(s) would you consider that would most likely provide a definitive diagnosis? A : Repeat standard pelvic radiograph and check a chemical profile. B : Bone scan or Magnetic Resonance Imaging (MRI). C : Dexa (dual energy radiograph absorptiometry) scan. D : Computerised Tomogram (CT) scan. E : Myeloma screen. Comment:Diagnosis of hip fractures may not always be evident on a plain radiograph.An anteroposterior view obtained with the hip internally rotated 15 to 20 degrees will provide an optimal image and may reveal a fracture not evident on on the standard ap view. If there is a high index of suspicion of a femoral fracture technetium-99 m bone scanning or magnetic resonance imaging is appropiate and will show a femoral fracture not evident on a plain radiograph. B : 43. The commonest cause of anaemia in rheumatoid arthritis (RA) is: A : anaemia of chronic disorder. B : non-steroidal anti-inflammatory drug (NSAID) induced iron deficiency anaemia. C : B12-deficiency anaemia due to associated pernicious anaemia. D : autoimmune haemolytic anaemia. E : bone marrow hypoplasia. Comment : All the types of anaemia mentioned are recognised causes of RA-associated anaemia. However, 'anaemia of RA' is a particular example of anaemia of chronic disorders, and is the commonest type of anaemia in RA. The haemoglobin levels are usually around 10 g/dL, and tend to vary inversely with the erythrocyte sedimentation rate (ESR) and other markers of disease activity. A :

Page 218: Masterclass Book Part 2

MOHAMMED IS-HAG 217

44. Which one of the following musculoskeletal disorders is NOT associated with diabetes mellitus? A : Dupuytren's contracture B : Chondrocalcinosis C : Scleroderma D : Carpal tunnel syndrome E : Frozen shoulder. Comment :Patients with either type 1 or type 2 diabetes may eventually have palmar flexor tendinitis, leading to Dupuytren's contracture (i.e. fingers locked in flexion). Characteristically, only one or two fingers (usually the middle and ring fingers) are affected. Diabetic cheiropathy, which is another form of flexor tendinitis, occurs primarily in patients with long-standing, severe type 1 diabetes. Affected patients have generalized puffiness of the hands resembling scleroderma and are unable to place their palms together.Patients with severe diabetes sometimes have scleredema, which is a thickened, indurated infiltrative skin disease that resembles scleroderma. However, it occurs mostly on the upper back, unlike scleroderma, which occurs on the extremities. Scleredema is not associated with either Raynaud's phenomenon or antinuclear antibodies,and it is best treated by rigid control of hyperglycemia . For reasons unknown, diabetic patients are also prone to carpal tunnel syndrome as well as to chondrocalcinosis (i.e. calcium pyrophosphate dihydrate deposition disease) and its accompanying clinical illness, pseudogout. Frozen shoulder (i.e. adhesive capsulitis) also may occur more frequently in diabetic patients than in the nondiabetic population; however, the syndrome is very common in the general population. The mechanisms of the associations of these syndromes with diabetes are not precisely known. C : 45. A 68-year old man presented with sudden severe pain and swelling in the left knee. Synovial fluid analysis shows abundant calcium pyrophosphate dihydrate (CPPD) crystals. Which of the following tests is NOT appropriate for further assessment of this patient illness? A : Creatinine kinase B : Serum calcium C : Thyroid function test D : Serum ferritin level E : Hb AIc. Comment : Pseudogout describes acute attacks of CPPD crystal-induced synovitis which clinically resembles urate gout. However, the majority of individuals with CPPD crystal deposition never experience such episodes. Chondrocalcinosis is the term used to describe the calcium-containing deposits that are found in cartilage and which are usually visible on joint radiographs. A variety of metabolic and endocrine disorders are associated with CPPD crystal deposition, which include: · diabetes mellitus, · hemochromatosis, · Wilson's disease, · hypothyroidism, · hyperparathyroidism, · hypomagnesemia, · hypophosphatasia. A : 46. Which one of the following antibodies, when present in high titre, is NOT regarded as diagnostic of the disease indicated? A : Anti-Sm antibodies in systemic lupus erythematosus (SLE) B : Ads-DNA antibodies in systemic lupus erythematosus (SLE) C : Anti-centromere antibodies in limited systemic sclerosis D : Rheumatoid factor in rheumatoid arthritis (RA) E : Anti-Scl 70 antibodies in diffuse systemic sclerosis. Comment : Antinuclear antibody (ANA) production in SLE is highly diverse, involving antibodies occurring only in patients with SLE as well as those with a wider distribution among connective tissue diseases (CTD). Among these ANA, two have the status as diagnostic markers: anti-DNA and anti-Sm. Anti-DNA are directed to determinants on single stranded (ss) and double stranded (ds) DNA, whereas anti-Sm antibodies bind protein components of an RNA-protein complex, termed snRNP. Although both antibodies are diagnostic of SLE, they occur independently in patients and display a distinct relation to disease activity. Anti-centromere (kinetochore) antibodies are directed against components of the mitotic spindle apparatus. Their presence correlates with Raynaud's phenomenon, oesophageal dysmotility, sclerodactyly, telangiectasias and limited cutaneous involvement (CREST syndrome). They are specific for limited systemic sclerosis.Anti-topoisomerase I (Scl-70) antibodies are associated with generalized skin involvement among patients with diffuse systemic sclerosis SSc, pulmonary fibrosis, cardiac involvement and digital pitting scars. They are regarded as diagnostic for diffuse systemic sclerosis. Rheumatoid factor is neither specific nor sensitive test for RA. It is positive in 50-70% of cases. False positive tests are encountered in many other conditions such as other inflammatory arthritis, connective tissue disease, chronic infections and even in normal elderly people. D :

Page 219: Masterclass Book Part 2

MOHAMMED IS-HAG 218

47. A 20-year-old man presents to casualty with a short history of malaise. Over the last day he has noticed a few bruises on his limbs. Blood tests reveals a white cell count (WCC) of 34 x 109/l, platelet count of 25 x 109/l, Hb of 7.8 g/dl, prothrombin time (PT) of 25 and activated partial thromboplastin time (APPT) of 42, fibrinogen of 0.5g/l. The blood film is shown (see image). What is the most likely diagnosis? A : Acute lymphoblastic leukaemia B : Acute myeloid leukaemia FAb type M5 C : Acute myeloid leukaemia FAb type M3 D : Chronic myeloid leukaemia E : Haemophilia A. Comment : The differential diagnosis of the blood count above is very broad. However, in the context of deranged clotting tests (suggestive of a disseminated intravascular coagulation (DIC)) AML M3 is the most likely of the options given. In addition to chemotherapy, patients with AML M3 should be treated aggressively with platelet/ clotting factor support. They are also treated with ATRA (all-trans-retinoic-acid), as the characteristic chromosomal abnormality renders them susceptible to this drug. C : 48. A 25-year-old man with two episodes of meningococcal meningitis due to unusual serotypes has haemolytic complement activity performed to assess the integrity of his classical (CH 100)and alternate pathways (AP100). He has no family history of male relatives with infections . The most likely abnormality on investigation of his complement pathways is likely to be: A : Absent classical and alternate pathway activity B : Normal classical pathway but absent alternate pathway activity C : Normal alternate pathway but absent classical pathway activity D : Normal classical and alternate pathway activity E : Marked increase in classical and alternate pathway activity. Comment : Recurrent meningococcal meningitis due to unusual serotypes is highly suggestive of primary complement deficiency. The most likely site of the complement deficiency is likely to be the terminal complement complex (C5 -C9) which would result in absent classical as well as alternate pathway activity, because the terminal complement complex is common to both these pathways. The absence of a family history of affected male relatives makes Properdin deficiency unlikely. A : 49. A 20-year-old woman is rejected as a blood donor on account of Fe deficiency anaemia. Which one of the following immunological investigations is most likely to be helpful in explaining her Fe deficiency? A : Anti-nuclear antibody B : Anti-neutrophil cytoplasmic antibody C : Anti-smooth muscle antibody D : Anti-glutamic acid decarboxylase antibody E : Anti-endomysial antibody. Comment : Coeliac disease is a common cause of iron deficiency in both general and hospital practice and should be excluded in this patient. IgA endomysial antibodies are highly specific markers of coeliac disease (specificity 95-100%, sensitivity 70-90%, positive predictive value 95-100%). IgA endomysial antibodies are directed against tissue transglutaminase, the recently described autoantigen in coeliac disease. E : 50. A 77-year-old man presents with persistent head ache and progressive deafness. On examination the patient has frontal bossing of the forehead and conductive deafness, more severe in the right ear. His serum alkaline phosphatase is significantly raised at 870 u/L. Which of the following statements is most accurate about this disease? A : It usually affects a single bone. B : The skull is the most commonly affected bone. C : Bone pain is the most common presenting feature. D : Hearing loss is often due to involvement of the middle ear ossicles leading to conductive

deafness. E : Bone pain is typically increased with rest and on weight bearing. Comment : Paget's disease of bone is a focal disorder of bone remodelling characterized by an increase in the number and size of osteoclasts in affected skeletal sites while the rest of the skeleton is spared. Paget’s disease most commonly involves the axial skeleton, the pelvis being the most common, but it can affect any area. In the majority of patients, the disease affects at least two bones, but in one third of patients only one bone is affected. In the skull, the 8th nerve can be compressed, resulting in hearing loss. This is one of the more common complaints, being present in 37% of respondents in a recent survey of 2000 patients with Paget's disease . Other causes of hearing loss include pagetic involvement of the middle ear ossicles, which dampens the motion of these ossicles. Unlike osteoarthritis, pagetic bone pain usually increases with rest, on weight bearing, when the limbs are warmed, and at night. An estimated 70% of patients who have Paget’s disease have no symptoms. The diagnosis is typically found incidentally on radiographs and laboratory investigations. E :

Page 220: Masterclass Book Part 2

MOHAMMED IS-HAG 219

51. A 68-year-old woman complained of progressive pain in her left hip after slipping on ice and sustaining a sub-capital fracture of the left femoral neck hip. Following recovery from a hemiarthroplasty, bone mineral density (BMD) measurement of the opposite femur confirmed the diagnosis of osteoporosis with a T score of -2.67. She did not wish to take oestrogen replacement therapy and was given alendronate 70 mg weekly. Which of the following statements is true? A : Normally the peak bone mass in a woman is achieved just before the menopause. B : Plain radiographs are very sensitive in identifying minor reduction in bone mineral density and

hence a very useful screening test. C : The T score compares a patient’s BMD with the mean value for young, healthy adults of the same

sex. D : A high serum phosphate level may be suggestive of chronic alcoholism. E : Bisphosphonates such as alendronate act by stimulating bone formation. Comment : Bone mass increases during childhood and adolescence and peaks between the ages of 20 and 30 years. Peak bone mass is influenced by age, sex, genetic factors, hormonal status, exercise, and calcium intake. As women in general have a lower peak bone mass than men, it is understandable why osteoporosis is predominantly seem in women. Plain radiographs are not sensitive enough to diagnose osteoporosis. Results of bone mineral density (BMD) tests are typically reported as T scores and Z scores. · The T score compares a patient's BMD with the mean value for young, healthy adults of the same sex. · The Z score compares a patient's BMD with the mean value for persons of the same age and sex. Both scores are expressed in terms of standard deviations from the mean. The World Health Organization has defined osteoporosis as a BMD of at least 2.5 standard deviations (SD) below the mean value in young normal adults (i.e. T<=-2.5). Chronic alcoholism is another important cause of low serum phosphate level, in which case the liver enzymes will be elevated. All bisphosphonates act similarly on bone in binding permanently to mineralized bone surfaces and inhibiting osteoclastic activity. Thus, they inhibit bone resorption and less bone is degraded during the remodelling cycle. They do not stimulate bone formation. C : 52. A 29-year-old woman has a 3-year history of arthralgia and Raynaud's phenomenon. A year ago she had a miscarriage at 29 weeks, complicated by a deep vein thrombosis. Investigations show:Hb11.2Wcc 4.3 Platelets 145,ANA positive1/160,DNAnegative,ENA Ro positive, Anticardiolipin antibodies present at moderate titre, lupus anticoagulant present. Which statement is true? A : She has primary antiphospholipid syndrome. B : She should be anticoagulated for a further 6 months. C : She has active lupus. D : A high C reaction protein (CRP) and erythrocyte sedimentation rate (ESR) would be consistent

with active lupus. E : Hydroxychloroquine may improve her arthralgia. Comment : This history is strongly suggestive of systemic lupus erythematosus (SLE) with secondary antiphospholipid syndrome. Foetal loss is common due to placental infarction and insufficiency. She is at risk of further thrombosis and should have lifelong anticoagulation. If she is planning further pregnancies she will need specialist referral for intensive monitoring. Active lupus is associated with high ESR and DNA antibodies, with reduced complement (especially C4) levels. High CRP suggests the possibility of infection.Hydroxychloroquine is effective for joint and cutaneous manifestations of SLE. E: 53. A 19-year-old woman collapses in a restaurant, shortly after beginning her meal. Which of the following statements is true? A : Anaphylaxis is not the cause if neither stridor nor wheeze is audible. B : If pallor and hypotension are present: anaphylaxis is the likely diagnosis. C : If she has facial swelling and wheeze: anaphylaxis is likely even if there is no hypotension. D : A history of chronic urticaria implies that she is allergic to something. E : Scrombrotoxin poisoning is a possibility and will not respond to intramuscular epinephrine. Comme:Anaphylaxis is caused by histamine release in response to contact with an allergen. In sensitized people, specific IgE attached to mast cells binds to ingested or inhaled allergen, causing the mast cell to degranulate, releasing histamine. If sufficient histamine is released, anaphylaxis occurs. The symptoms of anaphylaxis are facial and laryngeal oedema, wheeze and hypotension.Uriticaria and flushing may also occur. Not all features are present in every case. Acute urticaria in this situation would be suggestive of anaphylaxis but chronic urticaria is usually non-allergic in nature.Pallor and hypotension are features of a simple faint. Patients have a slow weak pulse. In situation B this is the most likely diagnosis unless additional features of anaphylaxis are present.Scrombrotoxin poisoning occurs when fishes such as mackerel or tuna are spoiled, when they may produce large amounts of histamine. Unlike anaphylaxis, where symptoms are usually fairly immediate, signs of scrombrotoxin poisoning occur at least an hour after ingestion of the suspect food. Symptoms would be expected to respond to epinephrine. C :

Page 221: Masterclass Book Part 2

MOHAMMED IS-HAG 220

54.

PLATE 38 A 56-year-old patient presented with 6-month history of excessive tiredness, stiffness and pain across the upper and lower limbs. On examination he has proximal muscle weakness. The creatinine kinase was 10 times the upper limits of normal. The skin lesions shown in the image are most likely to be: A : garrods pads B : tendon xanthoma C : Gottron's papules D : rheumatoid nodules E : gouty tophi. Comment : Gottron's papules, violaceous papules overlying the dorsal interphalangeal or metacarpophalangeal areas, elbow or knee joints, occur in approximately 70% of patients with dermatomyositis. Garrods fatty pads (knuckle pad syndrome) is a relatively common and harmless condition characterized by discrete fibromatous skin pads over the dorsum of the interphalangeal joints. C : 55. A 30-year-old woman presents with several attacks of pain and swelling affecting the metacarpophalangeal and proximal interphalangeal joints over the past few months, with morning stiffness and fatigue. What are the two most likely diagnoses? A : Ankylosing spondylitis B : Pseudogout C : Systemic lupus erythematosus (SLE) D : Primary generalized osteoarthritis E : Reactive arthritis F : Gout G : Rheumatoid arthritis H :Psoriatic arthropathy I : Systemic sclerosis J : Rheumatic fever. Comment : The most likely cause of these symptoms is rheumatoid arthritis, but SLE requires careful consideration. Rheumatoid arthritis can present as an acute monoarthritis, an acute polyarthritis, a subacute insidious polyarthritis, or with a polymyalgic presentation (particularly in the elderly, when it needs to be differentiated from polymyalgia rheumatica). Remember that acute synovitis in early rheumatoid arthritis is associated with very little fixed joint deformity and no extra-articular features. This contrasts with the deforming arthritis that is characteristic of chronic disease. C :G:

Page 222: Masterclass Book Part 2

MOHAMMED IS-HAG 221

56.

PLATE 39 A 75-year-old retired farmer has been seen in the outpatient clinic. He has been generally unwell for the last month with fever and weight loss, examination reveals this rash (see image). While in the clinic he starts to cough up blood and becomes acutely breathless. He is admitted and initial investigations reveal a serum creatinine of 170 micromol/l, a positive myeloperoxidase (MPO) ELISA, negative proteinase 3 (PR3) ELISA, negative GBM ELISA, a negative ANA and normal complement levels. Red cell casts are seen on microscopy of his urine. Which two of the following diagnoses are most compatible with the clinical picture? A : Sjogren’s syndrome B : Goodpasture’s disease C : Lupus D : Henoch schonlein purpura E : Cryoglobulinaemic vasculitis F : Polyarteritis nodosa G : Anti-phospholipid syndrome H : Wegener's granulomatosis I : Microscopic polyangiitis J : Takayasu’s arteritis. Comment : This patient has presented with the pulmonary-renal syndrome, which is most commonly caused by the small vessel vasculitides (Wegener’s granulomatosis and microscopic polyangiitis (MP)) although Goodpasture’s disease, lupus and cryoglobulinaemic vasculitis should also be included in the differential diagnosis. ANCA positivity in this patient group has a positive predictive value >90% for diagnosing small vessel vasculitis. Although specificity to MPO is more commonly associated with microscopic polyangiitis, 25% of patients with Wegner’s will also have a positive MPO. It is therefore not possible to differentiate these two diseases, based on ANCA specificity, nor is it important as immunosuppressive treatment is indicated regardless. A diagnosis of lupus is highly unlikely with a negative ANA and >90% of patients with cryoglobulinaemic vasculitis will have a low C4 as a result of classical pathway activation.H:I: 57. A 36-year-old man presents with a 2-day history of severe pain in the left knee. He has recently returned from a holiday in Spain. Examination shows low grade fever (37.5?C) and marked synovitis of the left knee with a tense effusion. Which are the two most likely diagnoses? A : Lyme disease B : Rheumatoid arthritis C : Pseudogout D : Gout E : Septic arthritis F : Osteosarcoma G : Osteoarthritis H : Reactive arthritis I : Torn medial meniscus J : Spontaneous haemarthrosis. Comment : Reactive arthritis, consequent upon either genital chlamydia trachomatis or enteric infection, is the most likely diagnosis. He is a little younger than many patients presenting with gout, but this is not a rare diagnosis in this age group, particularly if alcohol intake is high. The important diagnosis to exclude is septic arthritis, but this is a rare cause of monoarthritis in this age group. Aspiration of synovial fluid is the key diagnostic tool to differentiate between these diagnoses.D:H:

Page 223: Masterclass Book Part 2

MOHAMMED IS-HAG 222

58.

PLATE22 A 30-year-old woman presents with arthralgia and tiredness. She is found to be hypertensive (160/100 mmHg). The creatinine is 300 micromol/l, there is 0.9 g proteinuria per 24 hours and the kidneys are normal size on ultrasound. Full blood count shows a slightly reduced platelet count, mild anaemia and evidence of microangiopathy. The patient's hand is shown (see image). Select one of the following answers: A : High-dose oral or intravenous steroids are the most effective treatment. B : Angiotensin-converting enzyme (ACE) inhibitors should be avoided in this setting. C : The most likely diagnosis is haemolytic–uraemic syndrome. D : Autoantibodies to glomerular basement membrane are likely to be present in the serum. E : Renal biopsy is likely to show arteriolar changes, fibrin thrombi and fibrinoid necrosis. Comment : The cutaneous appearances and renal presentation are typical of scleroderma with a renal crisis. In patients who develop a renal crisis this often occurs early in the course of the disease. There is usually relatively minor proteinuria, hypertension and evidence of microangiopathy. Typically the renal biopsy will show arteriolar changes and fibrinoid necrosis, and the best current treatment is with ACE-inhibitors. E : 59. A 32-year old lady with systemic lupus erythematous (SLE) is seen in clinic. She had a flare of her disease 3 months ago, with arthralgia and rash, but has been asymptomatic since a short course of moderate dose prednisolone. She currently takes hydroxychloroquine 200 mg bd and prednisolone 8mg od. Clinical examination is unremarkable. Her ESR is 60 mm/h and her CRP is 8 mg/dl. Which two of the following investigations could explain her discordant inflammatory markers? A : Serum immunoglobulins B : Chest radiograph C : Plasma viscosity D : Urinalysis E : Complement levels F : Lipids G : Urea H : ANA I : Anti-DNA J : ANCA. Comment : The C-reactive protein is a more sensitive marker of inflammation and infection than the ESR. The normal CRP in this lady fits with the clinical picture of disease inactivity. Therefore, the raised ESR must be secondary to a cause other than inflammation or infection. Such causes include hyperlipidaemia, anaemia and hypergammaglobulinaemia, all of which frequently occur in SLE. A : F:

Page 224: Masterclass Book Part 2

MOHAMMED IS-HAG 223

60. Contraindications to receiving intravenous immunoglobulin therapy include: A : acute febrile illness B : chronic supporative chest disease C : myeloma D : hepatitis C infection E : mild renal impairment Comment : Intravenous immunoglobulin (IVIG) replacement is indicated for the prevention of infection in patients with antibody deficiency. This may be primary or secondary, as in some cases of myeloma. Immunological adverse reactions are increased if acute infection is present. IVIG should be deferred for 24-48 hours and antibiotics commenced. With adequate immunoglobulin replacement serious infections and progression to chronic disease such as bronchiectasis is prevented. However, patients often present late and will benefit even in the presence of suppurative lung disease. Hepatitis C outbreaks have been associated with IVIG, although modern production methods minimise this risk. Acute renal failure has been associated with IVIG, particularly in older people with pre-existing renal disease, as may occur in myeloma. IVIG should be used with caution in these people but is not contraindicated. A : An 82-year-old woman is admitted having sustained a hip fracture. Two years ago she sustained a wrist fracture. On examination she has bilateral cataracts and reduced visual acuity. In passing she mentions that she has been on prednisolone 5 mg for 8 years for polymyalgia but takes no other medication and has not seen a doctor for 2 years. Which of the following statements about corticosteroid induced osteoporosis is correct? A : Corticosteroids increase osteoblastic activity. B : Corticosteroids increase bone mass. C : Corticosteroids reduce intestinal calcium absorption. D : Corticosteroids increase circulating sex steroid levels. E : Corticosteroid induced osteoporosis is greatest 12 months after starting treatment. Comment : The pathogenesis of corticosteroid induced osteoporosis is multifactorial: · Corticosteroids reduce osteoblastic activity, and the resulting osteoblast/osteoclast imbalance causes loss of bone. · Corticosteroids reduce intestinal calcium absorption and lower circulating sex steroid levels. · Corticosteroid induced bone loss is fastest in the first 6-12 months of therapy C : 61. A 35-year-old woman recounts a 2-month history of weakness; she is having problems climbing the stairs and rising from a chair. On examination, you observe that she has an erythematous rash on her cheeks. An ANA comes back as strongly positive with a negative ENA and DNA. Which is the most likely diagnosis? A : SLE B : Dermatomyositis C : Mixed connective tissue disease D : Anti-phospholipid syndrome E : Primary Raynaud’s disease. Comment : Her history of proximal muscle weakness and rash is compatible with an inflammatory myositis - either complicating SLE or dermatomyositis. Although she has a positive ANA, her negative ENA and DNA and normal complement make dermatomyositis more likely. Patients with idiopathic inflammatory myositis frequently have a positive ANA without ENA positivity. The ANA is usually directed against other nuclear antigens such as PM-Scl and U1RNP. B : 62. A patient on dialysis presents with a large swollen tender shoulder. Aspiration yields a milky slurry. Plain films show this appearance (see image). What is the most likely underlying cause of this problem? A : Poor control of calcium and phosphate B : Primary hyperparathyroidism C : Osteolytic metastasis D : Myeloma E : Septic arthritis. Comment : This is an example of tumoural calcinosis. Excess calcium and phosphate (which are at best only metastable in plasma) precipitate in either crystalline or amorphous forms in the skin, joints, blood vessels and other places. This ectopic calcification can cause local effects as here, including pain (though often these are painless) and deformity. Treatment is by engendering a negative calcium and phosphate balance, e.g. increased dialysis clearance, stopping positive calcium balance, and recourse to renal transplantation. A :

Page 225: Masterclass Book Part 2

MOHAMMED IS-HAG 224

63.

A 32-year-old woman with systemic lupus erythematosus (SLE) and antiphosphlipid syndrome presented with a three week history of increasing pain in the right groin and anterior thigh associated with excessive fatigue and lethargy. She was limping and right hip movement was limited with reduced flexion and internal rotation. Although afebrile with a temperature of 37°C she was given intravenous antibiotics and kept on her regular maintenance dose of prednisolone at 7.5 mg/day. Magnetic resonance image (T1 weighted axial) of the hips (see image) shows the entire femoral head is reduced in signal on the right, indicating diffuse oedema. It also shows subchondral collapse with flattening of the right femoral head. A serpiginous band of low signal surrounds an area of normal signal. The most likely diagnosis is: A : SLE-associated acute inflammatory arthritis of the right hip B : Septic arthritis of the right hip C : Right psoas abscess D : Avascular necrosis of the right hip E : Thrombosis of the right ilio-femoral vein. Comment : All of the above options are possible causes for this patient's complaint. However, the magnetic resonance image is almost diagnostic of avascular necrosis (AVN). Corticosteroids such as prednisolone are commonly used to treat diseases in which there is inflammation, such as systemic lupus erythematosus. Both steroid therapy and SLE could predispose to avascular necrosis. If septic arthritis is suspected, immediate joint aspiration must be performed in order to obtain cultures to determine the infectious agent. Underlying bone changes are not typically present. Inflammatory arthritis in general is associated with synovial tissue thickening but again lacks significant bone changes. Ilio-femoral vein thrombosis and psoas abscess are not known to be associated with structural damage of the femoral head. In differentiating AVN from non-AVN disease of the femoral head, MR imaging has been reported to have a specifity of 98% and sensitivity of 97%.Mnemonic to remember causes of AVN (taken from Dahnert's Radiology Review Manual), "PLASTIC RAGS": P - Pancreatitis, Pregnancy L - Legg-Perthes Disease, Lupus A - Alcoholism, Atherosclerosis S - Steroids T - Trauma I - Idiopathic (Spontaneous osteonecrosis of knee, Legg-Calve-Perthes Disease, Freiberg Disease), Infection C - Caisson Disease*, Collagen Vascular Disease R - Rheumatoid arthritis, Radiation tx A - Amyloid G - Gaucher's Disease S - Sickle Cell Disease *Caisson Disease: decompression sickness in divers - ask about occupation! Corticosteroid use and alcohol are the most common associations in the UK. D :

Page 226: Masterclass Book Part 2

MOHAMMED IS-HAG 225

64. A 68-year-old lady presents with a 4-day history of anuria. She has a past history of intermittant claudication and is a long-term smoker. Which of the following statements are correct? A : If kidneys are of normal size renal artery occlusion can be excluded. B : If there is evidence of renal asymmetry and first investigation should be a biospy of the smaller

kidney. C : An IVU would be the investigation of choice. D : One should consider urgent arteriography and revascularisation. E : Weakly positive pANCA serology would establish a diagnosis of microscopic polyangitis. Comment : Absolute anuria would be a rare presentation of a rapidly progressive glomerulonephritis. A weakly positive pANCA is a relatively common finding and in many instances is a false positive result. One would need histology to confirm the diagnosis in these circumstances. Renal artery occlusion can present in this way and the presence of two normal size kidneys does to exclude the diagnosis. An IVU is not helpful in these instances, as non-opacification of the kidneys would occur in the presence of poor renal function irrespective of the cause. When there is renal asymmetry the larger of the two kidneys should be biopsied as the diagnostic yield is likely to be greater. Arteriography and revascularisation should be considered as this may well salvage the situation is carried out early enough. D : 65. A 20-year-old woman presented with sudden onset of swelling of the lips and tongue. She also had abdominal pain and vomiting. Her mother confirmed that her daughter had similar attacks over the years and even as a child. A brother and older sister have the same disorder. Which of the following statements about this disease is accurate? A : It has sex-linked inheritance. B : Animal allergen is often identified in the house C : Serum C4 levels are often low. D : Antinuclear antibodies (ANA) is often positive. E : Raised IgE helps differentiate it from other immune disorders. Comment : Hereditary angioneurotic oedema is an autosomal dominantly inherited condition caused by a deficiency of C1 esterase inhibitor. This results in intermittent episodes of spontaneous complement activation. Clinically the patient suffers oedema of the skin and mucosal surfaces. Fatalities may occur if the airway is compromised. C4 levels are typically low during an attack; they may be normal in between attacks.Acquired angioedema (AAE) is angioedema associated with allergic reactions, which is often associated with urticaria. Approximately 94% of cases of angioedema are drug-induced. Most are patients taking angiotensin-converting enzyme (ACE) inhibitors. Insect stings, and foods are other predisposing factors. C : 66. A 72-year-old man suffers a stroke causing left sided weakness. He has very little voluntary movement of the left arm and needs assistance to transfer. Whilst on the rehabilitation ward he complains of significant pain around his left shoulder. Which of the following statements is correct? A : Poor handling and positioning of the left arm by staff are unlikely to have contributed to the pain B : Shoulder pain causes distress but does not affect outcome C : A radiograph of the shoulder is not necessary D : Treatment with simple analgesics is a sensible initial approach E : Intra-articular steroids are likely to exacerbate the problem. Comment : Shoulder pain is a common complication of stroke. The aetiology is thought to be multifactorial and includes poor handling of the affected arm causing trauma to the shoulder structures, glenohumeral subluxation, abnormal tone (spasticity and flaccidity) and pain due to the stroke itself (central post-stroke pain). It is associated with prolonged hospital stay and poor recovery of arm function. A radiograph should be performed to exclude fracture or dislocation. Treatment usually starts with simple analgesics or non-steroidals. Physiotherapists may treat subluxation with strapping.If the pain persists intra-articular steroid injections and TENS may help. D : 67. In which one of the following conditions is HLA B27 rarely identified? A : juvenile chronic arthritis B : Reiter’s syndrome C : isolated uveitis D : Behcet’s diseaseE : ulcerative colitis. Comment : HLA B27 is associated with seronegative spondyloathropathy including: * ankylosing spondylitis * Reiter’s syndrome * spondyloarthropathy of inflammatory bowel disease * psoriasis * juvenile chronic arthritis (juvenile spondyloarthropathy). HLA-B27 is found in approximately 50% of patients who have uveitis in the absence of rheumatic disease. Behcet’s disease is associated with high prevalence of HLA B5. D :

Page 227: Masterclass Book Part 2

MOHAMMED IS-HAG 226

68. A 30-year-old woman presents to her general practitioner with a 3-month history of fatigue. She has read about lupus on the world wide web and asks her GP to carry out appropriate blood test(s) to rule out lupus. Which single blood test is best suited to screen for lupus in this situation? A : Antinuclear antibody B : Serum Immunoglobulin levels C : Anti-cardiolipin antibodies D : Anti-DNA antibodies E : Anti-ENA antibodies. Comment : ANA testing carried out on Hep-2 cells offers the highest degree of sensitivity for lupus. Conversely, a negative ANA in this situation effectively rules out systemic lupus erythematosus. The other tests although useful do not, if normal or negative, enable you to rule out lupus. A : 69. A young patient is attending the rheumatology monitoring clinic. She develops leucopaenia. Which of the following is unlikely to cause this abnormality? A : Azathioprine B : Methotrexate C : Systemic lupus erythematosus (SLE) D : Cyclosporin AE : Etanercept. Comment : Patients taking azathioprine, methotrexate and etanercept require regular monitoring of the full blood count to exclude marrow suppression. Patients with SLE often have a low white cell count. D : 70. A 34-year-old woman presents to your clinic complaining of cold hands particularly in the winter months. On examination, she has cold dusky hands and a petechial rash. Investigations are as follows: Hb 10.9 g/dL; Wbc 4.2 109/L; Platelets 407 x 109/L Urea and electrolytes: normal; Liver function tests: normal; Albumin 36 g/L; Globulin 90 g/L; Protein electrophoresis: polyclonal increase in gammaglobulins. Antinuclear antibodies present (1/160, speckled pattern) Complement C3 0.79 (NR 0.75-1.25), C4 <0.04 (NR 0.14-0.6) Which of the following statements is true? A : Active systemic lupus erythematous (SLE) is unlikely if DNA antibodies are present. B : A blood sample sent to the lab on ice may show cryoglobulins. C : Sjorgren’s syndrome is unlikely if rheumatoid factor is present. D : Sjorgren’s syndrome is likely if Ro and La extractable nuclear antigens are present. E : Hepatitis C is unlikely in this case. Comment : The symptoms and signs and low C4 are suggestive of cryoglobulinaemia. Sludging of proteins at reduced temperatures (for example hands on a cold day) can cause ischaemia and sometimes vasculitis, particularly of skin or kidneys. Cryoglobulinaemia is commonly associated with hepatitis C or connective tissue disease, such as Sjorgren's syndrome. The positive ANA and high globulins suggest Sjorgren's but could also be associated with chronic infection, such as hepatitis C. Since they precipitate at low temperatures, cryoglobulins should always be transported to the lab at 37?C. Failure to do this will result in a false negative result as the cryos will precipitate and be removed with the clot. D : 71. A 65-year-old man is admitted as an emergency with a very hot, swollen left knee. On examination he is unwell and pyrexial. He has marked loss of range of movement, secondary to pain. You have aspirated his knee and have sent the purulent-looking fluid for microscopy. Which of the following is true? A : He is likely to have acquired this infection after an arthroscopy or arthrocentesis. B : Seventy to eighty percent of cases will have an accompanying bacteraemia. C : Results of the culture should be awaited before commencing antibiotics. D : Antibiotics should cover beta-haemolytic streptococcus and staphylococcus infections. E : A plain x-ray of the knee will confirm the diagnosis. Comment : This is likely to be septic arthritis. Obviously, examination of the synovial fluid will help to exclude differentials such as gout and pseudogout. The most likely organisms are beta-haemolytic streptococci (20%) and staphlococci (70%). Antibiotics should be started empirically to cover these if the clinical suspicion is high. Ideally these should be intravenous for 2 weeks and then oral for 4 weeks. Arthroscopy is a risk factor for septic arthritis, but is rare. Fifty percent of cases will have an associated bacteraemia. Early x-rays are almost always normal. D :

Page 228: Masterclass Book Part 2

MOHAMMED IS-HAG 227

72. Lyme disease is one of the most common vector-borne diseases in the western hemisphere. It is caused by the spirochete, Borrelia burgdorferi. Which of the following statements regarding Lyme disease is true? A : Vaccination using recombinant antigen is protective in 70% of cases B : Arthritis is the presenting feature in the majority of cases C : Unlike other spirochete infections Borrelia burgdorferi is usually resistant to penicillins D : Erythema nodosum is the typical lesion seen at the site of the tick bite E : The causative micro-organism can be readily identified on Gram stains of serum. Comment : Lyme disease is a tick-borne inflammatory disorder caused by a spirochete, Borrelia burgdorferi. Its clinical hallmark is an early expanding skin lesion, erythema chronicum migrans, which may be followed weeks to months later by neurologic, cardiac, or joint abnormalities. All of the three spirochetes pathogenic for man - Treponemes, Leptospires and Borrelia - are sensitive to penicillins. Lyme disease develops after inoculation of skin with B. burgdorferi from an infected tick. Lyme disease typically progresses through a series of clinical stages from early, localized, cutaneous involvement (erythema chronicum migrans) through an early disseminated stage characterized by multiple secondary skin lesions, headache, which may reflect neurologic spread, and musculoskeletal, flu-like symptoms (stage 1); through a period weeks to months later during which cardiac and acute neurologic manifestations predominate (stage 2); to a third, chronic stage involving primarily joints and nervous system, arbitrarily defined as greater than one year (stage 3). Culture of B. burgdorferi from patients permits definitive diagnosis, but is rarely successful except from skin. Similarly, spirochetes are not often seen by direct examination of blood, plasma, plasma pellets, or skin transudate specimens. Determination of specific antibody titers is the most helpful diagnostic test for Lyme disease in routine clinical settings. A : 73. Which one of the following tests is the most specific method of diagnosing Paget's disease? A : Urinary hydroxyproline test B : Pyridoline urinary test C : Bone scan D : Total serum alkaline phosphatase level E : Plain radiographs. Comment : Diagnosis of Paget's disease may be suspected based on the symptoms, but radiographs are the most specific diagnostic test. Radiographs include both lytic (early) and sclerotic findings. Many patients are diagnosed incidentally in the asymptomatic phase by plain radiographs that show localized enlargement of bone. These radiographs often have a high specificity because of their classic nature, but a low sensitivity. Bone scans can be used to increase the sensitivity in patients suspected of having Paget's disease; however, the bone scan is less specific and should be interpreted cautiously. There are many biochemical markers for Paget's disease, but the two most important are total serum alkaline phosphatase and urinary pyridinoline. These markers may be normal in patients with the monostotic form of Paget's disease (15% of patients) therefore, serum bone-specific alkaline phosphatase measurements may be useful. Urinary hydroxyproline is no longer considered an accurate marker of activity or extent of the disease. E : 74. A 35-year-old woman presents to her doctor with a painful and swollen left leg. Ultrasound examination confirms a suspected deep venous thrombosis. A prolonged APTT is noted and anticardiolipin antibodies are detected. Which of the following is correct? A : She has primary antiphospholipid syndrome. B : She requires life-long anticoagulation. C : She requires a repeat cardiolipin antibodies after 6 weeks. D : She should be treated with life-long aspirin. E : Her prolonged APTT will correct with the addition of normal plasma. Comment : The history of venous thrombosis and the detection of cardiolipin antibodies is consistent with the anti-phospholipid syndrome. However, anticardiolipin antibodies are not specific and, therefore, have to be repeated after a 6-week interval. See Medical Masterclass, Rheumatology and Clinical Immunology module, Clinical Presentation 1.11, for further discussion. C : 75. A patient with psoriatic arthritis has active joints and troublesome plaque psoriasis. Which of the following will improve both the joint and skin problems? A : Sulphasalazine B : Hydroxychloroquine C : Gold D : MethotrexateE : Penicillamine. Comment : Hydroxychloroquine can exacerbate psoriasis. Sulphasalazine tends to only improve joint symptoms and not improve the psoriasis. Gold and penicillamine are not commonly used to treat this condition. D :

Page 229: Masterclass Book Part 2

MOHAMMED IS-HAG 228

76. A 50-year-old woman presents to casualty with a 2-year history of recurrent angioedema. Her clinical history fails to reveal an underlying trigger. Her serum complement profile is as follows: serum C3 1.2 g/l (ref range 0.75-1.65), serum C4 <0.02 g/l (ref range 0.2-0.6). The most likely diagnosis is: A : food allergy B : drug allergy C : Idiopathic angioedema D : C1 inhibitor deficiency E : venom allergy. Comment : Uncontrolled activation of the classical complement pathway leading to a marked reduction in serum C4 levels is a hallmark of C1 inhibitor deficiency. In the absence of a family history, it is possible that this represents a new mutation. Alternatively, the possibility of acquired C1 inhibitor deficiency due to lymphoproliferative disease should be considered. D : 77. A 28-year-old man is has recently been discharged from hospital after treatment for pneumococcal pneumonia. He has had repeated courses of antibiotics for sinus, ear and lower respiratory tract infections, and had sinus surgery the previous year. He is a life long non-smoker and is not on medication. His blood count prior to discharge was normal. In the absence of further clues in the history or examination, which single blood test is the most important? A : HIV antibody test B : Pneumococcal antibodies C : Immunoglobulin levels D : Liver function tests E : IgG subclass levels. Hypogammaglobulinaemia is associated with recurrent bacterial infections, most commonly of the respiratory tract. Delay of several years prior to diagnosis is usual, with associated morbidity. Patients with low immunoglobulin levels and recurrent infections should be treated with immunoglobulin replacement. More minor antibody defects, such as IgG subclass or specific antibody (to pneumococcus) defects can often be treated with appropriate vaccinations and/ or prophylactic antibiotics. C : 78. Which of the following statements is not accurate with regard to the risk of gastrointestinal complications secondary to non-steroidal anti-inflammatory drug (NSAID) therapy? A : Eradication of Helicobacter pylori in long-term NSAID-users does not necessarily decrease ulcer

complications. B : Patients will develop tolerance to the gastrointestinal side effects of NSAIDs over time. C : Concomitant use of oral corticosteroids, even in low doses, would increase the risk for ulcer complications. D : Advanced age is an independent risk factor. E : Almost 80% of patients hospitalized with serious NSAID-induced complications had no previous

gastrointestinal symptoms. Comment : Risk of gastrointestinal complications secondary to NSAIDs is not uniformly distributed in the population. Patient characteristics that indicate increased risk are as follows: * advanced age * history of ulcer * concomitant use of corticosteroids * higher doses of NSAIDs, including use of more than one NSAID concomitantly * concomitant administration of anticoagulants * serious systemic disorder (defined as comorbid nonrheumatic disease requiring drug therapy). Helicobacter pylori infection, cigarette smoking, and alcohol consumption are possible additional risk factors for NSAID ulcers. The relationship between H. pylori and NSAID-associated ulcers is controversial. Both H. pylori infection and NSAID use are independent risk factors for ulcer complications. However, the combination of the two factors is neither synergistic nor additive with respect to risk. Eradication of H. pylori in long-term NSAID users does not necessarily decrease ulcer complications.The risk of NSAID-induced gastrointestinal bleeding remains constant over time. Therefore, patients do not develop tolerance to the gastrointestinal side effects of NSAIDs. Most of the risk factors mentioned earlier are recognisable and unambiguous. However, the gastrointestinal risks from oral corticosteroids are dose- and context-specific. When used alone and in low doses (equivalent to 10 mg or less of prednisone daily), oral corticosteroids do not increase the risk for ulcer complications. However, when prescribed with NSAIDs concomitantly, oral corticosteroids increase the risk for upper gastrointestinal hemorrhage 10-fold. According to survey data, however, 81% of patients hospitalized with serious NSAID-induced complications had no previous gastrointestinal symptoms. Because mild gastrointestinal symptoms (e.g. mild nausea, dyspepsia, abdominal pain) do not necessarily precede severe complications of NSAIDs, strategies to prevent life-threatening events should not rely on warning signs or symptoms to prompt a patient to seek medical attention. B :

Page 230: Masterclass Book Part 2

MOHAMMED IS-HAG 229

79. A 30-year-old man has a swollen wrist, low back pain with early morning stiffness lasting 2 hours and mouth ulcers. He does not have a rash. What is the most likely diagnosis? A : Rheumatoid arthritis (RA) B : Psoriatic arthritis C : Ankylosing spondylitis D : Gout E : systemic lupus erythematosus (SLE) Comment : The history is suggestive of a seronegative spondyloarthropathy. RA is classically a peripheral, symmetrical polyarthritis. Gout can affect the wrist but is unusual in a young person. Patients with SLE can experience mouth ulcers but SLE is uncommon in young males. C : 80. A 40-year-old woman suddenly develops severe Raynauds that is troublesome in the summer as well as the winter. She starts to develop swelling of the fingers and feels tired. She also develops reflux oesophagitis and has difficulty swallowing. She has noticed that she has become more breathless. On examination, she has skin thickening affecting her hands, face and trunk. Which of the following is most likely to be positive? A : Ds DNA antibody B : Ro antibody C : Anticentromere antibody D : Jo-1 antibody E : Scl-70 antibody. Comment : This patient is developing diffuse systemic sclerosis. Anticentromere pattern is associated with limited systemic sclerosis. Ro antibody is common in patients with primary Sjogren's syndrome and systemic lupus erythematosus (SLE). Jo-1 may be positive in patients with polymyositis. E : 81. Which one of the following drugs does NOT cause proximal muscle weakness? A : Corticosteroid B : D-penicillamine C : Methotrexate D : Chloroquine E : Colchicine. A long list of drugs can cause myopathic changes. The list includes some drugs commonly used in a rheumatology practice.Glucocorticoid use causes proximal muscle weakness and wasting. Electromyographic changes are minimal and, if present, nonspecific. Biopsy of muscle shows only type II fiber atrophy. This condition may be difficult to diagnose when it develops in patients being treated for inflammatory muscle disease.Direct toxicity Vacuolor myopathy is associated with chloroquine and colchicine therapy. HMG-CoA reductase inhibitors (statins) are also associated with direct muscle toxicity especially when used in combination with other antilipid drugs such as gemfibrozil and clofibrate. Ciclosporin and erythromycin are also noted to enhance muscle toxicity of statins. Zidovudine (anti-HIV drug) causes myopathy through its direct toxic effect on myocyte mitochodria. Toxicity is invariably associated with high creatinine kinase (CK) levels. Withdrawal of the drug should rapidly lead to clinical improvement and normalization of the CK level.ImmuneThe exact mechanism by which many drugs cause myopathy is uncertain. Some myopathies, such as those caused by D-penicillamine, hydralazine, and procainamide, are immune mediated.Indirect toxicityIn addition, any drug or hormone that can raise or lower serum concentrations of sodium, potassium, calcium, phosphorus, or magnesium can induce myopathic symptoms.Muscle weakness and myopathy are not a recognised complication of methotrexate therapy. C : 82. Which one of the following rheumatological disorders is NOT associated with increased incidence of malignancy? A : Dermatomyositis B : Polymyalgia rheumatica C : Sj?gren's syndrome D : Diffuse systemic sclerosis E : Hypertrophic pulmonary osteoarthropathy. Dermatomyositis and, to a lesser degree, polymyositis and inclusion body myositis are associated with a wide range of solid tumors, carcinomas, sarcomas, and lymphomas. The cancer types most commonly associated with dermatomyositis are ovarian, pulmonary, pancreatic, gastric, and colorectal as well as non-Hodgkin's lymphoma. Sj?gren's syndrome has an established association with lymphoproliferative malignant disease.Patients with rheumatoid arthritis have a 10-fold increased incidence of lymphoproliferative disorders; the incidence is increased 33-fold when Sj?gren's syndrome is also present.Primary hypertrophic osteoarthropathy is not associated with other systemic disease, whereas secondary hypertrophic osteoarthropathy is associated with lung cancer(most commonly adenocarcinoma),other tumours (e.g. nasopharyngeal cancer), nonmalignant pulmonary disorders, and right-to-left cardiac shunts. A retrospective cohort analysis of 917 Swedish patients who had progressive systemic sclerosis/scleroderma found an increased incidence of cancer. Lung cancer was the most common type, followed by skin cancer, hepatoma and hematopoietic malignant lesions . Despite reports of cancer in patients with polymyalgia rheumatica (PMR) or giant cell arteritis (GCA), the incidence of cancer and overall life expectancy among patients with GCA and PMR is essentially identical to the general population. B :

Page 231: Masterclass Book Part 2

MOHAMMED IS-HAG 230

1.

PLATE 40 Identify the two abnormalities on this blood film. A : Lymphocytosis B : Blast cells C : Smear cells D : Thrombocytopenia E : Macrophages F : Plasma cells G : Hypochromasia H : Leucopenia I : Neutrophilia J : Thrombocytosis. Comment : Chronic lymphocytic leukemia (CLL) is characterized on the blood film by the presence of numerous, small, mature lymphocytes. These may be disrupted during the preparation of the film resulting in ‘smear’ cells. Co-existing haemolytic anaemia, with spherocytes seen on the blood film, is not an uncommon accompanying feature. A : C : 2. A GP contacts you about one of his patients. The following is the result of the patient's full blood count: Haemoglobin 10.2 g/dl, white cell count 115x10^9/l, platelet count 112 x 10^9/l, neutrophils 1.9x10^9/l, lymphocytes 112x10^9/l. Furthermore, a diagnosis of chronic lymphocytic leukaemia (CLL) has been correctly made by the laboratory. Which of the following statements regarding CLL are incorrect? A : CLL is a disease of the middle aged and elderly. B : Physical examination may be normal in the early stages of the disease. C : Chlorambucil may be used as first line therapy. D : Advanced disease may be associated with hepatosplenomegaly. E : Smear cells are usually seen on the blood film. F : Immunophenotyping is usually undertaken to confirm the diagnosis. G : There is an association with autoimmune haemolytic anaemia. H : There is an associated increased susceptibility to infection. I : In the majority of cases of CLL the malignant lymphocytes arise from the T cell lineage. J : 10% of patients with CLL develop amyloidosis during the course of their illness. Comment : In almost all cases of CLL the malignant lymphocytes are B cells. Amyloidosis is a complication of myeloma. I : J :

Page 232: Masterclass Book Part 2

MOHAMMED IS-HAG 231

3.

PLATE 41 A young man has a full blood count performed. Although the blood counts are normal, the blood film has a number of abnormalities (see image). What abnormality/ disease would NOT cause these appearances? A : Inflammatory bowel disease B : Sickle cell disease C : Iron deficiency D : Coeliac disease E : Lymphoma F : Previous splenectomy G : Liver disease H : Sarcoidosis I : Amyloidosis J : Essential thrombocythaemia. Comment : The prominent abnormalities in the blood film are acanthocytes, Howell-Jolly bodies and target cells. This is the classic appearance in a patient with hyposplenism (due to infiltration, infarction or atrophy). Target cells may be present in liver disease (as may acanthocytes in advanced disease), but Howell-Jolly bodies are not seen.C:G: 4. A man wakes up noticing a generalized rash over his body. A week previously he had developed a 'flu -like' illness. His complete blood count was Hb 12.4g/dl, WBC 8 x 10^9/l and platelets 2 x10^9/l. A bone marrow showed no atypical cells but adequate megakaryocytes and confirmed peripheral consumption. Which of these treatments are options in an actively bleeding patient with Immune thrombocytopenic purpura (ITP)? A : Peripheral vasoconstrictors B : Corticosteroids C : Fresh frozen plasma (FFP) infusions D : Low molecular weight Heparin E : Intravenous immunoglobulin infusion F : Cautery and laser diathermy of bleeding point G : Single platelet transfusion H : Topical fibrin glue I : Hydroxyurea J : Leucocyte infusions. Comment : Immune mediated thrombocytopenia (ITP) is an immunological disorder characterized by premature platelet destruction due to production of cross-reacting antibodies directed against platelet antigens. Destruction is mediated by the spleen and liver. Treatment options in a bleeding patient at presentation are corticosteroids, e.g. Prednisolone at a dose of 1mg/kg body weight. Intravenous immunoglobulins can be used to bring about a rapid rise in the platelet count but the effect is not long-lasting. They block the Fc receptor on the macrophages thus preventing phagocytosis. Platelet infusions can be given in life threatening bleeds but a substantial amount (not a single unit) will be needed to swamp the antibody. Chemotherapeutic agents such as Vincristine, cyclophosphamide and even combination chemtherapy have been tried in refractory cases. Neither FFP or leucocyte infusions have a role in management of ITP and Heparin can exarcebate the bleeding diathesis. B : E :

Page 233: Masterclass Book Part 2

MOHAMMED IS-HAG 232

5. A 19-year-old student is admitted with fever, neck stiffness, photophobia and headache. He has a Glasgow Coma Score of 13/15, a widespread purpuric rash and prolonged bleeding at the sites of venepuncture. A presumptive diagnosis of meningococcal infection is made. Laboratory investigations are in keeping with him having an associated disseminated intravascular coagulation (DIC). Which of the following would you expect in a patient like this? A : Prolonged prothrombin time B : Thrombocytosis C : Raised fibrinogen D : Eosinophilia E : Macrocytosis F : Bleeding into joints G : Positive direct antiglobulin (DAG) test H : Raised C reactive protein (CRP) I : Normal activated partial prothrombin time (APTT) J : Positive dilute Russell's Viper Venom test (DRVVT). Comment : DIC occurs as a result of consumption of components required for normal haemostasis. As a result PT and APTT rise, and platelets and fibrinogen fall. Macrocytosis, eosinophilia and a positive DRVVT do not occur as a result of DIC. Joint bleeding is characteristic of congenital severe haemophilia. CRP is frequently raised as a result of the underlying severe illness precipitating the DIC.A positive DAG test implies autoimmune haemolysis. A : H : 6.

PLATE42 A 76-year-old woman had been unwell for 2 weeks. Headaches, limb aches and fatigue. Hb 10.2 g/dl, platelets 382, WCC 13.2 (neuts 9.1). Her erythrocyte sedimentation rate (ESR) tube is the one on the right. Which are the two most likely diagnoses? A : Iron deficiency anaemia B : Breast cancer C : Rheumatoid arthritis D : Acute myeloid leukaemia E : Polycythaemia F : Polymyalgia rheumatica G : Systemic lupus erythematosis H : Myeloma I : Diabetes mellitus J : Diverticular disease. Comment : The ESR tube on the right shows a very rapid sedimentation rate of >120 mm/hr. This is a feature of polymyalgia or myeloma. The protein excess of myeloma neutralizes the negative charge on the erythrocytes and allows them to get close together and settle rapidly. Anaemia and cancer also raise the ESR but usually in the order of 40 to 90 mm/hr (e.g. tube on the left). Plasma viscosity is more reliable than the ESR as there are no confounding influences from the haemoglobin concentration. F : H :

Page 234: Masterclass Book Part 2

MOHAMMED IS-HAG 233

7. A young woman is being investigated for recurrent fetal loss. The clinical suspicion is of antiphospholipid antibody syndrome. Which of the following DO NOT support this diagnosis? A : Previous DVT B : Previous arterial thrombosis C : A prolonged APPT which corrects on mixing with normal plasma D : A prolonged APPT which does not correct on mixing with normal plasma E : Thrombocytopenia F : Trolonged PT G : Migraines H : Anaemia I : Recurrent skin ulcers J : Joint pathology. Comment : Antiphospholipid antibody syndrome (APAS) represents a spectrum of abnormalities which is multi-system in nature. It may be idiopathic or associated with a number of underlying causes, including inflammatory arthritis, infarctions and medications. Classically the presence of a lupus anticoagulant (antiphospholipid antibody) will prolong the APPT and this will not be corrected by the addition of normal plasma (unlike factor deficiencies). The PT is not affected. C : F : 8. A medical registrar phones for your advice on a 35-year-old woman who has been transferred to his care from another hospital. Her full blood count has shown a raised MCV. A bone marrow aspirate performed at the previous hospital has been reported to show no megaloblastic changes. Which are the two LEAST likely underlying causes of the macrocytosis? A : Liver disease B : Pregnancy C : B12 deficiency D : Hypothyroidism E : Chronic obstructive airways disease F : Folate deficiency G : Scurvy H : Haemorrhage I : Haemolysis J : Atypical pneumonia. Comment : The causes of macrocytosis can be divided into those with a megaloblastic bone marrow and those with a normoblastic bone marrow. Megaloblastic changes are generally related to vitamin B12 or folate deficiency (and syndromes causing this) or to drugs which dierctly damage the DNA. A variety of medical conditions result in a raised MCV with a normal bone marrow. C : F : 9. A 24-year-old male veterinary worker presented with fever, generalised lymphadenopathy and deranged liver function tests AST 892 and a total bilirubin level of 153mmol/L. Which of the following diseases need to be excluded? A : Malaria B : Chronic Myeloid Leukaemia C : Lymme Disease D : Typhoid infection E : Myeloma F : Hepatitis C infection G : Aplastic anaemia H : Toxoplasmosis I : Sickle cell anaemia J : HIV infection K : Myelofibrosis L : Autoimmune Haemolytic anaemia M : Cyclical Neutropenia N : Infectious Mononucleosis O : Rheumatoid arthritis. Comment : The causes of generalised lymphadenopathy in a young individual raise the possibility of a malignant process, but viral infections are also a distinct possibility. Toxoplasmosis, CMV, Rubella and HIV infection should be excluded. People with mononucleosis may have deranged LFTs and a raised LDH. Hepatitis A infection may mimic the above presentation but generalised lymphadenopathy is not a feature. H : J : N:

Page 235: Masterclass Book Part 2

MOHAMMED IS-HAG 234

10. A 45-year-old man, previously well, receives a unit of blood on a surgical ward. Shortly after the transfusion commences you are bleeped to the ward and told that he has had a severe reaction to the blood. Which of the following are the two most common causes of this scenario? A : Incorrect blood unit being given (i.e. for another patient) B : Bacterial infection of blood C : Reaction to HLA antibodies D : Circulatory overload E : Incorrect sample sent to blood bank (resulting in the incorrect blood for the patient) F : Allergic reaction to white cells G : Graft versus host disease H : Reaction to plasma proteins I : Immune sensitization J : Allergic reaction to platelets. Comment : Although all of the above listed causes can result in a transfusion reaction (of varying severity), it has been reported that the most common reason for a severe reaction to blood is due to transfusion of the incorrect unit. In almost all cases this is due to human error, from incorrect labelling of blood samples to failing to check the details on the unit being given. No addressograph labels are allowed on the sample bottle in many hospitals and the forms and sample bottle are signed by the person performing the venepuncture. This is the latest guidance on blood safety. A : E : 11. A 70-year-old woman is admitted with anaemia and fatigue. Hb 6.7, MCV 101fl, WCC 76 (90% lymphs), platelets 105. Retics 14%.She is slightly jaundiced and has splenomegaly. The lab think she probably has chronic lymphatic leukaemia. She is on no medication. Which are the 2 most likely causes of the anaemia? Which are the most likely causes of the anaemia? A : B12 deficiency B : Bleeding C : Folate deficiency D : Autoimmune haemolysis E : Acquired G6PD deficiency F : Iron deficiency G : Acute transformation H : Marrow infiltration with leukaemia I : Hyperviscosity J : Myelodysplasia. Comment : The MCV is raised due to reticulocytosis - this is a feature of autoimmune haemolysis. A reticulocytosis does not occur in haematinic deficiency. Marrow infiltration also causes anaemia in CLL and advances the staging of the disease. D : H : 12. A 72-year-old man has had a full blood count for investigation of 'dizzy spells'. He is a life-long smoker and drinks three pints of beer a day. Otherwise he reports being fit and well. His haemoglobin is 20.3g/dl and haematocrit is 61%. Which two of the following statements are correct? A : He should be commenced on warfarin with a target INR of 2.5. B : He requires a red cell mass in order to confirm the diagnosis. C : In untreated primary proliferative polycythaemia rubra vera (PRV) there is an increased risk of

thrombosis. D : Untreated secondary polycythaemia is associated with a 15% risk of stroke over 1 year. E : PRV is confirmed with a red cell mass greater than or equal to 200% of predicted. F : Because of the risk of leukaemic transformation drugs such as hydroxyurea and busulphan

should be not be used in patients with PRV. G : Patients with PRV have a 50% chance of developing acute myeloid leukaenia over the following 5

years. H : Venesection is required in patients with PRV aiming to reduce the haematocrit to 35%. I : Patients with PRV are banned from driving because of the risks of stroke. J : Median survival for patients with PRV is 6 years. Comment : PRV is confirmed by a red cell mass of greater than 125% of predicted. Treatment is aimed at reducing the haematocrit to less than 45% by either venesection or drugs (such as hydroxyurea or busulphan). Risk of transformation to acute leukaemia is very low. Secondary polycythaemia is not associated directly with an increased risk of stroke. Patients are not banned from driving. Consult DVLA for advice regarding driving. The median survival for PRV is 10-15 years. B : C :

Page 236: Masterclass Book Part 2

MOHAMMED IS-HAG 235

13. A young man presents to A&E with a history of a febrile illness. FBC reveals lymphopenia. Which of the following are most likely to explain this finding? A : Infectious mononucleosis B : HIV infection C : Lymphoma D : Tuberculosis E : malaria F : Acute leukaemia G : Food poisoning H : Bacterial pneumonia I : Influenza J : Exercise. Comment : Both malaria and HIV (and AIDS) will classically present with a low lymphocyte count. All the other options would tend rather to be associated with a lymphocytosis B : E : 14. A 67-year-old man has become anaemic following cholecystectomy. He gives a history of jaundice as a child but otherwise has enjoyed good health. There is a history of anaemia in the family. Which are the three most important factors when making a diagnosis of hereditary spherocytosis? A : Spherocytes on the blood film B : Positive Coombs test C : Raised mean corpuscular volume D : Increased osmotic fragility E : Positive family history F : Raised mean corpuscular haemoglobin concentration G : Reduced mean corpuscular hamoglobin concentration H : Raised reticulocyte count I : DNA analysis J : Raised zinc protoporphyrin K : Normal B12 level L : Low Folate level M : Normal iron status N : Bone marrow O : CT scan of abdomen. Comment : Despite increasing technology, the most important factors when making a diagnosis are a family history and features on the blood count. Spherocytes are usually readily identified. Their spherical nature causes the MCHC rather than the MCV to become elevated. Osmotic fragility is now rarely used as a diagnostic indicator. Cryohaemolysis (increased cell lysis at 0C) and eosin-5-maleimide binding pattern to red cells are now preferred in cases where the diagnosis is less clear. Protein and DNA analysis is reserved for difficult cases or where there are combined abnormalities. A : E : F : 15. A 75-year-old man had high grade non-Hodgkin’s lymphoma. He received chemotherapy through a Hickman line. On his third course he developed persisting pancytopenia and a fever. The echocardiogram from the left ventricle is shown (see image). Which are the most important next two steps? A : Remove the Hickman line B : Commence heparin C : Obtain three separate blood cultures D : Speak to the cardiac surgeons E : Perform a Coomb's test F : Perform urinalysis G : Commence tazocillin, gentamycin and teicoplanin H : Perform ultrasound for splenomegaly I : Check liver function tests J : Commence aspirin. Comment : Endocarditis can be a problem with Hickman lines - in fact, this man had a mitral valve prolapse prior to therapy. It is vital to establish the organism by blood culture. Then commence broad spectrum antibiotics, including gram positive cover (associated with Hickman lines) - speak to the microbiologist!! All other actions can wait. Anticoagulation is not indicated in the absence of embolus. It may become necessary to remove the Hickman line, but this may be the only means of venous access in some patients - a tough decision (involve the patient!). C : G :

Page 237: Masterclass Book Part 2

MOHAMMED IS-HAG 236

16. The use of anti-D immunoglobulin has all but wiped out cases of haemolytic disease of the newborn in this country. Not all women need to be given anti-D and it should not be given to women who do not need it. Which of the following women would require further testing and anti-D following delivery? For the scenarios below Father =father’s blood group, Mother =mother’s blood group, Baby = baby’s blood group. A : Mother = A RhD -ve, Father = O RhD-ve, Baby = O RhD -ve B : Mother = AB RhD+ve, Father = B RhD-ve, Baby = O RhD -ve C : Mother = B RhD-ve, Father = B RhD+ve, Baby = O RhD +ve D : Mother = O RhD+ve, Father = B RhD-ve, Baby = O RhD +ve E : Mother = O Rh -ve, Father = AB RhD+ve, Baby = B RhD -ve F : Mother = B RhD+ve, Father = B RhD-ve, Baby = AB RhD +ve G : Mother = AB RhD+ve, Father = AB RhD-ve, Baby = ARhD +ve H : Mother = O RhD -ve, Father = B RhD-ve, Baby = O RhD -ve I : Mother = O RhD-ve, Father = B RhD+ve, Baby = ORhD +ve J : Mother = O RhD+ve, Father = O RhD +ve, Baby = ORhD -ve. Comment : The father’s blood group is not relevant (anyway, you should never assume paternity!). The ABO blood group is not relevant. Anti D is used when the mother is RhD-ve and the child RhD+ve. C : I : 17. You are contacted regarding a 62-year-old woman who has a platelet count of 620x109/l (normal range 150x109/l - 400x109/l). Which of the following would not account for this finding? A : Osteomyelitis B : Polycythemia rubra vera C : Thrombotic thrombocytopenia purupra D : 5 years post splenectomy E : 5 days post hip replacement F : Chronic myeloid leukaemia G : Acute lymphoblastic leukaemia H : Rheumatoid arthritis I : Essential thrombocythemia J : Mycoplasma pneumonia. Comment : Inflammatory and infectious processes can cause thrombocytosis, as can recent trauma or surgery. Platelets may be persistently raised following splenectomy. The myeloproliferative disorders (polycythemia rubra vera and essential thrombocythemia) are associated with a raised platelet count, as is chronic myeloid leukaemia. Acute lymphoblastic leukaemia would not cause a raised platelet count and frequently (but not always) would be associated with a low platelet count. As the name suggests, in thrombotic thrombocytopenic purpura, the platelet count is low. C : G : 18. A 36-year-old woman is admitted with a temperature of 39.6?C having been playfully bitten by her pet dog. The patient had her spleen removed at the age of 11 following a road traffic accident. Which of the following organisms is she particularly susceptible to? A : Streptococcus pneumonie B : Cryptococcus neoformans C : Serious acute respiratory (SARS) virus D : Salmonella typhimurium E : Candida glabrata F : Capnocytophaga canimorsus G : Polio virus H : Coagulase-negative staphyloccoci I : Shigella J : Listeria monocytogenes. Comment : People with an absent or dysfunctional spleen are prone to infections from encapsulated organisms such as Streptococcus pneumonie, Haemophilus influenzae and Neisseria meningitides. They are also prone to exotic bugs like Capnocytophaga canimorsus, a gram-negative bacillus which is present in the mouth of up to 40% of dogs. This can cause purpura fulminans and can be rapidly fatal. Protozoal illness like Falciparum malaria and babesiosis are also a real risk. The risk, estimated at 0.42/year following splenectomy, is lifelong and influenced by the indication for splenectomy. The risk can be reduced by a stringent vaccination strategy, prophylactic antibiotics and health education. Travel advice and wearing a Medic Alert bracelet or a credit card alert card may be potentially life saving.A : F :

Page 238: Masterclass Book Part 2

MOHAMMED IS-HAG 237

19. A 50-year-old man is homozygous for the C282Y HFE gene mutation. Venesection has brought the ferritin level into the normal range. Which three tests need to be monitored at least annually? A : Folate B : Liver function C : Renal function D : Calcium E : Parathormone F : Faecal occult bloods G : Ferritin H : Hand radiographs I : Zinc protoporphyrin J : Liver biopsy K : Hepatic magnetic resonance imaging L : Thyroid function M : Testosterone N : Alfa fetoprotein O : Plasma viscosity. Comment : Ferritin remains the best surrogate test for iron content and needs regular follow-up, as all patients have different rates of iron accumulation. Liver abnormalities need to be screened for, always be aware of the possibility of hepatoma. Other endocrine screening tests are not essential - apart from fasting glucose!!! B : G :N: 20. You see a patient in clinic who has anaemia, hypercalcaemia and osteolytic bone lesions with pain. Kidney function appears to be normal at this stage but he has an IgG kappa paraprotein of 50g/l. Bone marrow shows an infiltrate of 50% plasma cells. Which of the following treatments are NOT used at some point in time in the course of this illness? A : Paracetamol B : Melphalan C : Prednisolone D : Cyclophosphamide E : Thalidomide F : Morphine Sulphate Tablets G : Anagrelide H : Radiotherapy I : Pamidronate J : Asparaginase. Comment : This patient has myeloma. This is a relapsing and remitting disease without any known curative therapy. Some patients are elderly and die of an unrelated cause but most patients like this will die of their myeloma; median survival is about 2 to 3 years. Treatment is aimed at getting the disease to a state where by it (temporarily) stops causing end organ (kidney, bone marrow, bone) damage. Initial management includes intravenous combination chemotherapy generally for younger (under 65) patients. Older patients are generally treated with oral chemotherapy. All drug treatments are tried at some point in the course of the illness and are given on and off until the tumour is essentially unresponsive to all drugs. Radiotherapy, analgesic drugs and bisphosphonates are important in the management of myeloma. Anagrelide is used to control platelets in essential thrombocythaemia. Asparaginase is used in the treatment of acute lymphoblastic leukaemia. G : J : 21. A 32-year-old woman who is 9 weeks pregnant in her first pregnancy develops a painful swollen calf. Ultrasound scan confirms the presence of extensive deep vein thrombosis extending into femoral and illiac vessels. This is her first clot and there is no other obvious risk factor. Which of the following statements regarding this lady are correct? A : The patient should be commenced immediately on a therapeutic dose of low molecular weight heparin. B : Warfarin should be started and heparin can be stopped when the INR is greater than 2.0. C : In pregnant women the majority of DVT's occur in the right leg. D : This patient requires 3 months anticoagulation. E : 5mg warfarin tablets are pink. F : 1mg warfarin tablets are blue. G : She needs her APTT monitored whilst taking warfarin to titrate dose. H : Warfarin is contraindicated in patients taking Penicillin. I : Low molecular weight heparin is given intra-muscularly. J : She should be advised not to become pregnant again. Comment : This patient needs to start LMW heparin. Warfarin is teratogenic and should not be given in the first trimester. Because of bleeding during the time of labour warfarin should not be given in the

Page 239: Masterclass Book Part 2

MOHAMMED IS-HAG 238

third trimester. Although theoretically warfarin can be given in the second trimester generally speaking it is not used and pregnant patients get daily LMW heparin throughout pregnancy to treat DVT. Ninety per cent of DVT's in pregnancy occur on the left. Anticoagulation should continue until at least 6 weeks post partum in this patient.The APTT is used to monitor unfrationated heparin and the PT to monitor warfarin. Warfarin tablets: 5mg = pink, 3mg = blue, 1mg = brown. Penicillin will affect warfarin control but does not preclude its use. Pregnancy should not be avoided in the future but prophylactic anticoagulation should be offered during the next pregnancy. A : E : 22.

PLATE43 A 38-year-old man with sickle cell disease has done an internet search on his condition. He comes to your clinic with an extensive list of potential problems. Which three of the following are NOT part of the clinical spectrum of sickle cell disease? A : Priapism B : hypercalcaemia C : Stroke D : Leg ulcers E : Splenic sequestration crises F : Proliferative retinopathy G : finger clubbing H : Acute chest syndrome I : Marrow Aplasia J : Pneumoccocal septicaemia K : Bone pain crises L : Hand-foot syndrome M : Bone fractures N : Low birth weight babies O : Osteomyelitis. Comment : There is a constellation of clinical signs and symptoms directly attributable to small vessel and large vessel occlusion in sickle cell disease. The result can be ischaemic necrosis and organ damage in the affected part. Low birth weight babies are seen in affected mothers due to placental infarction, with foetal wastage in extreme cases. Osteoporosis is seen in sickle cell disease due to widening of the haemopoietic tissue and thinning of the cortices. Avascular necrosis of the long bones (especially femur) can occur, but bone fractures are not increased in those with sickle cell disease. Finger shortening can be seen, due to repeated infarction in a growth plate, but finger clubbing is not a feature of sickle cell disease. Hypercalcaemia is not a recognised feature of sickling disorders. B : G :M:

Page 240: Masterclass Book Part 2

MOHAMMED IS-HAG 239

23. A 61-year-old woman presents with tiredness. Her full blood count is as follows: Hb 9.1 g/dl, MCV 84 fl, MCH 28 pg, WBC 10.2 x 10^9/l, platelets 387 x 10^9/l. Which two descriptive terms are correct? A : Hypochromic B : Leucopenic C : Megaloblastic D : Microcytic E : Normochromic F : Normocytic G : Leucocytosis H : Thrombocytopenic I : Macrocytic J : Thrombocythaemic. Comment : The normal ranges are as follows: Hb 12-16 g/dl (women), 13-17 g/dl (men); MCV 82-102 fl, MCH 27-32 pg, WBC 4-11 x 10^9/l, platelets 130-400 x 10^9/l.This patient has a normochromic normocytic anaemia. A common cause for this would be the anaemia of chronic disease, e.g. secondary to rheumatoid arthritis or polymyalgia rheumatica, but it would be wrong to jump immediately to this diagnosis. A blood film should be examined because an MCV and MCH that fall within the normal range could result from two populations of cells – one microcytic and one macrocytic – which would be described as a dimorphic picture. This might result from, for example, combined iron and folate deficiency. E : F : 24. A 22-year-old woman has had heavy periods ever since she can remember, certainly for many years, and is now iron deficient. What two conditions must be considered? A : Haemophilia B : Coeliac disease C : Disseminated intravascular coagulation (DIC) D : Chronic liver disease E : Idiopathic thrombocytopenic purpura (ITP) F : Hypothyroidism G : Cushing’s syndrome H : Von Willebrand’s disease I : Chronic renal failure J : Hyperthyroidism. Comment : Von Willebrand’s disease occurs in 1% of the population and could present in this way. Acquired bleeding problems such as DIC, liver disease, ITP or uraemia are excluded by the length of the history.Von Willebrand’s factor (VWF) is the carrier for factor VIII and in Von Willebrand’s disease the activated prothrombin time (APTT) may be prolonged due to a low factor VIII level. Specific assays include those for VWF and ristocetin cofactor. Treatment in this case might include oral iron (to correct iron deficiency) and the use of DDAVP (which promotes release of VWF) intranasally at the time of the menses to reduce blood loss. Operative procedures can be covered with intravenous DDAVP. Do not forget to take a family history: others may be affected (autosomal dominant). F : H : 25. A young woman is diagnosed with antiphospholipid antibody syndrome, after being investigated for recurrent DVTs. Which of the following statements regarding treatment are valid? A : She requires aspirin alone. B : She requires long term warfarin therapy. C : Steroids are better treatment then heparin and aspirin. D : Low molecular weight heparin should not be used. E : She should not use the oral contraceptive pill. F : Obesity will not increase her risks. G : She requires long term heparin therapy. H : In pregnancy, warfarin should be given. I : In pregnancy, aspirin is sufficient treatment. J : Regular plasma-exchange is indicated. Comment : Antiphospholipid antibody syndrome (APAS) represents a spectrum of abnormalities which is multi-system in nature. It may be idiopathic or associated with a number of underlying causes, including inflammatory arthritis, infections and medications. In view of the recurrent DVTs she should be treated with lifelong warfarin with or without aspirin. Recurrent fetal loss is a feature and pregnancy should be managed with heparin (low molecular weight) and aspirin. Aspirin alone is insufficient and warfarin is teratogenic. Other risk factors for thromboembolism should be removed. B : E :

Page 241: Masterclass Book Part 2

MOHAMMED IS-HAG 240

26. A young woman presents with a swollen leg and is proven to have a DVT. On questioning she admits to a strong family history of thrombosis. Which of the following tests is unlikely to be helpful in determining genetic risk factors she may have? A : Protein C levels B : HFE gene studies C : Protein S levels D : Antithrombin III levels E : Prothrombin gene studies F : Homocytsteine levels G : Fibrin degradation products H : Antiphospholipid antibodies I : Beta-2 glycoprotein 1 assay J : Activated protein C resistance. Comment : All of the above tests, except B and G, may be included in a ‘thrombophilia screen’ and identify factors which may predispose individuals to venous and/or arterial thrombosis. The HFE gene mutation causes haemochromatosis and is not associated with thrombosis per se. Fibrin degradation products (FDPs) will be raised in a patient with a DVT, but are a non-specific marker and will return to normal on treatment. B : G : 27. A 56-year-old lady presented to A&E feeling ‘unwell’ and with headache. An MRI scan shows bilateral middle cerebral artery thrombosis and bilateral homonymous hemianopia. The counts are Hb 11g/dl, white blood count 15 x 109/l, platelets 540 x 109/l. The clotting was normal. The renal function is deranged. The blood film shows fragments and reticulocytes . The following are not recognized causes of intravascular haemolysis? A : Mismatched blood transfusion B : Sickle cell anaemia C : Paroxysmal nocturnal haemoglobinuria D : March haemoglobinuria E : Micro-angiopathic haemolytic anaemia F : Drugs G : Disemminated intravascular coagulation H : Blackwater fever I : Hereditary elliptocytosis J : Gram negative septicaemia. Comment : Intravascular haemolysis liberates free haemoglobin into plasma which binds to plasma haptoglobins. The complex is cleared and free haemoglobin gets filtered by kidneys and reabsorbed by proximal tubular epithelium. This is shed with the iron into urine giving haemoglobinuria, haemosiderinuria. The causes of intravascular haemolysis are usually acquired and the list is exhaustive but should be learnt. From the above list, only sickle cell and hereditary elliptocytosis cause extravascular haemolysis. B : I : 28. A 22-year-old man has splenomegaly. Which of the following would NOT cause splenomegaly in this patient? A : Non Hodgkin's lymphoma (NHL) B : Chronic myeloid leukaemia (CML) C : Beta thalassaemia major D : Sickle cell disease (HbSS) E : Acute lymphoblastic leukaemia (ALL) F : Falciparum malaria G : Hereditary spherocytosis H : Haemophilia A complicated by hepatitis C infection I : Idiopathic thrombocytopenia purpura (ITP) J : Essential thrombocythaemia (ET). Comment : The myeloproliferative disorders (although rare in this age group) may all cause splenomegaly. More common in a man of this age is leukaemia, which also may cause splenomegaly. Thalassaemia major, unlike beta thalassaemia trait, causes splenomegaly. Sickle cell disease causes splenomegaly in children but by the time individuals are adults the spleen has infarcted and is very small. Consequently penicillin prophylaxis and vaccinations (pneumococcus, meningococcus and HIB) are recommended to reduce the risk of infection with encapsulated organisms. ET, being a myeloproliferative disorder, is associated with splenomegaly but ITP is not. D : I :

Page 242: Masterclass Book Part 2

MOHAMMED IS-HAG 241

29. A 6-month old boy presented with skin necrosis and leg swelling. There is a strong family history of recurrent venous thromboembolism. Which of the following blood coagulation proteins inhibit blood coagulation? A : Fibrinogen B : Factor IX C : Prothrombin D : Factor VIII E : Protein C F : Factor XIII G : Factor V H : Antithrombin I : Factor XII J : Factor VII. Comment : Haemostasis is a finely balanced act between clotting and excessive bleeding due to the presence of natural anticoagulant and procoagulant proteins. The natural anticoagulants are Protein C, Protein S and Antithrombin while the rest listed serve to initiate and/or stabilise blood clots in normal circumstances. The boy has homozygous protein C deficiency which manifests as extensive skin necrosis E : H : 30. An obese 50-year-old man attends his GP's surgery because he is getting headaches. He has a history of mild hypertension, for which he takes bendrofluazide 5 mg once daily, and he smokes 15 cigarettes per day. He looks plethoric, and his full blood count reveals a haemoglobin concentration of 19.2 g/dl. The most likely two causes of his abnormal blood count are: A : Benign intracranial hypertension B : Iron deficiency C : Primary poliferative polycythaemia D : Chronic obstructive pulmonary disease E : Renal disease causing inappropriate erythropoietin excretion F : Previously unsuspected cyanotic congenital heart disease G : Diuretic treatment H : Liver disease causing inappropriate erythropoietin excretion I : Cerebellar haemangioblastoma J : Stress polycythaemia. Comment : Polycythaemias can usefully be classified into: 1. True primary - primary proliferative polycythaemia, also known as polycythaemia rubra vera 2. True secondary - due to hypoxia, which stimulates appropriate erythropoietin release, or to inappropriate erythropoietin excretion, which can be caused by renal disease, hepatic disease, uterine fibroids or cerebellar haemangioblastoma 3. Apparent relative polycythaemia – due to reduced plasma volume, caused by dehydration / diuretics or stress polycythaemia, commonly seen in middle-aged overweight smokers. The diagnosis of apparent relative polycythaemia is established by measurement of red cell mass and plasma volume. Treatment is by modification / removal of associated factors, but in some cases venesection may be needed, although this is a contentious issue. G : J : 31. An 80-year-old man came to A&E with a 2-day history of swelling in his right eye and diplopia. He was found to be anaemic, hypercalcemic and his erythrocyte sedimentation rate (ESR) was markedly elevated. He describes a 3-month history of generalised bone pain. The following investigations would be expected to be normal: A : Serum electrophoresis B : Skull radiographs C : Serum calcium level D : Urine for Bence Jones Protein E : Serum thyroid level F : Bone marrow biopsy G : Blood urea level H : Serum parathyroid hormone level I : Serum urate level J : Serum beta 2 microglobulin. Comment : This man's likely diagnosis is multiple myeloma. He has a secondary plasmacytoma of the orbit (and not thyrotoxicosis, which can also cause diplopia). Skull radiographs usually show lytic lesions which are surprisingly unpainful. The serum parathyroid hormone level is normal. E : H :

Page 243: Masterclass Book Part 2

MOHAMMED IS-HAG 242

32. A 47-year-old decorator has hereditary spherocytosis. His Hb is 11 g/dl, MCV 89 fl, bilirubin 23 mmol/l. He copes with his job but asks if splenectomy may help. Which are the most appropriate responses? A : Splenectomy is without serious consequences. B : Folic acid replacement can usually obviate the need for splenectomy. C : Splenectomy is indicated for symptomatic anaemia. D : Splenectomy should always be followed by B12 replacement. E : Splenectomy does not require vaccination in adults. F : Splenectomy should be routinely performed at the age of 7 years. G : Splenectomy is indicated for splenomegaly in spherocytosis H : Splenectomy is indicated for uncompensated haemolysis. I : Splenectomy always improves the Hb level. J : Splenectomy reduces the number of circulating spherocytes. Comment : Splenectomy is not without hazard. It is reserved for symptomatic anaemia which affects quality of life. Vaccination and lifelong penicillin are required in adults. Splenectomy removes the source of red cell destruction and spherocytes may rise in number on the blood film. Splenectomy is performed in older rather than young children, but is not routine. C : H : 33. A 45-year-old woman presents with tiredness and weight loss. She has 4cm splenomegaly. Her white cell count is 145. Which of the following are typical features of chronic myeloid leukaemia (CML)? A : Thrombocytopenia B : Lymphadenopathy C : Eosinophilia D : Translocation between chromosome 9 and 22 E : Skeletal fracture F : Raised plasma viscosity G : Raised leucocyte alkaline phophatase H : Low leucocyte alkaline phophatase I : Anaemia J : Abdominal pain. Comment : Thrombocytosis and basophilia are often seen in CML. Total blood viscosity is often raised, but plasma viscosity is normal. Abdominal pain may be associated with splenomegaly, but this is not typical. D : H : 34. A 57-year-old lady presented with pancytopenia, Hb 9.6g/dl, Wbc 2.0 x 10^9/l and plts of 56 x 10^9/l. A bone marrow biopsy showed widespread granulomas with no other features of haemopoietic malignancy. Which of the following diseases do not cause bone marrow granulomas? A : Sarcoidosis B : Foreign bodies C : Tuberculosis D : Hodgkin's lymphoma E : Myelofibrosis F : Cytomegalovirus infection G : Osteopetrosis H : AIDS I : Brucellosis J : Ehrlichiosis. Comment : Granulomas can be found in bone marrow as a manifestation of systemic disease such as sarcoidosis or tuberculosis, or non-specifically. In many cases the cause is not found. Ehrlichia is an infection caused by a protozoa in immunocompromised patients with characteristic inclusion bodies in macrophages. It is commonly associated with granuloma formation. Foreign bodies in the marrow can incite granuloma formation with epitheloid cell and multinucleate giant cells with fibrous tissue arranged in a concentric fashion. Myelofibrosis and osteopetrosis cause marrow fibrosis or replacement by sclerotic bone which leads to a leucoerythroblastic blood film. E : G : 35. A 67-year-old man presents with heart and renal failure. T he marrow is infiltrated with plasma cells and confirms the diagnosis of myeloma. Which of the following possibilities are features of this condition? A : Renal failure is the commonest presentation. B : The antigen specificity of the monoclonal band is easily established. C : Bisphosphonate therapy has no place in preventing bone disease. D : Amyloid is a common complication. E : Bence-Jones protein occurs without serum bands in 15% of myeloma. F : Immuneparesis is a typical. G : Coombs' positive anaemia is typical. H : An IgM monoclonal band is seen in many myeloma cases. I : Lymphadenopathy is frequently seen. J : Plasmapheresis should be considered for hypercalcaemia.

Page 244: Masterclass Book Part 2

MOHAMMED IS-HAG 243

Comment : Bone pain is the commonest presentation. The origin and specificity of the monoclone is very difficult to determine. Amyloid is not common. The presence of monoclonal immunoglobulins suppresses normal immune function – some patients require infusions of normal immunoglobulin. IgM bands are seen in Waldenstr?m’s macroglobulinaemia, not myeloma. Plasmapheresis is considered when the plasma viscosity threatens neurological or cardiac function; hypercalcaemia is treated with fluid and bisphosphonate infusion. E : F : 36. A 73-year-old woman is admitted with Hb 7.5, MCV 99fL, neuts 1.1, platelets 68, reticulocytes 1%. Her neutrophils appear hypogranular. Poikilocytosis and anisocytosis are present. She is only on atenolol for hypertension. She has had no previous illnesses. What features do NOT support a diagnosis of myelodysplasia? A : Low B12 level B : Normal B12 level C : Ring sideroblasts in the marrow D : Monocytosis E : Raised ferritin F : Cytogenetic abnormalities on marrow aspiration G : 5% blasts in the marrow H : Normal folate levels I : Normal retic count J : Negative Coombs' test. Comment : B12 and folate levels must be normal to establish the diagnosis. Blasts can be a feature of evolving myelodysplasia. The retic count is variable. A : E : 37. In which of the following clinical scenarios would you commonly consider giving a platelet transfusion? A : Idiopathic thrombocytopenia (ITP) B : Acute myeloid leukaemia (AML) C : Thrombotic thrombocytopenia D : Renal failure E : Myeloma F : Disseminated intravascular coagulation (DIC) G : Chronic lymphoid leukaemia (CLL) H : B12 deficiency I : Warfarin overdose J : May–Hegglin abnormality. Comment : Platelet transfusions are given for AML, before or after chemotherapy. It is rarely needed in myeloma or CLL. DIC is usually treated with blood products and platelets. The May–Hegglin abnormality is an inherited thrombocytopenia with little bleeding problems - it rarely requires platelet infusions. Platelets should not be infused in ITP as they are consumed by the patient's antibodies. Platelet transfusions are essentially contraindicated in thrombotic thrombocytopenia purpura. B : F : 38. You are given a blood count by the gastroenterology secretary. It is from a 67-year-old woman in last week’s clinic. It shows Hb 8.5 g/dl, MCV 122 fl, platelets 98, neutrophils 1.2. Your next two steps should be to: A : organise a Schilling test B : phone the GP to ask him/her to check B12 and folate levels C : get a reticulocyte count D : phone the consultant gastroenterologist E : check to see what the blood film showed F : look in Medical Masterclass G : phone the consultant haematologist H : write to the patient and ask them to commence oral folate I : make a note to check her Coomb's test at the next clinic J : leave the problem untill the next clinic. Comment : Although it is important to seek help from senior colleagues and Medical Masterclass, you should be able to make a decision on this as a medical SHO. Don't leave this sort of problem to a later date or to somebody else - if the patient is lost to follow up they could become quite ill. The blood film will show if there is polychromasia (immune haemolysis) or features of megaloblastic anaemia. The film will give you more information than just a reticulocyte count (is this high in megaloblastic anaemia?). The B12 and folate levels should be checked quickly because restoring these haematinics can make people feel a lot better very quickly. B : E :

Page 245: Masterclass Book Part 2

MOHAMMED IS-HAG 244

39. A 32-year-old woman is seen in the antenatal clinic. Her full blood count reveals: Hb 10.1 g/dl, MCV 103 fl, Platelets 112, Neutrophils 9.3, Lymphocytes 3.7, Monocytes 0.5. Which two of the following are the most likely cause for this picture? A : Chronic granulocytic leukaemia B : Folate deficiency C : Myelofibrosis D : Normal haematology of pregnancy E : Iron deficiency F : Thrombotic thrombocytopenic purpura G : Pre-eclampsia H : HIV infection I : Urinary tract infection J : Acute leukaemia. Comment : A macrocytosis, mild thrombocytopenia and neutrophilia are commonly seen in pregnancy and are of no significance. However, always be alert for folate deficiency in pregnancy as this is easily treated and has implications for the fetus. B : D : 40. A 29-year-old Asian woman is 16 weeks pregnant. There is no family history of haemoglobinopathy. She eats meat twice a week and has a balanced diet. She has had no gastrointestinal problems. Hb 9.9 gm/dl, MCV 68 fl, iron levels borderline low. What are the two most important steps to take next? A : Commence oral iron B : Commence oral folate C : Perform HbA2 and HbF D : Perform a Coomb's test E : Arrange a blood transfusion F : Suggest she eats meat more frequently G : Commence oral vitamin supplements H : Commence intramuscular iron I : Organise a chorionic villous biopsy J : Test her partner for any inherited haemoglobinopathy. Comment : Although iron and folate deficiency are common in pregnancy and require therapy, there may be something else happening here. Beta thalassaemia carriers have a raised HbA2 and HbF. If they reproduce with another carrier they could give birth to a severely affected homozygous child - that is why it is important to test the partner early to alert obstetricians and paediatricians about the possibility. C : J : 41. A 65-year-old man is admitted with a 7-day history of easy widespread bruising and 24 hours of developing blood blisters in his mouth. He is having difficulty swallowing solids and liquids. He is found to have a factor 8 inhibitor and is correctly diagnosed with acquired haemophilia. The on-call haematologist has been contacted immediately as this is a medical emergency! Which of the following statements are NOT true or are INAPPROPRIATE regarding this gentleman and his care? A : Joint bleeding is a common problem in acquired haemophilia. B : The haematologist recommends that recombinant factor 7 treatment 90mcg/Kg, three doses

each 2 hours apart should be given C : The haematologist recommends that recombinant factor 7 should be given through a central

line. D : The haematologist recommends that Prednisolone 1mg/Kg daily should be started. E : Regular frequent assessment of his breathing and swallowing is essential. F : Consideration should be given to the presence of a coexisting malignancy. G : Consideration should be given to the presence of a coexisting vasculitis. H : There is no association between acquired haemophilia and congenital haemophilia. I : Mortality in this condition is in the region of 25%. J : Acquired factor 8 deficiency is more common than acquired factor 9 deficiency. Comment : Joint bleeding though characteristic of congenital haemophilia is hardly ever seen in acquired haemophilia. In acquired haemophilia mucosal bleeding is a real problem. Bleeding in the mouth or around the airway constitutes an emergency and EXPERT help is needed immediately. This patient needs urgent clotting factor treatment and option C is not unreasonable with reassessment of treatment after the third dose. More may need to be given. Recombinant factor 7 has a short half life (1-2 hours) and may need to be given repeatedly. Some haematologists may recommend factor eight inhibitor bypassing agent (FEIBA), which is a blood product. Clearly a central line is going to risk serious further bleeding and recombinant factor 7 can be given through a venflon. It goes almost without saying that regular frequent assessment is required. Acquired inhibitors (F8 more common than F9) may be

Page 246: Masterclass Book Part 2

MOHAMMED IS-HAG 245

associated with underlying medical problems such as malignancy or vasculitis and there is no association with congenital haemophilia. Mortality is high. This appears to be an 'autoimmune' disorder and the mainstay of therapeutic treatment is immune suppression. Steroids should be prescribed in the first place, then consider drugs such as cyclophosphamide.A:C : 42. A 67-year-old man on Warfarin for Atrial Fibrillation presented with a right upper arm DVT confirmed on venography, despite being on Warfarin with an INR of 7.5. He denies external blood loss. His right upper limb is massively swollen. Which two explanations are most likely? A : Lack of compliance with Warfarin B : Stroke with monoparesis C : Henoch scholein purpura D : Severe megaloblastic anaemia E : Cardiac emboli to limb F : Disseminated carcinomatosis G : Acute lymphoblastic leukaemia H : Homozygous protein C deficiency I : Rethromboses with background acquired thombophilia J : Aspirin therapy. Comment : Despite what looks like excessive anticoagulation, thrombosis is rare. One should look for sinister causes such as a metastatic malignant process and other causes of acquired thrombophilia such as antiphospholipid syndrome. Homozygous protein C deficiency is not compatible with life or will have presented much earlier on in life. In fact, this patient was eventually diagnosed with carcinoma of the pancreas. F : I : 43. A 75-year-old lady was admitted via A&E with the following blood counts: Hb 4g/dl, white blood count 11 x 109/l, mean corpuscular volume 109 fl, and platelets 540. The reticulocyte count is 6%. Which of the following are not useful investigations in extravascular haemolytic anaemia? A : Bilirubin estimation B : Haemoglobin electrophoresis C : G6PD spot test D : Haemosiderinuria E : Absolute reticulocyte count F : Vitamin B12 estimation G : Serum haptoglobins H : Osmotic fragility test I : Sickle solubility test J : Blood film evaluation. Comment : In extravascular haemolysis, destruction of the damaged red cell occurs in the reticuloendothelial system (RE) with formation of haem and globin. The haem is broken down into iron which is bound to transferrin for recirculation and the globin is reutilized. Conditions which cause extravascular haemolysis include: · sickle cell · hereditary spherocytosis · thalassaemia. Haemosiderinuria is a by-product of ‘intravascular’ haemolysis and serum haptoglobins are low due to the haptoglobin–haemoglobin complex being cleared by the RE system in intravascular haemolysis. D : G :

Page 247: Masterclass Book Part 2

MOHAMMED IS-HAG 246

1. Which statement is accurate about the clinical use of 5HT3 antagonists? A : They have made a substantial impact in chemotherapy-induced delayed nausea and vomiting B : They have benefits if used for anticipatory nausea and vomiting C : They are useful as adjuncts to prevent morphine-induced intractable nausea in oncology patients D : In pre-clinical and clinical studies, the maximum-tolerated dose was never achieved E : They can antagonize the effects of metaclopromide and haloperidol. Comment : 5HT3 antagonists have revolutionized modern oncology practice for their benefit in the prevention of ACUTE chemotherapy-induced emesis. They are often prescribed for three to five days after chemotherapy erroneously, as they have no proven benefits for delayed emesis (over standard therapies and steroids). This practice is only justified in split-course regimens.In pre-clinical and clinical studies, the maximum-tolerated dose was never achieved, so that we have to “guesstimate” the best dose. Indeed the minimum effective dose is unclear and may be as little as one-tenth of currently used doses! Over the last five years there has been a gradual reduction of dosing in most large oncology centres with no obvious reduction in efficacy. D : 2. A 43-year-old man develops a left hemiparesis over 3 days. CT scan shows a complex enhancing mass with oedema in the right parietal lobe. A stereotactic biopsy reveals a glioblastoma multiforme. Which one of the following statements is untrue? A : Radical radiotherapy is indicated. B : Primary treatment is debulking surgery. C : Raised intracranial pressure should be treated with dexamethasone. D : Five-year survival is 30%. E : At relapse, chemotherapy may be useful. Comment : The mean age at diagnosis for patients with glioblastoma multiforme is 54 years. These patients most often have a single, large, supratentorial, contrast-enhancing lesion seen on MR imaging and a short history of neurologic findings noted before diagnosis. Most patients are treated with surgery and radiotherapy. When adjuvant chemotherapy is used, it is usually given at the initiation of radiotherapy and again after the completion of radiotherapy. The agent most commonly used is single-agent BCNU (carmustine). The goal of surgery is to make a specific diagnosis, relieve symptoms, and reduce tumour bulk to the maximum extent possible. With this approach, the median survival expectation is approximately 50 weeks. Various prognostic factors influence survival, with younger age being one of the strongest predictors of longer survival D : 3. A 76-year-old woman presents with haemoptysis and clubbing. A diagnosis of lung cancer is made. Which is the most likely histological subtype? A : Small cell lung cancer B : Bronchoalveolar carcinoma C : Carcinoid D : Squamous cell lung cancer E : Adenocarcinoma. Comment : Almost all forms of paraneoplastic syndromes with lung cancer are commoner with small cell lung cancer, except clubbing, hypercalcaemia and hypertrophic pulmonary osteoarthropathy (HPOA) which are more frequent with Squamous cell tumours. D : 4. A 49-year old man presents to his GP with a history of chronic intermittent diarrhoea. On examination he has a flushed appearance and facial telangectasia. He has a heart murmur and hepatomegaly that is nodular and firm. Na 130 mmol/l K 2.5 mmol/l Urea 12.2 mmol/l Creatinine 119 mmol/l Calcium 2.4 mmol/l Albumin 29 g/l ALT 27 U/l Which single test may assist in confirming the diagnosis? A : Carcinoembryonic antigen (CEA) B : The tumour marker, CA 19-9 C : 5-hydroxyindoleacetic acid D : Vanillylmandelic acid E : Serum catecholamines.

Page 248: Masterclass Book Part 2

MOHAMMED IS-HAG 247

Comment : This patient has carcinoid syndrome and it is due to an underlying carcinoid tumour. These tumours may contain and secrete a number of biologically active substances. Immunocytochemical or radioimmunoassay studies show that carcinoids can contain adrenocorticotrophic hormone (ACTH), gastrin, somatostatin, insulin, motilin, growth hormone, gastrin- releasing peptide, serotonin, calcitonin, neurotensin, melanocyte- stimulating hormone-beta, tachykinins (substance P, substance K, neuropeptide K), glucagon, pancreatic polypeptide (PP), vasoactive intestinal peptide (VIP), and prostaglandins. These substances may not be released in sufficient amounts to cause symptoms. In various studies of patients with carcinoids, elevated serum concentrations of PP occur in 43%, motilin in 14%, and subunits of human chorionic gonadotropin (HCG) in 12%; a slightly elevated level of gastrin was reported in 15%, and none were reported with an elevated VIP or plasma gastrin-releasing peptide level. Even though these gastrointestinal (GI) peptides were present in the serum, it is not apparent that any of these peptides contributed to any clinical symptoms. Foregut carcinoids are more likely to produce various GI peptides than midgut carcinoids. 5-hydroxyindoleacetic acid is the metabolic product excreted in urine from the metabolism of serotonin. Ectopic ACTH production with Cushing's syndrome is increasingly seen with foregut carcinoids, and in some studies, these tumours were the most common cause of the ectopic ACTH syndrome, accounting for 64% of all patients. Acromegaly due to release of growth hormone-releasing factors can occur with a number of carcinoids. C : 5. A 62-year-old man presents with altered bowel habit and has positive faecal occult bloods. Colonoscopy reveals a tight sigmoid adenocarcinoma. A staging CT scan shows two metastases in the left lobe of the liver. The initial treatment plan should be: A : chemotherapy prior to surgery B : left hemicolectomy and palliative chemotherapy C : chemo-radiotherapy to primary tumour, then continued palliative chemotherapy D : left hemicolectomy then consideration of liver resection E : palliative chemotherapy alone. Comment : The first priority should be resection of the sigmoid lesion to prevent large bowel obstruction. Two liver lesions confined to the same lobe may be suitable for liver resection, with chemotherapy either before or after resection. D : 6. An elderly man with carcinoma of the prostate and bone metastases presents with sudden worsening of a previous pain in his right leg. His drug regime included morphine sulphate continus (MST) 100mg bd. Which of the following would you do? A : X-ray the right femur B : Re-titrate his opiate requirement with oral morphine C : X-ray the lumbar spine and right femur D : Change to sub-cutaneous diamorphine E : X-ray lumbar spine. Comment : It is important to rule out a fracture or spinal collapse. The pain may originate from either the lumbar spine or femur. There may also be new metastases that may be amenable to radiotherapy. The patient will probably need an increase in oral opiates. C : 7. A 30-year-old man presented to his primary care physician with a complaint of right leg pain of one year's duration. The pain was worse at night but was fully relieved by aspirin. A plain radiograph reveals a focally sclerotic expanded area of the tibial cortex without overlying soft tissue or periosteal abnormalities. A radiolucent nidus is visible in the centre of the focal area of sclerosis. Radionuclide bone scintigraphy reveals a very prominent focal uptake of the radiotracer in the same region. What is the most likely diagnosis? A : Osteoid osteoma B : Stress fracture C : Metastatic deposit D : Trauma E : Osteogenic sarcoma. Comment : Osteoid osteoma is a benign bone tumour with a central small nidus of osteoid which incites a vigorous reaction in surrounding tissue. It is characteristically found in the femur, tibia, talus, spine and humerus, usually in the diaphysis or metaphysis of these bones. The common presentation is one of pain unrelated to activity or most pronounced after drinking alcohol. The pain is often relieved with non-steroidal anti-inflammatory drugs (NSAIDs). It is most common between 7 and 25 years of age and is three times more common in males. A : 8. Regarding medullary thyroid cancer, which one of the following statements is incorrect? A : A raised calcitonin is indicative of metastatic rather than primary medullary thyroid cancer. B : Five-year survival is 50%. C : Radioiodine is not indicated after thyroidectomy. D : First degree relatives should be screened for phaeochromocytoma. E : Diarrhoea is a common paraneoplastic feature.

Page 249: Masterclass Book Part 2

MOHAMMED IS-HAG 248

Comment : Medullary thyroid cancer (MTC) is a tumour of the parafollicular C cells that accounts for approximately 10% of all thyroid malignancies. An estimated 75% of MTC cases are sporadic, and the remaining 25% are familial. Calcitonin levels are most useful in screening individuals who are genetically predisposed to the disease and in following patients who have already been treated. Regardless of the form in which MTC presents, the primary treatments for MTC are total thyroidectomy and central lymph node dissection. A persistently elevated calcitonin level following resection suggests residual or recurrent MTC. A : 9. A 49-year-old man presents with a lump in his right breast. Which one of the following statements is true of male breast cancer? A : It generally has a better prognosis than similar stage disease in women. B : It accounts for 1% of male malignancies. C : It is treated in a similar fashion to breast cancer in women. D : It can be predicted by the presence of BRCA-1 and BRCA-2 gene mutations. E : It is a common cause of unilateral gynaecomastia in men. Comment : Male breast cancer is rare and there are approximately 200 cases per year in the UK. It accounts for 1% of all breast malignancies. It is often detected late because most cases of gynaecomastia are benign. Stage for stage the treatment is the same for men and women, although men have a worse prognosis. The presence of BRCA-1 and BRCA-2 gene mutations indicate a high risk for the development of breast cancer, but neither of these tests is useful for screening a population. C : 10. A 35-year-old woman develops bleeding from her nipple. The most likely underlying cause is: A : Duct ectasia B : Fat necrosis C : Papilloma D : Inflammatory carcinoma E : Sclerosing adenosis. Given the age of the patient,the bleeding from the nipple is most likely to be associated with a papilloma .The remaining options are less likely at this age or to cause bleeding as the presenting feature. C : 11. Which of the following is least likely to be associated with an increased risk of developing hepatocellular carcinoma? A : Alpha-1-antitrypsin deficiency B : Primary biliary cirrhosis C : Chronic hepatitis B infection D : Haemochromatosis E : Long-term aflatoxin ingestion. Comment : Eighty per cent of cases of hepatocellular carcinoma (HCC) are associated with chronic hepatitis B infection. Most cases of HCC not associated with hepatitis B infection are associated with hepatitis C infection. Alcoholic cirrhosis and autoimmune chronic active hepatitis also increase the risk of developing HCC. Several metabolic diseases are also associated with an increased risk for the development of HCC, such as haemochromatosis (iron accumulation), Wilson's disease (copper accumulation), alpha-1-antitrypsin deficiency and glycogen storage diseases.Alflatoxins are mycotoxins produced by the fungi Aspergillus flavus and Aspergillus parasiticus which contaminate food (e.g. peanuts and corn). A less common association is with primary biliary cirrhosis which is more typically associated with the subsequent development of cholangiocarcinoma. B : 12. A 60-year-old man presents with urinary retention and a short history of back pain. He has mild impairment of gait. Which investigation would you initiate urgently? A : Chest radiograph B : CT scan of brain C : Trans-rectal ultrasound and prostate biopsy D : CT Scan of pelvis E : MRI spine. Comment : This is suspicious of early spinal cord compression. This could be because of many causes including secondary cancer from a number of primary sites, including prostate. Neurosurgery would be the first choice treatment in a patient with a solitary lesion compressing the cord, particularly if there is no prior diagnosis of cancer. E : 13. Which one of the following is not true about carcinoid syndrome? A : Elevated urinary excretion of 5-hydroxyindoleacetic acid (5HIAA) may be associated with

nontropical sprue B : It is associated with primary metastatic tumours in the gastrointestinal tract C : Pharmacological blockade is clinically useful in only 10% of patients D : Flushing attacks may be associated with bronchoconstriction, periorbital oedema, salivation and

excessive lacrimation E : Niacin supplementation can prevent pellagra in patients with marked elevation of urinary 5HIAA.

Page 250: Masterclass Book Part 2

MOHAMMED IS-HAG 249

Comment : Carcinoid tumours are classified as neuroendocrine tumours and take origin from four main sites: bronchus, appendix, rectum and jejuno-ileum. The acute symptoms of the carcinoid syndrome are: · vasomotor flushing · fever · pruritic wheals · diarrhoea · asthma · wheezing · borborygmi · abdominal pain . Chronic complications include: · tricuspid regurgitation · arthropathy · pulmonary stenosis · mesenteric fibrosis · cirrhosis · pellagra · telangiectasia. 5-HT (5-hydroxytryptamine, serotonin) is synthesized from tryptophan which is an essential amino-acid and is a precursor of nicotinamide (niacin). If excessive 5-HT is produced a deficiency of the vitamin can lead to pellagra. Five per cent of patients with carcinoid tumours develop carcinoid syndrome after the development of hepatic metastases, when first-pass metabolism of 5-hydroxyindoleacetic acid (5HIAA) and kinins in the liver is avoided so that the systemic symptoms occur. 5HIAA is the metabolic product excreted in the urine from the metabolism of serotonin. Diagnosis relies on the measurement of serotonin or its metabolites (5HIAA) in urine. Octreotide, a somatostatin analogue, is now the drug of choice for controlling the symptoms of carcinoid. Only ~7% of patients fail to respond in some way. C : 14. A patient presents with facial swelling and distended veins in the chest and arms. A chest radiograph shows upper mediastinal widening. Which of the following should be done next? A : Doppler ultrasound B : Radiotherapy to the chest C : Bronchoscopy D : CT Scan of chest and abdomen E : Mediastinoscopy. urgent CT scan will help in deciding the best way of getting a tissue diagnosis, be this by CT guided biopsy, mediastinoscopy or bronchoscopy if there is a hilar lesion. In the vast majority of patients it should be possible to obtain a tissue diagnosis and institute appropriate treatment; in the case of lymphoma or small cell lung cancer this will be chemotherapy, rather than palliative radiotherapy. D : 15. Which of the following statements describes the relationship between ovarian germ cell tumours and serum markers? A : Pure dysgerminomas produce alpha-fetoprotein (AFP) or human chorionic gonadotropin (beta-

hCG) in more than 90% of cases. B : More than 40% of non-dysgerminomatous germ cell tumours produce no cell markers. C : Both beta-hCG and AFP should be measured in following the progress of a tumour. D : Measurement of tumour markers one week following surgery for localized disease is useful in

determining completeness of the resection. E : Beta-hCG is limited in its usefulness as a marker, because it is identical to human luteinizing

hormone. Comment : About 90% of persons with non-dysgerminomatous germ cell tumours produce either alpha-fetoprotein (AFP) or human chorionic gonadotropin (beta-hCG). In contrast, persons with pure dysgerminoma usually produce neither. These tumour markers are present for some time after surgery. If the presurgical levels are high, 30 days or more may be required before meaningful postsurgical levels can be obtained. The half-lives of AFP and beta-hCG are six days and one day respectively. After treatment, unequal reduction of beta-hCG and AFP may occur, suggesting that the two markers are synthesized by heterogeneous clones of cells within the tumour; thus, both markers should be followed. Beta-hCG is similar to luteinizing hormone except for its distinctive beta subunit. C :

Page 251: Masterclass Book Part 2

MOHAMMED IS-HAG 250

16. A 63-year-old woman presents with one episode of haemoptysis. The chest radiograph reveals a left hilar lesion, confirmed on CT scan which is otherwise clear. She had been treated for node-positive, oestrogen receptor positive left breast cancer four years previously with wide local excision and axillary dissection, adjuvant anthracycline chemotherapy, radiotherapy and tamoxifen, which she is still taking. Which of the following is appropriate initial management? A : Withdraw tamoxifen and await tamoxifen withdrawal response B : Commence second line hormone therapy with anastrazole C : PET scan D : Palliative chemotherapy with a taxane E : Bronchoscopy and biopsy. Comment : Although metastatic breast cancer can present in countless ways, the possibility of a second malignancy should be considered (in this case, possibly an operable lung cancer). If the pattern of recurrence is characteristic, for example multiple bone or liver metastases, then a confirmatory biopsy is not generally required. E : 17. Following chemotherapy a patient develops symmetrical paraesthesia and sensory loss over the toes of both feet. Which of the following cytotoxics is least likely to cause this side effect? A : Vincristine B : Doxorubicin C : Docetaxel D : Cisplatin E : Paclitaxel. Comment : Peripheral neuropathy, the most frequent neurotoxicity of chemotherapy, is commonly seen with vinca alkaloids, taxanes and platinum derivatives. It usually presents as symmetrical sensory loss that may progress to worsening paraesthesia, loss of tendon reflexes and eventually motor weakness due to axonal degeneration. Features usually slowly improve over several months following cessation of chemotherapy, although residual deficits may persist indefinitely. B : 18. A 65-year-old woman with increasing abdominal pain is found to have a pelvic mass on physical examination. After appropriate staging studies she undergoes a laparotomy and is found to have serous carcinoma of the ovary with involvement of one ovary and several omental implants. She then undergoes a hysterectomy, bilateral salpingo-oophorectomy, liver biopsy, omentectomy, cytological examination of abdominal washings, and extensive inspection. All evidence of disease is removed.Assuming generally good health, an uneventful postoperative recovery, and lack of proximity to a centre performing clinical trials, what treatment should she now receive? A : No further therapy. B : Combination chemotherapy. C : Combination chemotherapy only if serum CA125 level is elevated. D : Intraperitoneal chemotherapy. E : Whole abdominal radiation therapy. Comment : The overall 5-year survival of those with disease that extends beyond the ovaries is 40%. However, some patients who are able to undergo complete or nearly complete initial cytoreductive surgery, may be cured with combination chemotherapy. Presumably such therapy eradicates residual subclinical disease, which is invariably present despite the apparently complete resection. Effective drugs include taxol, cisplatin, cyclophosphamide, hexamethylmelamine, and doxorubicin. Paclitaxel plus cisplatin is the standard regimen. Since some patients may have recurrent disease without an elevation of CA125 (which is a useful antigen in monitoring response to therapy in those who have elevated levels) the delay of therapy, pending a rise in this level, would not be prudent. Clear survival benefits have yet to be shown for the fairly toxic regimen of whole abdominal radiation therapy. Intraperitoneal chemotherapy holds promise in the eradication of minimal disease, but its role needs to be defined by further clinical trials. B : 19. Radiotherapy treatment carries a risk for the development of secondary malignancies. Which of the following is not associated with being induced by radiation therapy? A : Breast cancer B : Acute leukaemia C : Thyroid cancer D : Neuroblastoma E : Soft tissue sarcoma Comment : Embryonal tumours such as neuroblastoma, Wilms' tumour and hepatoblastoma do not occur after radiotherapy treatments. This is most likely due to the fact that their origins develop in utero. Sarcoma of the bone and soft tissue are the most common secondary malignancy when the treatments are high-dose radiotherapy. Conversely, leukaemia and thyroid cancers are seen after much lower exposures to ionising radiation. Other tissues within a radiation field e.g. breast, thyroid and skin are also at risk. D :

Page 252: Masterclass Book Part 2

MOHAMMED IS-HAG 251

20. A 52-year-old man is admitted with a history of bleeding per rectum, tenesmus and weight loss. His father died of colorectal carcinoma aged 43 years and two of his siblings died of colorectal carcinoma aged 36 and 37 years of age. An older sister had died aged 64 from thyroid carcinoma. Barium enema showed an apple core lesion in his sigmoid colon in keeping with carcinoma, also multiple polyps throughout the colon and rectum. Which of the following statements is correct? A : A diagnosis of familial adenomatous polyposis (FAP) is unlikely. B : FAP is a poorly penetrant autosomal recessive condition . C : There is an increased incidence of papillary carcinoma of the thyroid in FAP. D : Duodeal adenomas do not develop in FAP. E : There are no extraintestinal features of FAP. Comment : FAP is a highly penetrant autosomal dominant condition in which multiple adenomatous polyps develop throughout the colon and rectum during adolescence. Polyps increase the risk of colorectal cancer, which usually develops by age 40. Duodenal adenomas are found in almost all FAP patients and are pre-malignant. There is an increased risk of extraintestinal tumours including brain tumours and papillary carcinoma of the thyroid. Extraintestinal features of FAP include multiple subcutaneous cysts and osteomas (Gardner Syndrome). C : 21. Which one of the following interpretations of physical signs is correct ? A : The presence of bloody discharge from the breast invariably suggests an underlying malignancy

that requires further investigation. B : The clinical assessment of axillary lymphadenopathy is inaccurate with 75% false-negative and

50% false-positive findings. C : The presence of pain is most common in the diagnosis of breast cancer and thus a pathological

specimen should be taken from the site of discomfort. D : Cyclic breast pain is usually due to perimenopausal hormonal surges and is associated with an

underlying endocrine sensitive tumour. E : Fibrocystic disease appears to blend into the surrounding tissue during palpation, whereas

cancerous masses usually have well defined borders and are fairly fixed to the rest of the glandular stroma.

Comment : · The most common cause of bloody nipple discharge is a benign papilloma, although an underlying malignancy must be excluded. · The palpation of axillary lymph nodes is very inaccurate, but the true figures from a large clinical trial are 38% false-negative and 25% false-positive. · Pain is an uncommon presenting feature of malignancy and is usually due to benign disease. · Cyclical breast pain during the perimenopausal period is invariably benign. · Malignant breast lesions normally have a well defined margin, as opposed to benign fibrocysic disease which tends to blend into the surrounding breast stromal tissue. Fibroadenomas are well defined but encapsulated and highly mobile when palpated. E : 22. Which of these factors is not associated with psychological morbidity in cancer patients? A : History of mood disorder B : History of alcohol misuse C : Poor social support D : Disfiguring tumour E : High expectation of successful treatment outcome. Comment : Psychological distress is frequent in patients with cancer and is often overlooked or even deliberately neglected by clinicians. However, over the last few decades, more oncologists have appreciated that psychological distress and psychiatric disorders such as anxiety, depression and delirium (in hospitalised patients) are frequent co-morbid conditions. Increasingly the outcome measures in clinical trials of new therapies have included quality of life evaluation and not just assessed survival end-points. A number of factors have been found to be associated with an increased risk of psychological distress in patients with cancer. Clinical features of anxiety include: · anorexia · fatigue · loss of libido · weight loss · anhedonia · insomnia · suicidal ideation. Many of these key symptoms are at times attributed to the cancer, and as few as one third of cancer patients who might benefit from antidepressants are prescribed them. E :

Page 253: Masterclass Book Part 2

MOHAMMED IS-HAG 252

23. In a patient with neutropenic sepsis, which of the following is not part of the initial assessment? A : Examination of the mouth and throat B : Midstream urine (MSU) C : Swabs of line exit site D : Digital rectal examination E : Chest radiograph. Comment : A rectal examination could introduce bowel organisms into the circulation. 24. The prognosis of acute myeloid leukaemia (AML) depends on many features. Which of the following has the greatest influence on prognosis ? A : Age B : French-American-British (FAB) classification C : Karyotype D : Chemotherapy regimen E : Bone marrow transplantation in first remission. Comment : The Cancer and Leukaemia Group B (CALGB) have determined that for patients aged below 60, high dose cytarabine given after induction can double the likelihood of disease-free survival compared to standard intensification. Immunotherapy has no established role in AML, nor dose maintenance therapy. Certain karyotypes have more favourable prognoses, with improved long-term disease-free survival. These include genetic alterations such as translocations t(8;21) and t(15;17) and an abnormal 16q22. In contrast, all other karyotype abnormalities confer an adverse prognosis. For those patients with AML who appear to have a normal diploid karyotype, the prognosis is somewhere in between. Although, there is some correlation of karyotype with morphology, the association is weak. Karyotype outweighs age in defining prognosis. Bone marrow transplantation is not significantly superior in first remission and the results are significantly influenced by karyotype. C : 25. A 28-year-old woman presents with an enlarging mole on her calf. Excision biopsy shows a superficial spreading melanoma of 0.5mm thickness. Subsequent treatment should be: A : Wide excision and adjuvant immunotherapy B : Wide excision with 1 cm margin C : Wide excision and sentinel node biopsy D : Wide excision and elective inguinal node dissection E : Wide excision with 5 cm margin. Comment : This is a low risk melanoma. Excision margins greater than 1-2 cm have no advantage and the role of adjuvant immunotherapy is debatable even in high risk patients, although it may prolong disease-free survival. Sentinel node biopsy is in clinical trial for thicker melanomas. B : 26. Which of the following factors confers the highest risk of developing ovarian cancer? A : Oral contraceptive B : Obesity C : 1-2 children D : Nulliparity E : Overexpression of HER-2 oncogene. Comment : Consistently identified risk factors for ovarian cancer include: · a positive family history · few or no children · older age at first marriage and childbirth. There is a protective effect for childbearing and use of the oral contraceptive pill. There is some evidence that patients with HER-2 over-expression have a worse response to chemotherapy and a worse prognosis, but the reported studies are retrospective only and there is no evidence that the gene is a risk factor for the development of ovarian cancer. Risk factor Relative risk General population 1 Family history 17-50 Nulliparity 4-5 1-2 children 3 Obesity 2 Oral contraceptive 0.5 D : 27. Radiotherapy to the mediastinum is most likely to cause which of the following complications? A : Coronary artery disease with myocardial infarction as a late complication B : Severe mitral incompetence due to destruction of the anterior mitral leaflet C : Constrictive pericarditis as a late complication D : Congestive cardiomyopathy due to primary myocardial (myocyte) destruction E : Acute pericarditis during treatment or during the early post-treatment period.

Page 254: Masterclass Book Part 2

MOHAMMED IS-HAG 253

Comment : Radiotherapy at high doses will affect all cardiac structures and any of the listed complications could occur. However, most of the listed features require a high local dose to occur. Some degree of pericarditis is seen following radiation therapy and may manifest with low-grade fever, pericardial friction rub, chest pain, and ECG changes. Most patients respond to non-steroidal medication. E : 28. A 41-year-old male presented with a node in his left lower neck region. A biopsy of the node revealed Hodgkin's lymphoma, nodular sclerosis type. Subsequently a mediastinal mass was found. Prior to Gallium scintigraphy he was initially staged as IIA; however, this revealed addition gallium avid tumours in the left para aortic region, and a small focus in the splenic hilar region. 67Gallium (67-Ga) scanning has been used in malignant lymphoma treatment for all but which of the following: A : Staging at diagnosis B : Detection of relapse C : Screening at risk populations D : Evaluation of response to therapy E : Predicting prognosis. Comment : 67-Ga scintigraphy has been used for more than 20 years in the detection of malignancies. Its exact role in malignant lymphoma has been disputed, mainly because of the high false-negative rate. The role of 67-Ga scintigraphy in lymphoma relies on its ability to detect viable residual disease with or without residual radiographic abnormalities and conversely to identify patients with no viable disease whose radiographic studies are still abnormal. This depends on high 67-Ga avidity in untreated tumours, which vary with histological subtype and anatomic distribution of the disease. C : 29. A 75-year-old man with a 30-pack year cigarette smoking history complains of continuous right shoulder pain, a persistent cough and weight loss. His chest radiograph shows a right apical shadow. On examination you note that he is clubbed, has a small right pupil and a right-sided ptosis. What is the most likely diagnosis? A : Small cell lung cancer B : Squamous cell carcinoma C : Bronchoalveolar carcinoma of the lung D : Adenocarcinoma of the left lung E : Bronchial carcinoid. Comment : It is likely that he has a right apical carcinoma of the lung causing a Horner’s syndrome (ipsilateral ptosis, meiosis and anhydrosis). Small cell lung cancer generally arises centrally as opposed to the more peripheral lesions of non-small cell lung cancer. Squamous cell carcinoma accounts for about 30% of all lung cancers and arises most frequently in proximal segmental bronchi. Most of adenocarcinomas of the lung are peripheral in origin. The tumour in this case is on the right and not the left. B : 30. Which sentence best describes the evidence-based treatment options in inoperable pancreatic cancer? A : In locally advanced disease, radiotherapy alone has demonstrated a survival benefit B : In locally advanced disease, radiotherapy, combined with chemotherapy, has demonstrated a

survival benefit C : In locally advanced disease, radiotherapy alone has demonstrated an objective benefit in terms of

quality-of-life D : In inoperable pancreatic cancer, 5FU based chemotherapy (either alone or in combination) has

demonstrated a survival benefit E : In inoperable pancreatic cancer, Gemcitabine-based chemotherapy has demonstrated a survival

benefit. Comment : Although, radiotherapy and chemo-radiotherapy are often sited as improving symptoms in locally advanced pancreatic cancer, there is a lack of objective evidence from phase III randomized trials. In the early 1980s, the American gastrointestinal study group Trials (GITSG), were closed early due an increased mortality with radiotherapy alone. The more recent ESPAC-1 trial suggest that quality-of-life deteriorates with radiotherapy, if used in the adjuvant setting. Again the evidence from chemotherapy trials in the pre-gemcitabine era are disappointing, with no consensus benefit from any regimen. There are two randomized trials with gemcitabine showing a significant survival benefit of similar magnitude, one versus 5FU and the other a metalloproteinase inhibitor (both of which can be thought of as a placebo in this context, as the control arms in both trials had never shown a benefit previously!). E : 31. Which of the following is NOT associated with an increased risk of osteosarcoma? A : Osteogenesis imperfecta B : Maffucci’s syndrome C : Gorlin’s syndrome D : Li–Fraumeni syndrome E : Hereditary retinoblastoma.

Page 255: Masterclass Book Part 2

MOHAMMED IS-HAG 254

Comment : Gorlin’s syndrome (basal cell naevus syndrome) is associated with basal cell carcinomas, ovarian fibroma, medulloblastoma, falx calcification, bifid ribs and macrocephaly. Maffucci’s syndrome is associated with enchondromas, osteosarcoma, cutaneous haemangioma, and skin pigmentation. C : 32. The following cytotoxics all penetrate the cerebro-spinal fluid, except: A : Dacarbazine B : 5 Fluorouracil C : Hydroxyurea D : Paclitaxel E : Capecitabine. Comment : A limited number of cytotoxics penetrate the CSF (nitrosoureas, dacarbazine, capecitabine, 5FU, hydroxyurea); however, these are not the only agents responsible for neurotoxicity. D : 33. A 63-year-old male non-smoker develops a dark purple nodule on his cheek. Immunohistochemistry of the biopsy is positive for neurone specific enolase. Which is the most likely diagnosis? A : Metastasis from Small cell lung cancer B : Melanoma C : Merkel’s cell cancer D : Kaposi’s sarcoma E : Mycosis fungoides. Comment : Merkel's cell cancer is the most likely diagnosis. C : 34. A 70-year-old man develops night blindness, ring scotomas and photosensitivity due to anti-recoverin antibodies. The most likely underlying pathology is: A : Small-cell lung cancer B : Multiple sclerosis C : Temporal arteritis D : Lyme disease E : Colon cancer. Comment : The most common causes of paraneoplastic retinopathy are SCLC and melanomas. A : 35. In what percentage of neutropenic patients is the mouth the identified source of infection? A : <5% B : 10% C : 20% : C Comment : The mouth is the most identifiable source of systemic bacterial infection in patients that are neutropenic and can be identified in about 20% of cases. C : 36. A 55-year-old man presents with haematuria, loin swelling and discomfort. A CT scan shows a large renal mass with tumour in the renal vein but not the inferior vena cava, and bilateral small lung metastases. What is the most appropriate treatment option? A : Immunotherapy B : Radical nephrectomy and medroxyprogesterone acetate C : Chemotherapy D : Renal artery embolisation and immunotherapy E : Radical nephrectomy and immunotherapy. Comment : In good performance status patients with limited metastatic disease, nephrectomy plus immunotherapy offers longer survival than immunotherapy alone. E : 37. Which of the following cancers is most likely to produce bone metastases that are osteoblastic rather than osteolytic? A : Choriocarcinoma in either gender B : Endometrial adenocarcinoma C : Granulosa cell tumour of the ovary D : Serous carcinoma of the ovary E : Prostate adenocarcinoma. Comment : Metastastic prostate cancer can cause osteolytic bone lesions, but is more likely to cause a sclerotic reaction due to osteoblastic activity. The remaining cancers listed are less likely to involve bone as part of the natural history of the disease, but when they do, it is more likely to be with an osteolytic reaction. E : 38. A 65-year-old man with prostate cancer is concerned that occupational exposure may have contributed to his cancer. Which of the following is known to increase the risk of prostate cancer? A : Aluminium B : Asbestos C : Cadmium D : Petrol E : Lead.

Page 256: Masterclass Book Part 2

MOHAMMED IS-HAG 255

Comment : Asbestos is most commonly associated with mesothelioma, but lung cancer is also at increased incidence following exposure to fibres. Aluminium does not directly increase the risk of cancer but workers in aluminium foundaries are exposed to higher levels of polycyclic hydrocarbons which can result in lung cancer. Petrol is manufactured from benzene substrates which are associated with acute leukaemias. C : 39. Which one of the following types of intestinal polyps is NOT considered precancerous? A : 4 cm adenoma B : Villous adenoma C : Tubular adenoma D : Tubulovillous adenoma E : Hyperplastic polyp Comment : Hyperplastic polyps of the bowel are not precancerous. Approximately 1% of villous adenomas less than 1 cm will have malignant cells, but this increases 10-fold for larger lesions and may even be 30% for lesions greater than 2 cm. Approximately 10% of all malignant colon polyps are villous adenomas. The tubular adenomas are usually sessile and present with rectal bleeding, mucous discharge or secretory diarrhoea. Approximately 65% of villous adenomas contain malignant cells and 50% of those are invasive. The risk of malignant cells in a tubulovillous adenoma is greater than in a tubular adenoma but less than that in villous adenoma. E : 40. Patients who have received mantle radiotherapy for Hodgkin's Disease are not at increased risk of: A : Skin cancer B : Infertility C : Breast cancer D : Hypothyroidism E : Osteoporosis. Comment : Mantle radiotherapy treats all lymph node areas above the diaphragm and the gonads do not receive a significant dose. Although it is less widely used in Hodgkin's disease now, many patients will continue to be at risk of complications. B : 41. A 29-year-old right handed woman presents with a five year history of a slowly growing mass on the wrist of her left hand that was firm and mobile. The histological diagnosis was Schwannoma. Which of the following is true of Schwannomas? A : They cause local invasion of nerve resulting in neurological deficit. B : Left in situ, 80% will undergo malignant transformation. C : They cannot be resected without removal of nerve fascicle. D : They cannot be distinguished from neurofibromas by MR scanning. E : They are always multiple. Comment : These relatively rare benign tumours are nonetheless the most common tumour of the peripheral nerves, and may affect any nerve in the body. They arise from the sheath surrounding the nerve (Schwann cells). In Schwannomas, cells are not organized like Schwann cells around the axons but instead in swirls and streaks across and among themselves. Schwannomas do not invade the nerve and can usually be surgically removed without damage to the adjacent nerve. Schwannomas usually grow very slowly but they can become very large. Very large tumours can be seen or felt as a bulge under the skin. Schwannomas (like neurofibromas) can occur singly or multiply. Multiple Schwannomas can occur on many different nerves or be limited to only one nerve or to one extremity. Schwannomas (and neurofibromas) are seen in association with Neurofibromatosis (‘von Recklinghausen’s disease’). Schwannomas and neurofibromas look very similar on MR. Although Schwannomas are benign (they do not metastasise), a small number (5-15% quoted in the literature) are more aggressive and some can even become malignant (malignant transformation) and very rarely metastasise. Watching with serial imaging and clinical examination is a reasonable approach but surgery may be best if there is any increase in size or more aggressive appearance. Schwannomas can often be removed with little injury to the nerve that they grow on, as opposed to neurofibromas whose removal usually requires division of the nerve around the tumour. D : 42. A 35-year-old woman presents with a 1 cm diameter lump in her left breast, which needle biopsy reveals to be invasive ductal carcinoma. What should her initial treatment be? A : Wide local excision and axillary node sampling followed by adjuvant breast radiotherapy in all cases B : Wide local excision and axillary node sampling followed by adjuvant breast radiotherapy if there

is histological spread to the axillary nodes C : Mastectomy and axillary node sampling followed by adjuvant breast radiotherapy in all cases D : Mastectomy and axillary node sampling followed by adjuvant chemotherapy if there is

histological spread to the axillary nodes E : Wide local excision and axillary node sampling followed by adjuvant breast radiotherapy and

adjuvant chemotherapy if there is histological spread to the axillary nodes.

Page 257: Masterclass Book Part 2

MOHAMMED IS-HAG 256

Comment : The standard treatment of early breast cancer is wide local excision and axillary node sampling followed by adjuvant breast radiotherapy. This achieves similar local control and survival rates to mastectomy with less mutilating surgery.Women without axillary node involvement can be divided into a high risk group (tumour >1 cm or does not express the Estrogen Receptor, ER), who should be given adjuvant treatment, and a low risk group (smaller ER-positive) who do not require adjuvant treatment.Women with histological spread to axillary nodes should receive adjuvant chemotherapy, which delays recurrence and improves survival. A : 43. A patient with lymphoma excretes 1.5g urinary protein but has a negative dipstick test. The reason for this seeming inconsistency is: A : dipsticks preferentially detect albumin rather than immunoglobulin because albumin is negatively charged B : the molecular weight of the excreted protein is too low to be detected by dipsticks C : Tamm-Horsfall proteins block the reaction of the secreted protein and the dipstick D : the urine is not sufficiently concentrated E : dipsticks only recognise heavy chain sequences. Comment : A normal person excretes up to 150 mg of protein per day which is chiefly Tamm-Horsfall mucoprotein. Dipsticks can detect as little as 50 mg protein per litre. However, a false negative result occurs with immunoglobulins which are positively charged. A : 44. A 20-year-old man presented with right leg pain of one year's duration. There is no history of trauma. The pain was worse at night but was fully relieved by aspirin. Which of the following features does not support a diagnosis of osteoid osteoma? A : Focal sclerotic lesion in tibia with periosteal reaction B : Male gender C : Age distribution usually 7-25 years D : Pain worse after alcohol and exercise E : Normal 99Tc-diphosphonate bone scan. Comment : Osteoid osteoma is characteristically found in the femur, tibia, talus spine and humerus, usually in the diaphysis or metaphysis of these bones. This is a benign bone tumour with a central small nidus of osteoid which incites a vigorous reaction in surrounding tissue. The common presentation is one of pain unrelated to activity or most pronounced after drinking alcohol. The pain is often relieved with non-steriodals. Osteoid osteoma is most common between 7 and 25 years of age and is three times more common in males. E : 45. A 38-year-old woman presents with abdominal distention and is found to have ovarian cancer. The method of staging ovarian cancer relies primarily on: A : CA-125 B : CT scanning C : Surgery D : Pelvic or transvaginal ultrasound E : Clinical examination Comment : The staging of ovarian cancer is primarily based upon the surgical findings following an exploratory laparotomy. Ovarian cancer is often confined to the peritoneal cavity and therefore it can only be accurately staged with a surgical procedure, not only to remove the primary tumour, but also to biopsy multiple areas to determine the true extent of disease. Peritoneal washings are also taken. Accurate staging is crucial as patients that have disease truly confined to one or both ovaries can avoid adjuvant chemotherapy. C : 46. Which of the following is associated with an increased risk of endometrial cancer? A : Polycystic ovaries B : Cigarrette smoking C : Oral contraceptive pill D : High parity E : Early menopause. Comment : Risk factors for endometrial cancer largely relate to prolonged exposure of the endometrium to circulating estrogen in the absence of differentiating progesterone. They include obesity, menstrual irregularity including long periods of anovulation and prolonged menstrual span. Exogenous non-cyclical estrogens and Tamoxifen also increase the risk A : 47. Bacterial infections are common in patients with neutropenia. Which of the following organisms is the least likely to be isolated from a neutropenic patient that is febrile? A : Staphylococcus aureus B : Pseudomonas aeruginosa C : Streptococcus viridans D : Enterococcus spp. E : Stenotrophomonas maltophilia

Page 258: Masterclass Book Part 2

MOHAMMED IS-HAG 257

Comment : There are many organisms that can cause infections during periods of neutropenia during chemotherapy. The majority of organisms are gram positive (50-70%). Stenotrophomonas maltophilia accounts for <5% of all infections and is the least common. The more common gram-negative organisms include: Escherichia coli, pseudomonas aeruginosa and klebsiella species. E : 48. Which one of the following tumour markers have a role in screening? A : Carcinoembryonic antigen for gastric cancer B : Human chorionic gonadotrophin for pure seminoma C : Calcitonin for medullary cell thyroid cancer D : Beta-2 microglobulin for myeloma E : Thyroglobulin for follicular thyroid cancer. Comment : Tumour markers are proteins produced by cancers that are detectable in the blood of patients. Tumour markers may be used for: 1. Population screening 2. Diagnosis 3. Prognostic factors 4. Monitoring treatment and the diagnosis of remission and relapse. C : 49. A 28-year-old woman reports that several of her family members have died of breast cancer, including her mother at age 43, her maternal aunt at age 62 and her maternal grandmother at age 65. What is the probability that this patient will develop breast cancer ? A : 5 % B : 8 % C : 20 % D : 30 % E : 50 % Comment : The risk of developing breast cancer in the general population is approximately 8%. Patients that have a strong family history of breast cancer are at a higher risk, and this is especially true if one of the family members was young at the time of their presentation. The overall relative risk of breast cancer in a woman with an affected first-degree relative is 1.7. Premenopausal onset in a first-degree relative is associated with a three-fold increase in relative risk, whereas postmenopausal diagnosis increases the relative risk by only 1.5. No increase in risk has been demonstrated when only a second-degree relative (aunt, cousin, grandmother) is affected. The risk in this patient is approximately three-fold higher than the general population.Some inherited breast cancers have been found to be associated with a gene called BRCA1 on chromosome 17. This gene is mutated in some families with early-onset breast cancer and ovarian cancer. It is estimated that about 85% of women with BRCA1 gene mutations will develop breast cancer in their lifetime. Other genes have been identified that are associated with increased risk of breast and other cancers, such as: · BRCA2 · ataxia-telangiectasia mutation · p53. p53 mutations have been found in approximately 1% of breast cancers in women under 40 years of age. Genetic testing is available for women at high risk of breast cancer. However, such testing is controversial, as problems associated with management of patients with identified mutations, their insurability, and potential social conflicts are anticipated. C : 50. A 76-year-old woman with metastatic breast cancer developed nausea and vomiting. She was known to have bone and liver metastases and her pain was well controlled on a stable dose of slow release morphine sulphate for the last 6 months. She was also on diuretics and digoxin for chronic heart failure and atrial fibrillation and her GP had prescribed antibiotics 3 days previously for a respiratory tract infection. Which of the following is least likely to be contributing to her nausea & vomiting? A : Hypercalcaemia B : Uraemia C : Opiates D : Erythromycin E : Squashed stomach secondary to liver metastases. Comment : Side effects of opiates: · Constipation: This affects almost all patients and all patients require prophylaxis with a stimulant laxative (e.g. senna, bisacodyl) and a softener (e.g. docusate sodium) or as a combined preparation (e.g. codanthramer, codanthrusate) · Drowsiness: Generally remits after a few days · Nausea: Affects one third of opioid-naïve patients but usually resolves within one week. Consider prophylaxis for 1 week · Hallucinations: An uncommon side effect that often features images in the peripheral vision · Nightmares: Vivid and unpleasant but rare · Myoclonic jerks: Occur usually with excess doses and may be mistaken for fits · Respiratory depression: Not a problem in patients with pain. C :

Page 259: Masterclass Book Part 2

MOHAMMED IS-HAG 258

51. A 58-year-old man with extensive stage small cell lung cancer is enrolled onto a phase I trial of carboplatin, etoposide and topotecan chemotherapy. What is the principal aim of a phase I study? A : Establish the maximum tolerated dose B : Determine drug interactions C : Compare results with gold standard treatment D : Normal volunteer study for drug toxicity E : Determine tumours types that respond to the therapy. Comment : Experimental therapies: · Phase I studies determine the relationship between toxicity and dose schedules of treatment · Phase II studies identify tumour types for which the treatment appears promising · Phase II studies assess the efficacy of treatment compared to standard treatment including toxicity A : 52. A 44-year-old woman is HIV positive. She is very concerned that she is at increased risk of cancer. Which one of the following cancers is NOT increased in HIV positive people? A : Hodgkin’s disease B : Anal cancer C : Non-small cell lung cancer D : Breast cancer E : Seminoma. Comment : The only cancer that is consistantly reduced in incidence in cohort studies of HIV positive patients is breast cancer. The other malignancies listed are all increased in incidence. D : 53. You are asked to see a 52-year-old woman with metastatic breast cancer in clinic who is tearful and feels that she can no longer cope. What would be the most appropriate course of action? A : Screen for depression / suicidal thoughts and liaise with primary care team B : Prescribe antidepressants C : Reassure her that these feelings are normal in this situation D : Listen to her concerns and refer her for counselling E : Refer her for psychiatric assessment. Comment : Depression is common in patients with advanced cancer (20-25% incidence) and can respond to treatment. Diagnosing depression with screening tools such as the Hospital Anxiety and Depression scale (HAD) or basic clinical interview is within the capabilities of all doctors. In a clinic setting you should screen for depression, which includes listening to her concerns. If her feelings overwhelm her it is important to ask if she has thought about taking her own life. Studies suggest that patients don’t often volunteer this but appreciate being asked and it does not ‘put ideas into their head’. Good communication with the primary care team about appropriate management, that might include antidepressants, is vital. A : 54. A 78-year-old lady with carcinoma of the ovary is admitted with a 4-day history of constipation associated with colicky abdominal pain and vomiting. Which of the following would you do? A : Prescribe a laxative and paracetamol B : Order a plain abdominal radiograph C : Prescribe oramorph and metoclopramide D : Arrange an abdominal ultrasound E : Give a stat dose of diamorphine F : Prescribe hyoscine butylbromide and cyclizine orally G : Seek a surgical opinion H : Insert a nasogastric tube I : Start a syringe driver with diamorphine and metoclopramide J : Start a syringe driver with diamorphine and cyclizine. Comment : It sounds as if she has developed bowel obstruction A plain abdominal radiograph will help exclude constipation as a cause. It is difficult to predict how effectively she is absorbing oral medication and it should therefore be administered by alternative routes such as via a syringe driver. Metoclopramide is a prokinetic agent and can make intestinal colic worse. Cyclizine is the antiemetic of first choice. Avoid a nasogastric tube if possible but it may give symptomatic relief if vomiting is profuce. Hyoscine butylbromide will help abdominal colic but should be given subcutaneously. Carcinoma of the ovary may cause multiple sites of obstruction and it is probably best to investigate first before seeking a surgical opinion. Also the patient’s fitness for surgery needs to be taken into consideration. B:J:

Page 260: Masterclass Book Part 2

MOHAMMED IS-HAG 259

55. A 56-year-old man with metastatic carcinoma of the prostate has an epidural nerve block for neuropathic leg pain. Following this, he is told to stop morphine sulphate continus (MST) but to take some Oramorph should he develop any symptoms of opioid withdrawal. These include: A : sweating B : abdominal pain C : dry mouth D : myalgia E : drowsiness F : cramps G : headache H : myoclonus I : hallucinations J : urinary retention. Comment : Symptoms of opioid withdrawal are often likened to 'cold turkey' i.e. shivering, sweating, lacrimation and rhinorrhea. Patients feel generally unwell and 'fluey' with muscle aches, nausea and vomiting and diarrhoea. A:D: 56. A 45-year-old man is dying from non-Hodgkin’s lymphoma. He was distressed and mildly agitated prior to his diamorphine dose being doubled. You are asked to see him as his agitation has worsened. Which of the following are the likely causes? A : Opiate toxicity B : Pain C : Urinary retention D : Hypercalaemia E : Hyponatraemia F : Septicaemia G : Acute confusional state H : Terminal agitation I : Constipation J : Dehydration. Many of the above can cause agitation and it is important to exclude those that are remediable. Many patients become agitated due to general distress at their predicament. Non-specific distress at end of life is called terminal agitation. Opiate overdose can be extremely unpleasant and rather than cause sedation leads to hallucinations and myoclonic jerks. Opiates should not be used as sedatives. A:H: 57. A 58-year-old woman with metastatic renal cancer presents with severe right leg pain radiating from her buttock to her toes. A recent CT scan demonstrates epidural invasion at L1-L4. The pain is described as sharp, tingling and she has altered sensations in her lower leg. There has been little response to 20mg bd of morphine. What initial approaches should be tried to control her pain? A : Arrange radiotherapy B : Titrate morphine dose C : Give intravenous bisphosphonate D : Arrange urgent intrathecal infusion E : Add in paracetamol F : Ask a counsellor to talk with her G : Add in amitriptyline or gabapentin H : Change to fentanyl patch I : Refer back to oncologist for further chemotherapy J : Set up diamorphine syringe driver Comment : Titrating opioids and adding in co-analgesic drugs for neuropathic pain are likely to produce initial improvements in her pain. Intrathecal infusions for neuropathic pain should be tried after standard treatment has failed. Further oncological management (radio or chemotherapy) is unlikely to benefit the patient in the short term. Topical treatments (fentanyl patch) are best used in stable pain situations because of the long dosing intervals. Parenteral approaches (syringe driver) are unlikely to offer better analgesia if the patient is able to absorb oral medicines. Bisphosphonates and paracetamol are appropriate approaches for bone pain rather than neuropathic pain. B:G: 58. A 58-year-old woman presents with left chest wall pain after a coughing fit. Eight years ago she had a mastectomy, adjuvant chemotherapy and endocrine therapy and chest wall radiotherapy of a T2N1 cancer of the left breast. Her bone scan now shows a solitary hot spot in the anterior left 5th rib. Which one of the following statements is INCORRECT? A : A follow-up bone scan in 6 months is indicated. B : The hot spot is likely to be a fracture from radiation induced osteoporosis. C : The hot spot is likely to be a radiation induced sarcoma. D : The hot spot is likely to be a fracture from metastatic disease. E : A raised alkaline phosphatase would be helpful in the differential diagnosis. Radiation-induced sarcomas are very rare. They occur from 7 years onwards after radiotherapy. C:

Page 261: Masterclass Book Part 2

MOHAMMED IS-HAG 260

59. A 45-year-old man is dying from non-Hodgkin’s lymphoma. He is increasingly agitated and distressed. He is not obviously in pain and has a urinary catheter in situ which is not causing specific distress. He has diamorphine 30mg in his syringe driver. You are asked to review him. What do you suggest? A : Increasing the diamorphine dose B : Adding hyocine hydrobromide C : No changes to his medication D : Reducing the diamorphine dose E : Adding midazolam 10mg to the syringe driver F : Giving a stat dose of midazolam G : Changing diamorphine to fentanyl H : Adding levomepromazine 75mg to the syringe driver I : Giving stat dose of levomepromazine 25mg J : Giving PR diazepam. Comment : The patient requires sedation rather than analgesia. A stat dose of either midazolam or levomepromazine will allow you to calculate the 24-hour dose needed for the syringe driver.F:I: 60. A 78-year-old lady with carcinoma of the ovary is admitted with colicky abdominal pain and vomiting. A plain abdominal radiograph shows multiple fluid levels and no stool. She is on diamorphine in a syringe driver but the pain persists. Would you: A : give diamorphine prn B : add a fentanyl patch C : increase the dose of diamorphine in the syringe driver D : give a stat dose of midazolam E : add hyoscine hydrobromide F : insert a nasogastric tube G : give diazepam PR H : add hyoscine butylbromide to the syringe driver I : add midazolam to the syringe driver J : organise an abdominal ultrasound. Comment : This lady has intestinal obstruction and requires analgesia as opposed to sedation. Diamorphine prn will allow you to calculate how much extra diamorphine needs to be added to the syringe driver. Hyoscine butylbromide is an antispasmodic and will reduce abdominal colic. A nasogastric tube may help, particularly if there is persistent vomiting, but should be avoided if possible. Further investigations of the obstruction would only be relevant if the patient were fit enough for surgery.A:H: 61. A 4-year-old boy presents with a 3 week history of headache, ataxia and vomiting. A head CT shows a posterior fossa mass that arises from the midline cerebellum involving the fourth ventricle. The most likely pathology is: A : haemangioblastoma B : oligodendrocytoma C : medulloblastoma D : craniopharyngioma E : glioblastoma multiforme. Comment : Oligodendrocytomas arise from oligodendrocytes in the white matter of the cerebral hemispheres and occurs in young and middle aged adults. Craniopharyngiomas arise in a suprasellar location. Glioblastoma multiforme, the highest grade of glioma, occur in the cerebral hemispheres of older adults. haemangioblastoma is a cyctic tumour associated with adults with von Hippel-Lindau. Medulloblastomas arise from the cerebellar vermis in children and less commonly in the cerebelar hemispheres of adults. C: 62. A patient with poorly controlled pain on full dose coproxamol requires a step up the World Health Organisation (WHO) analgesic ladder to strong opiates. Which of the following is NOT a strong opiate? A : Sevredol B : Fentanyl C : Oxycodone D : Codeine E : Hydromorphone. Comment : Strong opiates include: morphine, diamorphine, hydromorphone, methadone, oxycodone and fentanyl. D:

Page 262: Masterclass Book Part 2

MOHAMMED IS-HAG 261

63. A 48-year-old man presents with fever, weight loss, lymphadenopathy and hepatosplenomegaly. Which one of the following clinical features would NOT be consistent with the diagnosis of multicentric Castleman’s disease? A : Pyrexia B : Polyclonal gammaglobinaemia C : Splenomegaly D : Paraneoplastic pemphigus E : Cardiomyopathy. Comment : Castleman’s disease is a rare lymphoproliferative disorder characterised by angiofollicular lymphoid hyperplasia. Two histological variants are recognised; a hyaline vascular variant and a less common plasma cell variant. Castleman’s disease has germinal centre hyalinisation or atrophy surrounded by concentric layers of lymphocytes with prominent vascular hyperplasia, hyalinisation of small vessels and interfollicular sheets of plasma cells and immunoblasts. Multicentric Castleman’s disease (MCD) presents with constitutional symptoms including fever, weight loss and night sweats and clinical findings include lymphadenopathy, hepatosplenomegaly and rashes. Investigations frequently reveal microcytic anaemia, hypoalbuminaemia and polyclonal hypergammaglobulinaemia. Many of the paraneoplastic manifestations of Castleman’s disease are believed to be due to excess interleukin-6 (IL-6) production by the tumour, possibly from the viral IL-6 homologue gene of HHV8. The best treatment for MCD is not known. E: 64. A 38-year-old woman has suffered from headaches and pains in her legs for many years. Despite thorough investigation, no physical basis for any of her pains has been established. They seem to be neuropathic in nature, and you talk with her about the wide variety of drugs that can be helpful in treating these sorts of difficult pain. Which two of the following are NOT potentially beneficial for neuropathic pain? A : Flecainide B : Clonidine C : Propranolol D : Carbamezeine E : Imipramine F : Gabapentin G : Haloperidol H : Amitriptyline I : Lignocaine. J : Dothiepin hydrochloride. Comment :The drugs used for neuropathic pain are either membrane stabilisers such as antiepileptics or antiarrhythmics,or antidepressants such norepinephrine and serotonin reuptake inhibitors.Beta adrenergic receptors and dopamine receptors are not involved in neuropathic pain transmission. C:G: 65. A patient present to her GP with a pigmented lesion on her foot. The patient states that the lesion was apparently present from birth and does not itch or bleed. However, it is not as homogenous as it used to be. Which one of the following is true? A : Early diagnosis would not affect prognosis B : It would be dangerous to perform an incisional biopsy of this lesion C : Bleeding and tenderness would be the first signs of malignant change D : Since the lesion has been present since birth, the risk of malignancy is low E : Change in colour is suspicious for malignant change. Comment : Melanomas develop from melanocytes that are derived from neural crest tissue that migrate to the skin, eye, central nervous system and occasionally elsewhere. The incidence is rising and has been shown to be related to sun-exposure. The vast majority arise from pre-existing benign naevi. Changes in size, colour or edge of a naevus or bleeding should alert to the possibility of melanoma. The three major signs are: · Change in size · Change in shape · Change in colour. The four minor signs are: · Inflammation, crusting or bleeding · Sensory change, such as itch · Diameter greater than 7mm. Melanoma is essentially a surgically treated disease. Biopsy for melanoma can be either excisional or incisional. Whichever is utilized, full thickness biopsy into the the subcutaneous tissue must be performed to permit micro-staging of the lesion (for thickness and level of the lesion). Early diagnosis, when the melanoma is confined to the epidermis (Clark’s level I), never metastasizes and has a 100% cure rate. E:

Page 263: Masterclass Book Part 2

MOHAMMED IS-HAG 262

66. A 38 yr old woman presents with abdominal distention and is found to have ovarian cancer. The method of staging ovarian cancer relies primarily on: A : CA-125 B : CT scanning C : Surgery D : Pelvic or transvaginal ultrasound E : Clinical examination Comment : The staging of ovarian cancer is primarily based upon the surgical findings following an exploratory laparotomy. Ovarian cancer is often confined to the peritoneal cavity and therefore it can only be accurately staged with a surgical procedure, not only to remove the primary tumour, but also to biopsy multiple areas to determine the true extent of disease. Peritoneal washings are also taken. Accurate staging is crucial as patients that have disease truly confined to one or both ovaries can avoid adjuvant chemotherapy.C: 67. A 78-year-old man with advanced cancer presents with painless pitting oedema of both legs. Which of the following diagnoses is least likely to apply? A : Cardiac failure B : Venous thromboembolism C : Lymphoedema D : Hypocalcaemia E : Hypoalbuminaemia. Comment : Cardiac failure and hypoalbuminaemia are common causes of this condition in terminally ill patients. Venous thromboembolism is thought to be present in up to 50% of terminally ill patients and usually presents with insidious painless swelling that may be associated with episodes of breathlessness. Lymphoedema, a protein rich exudate, is indistinguishable from other causes of oedema in the early stages. In time (weeks to months) the protein becomes organised and fibrosed and clinically becomes as 'non-pitting'.Disorders of calcium homeostasis are not commonly linked with oedema in this (or any other) context. D: 68.

A 16-year-old boy presents to his GP with a limp. He had a recent minor injury and now has a continuous dull ache in his thigh that occasionally wakes him at night. The right thigh is swollen, reddened and warm. A biopsy shows he has an osteosarcoma (see image). Which three of the following are good prognostic indicators? A : Age B : Male gender C : Non-extremity site D : Stage of the disease at presentation E : Percentage tumour necrosis to induction therapy F : A long duration of symptoms prior to presentation G : Raised alpha-fetoprotein tumour marker H : Raised placental alkaline phosphatase tumour marker I : Translocation of chromosomes 11 and 22 [t(11; 22)(q24; q12)] J : Raised serum acid phosphatase.

Page 264: Masterclass Book Part 2

MOHAMMED IS-HAG 263

Comment : Osteosarcomas are the most common primary malignancy of bone. Patient age and gender do not appear to be an independent prognostic factors. The impression that older patients fare worse may be attributed to the increased proportion of unfavourable axial lesions with increasing age. Pelvic and axial lesions are associated with lower survival rates than tumours of the extremities, probably owing to surgical inaccessibility and incomplete removal. Small tumour size (less than one third of the extremity bone), absence of metastases and high percentage tumour necrosis induced by induction therapy have been shown to be important prognostic factors. Serum alkaline phosphatase (ALP) is increased in 45-50% of patients, but is not diagnostic. It is an important marker of tumour activity in patients with osteosarcoma. Alpha fetoprotein is a glycoprotein that is produced by the liver, gastrointestinal tract and the yolk sac of the human fetus. It is raised in germ cell tumours (GCT) (80% non-seminomatous), hepatocellular carcinoma (HCC), neural tube defects and Down’s pregnancies. It has a role in screening in pregnancy but not in cancer. It is prognostic for GCT not HCC. Transient increases may be seen in liver diseases. Placental alkaline phosphatase (PLAP) is an isoenzyme of alkaline phosphatase and is raised in seminoma and ovarian dysgerminoma (50%). Primitive neuroectodermal tumours and Ewing’s sarcomas share a common translocation involving chromosomes 11 and 22 [t(11;22)(q24;q12)]. D:E: 69. A 58-year-old patient with metastatic carcinoma of the breast has good pain control on MST 180mg bd. She is admitted with increasing weakness and has difficulty swallowing her tablets. It is therefore decided to convert her to a 24-hour diamorphine syringe driver. The correct dose of diamorphine is: A : 30mg with 5mg prn. B : 60mg with 10mg prn. C : 60mg with 5mg prn. D : 120mg with 10mg prn. E : 120mg with 20mg prn. Comment : The dose of diamorphine should be 1/3 of the total 24-hour dose of morphine. The prn dose should be 1/6 of the 24 hour dose of diamorphine.E: 70. A GP rings you about a 55 year old lady with metastatic carcinoma of the breast who has become acutely confused. What advice would you give? A : Start the patient on high dose steroids B : Start the patient on diazepam C : Ask the GP to exclude hypercalcaemia D : Start the patient on antibiotics E : Offer to send out a Community Macmillan Nurse Comment : Hypercalcaemia can present as acute confusion and can only be diagnosed by a blood test. It can be treated with intravenous fluids and bisphosphonates. It is important to exclude infection as a cause of confusion. If she is very agitated an antipsychotic such as haloperidol would be more appropriate than diazepam. C: 71. A 50-year-old man with metastatic colorectal cancer complains of regurgitation of food and a feeling that it sticks retrosternally. Chest radiograph and upper gastrointestinal endoscopy are normal. Which antiemetic is most likely to be of benefit? A : Cyclizine B : Haloperidol C : Prochlorperazine D : Metoclopramide E : Ondanestron. Comment : It sounds as though there may be a neuro muscular cause for his symptoms. Another possibility is external compression, but the endoscopy did not show this. Metoclopramide is both an antidopaminergic and gastrokinetic agent. It may improve oesophageal motility. D:

Page 265: Masterclass Book Part 2

MOHAMMED IS-HAG 264

1. A 16-year-old male presents with short stature. He had a normal birth and normal growth and development until about the age of 11 years when he started to fall behind his peers. He is otherwise well. General examination is normal except that he is pre-pubertal (testes 3ml, Tanner stage G1, P1) with height below the 3rd centile. His bone age is 4 years behind his chronological age. His mid-parental height is on the 50th centile. The likely diagnosis is: A : Growth hormone deficiency B : Kleinfelter's syndrome C : Kallman's syndrome D : Constitutional delayed puberty E : Coeliac disease. Comment : Constitutional delay is common in boys and by far the commonest cause of short stature with delayed pubertal development. In contrast constitutional delay of puberty is relatively uncommon in girls and an underlying cause should be aggressively investigated. A diagnosis of constitutional delayed puberty requires there to be no pubertal development by the age of 14 in boys (13 in girls) and for the bone age to be at least 3 years less than the chronological age (showing that there is still potential for growth). The condition may not require treatment other than reassurance that puberty and increased growth will eventually occur along with follow-up to monitor growth. However, if the condition is causing distress, puberty can be accelerated by treatment with testosterone which primes the hypothalamo-pituitary-gonadal axis. Coeliac disease and other chronic illnesses can also cause delayed puberty, but this would be a less common cause and you might expect there to be other associated symptoms or signs.Growth hormone deficiency in childhood is much rarer than constitutional delayed puberty and presents with failure of linear growth at any age, rather than the characteristic normal growth followed by falling behind as peers undergo their pubertal growth spurt described above. Kleinfelter’s syndrome (47XXY) is associated with primary hypogonadism (i.e. testicular failure) whilst Kallman’s syndrome is characterized by secondary (hypogonadotrophic) hypogonadism with anosmia. Both conditions are generally associated with tall rather than short stature. D : 2. A 42-year-old gentleman with type 1 diabetes has read about a new type of insulin. Which of the following is not true concerning insulin glargine? A : Insulin glargine can be used as a basal insulin. B : Insulin glargine is less painful to inject than human insulatard. C : Insulin glargine is a clear insulin. D : Insulin glargine is licensed for both type 1 and type 2 diabetes. E : Insulin glargine is thought to cause fewer severe hypoglycaemic episodes than isophane insulin. Comment : Insulin glargine has been available in the UK since September 2002. It is produced by recombinant DNA technology. Two arginine residues have been added to the beta chain and a glycine substituted for asparagine in the alpha chain. This alters the isoelectric point from being acidic to a near neutral pH. Therefore glargine remains in solution in the acidic conditions of the vial or cartridge and precipitates in the neutral pH of subcutaneous tissues. It is therefore a clear insulin. These changes mean delayed absorption, a late onset of action and prolonged duration of action. Given as a single daily dose glargine produces a flat pharmacokinetic profile. Because of the acidity glargine can be more painful to inject than Insulatard. There is evidence showing that there is a reduced risk of hypoglycaemia with glargine than with traditional basal insulins. B : 3. A consultant colleague seeks an on-call review of one of her patients because the biochemist has phoned to say the patient may have a thyroid crisis. After reviewing the patient which of the following should you not prescribe? A : Carbimazole B : Aspirin C : Potassium iodide D : Propranolol E : Dexamethsaone. Comment : Thyroid crisis is a serious and potentially fatal disorder and needs emergency treatment. Antithyroid drugs should be given immediately and then 6-8 hourly. Carbimazole or propylthiouracil can be used. After an antithyroid drugs has been given iodide should then be given to prevent release of any stored thyroid hormone. Propranolol and dexamethasone will help decrease the conversion to T4 to T3. Propranolol also blocks the peripheral actions of T3. Although it would act as an antipyretic agent, aspirin should be avoided as it can displace thyroid hormones from thyroid binding globulin. B :

Page 266: Masterclass Book Part 2

MOHAMMED IS-HAG 265

4.

PLATE 44 A 47-year-old male is referred to the clinic with a 6-month history of lethargy and intermittent bouts of feeling ‘completely washed out’. His past medical history is unremarkable aside from a bout of thyrotoxicosis during his mid-20’s, which was successfully treated with a short course of anti-thyroid drugs. Based on the appearances shown in the image, which of the following blood tests is most likely to yield an abnormal result? A : Oral glucose tolerance test (OGTT) with measurement of growth hormone (GH) levels B : Thyroid stimulating hormone (TSH) C : Full blood count D : Corrected calcium E : Short synacthen test. Comment : The most striking abnormality shown is the presence of marked pigmentation within the palmer and digital creases. In the clinical setting, this should raise very strong suspicions regarding the possibility of primary adrenal insufficiency (Addison’s disease). Although thyroid dysfunction (either relapse of his thyrotoxicosis or subsequent development of hypothyroidism – both of which would be most likely to be of autoimmune origin) should also be considered, the clinical appearances would mandate that an assessment of adrenal reserve be carried out in any case – this is of particular importance if thyroxine replacement is being contemplated, given its potential to provoke an acute adrenal crisis if hypoadrenalism goes unrecognized. It would also be sensible to check for the presence of anaemia or hypercalcaemia in any subject with tiredness and lethargy, although the appearances in figure 1 specifically point to primary adrenal insufficiency. There are no features to suggest acromegaly as an underlying diagnosis. E : 5. A 77-year-old lady has been referred by her general practitioner with an eight-month history of constipation, lethargy and weight increase. Thyroid function tests reveal her to be hypothyroid. Which of the following statements is correct? A : The commonest cause of primary thyroid failure is Hashimoto's thyroiditis. B : The most discriminatory clinical feature of hypothyroidism is absent ankle jerks. C : In primary hypothyroidism cardiac evaluation is unimportant. D : An electrocardiogram will show high voltage changes with bradycardia. E : Triiodothyronine therapy rather than laevothyroxine is preferred in older patients. Comment : The commonest cause of primary thyroid failure is Hashimoto's thyroiditis. The most discriminatory clinical feature for hypothyroidism are slow-relaxing tendon jerks. In primary hypothyroidism cardiac evaluation is important, especially in middle aged and older patients. An electrocardiogram will classically show low voltage changes with bradycardia. Laevo-thyroxine is preferred to triiodothyronine because of its longer half life and greater chance of compliance with once a day treatment. A :

Page 267: Masterclass Book Part 2

MOHAMMED IS-HAG 266

6. A 58-year-old female presents to her GP with symptoms suggestive of bilateral carpal tunnel syndrome. He suspects that she has acromegaly and refers her to the endocrine clinic for further evaluation. Which of the following findings would support his provisional diagnosis? A : Random growth hormone (GH) level of 6mU/l B : GH nadir of 1.0mU/l during an oral glucose tolerance test (OGTT) C : Elevated insulin-like growth factor 1 (IGF-1) level D : Left homonymous hemianopia on formal visual field testing E : Elevated gonadotrophin (FSH, LH) levels. Comment : Whilst GH has some direct actions (e.g. antagonism of insulin), many of its effects are mediated through IGF-1, which is produced by the liver and, accordingly, in acromegaly IGF-1 levels are typically elevated above the age- and sex-related reference range. Unless the GH level is dramatically elevated, the diagnosis of acromegaly should not be made on the basis of a single random GH measurement - if sampling coincides with a normal physiological ‘ spike’ in GH secretion a false positive result may ensue. Indeed, even in the presence of an obviously elevated GH, the OGTT is still considered the ‘gold standard’ test for diagnosing this disorder – in normal subjects GH levels should fall to a nadir of <2mU/l (many actually suppress to <1mU/l). In contrast, in acromegaly not only do the levels fail to suppress adequately, but in many instances a paradoxical rise in GH is observed in response to a glucose challenge. The majority of cases of acromegaly (approximately 75%) arise in the setting of a pituitary macroadenoma (>1cm in diameter). These tumours may encroach on the optic chiasm, classically resulting in a bitemporal visual field defect. In addition, the remaining normal anterior pituitary tissue is often compressed – in these circumstances one might expect the gonadotrophin levels to fall within or below the reference range for younger females, which would of course be inappropriate in the post-menopausal state, where the gonadotrophins are typically elevated. 7.

PLATE 45 Which of the following statements about necrobiosis lipoidica diabeticorum (see image) is true? A : Lesions can occur on legs or arms. B : Lesions usually resolve spontaneously after 4-5 years. C : Topical steroids are of benefit. D : Lesions should usually be biopsied to confirm the diagnosis. E : It is more common in males than in females. Comment : Necrobiosis lipoidica diabeticorum usually affects the shins but may occur on arms or even on the trunk. The lesions are chronic and rarely resolve. There is no effective treatment. Topical or injected steroids are sometimes used but have not been shown to be of any benefit. Females are affected much more than males. These lesions are unrelated to microvascular complications. The lesions show an atrophic centre with dilated capillaries and a slight raised pinkish rim. Ulceration may occur in the centre. A :

Page 268: Masterclass Book Part 2

MOHAMMED IS-HAG 267

8. Which one of the following groups of patients with diabetes, according to the current guidelines, is suitable for continuous subcutaneous insulin infusion (CSII) via a pump? A : All women with diabetes during pregnancy B : Poorly controlled patients with type 1 diabetes who are unable to do their own injections due to

blindness C : Poorly controlled patients with type 1 diabetes who have compliance problems D : Patients with type 1 diabetes who are able to afford to buy the pump E : Young type 1 diabetes patients with failed mutiple-dose insulin (MDI) regimens. Comment : NICE guidelines suggest that CSII is recommended as an option for people with type 1 diabetes provided that multiple-dose insulin (MDI) therapy has failed and that those receiving it have the commitment and competence to use the therapy effectively. It is also recommended in children, adolescents, pre-pregnant and pregnant women for whom multiple dose insulin therapy is deemed to have failed. But, because of the risks of ketoacidosis to the foetus, pregnant and pre-pregnant women who switch to CSII therapy should do so only on the advice and under the care of a specialist team. All other groups of patients are excluded by the guidelines but for obvious reasons it will not be indicated in people with poorly controlled diabetes who have problems with compliance or have disabilities preventing correct operation of the pump, such as blindness. E : 9. Regarding fine needle aspiration cytology (FNAC) of a suspected thyroid lump, which of the following statements is true? A : With improvements in imaging techniques, FNAC is now rarely required in the investigation of a thyroid

lump. B : Two diagnostically benign cytology results, 3 to 6 months apart, exclude neoplasia. C : Non-diagnostic histology on FNAC should be followed by an open or core biopsy. D : Suspicious histology results necessitate repeat FNAC after 1 year. E : FNAC should always be performed under ultrasound guidance. Comment : According to National Guidelines for the management of thyroid cancer published in March, 2002, fine needle aspiration cytology (FNAC) should be performed in planning for surgery in patients with suspected thyroid lesions. Ultrasound guidance may be of value in aiding FNAC but is not mandatory. A surgeon or physician with reasonable expertise may do it without assistance from ultrasound guidance. Non-diagnostic histology results should lead to repeat FNAC. Two diagnostically benign cytology results from the lesion at least 3 to 6 months apart are deemed adequate to exclude malignancy. Suspicious cytology should be taken seriously and the patient referred for excision, as in the case of malignant cytology from FNAC. B : 10. A 70-year-old man presents to A&E drowsy and irritable. The following results are available: sodium 159 mmol/l; potassium 4.5 mmol/l; chloride 105 mmol/l; bicarbonate 29 mmol/l; serum urea 15 mmol/l; serum glucose 50 mmol/l. Urinary examination is negative for ketones. The following treatments are advisable EXCEPT: A : Subcutaneous heparin 5000iu twice daily B : Blood cultures and commencement of broad spectrum antibiotics C : Regular serum potassium measurements D : Rapid infusion of 0.45% saline IVI E : Intravenous insulin using a sliding scale. Comment : This patient has hyperosmolar nonketotic coma with a sodium >150 mmol/l and severe hyperosmolality ((2x(Na+K)+glucose+urea)). This is often precipitated by intercurrent infection. These patients are profoundly dehydrated and at risk of venous thrombosis. They need resuscitation and rehydration. Careful use of 0.45% N/saline would seem advantageous as long as correction of hypernatraemia does not occur too quickly; use N/saline once sodium less than 150. Insulin should be given intravenously using a sliding scale. D : 11. A 20-year-old student presents with amenorrhoea. She had menarche at age 13 years and a regular menstrual cycle before going to college, where she has taken up serious running. She runs for about 10 miles every day and is training for next year’s London marathon. Examination is unremarkable, excepting that she is thin with BMI 19. The most likely cause of her amenorrhoea is: A : pregnancy B : prolactinoma C : excessive exercise D : premature ovarian failure E : polycystic ovarian syndrome. Comment : The ‘female athletic triad’ consists of disordered eating, amenorrhoea and osteoporosis. It is believed that energy imbalance (lots of exercise, low calorie intake) leads (mechanism uncertain) to suppression of the activity of the gonadotrophin-releasing hormone (GnRH) pulse generator, and thereby to amenorrhoea. C :

Page 269: Masterclass Book Part 2

MOHAMMED IS-HAG 268

12.

PLATE 46 This 74-year-old woman was seen as an emergency GP referral just before Christmas. She had been confined to her house for the previous 2 months with increasing lethargy, and had been experiencing worsening frontal headaches with occasional double vision over the preceding 3 weeks. She said she had had a knee replacement 2 years before, suffered mild angina, hypertension and hypothyroidism, and had had major abdominal surgery some 20 years before, though she wasn’t entirely clear why. Her medication consisted of aspirin 75mg od, atenolol 50mg od, L-thyroxine 75mcg od, hydrocortisone 10mg in the morning and 10mg at lunchtime, and fludrocortisone 100 mcg od. Her appearance is shown below. Blood pressure was 160/95. Urea and electrolytes, full blood count and glucose were within the normal laboratory range. Free T4 was 16 pmol/l and TSH<0.03 mU/l. Which of the following is most likely? A : She has untreated haemochromatosis B : She is non compliant with her therapy C : She has developed small bowel malabsorption D : She has an underlying small cell carcinoma of the lung E : She has hyperplasia of the anterior pituitary gland Comment : This patient has Nelson’s syndrome, as evidenced by her intense, seasonally-inappropriate pigmentation, the circumstancial evidence of hypopituitarism (drug regimen, thyroid function tests, and history consistent with bilateral adrenalectomy in the past, with recent expansion of pituitary mass), and the lack of historical or clinical evidence of hypoadrenalism. With improvement in techniques for tumour localisation and extirpation, bilateral adrenalectomy as a second line procedure is much rarer than formerly in the treatment of Cushing’s disease, but when performed, is associated with a risk of Nelson’s syndrome of between 8 and 45%. Nelson’s tumours can be aggressive and locally invasive, and prophylactic pituitary radiotherapy after adrenalectomy is favoured by many. Monitoring is with ACTH levels and serial pituitary imaging. Pigmentation arises from the MSH products of the proteolysis of POMC, which also produces ACTH. Normoglycaemia, normal electrolytes and a good blood pressure suggest that this is not inadequately treated Addison’s disease, while in ectopic ACTH production from an aggressive small cell carcinoma, diabetes, hypokalaemia and severe muscle weakness are characteristic. In this situation, there is generally only a short term between diagnosis and death. E :

Page 270: Masterclass Book Part 2

MOHAMMED IS-HAG 269

13. A 72-year-old woman is admitted with headaches, polyuria and polydipsia of recent onset. She has previously had mastectomy for breast cancer. A CT head scan shows multiple cerebral metastases. Her admission results are as follows: Sodium 153 mmol/l, potassium 4.0 mmol/l, urea 5.0 mmol/l, creatinine 110 micomol/l, glucose 5mmol/l, 24 hours urinary volume: 4.4 litres. Plasma osmolality: 320mOsm/kg, Urinary osmolality: 254mOsm/kg. Which one of the following treatments you will use? A : Hypotonic saline B : Hydrochlorthiazide C : Desmopressin (DDAVP) D : Demeclocycline E : Water restriction. Comment : This lady appears to have cranial diabetes insipidus secondary to metastatic disease from her breast cancer. Her investigations are not complete but out of the above choices, DDAVP in the form of nasal spray or subcutaneous injections, appears to be the most suitable treatment. Hypotonic saline will be risky and replacing ADH should be the aim. Patient should be allowed to drink according to her thirst. Intravenous 5% dextrose will be a better option to use if patient is unable to drink and sodium is rising. Hydrochlorthiazide may be used to treat nephrogenic diabetes insipidus. Demeclocycline and lithium carbonate actually induce nephrogenic diabetes insipidus and may be used in certain cases of SIADH. Water restriction will be very dangerous in the given clinical scenario for obvious reasons. C : 14. Regarding primary hyperlipidaemia: A : familial hypercholesterolaemia is associated with an increase in low-density lipoprotein (LDL)

receptors B : xanthelasmata are a feature of Tangier disease C : tendon xanthomas become more noticeable with age D : xanthelasmata are pathognomonic of primary hypercholesterolaemia E : familial hypercholesterolaemia is inherited in an autosomal recessive fashion. Comment : Familial hypercholesterolaemia (FH) is caused by either: · a reduction in the number of LDL receptors, or · by production of defective LDL receptors. This causes decreased clearance of LDL and hypercholesterolaemia. Homozygous FH is characterised by extreme hypercholesterolaemia and very premature ischaemic heart disease; heterozygous FH has an intermediate phenotype.Xanthelasmata are a feature of conditions with raised LDL cholesterol, either primary or secondary e.g. primary biliary cirrhosis. They do not occur in Tangier disease, a disorder of HDL metabolism in which lipid accumulates in reticuloendothelial tissue such as the tonsils.Tendon xanthomas also occur when LDLcholesterol levels are high, but are a particular feature ofFH.They increase in size with time but regress with lipid lowering therapy, unlike corneal arcus.Xanthomas are an abnormal deposition of lipid in skin and other tissues due to both the severity of and the duration of hypercholesterolaemia.Tendon xanthomas suggest familial hypercholesterolaemia and may occur in the: · extensor tendons of the hands · achilles tendon · patella tendon · olecranon. They increase in size and number with increasing age. C : 15. A 43-year-old lady is referred to the endocrine clinic with a corrected calcium of 1.56mmol/l. This was detected when she had cataract surgery done recently. Her previous medical history is non-significant and so is the family history. Her periods are normal and regular. She went through normal schooling and went to college and studied beauty therapy. She does admit to spasm in her hands when she gets anxious but otherwise no symptoms of chronic hypocalcaemia. She is short, 4 feet 10 inches, has horizontal ridges on her nails and Trousseau's sign is positive. There are no other abnormalities. Her blood results showed a raised phosphate at 2.56mmol/l, very low (undetectable) levels of parathyroid hormone (PTH). Her 0900 hours cortisol, thyroid function tests, urea and creatinine, and other electrolytes are normal. Which of the following diagnoses is most likely? A : Pseudohypoparathyroidism B : Idiopathic (acquired) hypoparathyroidism C : Pseudopseudohypoparathyroidism D : Autoimmune hypoparathyroidism (Polyglandular Autoimmune Syndrome type 1) E : Chronic renal failure with secondary hyperparathyroidism. CommentThis woman has hypoparathyroidism. Patients with pseudohypoparathyroidism have somatic features and PTH levels are high because of target organ resistance.Patients with pseudopseudohypoparathyroidism have the somatic features without biochemical abnormality. Renal failure is excluded by normal urea and creatinine.It is difficult to differentiate between idiopathic hypoparathyroidism and the familial/autoimmune hypoparathyroidism that occurs as a part of polyglandular autoimmune syndrome type 1. However, without any family history of autoimmune disorder and without any personal history of problems such as mucocutaneous candidiasis, vitiligo or alopecia, this woman’s hypocalcaemia is most likely to be due to idiopathic/acquired hypoparathyroidism. B :

Page 271: Masterclass Book Part 2

MOHAMMED IS-HAG 270

16. A 54-year-old woman is diagnosed as having acromegaly. She asks you what treatment the endocrinologist, who she is seeing next week, is likely to recommend in the first instance. You answer: A : Radiotherapy B : Surgery C : Octreotide (or other somatostatin analogue) D : Bromocriptine (or other dopamine antagonist) E : A growth hormone receptor antagonist. The first line treatment is usually surgery, with transsphenoidal adenectomy curing 80% of patients with microadenomas and 50% of those with macroadenomas.Somatostatin analogues are effective symptomatically in 60% of cases and produce a significant reduction in tumour size in 30%, but they can only be given as injections,are expensive,and have a number of side effects.Dopamine antagonists are of very limited efficacy, although they can be useful if the tumour co-secretes prolactin.B : 17.

PLATE 47 A 66-year-old gentleman with recently diagnosed type 2 diabetes is referred by his GP for retinal screening. Digital retinal photography is performed, and the appearances shown (see image) are noted. These findings are most likely to be attributable to: A : previously unrecognized hypertension B : poor glycaemic control C : age-related macular degeneration D : hyperlipidaemia E : previous laser therapy. Comment : The presence of flame and blot haemorrhages, cotton wool spots and blurring of the optic disc margins are typical of the retinal changes that are seen in advanced hypertensive retinopathy. Whilst some of these findings are also observed in diabetic eye disease (e.g. dot and blot haemorrhages, cotton wool spots), the absence of other features (e.g. hard exudates, venous beading) should alert the clinician to other possible diagnoses. A : 18. A 54-year-old gentleman is followed up in the endocrine clinic following his bilateral adrenalectomy in 1976 for Cushing's disease. He is on hydrocortisone 20 mg in the morning and 10 mg at teatime, and 200 micrograms of fludrocortisone a day. His most recent cortisol day profile is satisfactory. His sodium was 149 mmol/l (136-150 mmol/l) and potassium 3.2 mmol/l (3.6-5.0 mmol/l). His blood pressure, on two occasions, was greater than 170 systolic and 100 diastolic. What action will you take? A : Reduce hydrocortisone dose to half B : Start antihypertensive treatment C : Reduce fludrocortisone to 100 micrograms a day D : Start at a small dose of spironolactone and gradually increase E : Advise a low salt diet and review in clinic. Comment : It is quite obvious from his drug history, biochemical profile and blood pressure that he is being over treated with fludrocortisone. His recent cortisol day profile reflected satisfactory replacement with glucocorticoids. Treating him with antihypertensives may help but it does not make sense, as the cause appears to be remediable, and if not corrected, may lead to adverse events. If his blood pressure

Page 272: Masterclass Book Part 2

MOHAMMED IS-HAG 271

remained high despite readjusting his fludrocortisone doses to required levels, he may then need further investigations and treatment with antihypertensives. Spironolactone is anti-aldosterone; its simultaneous use with a mineralocorticoid like fludrocortisone does not make any sense. C : 19. A 40-year-old medical secretary is being investigated for hypertension. Her 24-hour urinary free cortisol levels were high on two occasions and she has significantly raised serum cortisol levels with loss of diurnal rhythm. Which of the following features would favour benign adrenal adenoma as the cause of her Cushing's syndrome over the other causes? A : Presence of hypokalaemia B : Absence of hirsutism C : Presence of weight loss D : Absence of typical features of Cushing's syndrome E : Normal MRI of the pituitary.. Comment : Patients with Cushing's syndrome may have different presentations and features depending upon the cause. Most patients will have features that are more or less typical of Cushing’s but those with ectopic ACTH from a malignant tumour or the ones with malignant tumour of the adrenal itself may present rather quickly and may have weight loss, hypertension, hypokalaemia and obvious tumour and features of its spread and not much in the way of Cushingoid features. Patients with Cushing's disease (pituitary dependent), adrenal adenoma and the ones with carcinoid tumour will have the usual features but those with adrenal adenoma do not have features of hyperandrogenaemia like hirsutism as benign adrenal tumours produce cortisol but not the androgens. Absence of hirsutism and virilisation in a patient with other features of Cushing's syndrome favours adrenal adenoma but needs further investigations. Normal MRI scan of the pituitary does not differentiate between different non-pituitary dependent causes. B : 20. A 22-year-old medical student is found to have Addison's disease. She asks an enormous number of questions. Which of the following of her statements is true regarding Addison’s disease? A : Addison’s disease has a recognised association with type 2 diabetes mellitus. B : Presence of macrocytosis suggests co-existing autoimmune haemolytic anaemia. C : Hypokalaemic alkalosis is a feature of Addison’s disease. D : Thyroid function abnormalities may revert to normal with glucocorticoid therapy. E : Diagnosis is made on detecting adrenal antibodies in the blood. Comment : Addison’s disease is associated with other autoimmune conditions in the autoimmune polyglandular syndromes (both type 1 and type 2). It is type 1 diabetes mellitus, another autoimmune condition, that is associated with Addison’s disease and not type 2 diabetes.Presence of macrocytosis in the context of autoimmunity associated with Addison’s disease raises the possibilty of co-existing pernicious anaemia. It is usually a normocytic normochromic anaemia that may be present in Addison’s disease. There may be neutropenia and eosinophilia as well.Typically, Addison’s disease, due to lack of steroids, is associated with hyponatraemia and hyperkalaemia. There may also be metabolic acidosis. Hypokalaemic alkalosis is a feature of steroid excess as in Cushing’s syndrome.As mentioned earlier, there is an association with other autoimmune conditions. Addison’s disease may be associated with autoimmune thyroid disease. At other times, there may be abnormalities of thyroid function present that may revert back to normal with glucocorticoid therapy.Adrenal antibodies are present in many patients with Addison’s disease but are not essential for making a diagnosis. The diagnosis of Addison’s disease is made on clinical grounds, along with typical abnormalities of biochemistry, cortisol and adenocorticotropic hormone (ACTH) levels, and provocative tests like the Short Syacthen Test. D : 21. Which of the following sets of investigations is not compatible with a non-functioning pituitary adenoma in a 52-year-old man? A : Thyroid-stimulating hormone (TSH) 1.02 mU/l (0.4 to 4.0 mU/l) and Free T4 of 7.1 pmol/l (9.0 to 21.0

pmol/l) B : Prolactin 1856 mU/l (0 to 450mU/l) C : Short Synacthen Test: Cortisol (0 min) 56 nmol/l & Cortisol (30 min) 167 nmol/l (Abnormal) D : IGF-1 raised at greater than 30 % above the upper limit of age-related reference range E : Follicle-stimulating hormone (FSH) 0.3 mIU/l (Low), LH 0.5 mIU/l (Low), Testosterone 5 pg/ml (Low). Comment : Non-functioning pituitary adenomas, as the name suggests, do not secrete any pituitary hormones. They may present with local pressure effects, varying degrees of hypopituitarism, hyperprolactinaemia due to pituitary stalk compression or they may be found incidentally. Pituitary stalk compression may lead to impaired dopamine delivery, hence causing increased prolactin release due to lack of inhibition by dopamine.All of the above sets of investigations, except raised IGF-1 levels, are compatible with hypopituitarism associated with a non-functioning pituitary adenoma. Such high levels of IGF-1 would, in fact, point towards the likelihood of a functioning adenoma (acromegaly) and would warrant further investigations to confirm it. D :

Page 273: Masterclass Book Part 2

MOHAMMED IS-HAG 272

22.

This 25-year-old man has type 1 diabetes of 10 years duration. Recently he has had erratic glycaemic control with both hypo- and hyperglycaemia. He shows you a painless lump in clinic (see image). On examination it is regular, smooth, soft and non-tender. What is the most appropriate intervention? A : To prescribe a different sort of insulin B : To prescribe an antibiotic with gram positive cover C : To prescribe an antibiotic with gram negative cover D : To perform a fine needle aspiration biopsy E : To advise more frequent rotation of injection sites. Comment : This is lipohypertrophy caused by repeated insulin injection into the same site. It is probably caused by a direct effect of insulin on the subcutaneous tissues promoting lipogenesis. Absorption of insulin from the hypertophied site is variable, hence the recent erratic glycaemic control. For this and for cosmetic reasons patients are advised to rotate insulin injection sites. Avoiding further injection into the hypertrophied area will usually allow the tissue to regress. Diabetic patients should have injection sites checked for lipohypertrophy at clinic visits, especially if there is otherwise unexplained deterioration of glycaemic control. E : 23. You see a 32-year-old man in clinic after several episodes of severe paroxysmal hypertension, associated with diaphoresis and headache. On the last occasion he had been admitted overnight until the blood pressure settled. A 24-hour urine collection revealed VMA excretion of 10 times the upper limit of normal on that occasion, while plasma catecholamines were similarly elevated. An emergency CT scan showed a 3cm left adrenal mass. He now attends for further advice. Which of the following is true? A : Arteriography is advisable to corroborate the CT finding prior to surgery. B : No alpha blocker should be given until beta blockade is established. C : There is a 60% chance that the tumour is malignant. D : Labetol would be an appropriate agent to use in a further hypertensive crisis. E : Perioperative diuresis is likely to be necessary in the event of resection. Comment : This man has a phaeochromocytoma, roughly 10% of which are malignant. Alpha blockade must be established prior to beta blockade (to relieve the intense peripheral vasoconstriction - prior to negative inotropic interventions on the heart). Labetalol, which has around 1:5 alpha: beta antagonism, is a satisfactory agent to use in the emergency setting, although a titrating dose of phenoxybenzamine, an irreversible inhibitor, is more common as preparation for surgery. Arteriography may precipitate a hypertensive crisis and should be avoided. Because high tonic levels of catecholamines deplete intravascular volume, fluid repletion is the mainstay of early post resection management. D :

Page 274: Masterclass Book Part 2

MOHAMMED IS-HAG 273

24.

The MRI appearances on T2-weighted images from a 46-year-old female with refractory hypertension are shown (see image). The most likely underlying diagnosis is: A : Cushing’s disease B : Conn’s syndrome C : Phaeochromocytoma D : Multiple Endocrine Neoplasia (M.E.N.) type 1 E : Nephrocalcinosis secondary to primary hyperparathyroidism. Comment : The image reveals a large left suprarenal mass. The appearances are typical of a phaeochromocytoma which, unlike most other adrenal tumours, demonstrates a distinctive ‘bright white’ signal on T2-weighted MRI. In Cushing’s disease (corticotroph pituitary adenoma) bilateral adrenal hyperplasia may be seen on adrenal imaging. Phaeochromocytoma is one component of the M.E.N. type 2 syndrome (medullary thyroid carcinoma, phaeochromocytoma, parathyroid hyperplasia) and not the M.E.N. type 1 syndrome (parathyroid hyperplasia, pancreatic tumours, pituitary tumours). C : 25. A 62-year-old man is found to have glycosuria on a routine urine test. His blood glucose is found to be 21 mmol/l and a diagnosis of maturity onset diabetes mellitus is made. He is very concerned about the implications for his driving licence: which one of the following statements is true? A : Tablet-treated diabetic patients are allowed up to a 3-year licence. B : Insulin-treated diabetic patients are allowed up to a 3-year licence. C : Diet-controlled diabetic patients are allowed up to a 5-year licence. D : Insulin-treated gestational diabetes patients are not allowed to drive E : Diabetic patients with eyesight complications have separate rules to non-diabetic patients. Comment : Patients managed by diet alone need not notify DVLA unless they develop relevant disabilities (such as eye problems) or they need drug treatment for their diabetes.Patients managed by oral hypoglycaemic agents can retain their licence up to the age of 70 years unless they develop relevant disabilites or they need insulin treatment for their diabetes.Patients managed on insulin must demonstrate satisfactory control, recognise warning symptoms of hypoglycaemia and meet required visual standards. They will be given a one, two or three-year licence.Patients with insulin-treated gestational diabetes must notify the DVLA but may retain their licence if they have good control. Patients with diabetes and eyesight complications have the same rules as non-diabetic drivers. B :

Page 275: Masterclass Book Part 2

MOHAMMED IS-HAG 274

26. Regarding primary hyperparathyroidism, which one of the following statements is true? A : Raised parathyroid hormone (PTH) with low serum calcium and high phosphate is diagnostic. B : Hypertension is frequently associated. C : Arcus senalis is a rare but classical sign. D : Preoperative localization of parathyroid adenoma is mandatory. E : Younger patients (<50) usually do not require surgery. Comment : Presence of raised or inappropriately normal PTH levels along with raised calcium levels is seen in primary hyperparathyroidism. Due to the phosphaturic effects of PTH, serum phosphate levels are low. Renal failure increases serum phosphate and decreases vitamin D3 levels. This leads to increased PTH levels as phosphate directly stimulates PTH secretion and vitamin D3 directly inhibits PTH secretion. Hypocalcaemia further increases PTH secretion. This state of affairs is called secondary hyperparathyroidism. Hypertension is frequently associated with hyperparathyroidism. The exact mechanism is not known but a parathyroid hypertensive factor secreted by parathyroid gland may be responsible. Band keratopathy is an irregular region of calcium phosphate deposition at the medial and lateral limbic margins of the cornea. It is a rare but classical sign of hyperparathyroidism. Arcus senalis is an age related concentric grey crescent that eventually encircles the cornea. It is unrelated to hyperparathyroidism. Experienced surgeons, more than 90-95 % of the time, can localize and remove a parathyroid adenoma without preoperative localization. Preoperative localization studies are usually necessary before a second operation as the failure in the first operation could have been due to aberrant lacation of the parathyoid adenoma (5-10 %).Younger patients (<50) with primary hyperparathyoidism are more likely to require surgery than older subjects. Because of the long-term effects of raised PTH on bone and kidneys, some authorities recommend surgery even in mild and relatively asymptomatic hyperparathyroidism in younger patients. B : 27. A 67-year-old lady is admitted following a fall. She is known to be hypertensive and is on treatment with an angiotensin-converting enzyme (ACE) inhibitor. She has chronic renal failure that is presumed to be secondary to hypertension. Routine bloods show: Na+ :136 mmol/l, K+: 5.2 mmol/l, Urea: 20.1 mmol/l, Creatinine: 363 mmol/l, Corrected calcium :2.80 mmol/l, PO42- :1.9 mmol/l. Creatinine clearance 25 ml/min Parathyroid hormone (PTH) performed to assess the hypercalcaemia is 65.5 pmol/l. The most likely diagnosis is: A : Primary hyperparathyroidism B : Secondary hyperparathyroidism C : Tertiary hyperparathyroidism D : Pseudohypoparathyroidism E : Hypercalcaemia secondary to exogenous replacement therapy. Comment : PTH is raised, calcium is raised and so is phosphate, whilst creatinine clearance is deceased. The raised calcium excludes secondary hyperparathyroidism, since calcium is normal and PTH is raised in this condition. Pseudohypoparathyroidism may lead to PTH and phosphate levels this high, but calcium should be low. Exogenous replacement therapy(unless delivered in primary hyperparathyroidism) should suppress PTH. Primary hyperparathyroidism should lead to low phosphate levels and rarely leads to PTH levels this high (though it has been known). The situation is complicated by renal failure however, and it is possible that decreased renal phosphate excretion due to renal failure in the presence of primary hyperparathyroidism could lead to these results thought the most likely explanation is tertiary hyperparathyroidism. In this condition prolonged hypocalcaemia (due to deficient vitamin D hydroxylation in the kidney in this case but also occurring in malabsorption), leads to parathyroid over-activity to maintain serum calcium concentrations. This initially corrects hypocalcaemia (secondary hyperparathyroidism) then may proceed to unrestrained parathyroid over-activity (autonomous gland function and hyperplasia) with hypercalcaemia. PTH levels are typically high, phosphate levels are high secondary to renal failure, and creatinine clearance is markedly reduced (typically < 30ml/min). All these features are present in this patient. C: 28. A 79-year-old frail woman has a toxic adenoma of thyroid and has failed to respond to radioactive iodine, which she had 8 months ago. She remains on 20 mg of carbimazole a day and her latest thyroid function test showed a TSH of 0.3 mU/l (0.4-4.0 mU/l), free T4 of 22 pmol/l (10-20 pmol/l) and free T3 of 6.3 pmol/l (3-5.5pmol/l). What would be the most appropriate line of action? A : Leave her on carbimazole indefinitely B : Block and replace (use high dose carbimazole to block and replace with thyroxine) C : Change carbimazole to propylthiouracil D : Refer her for a second dose of radioactive iodine E : Refer her to surgeons for thyroidectomy. Comment : Long-term carbimazole is an option, but dose requirement is high and she may run into side effects. Giving another dose of radioactive iodine is simpler, quite likely to be successful and may be curative. Surgery is also an option, but in a frail patient it would be standard practice to try at least one further dose of radioactive iodine before considering this. Propylthiouracil has no advantage over carbimazole. Block and replace therapy is not used in this clinical situation. D :

Page 276: Masterclass Book Part 2

MOHAMMED IS-HAG 275

29. An 18-year-old girl presented to the endocrine clinic accompanied by her mother. The family were concerned that the girl was excessively tall compared to other members of her family and that she had still not had her first period, although she had developed pubic hair at the age of 13 years. She had a cleft palate repaired as an infant. On examination, she was 180cm with a weight of 67kg. Her mother was 150cm and her father 175cm. Pubic hair was stage 3 but breast and nipple development were prepubertal (stage 1). Baseline tests showed that plasma estradiol levels were undetectable with baseline LH levels of 1.9 IU/l and FSH 3.2 IU/l. All other baseline anterior pituitary hormones were within the reference range. Which of the following statements is correct? A : The patient has premature ovarian failure B : The patient should be counselled that she will not be able to conceive without egg donation C : There is a pathological process affecting the anterior pituitary gland D : An MRI of the brain is likely to reveal abnormalities of the olfactory bulbs or sulci E : Oestrogen replacement therapy is not indicated Comment : This patient had Kallmann’s syndrome. This is the most common form of isolated hypogonadotrophic hypogonadism with delayed puberty in which anosmia resulting from agenesis of the olfactory lobes or sulci is associated with LHRH deficiency. The condition is four times more common in boys than in girls. In some cases the LHRH deficiency may be partial (the fertile eunuch syndrome) and rarely affected men who had a delayed puberty may recover spontaneously, experience an increase in testicular size and enter full puberty. Undescended testes and gynaecomastia are common in boys, as with other forms of hypogonadotrophic hypogonadism. Associated defects that may be variably present include cleft lip, cleft palate, imperfect facial fusion, seizure disorders, short metacarpals, pes cavus, neurosensory hearing loss, cerebellar ataxia and nystagmus, oculomotor abnormalities and unilateral or rarely bilateral renal aplasia. In a review of MRI findings of patients with Kallmann’s syndrome, 56% had absent or abnormal olfactory sulci bilaterally; in 17% the agenesis was unilateral. The syndrome can be transmitted as an X-linked, autosomal dominant or autosomal recessive trait. The molecular genetics of the X-linked form have been well established. The KAL 1 gene is located at the Xp22.3 locus and encodes a 680 amino acid glycoprotein, named anosmin-1. This glycoprotein is thought to play a part in the guidance of LHRH neurons to the medial basal hypothalamus. A wide variety of deletions and mutations of the KAL gene have been described. Management includes sex steroid replacement therapy to induce and maintain primary and secondary sexual characteristics and maintain bone and muscle mass. Premenopausal women a combined oral contraceptive pill may be used or alternatively oestrogen patches or gels with concomitant cyclic progesterone therapy in women with an intact uterus to prevent unopposed endometrial proliferation. For males, who do not wish to be fertile, intramuscular testosterone (sustanon) 250mg three times weekly is effective. Alternatively, testosterone patches, gels or buccal preparations can be used. Women who wish to become pregnant respond well to gonadotrophin or pulsatile GnRH therapy. The latter is less likely to result in multiple pregnancy.Men who wish to be fertile should have their testosterone replacement stopped, since testosterone therapy may suppress spermatogenesis. These patients also respond well to pulsatile GnRH therapy. However, adequate spermatogenesis may take up to two years to achieve and frozen aliquots should be stored for further use. Alternatively, patients may respond to ? hCG (500-2000 IU) given intramuscularly or subcutaneously three times a week;FSH 75IU may be added to improve sperm quantity after six months,if necessary. D : 30. A 45-year-old man is referred with severe tiredness, frontal headaches and lack of libido. He has a bitemporal hemianopia on confrontation. Which of the following results would not be consistent with the presence of a pituitary adenoma? A : Low insulin-like growth factor-1 (IGF-1) levels B : High prolactin levels C : High follicle-stimulating hormone (FSH) and luteinizing hormone (LH) levels with a low testosterone level D : Secondary hypothyroidism E : Poor (sub-normal) cortisol response to a short synacthen test. Comment : This patient's clinical picture is highly suggestive of a pituitary adenoma. It is difficult on the given information to ascertain whether he has a secretory or a non-secretory tumour. High prolactin levels may be seen in prolactinomas or any other pituitary or non-pituitary tumours where pressure from the tumour mass on the pituitary stalk functionally disconnects the inhibitory input from the hypothalamus. Some growth hormone secreting adenomas also secrete prolactin. Low levels of certain pituitary hormones occur due to local effects of the pituitary adenoma (whether secretory or non-secretory). Secondary hypothyroidism due to lack of thyroid-stimulating hormone (TSH), adrenocortical deficiency due to lack of adrenocorticotrophic hormone (ACTH), low IGF-1 associated with growth hormone deficiency and low testosterone associated with FSH and LH deficiency (hypogonadotrophic hypogonadism) can all occur together, in different combinations or on their own. C :

Page 277: Masterclass Book Part 2

MOHAMMED IS-HAG 276

31. A 24-year-old female with known type 1 diabetes mellitus is admitted via the accident and emergency department in a semi-concious state (Glasgow Coma Score [GCS] = 10). The admitting SHO documents her to be agitated and tachypnoeic with a respiratory rate of 36 breaths/minute. The staff nurse reports that the blood sugar checked with a bed-side stick test is ‘high’. In these circumstances: A : intravenous fluids must be given with caution until a chest radiograph has been undertaken to

exclude pulmonary oedema B : a ventilation/perfusion (V/Q) scan should be arranged once the patient has been stabilized and adequately

resuscitated C : broad-spectrum antibiotics should be commenced as soon as blood cultures have been taken D : intravenous fluids must not be supplemented with potassium chloride until the result of repeat

electrolytes performed at 4 hours post-admission are available E : oral fluids should be with withheld. Comment : The picture here is one of moderate to severe diabetic ketoacidosis (DKA) – a condition that is still associated with significant morbidity and mortality. The tachypnoea almost certainly reflects the presence of a significant metabolic acidosis (with an attempt to compensate through ‘blowing off’ carbon dioxide - Kussmaul’s respiration), rather than primary lung pathology (although the latter may of course coexist and could have precipitated the acute episode). Patients with DKA are usually severely dehydrated (5-10L) and aggressive fluid resuscitation is a cornerstone of management – aim to give 5-6L within the first 24 hours, but bear in mind the clinical setting, for example a fit 24-year-old is likely to tolerate more aggressive fluid replacement than an elderly patient with a history of cardiac disease – consider central venous pressure (CVP) monitoring in the latter group. In either case, do not delay/withhold fluids whilst awaiting a chest radiograph (which may take several hours to obtain!). Unless there are specific clinical indications, a V/Q scan should not be routinely requested and broad-spectrum antibiotics should not be routinely given – remember that the white cell count may be raised in DKA even in the absence of infection/sepsis. Although potassium levels are often normal or even slightly elevated at the point of admission, life-threatening hypokalaemia may develop rapidly, especially in response to insulin (which ‘pushes’ potassium inside cells). Therefore, whilst it is reasonable (and usually advisable) not to add potassium to the first bag of fluid, thereafter supplementation should be instituted unless there are strong clinical indications to the contrary. In a semi-conscious patient the risk of aspiration is high (especially in the presence of delayed gastric emptying as is typically the case with DKA), and accordingly the patient should be made nil by mouth and a nasogastric tube sited. E: 32. Which of the following statements is true regarding thyroid malignancy? A : Papillary cell carcinoma is the commonest thyroid tumour. B : Medullary thyroid carcinoma (MTC) is associated with MEN Type-1 syndrome. C : Follicular cancer has the worst prognosis of all. D : Anaplastic carcinoma occurs in relatively younger age group as compared with other types of thyroid

malignancy. E : Lymphoma of the thyroid gland almost always arises as part of generalised lymphoma. Comment : Papillary carcinoma of thyroid is the commonest (~70% of all cases) type of thyroid cancer. It is probably the slowest growing and has the most favourable outlook of all.Anaplastic carcinoma occurs in relatively older age groups with peak incidence in the 7th decade. It is the most aggressive of all thyroid cancers and has the worst prognosis.Follicular carcinoma of the thyroid has a less favourable outcome than papillary carcinoma but better than anaplastic carcinoma.Medullary thyroid carcinoma is a part of multiple endocrine neoplasia (MEN) syndromes types 2A and 2B, and may be associated with pheochromocytoma and parathyroid hyperplasia. MEN type 1 syndrome has parathyroid hyperplasia, pancreatic and pituitary tumors as its constituents.Lymphoma of the thyroid is very rare (less than 1% of all cases). It can arise as a part of generalised lymphoma or as a primary lymphoma of the thyroid. A : 33. A 54-year-old man presents with renal calculi and bone pain. His mother was told she had a high calcium level and his grandmother died of a perforated duodenal ulcer. Which of the following findings favours a diagnosis of Multiple Endocrine Neoplasia Type 1? A : Raised serum calcium with a suppressed parathyroid hormone (PTH) B : Family history consistent with autosomal recessive pattern of inheritance C : A prolactin level within the reference range D : Medullary thyroid carcinoma E : Hypercalcuria. Comment : The association of parathyroid, pancreatic islet and pituitary hyperplasia or neoplasia is called MEN1, which is an autosomal dominant disease. Findings consistent with the diagnosis include a raised corrected serum calcium with a normal or elevated PTH measurement, indicating primary hyperparathyroidism. Hypercalciuria resulting from a high serum calcium level may be found, as may elevated prolactin measurements, possibly indicating prolactinoma.Medullary thyroid carcinoma is associated with MEN 2. E :

Page 278: Masterclass Book Part 2

MOHAMMED IS-HAG 277

34. A 55-year-old man with type 2 diabetes mellitus, who has been lost to follow-up for over 2 years, is admitted with poorly controlled diabetes. Apart from polyuria, polydipsia and tiredness, he is also complaining about blurred vision, especially in right eye. He has lost weight but has dependent oedema of his lower legs. His urine dipstix test reveals 4+ protein and 4+glucose. He has signs of peripheral neuropathy and there's clinical evidence of right Charcot's foot. His laboratory results are as follows:Hb. 11gm/dl, MCV 84, WBC 9.0, Platelets 323, ESR 18 mm 1st. hour Sodium 138mmol/l, Potassium 4.7mmol/l, Urea 11.8mmol/l, Creatinine 198 micromol/l, Albumin 32 g/l, HbA1c 11.9% What do you think the preferred next step should be? A : Urinary albumin:creatinine ratio B : Radiograph right foot C : Fundoscopy through dilated pupils D : Ultrasound scan of renal tract E : 24-hour urinary protein estimation. Comment : This patient has poorly controlled diabetes. He has not had his annual screening for at least 2 years. He appears to have diabetic nephropathy and may well have nephrotic syndrome. Furthermore, he has signs of peripheral neuropathy and possibly Charcot foot. It is quite likely that he has diabetic retinopathy and it is very important to exclude sight threatening retinopathy like maculopathy or proliferative retinopathy. Blurred vision may be due to poor glycaemic control but it is vital to perform fundoscopy and exclude any retinopathy that requires urgent ophthalmological attention. The patient has dipsticks positive proteinuria, hence urinary albumin:creatinine ratio is pointless. 24-hour urinary protein excretion will be required to quantify the proteinuria. Ultrasound of renal tract to see the size of kidneys and to exclude obstructive uropathy is important and should ideally be carried out as in-patient. Radiograph of his foot and possibly some other investigations will also be needed to assess the problem and extent of his foot disease but urgent fundoscopy can lead to preservation of his sight from imminent threats. C : 35. A 28-year-old woman is diagnosed as having Graves’ disease. She has prominent eyes. A medical student asks you to define what is meant by the term ‘lid lag’. You reply: A : When the patient is looking straight ahead, you can see a bit of the sclera all around the iris. B : When the patient tries to close her eyes, the lids only touch each other with difficulty. C : When the patient follows your finger, moving upwards from below, the upper eyelid has difficulty

in moving out of the way. D : When looking at the patient from the side, you see that the eyes are proptosed. E : When the patient follows your finger, moving downwards from above, the sclera can temporarily be seen

above the iris. Comment : Option A is the definition of lid retraction.Other ocular features of Graves’ disease include proptosis, ophthalmoplegia, periorbital oedema and chemosis. E : 36. A 25-year-old woman is referred complaining of excessive hair growth on her face and stomach. On further questioning she says this has worsened greatly over the past 2 years together with significant weight gain after a change of job. Her menses have always been rather erratic, but this has also become more marked recently. She takes no medication. On examination her BMI is 30 kg/m2 with a blood pressure of 140/85. She has recently depilated her face, but has a marked escutcheon and sparse hairs on her nipples. There is no sign of masculinisation or other obvious abnormality. The serum testosterone level is 3 nmol/l (ULN 2). Which of the following is true with respect to her further management? A : Androstenedione measurement would specifically be useful in detecting adrenal disease. B : The elevated testosterone is strongly suggestive of an underlying virilising tumour. C : Cyproterone acetate alone would be an appropriate initial treatment. D : A further assessment should be made in 4-6 weeks, and treatment changed if ineffective. E : Improving insulin sensitivity would be likely to lessen the hirsutism. Comment : Androstenedione is produced from both the adrenals and ovaries, while dehydroepiandrosterone sulfate (DHEAS) is adrenal-specific. Mildly elevated testosterone levels are common in benign cases of hirsutism (such as those due to polycystic ovary syndrome (PCOS)), but 5nmol/l has been suggested as a cut-off for more urgent investigation of possible underlying malignancy. Antiandrogens must never be prescribed alone where pregnancy is a possibility due to their teratogenicity, and the slow hair growth cycle means that at least 3, and preferably 6-12 months should elapse before a treatment is deemed to have failed. In this case, the likely cause of hirsutism is PCOS exacerbated by the insulin resistance of obesity. Insulin sensitisation through weight loss or treatment with, e.g. metformin (currently unlicensed for this although evidence exists) would have a salutary effect. E :

Page 279: Masterclass Book Part 2

MOHAMMED IS-HAG 278

37. A 28-year-old woman is seen in outpatients with a 10-month history of oligomenorrhoea and 6 months of weight gain, hirsutism and male pattern alopecia. Which one of the following five investigations is unnecessary? A : 17-OH progesterone B : 24 hours urinary free cortisol C : Serum testosterone D : MRI scan of the adrenals E : Gonadotrophin-releasing hormone test (GnRH test). Comment : The above presentation is compatible with androgen excess. There may be several causes. Important differentials should include: · polycystic ovarian syndrome (PCOS) · androgen-producing adrenal and ovarian tumours · congenital adrenal hyperplasia (CAH) · Cushing's syndrome. Estimation of 17-OH progesterone is relevant regarding CAH but the levels may be raised in certain other adrenal conditions. Short synacthen test, involving estimation of 17-OH progesterone alongside cortisol in response to synacthen, is the standard test for adult type CAH. Menstrual irregularities, hirsutism with virilisation and weight gain are features that may be present in Cushing's syndrome, hence estimation of 24 hours urinary free cortisol on at least two occasions is a good screening test. Estimation of serum testosterone, other androgens, sex hormone binding globulin (SHBG) and free androgen index (FAI), will be important in all of the above conditions as hyperandrogenism may be present in all of them. Imaging of adrenals and ovaries, such as CT or MRI scans, is relevant in the above setting but obviously there will be some further indications on basic examination and investigations to suggest the possibility of adrenal tumour, hyperplasia or ovarian tumour.Estimation of gonadotrophins (follicle-stimulating hormone (FSH) and luteinizing hormone (LH)) along with androgens and estrogen, is important in the given presentation but there is no indication to perform a gonadotrphin-releasing hormone test. This test is done as a part of the combined triple test in suspected hypopituitarism and in the investigation of delayed puberty. E : 38. A 58-year-old man is seen for the first time in the diabetes clinic after several years of management by his GP. His blood pressure is 139/87, and his annual review tests include the following: HbA1c 8.6%, cholesterol 5.6 mmol/L (HDL 1.0, LDL 3.7), triglycerides 3.1 mmol/L. His urinary albumin : creatinine ratio is 4.1, and subsequent timed urine collection reveals an albumin excretion rate of 250 mg/24 hours. Which of the following is correct with respect to his further management? A : The urinary protein loss will increase inexorably. B : ACE inhibitors and angiotensin II receptor antagonists have both been proven to improve the

renal prognosis. C : The main focus of effort should be the preservation of renal function. D : There is no benefit in further reduction of blood pressure. E : 10 year coronary risk should be estimated by reference to the Joint Societies prediction charts, as

published in the British National Formulary. Comment : The great importance of microalbuminuria is that it confers an additional risk of cardiovascular disease above that resulting from other established risk factors, perhaps because it is a surrogate marker of generalised endothelial dysfunction. Importantly, this excess risk is not encompassed by the Joint Societies charts in the B.N.F., which thus underestimate the risk. Hence, it is important to intensify control of conventional risk factors, as the greatest threat to a patient such as this is macrovascular disease. Microalbuminuria is also important as the most easily measured early stage of diabetic nephropathy. It is, moreover, also reversible; the chances of this being maximised by scrupulous blood pressure management and blockage of the renin-angiotensin system. Both ACE inhibitors and ATII antagonists have been shown in large trials to improve the renal prognosis, and emerging evidence suggests their effects are comparable. Trial data suggests that lowering blood pressure improves prognosis even within the normal range, and it may be that targets below the current one of 135/80 may be justified. B : 39. Male hypogonadism can be classified as primary or secondary. Which of the following conditions would be expected to give rise to primary hypogonadism? A : Constitutional delayed puberty B : Klinefelter's syndrome C : Hyperprolactinaemia D : Kallmann's syndrome E : Pituitary macroadenoma. Comment : Male hypogonadism denotes deficiency of both testosterone secretion and sperm production. Primary hypogonadism is due to testicular dysfunction, whereas secondary hypogonadism reflects either hypothalamic or pituitary dysfunction. Klinefelter's syndrome is a congential disorder with

Page 280: Masterclass Book Part 2

MOHAMMED IS-HAG 279

seminiferous tubule dysgenesis. Testosterone production is variable and therefore different degrees of sexual development are possible within the syndrome. Kallmann's syndrome is associated with isolated gonadotrophin deficiency. Patients may have undescended testes and gynaecomastia - but these are secondary not primary events. B : 40. A 44-year-old woman presents with weight gain and hypertension. Urinary free cortisol: First specimen: 330nmols/24 hours; second specimen: 376 nmol/24 hours (normal values up to 270 nmol/l) 0900 hrs cortisol 1450 nmol/l (Normal: 200-700 nmol/l) 0900 hrs cortisol 82 ng/l (Normal: up to 80 ng/l) 0900 hrs cortisol suppressed to <50 nmol/l following a 48 hours high dose dexamethasone suppression test Corticotrophin-releasing hormone (CRH) test: Exaggerated Adrenocorticotrophic hormone (ACTH) and cortisol response. Which of the following conditions is most likely to be present? A : Ectopic ACTH production B : Pseudocushings C : Cushing's disease D : Exogenous steroids E : Cortisol producing adrenal tumour. Comment : This patient presents with weight gain and hypertension. Further history and examination are obviously very important. If one suspects Cushing's syndrome, 24 hours urinary free cortisol should be checked as a screening tool and repeated.As the patient seems to have significantly raised levels of cortisol in urine, further investigations should be carried out. Basal cortisol is high and basal ACTH is high normal. This would point towards pituitary dependent Cushing's disease and away from other causes. But the patient needs further evaluation to confirm this.Suppression of cortisol levels to high dose dexamethasone test and an exaggerated ACTH and cortisol response to CRH test points to the diagnosis of pituitary dependent Cushing’s disease and excludes ectopic ACTH, pseudocushings and other causes. Petrosal venous sampling before and after CRH stimulation, showing greater than twice the levels of ACTH as compared to peripheral blood, confirms Cushing's disease and may also help in lateralizing the pituitary adenoma. C : 41. A 36-year-old schoolteacher with a longstanding history of headaches and non-specific variable muscular pains gives a 1-year history of marked polyuria and thirst. On questioning she describes a continually dry mouth and passing large volumes of clear urine. She passes urine 2-3 times a night and always has a glass of water on her bedside table. Examination is unremarkable. She is booked for a water deprivation test and attends after a light breakfast. Baseline bloods:Glucose 7.8 Na+ 134 mmol/l, K+ 3.5 mmol/l, Urea 3.0 mmol/l, Creatinine 60 mmol/l, Ca2+ 2.70 mmol/l, Albumin 38 g/l Time: 0hrs 2 hrs 4 hrs 6 hrs 8hrs Urine osmolality (mOsm/kg)

270 270 310 380 420

Plasma Osmolality (mOsm/kg)

281 281 285 288 290

You are called to review the results to determine the next course of action. What should you do? A : Diagnose syndrome of inappropriate antidiuretic hormone secretion (SIADH) and stop the test B : See that the results are suggestive of psychogenic polydipsia but inconclusive and continue the

dehydration phase for another hour to see if plasma osmolality rises before taking other measures

C : See that nephrogenic diabetes insipidus is most likely and give 2 mcg of IM desmopressin before collecting urine samples for another 4 hours

D : See that cranial diabetes insipidus is most likely and give 2 mcg of IM desmopressin before collecting urine samples for another 4 hours

E : Diagnose diabetes mellitus as the likely cause for the polyuria and stop the test. Comment : Syndrome of inappropriate antidiuretic hormone secretion is characterised by low plasma osmolality, low plasma sodium and relatively high urine osmolality. These basic criteria are not met in this instance. The plasma glucose sample is a post-prandial sample and therefore a result of 7.8 mmol/l is not diagnostic of diabetes mellitus, so this is not a consideration in this case. Both cranial and diabetes insipidus (DI) are characterised by an inability of the kidneys to concentrate the urine. In cranial diabetes insipidus this is because the hypothalamus/pituitary has lost the ability to produce vasopressin. In nephrogenic DI there is no response to the action of vasopressin in the distal renal tubule. In both types of DI plasma osmolality is usually raised and the urine is not appropriately concentrated (urine osmolality: plasma osmolality <2.0). It would be very unusual for plasma osmolality not to rise above 295 mOsm/kg during the course of a water deprivation test in DI (with the lack of an appropriate urine concentrating response). Cranial DI is differentiated from nephrogenic DI by the

Page 281: Masterclass Book Part 2

MOHAMMED IS-HAG 280

response to 2mg IM desmopressin at the end of the test (when the individual is allowed to drink free fluids). An individual who has developed plasma osmolality >295mOsm/kg and who has failed to appropriately concentrate their urine (urine osmolality: plasma osmolality < 2.0) and who has cranial DI, should concentrate their urine normally in response to desmopressin. In nephrogenic diabetes insipidus the urine fails to concentrate normally in response to desmopressin. A normal individual would concentrate their urine throughout the test and there should be a fall off in urine volume. Plasma osmolality rises but remains below 295 mOsm/kg. Urine osmolality: plasma osmolality ratio should rise above 2.0 during the test. In psychogenic polydipsia, continual water loading can “washout” the renal concentrating mechanism, but urine osmolality: plasma osmolality rises to > 2.0 by the end of the test provided adequate dehydration is achieved. Since, however, many of these subjects are fluid overloaded at the start of the test (as in this case) 8 hours may not be enough to achieve adequate dehydration. If at the end of 8 hours urine output has not tailed off and/or urine osmolality: plasma osmolality is < 2.0, but plasma osmolality is < 295 then water deprivation should be continued for another hour before administration of desmopressin. If results are still equivocal then a modified Miller-Moses test or hypertonic saline infusion may be considered. B : 42.

PLATE 49 A 62-year-old gentleman with type 2 diabetes for 17 years is found to have decreased visual acuity at his annual review. Which of the following is true regarding maculopathy? A : Maculopathy causes loss of peripheral vision B : Maculopathy requires routine referral for an ophthalmology opinion C : Maculopathy is unaffected by poor glycaemic control D : Circinate exudates may indicate maculopathy E : Maculopathy is unaffected by hypertension. Comment : Maculopathy is the commonest threat to vision in type 2 diabetes. It typically produces loss of central vision and if discovered or suspected requires urgent referral to an ophthalmologist. The 3-year risk of severe visual loss in maculopathy is reduced by over 50% with photocoagulation. Macular grid laser therapy is usually used. Both glycaemia and hypertension should be as well controlled as possible. Circinate exudates occur around areas of microvascular leakage and should therefore lead to the suspicion of macular ischaemia. D : 43. In which of the following patients can diabetes be diagnosed from the information given? A : A 46-year-old gentleman with polydipsia, polyuria and a glucose of 12.1 mmol/l. B : A 33-year-old lady with a fasting plasma glucose of 9 mmol/l. C : A 33-year-old lady with headaches and a random plasma glucose of 14 mmol/l. D : A 62-year-old lady with polydipsia, polyuria and a glucose of 10.4 mmol/l. E : A lady who is 32 weeks pregnant and has a random glucose of 14.1 mmol/l. Comment : In people with symptoms of hyperglycaemia (e.g. thirst, polyuria, weight loss) diabetes can be diagnosed with a random glucose greater than 11.1 mmol/l. Headaches are not hyperglycaemic symptoms. If there are no hyperglycaemic symptoms then there have to be two glucose results within the diagnostic range on separate daysA fasting plasma glucose greater than or equal to 7 mmol/l or a 2-

Page 282: Masterclass Book Part 2

MOHAMMED IS-HAG 281

hour post 75 glucose load plasma glucose greater than or equal to 11.1 mmol/l would meet diagnostic criteria. The diagnostic criteria are the same in pregnancy. In children the standard 75-g OGTT does not apply. Instead 1.75g/kg of glucose is given. These glucose values were agreed by the World Health Authority in 1997 as the levels required to diagnose diabetes. A : 44. A 66-year-old lady was diagnosed with polymyalgia rheumatica and was commenced on long-term steroid therapy. Looking at her chest radiograph, your house officer thinks that she may already have osteoporosis. Which one of the following steps would you take? A : Arrange out-patient DEXA scan and treat only if she has osteoporosis. B : Commence anti-resorptive therapy like Alendronate. C : Advise her to take high calcium diet and exercise regularly. D : Start her on Hormone Replacement Therapy (HRT). E : Consider antiresorptive therapy only if steroids are continued beyond 2 years. Comment : According to the recent guidelines from the Royal College of Physicians, men and women over the age of 65 yr who are on chronic (more than 3 months) steroid therapy should be started on treatment such as antiresorptive drugs to prevent osteoporosis. They should go onto such therapy at the same time as they commence steroids. A DEXA scan is not required as they need preventive therapy regardless of the result such a scan might show. HRT may have a limited role early in the postmenopausal period, but only for a limited time and mainly for vasomotor symptoms. At the age of 66 yr HRT is not recommended for osteoporosis prevention therapy. B : 45.

PLATE 50 A medical student attached to the endocrine firm is asked to comment on the eye signs exhibited by the patient shown below. She specifically comments that the patient has Graves’ dysthyroid eye disease on the basis of the clinical findings listed below. Which of her observations is true? A : Ophthalmoplegia B : Chemosis C : Periorbital oedema D : Lid retraction E : Visible nodular goitre. Comment : Both lid lag and lid retraction reflect enhanced sensitivity to circulating catecholamines, and may therefore be found in most thyrotoxic states. The following eye signs are specific to Graves’ disease: · proptosis · ophthalmoplegia · chemosis · periorbital oedema . This patient exhibits neither ophthalmoplegia nor chemosis. The goitre in Graves’ disease is typically smooth rather than nodular. C :

Page 283: Masterclass Book Part 2

MOHAMMED IS-HAG 282

46. A 56-year-old gentleman is referred by his GP for advice with regards abnormal thyroid function tests, which were discovered when he attended for routine follow-up in the cardiology clinic. The patient is currently asymptomatic, but was started on amiodarone recently, following a hospital admission with 'troublesome atrial fibrillation': Free T4 15.2 pmol/L (9-20) TSH 9.8 mU/L (0.4-4) You should advise the GP: A : to discontinue the amiodarone immediately B : that the patient is developing hypothyroidism and requires thyroxine replacement C : that the patient's thyroid dysfunction is the likely underlying cause for his 'troublesome atrial

fibrillation' D : to simply repeat the thyroid function tests in six to eight weeks providing the patient remains

well E : to commence carbimazole to minimise the risk of further cardiac upset due to uncontrolled thyroid

dysfunction. Comment : The anti-arrhythmic agent amiodarone has a high organic iodine content (approximately 1/3 by weight) and bears structural similarities to both T4 and T3. Standard maintenance doses of amiodarone result in a massive expansion of the iodide pool and can influence thyroid physiology in a variety of ways, not all of which require intervention ! For example, it is not uncommon to observe a rise in TSH levels during the early phases of treatment (possibly indicating an effect on the pituitary specific type 2 deiodinase enzyme), which may, or may not, be associated with an increase in T4 levels at the expense of T3 (through inhibition of the peripheral type 1 deiodinase enzyme). D : 47. Regarding the insulin tolerance test, which of the following statements is true? A : It is contraindicated after pituitary surgery. B : It is the gold standard for assessment of growth hormone reserve in adults. C : The plasma glucose should not be allowed to fall below 2.2 mmol/l. D : It is a useful outpatient screening test. E : It is not indicated for the assessment of growth hormone (GH) reserve in children. Comment : The insulin tolerance test (ITT) exploits the principal that hypoglycaemia activates the hypothalamo-pituitary-adrenal axes. It is primarily used in adults with suspected hypopituitarism or isolated adrenocorticotrophic hormone (ACTH) or GH deficiency to assess ACTH cortisol and GH reserve. It is used to assess GH reserve in children with definite growth retardation and a subnormal growth hormone stimulation test to stimuli such as exercise. ITT remains the gold standard investigation for these conditions although other stimulatory tests may need to be used due to contraindications to ITT. In the investigation of possible panhypopituitarism (due to any cause including tumour or after pituitary surgery), the ITT is sometimes combined with thyrotrophin-releasing hormone (TRH) and gonadotrophin-releasing hormone (GnRH) stimulation as a ‘combined pituitary function test’. ITT is a useful but potentially dangerous investigation and should only be performed as an inpatient under specialist supervision. Contraindications to ITT are: · Ischaemic heart disease, dysrhythmia and/or abnormal resting ECG · Epilepsy · Untreated hypothyroidism (impairs the GH and cortisol response) · Severe glucocorticoid deficiency (9am cortisol < 100nmol/l). NB: Do not replace thyroxine if significant adrenal insufficiency is suspected until glucocorticoid replacement has been established, as this may precipitate a hypoadrenal crisis. It is a requirement of the ITT that adequate hypoglycaemia (<2.2mmol/l) is achieved. Failure to achieve adequate hypoglycaemia indicates an inadequate stimulus to the HPA axes and requires repetition of the test with an increased dose of insulin. B : 48. A 38-year-old man presents with recurrent ‘funny turns’. During one of these he is found to have a blood glucose concentration of 1.6 mmol/l. A clever pre-registration house officer sent a concurrent serum sample to the biochemistry laboratory: this is found to contain insulin, but C-peptide levels are low. This means that hypoglycaemia is likely to have been caused by: A : endogenous insulin, e.g. produced by an insulinoma B : consumption of an oral hypoglycaemic agent C : liver disease D : starvation E : exogenous insulin, e.g. self-administration. Comment : If endogenous insulin were the cause, whether generated by a tumour (insulinoma) or in response to stimulation by an oral hypoglycaemic agent, then levels of both insulin and C-peptide would be the same. The only thing that can produce dissociation between insulin and C-peptide levels is administration of exogenous insulin. E :

Page 284: Masterclass Book Part 2

MOHAMMED IS-HAG 283

49. A GP rings you about a 55-year-old lady with metastatic carcinoma of the breast who has become acutely confused. What advice would you give? A : Start the patient on high dose steroids B : Start the patient on diazepam C : Ask the GP to exclude hypercalcaemia D : Start the patient on antibiotics E : Offer to send out a Community Macmillan Nurse. Comment : Hypercalcaemia can present as acute confusion and can only be diagnosed by a blood test. It can be treated with intravenous fluids and bisphosphonates. It is important to exclude infection as a cause of confusion. If she is very agitated an antipsychotic such as haloperidol would be more appropriate than diazepam. C : 50. A 38-year-old sales representative is referred to your endocrine clinic. He tells you he has a lifelong history of compulsive water drinking. He suffered from nocturnal enuresis as a child and the diagnosis of psychogenic polydipsia was agreed by a number of doctors at that time who instructed the family on strict behavioural measures to combat the problem. He had not seen a doctor since his early teenage years but had become worried that his psychological condition was worsening when he found himself drinking water from a roadside drain on a warm day when on a business trip abroad. He attends reluctantly since his GP is requesting one final medical opinion prior to referral to the psychiatrists. He attends for the water deprivation test you arrange with 4l of fluid concealed in his bags and becomes agitated when they are removed. Baseline bloods from clinic:

TSH 1.21 iu/l

glucose 5.4 mmol/l

Na+ 138 mmol/l

K+ 3.5 mmol/l

Urea 7.1 mmol/l

Creatinine 110 mmol/l

Ca2+ 2.78 mmol/l

Albumin 45 g/l

Water deprivation test:(2mg IM desmopressin given at 8 hours and allowed to drink from then)

Time (hrs) 0 2 4 6 8 10

Urine osmolarity (mOsm/kg) 205 205 220 240 265 588

Plasma Osmolarity (mOsm/kg) 285 290 298 305 310 293

What should you do? A : Water restrict the patient and suggest to the GP that they proceed with the psychiatry referral B : Investigate hypercalcaemia as a possible cause of polyuria C : Prescribe oral or nasal desmopressin D : Prescribe a thiazide diuretic and mild sodium restriction E : Prescribe oral or nasal desmopressin and arrange an outpatient MRI scan of the pituitary. Comment : This gentleman fails to concentrate his urine despite a rise in plasma osmolality above 295 mOsm/kg and this is diagnostic of diabetes insipidus (DI). Following administration of desmopressin he concentrates his urine, the urine osmolality rising by more than 50% and producing a urine osmolality: plasma osmolality ratio of > 2.0, demonstrating a response of the kidneys to exogenous desmopressin. This means that he must have cranial DI, the treatment for which is oral or intranasal DDAVP. It would therefore be wise to perform an MRI of the pituitary to look for a structural pituitary lesion that might be the cause. The treatment of nephrogenic DI is often achieved by the use of thiazide diuretics and sodium restriction. E : 51. Which of the following cannot be included as a cause of primary hyperaldosteronism? A : Conn's syndrome B : Bilateral adrenal hyperplasia C : Adrenal carcinoma D : Glucocorticoid remediable aldosteronism E : Cushing's disease. Comment : Primary hyperaldosteronism results from increased production of aldosterone by abnormal zona glomerulosa tissue from an adenoma or hyperplasia. Benign aldosterone producing adenomas - first described by Conn - account for 75% of all cases of primary hyperaldosteronism. Primary aldosteronism is a disease of the zona glomerulosa, and other adrenal products may be increased including 11-deoxycorticosterone, corticosterone and 18-hydroxycorticosterone. Cells in the zona glomerulosa cannot produce cortisol and therefore cortisol levels should be normal. However, extremely high levels of cortisol may result in cross-activation of the mineralocorticoid receptor and give a biochemical picture similar to primary hyperaldosteronism. E :

Page 285: Masterclass Book Part 2

MOHAMMED IS-HAG 284

52. A 32-year-old man is referred to your clinic by his general practitioner who has diagnosed him as having hypogonadotrophic hypogonadism. He went to see his GP because of difficulty maintaining erections and concerns regarding the size of his penis, which he regards as small. He went through puberty later than his peers at school, though thinks he started by age 16. He has attained a normal adult height of 1.78 m, and weighs 108 kg; BMI is therefore 34 kg/m2. He shaves most days. Examination reveals scant axillary and body hair, normal pubic hair, stretched penile length of 10.4cm and testicular volume of 25ml (orchidometer assessment). Testosterone (9am) is 8nmol/L (normal 10-35 nmol/L), LH 4IU/L (1.3-13) and FSH 3.6 IU/L (0.9-15). Thyroid function tests performed at GP surgery are reported as normal and prolactin was 40 mmol/L (10-110). Which of the following statements is correct? A : He has hypogonadotrophic hypogonadism. B : Luteinizing hormone (LH) stimulates spermatogenesis. C : MRI pituitary is indicated. D : Measurement of sex hormone binding globulin (SHBG) would be useful. E : Full pituitary function testing should be conducted. Comment : Simplistically, LH stimulates testosterone production and follicle-stimulating hormone (FSH) spermatogenesis, so B is wrong. This man has a normal testicular volume (20-30ml) and therefore is highly unlikely to be hypogonadotrophic. Penile length is also normal (mean stretched length of flaccid erectile tissue 12.4±2.7cm in Caucasian males, 14.6cm African/African Caribbean descent, 10.6 cm Asian males) and he shaves daily with normal pubic hair all making hypogonadism unlikely. He has no evidence of pituitary insufficiency and his low testosterone concentration is likely to be a reflection of lowered SHBG concentrations due to his obesity (insulin downregulates SHBG expression and secretion and thus the hyperinsulinaemia of obesity leads to lower SHBG levels). Measurement of his SHBG would confirm low levels and imply that free testosterone concentrations are normal (some people use a ‘free androgen index’ for the calculated free testosterone though there is debate about its utility in this situation). His sexual difficulties are likely to have other origins and counselling may prove the best intervention here. D : 53. A 25-year-old man presents to the endocrine clinic complaining that he and his wife have been trying to conceive for almost 2 years without success. His wife has a 2-year-old daughter by a previous partner and so the patient is concerned that there may be ’something wrong with him‘. On further questioning he admits that he has only ever shaved twice a week, has poor libido and had a left orchidopexy as a small child. On examination he is a tall gentleman with very little facial hair, subtle bilateral gynaecomastia and small firm testis, approximately 5mls in volume. Baseline tests reveal a testosterone of 10.1nmol/l, LH 17 IU/l and FSH 23 IU/l. The single most important test in confirming the diagnosis is: A : A semen analysis B : An MRI pituitary C : Measurement of plasma estradiol levels D : A karyotype analysis E : A testicular biopsy Comment : This gentleman was suffering from Klinefelter’s syndrome, which not uncommonly presents in adult life rather than at adolescence. This syndrome is characterised by dysgenesis of the seminiferous tubules and is a common cause of primary hypogonadism and male infertility. Karyotype analysis of unselected newborns has estimated an incidence of 1 per 800 to 1000 males, the most common human chromosomal abnormality. Patients are phenotypically male with testes of usually < 3cm in length and azoospermia. Gynaecomastia is also common. Karyotype analysis typically demonstrates the 47, XXY karyotype. Patients show poor to normal virilization at puberty and tend to have a tall stature due to disproportionately long legs.Testosterone levels vary but are usually decreased and gonadotrophins are raised – particularly FSH. Plasma estradiol levels are usually normal or elevated, similarly SHBG.Undescended testes are 3 times as common as in normal boys. Prepubertal studies indicate that children with the 47, XXY Karyotype have lower birth weights, smaller mean head circumferences, a slightly increased risk of congenital anomalies, height percentiles that increase with age, a lower verbal IQ than normal boys and poor motor and muscle tone. Associated abnormalities include an increased frequency of diabetes mellitus; 19% of patients have been reported to have impaired glucose tolerance and 8% to be diabetic. 47 XXY patients with gynaecomastia also have an increased predisposition to cancer of the breast. In addition, 20-50% of boys 8 years or older with primary mediastinal germ cell tumours have Klinefelter’s syndrome.25% of adults with Klinefelter’s have osteoporosis, although this is uncommon in patients on testosterone replacement. Chronic lung disease and varicose veins are also thought to be more common. It seems that normal complements of germ cells are present in these patients in early foetal life but during late gestation and early infancy there is a dramatic loss of spermatogonia. In adult patients, permatogenesis may rarely be present but most fertile patients have proved to have sex chromosome mosaicism (46, XY/47 XXY) and often lack the features which distinguish them from typical patients with Klinefelter’s syndrome. The technique of

Page 286: Masterclass Book Part 2

MOHAMMED IS-HAG 285

intracytoplasmic sperm injection (ICSI) has been used with some success in this group of patients – however, there is also an increased risk of trisomy 21 in children of patients with Klinefelter’s syndrome. The mainstay of treatment is testosterone replacement. Patients diagnosed pre-pubertally may be given small doses of testosterone intramuscularly (e.g. 50mg im monthly) to initiate puberty and avoid the psychological and physical consequences of hypogonadism. The dose may be gradually increased (100mg im, monthly) once bone age has advanced to allow full growth potential to be realised. Once the final height has been reached an adult replacement dose of testosterone, e.g. 250mg im 3 weekly. Alternatively testosterone patches or the newer gels or buccal preparations may be used in adults. D : 54. A 15-year-old girl is admitted for parathyroidectomy. Her mother had had the same operation 16 years previously though it had had to be repeated 6 months later because of recurrent hypercalcaemia. Investigations showed: Calcium: 2.88 mmol/l Phosphate: 0.81 mmol/l Creatinine: 70 micromol/l Albumin: 42g/l PTH: 36 ng/l (10-65) Urinary calcium excretion: 2 mmol/24 hrs (normal <7.5 mmol/24 hrs) Urine creatinine concentration 6 mmol/l What is the most likely diagnosis? A : Primary hyperparathyroidism B : MEN 1 C : MEN 2a D : MEN 2b E : Familial benign hypocalciuric hypercalcaemia. Comment : Calcium excretion is low whereas it is usually increased in primary hyperparathyroidism. A useful tool for assessing this is the calcium clearance/creatinine clearance ratio: {[urine Ca++]X[plasma creat]}/{[plasma Ca++]X[urine creat]} (NB: correct plasma creatinine concentrations to mmol/l from micromol/l)A value > 0.01 is suggestive of hyperparathyroidism. A value less than this implies inappropriately low calcium excretion (corrected for plasma calcium and renal function)1. Certainly the presentation of hypercalcaemia in such a young individual should prompt investigation into the possibility of MEN 1 or MEN 2a if hyperparathyroidism is confirmed. A family history would also be important. MEN 2b is not associated with hyperparathyroidism and so need not be considered here. In this case the family history (particularly of failed parathyroidectomy), high calcium, normal parathyroid hormone (PTH) and low calcium excretion suggest the diagnosis of familial benign hypocalciuric hypercalcaemia (FBHH). This is a disorder of the calcium sensing receptor and is inherited in an autosomal dominant manner. Individuals are asymptomatic, calcium concentrations are usually less than 3 mmol/l and PTH is usually in the normal range (though inappropriately high for the prevailing calcium concentrations). Urine calcium excretion is reduced. The homozygous form presents in the neonatal period with severe hypercalcaemia and requires urgent total parathyroidectomy to prevent death from rib fractures amongst other complications2. E : 55. A 42-year-old man has proven growth hormone deficiency post pituitary surgery for pituitary adenoma. He is hyperlipidaemic with a family history of ischaemic heart disease. He complains that he has lost his energy and complains that he finds he is unable to work out in the gym as he used to. Other pituitary axes are either normal or adequately replaced. He is interested in a trial of growth hormone therapy. Which of the following statements are true? A : He should be commenced on growth hormone as there is evidence that it will reduce his risk of

cardiovascular mortality and morbidity. B : He should not commence on growth hormone as it may adversely affect his lipid profile. C : Treatment with growth hormone may improve his lipid profile and exercise tolerance. D : Growth hormone treatment is contraindicated due to the risk of re-growth of his pituitary tumour. E : Growth hormone therapy is important to reduce his risk of osteoporotic fracture. Comment : Growth hormone deficiency is associated with loss of lean body mass and increase in fat mass, hyperlipidaemia, increased risk of osteoporosis and cardiovascular morbidity and mortality. However treatment with GH in adults is controversial and many clinicians only consider it in patients who are symptomatic in addition to biochemically GH deficient. This is because GH is relatively expensive and as yet there is no hard end-point data on reduction of cardiovascular adverse events or osteoporotic fractures. Several studies have shown improvement in surrogate markers of these diseases, including markers of bone turnover and plasma lipids. Wellbeing scores and exercise tolerance have also been shown to improve, hence the concentration on symptomatic patients. Although there is a theoretical concern about promoting tumour growth, the dose of GH is titrated against IGF-1 to try to prevent over-replacement. So far there is no evidence to suggest an increased risk of new tumour formation or recurrence of a previously treated pituitary tumour. C :

Page 287: Masterclass Book Part 2

MOHAMMED IS-HAG 286

56. In a male patient found to have an elevated prolactin level which of the following is most likely to be the cause: A : Addison's disease B : Phaeochromocytoma C : Primary hypothyroidism D : Insulinoma E : Primary hyperparathyroidism. Comment : Thyrotrophin-releasing hormone (TRH) will be elevated in patients with primary hypothyroidism and TRH stimulates prolactin release. C : 57.

A 42-year-old woman was referred by her GP on account of thyrotoxicosis picked up during assessment of a sore neck. Further history revealed she had been feeling out of sorts for the previous week with aching joints and muscles, which had now been replaced by discomfort in the neck. On examination there was a moderate goitre, slightly more prominent on the right, which was mildly tender to palpation. There was lid lag and eyelid retraction, and she was clinically hyperthyroid. Free T4 was 29 pmol/l (9-20), free T3 11 pmol/l (3.0-7.5), TSH <0.03 mU/l (0.4-4). The thyroid scintigram is shown (see image). Which of the following is true? A : After 6 months of a blocking dose of carbimazole and thyroxine, there is about a 50% chance of cure. B : Medical therapy, radioiodine and subtotal thyroidectomy should be discussed as options with the patient. C : She is most likely to have Riedel’s thyroiditis D : Oral prednisolone would be likely to relieve symptoms rapidly. E : Anti thyroid peroxisomal antibodies are likely to be present in high titre. Comment : The presentation with symptoms suggestive of a viral infection, thyroid pain and tenderness, thyrotoxicosis with free T4 and T3 in proportion, and the silent scintigram all point strongly to de Quervain’s thyroiditis. This destructive thyroiditis is most commonly associated with viral infection, and has a biphasic pattern, with 2-3 months of thyrotoxicosis typically followed by a similar period of hypothyroidism before reestablishment of euthyroidism. Treatment is with simple analgesia or steroids if necessary, with symptoms settling rapidly on oral prednisolone. The condition is only around one fifth as common as Graves’ disease. The ESR is generally very high. Occasionally the early phase of Hashimoto’s thyroiditis can be associated with thyroid discomfort and hyperthyroidism, but more commonly it is insidious, silent and progressive with high autoantibody titres, unlike de Quervain’s thyroiditis, where any immunological perturbation is transitory and mild. D :

Page 288: Masterclass Book Part 2

MOHAMMED IS-HAG 287

1. A 78 yr old man has been ventilated on ICU for 10 days following surgical repair of a ruptured abdominal aortic aneurysm. His respiratory function is deteriorating and it is thought that he has developed a ventilator-associated pneumonia. Which of the following drugs might be most suitable for treatment? A : Benzyl penicillin B : Cefuroxime C : Augmentin D : Vancomycin E : Piperacillin/tazobactam (Tazocin). Comment : Important organisms to consider in this context are resistant gram negative rods, including pseudomonas. The only drug with such cover in this list is tazocin, but other anti-pseudomonal regimens may be suitable. A and B cover community acquired pneumonias but have weaker gram negative cover. E : 2. Which of the following is NOT true for aminoglycosides: A : They cause cumulative ototoxicity. B : They can cause agranulocytosis. C : They commonly causes pseudomembranous colitis. D : They are nephrotoxic. E : They can cause neuromuscular blockade in myasthenics and during anaesthesia. Comment : Pseudomembranous colitis is rare with aminoglycosides, probably because they have little activity against anaerobic gut flora. Long-term therapy and coexistant renal impairment increase the risk of ototoxicity. Neutropenia and aplastic anaemia may also be seen. Nephrotoxicity is usually of an ATN/proteinuria type and is usually reversible on withdrawal of the drug. Neurotoxicity is due to reduction in Ach release at motor neurone terminals. C : 3. Which of the following drugs might be suitable for treatment of enterococcal infection? A : Benzyl penicillin B : Ceftazidime C : Cefuroxime D : Metronidazole E : Vancomycin. Comment : Enterococci are may cause sepsis and endocarditis. Some are ampicillin sensitive, all are cephalosporin resistant - but most are vancomycin sensitive. 4. In addition to ceftriaxone, sometimes ampicillin is added to 'blind' antibiotic regimens in cases of meningitis. What organism is this intended to cover in such a case? A : Penicillin-resistant pneumococci B : Meningococcus group B C : Meningococcus group C D : Listeria E : Enterococci. Comment : Listeria can cause disease in the immunosuppressed (including pregnant women). Ampicillin is the drug of choice. A-C are covered well by ceftriaxone and E is not an important cause of meningitis except in special cases. D : 5. A 26-year-old woman with epilepsy has been diagnosed with pulmonary tuberculosis and is about to be started on quadruple therapy. The following need to be discussed with her except: A : Method of contraception B : The need for screening for ocular complications with ethambutol C : Interactions with other medications D : Discoloration of urine due to isoniazid E : Compliance. Comment : Isoniazid inhibits carbamazepine and phenytoin and ethosuximide. Her anti-epileptic medication will need reviewing. She should also be reminded to check with a doctor or pharmacist before starting other prescribed or over the counter medications. Rifampicin turns the body fluids including urine orange, which can be alarming to a patient if they are not pre-warned. Methods of contraception will be important for any sexually active female. Rifampicin is a hepatic enzyme inducer. Accelerated metabolism leads to a reduced contraceptive effect of both the progesterone only and combined pill. Ethambutol has ocular toxicity and vision should be checked routinely at follow-up. D : 6. The rash in smallpox differs from the rash in chickenpox in that the smallpox rash typically:

Page 289: Masterclass Book Part 2

MOHAMMED IS-HAG 288

A : is central B : is centrifugal C : spares the soles of the feet and palms of the hands D : crops with lesions in different stages E : is very superficial. Comment : Chickenpox, which is clearly very much the more common disease, causes a rash which crops, is typically central, and does not affect the hands and feet. Smallpox rash is more distal and causes deep lesions which leave scars (typically on the face). B : 7. A 26-year-old female presents with a short history of confusion, diarrhoea and breathlessness. On examination she is pyrexial at 38.5 C, pulse 120 bpm, Bp 80/60 and respiratory rate 26 breaths/min. She has a faint blanching macular rash across the trunk. She is disorientated without meningism. Which of the following diagnoses is most likely? A : Salmonella enteritidis infection B : Pneumococcal meningitis C : Anaphylaxis D : Ecstasy overdose E : Toxic shock syndrome. Comment : The presentation with fever, diarrhoea, shock and a macular rash are characteristic of staphylococcal toxic shock syndrome due to focal infection with a toxin-producing strain of S. aureus. Confusion, breathlessness (due to metabolic acidosis) and oliguria are commonly present. The toxin acts as a superantigen bypassing the normal antigen-restricted pathway of T-cell activation leading to widespread cytokine release shock and organ failure. Approximately 50% of cases occur in young women due to vaginal infection with S. aureus at the time of menstruation. A retained vaginal tampon increases the risk and should be looked for in this type of presentation. A similar syndrome may also be seen with toxin producing streptococci. E : 8. Which of the following forms of acute viral hepatitis has a much higher mortality in pregnant than non-pregnant females? A : Hepatitis A B : Hepatitis B C : Hepatitis C D : Hepatitis E E : Hepatitis G. Comment : The mortality of hepatitis E in pregnant women, particularly in the third trimester, are of the order of 20-30%. It is transmitted by the faecal oral route like hepatitis A and epidemics have been associated with faecal contamination of water. There is no chronic phase of infection. Hepatitis G virus is currently not known to cause any human disease, not even hepatitis, despite its name. D : 9. Which of the following is least characteristic of candidal infections? A : Vulvovaginitis B : Opthalmitis C : Mucositis D : Line associated sepsis E : Pneumonia. Comment : The commonest manifestation of candidal infection is mucosal disease (eg. Vulvovaginitis and oral thrush) and can occur inimmunocompetent and immunocompromised individuals alike. Disseminated disease is seen most often in the immunocompromised patient, those with intravenous catheters and intravenous drug users. Opthalmitis is a well recognized complication of disseminated disease. It should always be checked for in all patients with candidaemia. E : 10. A 25-year-old woman complains of an increasing offensive vaginal discharge over the last 2 weeks. She is feeling feverish and has lower abdominal pains. She is not sexually active and has had a sexual health screen after she finished her last relationship some 3 months ago when a cervical erosion was noted. She has never had a sexually transmitted infection. She is on no medication and her last normal menstrual period was 17 days ago. She discontinued her oral contraceptive some 4 weeks ago. What is the most likely diagnosis? A : Candidal infection B : Neisseria gonorrhoeae infection C : Cervical malignancy D : Retained foreign body in the vagina E : Pregnancy. Sexually transmitted infections are highly unlikely in view of the history.Toxic shock syndrome is associated with retained tampons and can present with fevers, hypotension and rash. Unless diagnosed early patients may deteriorate rapidly and fatalities have been reported. Removal of the foreign body should be accompanied with antibiotic therapy regardless of the risk for STI. D :

Page 290: Masterclass Book Part 2

MOHAMMED IS-HAG 289

11.

PLATE51 A 54-year-old woman received an allogeneic bone marrow transplant for acute myeloid leukaemia. Eight weeks post transplant she complains of blurred vision and the lesion shown in the image is seen on fundoscopy. Which of the following is the likely diagnosis? A : Cytomegalovirus retinitis B : Bacterial endophthalmitis C : Reactivation of toxoplasmosis D : Leukaemic deposits in the retina E : Candida endophthalmitis. Comment : The image shown includes a black scar on the retina typical of old toxoplasma choroidoretinitis. Such scars are often found as incidental findings on fundoscopy and indicate a risk for subsequent reactivation. In this case there are also exudates within the old scar plus a new lesion (underneath old scar) with florid exudate indicating reactivation of toxoplasmosis as a consequence of intense immunosuppression. None of the other diagnostic options given will have black areas. CMV retinitis typically consists of haemorrhage plus exudates (cheese and tomato pizza appearance). Candida appears as white lesions extending from the retina into the vitreous. Bacterial endophthalmitis generally has signs of inflammation throughout the eye. C : 12. A 70-year-old man with carcinoma of the prostate and bone metastases was admitted to the hospice for symptom control. On admission he had bone pain and anorexia. Five days later he became breathless and confused. What is the most likely cause of his symptoms? A : Carcinomatous lymphangitis B : Left ventricular failure (LVF) C : Lung fibrosis due to radiotherapy D : Lung secondaries E : Pneumonia. Comment:Carcinoma of the prostate rarely metastasises to the lungs and the acuteness of this illness makes LVFor pneumonia more likely.He is toxic and confused and pneumonia is therefore the choice.E: 13. Listeria monocytogenes does NOT typically cause which of the following? A : Disease during pregnancy B : Food poisoning syndrome C : Neonatal infection D : Meningitis E : Septicaemia. Comment : Listeria is spread by ingestion of foodstuffs such as paté and soft cheese, but typically presents with non gastro-intestinal disease, particularly meningitis in immunosuppressed individuals (including pregnant women). Listeria has been occasionally implicated in outbreaks of gastroenteritis, but in most patients this is mild and self-limiting and thus does not come to medical attention. B :

Page 291: Masterclass Book Part 2

MOHAMMED IS-HAG 290

14.

PLATE 16 A 32-year-old male presents with fever, dry cough and breathlessness. Resting oxygen saturation on air is 97% and his chest radiograph was reported as normal. This appearance of his tongue was noted (see image). What is the most likely diagnosis? A : Pneumocytsis carinii pneumonia B : Q fever C : Legionnaire’s disease D : Infective endocarditis E : Pulmonary candidiasis. Comment : There is a white patch on the side of the tongue indicating oral hairy leukoplakia (OHL). OHL is almost pathognomonic of underlying immunodeficiency due to HIV infection. OHL is generally seen when the CD4 count is less than 350 and is due to local Ebstein Barr virus infection of the skin. Pneumocytsis carinii pneumonia (PCP) is one of the commonest causes of breathlessness in patients with a CD4 count of less than 200. The presentation described is typical of early PCP when the patient may still have a normal chest radiograph and only becomes hypoxic on exercise. A : 15. In the management of hepatitis C infection: A : treatment of acute infection is rarely successful B : interferon gamma plus ribavirin is the treatment of choice C : therapy is well tolerated with few side effects D : genotype 1 carries a more favourable outlook E : Hepatitis A vaccination is indicated. Comment : The current gold standard (NICE approved) therapy of chronic hepatitis C infection is alpha-interferon plus ribavirin for between six and 12 months. Side effects are common including depression and neutropenia (interferon) and anaemia (ribavirin). Sustained response rates remain poor (<50%) and are lowest for genotype 1 which is the commonest genotype in Western Europe. A recent study from Germany has shown a sustained response of over 90% when treatment was initiated within six months of hepatitis C seroconversion. Hepatitis A infection is associated with a high risk of fulminant hepatic failure in patients with chronic hepatitis C infection. Patients with chronic hepatitis C should be screened for previous infection of both hepatitis A and hepatitis B and offered vaccination if non-immune. E :

Page 292: Masterclass Book Part 2

MOHAMMED IS-HAG 291

16.

A 32-year-old man presented to A&E following a grand mal seizure. He gave a history of occasional blackouts over the previous five years. He entered the UK from India 3 years earlier and was otherwise well with no fever and normal blood tests. Unenhanced CT brain scan is shown (see image). Which of the following is the likely diagnosis? A : Cerebral tuberculosis B : Cerebral metastases C : Schistosomiasis D : Cystercicosis E : Cerebral toxoplasmosis. Comment : The CT scan shows small calcified lesions with no surrounding mass effect or oedema, this is the typical appearance of neurocystercicosis. Cystercicosis due to the pork tapeworm Taenia solium is prevalent in Central and South America and parts of Asia including the Punjab. Tissue cysts may be found in many tissues including skeletal and cardiac muscle, retina and brain. Active cysts may be surrounded by oedema but as the organisms die the cysts become inactive and calcified. Many cases are asymptomatic. Epilepsy is the commonest clinical problem but encephalitis, mass lesions and hydrocephalus may also occur. Toxoplasmosis and tuberculosis form cerebral abscesses with a low-density area, surrounding oedema and enhancement with contrast. Schistosomiasis may rarely produce focal CNS lesions.Schistsomiasis is found in Africa and parts of the far East and this patient has no epidemiological exposure. D : 17. An intubated patient on the ICU for 8 days following a road traffic accident has a persistent fever and some lung shadowing. Which of the following organisms is most likely to be involved? A : Streptococcus pneumoniae B : Staphylococcus epidermidis C : Staphylococcus aureus D : Pseudomonas aeruginosa E : Legionella pneumophila. Comment : Pseudomonas species are very common causes of nosocomial pneumonia on the ICU. Strep pneumoniae is a very common cause of community-acquired pneumonia. Staphylococcal disease due to methicillin-resistant S. aureus (MRSA) or methicillin-susceptible S. aureus (MSSA) is also reasonably common. Legionella is usually community-acquired and can lead to ICU admission, but is now an uncommon nosocomial infection. 18. The case fatality rate for smallpox is approximately: A : 100% B : 5% C : 30% D : 70% E : 95%. Comment : Ordinary smallpox (variola major) accounts for the majority of cases and kills about one in three people. Other forms are often milder, particularly in vaccinated patients. C :

Page 293: Masterclass Book Part 2

MOHAMMED IS-HAG 292

19. Severe acute respiratory syndrome (SARS) is caused by? A : A coronavirus B : A rhinovirus C : A herpesvirus D : An adenovirus E : A metapneumovirus . Comment : SARS is caused by a novel coronavirus. Other human coronaviruses can cause upper respiratory tract symptoms, but generally do not cause significant pneumonic illness. A : 20. Which of the following investigations is unlikely to be helpful in the investigation of a patient with suspected meningococcal disease in the absence of meningism. A : Blood cultures B : Skin scrape of petechial rash C : Arterial blood gas analysis D : Full blood count E : Lumbar puncture. Comment : Lumbar puncture is a potentially hazardous procedure in a septicaemic patient, particularly in the context of abnormal clotting or reduced conscious level, and is unlikely to yield a positive result in the absence of meningism. Furthermore, a negative lumbar puncture can be misleading as it does not preclude the patient having meningococcal disease.Blood culture results may be delayed, and will not influence immediate treatment decisions, although they may enable confirmation of the diagnosis for public health purposes.Skin scrape of the petechial rash may enable rapid identification of the causative organism by microscopy and Gram stain. Thrombocytopaenia, neutropaenia and an arterial base defecit lower than -5 are indicators of a poor prognosis. E : 21. A 67 yr old man with chronic renal failure (cause unknown) for which he receives regular haemodialysis treatment has been admitted to the renal ward on many occasions with access difficulties. He is now admitted with fever and malaise. Blood cultures grow a vancomycin-resistant enterococcus (VRE). Which of the following drugs would be most suitable treatment? A : Meropenem B : Linezolid C : Enterocid D : Tobramycin E : Septrin. Comment : VREs are an emerging problem in hospitals, where extensive glycopeptide use may select for resistant strains. Some are teicoplanin sensitive. The new agents Linezolid and Synercid may be active. Septrin is active against some highly resistant gram negative bacteria such as Stenotrophomonas maltophilia. B : 22. The last case of endemic smallpox occured in 1977 in which country? A : USA B : Iraq C : USSR D : Somalia E : The Gambia. Comment : The last case occured in Somalia. Isolates were kept in a single lab in the USSR and the Communicable Diseases Centre (CDC) Atlanta, USA, but there is concern that stocks may have reached other countries. D : 23. Which of the following is an effective therapy for hepatitis B virus infection? A : Adefovir B : Interferon-gamma C : Ribavirin D : Ritonavir E : Milk thistle. Comment : Adefovir is a novel therapy for HBV which appears to be successful in recent trials of chronic infection. The rate of accumulation of resistance mutations is much lower than lamivudine. Unlike interferon-alpha, interferon-gamma is not a drug used against HBV (or HCV). A : 24. Foot and mouth disease in cattle is caused by an RNA virus that is genetically related to viruses such as the Hepatitis A virus. Which is the closest relative to hepatitis A virus? A : Hepatitis B virus B : Hepatitis C virus C : Hepatitis D virus D : Coxsackie A virus E : Japanese B encephalitis virus. Comment : All these viruses fall into the picornavirus group. D :

Page 294: Masterclass Book Part 2

MOHAMMED IS-HAG 293

25.

A 74-year-old patient previously treated for pulmonary tuberculosis presents with cough and a 250 ml haemoptysis. The chest radiograph is shown (see image). Which of the following diagnoses is correct? A : Active tuberculosis B : Carcinoma of the bronchus C : Pulmonary infarction D : Aspergillus infection E : Bacterial pneumonia. Comment : The patient has scaring and cavitation from old tuberculosis. The lesion in the right apex is a fungal ball within and existing pulmonary cavity – an aspergilloma. The diagnosis can be confirmed by CT scanning and growth of aspergillus from respiratory specimens. Massive haemoptysis in a person with known cavitary lung disease is highly suggestive on the development of an aspergilloma. Aspergillomata respond poorly to antifungal therapy. Definitive therapy is surgical and bronchial artery embolization may be used to stop haemorrhage. D : 26. Superantigen toxin release from Staphylococcus aureus causes: A : T cell toxin syndrome B : scalded skin syndrome C : toxic shock syndrome D : food poisoning E : abscess formation. Comment : Toxic shock syndrome is characterized by fever, shock and rash and may lead to multi-organ dysfunction and death. Toxic shock syndrome toxin type 1 (TSST-1) acts as a superantigen by binding both to MHC and T cell receptors leading to cell activation. Subsequent release of pro-inflammatory mediators may then initiate the pathophysiological changes leading to shock and organ failure. Other toxins released by some strains of S. aureus cause scalded skin syndrome or food poisoning but these do not act as superantigens. C : 27. A 40-year-old female traveller recently returned from a 2-month trip to India presents with a week’s history of fever, malaise, myalgia and headache. Examination is non-localising and a full blood count shows Hb 13.6, WCC 14.2 Plt 148 neutrophils 12.4. A malaria film is negative. Which of the following would be unlikely? A : Rickettsial disease B : Amoebic liver abscess C : Leptospirosis D : Urinary tract infection E : Sepsis from pyodermic insect bites.

Page 295: Masterclass Book Part 2

MOHAMMED IS-HAG 294

Comment : Other than rickettsial infection, which usually presents with a localised lymphadenopathy +/- eschar and rash and normal white blood count, all the other conditions listed can be associated with peripheral neutrophilia. Malaria is rarely associated with raised neutrophils but should always be excluded in travellers returning from endemic areas. Amoebic liver abscess is associated with neutrophilia and not eosinophilia. It should be remembered that a large proportion of travellers with fever would have cosmopolitan rather than exotic causes for their fever, but that a careful history and examination should be undertaken to exclude the exotic causes. A : 28. The following are included in standard childhood immunization schedules in the UK, except: A : Mumps B : Tetanus C : Varicella zoster D : BCG E : Meningococcus Group C. Comment : Although there is a vaccination against Varicella zoster it is not included in the routine childhood immunizations in the UK unlike some other countries e.g. USA. Immunization schedule BCG, 1st Hep B Birth (if at risk) 2nd and 3rd Hep B 1 & 2 months (if at risk) HiB 2, 3 and 4 months Diptheria/Pertussis/Tetanus (DPT) Meningococcus Group C Oral Polio vaccine (OPV) 4th Hep B 12 months (if at risk) Measles/Mumps/Rubella (MMR) 13 months D/T 4–5 years OPV 2nd MMR BCG 10–14 years C : 29. A 40-year old man from Saudi Arabia (in UK 4 days) presents with painful episodes of trismus. He has never been vaccinated against tetanus and sustained a slight cut to his foot while walking barefoot in his garden in Saudi Arabia 3 days before arriving in the UK. You suspect tetanus and he then has an episode of generalised tetanus with respiratory arrest. Which of the following is incorrect as part of his management? A : Admit to side ward in intensive care unit B : IV gentamicin to kill any Clostridium tetani bacteria left in wound C : Tetanus toxoid as he will not be immune to tetanus even after recovery from infection D : Clean, debride and dress his foot lesion E : IM tetanus antitoxin. Comment : These patients should be nursed in a quiet environment as noises often trigger generalized spasms. If he is having generalized spasms he will require ventilation and muscle relaxants with or without neuromuscular blockade. IM antitoxin should be given immediately to mop up any unbound tetanus toxin. The antibiotic of choice is IV metronidazole. After recovery from infection he will still have no immunity to tetanus and the first dose of tetanus toxoid (at a different site to the antitoxin) is therefore given, with two more doses to follow. B : 30. Which of the following is NOT a cause of bloody diarrhoea in travellers? A : Giardia B : Shigella C : Entamoeba histolytica D : Schistosoma mansoni E : E coli. Comment : Giardia is an important cause of diarrhoea but does not cause bloody diarrhoea. Only certain E coli strains cause bloody diarrhoea. A : 31. Which of the following viruses is not a significant cause of diarrhoea in the immunocompetent? A : Norwalk agent B : Rotavirus C : Adenovirus D : Cytomegalovirus (CMV) E : Astrovirus. Comment : CMV is a cause of gut pathology in the immunosuppressed, especially late-stage HIV, but not in individuals with normal immunity. D :

Page 296: Masterclass Book Part 2

MOHAMMED IS-HAG 295

32.

PLATE 52 A 28-year-old woman has noticed this blistering eruption appear over her body every 2-3 months for the last year (see image). On each occasion she has had symptoms of genital soreness and has taken some Potassium citrate and cranberry juice. What is the most likely diagnosis? A : Fixed drug eruption B : Erythema multiforme C : Disseminated gonoccacal disease D : Bullous phemphigus E : Urticaria. Comment : Erythema multiforme is a common complication of genital herpes infection. The target-like lesions appear either as the genital lesion is evolving or soon after. Attacks can be controlled with continuous oral aciclovir to prevent HSV reactivation, but aciclovir has no effect on the erythema multiforme during an acute attack. B : 33. Which of the following is NOT a potential treatment for cytomegalovirus (CMV) infection? A : Ganciclovir B : Foscarnet C : Cidofovir D : Valganciclovir E : Aciclovir. Comment : Aciclovir is effective therapy for Herpes Simplex and Varicella Zoster but not CMV. Valganciclovir is a new oral agent that shows promise in the treatment of CMV retinitis in HIV-infected individuals. E : 34. Which of the following is not a recognised side effect of anti-HIV therapy? A : Facial thinning B : Hypocalcaemia C : Lactic acidosis D : Hypertriglyceridaemia E : Nail dystrophy. Comment : Anti-HIV drugs are associated with numerous side effects but are not linked to hypocalcaemia. Some data, however, do suggest a link with premature osteoporosis. Facial thinning is caused by fat redistribution (lipodystrophy) associated with insulin resistance, and disturbed fat and glucose metabolism. Zidovudine induces a grey discolouration of the nails in persons with pigmented skins. Indinavir crystallises in the renal tract leading to renal impairment and stones. One of the most serious side effects is lactic acidosis and liver failure associated with nucleoside reverse transcriptase inhibitors. B :

Page 297: Masterclass Book Part 2

MOHAMMED IS-HAG 296

35.

PLATE 53 A 28-year-old man presents with a lesion in his mouth (see image). What is the diagnosis? A : HIV infection B : Squamous cell carcinoma C : Oral syphilis D : Normal variant E : Kaposi’s Sarcoma Comment : The hard palate is a common site for Kaposi’s sarcoma (KS) and this is the classical appearance. The patient may be unaware of the lesion. Most commonly there will, in addition, be cutaneous KS lesions, but these may be absent or inconspicuous. This presentation would be most consistent with underlying HIV infection and the development of KS would be an AIDS-defining illness. KS may be seen on the legs of elderly men or complicating other causes of significant immunodeficiency such as after transplantation. Endemic KS is also seen in parts of Africa where young HIV-negative adults may be affected. Lymph nodes, genito-urinary tract and lung involvement may be seen in advanced cases. All types of KS are closely linked to the presence of human herpesvirus 8. E : 36. Current meningococcal vaccines cover which major groups? A : Group A only B : Group B only C : Groups B and C D : All groups E : Groups A and C Comment :There is also a vaccine which covers groups Y and W135, in addition to groups A and C. E : 37. A 74 yr old woman with chronic leg ulceration for which she has been admitted to hospital many times is admitted once again with fever and malaise thought to be due to infection of these ulcers. Swabs of the ulcer grow Methicillin-resistant Staphylococcus aureus (MRSA). Which of the drugs listed can be used to treat this condition? A : Flucloxacillin B : Augmentin C : Vancomycin D : Cefuroxime E : Meropenem. Comment : MRSA is resistant to beta-lactam antibiotics. Glycopeptides such as Vancomycin are commonly used. An alternative is Teicoplanin. Other drugs such as gentamicin, rifampicin and doxycycline may be active. C :

Page 298: Masterclass Book Part 2

MOHAMMED IS-HAG 297

38. Which is an effective therapy for HBV? A : Adefovir B : Interferon-gamma C : Ribavirin D : Nevirapine E : Milk thistle Comment : Adefovir is a novel oral therapy for HBV (reverse transcriptase inhibitor) which appears very effective with low rates of resistance. Interferon-alpha is used but there is no current role for gamma. Ribavirin is an effective therapy in combination with interferon-alpha for HCV but not HBV, which has a different polymerase (the mode of action of ribavirin is not clear, but it is known to interfere with polymerase function).Nevirapine is not used, while milk thistle is often taken by patients with chronic viral hepatitis, without a proven influence on the outcome. A : 39. The following are well absorbed and systemically effective after oral administration, except: A : Penicillin B : Metronidazole C : Amphotericin B D : Ciprofloxacin E : Clindamycin. Comment : Amphotericin B is not effective systemically when taken orally. Severe fungal infections require intravenous administration. All the other drugs give effective systemical levels from oral administration. C : 40. Which drug is used for the treatment of both HIV and HBV? A : Zidovudine (AZT) B : Didanosine (DDI) C : Lamivudine (3TC) D : Interferon alpha E : Ritonavir Comment : Lamivudine acts as a RT inhibitor in HIV infection and blocks the same activity in HBV infection. HBV is not a classical retrovirus but does have an RT step in its life cycle. AZT and DDI are not used for HBV although they have similar activities in HIV. Interferon alpha is effective therapy for HBV but not HIV, and ritonavir is a protease inhibitor specific for HIV. C : 41. A 20-year-old woman has recently given birth to a well full-term baby delivered vaginally. Some 5 days after delivery she is told by her regular sexual partner that he has recently been treated for genital chlamydial infection. The doctor looking after her partner has advised him to ensure that the baby is taken to a doctor for treatment of possible chlamydial infection. This is because if untreated the baby may develop all of the following EXCEPT: A : Neonatal opthalmia B : Sinusitis C : Pneumonia D : Failure to thrive E : Pelvic inflammatory disease/ epididymitis. Babies delivered through an infected genital tract are susceptible to a variety of severe manifestations of bacterial and viral STIs. Infective agents can become established in any mucous membrane site. Inhalation can lead to upper and lower respiratory tract problems. Late presentations of sinusitis and failure to thrive have been described.Upper genital tract infection in neonates does not occur. E : 42. Which would be the best antibiotic regime to use in a patient who has spreading methicillin-resistant Staphylococcus aureus (MRSA) infection of a skin wound? A : Flucloxacillin B : Gentamicin C : Vancomycin D : Rifampicin E : Doxycycline. Comment : MRSA is often sensitive to all of these antibiotics, excepting flucloxacillin. Rifampicin and doxycycline may be used in combination in certain situations. Gentamicin can be added to treat staphylococcal infections (e.g. endocarditis) but is not usually used in cellulitis. C : 43. Vaccines exist in the UK for which major groups of meningococcus? A : B B : C C : B and C D : A and C E : A, B and C.

Page 299: Masterclass Book Part 2

MOHAMMED IS-HAG 298

Comment : The vaccine for A has been used for travellers for some years, and the vaccine for C has been introduced more recently for risk groups. No vaccine for B exists yet in the UK, although a local version is available in Cuba. D : 44. Which of the following is NOT a recognised risk factor for peritoneal tuberculosis? A : Peritoneal dialysis B : Chronic hepatitis B infection C : Cirrhosis D : Diabetes mellitus E : Previous small bowel surgery. Comment : While there are anecdotal reports of tuberculosis localising at the site of previous trauma, there is little controlled data to confirm this. Chronic liver disease, peritoneal dialysis and states of relative immune suppression all create a predisposition to peritoneal tuberculosis. E : 45. In previous centuries Bacillus anthracis was a significant cause of mortality in: A : Travellers B : Bakers C : Sheep farmers D : Millers E : Woolsorters. Comment : Woolsorters were at particularly high risk due to serious inhalational disease. Tanners exposed to infected animals were also susceptible but tended to develop the cutaneous form of the disease. E : 46. In previous centuries Bacillus anthracis was a significant cause of mortality in: A : Travellers B : Bakers C : Sheep farmers D : Millers E : Woolsorters. Comment : Woolsorters were at particularly high risk due to serious inhalational disease. Tanners exposed to infected animals were also susceptible but tended to develop the cutaneous form of the disease. E : 47. Which of the following causes of bloody diarrhoea is more frequently not associated with fever? A : Entamoeba histolytica B : Shigella C : Campylobacter D : Clostridium difficile E : Vibrio parahaemolyticus . Comment : Fever during infection with Entamoeba histolytica occurs in fewer than 30% of cases. Amoebic liver abscess, which usually develops 2-5 months post-exposure, presents with fever, weight loss and right upper quadrant pain. A : 48. Which of the following is Bacillus anthracis resistant to? A : Septrin B : Penicillin C : Ampicillin D : Cefuroxime E : Ciprofloxacin. Comment : Penicillin is the drug of choice for the treatment of human disease caused by anthrax. Ciprofloxacin, however, is preferred for mass prophylaxis due to greater efficacy in eradicating carriage and a superior side effect profile. D : 49. What is the risk of allergy to a cephalosporin antibiotic in someone with a penicillin allergy? A : 1 in 1000 B : 1 in 100 C : 1 in 10 D : 50:50 E : virtually 100%. Comment : The risk is usually quoted as about 5-10%. C : 50. Lesbians (women who have sex with women) are at greater risk of acquiring which of the following infections: A : Candidiasis B : Human papilloma virus infection C : Herpes simplex infection D : Bacterial vaginosis E : Gonorrhoea.

Page 300: Masterclass Book Part 2

MOHAMMED IS-HAG 299

Comment : Unlike homosexual men, lesbians appear to be at lower risk through same-sex activity for most genital pathogens. The only infection commonly seen in this group - probably at higher frequency then background levels - is bacterial vaginosis. D : 51. A HIV-positive patient with a CD4 count of 150 is travelling to sub-Saharan Africa and requests vaccination advice. Which of the following would be contraindicated? A : Tetanus B : Cholera C : Meningococcal A/C vaccine D : Yellow fever E : Hepatitis A. Comment : Yellow fever vaccine is a live attenuated virus and should be avoided in immunocompromised individuals. Death from multi-organ failure has been reported following yellow fever vaccination to this group.The meningococcal A/C is a polysaccharide vaccine, cholera and hepatitis A are both inactivated viral vaccines and tetanus is a toxoid. If polio or typhoid vaccination is required then it is important to choose the correct formulation as both are available as live and inactivated vaccines. Proof of yellow fever vaccination is required by some countries prior to entry. You may provide your patient with a letter stating that they have a contraindication to vaccination, but this does not guarantee entry. Advise against travel to any region in the midst of a yellow fever outbreak. D : 52.

PLATE 54 What is the aetiological agent for this asymptomatic lesion found in an HIV patient? A : Epstein Barr virus B : Herpes simplex virus C : Candida albicans D : Candida glabrata E : Enterovirus. Comment : This lesion is called oral hairy leucoplakia and is caused by Epstein Barr virus. It is usually asymptomatic and occurs in patients with HIV infection and very rarely in immunosuppressed organ transplant recipients. It is useful because if present it is almost diagnostic of HIV infection in the absence of an organ transplant. A : 53. Hepatitis C is what kind of virus? A : A retrovirus B : A DNA virus C : A variable RNA virus D : A close relative of hepatitis B virus E : A defective helper virus. Comment : Hepatitis C virus is highly variable, as are many RNA viruses, and exists in multiple different genotypes. C :

Page 301: Masterclass Book Part 2

MOHAMMED IS-HAG 300

54. Hepatitis C is what kind of virus? A : A retrovirus B : A DNA virus C : A variable RNA virus D : A close relative of hepatitis B virus E : A defective helper virus. Comment : Hepatitis C virus is highly variable, as are many RNA viruses, and exists in multiple different genotypes. C : 55. Early treatment for acute HCV infection should be instituted... A : Immediately after any needle stick incident B : Immediately after a needle stick incident with HCV+ blood C : Upon HCV seroconversion D : Upon first detection of HCV by PCR E : If virus is not spontaneously cleared within 3 months Comment : Guidelines may vary, but the risk of transmission of HCV even after a needle stick with PCR+ blood is about 3%, so prophylactic therapy (which is used for needle stick injuries involving HIV+ blood) is not currently recommended. Seroconversion can occur quite late so this is not a good guide. PCR is a much better guide – however, since the virus can be cleared spontaneously, most clinicians would allow a short window for this to occur before instituting therapy. Therapy is very effective if started early – around 95% recovery if treated within 3 months. It is not yet clear how long treatment can be delayed before losing the efficacy. E : 56. Which of the following is NOT a significant side effect of interferon alpha therapy? A : Depression B : Auto-immune disease C : Fatigue D : Hearing disturbance E : Low white count. Comment : Interferon alpha has numerous side effects, which vary in intensity from individual to individual. Hearing disturbance is not prominent amongst these but visual disturbance can occur. D : 57. A 30-year-old male recently returned from trekking in Nepal presents with a 6-day history of bloody diarrhoea with abdominal cramps but no fevers. He has taken some antibiotics, obtained in Nepal, with little effect. Which of the following would be an unlikely cause? A : Entamoeba histolytica B : Trichuris C : Acute schistosomiasis D : Ulcerative colitis E : Clostridium difficile colitis. Comment : Schistosomiasis is not endemic in the Indian subcontinent, though would be a cause of the above elsewhere in the tropics. Heavy worm loads with Trichuris can cause bloody diarrhoea. Cosmopolitan causes of bloody diarrhoea should always always be considered as a cause despite the travel history and C. difficile disease excluded when antibiotics have been taken. C :

Page 302: Masterclass Book Part 2

MOHAMMED IS-HAG 301

1. A 78-year-old woman is admitted after a syncopal episode. She is tachypnoeic with a respiratory rate of 28, hypotensive with BP 90/52 and tachycardiac at a rate of 124/minute. Her 12 lead ECG shows a sinus tachycardia with S1 Q3 T3 pattern and a subsequent spiral CT shows a massive central pulmonary embolism. Which of the following would you treat her with? A : Streptokinase 1,500 000 units over 60 minutes B : Streptokinase 100, 000 units over 30 minutes C : Streptokinase 250,000 units over 30 minutes then 100,000 units every hour for 12-72hours to D : Stretokinase 250,000 units over 60 minutes E : Streptokinase 100, 000 units over 30 minutes with 250,000 units every hour for 12-72 hours. Comment : Give 250,000 units over 30 minutes then 100,000 units every hour for 12 to 72 hours. C : 2. A 50-year-old woman presents to the emergency department with short history of severe occipital headache with vomiting and impaired balance. Her past medical history includes hypertension. On examination she has nystagmus to the right, ataxia of her right limbs and gait ataxia. What is the most likely diagnosis? A : Basal ganglia haemorrhage B : Subdural haemorrhage C : Left temporal lobe haemorrhage D : Pontine haemorrhage E : Cerebellar haemorrhage. Comment : She has posterior circulation signs, in particular right cerebellar signs, and with the fact she has hypertension we need to exclude an acute cerebellar haemorrhage, which may need surgical intervention E : 3. A 55-year-old man with a history of severe asthma and ischaemic heart disease is brought to A&E complaining of palpitations and syncope. On examination he has a weak, regular pulse with a rate of 180 / minute with a blood pressure of 110/70. 12 lead ECG reveals a broad complex tachycardia. Which of the following statements is correct? A : Absence of capture or fusion beats on a long rhythm strip is strongly suggestive of

supraventricular tachycardia (SVT) with aberrant conduction. B : SVT is more likely than ventricular tachycardia (VT) if the patient gives a history of recent

myocardial infarction. C : Intravenous adenosine should be used to distinguish SVT from VT. D : QRS calibre greater than 0.2 is usually indicative of VT. E : SVT with aberrant conduction is unlikely if a recent ECG showed no evidence of bundle branch

block. Comment : Although the presence of capture and fusion beats is pathognomonic of VT, they are rarely seen, and their absence cannot therefore be relied upon to rule out the diagnosis. Broad complex tachycardia in the first week after a myocardial infarction is proven to be ventricular in origin in over 90% of cases. Adenosine may induce bronchospasm, and therefore should be avoided in patients with prior history of asthma. The absence of bundle branch block on a recent ECG is an unreliable indicator of the source of the tachycardia, since bundle branch block is frequently rate-related, and often resolves on termination of the dysrhythmia. D : 4. A 30-year-old female, with a history of previous suicide attempts, presents following an overdose of paracetamol. She stated that she has taken 8g, three hours earlier. Which of the following is the most appropriate management? A : Take immediate plasma paracetamol levels. B : Take no immediate action. C : Transfer to psychiatric care. D : Start intravenous N-acetylcysteine. E : Administer oral-activated charcoal 50g. Comment : Paracetamol levels should be taken at least 4 hours after ingestion (serial levels can be taken if timing of ingestion is in doubt). Earlier levels can be misleading. Psychiatric input may be required once the patient is medically stable. It is normal to start N-acetylcysteine immediately for patients who present late (8-24 hours), or if a significant amount has been taken (>150mg/kg). A single dose of activated charcoal (50g for adults) can be given up to 1 hour after ingestion of toxins. Note that certain drugs are not readily adsorbed by activated charcoal. B :

Page 303: Masterclass Book Part 2

MOHAMMED IS-HAG 302

5. A 70-year-old man presented with severe anterior chest pain which radiated to his back. Chest x-ray revealed a widened mediastinum and an acute type B thoracic aortic dissection was confirmed on angiography. Which of the following is the most appropriate treatment? A : Oral Nifedipine B : Intravenous Labetalol C : Intravenous Sodium nitroprusside D : Oral Bendroflumethiazide E : Oral Lisinopril Comment : Type A aortic dissections require surgical treatment. Type B dissections are usually managed medically with careful blood pressure control. In the acute phase, control of blood pressure with an intravenous infusion of Labetalol would be the ideal option. Labetalol has combined alpha and beta-adrenoceptor blocking activity and causes peripheral arteriolar vasodilatation without reflex tachycardia. Intravenous Nitroprusside causes tachycardia and would normally be combined with a beta-blocking agent. The other agents should not be used in the acute setting when very careful titration of blood pressure is needed. They could be used later for more chronic control of blood pressure after the emergency has been managed. B : 6. A frail 73-year-old woman is admitted via A&E with agitation, restlessness and confusion. On examination she is apyrexial and has no chest signs. Abdominal examination is normal and rectal examination reveals a small amount of normal stool. Her son reports that she had been staying with him for a long weekend just to give her a break, but unfortunately she had forgotten to bring her medications with her. He says that she has a long history of agitation and anxiety. Her blood tests are all normal. Urine dipsticks is negative for nitrites and leucocytes. The most likely cause for her confused state is: A : urinary tract infection B : chest infection C : alcohol withdrawal D : constipation E : benzodiazepine withdrawal. Comment : She has no signs of infection and is not clinically constipated. There is no history of alcohol abuse and there would often be a biochemical or haematological pointer to high alcohol intake such as abnormal liver function tests or raised MCV. Benzodiazepine withdrawal may present with acute confusion. E : 7. Which of the following will not result in an abnormally raised central venous pressure (CVP)? A : Tension pneumothorax B : Right ventricular infarction C : Cardiac tamponade D : Occlusion of the central line E : Anaphylaxis. Comment : Tension pneumothorax raise intrathoracic pressure resulting in an artificially elevated CVP. Right ventricular infarction will also cause elevation of the CVP - importantly, part of the treatment for RV infarction is to keep the filling pressure very high to ensure adequate cardiac output. In cardiac tamponade, RA and RV pressures are elevated significantly. A blocked catheter results in a high CVP with a damped waveform. Anaphylaxis is associated with cardiovascular collapse. E : 8. A 75-year-old man is admitted to hospital. He is receiving warfarin as prophylaxis for a previous DVT. His international normalised ratio (INR) has been stable 2-2.5 for the past 8 weeks. While in hospital his INR increases to >8. Which of the following drugs prescribed in hospital could cause his increased INR? A : Ciprofloxacin B : Aspirin C : Carbamazepine D : Rifampicin E : Co-Dydramol. Comment : Commonly prescribed antibiotics Clarithromycin, ciprofloxacin, and metronidazole enhance the anticoagulant effect of warfarin whereas rifampicin (a potent enzyme inducing drug) diminishes the anticoagulant effect. Although aspirin may increase the likelihood of bleeding due to its antiplatelet and gastric irritant effects, it should not cause prolongation of the INR; other non-steroidal anti-inflammatory (NSAIDs), especially azapropazone may cause prolongation of the INR. Carbamazepine, primidone and phenobarbitone induce liver enzymes and therefore reduce anticoagulant effect, while valproate probably increases the INR. The effect of phenytoin is unpredictable, as both prolongation and reduction of the INR have been reported. Unlike co-proxamol, which causes prolongation of the INR, codydramol has no significant effect on warfarin, and is therefore a useful analgesic for these patients. A :

Page 304: Masterclass Book Part 2

MOHAMMED IS-HAG 303

9. A 50-year old man presented with lethargy and palpitations. Investigations revealed: Serum sodium 144 mmol/L (137-144) Serum potassium 7.9 mmol/L (3.5-4.9) Serum bicarbonate 9 mmol/L (20-28) Serum urea 40.5 mmol/L (2.5-7.5) Serum creatinine 510 micromol/L (60-110) The best immediate therapy is: A : intravenous calcium gluconate B : intravenous dextrose and insulin C : intravenous sodium bicarbonate D : nebulised salbutamol E : rectal calcium resonium. Comment : The immediate danger is of dangerous hyperkalaemia. The patient is complaining of palpitations and clearly arrhythmias are a possibility. The immediate priority is to protect the heart from the hyperkalaemia and this can be done with intravenous calcium gluconate 10% 10-20mls slow iv. The other options may be valuable later to treat hyperkalaemia but are not the best immediate therapy. A : 10. A 40-year-old women presented four hours after an overdose of amitriptyline and diazepam. On examination her Glasgow Coma Scale score was 10. She had dilated pupils, a blood pressure of 100/70 and a pulse of 140 beats per minute. Investigations revealed: SaO2(ward oximetry) 95% BM 7mmol/l Which is the most appropriate other immediate investigation? A : CT brain scan B : Serum urea and electrolytes C : ECG D : Serum paracetamol level E : Serum salicylate level. Comment : She has taken a combination of benzodiazepine and TCA in overdose. TCA overdose can cause coma, convulsions and arrhythmias in serious cases. The single most important test to guide therapy and prognosis remains the 12 lead ECG. A QRS>160 ms is associated with risk of arrhythmias. Always check for other poisons in cases of polypharmacy overdose. Also check arterial blood gases for signs of hypoventilation and acidosis. C : 11. A 25-year-old man with a history of bipolar affective disorder and opiate abuse is brought into A&E by ambulance. His GCS is 9/15. He smells strongly of alcohol and has pinpoint pupils. Which of the following statements about his management is NOT correct? A : It is essential to check his BM stix and give dextrose if this is low. B : He should be given naloxone 400mcg iv. C : Treatment with 50g of activated charcoal is not indicated. D : Gastric lavage is not indicated. E : There is no benefit in taking blood for paracetamol and salicylate levels. Comment : Basic management should always include maintenance of airway, fluid resuscitation where appropriate and correction of hypoglycaemia. He is known to have a history of opiate abuse and has a reduced GCS and pinpoint pupils so it would be reasonable to give naloxone iv. This is the correct dose and can be repeated as often as necessary. Activated charcoal does not adsorb lithium, opiates or alcohol. Gastric lavage has not been shown to be of benefit, indeed there is some evidence that it increases absorption by forcing the drugs through the pylorus. E is not true. Overdoses are usually of multiple medications and this should be done. E : 12. A 45-year-old patient with a past history of alcohol-related chronic liver disease presents to A&E following a 400ml fresh haematemesis. On examination he is jaundiced with palmar erythema and marked ascites. Pulse is 120 beats per minute and blood pressure 100/70. In addition to fluid resuscitation, which of the following treatments is most likely to be beneficial in his initial management, while awaiting upper GI endoscopy? A : Ranitidine 50mg intravenously B : Omeprazole 40mg intravenous bolus C : Tranexamic acid 1g intravenously D : Terlipressin 2mg intravenous bolus E : Propranolol 40mg orally. Comment : It is likely that this patient is bleeding from oesophageal varices, resulting from alcohol-induced portal hypertension. Terlipressin reduces the likelihood of continued bleeding by reducing portal pressure, and may be helpful either prior to endoscopy, or as an adjunct to endoscopic therapy. Intravenous omeprazole has been shown to reduce the likelihood of peptic ulcer rebleeding after endoscopic therapy, but there is no evidence to support its use as an empirical therapy prior to endoscopy or in the management of variceal haemorrhage. Tranexamic acid is an antifibrinolytic and

Page 305: Masterclass Book Part 2

MOHAMMED IS-HAG 304

has been shown to provide a slight reduction in mortality following peptic ulcer haemorrhage in one meta-analysis, while intravenous ranitidine has no impact on outcome of upper GI bleeding from any source. Propranolol is helpful in the primary and secondary prevention of variceal haemorrhage but has no role in the acute setting. D : 13. A 26-year-old woman presents with a week's history of progressive numbness and weakness in her lower limbs. Which of the following suggests a diagnosis of Guillain-Barré syndrome (GBS)? A : Optic atrophy on fundoscopy B : A sensory level C : Ankle weakness with saddle area sensory loss D : Autonomic dysfunction E : Proximal weakness > distal. Comment : Optic atrophy suggests a diagnosis of demyelination. Clear-cut sensory levels classically occur with cord compression. Ankle weakness with saddle area sensory loss would point towards a cauda equina lesion (always examine the saddle area!). In Guillain Barré syndrome there are often cardiovascular abnormalities reflecting autonomic dysfunction. Indeed the autonomic dysfunction can result in sudden death and can manifest as extreme hyper or hypotension, tachycardia and bradycardia. In GBS the weakness is greater distally. D : 14. An elderly man is brought into A&E by ambulance because he had been found wandering down his street early in the morning. He gives a fluent history of his past life, but is unable to explain what he had been doing. On examination he smells of alcohol. He has nystagmus and bilateral lateral gaze palsies. Which of the following statements is incorrect? A : The lesions are in the mamillary bodies and thalamus. B : His red cell transketolase is low. C : Examination of his pupils is normal. D : All of his deficits will resolve with 3 days of parenteral thiamine. E : A CT scan of his head is likely to be normal. Comment : This man has Wernicke–Korsakoff syndrome. It is caused by thiamine deficiency and is common in alcoholic patients. However, it should be considered in all patients with malnutrition. The classical presentation is of confusion, ataxia, ophthalmoplegia and nystagmus. The neurological signs do tend to improve with 3 days of parenteral thiamine, but there are often residual memory problems. CT of the head is likely to be normal. An MRI may show evidence of neuronal loss and demyelination in the midbrain structures. D : 15. A 47-year-old man has re-bled on the ward following emergency endoscopic treatment of his oesophageal varices. In relation to Sengstaken–Blakemore tube placement, which of the following statements is incorrect? A : Inflation of the oesophageal balloon is likely to be required if bleeding is originating above the cardia. B : Formal airway protection with the insertion of a cuffed endotracheal tube with appropriate

anaesthesia is recommended if the patient has grade II encephalopathy. C : If the oesophageal balloon is inflated, pressure should be released for up to 5 minutes every 30 minutes. D : Aspiration ports should be left to drain freely and aspirated regularly. E : Gastric aspiration port may be used for oral medication. Comment : Since oesophageal varices usually originate in the stomach, tamponade of the varices at the gastrooesophageal junction by inflation of the gastric balloon is usually effective in controlling haemorrhage. Inflation of the oesophageal balloon is rarely required and risks necrosis of the oesophageal wall, necessitating regular reduction of the pressure. Regular aspiration of the appropriate ports prevents build-up of secretions; the gastric aspirate port may be used as a nasogastric tube for delivery of essential oral medication, such as lactulose for the treatment of encephalopathy. Aspiration pneumonia is a major cause of death and significant morbidity in these patients; the risk is particularly high in confused or encephalopathic patients who should be intubated for airway protection prior to insertion of the tube. A : 16. A 35 year-old women presented 2 hours after collapsing at home with severe headache. On examination she was drowsy and had neck stiffness. Her temperature was 37.5. She had a mild right hemiparesis. Which of the following is the most appropriate first diagnostic investigation? A : CT brain scan B : Lumbar puncture and examination of the cerebrospinal fluid C : Four-vessel angiography D : MR angiogram E : MR brain scan. Comment : The history is suggestive of a subarachnoid haemorrhage. Urgent CT brain scan will identify more than 95% of patients with suspected subarachnoid haemorrhage if performed within 1-2 days after headache onset (van Gijn 1982). Lumbar puncture is potentially dangerous and will add no extra information if brain CT shows definite evidence of extravasated blood. If CT is negative and there

Page 306: Masterclass Book Part 2

MOHAMMED IS-HAG 305

are no contraindications, LP should be performed. Four vessel angiography will be needed later once the patient is stabilised to identify the source of the bleeding and surgical or endovascular treatment. In the acute stages of SAH (because of the speed of investigation and availability) CT imaging is superior to MRI. MRI imaging becomes much more useful if presentation is delayed after the first few days (>4) when CT sensitivity for subarachnoid blood rapidly declines.MR angiography is without risks and reasonably sensitive (90%) – useful for screening people at risk of intracranial aneurysms but less suitable for patients with subarachnoid haemorrhage. A : 17. A 70-year-old man presented with increasing shortness of breath. He had suffered a myocardial infarction complicated by arrhythmias two years earlier. Investigations revealed: Ward oximetry: O2 saturation 84% on air; Chest radiograph: Bilateral patchy infiltration of lung fields, cardiothoracic ratio of 20/31 cm. No evidence of pulmonary oedema. Which one of the following drugs he was prescribed might explain these findings? A : Aspirin B : Amiodarone C : Lisinopril D : Furosemide E : Atorvastatin. Comment : Amiodarone, an antiarrhythmic drug, can cause pulmonary toxicity – an acute alveolitis that may or may not be followed by fibrosis. It is estimated that approx 6% of patients taking 400mg or more per day for 2 months or more will develop overt pulmonary toxicity. The mechanism may include both immunologically mediated and direct toxic effects. Radiographic appearances are varied: most frequently there is a diffuse nodular or alveolar pattern sometimes with upper lobe predominance. Drug cessation will normally be required. B : 18. A 38-year-old man presented with a short history of increasing lethargy and 3 days of irregular palpitations. Investigations revealed: serum sodium 142 mmol/L (137-144) serum potassium 7.9 mmol/L (3.5-5.0) serum bicarbonate 10.0 mmol/L (20-28) serum creatinine 550 µmol/L (60-110) What is the best immediate therapy? A : Nebulised salbutamol B : Intravenous calcium gluconate C : Intravenous sodium bicarbonate D : Intravenous insulin and dextrose E : Oral calcium resonium Comment : This man has dangerous hyperkalaemia resulting from acute renal failure. Although there is no mention of ECG changes, the history of palpitations should raise the possibility of arrhythmias. As such, 10mL 10% calcium gluconate i.v. will immediately reduce myocardial excitability. This should prevent any dangerous cardiac arrhythmias. It does not reduce plasma potassium levels. All the other agents could be used to lower potassium whilst arrangements for urgent dialysis are made. B : 19. A 45-year-old man with chronic alcoholic liver disease was admitted earlier in the day following a large haematemesis. He was treated with intravenous terlipressin and urgent endoscopy was arranged after initial fluid resuscitation and correction of a mild coagulopathy with fresh frozen plasma. Endoscopy revealed bleeding oesophageal varices, which were injected with sclerosant with apparently good effect. He has been stable on the ward for the past 6 hours, but you are called to see him in the early hours of the following morning because he has had a further 500ml haematemesis, and his blood pressure has dropped from 130/90 to 90/50. Which of the following measures would be most appropriate in addition to fluid resuscitation? A : Urgent repeat endoscopy and sclerotherapy B : Change terlipressin to octreotide C : Administer 10mg vitamin K intravenously D : Insert sengstaken tube, inflate gastric balloon and apply traction E : Urgent surgical intervention. Comment : Balloon tamponade is the most effective treatment for control of variceal bleeding if endoscopic therapy has failed. Even though the varices were bleeding from within the oesophagus, inflation of the gastric balloon with application of traction usually stops the bleeding by compressing the vessels as they cross the gastrooesophageal junction. Rarely, inflation of the oesophageal balloon may be required. It is unlikely that repeat endoscopy would prevent on-going bleeding if he has already received successful sclerotherapy, unless there was doubt about the source of bleeding previously. Octreotide is less effective than terlipressin in controlling portal pressure, and therefore switching therapy is unlikely to be helpful. Although the effects of the fresh frozen plasma administered earlier will probably have worn off, vitamin K will probably not help as the coagulopathy is probably due to synthetic dysfunction rather than vitamin K deficiency. Surgical transection of the oesophagus is rarely indicated, and has largely been superseded by radiological shunt insertion (e.g. TIPSS). D :

Page 307: Masterclass Book Part 2

MOHAMMED IS-HAG 306

20. You are called to see a 78-year-old man who is deteriorating rapidly following surgery for gallstones. He suffers from hypotension, pyrexia, tachypnea and falling urine output. Which of the following statements about severe sepsis do you agree with? A : Severe sepsis accounts for 10% of all critical care admissions in England, Wales and Northern

Ireland. B : No drugs are recommended by NICE for the treatment of severe sepsis. C : Severe sepsis kills almost half the patients affected. D : Drotrecogin alpha (activated ) is a recombinant IL-6 inhibitor. E : If the patient survives severe sepsis, sequelae are rare. Comment : Severe sepsis accounts for over a quarter of admissions to critical care. Despite advances in critical care, almost half of those patients die and those who survive often suffer permanent organ damage. Drotrecogin alpha (activated) has been recommended by NICE for treatment of adult patients with severe sepsis resulting in the failure of two or more organ systems. It is a recombinant activated protein C, but its main mechanism of action clinically appears to be anti-inflammatory. C : 21. A 25-year-old woman presents to A&E 1 hour after consuming 28 x 500mg paracetamol tablets. Which of the following statements is true? A : If the INR is normal on a sample taken four hours from the time of ingestion, liver damage is

unlikely to occur. B : Alcohol ingestion at the time of consumption of paracetamol is an indication for N-acetyl-

cysteine treatment if paracetamol level at 4 hours exceeds the ‘high-risk’ line. C : Activated charcoal may be beneficial if given immediately. D : Onset of tinnitus may be an early symptom of liver failure. E : Deterioration in conscious level within the first 24 hours usually suggests hepatic

encephalopathy. Comment : Abnormal blood clotting following paracetamol overdose results from loss of production of clotting factors by hepatocytes, and is therefore a good early marker of synthetic liver function. The INR rises first because Factor VII has the shortest half-life and is therefore the first to disappear from the blood. However it is unusual to see any abnormality in blood clotting less than 18 hours from ingestion, so normal INR at 4 hours is unhelpful. Abnormal INR at the time of admission may indicate prior chronic liver disease, warfrin ingestion or suggest that ingestion of the drug occurred earlier than the patient reports. Because acute alcohol intoxication inhibits liver enzyme function, less paracetamol will be metabolised to the toxic metabolite, so that liver damage is, if anything, less likely. Chronic alcohol abuse causes liver enzyme induction and is an indication for treatment with N-acetyl cysteine if paracetamol level exceeds the ‘high risk’ line. Activated charcoal is only likely to be beneficial if given within 1 hour of ingestion of paracetamol.If the patient complains of tinnitus, this suggests concurrent salicylate consumption, which requires specific treatment according to the plasma level. Hepatic encephalopathy rarely occurs less than 48 hours from consumption; reduction in level of consciousness in the first 24-48 hours is usually a result of concurrent consumption of sedative drugs (particularly opiates in combination drugs such as co-dydramol of co-proxamol) or hypoglycaemia. Regular monitoring of blood glucose is recommended for patients with elevated paracetamol level requiring treatment with N-acetyl cysteine, or following any change in conscious level. C : 22. Which of the following is NOT a strong indication for urgent CT scanning prior to lumbar puncture in a patient with meningism? A : Abnormal level of consciousness B : AIDS C : Abrupt onset of headache without fever D : Age <60 E : Arm drift on motor testing. Comment : If the diagnosis of meningitis is suspected in a patient under 60, the risk of ‘coning’ (and the likelihood of an abnormal CT) has been shown to be extremely low in the absence of focal neurology, impaired consciousness, previous CNS disease, immunocompromise, or seizure within 1/52 of presentation. An abrupt onset of headache should raise the question of subarachnoid haemorrhage, in which case CT scanning is usually undertaken prior to lumbar puncture. D : 23. Which one of the following statements regarding troponins is correct? A : Elevated plasma concentrations are specific markers for ischaemic heart disease. B : Elevated plasma concentrations would typically be found two weeks after an acute myocardial infarction. C : The sensitivity of troponins for cardiac muscle damage is similar to that of CK-MB. D : Reduced plasma concentrations are typically found in patients who are in atrial fibrillation. E : Troponins have a key role in the decision regarding thrombolysis in patients presenting with chest pain. Comment : Troponins I and T may remain elevated for up to 3 weeks after MI, and hence not very useful for repeat events in this time, unless a rising plasma concentration can be demonstrated. They are specific markers for cardiac muscle damage, but not specific markers of ischaemia; other conditions, e.g. myocarditis, may also produce an elevated level. A good MI history along with ECG changes is the

Page 308: Masterclass Book Part 2

MOHAMMED IS-HAG 307

only evidence-based criteria on which to base a decision on thrombolysis. Troponins are more sensitive than CK-MB in the detection of minor degrees of cardiac muscle damage, and can be used as prognostic markers in the risk stratification of Acute Coronary Syndromes. B : 24. Administration of sodium bicarbonate in cardiac arrest may result in all of the following except: A : exacerbation of hyperkalaemia B : paradoxical intracellular acidosis C : shift of the oxygen dissociation curve to the left D : increased carbon dioxide production E : late alkalosis. Comment : Sodium bicarbonate administration may exacerbate hypokalaemia. In the USA it is still used as a treatment for hyperkalaemia although its effectiveness is questioned. Paradoxical intracellular acidosis results from increased carbon dioxide production. A : 25. A 45-year-old man with no significant past medical history has been experiencing increasingly frequent episodes of exertional chest pain over the past 4 weeks. He comes to A&E on the recommendation of NHS Direct after experiencing an episode of pain at rest the previous night. On arrival in A&E he is pain free and looks well. 12-lead ECG reveals 1mm planar ST segment depression in leads II, III and AVF. Troponin-I measured 12 hours from the onset of his symptoms is moderately elevated at 0.8mg/l. Which of the following drug combinations is most likely to improve his long-term morbidity and / or mortality? A : Aspirin 300mg B : Aspirin 300mg + Enoxaparin 1mg / kg bd C : Aspirin 300mg + Enoxaparin 1mg / kg bd + Clopidogrel 300mg D : Aspirin 300mg + Enoxaparin 1mg / kg bd + Clopidogrel 300mg + Atenolol 25mg E : Aspirin 300mg + Enoxaparin 1mg / kg bd + Clopidogrel 300mg + Atenolol 25mg + Intravenous

infusion of glyceryl trinitrate (GTN). Comment : Aspirin, Enoxaparin, Clopidogrel and beta-blockers have all been shown to reduce morbidity in patients presenting with acute coronary syndromes. The abnormal ECG and elevated troponin confirm increased risk and hence aggressive medical treatment. Glyceryl trinitrate may offer pain relief for patients with on-going angina but has no effect on long-term mortality. D : 26. A 57-year-old man with a past history of heavy alcohol intake presents to the acute medical take after vomiting approximately 500ml fresh blood. Examination reveals blood pressure 110/60, pulse 90. He is jaundiced and has widespread spider naevi. Abdominal examination reveals tense ascites. Which of the following treatments is most likely to be beneficial in his initial management? A : Propranolol 40mg orally B : Terlipressin 2mg intravenous bolus C : Omeprazole 40mg intravenous bolus D : Tranexamic acid 1g intravenously E : Octreotide 50mg per hour by intravenous infusion. Comment : Given the clinical evidence of chronic liver disease and portal hypertension, this man should be considered to have had variceal bleeding until proven otherwise. Early diagnostic and therapeutic endoscopy should be undertaken as soon as possible following resuscitation. Terlipressin 2mg stat followed by 2mg 6-hourly has been shown to be as effective as balloon tamponade in controlling variceal haemorrhage, although somatostatin has been shown to be effective in establishing haemostasis there are no consistent data to support the use of its synthetic analogue Octreotide. Propranolol is beneficial in secondary prevention, but has no role in the acute setting. Intravenous omeprazole and tranexamic acid may be beneficial in the context of peptic ulcer bleeding, following endoscopic confirmation, but have no place in the management of variceal bleeding or undiagnosed upper GI haemorrhage. B : 27. A previously well 40-year-old woman presented to the emergency department with a 72-hour history of increasing confusion and headaches. She had no recent foreign travel and was not at high risk for HIV. She was febrile at 38.5 C. On examination she was drowsy with mild neck stiffness and her right plantar was extensor in response. Investigations revealed: CT brain scan - low attenuation area in the left temporal region. What is the most likely diagnosis? A : Meningococcal meningitis B : Cerebral infarct C : Herpes simplex encephalitis D : Cerebral toxoplasmosis E : Pyogenic brain abscess. Comment : Herpes encephalitis presents with fever and headache for several days, followed by behavioural changes, seizure (40%), hemiparesis (33%) and a depressed level of consciousness.

Page 309: Masterclass Book Part 2

MOHAMMED IS-HAG 308

Infection occurs from exposure to infected saliva or respiratory secretions with virus ascending along the olfactory nerve into the limbic lobe, or from reactivation of virus from the trigeminal ganglion. 90% of patients show evidence of temporal lobe involvement.Diagnostic test of choice is CSF PCR (ensure that the lumbar puncture is not contra-indicated because of mass effect). Sensitivity is 99%. Treatment: aciclovir 10mg/kg q8h IV for 10-14 days. Having been previously well and not likely to be immunocompromised, other diagnosis can be excluded especially with CT findings. C : 28. A 50-year-old man presents with sudden onset shortness of breath and pleuritic chest pain. He has a CT pulmonary angiogram, which shows a large pulmonary embolus. Which of the following is NOT an indication for thrombolysis in this patient? A : Cardiac arrest B : Falling blood pressure C : D dimer greater than 4000 D : Engorgement of neck veins E : Right ventricular gallop. Comment : Thrombolysis improves the outcome of patients with large pulmonary embolus with signs of right heart failure. Any signs of incipient right heart failure are indications for thrombolysis. D dimer is really more useful as a negative predictor and is not helpful as a measure of the size of an embolus. Contraindications to thrombolysis include recent surgery, trauma and intracranial pathology. Thrombolysis removes the clot obstructing the large pulmonary arteries as well as any clot in the pelvic or deep leg veins. It reduces the release of serotonin and other neurohumoral factors, which would tend to exacerbate pulmonary hypertension. C : 29. A 78-year-old woman is admitted with a massive pulmonary embolism. Her 12 lead ECG shows a sinus tachycardia at 124 beats per minute with an S1 Q3 T3 pattern. Which of the following statements is correct? A : Most patients with a pulmonary embolism have an S1 Q3 T3 pattern on their ECG. B : Most patients with a pulmonary embolism have right bundle branch block. C : Most patients with pulmonary embolism have atrial fibrillation. D : Most patients with pulmonary embolism have atrial fibrillation with right bundle branch block. E : The most frequent ECG abnormality is T wave inversion in the anterior leads. Comment : The most frequent ECG abnormality is T wave inversion in the anterior leads especially V1 to V4. These are probably produced by reciprocal changes due to compression of the right coronary artery by the right ventricle as a result of pressure overload. E : 30. Which of the following statements concerning diabetic ketoacidosis is incorrect? A : It may present as an acute abdomen. B : Serum amylase may be elevated without evidence of pancreatitis. C : The white cell count may be elevated without evidence of infection. D : The anion gap is normal. E : Although serum potassium is often raised, total body potassium is reduced. Comment : DKA is associated with a high anion gap acidosis. D : 31. A 47-year-old patient with a past history of relapsing-remitting multiple sclerosis presents to the acute medical take following a flare-up of his condition. Which of the following statements regarding steroid treatment is untrue? A : Corticosteroid treatment may limit the duration of visual loss due to optic neuritis. B : Avascular necrosis of the femoral head is a recognised complication of corticosteroid treatment. C : Corticosteroid treatment may reduce the frequency of future relapses. D : Oral corticosteroid treatment has no impact on rate of recovery from a relapse. E : 1g methylprednisolone daily for 3 days is an appropriate dose. Corticosteroids reduce the duration of a relapse, including optic neuritis, but have no effect on the disease progression or frequency of relapses. Short courses of oral or intravenous methylprednisolone have been shown to be equally effective. Avascular necrosis is a recognised side effect. C : 32. A man is brought to the accident and emergency department by ambulance. He is unconscious (GCS 5) with pin-point pupils and a slow respiratory rate. Immediate specific treatment should be: A : Naloxone (0.4 mg) intravenously, repeated if no effect B : N-acetyl cysteine (150 mg/kg over 15 min) intravenously, then 50 mg/kg over 4 hours, then 100

mg/kg over 16 hours C : Dextrose (50 ml of 50% solution) intravenously, repeated if no effect D : Naloxone (4 mg) intravenously, repeated if no effect E : Insert stomach tube (after securing airway) and give activated charcoal. Comment : The working diagnosis must be opioid overdose, the treatment for which is intravenous naloxone (0.4 mg), repeated up to a total dose of 2 mg depending on clinical response. The half-life of naloxone is shorter than that of opioids, hence if this man wakes up it can be anticipated that he will 're-narcose'. A naloxone infusion may be necessary. A :

Page 310: Masterclass Book Part 2

MOHAMMED IS-HAG 309

33. A 72-year-old man is admitted to the coronary care unit with an acute myocardial infarction. He suffers a cardiac arrest. Basic life support is being given as you arrive. The ECG monitor reveals ventricular fibrillation (VF). The first defibrillation attempt should be made at: A : 200 J B : 400 J C : 360 J D : 20 J E : 100 J. Comment : The European Resuscitation Council guidelines for adult Advanced Life Support suggest that - after Basic Life Support and a precordial thump (if appropriate) - VF/VT should be treated with up to three defibrillation shocks, the first two at 200 J and the third at 360 J. A : 34. A 42-year-old man presents with a sudden onset of headache followed by collapse. On arrival in A&E he has a heart rate of 76 bpm, a blood pressure of 220/140 and a Glasgow Coma Score of 9 (E2, M5, V2). Which of the following should be done immediately? A : Give nimodipine 60 mg 4-hourly via a nasogastric tube B : Start a labetalol infusion 15-30 mg/hour intravenously C : Give codeine phosphate 60 mg IM D : Give high flow oxygen via a non-rebreather mask E : Give 50 ml 50% dextrose intravenously. Comment : He has likely suffered a subarachnoid haemorrhage or an intracerebral bleed. The priority is to prevent secondary brain injury. His airway is likely to be protected with a GCS of 9 (although he may benefit from a nasal or oral airway, and close consideration paid to his airway if going for a CT scan). He should receive high flow oxygen. His blood pressure should not be treated acutely as it is often appropriate to compensate for a rise in intracranial pressure. Nimodipine should be given if a subarachnoid haemorrhage is proven. Attention should be paid to maintaining a normal blood sugar as hyperglycaemia worsens outcome. D : 35. A 27-year-old woman develops difficulty breathing and her lips and tongue swell about five minutes after starting to eat a curry. She is brought to the accident and emergency department by ambulance. She is cyanosed and wheezing. Aside from high flow oxygen via a reservoir bag, which of the following treatments would be your top priority? A : Hydrocortisone 200 mg intravenously B : Chlorpheniramine 10 mg intravenously C : Epinephrine (adrenaline) - 0.5 ml of 1/1000 solution intravenously D : Epinephrine (adrenaline) - 0.5 ml of 1/1000 solution intramuscularly E : Salbutamol 5 mg nebulized. Comment : The history clearly suggests anaphylaxis and treatment with intramuscular epinephrine (0.5 ml of 1/1000) is required.In extremis, epinephrine can be given intravenously, but at reduced dosage: make a 1/10,000 solution (by diluting 1 ml of 1/1000 to 10 ml with 0.9% saline) and give this at 1 ml/min (0.1 mg/min) until a response has been obtained (or a total of 0.5 mg - 5 ml - has been given). D : 36. A 35-year-woman presents 6 hours post a deliberate overdose of paracetamol. The paracetamol level at 6 hours is above the treatment line. Thirty minutes after starting an infusion of N-acetyl cysteine (NAC) she becomes flushed and hypotensive with a blood pressure of 80/55 mmHg. The infusion is stopped immediately and 500ml IV 0.9% saline administered over 30 minutes. Which of the following is the correct ongoing management? A : IV chlorpheniramine and restart NAC infusion at lowest rate once symptoms resolved B : IV chlorpromazine and restart NAC infusion at lowest rate once symptoms resolved C : IV chlorpheniramine and give 2.5g of oral methionine D : IV chlorpromazine and give 2.5g of oral methionine E : Withhold treatment and recheck paracetamol level at 12 hours Comment : Reactions to NAC are well recognized and are not related to hypersensitivity. NAC can almost always be safely restarted and total dose safely administered after symptomatic treatment. Oral methionine may be an alternative but is definitely second line. IV chlorpromazine is now never given and would make hypotension worse. Withholding treatment and waiting more than 12 hours would expose patient to risk of liver failure. A :

Page 311: Masterclass Book Part 2

MOHAMMED IS-HAG 310

1. A 25-year-old man with atopic eczema has a positive skin-prick test reaction to house dust mites. Which of the following is true? A : House dust mite reactivity is definitely the cause of his eczema. B : He should spray acaricide throughout the house. C : He should remove bedroom carpets. D : He should install a de-humidifier. E : None of the above. Comment : Individuals with atopic dermatitis may show reactivity to many different environmental antigens (e.g. house dust mites, grass pollen, cat and dog dander) on skin prick testing, but it is unclear to what extent this causes disease. House dust mite avoidance for individuals with atopic dermatitis is still controversial, but may benefit some patients. A number of measures do help to reduce house dust mite levels but it is difficult to predict who will benefit. Impermeable mattress, pillow and duvet covers are thought to be relatively effective at reducing house dust mite exposure. E : 2. A 60-year-old lady presents with a 3-year history of progressive hair loss. Examination reveals a diffuse thinning of hair over the vertex, with no scarring of the scalp. The most likely diagnosis is A : Alopecia areata B : Androgenetic alopecia C : Discoid lupus erythematosus D : Lichen planopilaris E : Telogen effluvium. Comment : Androgenetic alopecia is a common cause of alopecia in men and women. It classically causes diffuse hair loss over the vertex with sparing over the occiput, with no scarring of the scalp. Discoid lupus and lichen planus can cause patchy, scarring alopecia. Alopecia areata is non-scarring but normally causes patchy hair loss. Telogen effluvium is a cause of diffuse non-scarring alopecia but normally occurs approximately 4 months after an event such as childbirth or severe illness. B : 3. A 58-year-old man presents with a scaly rash. You consider the diagnosis of psoriasis. Which one of the following statements is true of this condition? A : Psoriasis shows the Koebner phenomenon. B : Nail involvement in psoriasis is rare. C : Guttate psoriasis is the commonest form. D : Sterile pustules are frequently seen within lesions. E : Intense pruritus is a typical symptom. Comment : The Koebner phenomenon is the localization of cutaneous disease to sites of trauma and is shown by several disorders including psoriasis, lichen planus, viral warts and vitiligo.Nail involvement in psoriasis is common and is characterized by the presence of “thimble pitting”, onycholysis (separation of the nail from the nail bed) and subungual hyperkeratosis. Chronic plaque psoriasis localizing to the extensor surfaces is by far the commonest form. Widespread pustules within lesions or on erythematous skin should raise the possibility of generalized pustular psoriasis which is a rare but serious complication with a significant mortality.Although pruritus can be feature of psoriasis, it is rarely intense. A : 4. A 30-year-old female develops multiple hot red tender nodules on both lower legs. Which of the following is least likely in your differential diagnosis? A : Sarcoidosis B : TB C : Lymphoma D : Chronic myeloid leukaemia E : Crohn's disease. Comment : The clinical history is highly suggestive of erythema nodosum (EN) which may be associated with options A-C and E. Myeloid malignancies are not associated with EN, but are associated with other skin conditions such as pyoderma gangrenosum and Sweet's syndrome (hot, red, tender plaques often in a pyrexial patient and usually on the upper body). D : 5. A 30-year-old insulin-dependent diabetic presents with a rash on her shins. Which is the most likely diagnosis? A : Erythema nodosum B : Pretibial myxoedema C : Candidiasis D : Necrobiosis lipoidica E : Granuloma annulare.

Page 312: Masterclass Book Part 2

MOHAMMED IS-HAG 311

Comment : Answers A and B are both rashes with a predeliction for the shins, but they do not occur more commonly in diabetics. Answers C and E do occur more commonly in diabetics but do not classically affect the shins. D : 6. Which of the following is not a well-documented feature of dermatomyositis? A : Association with underlying malignancy B : Raynaud's phenomenon C : Calcinosis D : Dysphagia E : Palpable peripheral nerves. Comment :Internal malignancy is present in up to 40% of males with dermatomyositis over 40 years of age. E : 7. In which of the following is direct immunofluorescence essential to make the diagnosis? A : Erythema nodosum B : Allergic contact dermatitis C : Pemphigoid gestationis D : Erythema multiforme E : Acanthosis nigricans. Pemphigoid gestationis is an autoimmune blistering disease of pregnancy. It is immunologically similar to bullous pemphigoid. Direct immunofluorescence is essential to demonstrate the deposition of complement and IgG along the dermoepidermal junction. Histology alone cannot make the diagnosis.C : 8. A patient is due to have a skin biopsy. Which of the following statements about an incisional skin biopsy are incorrect? A : It is essential in the diagnosis of pemphigus vulgaris. B : It is essential in the management of malignant melanoma. C : Lignocaine with adrenaline can be used to anaesthetise the nose. D : Keloid scar formation may occur. E : Is not helpful in persistent cases of acne. Comment : Scarring will follow any skin biopsy and patients must be warned. Keloid scars are also a risk, particularly in black skin, and the highest risk sites are the upper torso and shoulders. Adrenaline aids haemostasis but should be avoided on the digits where vasoconstriction of the digital arteries could cause ischaemia. A skin biopsy is needed to confidently make the diagnosis of pemphigus vulgaris and some skin should be sent for direct immunofluorescence, the gold standard investigation. Optimal management of a suspected malignant melanoma is an excisional biopsy. Incisional biopsies are only performed under certain circumstances e.g. confirmation of the diagnosis if there is doubt, particularly when an excisional biopsy is not straightforward.As a general rule, acne is a clinical diagnosis and a biopsy is never required, even in persistent cases. A biopsy would only be considered in cases where there is diagnostic doubt. B : 9. A 30-year-old male presents with a 6-month history of a recurring itchy rash. Clinically, you suspect chronic idiopathic urticaria. Which of the following is NOT appropriate management for him? A : Prescribing oral antihistamines B : Advising avoidance of aspirin and NSAIDS C : Reassuring patient that it should be self-limiting D : Referring for patch testing E : Advising keeping cool and avoiding excess alcohol when the rash is present. Comment : Chronic idiopathic urticaria is a very common condition which is self-limiting and is usually controlled by oral antihistamines. It is known to be exacerbated by aspirin, NSAIDS, heat and alcohol. Most cases are idiopathic with no underlying allergic cause. Referral for allergy tests is therefore not appropriate for the majority of cases. Those cases of urticaria which are allergic in aetiology are thought to be type 1 hypersensitivity reactions, but patch testing identifies type 4 delayed hypersensitivity reactions (response D is therefore wrong for two reasons). D : 10. A 50-year-old man presents with hair loss. Examination reveals patches of scarring alopecia with surrounding inflammation. Which is the most likely diagnosis? A : Lichen planopilaris B : Androgenetic alopecia C : Alopecia areata D : Traction alopecia E : Trichotillomania. Comment : The two most common causes of scarring alopecia are lichenplanopilaris (lichen planus affecting the scalp hair follicles) and discoid lupus erythematosus. Both conditions cause inflammation, and can be difficult to differentiate clinically. B and C are both non-scarring causes of alopecia. Traction alopecia can cause scarring but is not usually associated with inflammation. Alopecia occurs at sites of traction and is usually caused by hair styling practices, such as braiding. A :

Page 313: Masterclass Book Part 2

MOHAMMED IS-HAG 312

11. A 30-year-old lady presents with a 5-year history of flushing of her facial skin and a spotty rash. Examination reveals a pustular rash on the cheeks with no comedones. What is the most likely diagnosis? A : Rosacea B : Acne vulgaris C : Carcinoid syndrome D : Systemic lupus erythematosus (SLE) E : Allergic contact dermatitis. Comment : Rosacea often presents with easy facial flushing before the onset of a pustular rash. Rosacea is often mistaken for acne, but comedones are the clinical hallmark of acne and are absent in rosacea.Carcinoid syndrome often presents with flushing but is not associated with a pustular rash. SLE can cause erythema of the cheeks but pustules are not seen. Allergic contact dermatitis would most commonly present with a scaly, eczematous rash. A : 12. Which of the following is not true for urticaria? A : It can be associated with angioedema B : It can be caused by a non-immunological reaction to aspirin C : It can be caused by a type I allergic reaction to foods D : Lesions typically fade to leave small scars E : Lesions are usually itchy and last less than 24 hours. Comment : The characteristic pruritic weals are due to vasodilatation and leakage of fluid into the tissues. The lesions typically last less than 24 hours and fade without scarring.Urticaria can be caused by an immediate type I reaction to food but frequently no precipitating cause can be identified. Codeine and morphine are able to directly induce mast cell release of histamine. Aspirin can also cause mast cell release of histamine possibly via diversion of the arachidonic acid pathway to produce more lipoxygenase pathway products.Angioedema is a variant of urticaria in which the subcutaneous tissues rather than the dermis are predominantly involved. D : 13. The commonest form of drug eruption is: A : erythema B : purpura C : fixed drug eruptions D : eczema E : exfoliative dermatitis. Comment : Erythema with or without urticaria, pruritus or erythema-multiforme-like lesions is the commonest form of drug eruption.Exfoliative dermatitis carries a significant mortality when extensive.A: 14. Which is not true of vitiligo? A : It is histologically characterized by loss of epidermal melanocytes B : A positive family history is rare C : Vitiligo shows the Koebner phenomenon D : Commonly involved sites include hands, wrists, knees, neck E : It is associated with autoimmune disease. Comment : The prevalence of vitiligo is 0.5-1% of populations worldwide. The commonest generalized form is characterized by the development of sharply defined white patches distributed symmetrically. Typical sites include backs of hands, wrists, knees, neck and around body orifices. Hair may de-pigment. The course is unpredictable with lesions remaining static or expanding or showing signs of follicular re-pigmentation. 30% have a positive family history and there are strong associations with autoimmune diseases, particularly thyroid disease and pernicious anaemia. The Koebner phenomenon is the localization of cutaneous disease to sites of trauma and is shown by several disorders including psoriasis, lichen planus, viral warts and vitiligo. B : 15. A 16-year-old male is given amoxycillin for a sore throat. Five days later, he develops a rash consisting of erythematous papules and plaques. Individual lesions come and go over several hours, leaving no trace. What is the most likely diagnosis? A : Epstein Barr virus (EBV) infection B : Drug hypersensitivity syndrome C : Scarlet fever D : Cutaneous vasculitis E : Urticaria. Comment : Lesions which come and resolve without trace, typically within 24 hours, are characteristic of urticaria. The other diagnoses would all fit the clinical scenario but individual lesions would take several days minimum to resolve. Amoxycillin in the context of EBV infection can give a florid maculo-papular eruption. Penicillin antibiotics may trigger a variety of drug eruptions including a cutaneous vasculitis and the drug hypersensitivity syndrome. The latter term should be reserved for a multi-system drug reaction consisting of a widespread eruption which may progress to erythroderma, often with facial oedema, plus peripheral lymphadenopathy, peripheral blood eosinophilia, atypical lymphocytosis and visceral involvement which may include the liver and lungs.Scarlet fever is produced by a streptococcal toxin. E :

Page 314: Masterclass Book Part 2

MOHAMMED IS-HAG 313

16. A 30-year-old woman presents with three patches of hair loss. Clinically, the scalp is normal within the patches and you suspect a diagnosis of alopecia areata (AA). Which of the following is a sign that her prognosis for complete hair regrowth / lack of progression may not be good. A : Hair loss at the occiput (ophiasis) B : Hair loss at the anterior scalp margin C : Normal nails D : Lack of 'exclamation mark' hairs at edge of patches E : Presence of seborrheic dermatitis. Comment : Hair loss in an 'ophiasis' pattern is associated with a poor prognosis. Pitted nails are sometimes associated with AA, and maybe a marker of severity. ‘Exclamation mark’ hairs are short stubby hairs found at the edge of all 'active' patches of hair loss in AA. Seborrheic dermatitis has no association with AA. A : 17. When a 21-year-old female returns from vacation abroad, she notices an eruption composed of hypopigmented macules over her upper central back and chest. What is the most likely diagnosis? A : Post-inflammatory hyperpigmentation B : Vitiligo C : Pityriasis versicolor D : Leprosy E : Hypopituitarism. Comment : All of the above can cause hypopigmentation, and options A-D produce localized hypopigmentation. However, the distribution is very characteristic of pityriasis versicolor, an infection with the yeast malassezia furfur. Warm, moist conditions favour its growth. Furthermore, infected skin does not pigment normally so tanning of the normal, surrounding skin makes the infection more obvious. For these two reasons, patients may not be aware of the problem until they take a vacation. Another clue to the diagnosis is scaling of the skin which may not be obvious until the skin is gently abraded. C : 18. Which of the following statements about atopic dermatitis is true? A : Less than 30% of childhood atopic dermatitis clears in adolescence. B : Risk factors for recurrent disease in adulthood include family history of atopic dermatitis. C : Atopic dermatitis is only rarely associated with asthma and hayfever. D : Urticaria is a characteristic clinical finding. E : Flexural involvement is rare in adult disease. Comment : Up to 60% of childhood atopic dermatitis is believed to clear during adolescence, but risk factors for later recurrence in adulthood are: · positive family history · concomitant hayfever and/or asthma · severe disease in childhood · early onset childhood disease. Although urticaria is well described in individuals with atopic dermatitis it is not a hallmark feature. Flexural involvement is the commonest distribution in adulthood, but other patterns are well documented. The UK Working Party's Diagnostic Criteria for atopic dermatitis are: · prescence of an itchy skin condition within the last 12 months, plus · at least 3 of the following: o history of flexural involvement o visible flexural involvement o onset below 2 years of age o personal history of asthma or hayfever o history of dry skin. B : 19. Which one of these features is typical of dermatomyositis? A : Heliotrope rash around external auditory meatus B : Gottron's papules over knuckles of fingers C : Distal muscle weakness D : Rare association with malignancy in the elderly E : Sclerotic digital skin. Comment : The lilac heliotrope discolouration around the eyes can be associated with a malar erythema and oedema. Gottron's papules are slightly atropihic lilac-coloured papules over the knuckles of the hands and can be associated with linear streaks of erythema over the extensor tendons. Most patients have clinical muscle involvement with a predominantly proximal pattern. Internal malignancy occurs in up to 66% of males over 40 years. B :

Page 315: Masterclass Book Part 2

MOHAMMED IS-HAG 314

20.

PLATE 55 A 79-year-old woman complains of a rash (see image). Which of the following statements is NOT correct? A : The patient complains of itch. B : A diagnosis of pemphigus vulgaris is likely. C : A drug may be responsible. D : A biopsy for direct immunofluorescence (DIF) should be taken from normal skin. E : There is likely to be IgG and C3 deposition along the dermoepidermal junction. Comment : This is a bullous disorder, and as the blister is intact, pemphigoid is much more likely than pemphigus. Drugs can cause bullous eruptions. DIF is the investigation of choice, and the findings in pemphigoid are as described in E, but unaffected skin should be sampled as false negatives can be obtained if blistered skin is sampled. B : 21. A 30-year-old male complains of itchy buttocks. There is a papular rash on examination. On further questioning, he complains of diarrhoea and weight loss. What is the most likely diagnosis? A : Folliculitis B : Dermatitis herpetiformis C : Intestinal threadworms D : Cutaneous larva migrans E : Urticaria. Comment : Dermatitis herpetiformis has a predilection for the extensor surfaces including the elbows, knees and buttocks. It is characteristically very itchy. It is associated with gluten-sensitive enteropathy, which may explain this man’s diarrhoea. Urticaria is itchy and is occasionally associated with intestinal parasites, which would link with diarrhoea. However, it does not have a predilection for the buttocks. Intestinal threadworms are associated with peri-anal itching due to migration of worms onto the peri-anal skin. Cutaneous larva migrans is a characteristic itchy eruption of creeping tortuous tracks produced by hookworms as they migrate through the skin. Hookworms are acquired through direct contact with faeces-contaminated sand or soil. The feet, hands and buttocks are commonly affected. Folliculitis is common on the buttocks and may be itchy. The papules would be centred on hair follicles. It does not tie in with the history of diarrhoea. B :

Page 316: Masterclass Book Part 2

MOHAMMED IS-HAG 315

22. A 70-year-old lady is an inpatient with a diagnosis of bullous pemphigoid. She has many new, intact blisters over her trunk and limbs. She is started on 40mg of prednisolone as treatment. Which of the following additional management suggestions are not recommended or unnecessary? A : Regular dipstick urine for glucose B : Regular blood pressure C : Oral bisphosphonate D : Leave blisters intact to prevent infection E : Regular count of new blisters. Comment : Any patient started on oral steroids, and particularly the elderly, should be monitored for steroid side effects. This includes hypertension, diabetes and gastric erosions. An oral bisphosphonate will protect against osteoporosis. A blister chart is useful for inpatients to monitor disease progression and control. If no new blisters are occurring then steroid doses can be reduced. Current practice is to burst tense blisters with a sterile needle to release fluid. The roof of the blister is left in situ over the eroded surface. No increase in infection rate is seen. It is more comfortable for the patient and healing is felt to be enhanced. D : 23. A 17-year-old girl has developed an eczematous patch just below her umbilicus, thought to be due to allergy to a stud fastener in her jeans. Which one of the following statements is true regarding allergic contact dermatitis to nickel? A : It is diagnosed by prick testing. B : It is a type 3 allergic reaction. C : It affects males and females equally. D : It occurs more commonly in atopic patients. E : It can be caused by wearing gold jewellery. Comment : Allergic contact dermatitis is a type 4 (delayed) hypersensitivity reaction and is diagnosed by patch testing. Nickel is one of the commonest causes, with an increased frequency in women, probably due to increased exposure to nickel-containing jewellery, particularly through earrings for pierced ears. Men with nickel allergy are more likely to have an occupational cause. Atopic patients are thought to be less prone to developing allergic contact dermatitis (other than to medicaments). Low carat gold often contains significant amounts of nickel. This can be detected by the nickel spot test: dimethylglyoxime rubbed against a metal will turn pink in the presence of nickel. E : 24. Which of the following would be expected to produce alopecia (hair loss) if there was scalp involvement? A : Psoriasis B : Seborrhoeic dermatitis C : Bullous pemphigoid D : Discoid lupus erythematosus E : Rosacea. Comment : Discoid lupus erythematosus produces hair loss which may be permanent because of scarring of the hair follicles (a scarring alopecia). Psoriasis and seborrhoeic dermatitis commonly involve the scalp but do not produce hair loss.Rosacea affects the face.Bullous pemphigoid does not typically affect the scalp but would not be expected to produce hair loss. D : 25. A 50-year-old male is referred with periorbital oedema and a rash on the back of his hands. He also has a cough and is having difficulty stacking the high shelves at work in a supermarket. What is the most likely diagnosis? A : Angioedema B : Allergic contact dermatitis C : Dermatomyositis D : Hypothyroidism E : Polyarteritis nodosa Comment : Dermatomyositis (DM) clasically presents with a periorbital heliotrope rash, but at times the purple discoloration is not seen and the diagnosis of DM is one to consider in a patient with periorbital oedema. The finding of erythematous papules over the knuckes (Gottren's papules), linear erythema along the dorsal surface of the fingers, and ragged, erythematous nailfolds indicates DM. There is often a detectable proximal myopathy and patients will struggle with activities that involve lifting their arms above the shoulders, as in this case. There is an association with malignancy, particularly lung and upper aerodigective tract. The patient's cough suggests he may have an underlying lung cancer. Hypothyroidism can cause periorbital oedema and a proximal myopathy but doesn't explain the rash. PAN often presents with middle aged men and could explain his cough but causes myalgia and a vasculitic rash. C :

Page 317: Masterclass Book Part 2

MOHAMMED IS-HAG 316

26. A 30-year-old man presents with a 3-month history of a sore mouth and a 2-week history of ulcers on the upper chest and back. What is the most likely diagnosis? A : Behçet’s disease B : Crohn's disease C : Pemphigus vulgaris D : Pyoderma gangrenosum E : Bullous pemphigoid. Comment : Oral ulceration can be a feature of both Behçet’s and Crohn’s disease. Both have cutaneous manifestations but ulceration over the upper torso would not be typical. Both are associated with pyoderma gangrenosum, which does present with cutaneous ulceration but the upper torso would be an unusual site. The legs are most common. Pyoderma gangrenosum very rarely produces oral ulceration. The clinical history is very typical of pemphigus vulgaris. Oral ulceration is almost universal and typically precedes cutaneous involvement by weeks to months. The upper torso is a site of predilection. Patients can present at any age but most commonly in the 3rd-6th decades. Bullous pemphigoid is most often seen in the elderly and oral ulceration is seen in a minority of cases. C : 27. A 60-year-old female with a recent onset of gout presents with a 5-day history of a widespread erythematous skin eruption and mouth ulcers. She has lesions on the palms and some areas of skin are beginning to blister and ulcerate. What is the most likely diagnosis? A : Bullous pemphigoid B : Fixed drug eruption C : Stevens-Johnson syndrome D : Pemphigus vulgaris E : Erythema multiforme. Comment : Allopurinol, prescribed for gout, is a well-recognised trigger of Stevens-Johnson syndrome (SJS). The palmar lesions, mouth ulcers and short history suggest this diagnosis. Erythema multiforme is also a possible diagnosis but the eruption tends to be less widespread than SJS, confined mainly to the distal limbs, with less tendency blister. An autoimmune blistering disease, such as bullous pemphigoid or pemphigoid vulgaris, would tend to have a longer history before presenting in a widespread form and palmar lesions are not typical of either. A fixed drug eruption usually presents with a small number of erythematous plaques, which always recur at the same site upon re-exposure to the drug. C : 28. A 25-year-old female presents with a 3-week history of bald patches in the scalp. On examination, there are several round patches (2-3cm), of completely hairless skin. The skin looks normal. What is the most likely diagnosis? A : Alopecia totalis B : Tinea capitis C : Discoid lupus erythematosus D : Lichen planus E : Alopecia areata. Comment : All of the above can cause loss of scalp hair. Discoid lupus erythematosus and lichen planus are inflammatory skin diseases that can cause permanent hair loss due to scarring of the hair follicles (scarring alopecia). The scalp would look red and scaly in these two conditions. Tinea capitis can produce patchy hair loss but the scalp would not look normal; there would be scaling and erythema. Examination of plucked hairs or skin scrapings would confirm the diagnosis.Alopecia areata and alopecia totalis are believed to be organ-specific autoimmune diseases targeting hair follicles. Alopecia areata characteristically presents with round patches of complete hair loss and the scalp looks normal. There is often a short history. Alopecia totalis is a more extensive form of alopecia areata in which the entire scalp hair is lost. E : 29. A 25-year-old female presents with hypopigmentation on the knees and elbows. What is the most likely diagnosis? A : Leprosy B : Vitiligo C : Post-inflammatory hypopigmentation D : Pityriasis versicolor E : Halo naevi. Comment : All of the above are hypopigmented. However, the symmetrical distribution on pressure points is very typical of vitiligo. B : 30. An 80-year-old female is referred with widespread, refractory eczema of 3 months’ duration. There is no past medical history other than a hospital admission for a fractured neck of femur six months ago. What is the most likely diagnosis? A : Bullous pemphigoid B : Atopic eczema C : Scabies D : Varicose eczema E : Xerosis.

Page 318: Masterclass Book Part 2

MOHAMMED IS-HAG 317

Comment : This is a commonly recounted history in the elderly with scabies. The hospital admission is an important clue because institutions are a common source of infestation, particularly if intensive nursing was involved. Symptoms do not occur for several weeks after the initial infestation because the symptoms and signs are due to a hypersensitivity reaction. This can build up to produce a widespread eczema. It is important to examine the hands and feet where the typical burrows of scabies may be seen most easily. Bullous pemphigoid can be mistaken for eczema in its early stages, prior to the development of blisters, and is most commonly seen in the elderly. Varicose eczema is also common in the elderly. It is usually sited on the lower legs but can become widespread. Atopic eczema is also possible in this case but less likely given the recent onset with no history of atopy. Xerosis is dry skin, is common in the elderly and is a cause of itching. C : 31. A 30-year-old woman with arthritis presents with a painful, rapidly enlarging ulcer on the lower leg. What is the most likely diagnosis? A : A venous leg ulcer B : Pyoderma gangrenosum C : Necrobiosis lipoidica D : Cutaneous vasculitis E : Squamous cell carcinoma. Comment : All of the above can cause leg ulceration. A squamous cell carcinoma would be very unusual in a 30-year-old, and they tend to be slow growing and are often painless. Venous leg ulceration would also be unusual in a young person.Necrobiosis lipoidica has a predilection for the shins but ulceration is not very common. It is often associated with diabetes. The clinical history including the site would fit the diagnosis of pyoderma gangrenosum, and the history of arthritis in a young patient may indicate rheumatoid arthritis, tying the case together.Vasculitis may also occur in rheumatoid arthritis and necrotic areas may ulcerate,but one would expect to see areas of purpura in addition to the leg ulcer. B: 32. A 30-year-old man presents with intermittent episodes of lip swelling, at random times. There is no associated wheeze, swallowing problems or abdominal pain. He is well other than a history of migraine. What is the most likely diagnosis? A : Hereditary angioedema B : Food allergy C : Chronic idiopathic angioedema D : Angiotensin-converting enzyme (ACE) inhibitor-induced angioedema E : Hypothyroidism. Comment : Chronic idiopathic angioedema is a common condition. It is often exacerbated by analgesics such as aspirin, codeine and non-steroidal anti-inflammatory drugs (NSAIDs), which act as general mast cell destabilisers. Hereditary angioedema is less common and caused by C1 esterase inhibitor deficiency. Attacks are often associated with abdominal pain (due to bowel oedema). Type 1 food allergy can present with lip swelling but the timing is not random, occurring within 30 minutes of contact with the food. ACE-inhibitors can trigger angioedema. Hypothyroidism is associated with angioedema, however our patient was well. C : 33. A 60-year-old cleaner presents with a rash on both hands. An irritant hand dermatitis is suspected. Which one of the following statements is true regarding this condition? A : It classically causes a finger tip dermatitis. B : It is diagnosed by patch testing. C : It is more common in atopic patients. D : It can be differentiated from allergic contact dermatitis histologically. E : It should not be treated with topical steroids. Comment : Irritant hand dermatitis classically causes dermatitis in the finger webs, beneath rings and over the dorsum of the hands. It can be caused acutely by contact with stong alkalis or acids but is usually a chronic problem caused by repeated exposure to detergents and wet work. Atopic patients are more prone to develop irritant dermatitis. Patch tests demonstrates type IV allergic reactions and are used to diagnose an allergic contact dermatitis. Patch tests are negative in an irritant dermatitis. Histology is similar in allergic and irritant dermatitis. Therefore, the diagnosis of an irritant contact dermatitis is based on the the history, clinical signs and negative patch tests. The treatment of irritant and allergic dermatitis is similar and involves avoidance of irritants/allergens, use of soap substitutes, regular emollients and topical steroids. C :

Page 319: Masterclass Book Part 2

MOHAMMED IS-HAG 318

34.

PLATE 56 A 62-year-old man presented with a scaly rash on his face and hands (see image). What is the likely diagnosis? A : Photosensitive eczema B : Psoriasis C : Contact dermatitis D : Systemic lupus erythematosus E : Seborrhoeic dermatitis. Comment : The V-shaped cut-off at the neck indicates that the rash is photo-induced. Look also for sparing behind the ears, under the chin and nose, and in the upper eyelid fold. Seborrhoeic dermatitis typically affects the eyebrows and nasolabial folds. Systemic lupus erythematosus can cause a photosensitive rash, but is not a likely diagnosis in a middle-aged man. A : 35. A mother and her three children, one a 6-month-old baby, present to their GP with an itchy rash that has been present for two weeks. The father has no rash and no symptoms of itch. Clinically scabies infestation is suspected. Which of the following advice is not appropriate? A : The whole family should be treated with a topical scabecide. B : Application of the scabecide from the neck down is only needed for adults. C : No scabecide is needed for father as he is unaffected. D : Scabecide should be applied head to toe for the baby. E : Wash the bedclothes and linen. Comment : Scabies infestation is extremely common. All people living in a house with an affected person should be treated with a topical scabecide. General advice is to apply treatment topically from neck down in adults and children, but from head to toe in babies where the scalp can be affected. The rash and itch associated with scabies is usually delayed from time of infestation, as symptoms occur due to an allergic type response to the mite eggs or faeces. The father must therefore be treated even if asymptomatic at present. Mites do not usually survive in clothing and bedding, therefore boil washes are not needed. However, normal washing of linen is usually recommended. C :

Page 320: Masterclass Book Part 2

MOHAMMED IS-HAG 319

36. A 24-year-old keen hiker presents to his GP with a rash on his leg and 'flu-like symptoms. He reports that the rash started as a small raised red area 2 days ago, which has now spread. On examination he is apyrexial and has mild lymphadenopathy in his right groin. On examination of his right lower leg, he has a ring of erythema surrounding a central area of normal skin. What is the most likely diagnosis? A : Weil's disease B : Lyme disease C : Coeliac disease D : Graves' disease E : Erythema nodosum. Comment : The rash is erythema chronicum migrans, which is a feature of early Lyme disease (Stage 1) in 70% of cases. Regional lymphadenopathy may also be a feature. The diagnosis is confirmed by serology. The treatment for early disease is 2-3 weeks of doxycycline or amoxicillin. B : 37. A 60-year-old cleaner presents with a rash on both hands. An irritant hand dermatitis is suspected. Which one of the following statements is true regarding this condition? A : It classically causes a finger tip dermatitis. B : It is diagnosed by patch testing. C : It is more common in atopic patients. D : It can be differentiated from allergic contact dermatitis histologically. E : It should not be treated with topical steroids. Comment : Irritant hand dermatitis classically causes dermatitis in the finger webs, beneath rings and over the dorsum of the hands. It can be caused acutely by contact with stong alkalis or acids but is usually a chronic problem caused by repeated exposure to detergents and wet work. Atopic patients are more prone to develop irritant dermatitis. Patch tests demonstrates type IV allergic reactions and are used to diagnose an allergic contact dermatitis. Patch tests are negative in an irritant dermatitis. Histology is similar in allergic and irritant dermatitis. Therefore, the diagnosis of an irritant contact dermatitis is based on the the history, clinical signs and negative patch tests. The treatment of irritant and allergic dermatitis is similar and involves avoidance of irritants/allergens, use of soap substitutes, regular emollients and topical steroids. C : 38. A 50-year-old male presented with thickening, discolouration and onycholysis of several toenails. There was erythema and scaling in the toewebs. What is the most likely cause of his nail dystrophy? A : Lichen planus B : Psoriasis C : Twenty nail dystrophy D : Onychomycosis E : Alopecia areata Comment : All of the above can cause nail dystrophy. Onycholysis describes distal separation of the nail from the nail bed and is most commonly seen in psoriasis and onychomycosis. It is not a feature of the other diseases above. Thickening and discolouration are also signs seen in both onychomycosis and psoriasis. The presence of erythema and scaling between the toes could be psoriasis but is more likely to be tinea pedis (athlete's foot) and the organisms responsible may infect the nails to cause onychomycosis. D : 39. Which of the following is NOT true of xeroderma pigmentosum: A : Typically autosomal recessive B : Typically due to a defect in DNA repair C : The skin is usually abnormal at birth D : Eyes are commonly affected E : Many patients die before the age of 20 years. Comment : Eye problems include photophobia, conjunctivitis and ectropion. Many tumours develop on light-exposed sites and include basal cell carcinomas, squamous cell carcinomas, kerato-acanthomas and melanomas. C : 40. Allergic drug reactions include: A : mouth ulcers and methotexate B : striae and steroids C : argyria and silver-based preparations D : hair loss and oral contraceptive E : erythema multiforme and ampicillin. Comment : All other answers A-D are known associations or side effects of the drug but not a specific allergy. E :

Page 321: Masterclass Book Part 2

MOHAMMED IS-HAG 320

41. A 60-year-lady with rheumatoid arthritis presents with a rapidly enlarging, painful, sloughy leg ulcer on the anterior shin. Doppler examination is normal. There has been no improvement with dressings and bandages. What would be the most appropriate next management step? A : Referral to surgeons for debridement B : Referral to surgeons for grafting C : Maggot therapy D : Compression E : Immunosupression. Comment : Rheumatoid arthritis is a risk factor for pyoderma gangrenosum (PG). The anterior shin is an unusual site for venous ulceration. Dopplers are normal, making arterial disease unlikely. PG ulcers are often painful and do not respond to conventional treatment. PG is associated with pathergy, such that trauma to the ulcer via debridement, or removal of skin at a distant site for grafting, would be contraindicated. PG is diagnosed by exclusion of other causes of ulceration and by its improvement with immunosuppression. E : 42. A 30-year-old man with a history of atopic eczema presents a painful widespread vesicular rash (which he has had for two days) and constitutional upset. What is the most likely diagnosis? A : Eczema herpeticum B : Severe exacerbation of atopic eczema C : Bacterial infection D : Drug reaction E : Molluscum contagiosum. Comment : Eczema herpeticum can develop in the absence of a history of herpes simplex reactivation or contact. The other diagnoses are all seen in individuals with atopic eczema, but the hallmark features of pain and vesicals are highly suggestive. A : 43.

PLATE 57 A 23-year-old woman presents with a rash on her eyelids (see image). She has no other problems with her skin. The likely diagnosis is: A : atopic eczema B : contact dermatitis C : psoriasis D : systemic lupus erythematosus E : sarcoidosis. Comment : Her erythematous, scaly, lichenified eyelids were due to contact dermatitis caused by allergy to the eye make-up she had been using.Atopic eczema could produce a rash on the eyelids like this, but not in isolation. Look in particular for a history of atopy and involvement of the limb flexures when considering this diagnosis.Systemic lupus erythematosus characteristically causes a photosensitive rash, which would spare the upper eyelid and not affect it. B :

Page 322: Masterclass Book Part 2

MOHAMMED IS-HAG 321

44. A 25-year-old woman presents with scaly red plaques on the face, scalp and upper torso. She reports that it is worse in the summer. Which of the following is the best option? A : Nail pitting may be detected. B : A trial of topical corticosteroids is indicated. C : Direct immunofluorescence would be unhelpful. D : Patch tests are indicated. E : A skin biopsy is necessary. The history suggests discoid lupus erythematosus (DLE). Individual lesions of psoriasis and DLE may look similar but the distribution in this case is typical of DLE, not psoriasis. Therefore nail pitting, a feature of psoriasis, would be unlikely. DLE is photoaggravated and therefore worse in the summer, whereas psoriasis usually improves with sun exposure. In cases of suspected DLE, the diagnosis should be confirmed with a skin biopsy prior to commencing treatment. If lesional skin is also sent for direct immunofluorescence, granular deposits of immunoglobulins and complement will be seen along the basement membrane zone (a lupus band) in the majority of cases. Patch tests are unlikely to help in this case. E : 45. A very anxious 28-year-old woman with atopic dermatitis comes to see you in outpatients. She is concerned about the chances of her children inheriting the condition, and of whether 'lifestyle' issues have any impact on the condition. Which one of the following statements about atopic dermatitis is true? A : It affects females far more frequently than males. B : It has a monozygotic twin concordance of less than 50%. C : It has greatly increased prevalence in rural areas compared to urban areas. D : has had a two to three-fold increase in prevalence in developed countries in the last 30 years. E : Paternal atopy carries a greater risk of offspring disease than maternal atopy. Some studies have shown a slight increase in prevalence in females compared to males in childhood. The monozygotic twin concordance rate is as high as 85%, with the dizygotic rate being approximately 20%. Maternal atopy carries greater risk of disease in the offspring than paternal atopy. Environmental factors are thought to be important in explaining the increased prevalence in developed countries over the last century. For example, migrant individuals from countries with a low prevalence tend to assume rates of disease which are closer to that found in the adopted country. There is also a strong link between atopic dermatitis and higher socio-economic group and smaller family size. These findings have been confirmed for reported and examined dermatitis, and similar trends have been observed in skin prick test reactivity. D : 46. A 50-year-old man presents with erythroderma. Which of the following is least likely to be the underlying cause? A : Eczema B : Psoriasis C : Drug eruption D : Sezary syndrome E : Lichen planus. Comment : Answers A-D are all well recognized causes of erythroderma. Lichen planus has many different clinical presentations. It classically causes itchy, violacious papules at the wrists and very rarely causes erythroderma. E : 47. A 70-year-old female complains of swollen, red, itchy legs. Which is the most likely diagnosis? A : Lymphedema B : Lipodermatosclerosis C : Varicose eczema D : Cellulitis E : Necrobiosis lipoidica. Comment : Varicose eczema is a common problem, particularly in elderly patients with peripheral edema, which tends to exacerbate the problem. It is often mistaken for cellulitis but cellulitis is rarely bilateral and is painful rather than itchy. Lipodermatosclerosis describes the woody, firm, thickening of the skin of the lower legs that can occur in patients with venous hypertension. Necrobiosis lipoidica is usually asymptomatic and typically occurs on the shins of younger patients. Lymphedema is not red and itchy unless complicated by eczema. C : 48. A 60-year-old male with weight loss complains of recent darkening of the skin under his arms. Which of the following statements about acanthosis nigricans is incorrect? A : It is associated with acne. B : It is a cause of hyperpigmentation. C : It is associated with gastric adenocarcinomas. D : It is associated with insulin resistance. E : It is associated with rosacea. Acanthosis nigricans is a warty, velvety, hyperpigmented thickening of the skin, which occurs most commonly in the flexures. It can be idiopathic, familial or drug induced. Many cases are associated with insulin resistance syndromes, including the HAIR-AN syndrome (HyperAndrogenism, Insulin Resistance and Acanthosis Nigricans). Acne can be one of the features of hyperandrogenism. Malignancy is another cause, particularly adenocarcinomas of GI and GU tract.This case is most likely to be paraneoplastic given the age of the patient, recent onset and history of weight loss. Rosacea is not associated with acanthosis nigricans. E :

Page 323: Masterclass Book Part 2

MOHAMMED IS-HAG 322

49. A 16-year-old girl presents with an itchy rash 3 days after arriving in Majorca on holiday. Examination reveals an erythematous papular rash over the arms and trunk, sparing sites under her swimming costume and sparing her face and hands. What is the most likely diagnosis? A : Systemic lupus erythematosus B : Polymorphic light eruption C : Photoallergic contact dermatitis D : Scabies E : Xeroderma pigmentosum. Comment : Polymorphic light eruption is a common photosensitivity disorder which particularly affects young females. The skin shows 'hardening', such that areas frequently exposed to the sun, such as the face and hands, may not be affected whilst newly exposed sites are most severely affected. The rash often develops after a few days of sun exposure and is most severe at the beginning of the summer and improves at the end of the summer as the skin 'hardens'. Systemic lupus erythematosus is a photoaggravated disorder in which the face is usually affected. Photoallergic contact dermatitis can occur due to sunscreen allergy. A rash would appear at all sites where sunscreen had been applied and subsequently exposed to the sun. Scabies is not photoaggavated. Patients with xeroderma pigmentosum describe easy burning after minimal sun exposure. They subsequently develop freckling, chronic solar damage and skin tumours. B : 50. A 60-year-old female has been erythrodermic, scaly and itchy for several months. There is peripheral lymphadenapathy and a blood film shows atypical lymphocytes. What is the most likely diagnosis? A : Systemic sclerosis B : Psoriasis C : Sezary syndrome D : Atopic eczema E : Staphylococcal scalded skin syndrome. Comment : Options B, C and D can cause erythroderma which describes widespread erythema of the skin affecting at least 90% of the surface area. Systemic sclerosis produces diffuse fibrosis of the skin but not erythema. Staphylococcal scalded skin syndrome is due to haematogenous dissemination of a toxin (exfolative toxin) produced by certain strains of staphylococcus aureus. Its substrate is a protein in the epidermis called desmoglein 1. It presents acutely, usually in children, causes a burning sensation in the skin and superficial blistering. Psoriasis, atopic eczema and the sezary syndrome can all present with a scaly erythroderma. Erythrodermic psoriasis tends to be sore rather than itchy. Modest peripheral lymphadenopathy can occur in erythroderma, whatever the cause, as a reactive phenomenon. However, it is a diagnostic feature of the sezary syndrome, a type of cutaneous T-cell lymphoma in which there is systemic involvement. The lymph nodes may be large. The finding of atypical lymphocytes in the peripheral blood and lymphadenopathy indicates Sezary syndrome is the most likely diagnosis in this scenario. C : 51. Which of the following is not a presenting feature of lichen planus? A : Itch B : Mucosal ulceration C : Nail dystrophy D : Alopecia E : Hypopigmentation. Comment : Lichen planus can produce quite marked post-inflammatory hyperpigmentation, even in white skin. It is also very itchy and can produce nail dystrophy and alopecia. Erosive lichen planus is a variant which presents with mucosal ulceration. E : 52. A 22-year-old woman presents with atopic dermatitis. Investigations in individuals with this condition show: A : most patients have elevated total IgE levels. B : IgE titres do not correlate with disease activity. C : most have positive radioallergenosorbent (RAST) tests to food allergens such as nut, wheat or milk. D : few have positive RAST tests to house dust mite, cat, dog, grass. E : skin prick tests show poor correlation with RAST tests to defined allergens. Comment : Total IgE titre does show some correlation with disease activity and it can be a useful screening tool in patients in whom a diagnosis of atopic dermatitis is being considered. Positive radioallergenosorbent (RAST) tests to foods such as nut, wheat and milk are rare in patients with atopic dermatitis, but in Northern European affected adults there are positive RAST tests to: · house dust mite in 50-60% · cat dander in 50% · grass in 75% · birch pollen in 65%. There is generally a good correlation between skin prick test reactivity and RAST test to these allergens. Prick tests involve the intradermal injection of allergen with assessment of the immediate urticarial reaction and are believed to reflect type I allergic reactions. In contrast patch tests involve applying antigen to the skin for 48 hours and assessing the subsequent delayed eczematous reaction.This is

Page 324: Masterclass Book Part 2

MOHAMMED IS-HAG 323

believed to more closely reflect type IV delayed hypersensitivity reactions and is used clinically to investigate patients with suspected contact allergic dermatitis to environmental antigens such as nickel.A: 53. A 40-year-old man with severe atopic eczema fails to respond to potent topical steroids. Possible treatment options do not currently include: A : Azathioprine B : Cyclosporin C : Ultraviolet light D : Mycophenolate mofetil E : Tumour necrosis factor inhibitors. Comment : Many systemic therapies have been used in individuals with severe atopic eczema. Tumour necrosis factor inhibitors (e.g. infliximab) can be helpful in patients with psoriasis, but there is no data on their use for treatment of eczema. E : 54. A 35-year-old woman recounts a 2-month history of weakness; she is having problems climbing the stairs and rising from a chair. On examination, you observe that she has an erythematous rash on her cheeks. An ANA comes back as strongly positive with a negative ENA and DNA.Which is the most likely diagnosis? A : SLE B : Dermatomyositis C : Mixed connective tissue disease D : Anti-phospholipid syndrome E : Primary Raynaud’s disease. Comment : Her history of proximal muscle weakness and rash is compatible with an inflammatory myositis - either complicating SLE or dermatomyositis. Although she has a positive ANA, her negative ENA and DNA and normal complement make dermatomyositis more likely. Patients with idiopathic inflammatory myositis frequently have a positive ANA without ENA positivity.The ANA is usually directed against other nuclear antigens such as PM-Scland U1RNP.B : 55. A 70-year-old female with cardiac failure has dry, red, itchy and swollen legs. What is the most likely diagnosis? A : Allergic contact dermatitis B : Cellulitis C : Erythema nodosum D : Worsening cardiac failure E : Gravitational eczema Gravitational eczema is common in elderly patients with venous hypertension and peripheral oedema. It is also referred to as stasis eczema and in the context of varicose veins, is also referred to as varicose eczema or venous eczema. The skin is typically red, dry, scaly and itchy. Making the diagnosis of cellulitis is a common mistake. Cellulitis is not scaly nor itchy and is rarely bilateral. It is usually painful and patients are unwell with a fever.Treatment of gravitational eczema involves emollients and topical corticosteroids plus treatment of the underlying oedema and venous hypertension with compression hosiery. Allergic contact dermatitis can complicate gravitational eczema if patients are allergic to one of the creams they are using to treat themselves, or to the bandages being applied. E : 56. A 70-year-old man presents to a Dermatology clinic with generalized itching for 6 months. On examination no rash is seen. Several blood tests are taken to exclude a systemic cause for his pruritus. Which of the following is NOT likely to be helpful? A : Urea and electrolytes B : Liver function tests C : Ferritin D : B12 E : Thyroid function tests. Comment : B12 deficiency does not normally cause pruritus. Iron deficiency, investigated with the serum ferritin, is a cause of generalized pruritus.Chronic renal impairment and cholestasis can both cause itching. Hyperthyroidism and hypothyroidism are also systemic causes of itch. Other possible systemic causes of itch are polycythaemia rubra vera and underlying malignancy. D : 57. A 60-year-old man presents with a palpable, non-blanching rash on the lower legs. Which of the following investigations is least likely to help with the underlying diagnosis? A : Antistreptolysin-O test (ASOT) B : Antinuclear antibodies (ANA) C : Hepatitis B antibodies D : Thyroid function tests E : Cryoglobulins. Comment :A palpable, purpuric rash clinically represents an underlying vasculitis. Common causes for cutaneous vasculitis are infections (e.g. Streptococcal, Hepatitis), drug reactions and autoimmune diseases.Thyroid function abnormalities are not usually associated with vasculitis. D :

Page 325: Masterclass Book Part 2

MOHAMMED IS-HAG 324

1. A 72-year-woman is admitted after a fall. She lives alone and works in the hospital benevolent society shop. You are called to see her because she is agitated, accusing staff of stealing her jewellery and wanting to leave. Which of the following would be your first intervention? A : Prescribe haloperidol 0,5mg to calm her down and call the psychiatrist on call to assess whether

she should be detained under the Mental Health Act. B : Order an urgent CT head scan. C : Do a physical examination and review the blood and urine test results D : Interview the patient to establish her concerns, take a history, assess her mental state and try to

reassure her. E : Ask the nurses to move her to a quiet, well lit room and try to get a family member to come and

sit with her. Comment : A sudden change in the mental state or sudden onset of behaviour that is out of character in an older person is most likely to be due to an acute confusional state (delirium). Even if a patient is confused it is paramount to take a history, which may give clues to the cause of the delirium. It also gives you the opportunity to assess the severity of confusion. The patient will benefit from reassurance. Of course, don’t dismiss the possibility that the patient is not confused and her jewellery is indeed missing! A physical review and review of investigations must be done as soon as is possible as delirium is a serious condition with a high morbidity and mortality. Sedation should only be used as a last resort and preferably only once the cause of the delirium has been established. D : 2. A 20-year-old university student is brought to A&E by his friends. He is complaining of 'feeling unreal'. Identify which one of the following statements is FALSE? A : Olfactory hallucinations and déjà vu suggest temporal lobe epilepsy (TLE). B : Visual hallucinations and intense fear suggest recent LSD consumption. C : Third person auditory hallucinations suggest schizophrenia. D : Hyperventilation and disproportionate worrying suggest mania. E : The presence of passivity and primary delusions suggest schizophrenia. Comment : In dissociative disorders there is a temporary but drastic modification of a person’s character or sense of personal indentity.Conditions that can present with dissociative symptoms include schizophrenia, TLE, depression, head injury, delirium, dementia, drugs and acute stress disorder. Other features of TLE can include epigastric discomfort, jamais vu, and intense anxiety or depression. LSD can cause hallucinations in any modality.Hyperventilation and disproportionate worrying suggest acute stress disorder. D : 3. A 28-year-old woman is brought to the Accident and Emergency Department after taking an overdose of 12 x 10 mg temazepam tablets. In assessing her risk of suicide, which of the following statements is FALSE? A : If she seems to have memory impairment, then her risk of suicide is relatively high. B : The overdose is not life-threatening, indicating that her risk of suicide is relatively low. C : If she had prepared by hoarding tablets, then her risk of suicide is relatively high. D : If she has insomnia, then her risk of suicide is relatively high. E : A past history of deliberate self harm (DSH) means that she has a relatively high risk of suicide. Comment : High risk clinical factors for suicide include severe insomnia, self neglect, memory impairment, agitation, panic attacks, pessimism, despair, anhedonia and morbid guilt. Other factors predicting high risk are declared intent, preparation, past history of DSH, severe depression, schizophrenia and substance abuse, and the use of a potentially lethal method. However, patients are not experts in pharmacology, and taking a small overdose of a relatively safe drug (as in this case) does not mean that the suicide risk is necessarily low.For 10 yrs after an episode of DSH the risk of suicide is increased 30-fold over that expected. 1% of patients kill themselves in the year after an episode of DSH, and 20-25% of people who die by suicide have presented to hospital after episodes of DSH in the year before their death. B : 4. A very thin 18-year-old girl is referred for investigation of weight loss. You suspect that she has anorexia nervosa, but perform a range of screening tests. Which one of the following would be compatible with the diagnosis of anorexia nervosa? A : Hyperkalaemia B : Low serum bicarbonate C : Low serum cholesterol D : Low white cell count E : Elevated ESR.

Page 326: Masterclass Book Part 2

MOHAMMED IS-HAG 325

Comment : A wide range of biochemical / haematological abnormalities can be found in anorexia nervosa. These include hypokalaemia, hypochloraemic alkalosis (both due to vomiting and/or diuretic/laxative abuse) and hypercholesterolaemia (mechanism unknown). The ESR is normal or reduced. D : 5. A 58-year-old man with no known past medical or psychiatric history is admitted to coronary care with an inferior myocardial infarct. He receives treatment with thrombolysis and is started on a beta-blocker, aspirin and a statin. He is recovering well and is transferred to one of the general medical wards. On day 5 he becomes mentally disturbed, starts to hear voices in his head, and accuses staff on the ward of trying to kill him. He says that he wants to go home and you cannot convince him to agree to stay. It would be appropriate for you to explain to him that you: A : have no right to detain him, but will ask him to sign to say that he is discharging himself against

medical advice B : have the right to detain him under Section 2(2) of the Mental Health Act C : have the right to detain him under Section 3(1) of the Mental Health Act D : have the right to detain him under Section 5(2) of the Mental Health Act E : have the right to detain him under Section 7(1) of the Mental Health Act. Comment : If a patient is already being nursed on a medical, surgical or obstetric ward, or in an intensive care or high-dependency unit, and they develop a mental illness de novo, or have an exacerbation or relapse of a pre-existing disorder, their physician or surgeon can authorise their compulsory detention for up to 72 hours under section 5(2) of the Mental Health Act. During those 72 hours the medical or surgical team must request a formal assessment by a consultant psychiatrist, the patient’s own general practitioner and an approved social worker. D : 6. A 40-year-old woman is referred to the general medical clinic with a 9-month history of feeling exhausted. She is unable to do anything that requires physical exertion and feels miserable because of this. A comprehensive battery of tests fails to establish any clear medical cause for her symptoms and you think that she has chronic fatigue syndrome. Which one of the following treatments is of definite benefit in this condition? A : Graded exercise programme B : Prolonged rest C : Corticosteroids D : Benzodiazepines E : Allergen avoidance. Comment : Graded exercise programmes and cognitive-behavioural therapy are the only two treatments shown to be of definite benefit in chronic fatigue syndrome. Prolonged rest is ineffective and tends to be harmful. A : 7. A 38-year-old man presents with depression and you think that he would benefit from an antidepressant. Which one of the following statements regarding antidepressant medications is true? A : Selective serotonin reuptake inhibitors are more potent antidepressants than tricyclics. B : Monoamine oxidase inhibitors increase the amount of noradrenalin and 5HT released from nerve

terminals. C : Fluoxetine has no anticholinergic side effects. D : Lofepramine is more cardiotoxic than amitriptyline. E : Selective serotonin reuptake inhibitors are more sedative than tricyclics. Comment : Selective serotonin reuptake inhibitors (SSRIs), eg. fluoxetine, are of equal efficacy to tricyclics, but are less sedative and have fewer cardiovascular effects. They also have no anticholinergic side effects, but they can cause gastrointestinal problems, agitation, insomnia and headache. Side effects of tricyclics are anticholinergic (eg. dry mouth, blurred vision, constipation, urinary retention), noradrenergic (eg. postural hypotension), cardiac arrhythmias (particularly in overdose, lofepramine less so than amitriptyline), reduction of seizure threshold, weight gain and sexual dysfunction.Monoamine oxidase inhibitors inhibit re-uptake of NA and 5HT, thereby increasing the amount of neurotransmitter available at the synapse. C : 8. A 30-year-old man is involved in a multiple car crash in which several people die. He is not physically injured himself, but in the days that immediately follow he feels numb and detached, dazed and disorientated, with physical symptoms of sweating and shakiness. The most likely diagnosis is: A : Acute stress disorder B : Post-traumatic stress disorder C : Adjustment disorder D :Panic attack E : Depression.

Page 327: Masterclass Book Part 2

MOHAMMED IS-HAG 326

Acute stress disorder is a short-lived but severe disorder caused by an overwhelming, psychologically traumatic experience. The symptoms develop rapidly, but tend to resolve within a matter of days. These include psychological symptoms, such as feeling numb and detached, dazed and disorientated, and physical symptoms, such as sweating, shakiness, palpitations and insomnia. Some patients will go on to develop post-traumatic stress disorder.If there is persistent denial that the event has occurred, the patient should be cautiously prompted to recall the facts. A short course of a benzodiazepine tranquilliser and/or hypnotic may help severe agitation or insomnia. A : 9. You need to decide which antidepressant to prescribe for an emaciated agitated 78-year-old man who you are seeing as an out-patient. Which one of the following statements is true? A : Mirtazapine, which enhances both noradrenergic and serotinergic transmission, would be a good

choice. B : Fluvoxamine may be useful because it rarely causes nausea. C : Antidepressants that are H1 receptor agonists help patients to lose weight. D : An SSRI should be avoided because of concern about cardiotoxicity in an older patient. E : St John's Wort may be useful as it is not associated with any drug interactions. Comment : All the SSRIs can cause nausea, which is more common with fluvoxamine. The major advantage of SSRIs as a class of drug is that they are safer in overdose than tricyclics, particularly with respect to cardiac problems. Mirtazapine blocks alpha-2, 5-HT2A and 5-HT3 receptors, thus increasing the amounts of both noradrenaline and serotonin in the synaptic gap. It also has a high affinity for H1 receptors so it tends to cause weight gain and drowsiness, a good choice for a thin agitated patient. St John's Wort is associated with drug interactions, which can cause problems, particularly since it is available over the counter. A : 10. A very thin 17-year-old girl is referred for medical opinion. A comprehensive battery of tests fails to establish any clear medical cause for her condition and you think that she has an eating disorder. Which one of the following statements about anorexia nervosa and bulimia nervosa is true? A : In both anorexia nervosa and bulimia nervosa body weight is significantly reduced. B : Patients with anorexia nervosa judge self-worth largely or exclusively in terms of shape and

weight, but those with bulimia nervosa do not. C : In anorexia nervosa body weight is significantly reduced, but in bulimia nervosa it is often

normal. D : Patients with anorexia nervosa and bulimia nervosa are both in control of their eating, but exert

this control in different ways. E : Patients with anorexia nervosa tend to give a reliable history of how much (and how little) they

eat, whereas patients with bulimia nervosa tend to conceal their history of vomiting. Comment : The ICD-10 critera for anorexia nervosa require BMI <17.5, self-induced weight loss, body image distortion, and abnormalities of the hypothalamic-pituitary-gonadal axis. The DSM-IV criteria for bulimia nervosa require recurrent episodes of binge eating, recurrent inappropriate compensatory behaviour to prevent weight gain, with these features occurring more than twice weekly for three months. There is no diagnostic requirement for weight loss. In both anorexia nervosa and bulimia nervosa self-worth is judged largely or exclusively in terms of shape and weight, and in both conditions a reliable dietary history is unlikely to be obtained from the patient. A key feature in the binge eating of bulimia nervosa is a feeling of loss of control. C : 11. A 54-year-old man is admitted to hospital in a neglected state. He appears anxious, agitated, shaky and sweaty. He reports that he stopped drinking alcohol two days previously. After initial treatment to control his withdrawal symptoms, you talk to him about his usage of alcohol. Which of the following is NOT one of the DSM-IV criteria for alcohol dependence? A :Inability to cut down B : Repeated efforts to control drinking C : Serum gamma-GT over three times the upper limit of normal D : Drinking increasing amounts E : Withdrawal symptoms. Comment : The six DSM-IV criteria for alcohol dependence are: 1. inability to cut down 2. repeated efforts to control drinking 3. amnesic periods 4. drinking increasing amounts 5. ongoing drinking despite detrimental consequences 6. withdrawal symptoms. C :

Page 328: Masterclass Book Part 2

MOHAMMED IS-HAG 327

12. You are asked to see a 52-year-old woman with metastatic breast cancer in clinic who is tearful and feels that she can no longer cope. What would be the most appropriate course of action? A : Screen for depression / suicidal thoughts and liaise with primary care team B : Prescribe antidepressants C : Reassure her that these feelings are normal in this situation D : Listen to her concerns and refer her for counselling E : Refer her for psychiatric assessment. Comment : Depression is common in patients with advanced cancer (20-25% incidence) and can respond to treatment. Diagnosing depression with screening tools such as the Hospital Anxiety and Depression scale (HAD) or basic clinical interview is within the capabilities of all doctors. In a clinic setting you should screen for depression, which includes listening to her concerns. If her feelings overwhelm her it is important to ask if she has thought about taking her own life. Studies suggest that patients don’t often volunteer this but appreciate being asked and it does not ‘put ideas into their head’. Good communication with the primary care team about appropriate management, that might include antidepressants, is vital. A : 13. A 27-year-old man with a body builder’s physique is brought to you handcuffed by the police. He is agitated, sweating and lashing out at the police. He has pressured speech that does not really make sense. What is the first thing you would do? Choose the best option from the following: A : Ask the police to release him so you can do a physical examination B : Sedate him with Haloperidol 10mg IMI C : Quickly get as much collateral information as you can from all sources D : Try talking to him in a calm way to find out what is wrong with him E : Immediately transfer him to a psychiatric ward. Comment : At this point information is what you most need. Get collateral information, including from the police, A&E staff who might recognize him or family if you have a phone number. The five minutes doing this will be well spent. Has he recently been physically ill with a fever? Does he have a psychiatric or substance use history? Is he habitually violent? Having background information will guide you to the more likely conditions. Unfortunately, sometimes no collateral information is available. Of course, the second step that should be started urgently is to talk to the patient and begin assessing him. C : 14. A 28-year-old woman with a history of manic depression has been well controlled on lithium for several years. She wishes to get pregnant and asks for advice regarding the safety of lithium in pregnancy and when breast-feeding. Which one of the following statements is true? A : Lithium is safe during pregnancy and when breast-feeding. B : Lithium is contraindicated during the first trimester and when breast-feeding. C : Lithium is contraindicated in the first trimester but is safe when breast-feeding. D : Lithium is safe during pregnancy but is contraindicated when breast-feeding. E : Lithium is safe during the first trimester, but contraindicated during the second and third

trimesters and when breast-feeding. Comment : In the first trimester lithium can cause atrialisation of the right ventricle. During the second and third trimesters lithium can be used, but dose requirements are increased. Immediately after delivery lithium dose requirements return to normal abruptly. Lithium levels can rise dangerously if a high dose is continued. Lithium is excreted in breast milk and if the infant becomes dehydrated, then toxic lithium levels develop rapidly. B : 15. A 23-year-old man, studying for a higher degree in pharmacology, develops schizophrenia. You suggest that he would benefit from treatment with thioridazine. He is very concerned about the possibility of drug toxicity and produces a long list of possible side effects. Which one of the following is NOT a recognised side effect of thioridazine? A : Sedation B : Jaundice C : Leucopenia D : Orthostatic hypotension E : Diarrhoea. Comment : The side effects of antipsychotics include extrapyramidal symptoms (stiffness, tremor, hypersalivation, acute dystonia, akathisia, tardive dyskinesia), anticholinergic symptoms (blurred vision, constipation, urinary retention, dry mouth, confusion, agitation, seizures), antihistaminergic symptoms (sedation), alpha-blocking effects (orthostatic hypotension), leucopenia, increased prolactin secretion (amenorrhoea, galactorrhoea, sexual dysfunction), weight gain, obstructive jaundice, retinitis pigmentosa (with thioridazine >600 mg/d), allergic dermatitis / photosensitivity and neuroleptic malignant syndrome. E :

Page 329: Masterclass Book Part 2

MOHAMMED IS-HAG 328

16. A 42-year-old woman is referred to the medical outpatient clinic because she is ‘always exhausted’. She says that she cannot do anything that requires any physical effort and that she sleeps all the time. Her general practitioner cannot find any explanation for her symptoms. You consider the diagnosis of chronic fatigue syndrome. Which one of the following findings would NOT be consistent with this diagnosis? A : Subjective memory impairment B : Tender lymph nodes C : Muscle pain D : Joint pain E : Weight loss. Comment : Chronic fatigue syndrome is defined as clinically evaluated, medically unexplained fatigue of at least six months’ duration that is: · Of new onset · Not a result of ongoing exertion · Not substantially alleviated by rest · A substantial reduction in previous levels of activity. Together with the occurrence of four or more of the following symptoms: · Subjective memory impairment · Tender lymph nodes · Muscle pain · Joint pain · Headache · Unrefreshing sleep · Postexertional malaise E : 17. Mr Jonas is admitted from his residential home with pneumonia. He is on long-term thioridazine for chronic schizophrenia. Which one of the following statements is true? A : Agranulocytosis can occur with all antipsychotics but is most frequent with clozapine. B : On examination you are surprised to find that that he has Parkinsonian features because these

are not usually a feature of thioridazine. C : Thioridazine has been withdrawn from routine use because it causes agranulocytosis. D : Risperidone should be avoided. E : Akathisia is pins and needles in the extremities. Comment : All antipsychotic medication is associated with the development of Parkinsonian features; this is most marked with drugs such as haloperidol and trifluoperazine and least marked with the newer 'atypicals'.Thioridazine has been withdrawn because of its potential cardiac toxicity (long QT interval) . Risperidone is a popular antipsychotic in older people though it is expensive. Clozapine is an excellent antipsychotic but its use is very carefully restricted because of the high incidence of agranulocytosis. Akathisia describes the motor restlessness seen in drug induced Parkinsonism (Latin 'out of a seat'). A : 18. A 48-year-old man with a long history of alcohol abuse is admitted after being found collapsed in the street. He is agitated and tremulous. You make a diagnosis of alcohol withdrawal. Which one of the following is appropriate management for this condition? A : Nurse in slightly darkened room B : Monitor for hyperglycaemia and control blood glucose using an insulin infusion if necessary C : Give antipsychotics to reduce agitation and likelihood of seizures D : Avoid benzodiazepines to reduce likelihood of respiratory depression E : High-calorie, high-carbohydrate diet. Comment : Important aspects of immediate management of alcohol withdrawal include benzodiazepines (reducing schedule), nursing in well-lit quiet environment, thiamine supplementation, monitoring for and treating hypoglycaemia, rehydration, high-calorie high-carbohydrate diet, and avoiding antipsychotics (which lower seizure threshold). E : 19. A 32-year-old man with a history of opioid abuse is admitted to hospital after a road traffic accident in which he fractures his femur. He becomes unwell and you are asked to assess him on the ward. Which one of the following features would NOT be compatible with a diagnosis of an opioid withdrawal reaction? A : Nausea / vomiting B : Hypothermia C : Lacrimation D : Yawning E : Diarrhoea. Comment : Symptoms of opioid withdrawal include dysphoric mood, yawning, insomnia, muscle aches, lacrimation / rhinorrhoea, papillary dilatation, piloerection, fever, sweating, nausea / vomiting, diarrhoea.If the patient is having an opioid withdrawal reaction, then give 10 mg of methadone syrup and wait about 60 min to determine its effect. B :

Page 330: Masterclass Book Part 2

MOHAMMED IS-HAG 329

20. A 34-year-old woman is brought into A&E after taking an uncertain quantity of paracetamol two hours previously and trying to hang herself. She becomes agitated and insists that she wants to go home immediately. You judge that she is at high risk of suicide. How should you proceed? A : Call the duty psychiatrist, and with other staff in the A&E department attempt to restrain her

under Common Law until they arrive B : Ask her to sign a ‘discharge against medical advice’ form and let her go C : Call the duty psychiatrist, but let the patient go if she insists and the duty psychiatrist does not

arrive in time to see her D : Detain her under section 5(2) of the Mental Health Act E : Call the hospital security services, restrain her and sedate her. Comment : In an A&E department the suicidal patient who declines to be admitted for observation and treatment should be managed as follows: · Ensure that a member of staff stays with them at all times · Call the duty psychiatrist · If they attempt to abscond before or during psychiatric assessment, the staff of the A&E department have a duty under Common Law to restrain the patient If a patient who is already being nursed on medical, surgical or obstetric ward, or in a high dependency or intensive care unit, develops a mental illness (or has an exacerbation of a pre-existing disorder), their physician or surgeon can authorise their compulsory detention for up to 72 hours under section 5(2) of the Mental Health Act. A : 21. A 78-year-old man, without significant past medical history and taking no regular medications, has become increasingly forgetful over the last 18 months, to the point where he now finds it difficult to remember the names of some members of his family. He has recently been having visual hallucinations. On examination he seems rather expressionless and has cogwheel rigidity of both arms. The most likely diagnosis is: A : Alzheimer’s disease B : Lewy body dementia C : Vascular dementia D : Frontal lobe dementia E : Alzheimer’s disease and Parkinson’s disease. Comment : Lewy body dementia is characterised by fluctuating cognition, visual hallucinations, parkinsonism, neuroleptic sensitivity, falls / transient loss of consciousness / syncope and delusions. Vascular dementia typically occurs in those with widespread vascular disease. The course is typically fluctuating with stepwise progression. Differentiation from Alzheimer’s disease can be difficult, although a history of strokes or the presence of focal neurological signs are very suggestive. B : 22. An 85-year-old man is admitted from home because he has become increasingly confused and is not coping. He is known to have metastatic carcinoma of the prostate and takes Zoladex 3 monthly. You note that he has bruising over the left side of his forehead. Which of the following investigations will not help you diagnose and treat his confusion? A : CT scan of the head B : Midstream urine sample C : Calcium D : Urea and electrolytes E : Prostatic specific antigen (PSA). Comment : This patient deserves a CT scan of his head to exclude not only coexisting vascular dementia but also a possible subdural haemorrhage, as there is evidence that he has had at least one fall. It is important to exclude intercurrent infections including a UTI and chest infection. Dehydration and hypercalcaemia should also be excluded. The PSA may give you some information about the extent of the prostatic disease, but not the cause of the confusion. E : 23. You are asked to see a very thin 18-year-old girl. You suspect that she has anorexia nervosa. On examination, which one of the following features would NOT be expected if this diagnosis were correct? A : Tachycardia B : Hypotension C : Oedema D : Lanugo E : Hypothermia. Comment : When examining this patient you should: · Establish her height and weight and calculate her BMI · Look for signs of starvation: hypothermia, lanugo, loss of muscle mass, dependent oedema, bradycardia, hypotension, neuropathy. · Look for signs that she is inducing vomiting: salivary gland enlargement, dental erosion, calluses on the fingers / knuckles. A :

Page 331: Masterclass Book Part 2

MOHAMMED IS-HAG 330

24. A 58-year-old man, with a long history of alcohol abuse, is admitted in a dishevelled and confused state. He is very unsteady on his feet. Which of the following findings would most strongly support a diagnosis of Wernicke’s encephalopathy? A : Fluctuating conscious level with focal neurological signs B : Nystagmus and intention tremor C : Nystagmus and ataxia of gait D : Tachycardia and tremulousness E : Drowsiness, jaundice and metabolic flap. Comment : Wernicke’s encephalopathy typically presents with ophthalmoplegia (horizontal and vertical nystagmus, weakness / paralysis of the lateral rectus muscles, weakness / paralysis of conjugate gaze), ataxia (predominantly affecting stance or gait, and often without clear-cut intention tremor) and confusion.Presentation A would suggest subdural haematoma, presentation D alcohol withdrawal, and presentation E hepatic encephalopathy. C : 25. A 34-year-old woman develops chest pain after an argument with her 17-year-old daughter. She is brought to the Accident and Emergency department where you are asked to see her. She is hyperventilating and looks very anxious. She is tender to light pressure on the front of her chest, but examination is otherwise unremarkable. Breathing room air, her oxygen saturation (finger probe) is 99%. Her ECG is normal. You should: A : Check troponin. Plan to repeat ECG in 2 hours. Explain that you think that there is probably

nothing serious going on, but you want to make sure that she has not had a heart attack B : Check D-dimer and troponin. Explain that you think that there is probably nothing serious going

on, but you want to make sure that she has not had a clot of blood in the lung or a heart attack C : Explain to her that she may be having a heart attack and that you wish to admit her to the

coronary care unit for close monitoring D : Explain to her that you are going to ask one of the nurses to put a paper bag over her head to

help her breathing E : Explain that she has had a panic attack and that her symptoms are a consequence of this. Help

her to control her breathing rate, and say that you think everything will settle down and she will be able to go home

Comment : It was essential to check the ECG and pulse oximetry, but the clinical context and examination findings all point to the diagnosis of a panic attack, and – given that these tests gave normal results – it would be appropriate to explain the diagnosis to the patient and reassure accordingly. Psychological symptoms of an anxiety state include: · irritability · Intolerance of noise · Poor concentration / memory · Fearfulness · Apprehensiveness · Restlessness · Continuous worrying thoughts. Physical symptoms of an anxiety state include: · Dry mouth · Difficulty in swallowing · Chest pain · Shakiness · Diarrhoea · Urinary frequency · Paraesthesiae · Hot flushes. Physical signs of an anxiety state include: · Tenseness · Sweating · Shaking · Pallor · Restlessness · Sighing. E : 26. A 48-year-old with recurrent carcinoma of the colon admits to poor sleeping, anhedonia and feelings of hopelessness. Which of the following would you do? A : Reassure him that this is a normal response to his predicament B : Start anti-anxiolytic drugs C : Suggest counselling D : Start an anti-depressant E : Suggest relaxation.

Page 332: Masterclass Book Part 2

MOHAMMED IS-HAG 331

Comment : Depression is the most likely diagnosis. This needs to be treated along conventional lines. Other modalities may be of additional benefit. D : 27. A 23-year-old man is brought to A&E by ambulance. He is known to have schizophrenia and he is being prescribed oral trifluoperazine. He is mute. His head is turned to the right in an awkward and extreme position. He appears distressed. The ambulance crew tell you he has been like this for about 30 minutes. Other than this odd posture his physical examination was normal. From the list below choose the immediate intervention that is most likely to be appropriate: A : Administer an extra dose of trifluoperazine. B : Reduce the dose of trifluoperazine. C : Administer intramuscular dose of procyclidine. D : Refer to on call psychiatrist. E : Prescribe dose of oral lorazepam. Comment : This is most likely an acute dystonic reaction to the trifluoperazine. Acute dystonia as an adverse effect of antipsychotic drugs affects about 2% of patients. It occurs abruptly and early in the course of treatment. It is generally associated with piperazine phenothiazines (e.g. trifluoperazine) and butyrophenones (e.g. haloperidol). Males, young adults and children are more susceptible. It is best treated with parenteral anticholinergic drugs and relief of symptoms should be swift. C : 28. A 30-year-old woman, with a long history of intravenous drug abuse, is admitted with an abscess in her right groin related to the use of non-sterile needles. This is drained, and she is admitted to a medical ward for intravenous antibiotic treatment. She ‘goes missing’ from the ward at 14.00h, but is found outside a back entrance of the hospital smoking a cigarette at 15.30h and then returns to the ward. At 21.30h you are asked to see her by the ward nurses because she says she feels very unwell. She is nauseated, has had some diarrhoea, and yawns as you talk to her. Her temperature is 37.6 C and her pupils are dilated. The most likely diagnosis is: A : Opioid toxicity B : Alcohol withdrawal C : Clostridium difficile infection D : Opioid withdrawal reaction E : Toxic shock syndrome. Comment : The symptoms and signs of opioid withdrawal include dysphoric mood, yawning, insomnia, nausea, vomiting, diarrhoea, muscle aches, lacrimation / rhinorrhoea, pupillary dilatation, piloerection, sweating and fever.Initially give 10 mg of methadone syrup and wait about 60 min to determine its effect. Continue administering in 10 mg doses until symptoms are under control. It is rare to exceed a total dose of 40 mg over 24 hours. D : 29. A 52-year-old man, with a history of heavy alcohol consumption, is admitted to hospital with a chest infection. He is tremulous and confused due to alcohol withdrawal. Along with treating his chest infection, giving thiamine / other vitamins, and preventing hypoglycaemia, you will prescribe chlordiazepoxide at a dose of: A : 500 mg po four times daily B : 10 mg po four times daily C : 50 mg IM as needed D : 30 mg po four times daily E : 10 mg IM as needed. Comment : If in doubt regarding a drug dose, always look in the British National Formulary (BNF). D : 30. A 59-year-old school teacher was diagnosed with depression and anxiety when he felt unable to cope with the pressures of work. When you saw him his mood and anxiety were improved. His wife reported that he was increasingly absent-minded. On cognitive testing his recall was poor and he made errors on the serial seven subtraction exercise. Choose the two most appropriate management options listed below: A : Prescribe an antidepressant. B : Advise retirement on medical grounds. C : Arrange electroconvulsive therapy. D : Arrange psychometric testing. E : Refer for cognitive behaviour therapy. F : Arrange a dementia screen including neuroimaging. G : Refer for marital therapy. H : Observe for a few months and then review. I : Prescribe lithium. J : Prescribe a benzodiazepine. In the early stages of dementia, patients may present with low mood and anxiety symptoms. Depressive symptoms affect up to 50% of patients with dementia. This is confounded by the fact that patients with depression can present with a ‘pseudodementia’. Cognitive impairment must always be actively investigated and a diagnosis made so that appropriate treatment options can be explored. D : F:

Page 333: Masterclass Book Part 2

MOHAMMED IS-HAG 332

31. Of the following, the commonest underlying organic cause of anxiety symptoms is: A : phaeochromocytoma B : carcinoid C : carcinoma of the bronchus D : hypoglycaemia E : hyperparathyroidism. Comment : In addition to hypoglycaemia, relatively more common organic causes of anxiety include alcohol withdrawal, drug intoxication or withdrawal, thyrotoxicosis and paroxysmal supraventricular tachycardias. Phaeochromocytoma is rare. Carcinoid does not cause anxiety. Carcinoma of the bronchus and hyperparathyroidism are more likely to present with depression. D : 32. A 40-year-old man from Uganda is brought in by the police under section 136 of the Mental Health Act. He was arrested because he was standing naked in a busy street attempting to ‘direct’ the traffic. On examination he declared that he was the Minister of Transport. He spoke rapidly and paced around the room issuing orders in a loud voice. He was found to have oral thrush. From the following choose the two LEAST likely diagnostic possibilities: A : Mania B : Tertiary Syphilis C : Amphetamine use D : Gonorrhoea E : Adverse effects of antiretroviral drugs F : Neuropsychiatric sequelae of HIV G : Delirium due to secondary infection H : Cocaine use I : Post traumatic stress disorder J : Psychological reaction to the diagnosis of AIDS. The most common aetiological factors in HIV/AIDS-associated psychotic disorders are: prexisiting psychotic disorders, substance misuse, psychogenic reaction, iatrogenic factors and HIV related brain disease. D : I: 33. A 46-year-old with pancreatic cancer attends clinic. Her pain is now well controlled on methadone; however, she spends much of the day in bed and is refusing food. What are the most helpful factors in distinguishing the disease process and appropriate sadness from depression? A : Anorexia B : Weight loss C : Hypersomnia D : Social withdrawal E : Feelings of worthlessness F : Nocturnal wakening G : Fatigue H : Anhedonia I : Reduction in normal activities J : Thoughts of death. It can be difficult to differentiate between depression and the sadness associated with an impending and untimely death. Thoughts of guilt and self-worthlessness are usually found in those who are depressed, and it is unusual for those who are sad not to gain some enjoyment from some activities. Other questions should include those about suicidal thoughts. E : H: 34. A known alcoholic presents to A&E with confusion. There are no signs of acute alcohol intoxication. You suspect Wernicke's encephalopathy. Which of the following features are not compatible with this diagnosis? A : bilateral lateral recti gaze palsy B : disorientation C : hypothermia D : markedly elevated blood ammonia level E : seizures F : postural hypotension G : apathy H : multidirectional nystagmus I : memory loss J : pupillary abnormalities. Wernicke's encephalopathy (WE) is seen in patients with a poor nutritional state, due to either poor intake or absorption, or increased metabolic requirement of thiamine. It may be precipitated acutely in at-risk patients by intravenous glucose administration. The main features are confusion, developing over days or weeks with inattention, apathy,disorientation, and memory loss.Coma is rare. Most patients have bilateral lateral recti palsies,occasionally with other muscles involved, multidirectional nystagmus, and pupillary abnormalities. Truncal ataxia, and brain stem autonomic dysfunction are also seen. There are no distinctive laboratory findings; an elevated ammonia is seen in hepatic encephalopathy. The mortality rate of treated WE is 10-20%. Often patients are left with a disorder of impaired memory and learning - Korsakoff's syndrome. Thiamine is the only treatment known to alter the outcome, and should be given parenterally in the acute setting. D : E :

Page 334: Masterclass Book Part 2

MOHAMMED IS-HAG 333

1. A 78-year-old woman, who has been feeling generally rotten and troubled by headaches recently, suddenly loses vision in one eye. The most likely diagnosis is: A : Migraine B : Retinal detachment C : Scleritis D : Demyelinating optic neuritis E : Ischaemic optic neuritis. Comment : An elderly patient with acute and painful visual loss has giant cell arteritis until proved otherwise.Any middle-aged or older patient presenting with acute loss of vision should be asked about headaches, scalp tenderness, jaw pain and symptoms of polymyalgia rheumatica.Ischaemic optic neuritis is a medical emergency: visual loss in the affected eye is irreversible, and the second eye is at immediate risk. Treatment with oral prednisolone 1 mg/kg/day is indicated, and some physicians would give intravenous methylprednisolone if both eyes were affected. E : 2. A middle-aged man reports that his vision has gradually become blurred, especially in the right eye. He is finding it difficult to read, even in bright light, and has great difficulty when driving at night because of glare from oncoming headlights. He had a successful renal transplant seven years ago and is on low dose maintenance immunosuppression and antihypertensive medication. His mother had renal failure and glaucoma. What is the diagnosis? A : Iritis B : Retinitis pigmentosa, related to the underlying cause of renal failure C : Glaucoma D : Cataract E : Side effect of antihypertensive medication. Iritis causes photophobia but will have an acute onset, usually with a red eye. These symptoms usually predominate so that iritis rarely presents with blurring of vision as a cardinal symptom. It is also usually unilateral.Retinitis pigmentosa (RP) can occur in some types of renal failure, especially Alport’s syndrome, associated with deafness and a positive family history. The cardinal symptom in RP is poor night vision, but glare is not a typical description and symptoms are very gradually progressive from a relatively early age.Glaucoma causes gradual loss of visual field and is rarely symptomatic in middle age, even with a positive family history. Glaucoma is not usually associated with renal failure or systemic medication.Cataract is relatively common in renal transplant patients, as risk factors include renal failure and long-term systemic corticosteroid medication. The symptoms are typical, as vision becomes worse in bright light when the pupil constricts, confining the light path to the central part of the lens where it is thickest and the typical steroid-induced cataract most pronounced.Antihypertensive medications cause many side effects, but blurring of vision is not usually one of them.D : 3. A 58-year-old man suddenly loses all vision in one eye. It begins to improve 30 minutes later, and a few hours later is back to normal. The most likely diagnosis is: A : Retinal arterial occlusion B : Retinal vein occlusion C : Retinal detachment D : Vitreous haemorrhage E : Acute optic neuropathy. Comment : Complete loss of vision in one eye only, even if only in part of the field of vision, is likely to be due to an arterial event in the retina, either amaurosis fugax or a completed stroke. Retinal transient ischaemic attacks are usually brief (<30 mins) and almost always due to an embolic event, most typically from the ipsilateral carotid bifurcation.Occlusion of the retinal vein presents acutely, although not as abruptly as an arterial event, and the loss of vision is not transient. A : 4. An 83-year-old woman is troubled by poor vision, although she hasn't mentioned this to her doctor. Which of the following statements is NOT correct? A : Visual impairment is associated with reduced quality of life. B : Visual acuity of less than 6/12 is found in around 20% of those aged 75 years or older. C : Visual impairment screening programmes should be set up for older people. D : Poor vision is a risk factor for hip fracture. E : Age related macular degeneration and cataracts account for over three quarters of older people

with impaired vision. Comment : Although visual impairment is common, a recent randomised controlled trial of screening older people for impaired vision in primary care concluded, disappointingly, that including a visual screening component in multidimensional screening for older people did not lead to improved visual outcomes. Unless ways are developed to improve vision of older people,screening is not worthwhile. C :

Page 335: Masterclass Book Part 2

MOHAMMED IS-HAG 334

5. A 48-year-old man with diabetes finds that the vision in one eye is blurred when he reads, but not at other times. The most likely diagnosis is: A : Macular oedema B : Floaters C : Cataract D : Glaucoma E : Stroke. Comment : Visual impairment more marked for reading than distance is very suggestive of macular disease, and the likely cause of symptoms in this case is diabetic maculopathy, when the central fovea becomes affected by retinal oedema or frank hard exudate. Age-related macular degeneration would be unlikely in a man of this age. A : 6.

PLATE 58 This image of the fundus of a 68-year-old man with type II diet-controlled diabetes mellitus and a blood pressure of 150/100 shows: A : a normal fundus B : diabetic retinopathy with dot and blot haemorrhages and new vessels on the optic disc C : background diabetic retinopathy with dot and blot haemorrhages D : background diabetic retinopathy with dot and blot haemorrhages and hard exudates at the fovea E : grade 2 hypertensive retinopathy. Comment : Ten years after the diagnosis of type 1 diabetes about 80% of patients have detectable retinopathy. In type 2 diabetes retinopathy may be established at presentation in up to 25%. Mild to moderate non-proliferative retinopathy is characterised by microaneurysms and haemorrhages, sometimes referred to as 'dots and blots', as shown in the figure. Leakage from these can result in retinal oedema and hard exudates (not present here), but this is asymptomatic unless the fovea is involved (maculopathy). C : 7. A 60-year-old woman develops painful bloodshot eyes. Which one of the following features would make you think that she has scleritis rather than iritis? A : Photophobia B : Pain that disturbs sleep C : Visual blurring D : Keratitic precipitates E : Irregular pupil. Comment : Photophobia, visual blurring, keratitic precipitates (best seen with a slit lamp) and irregularity of the pupil are all features of iritis. Severe pain suggests scleritis and is not typical of iritis or the milder condition of episcleritis, where the inflammation usually affects just a sector of the globe of the eye. In scleritis the pain often disturbs sleep, and can literally drive people to bang their head against the wall and ask for the eye to be removed.Scleritis is often associated with systemic symptoms and a manifestation of vasculitis: check an MSU for blood / protein, full blood count, renal / liver / bone function, inflammatory markers and autoimmune / vasculitic serology (in particular rheumatoid factor and serum ANCA). B :

Page 336: Masterclass Book Part 2

MOHAMMED IS-HAG 335

8.

PLATE 59 A 25-year-old woman reports that the vision in her right eye has become blurred over the last three days and she can now see very little with it. The fundus (see image) shows: A : papilloedema. B : grade IV hypertensive retinopathy. C : normal appearance. D : optic disc swelling probably due to an acute optic neuropathy. E : central retinal artery occlusion. Comment : The optic disc is swollen. If vision were normal, this appearance could be due to papilloedema secondary to raised intracranial pressure (but not due to hypertension in the absence of any retinal haemorrhages or cotton wool spots). However, vision is impaired, making optic disc swelling secondary to an acute optic neuropathy the correct diagnosis. The commonest cause of acute optic neuropathy in a young woman is multiple sclerosis. It will be important to pursue this possibility in the history: · has this ever happened before? · have you ever had problems with your arms / legs / walking / balance before? · have you ever seen a neurologist about any problems before? Aside from fundoscopy, key issues to check on examination of the eye are: · visual acuity · presence of a central scotoma · impaired colour appreciation so that, compared with the normal eye, colours look washed out · a relative afferent pupillary defect. A full neurological examination to look for evidence of deficit elsewhere will be appropriate. The eventual level of recovery of visual acuity is difficult to predict, hence a guarded prognosis must be given. Deciding whether or not to engage the patient in discussion about multiple sclerosis and/or further tests to pursue this diagnosis requires careful consideration. D : 9. A 50-year-diabetic woman with ischaemic heart disease complains of a problem with the vision in her left eye. To get a good view of the ocular fundi for diagnostic purposes the best agent to dilate her pupils with is: A : Cyclopentolate 1% B : Tropicamide 0.5% C : Tropicamide 1% D : Phenylephrine 2.5% E : Phenylephrine 10%. Comment : All of the preparations listed can be used to dilate the pupils, but tropicamide (0.5% for children, 1% for adults) is best for diagnostic purposes, dilating the pupil for two to four hours by blocking the parasympathetic terminals in the papillary constrictor muscle. Cyclopentolate works in a similar manner but lasts for six to eight hours.Phenylephrine drops dilate the pupil by stimulating the sympathetic system. They should be used with caution both in children and in adults with ischaemic heart disease because they can induce hypertension, exacerbate angina or induce arrhythmias. C :

Page 337: Masterclass Book Part 2

MOHAMMED IS-HAG 336

10.

PLATE 60 A 60-year-old woman has felt generally unwell for several weeks. She has joint pains, sinus congestion, feels that her ears are blocked up, and her eyes have become red and painful enough to stop her sleeping properly. One of them is shown in the figure. What is the diagnosis of her ocular condition? A : Conjunctivitis B : Scleritis C : Iritis D : Uveitis E : Episcleritis. Comment : An adult who presents with red eyes and severe pain has scleritis until proven otherwise. This is much less common than iritis, and should prompt a search for underlying systemic vasculitis such as rheumatoid arthritis or Wegener's granulomatosis, the latter being most likely in this case in view of the upper respiratory tract symptoms. Scleritis is a sight-threatening condition that requires treatment with systemic immunosuppression. B : 11. A 68-year-old man is found to have glycosuria on a routine dipstick test of his urine. Subsequent investigation confirms that he has type II diabetes mellitus, for which he receives treatment with dietary advice and then a sulphonylurea. He is terrified that he is going to become blind, because he knows that this happens in diabetes. Which of the following statements about retinopathy in type II diabetes is true? A : Patients are more likely to develop proliferative disease than maculopathy. B : Retinopathy may already be established when diabetes is diagnosed. C : The most important means of preventing blindness is good control of blood pressure. D : Angiotensin-converting enzyme inhibitors (ACE)-inhibitors prevent progression of diabetic eye

disease. E : It is worse in patients with significant carotid arterial disease. Comment : About 80% of patients with type I diabetes will have retinopathy 10 years after presentation, but it is virtually unheard of at presentation. By contrast, in type II diabetes, where the time of onset is uncertain, up to 25% of patients will have retinopathy at the time of diagnosis. In type II diabetes maculopathy is more common than proliferative disease. The most important means of preventing blindness is to achieve good diabetic control. Diabetic retinopathy can sometimes be asymmetrical in the presence of significant carotid stenosis, which appears to ‘protect’ the ipsilateral eye. B :

Page 338: Masterclass Book Part 2

MOHAMMED IS-HAG 337

12.

PLATE 61 A middle-aged man suddenly looses vision in his right eye. The optic fundus shows: A : Normal appearances B : Acute central retinal artery occlusion C : Vitreous haemorrhage D : Pre-retinal haemorrhage E : Acute central retinal vein occlusion. Comment : The optic disc can just be seen through a diffuse vitreous haze caused by red cells. With a dense haemorrhage the red reflex may be entirely obscured. The differential diagnosis of sudden painless loss of vision in one eye includes: central retinal artery occlusion, central retinal vein occlusion, retinal detachment, vitreous haemorrhage (as in this case), acute optic neuropathy, migraine and giant cell arteritis. C : 13. A middle-aged Afro-Caribbean woman presents with a history of fatigue, widespread joint pains and shortness of breath with a dry cough. Her ears and sinuses have been normal, but she mentions that she has recently attended the local eye casualty department with a painful, photophobic red eye. This was successfully treated with Maxidex drops hourly during the day, and another drop to dilate the pupil at night. The most convincing diagnostic possibility is: A : Rheumatoid arthritis B : Sarcoidosis C : Wegener’s granulomatosis D : Relapsing polychondritis E : Ankylosing spondylitis. Comment : While rheumatoid arthritis may cause fatigue and joint pain, acute arthritis is a more typical presentation. Dry eye rarely presents so dramatically in one eye, and the treatment regime would be inappropriate for this. Iritis is not a common association. Sarcoidosis is more common in Afro-Caribbean patients. A chest radiograph may show typical sarcoid changes to account for her chest symptoms. The presentation of a painful red eye, especially with photophobia, and treatment with frequent and potent topical corticosteroid (Maxidex is dexamethasone 0.1%) together with pupillary dilatation, strongly suggests a slit-lamp diagnosis of acute iritis (anterior uveitis). Wegener’s granulomatosis may account for the non-specific systemic features, but uveitis is a much less common association than acute scleritis. Though topical corticosteroids might be used in scleritis, they are less appropriate, less likely to succeed, and dilating the pupil is unnecessary. Relapsing polychondritis is a contender as iritis may be associated, but there are no other specific features, such as swelling of the pinna or nasal cartilage, to suggest this particular uncommon diagnosis. Ankylosing spondylitis may be associated with acute iritis but multiple peripheral joint involvement is atypical and chest symptoms incongruous. B :

Page 339: Masterclass Book Part 2

MOHAMMED IS-HAG 338

14.

PLATE 62 74-year-old woman woke at 7.30 am and discovered she couldn't see with her right eye. Her husband drove her to A&E and you see the fundal appearance shown (see image). Which response is the best? A : You are surprised because this is a very rare retinal condition. B : You expect to demonstrate a relative pupillary afferent defect RAPD in this eye. C : In comparison with a 54-year-old, you are more likely to identify an underlying cause in this patient. D : When you examine her eye, she is complaining of a painful pulling sensation. E : The red reflex is obscured. The appearance of global flame and blot haemorrhages and cotton wool spots is characteristic of a central retinal vein occlusion. This is painless (although a painful red eye from neovascular glaucoma is a late complication). It is associated with HT, diabetes, hyperlidaemia and raised viscosity and the younger the patient the more likely you are to identify an underlying cause. RVO (central or branch vein) is the second most common retinal vascular disease after diabetic retinopathy. You can see the fundus clearly so you would not expect loss of the red reflex - this occurs with significant vitreous haemorrhage. The degree of retinal ischaemia here will produce a relative afferent pupil defect. B : 15. A 32-year-old woman visits her GP with a 9-month history of headaches. These are intermittent and last for about 1 hour. When the pain is at its worse she experiences visual blurring and nausea. The GP has been treating her for migraine, but at a return visit notices that her right pupil is larger than the left. The rest of the examination is normal except that her reflexes are all diminished. Her eye movements are normal and there is no ptosis. What is the most likely cause of the pupillary abnormality? A : Physiological anisocoria B : Holmes–Adie pupil on the right C : Right third cranial nerve palsy caused by posterior communicating artery aneurysm D : Argyll–Robertson pupil on the left E : Left sided Horner's syndrome. Comment : In this situation it is not always clear which is the abnormal pupil. The pupils should be examined in both dark and bright lighting conditions. Normal physiological anisocoria (unequal pupils) will not be altered. However, in the dark, an abnormally small pupil will fail to dilate, and in very bright light an abnormally large pupil will fail to constrict.This woman's headache is likely to be migraine, which when chronic can be associated with Horner's syndrome. However, there is no ptosis, although this can be extremely difficult to appreciate.There are no additional features to suggest a third cranial nerve palsy, so this is unlikely.Argyll–Robertson pupils are small, fail to react to light or accommodation, and are usually bilateral.A Holmes–Adie pupil is usually large and fails to react to light, with sluggish reaction to accommodation. The presence of diminished reflexes make this diagnosis the most likely. Thus her pupillary findings are unrelated to migraine. B :

Page 340: Masterclass Book Part 2

MOHAMMED IS-HAG 339

16.

PLATE 63 What does this optic fundus show? A : Proliferative diabetic retinopathy B : Background diabetic retinopathy C : Grade III hypertensive retinopathy D : Branch retinal vein occlusion E : Central retinal vein occlusion Comment : Flame and blot haemorrhages and cotton wool spots (retinal microinfarcts) are seen in a wedge distribution, appearances typical of branch retinal vein occlusion. Central retinal vein occlusion leads to these appearances throughout the retina (‘bloodstorm’). Central retinal vein occlusion results in painless, sudden loss of vision. Branch retinal vein occlusion causes visual loss when the macula is involved, but if the macula is spared the occlusion may be an incidental finding. D : 17. A sixty-year-old diabetic woman complains that her reading vision has become distorted in one eye and the image appears smaller than with the other eye. She has been diabetic for fifteen years and has maintained quite good diabetic control on diet and oral medication, with normal weight, blood pressure and plasma lipids. She is a non-smoker. What is the diagnosis? A : Proliferative diabetic retinopathy B : Diabetic maculopathy C : Cataract D : Retinal vein occlusion E : Complication of oral hypoglycaemic medication. Comment : Her symptoms are typical of oedema within the central retina, affecting the fovea. Distortion and micropsia arise when the photoreceptors within the deeper layers of the retina become irregularly spaced. Such symptoms are typical of diabetic maculopathy, but not typical of proliferative retinopathy which is characteristically asymptomatic until an acute vitreous haemorrhage occurs. Cataract occurs at an earlier age than usual in diabetes, but these symptoms are not typical. Retinal vein occlusion may also present with foveal oedema but is less likely as a cause, especially in a relatively young woman without extra vascular risk factors. Oral hypoglycaemic medication does not cause foveal damage. Although transient blurring of vision may occur when the blood sugar is first brought under control, this is more typical of type 1 diabetes, in which normalization is more rapid and more profound after using insulin for the first time. B :

Page 341: Masterclass Book Part 2

MOHAMMED IS-HAG 340

18.

PLATE 64 young woman presents with a few days of visual blurring. Her optic fundus shows: A : Background diabetic retinopathy B : Papilloedema C : Normal appearances D : Proliferative diabetic retinopathy E : Grade II hypertensive changes. Comment : The optic disc is swollen. If visual acuity is normal, then papilloedema is probably secondary to raised intracranial pressure or systemic hypertension. If vision is reduced, then an acute optic neuropathy is likely. It would be important to test carefully for signs of optic nerve dysfunction: reduced acuity, central scotoma, impaired colour appreciation (compared with the normal eye, colours look ‘washed out’), relative afferent pupillary defect. B : 19. A 38-year-old woman presents with rapid loss of central vision in the right eye, which is painful when she moves it. Which one of the following physical signs would NOT support a diagnosis of optic neuritis? A : Cotton wool spots B : Reduced colour perception C : Central scotoma D : Relative afferent pupillary defect E : Swollen optic disc. Comment : This presentation would be typical of optic neuritis. Physical signs to support this diagnosis include reduced visual acuity, reduced colour perception, central or centrocaecal scotoma, relative afferent pupillary defect and a swollen optic disc. Cotton wool spots are not a feature. The differential diagnosis in an older patient is acute anterior ischaemic optic neuropathy, when the visual field defect is usually altitudinal. If visual loss or pain is severe, IV methylprednisolone (1g daily for 3 days) should be given, but oral steroids should not be used because they are associated with an increased risk of recurrent episodes. A : 20. A 38-year-old woman presents with a 24 to 48-hour history of a red and aching eye with photophobia. Vision in that eye is blurred, but acuity is not severely affected. On examination there is sediment at the bottom of the anterior chamber (hypopyon). What is the diagnosis? A : Iritis B : Episcleritis C : Scleritis D : Keratitis E : Conjunctivitis.

Page 342: Masterclass Book Part 2

MOHAMMED IS-HAG 341

Comment : Iritis typically presents with symptoms of a red, aching eye with photophobia, which tends to worsen over hours to a few days. Vision may be blurred, but acuity is not severely affected. The pupil tends to be small and may be irregular because the iris has adhered to the anterior lens, in which case it festoons on dilatation. Other features of iritis include: 1. ‘flare’– leak of protein from inflamed vessels into the anterior chamber making the fluid in the aqueous look hazy; 2. keratitic precipitates – deposits of inflammatory cells on the inner surface of the cornea; 3. hypopyon – characteristic of Behçet’s disease or endophthalmitis. A : 21.

PLATE 65 What does this optic fundus show? A : Normal appearance B : Optic atrophy C : Glaucoma D : Background diabetic retinopathy E : Papilloedema. Comment : The peripheral rim of the optic disc is narrow and pale, the central cup is wide - features typical of chronic glaucoma. C : 22. A 28-year-old woman reports that vision in her right eye, which aches, has become blurred over a few days. The most likely diagnosis is: A : Foveal oedema B : Ischaemic optic neuritis C : Optic nerve compression D : Demyelinating optic neuritis E : Pre-retinal haemorrhage. Comment : To confirm the clinical suspicion of an optic nerve lesion, look for the following: · Reduced visual acuity (to variable degree) · Central scotoma, partial to colour or total · Impaired colour appreciation, so that colours look ‘washed out’ compared with the normal eye · Relative afferent papillary defect · Swollen optic nerve head (in the acute stage, and not if inflammation is ‘retrobulbar’) D : .

Page 343: Masterclass Book Part 2

MOHAMMED IS-HAG 342

23.

PLATE 66 What does this optic fundus show? A : Proliferative diabetic retinopathy B : Background diabetic retinopathy C : Grade III hypertensive retinopathy D : Branch retinal vein occlusion E : Central retinal vein occlusion. Comment : The appearances are of background diabetic retinopathy with hard exudates in a circular or circinate pattern at the fovea. Blot haemorrhages are also seen, but there are no new vessels to indicate diabetic proliferative retinopathy. The fact that the fovea is involved in this case will mean that the patient’s vision is affected and the eye can be described as showing ‘maculopathy’. Diabetic maculopathy as seen here will be treated by focal or ‘grid’ laser coagulation, the primary goal being to seal leaking areas close to the fovea. B :

Page 344: Masterclass Book Part 2

MOHAMMED IS-HAG 343

1. A 50-year-old woman has increasing frequency of migraine attacks. You decide to start some prophylactic therapy. Which one of the following drugs would not be appropriate for prophylaxis against migraine? A : Rizatriptan B : Sodium valproate C : Propranolol D : Amitriptyline E : Pizotifen. Comment : Rizatriptan is not used as prophylaxis against migraine. It is a 5HT1 agonist and may be useful in the treatment of acute migraine attacks. It is available as either tablets or ‘melt wafers’, which dissolve on the tongue. Propranolol and pizotifen are licensed for use as prophylaxis against migraine. Pizotifen may cause drowsiness and weight gain. Although sodium valproate and amitriptyline are unlicensed for migraine prophylaxis, they may be effective in some patients. A : 2. Clozapine is an atypical antipsychotic drug that appears to have fewer problems with adverse effects than older antipsychotics. The relative safety of clozapine stems from one of the following properties: A : Low affinity for dopamine D2 receptors B : Low affinity for 5HT receptors C : Increase in prolactin levels D : Does not cause tachycardia E : No effect on white cell counts. Comment : Clozapine appears to have fewer extrapyramidal adverse effects than older antipsychotics. This has been attributed to its relatively low affinity for D2 dopamine receptors. Unlike older antipsychotics, clozapine has relatively high affinity for 5HT receptors, and also has little effect on prolactin levels. Myocarditis and cardiomyopathy have been reported with atypical antipsychotics, and persistent tachycardia is an early warning sign. Agranulocytosis is a well-recognized complication of clozapine, and patients should be supervised under the Clozaril Patient Monitoring Service. A : 3. A 58-year-old gentleman with elevated cholesterol has failed to reach a desired cholesterol level on statin treatment. You decide to commence him on ezetimibe. Which of the following is true concerning ezetimibe? A : Prescription with statin treatment is contraindicated. B : Decreased absorption of fat soluble vitamins is an unwanted effect. C : Its main action is to prevent cholesterol synthesis by the liver. D : It causes an elevation in plasma triglyceride concentrations. E : It causes a reduction in low-density lipoprotein (LDL)-cholesterol of approximately 20%. Comment : Ezetimibe is the first of a novel class of drugs for the treatment of hyperlipidaemia whose main action is to specifically prevent cholesterol absorption from the small intestine. Typically it reduces LDL-cholesterol by approximately 20%, triglycerides by up to 5% and raises HDL-cholesterol by approximately 5%. It does not inhibit the absorption of fat-soluble vitamins unlike the anion-exchange resins (e.g. colestyramine). Ezetimibe can be safely co-administered with statins and is currently licensed for use in combination with a statin in patients who fail to reach desired lipid profiles or as monotherapy in patients intolerant to a statin. There is no increased risk of myopathy with ezetimibe prescription. E : 4. A GP refers an elderly woman to your day hospital for rehabilitation. She has been reluctant to go out for some time. She walks slowly with a stooped posture. She fell in her garden 4 months previously and has been terrified of falling ever since. There are no features of Parkinsonism on examination. Which one of these options is correct? A : Roughly 20% of people aged over 65 years will fall each year B : Falls tend to occur when carrying out hazardous, ambitious tasks C : Fear of falling can occur independently of previous falls / fall related injuries D : The psychological consequences of falling are independent of physical risk factors for falling E : Physical treatment modalities do not have a role in fall-related anxiety management. Comment : Around one third of people over the age of 65 will fall every year, most often during everyday essential activities.The psychological consequences of falling are often overlooked and include depression, generalised anxiety and fear of falling. Fear of falling can be present even when there have been no falls-related injuries, and can itself be a predictor of functional decline, loss of independence and future falls. In the short term worrying about falls leads to distraction, increased muscle tension and poor posture (as in this case). In the long term the associated reduction in activity leads to reduced body strength, which will increase the risk of falling when doing everyday activities.

Page 345: Masterclass Book Part 2

MOHAMMED IS-HAG 344

Interventions to manage fear of falling should be integrated into a broad, multidisciplinary approach, the aim being to manage fear rather than eradicate it. Measures include education to understand risk factors for falling, and reduction of the risk using exercise programmes, environmental assessment and relaxation and breathing techniques. Specific cognitive-behavioural techniques include practised methods of dealing with negative thoughts, and having reminder lists of how to 'walk well'. C : 5. A 26-year-old woman consults her general practitioner in the 12th week of her pregnancy. She complains of fever and dysuria. There is no other significant history but direct questioning reveals a self-limiting rash in the past after taking penicillin. Her GP sends off a sample of urine that shows a significant growth of Gram negative bacilli. The organism is sensitive to the antibiotics listed below. Her GP is uncertain as to the best agent in the context of the pregnancy and contacts you by telephone for advice. Which of the following would be the best choice agent in this situation? A : Ciprofloxacin B : Gentamicin C : Cefaclor D : Trimethoprim E : Co-amoxiclav. Comment : Ciprofloxacin has been associated with arthropathy and cartilage erosions in young animals.Gentamicin needs to be given parenterally and is not suitable for outpatient use. There is also a risk of fetal nephrotoxicity and ototoxicity. Trimethoprim is a folate antagonist and can increases the risk of neural tube defects. Co-amoxiclav is a combination of amoxycillin and clavulanic acid, and although there is no definite risk of teratogenicity, it should not be used unless absolutely necessary. Furthermore, this patient has previously had a rash with penicillin. Although there is a small risk of cross-allergy (10%) with cephalosporins, cefaclor would be a reasonable choice given that the penicillin allergy was relatively mild and not a full-blown anaphylactic reaction. C : 6. Acupuncture is playing an increasing role in pain management. Which structures are involved in mediating the effects of acupuncture? A : Dorsal horn of spinal cord B : Hypothalamus C : Hypothalamus and A beta and A gamma nerve fibres D : Cerebral cortex E : Cerebral cortex and A beta nerve fibres. Comment : The A beta nerve fibres are the path for fast transmission of sensation. Acupuncture also has a central effect. E : 7. A 21-year-old male reports having taken an overdose of 'some tablets'. For which of the following would you not use activated charcoal within the first hour? A : Paracetamol B : Aspirin C : Diazepam D : Atenolol E : Lithium. Comment : Activated charcoal provides a large area for adsorption of many drugs ingested as an overdose. Examples of agents not adsorbed by activated charcoal include: · metals (lithium, iron) · hydrocarbons and solvents · alcohols · acids · alkalis. E : 8. An 82-year-old man is admitted after a syncopal episode. His pulse rate is 40/min and ECG confirms complete (3rd degree) heart block. His pulse slows to 24/min and he feels very faint. Whilst arrangements are being made for temporary pacing you give: A : Adrenaline 0.5 mg intravenous bolus B : Isoprenaline 50 mg intravenous bolus C : Atropine 0.5 mg intravenous bolus D : Atropine 5 mg intravenous bolus E : Isoprenaline 500 mg intravenous bolus. Comment : The options to be considered, prior to temporary transvenous pacing, in this context are: 1. Atropine 0.5-1.0 mg intravenous bolus, repeated as required. 2. Isoprenaline, intravenous infusion at 2-10 microg/min. 3. External cardiac pacing. C :

Page 346: Masterclass Book Part 2

MOHAMMED IS-HAG 345

9. You are treating a 72-year-old man with moderate peripheral vascular disease. He exercises regularly but finds that his walking distance is diminishing due to pain. Which drug might help improve pain-free walking distance? A : Naftidrofuryl B : Cinnarizine C : Inositol nicotinate D : Simvastatin E : Diltiazem. Vasoactive drugs have limited benefit in treating intermittent claudication. There is modest evidence for the use of drugs such as naftidrofuryl and pentoxifylline, but little benefit from cinnarizine or inositol nicotinate. Simvastatin may be prescribed for patients with peripheral vascular disease who have elevated cholesterol levels, but there is no data on improvements in walking distance. A : 10. A general practitioner seeks your opinion on a patient who had developed abnormal thyroid function tests after being started on amiodarone recently. Which one of the following features, in conjunction with clinical symptoms and signs, is helpful in diagnosing overt hypothyroidism? A : Increase in thyroid-stimulating hormone (TSH) up to 20mU/L B : Decrease in T3 C : Elevated free T4 and T3 D : T4 at upper end of or just above normal range E : Low free T4, and low T3. Comment : Amiodarone is an iodine rich structure which resembles T4. Daily dose 200mg generates 7mg free iodine (WHO optimal intake 0.15-0.3mg /day) This is a high iodine load for body which blocks further thyroid iodide uptake and hormone synthesis. It also blocks conversion of T4 to T3 and affects pituitary thyroid axis. The following changes in thyroid function tests occur within 3 months of starting amiodarone and are not indicative of thyroid disease: increase in TSH up to 20mU/L, increase in T4 to upper limit of normal range, and decreased T3 levels. Diagnosis of hypothyrodism should be based on clinical assessment, together with the following features: high TSH - > 20mU/l, low free T4, low T3. Treatment is with thyroxine and we should aim for free T4 levels close to upper range. E : 11. A 58-year-old with carcinoma of the ovary has been taking morphine sulphate continus (MST) 200mg b.d. for 3 months. She presents with a 2-day history of twitching and drowsiness. Examination reveals her to be fluid overloaded and have pin point pupils. What is the most likely cause of her symptoms? A : Acute renal failure B : Morphine overdose C : Renal failure leading to accumulation of morphine D : Brain secondaries E : Right sided cardiac failure. Comment : Patients with relapsed ovarian cancer may develop an obstructive nephropathy due to pelvic recurrence. If they are on morphine they may get accumulation of this drug and signs of opiate toxicity superimposed on the signs of renal failure. C : 12. A 58-year-old patient with metastatic carcinoma of the breast has good pain control on MST 180mg bd. She is admitted with increasing weakness and has difficulty swallowing her tablets. It is therefore decided to convert her to a 24-hour diamorphine syringe driver. The correct dose of diamorphine is: A : 30mg with 5mg prn. B : 60mg with 10mg prn. C : 60mg with 5mg prn. D : 120mg with 10mg prn. E : 120mg with 20mg prn. Comment : The dose of diamorphine should be 1/3 of the total 24-hour dose of morphine. The prn dose should be 1/6 of the 24 hour dose of diamorphine. E : 13. In the management of cellulitis, which of the following statements is correct? A : Cellulitis is associated with less than 0.5% mortality in hospitalised patients. B : Routine needle aspiration of of the afftec area is helpful in determining the bacterial cause. C : The commonest cause of cellulitis in adults is Staphylococcus aureus. D : Oral flucloxacillin 50mg qds is an appropriate first line regime for simple uncomplicated

cellulitis. E : Prophylactic oral erythromycin may be useful in preventing recurrent episodes of cellulitis. Comment : The commonest cause of cellulitis in adults is Streptococcus pyogenes although the infection may also involve Staphylococcus aureus. As such, antibiotic cover should reflect this. Oral phenoxymethylpenicillin (or benzylpenicillin intravenously), with replacement using flucloxacillin where

Page 347: Masterclass Book Part 2

MOHAMMED IS-HAG 346

there is no response, is suitable for 'blind' treatment of cellulitis. However, the BNF and Public Health Laboratory Service recommend that a combination of phenoxymethylpenicillin and flucloxacillin be used for treating cellulitis. Some experts advocate high dose flucloxacillin (1-2g intravenously qds) as an alternative first-line treatment as this should be effective against both streptococci and staphylococci.Cellulitis can lead to serious early complications including septicaemia. Death occurs in up to 5% of hospitalised patients. Although there is weak and inconclusive evidence, a randomised trial has suggested that erythromycin may be useful in preventing recurrence of cellulitis in those with a history of 2 or more previous episodes. Routine needle aspiratin of the affected area rarely helps to determine the bacterial cause in cellulitis. E : 14. A 58-year-old woman with a past medical history of hypertension, congestive cardiac failure, osteoarthritis, urinary tract infections and depression presents with general malaise. Blood tests show that she has acute renal failure, with serum creatinine 700 micromol/l. Ultrasound examination reveals two normal-sized kidneys and she proceeds to renal biopsy that demonstrates acute interstitial nephritis. There have been many recent changes in her medications. Which of the following is most likely to be responsible for this condition? A : Candesartan B : Coproxamol C : Enalapril D : Ibuprofen E : Trimethoprim Comment : The drugs that most commonly cause acute interstitial nephritis are penicillins, non-steroidal anti-inflammatory drugs and thiazide diuretics.When the circulation is compromised, non-steroidal anti-inflammatory agents, angiotensin converting enzyme inhibitors (such as enalapril) and angiotensin-II receptor antagonists (such as candesartan) can have adverse haemodynamic effects on the kidney leading to a reduction in GFR, but ACE inhibitors and AT2 blockers are not associated with interstitial nephritis. D : 15. A 54-year-old man with renal cell carcinoma and bone metastases is admitted with confusion and constipation. He is on MST 100mg bd and a non-steroidal anti-inflammatory drug. How would you manage him? A : Avoid intrusive investigations and treat symptoms as they arise B : Stop opiates which may be the cause of his symptoms C : Reduce opiates D : Change opiates E : Check serum calcium. Comment : Hypercalcaemia may well cause these symptoms and should always be checked unless a patient is dying. Another possible metabolic cause of this presentation is renal failure. Opiates rarely cause confusion in the absence of renal failure or overdose for other reasons. E : 16. A 68-year-old woman with ascites secondary to carcinoma of the ovary is complaining of early satiety and intermittent vomiting. The antiemetic of choice is: A : Cyclizine B : Haloperidol C : Ondanestron D : Metoclopramide E : Dexamethasone. Comment : Metoclopramide as a prokinetic agent will increase the rate of transit of food through the gastrointestinal tract and alleviate her symptoms. She should also be advised to eat 'little and often'. D : 17. When considering the use of aromatase inhibitors in the medical management of breast cancer, which statement is true? A : Aromatase inhibitors are reserved for those with oestrogen receptor negative tumours. B : Aromatase is responsible for the peripheral conversion of androgens to oestrogen. C : Anastrozole and letrozole are non-selective inhibitors of aromatase. D : Aromatase inhibitors are contra-indicated in patients with venous thromboembolism. E : Musculoskeletal adverse effects are uncommon with aromatase inhibitors. Comment : In the UK, about two-thirds of women with breast cancer have tumours which are oestrogen and/or progesterone receptor positive. These tumours appear to respond to agents such as oestrogen antagonists (tamoxifen) and drugs that lower oestrogen concentrations e.g. aromatase inhibitors. Circulating oestrogens in postmenopausal women are derived almost exclusively from peripheral conversion of androgens by aromatase (a cytochrome p450 enzyme found in the adrenal glands and subcutaneous fat). Aminogluthetimide is a non-selective aromatase inhibitor, while anastrozole and letrozole are selective inhibitors which do not affect mineralocorticoid and glucocorticoid synthesis. Aromatase inhibitors appear to be less thrombogenic than tamoxifen and may be a better option in patients with a history of venous thromboembolism. However, the lowering of oestrogen levels increases the risk of bone fractures. B :

Page 348: Masterclass Book Part 2

MOHAMMED IS-HAG 347

18. You see a 70-year-old lady who has severe Parkinson’s disease and is on co-careldopa and apomorphine. She complains of nausea and vomiting due to her medication. Which one of the following drugs would be prescribe for these symptoms? A : Domperidone B : Metoclopramide C : Prochlorperazine D : Entacapone E : Betahistine. Drugs such as apomorphine and bromocriptine cause vomiting through peripheral stimulation of the chemoreceptor trigger zone. Worsening of Parkinson’s disease may result from the use of dopamine antagonists; however, domperidone is much less likely to cross the blood–brain barrier and is therefore the preferred agent in this case. Entacapone is a catechol-O-methyltransferase (COMT) inhibitor which increases levodopa levels, thus worsening nausea and vomiting. Betahistine is used in vertigo. A : 19. You see a woman in late pregnancy who has just been diagnosed with thyrotoxicosis. She is planning to breast-feed her baby after delivery. Which treatment would you recommend for her? A : Carbimazole B : Blocking dose of carbimazole with added thyroxine C : Potassium perchlorate D : Propylthiouracil E : Lugol’s iodine. Comment : Anti-thyroid drugs can cross the placenta and breast milk, thus causing hypothyroidism in the child. The carbimazole-blocking regimen is worse in this respect, as the carbimazole crosses the placenta, but the thyroxine does not. Potassium perchlorate is no longer used in the UK; Lugol’s iodine may occasionally be prescribed for patients undergoing thyroid surgery, but causes goitre in infants. Propythiouracil is more highly protein bound and is ionized at pH 7.4, thus making it less likely to cross the placenta or breast milk. D : 20. An 18-year-old girl is admitted having deliberately taken a large overdose of her father's atenolol tablets. Which of the following statements is false? A : Dizziness is a common feature of beta-blocker overdose. B : A temporary pacing wire may be required. C : Atropine should not be given due to the risk of causing tachycardia. D : Glucagon is a useful therapy in the management of beta-blocker overdose. E : Paracetamol levels should be measured. Comment : Beta blocker overdose may cause dizziness, hypotension, syncope and heart failure. Bradycardia is a common feature of significant overdose and should be treated by the administration of atropine. Intravenous glucagon may also be given, particularly in patients with haemodynamic compromise. Temporary cardiac pacing may be necessary in patients unresponsive to drug therapy. Paracetamol concentrations should be measured as these may also have been taken at the time of the overdose and may require specific additional treatment. There is no evidence that salicylate concentrations should be measured unless there is a history of ingestions, signs of salicylate toxicity, in patients with reduced Glasgow coma scale (GCS) and patients unable to provide a reliable history. C : 21. Which one of the following antiemetic drugs used in palliative care is not associated with constipation A : Ondansetron B : Hysocine C : Cyclizine D : Haloperidol E : Levomepromazine. Comment : Hyoscine, cyclizine and levomepromazine have significant antimuscarinic effects which reduce colonic peristalsis. All 5HT3 antagonists cause constipation by inhibiting large bowel transit (by blocking cholinergic mechanisms). Haloperidol is a dopamine antagonist and is not commonly associated with constipation. D : 22. During the development of a drug, it is given to patients with disease to determine optimal dosing and monitor for side effects. Which phase of study is this? A : Phase I B : Phase II C : Phase III D : Phase IV E : Phase V Comment : Phase I trials test drug handling by normal volunteers. Phase II trials test drug handling and effects in patients with disease. Phase III trials formally assess the effectiveness and safety of the drug in people with disease, the gold-standard phase III study being a double-blind randomised controlled trial. Phase IV trials consist of post-marketing surveillance. B :

Page 349: Masterclass Book Part 2

MOHAMMED IS-HAG 348

23. An 18-year-old girl presents with a paracetamol overdose. Which of the following factors would not be considered to increase the toxicity of the overdose? A : Alcohol consumption of 10 units per week B : BMI of 14 C : Recent glandular fever D : History of cystic fibrosis E : Use of phenytoin. Comment : Paracetamol ingestion of greater than 150mg/kg in 'normal risk' and greater than 75mg/kg in 'high risk' individuals is associated with hepatotoxicity if left untreated. Alcohol consumption of greater than 14 units per week in females and 21 units per week in males is associated with increased toxicity of paracetamol, and in view of the fact she drinks only 10 units per week this is the correct answer. Depletion of glutathione from the liver associated with eating disorders, malnourishment, cystic fibrosis and HIV is associated with increased toxicity. Additionally, viral infections such as EBV and CMV reduce glutathione levels in children. The use of enzyme inducers such rifampicin, phenytoin and carbamazepine are associated with increased paracetamol toxicity. A : 24. A 75-year-old woman with metastatic carcinoma of the colon is admitted semi-conscious and dying. Her symptoms had been previously well controlled on oxycodone SR 80mg bd. What would you do about analgesia? A : Nothing at present as she is semi-conscious and not obviously in pain. B : Chart prn oxycodone orally C : Change to im morphine D : Chart prn paracetamol pr E : Start a syringe driver with diamorphine. Comment : A good strategy is to anticipate problems and avoid the patient developing pain. A syringe driver with the correct dose of diamorphine and 1/6 dose as breakthrough medication is correct. E : 25. A 35-year-woman presents 6 hours post a deliberate overdose of paracetamol. The paracetamol level at 6 hours is above the treatment line. Thirty minutes after starting an infusion of N-acetyl cysteine (NAC) she becomes flushed and hypotensive with a blood pressure of 80/55 mmHg. The infusion is stopped immediately and 500ml IV 0.9% saline administered over 30 minutes. Which of the following is the correct ongoing management? A : IV chlorpheniramine and restart NAC infusion at lowest rate once symptoms resolved B : IV chlorpromazine and restart NAC infusion at lowest rate once symptoms resolved C : IV chlorpheniramine and give 2.5g of oral methionine D : IV chlorpromazine and give 2.5g of oral methionine E : Withhold treatment and recheck paracetamol level at 12 hours Comment : Reactions to NAC are well recognized and are not related to hypersensitivity. NAC can almost always be safely restarted and total dose safely administered after symptomatic treatment. Oral methionine may be an alternative but is definitely second line. IV chlorpromazine is now never given and would make hypotension worse. Withholding treatment and waiting more than 12 hours would expose patient to risk of liver failure. A : 26. Which statement is true with regards to the safety profile of metformin in diabetes mellitus? A : Gastro-intestinal adverse effects are uncommon. B : There is a high incidence of weight gain C : Metformin should be stopped temporarily in patients undergoing iodinated contrast media

studies. D : Hypoglycaemia is a common problem. E : It can safely be used in patients with acute myocardial infarction. Comment : Metformin combats insulin resistance and has a lower incidence of weight gain and hypoglycaemia, although GI adverse effects are common. In view of the risk of lactic acidosis, avoid metformin if creatinine >150 micromols. Similarly, it should be withdraw temporarily in the following instances: Acute tissue hypoxia – MI, sepsis, and after ionic contrast media radiological studies. C : 27. You see a patient in the outpatient clinic with multiple myeloma whose recent blood tests have shown a serum potassium of 5.7mmol. His recent drug history includes the use of melphalan, interferon, and pamidronate, and he is clinically well, with no ECG changes. Which one of the following oral agents would you prescribe for treatment of hyperkalaemia in this patient? A : Calcium carbonate B : Calcium polystyrene sulphonate (Resonium) C : Sodium polystyrene sulphonate (Resonium) D : Bendrofluazide E : Salbutamol.

Page 350: Masterclass Book Part 2

MOHAMMED IS-HAG 349

Comment : Ion-exchange resins may be used to remove excess potassium in mild to moderate hyperkalaemia. They can either be calcium or sodium based, with the site of ion exchange taking place mainly in the large intestine, thus allowing either oral or rectal administration. Calcium based resins may provoke hypercalcaemia in patients with multiple myeloma, or metastatic cancer and are best avoided in such instances (especially as this patient has recently received pamidronate, which suggests that hypercalcaemia has been a problem). Salbutamol can be given by nebuliser for more severe cases of hyperkalaemia, especially in patients who are fluid overloaded. C : 28. A 73-year-old man presents to A&E with drowsiness and confusion. He is noted to be tachycardic and tachypnoeic. His pulse oximeter reading is 90% on room air and he is not cyanosed. Apparently his wife had been admitted with similar symptoms earlier in the week. Which one of the following is most likely? A : Paracetamol overdose B : Salicylate overdose C : Carbon monoxide poisoning D : Cerebrovascular accident E : Pneumonia. Comment : This man has carbon monoxide (CO) poisoning. Pulse oximeters cannot distinguish between COHb and HbO2, therefore it is essential to take arterial blood gases and – to make the specific diagnosis – measure the level of CO. It is important to think about prevention: CO alarms are cheap and readily available. C : 29. A 60-year-old woman is admitted feeling generally unwell. Her serum potassium is found to be elevated at 7.0 mmol/l. Which of the following drugs is the LEAST likely to have contributed to her hyperkalaemia? A : Lisinopril B : Bendrofluazide C : Losartan D : Spironolactone E : Slow K. Comment : Bendrofluazide tends to cause hypokalaemia. The other drugs listed may cause hyperkalaemia, although this is not usually clinically significant when used alone at therapeutic doses. The development of significant drug-induced hyperkalaemia is more likely when more than one of these agents is used in combination, or if the patient has co-existing renal impairment. B : 30. A cachectic 87-year-old man is admitted with acute urinary retention and is found to have a creatinine of 520 micro mol/l. He has been on a number of medications. Which of the following is least likely to be nephrotoxic in this situation? A : Ibuprofen B : Ramipril C : Allopurinol D : Lansoprazole E : Sulfasalazine. Comment : As he has muscle wasting, a creatinine this high probably indicates severe renal impairment. This should improve with catheterisation and a good fluid input, but it is essential to stop all potentially nephrotoxic drugs. Do not rely on your memory -always check with the BNF as continuing a nephrotoxic drug at this stage may lead to irreversible renal failure. Of this list, only lansoprazole should be continued until his renal function has improved. D : 31. A 20-year-old woman, previously fit, well and with no significant past medical history, presents 10 weeks into pregnancy with a painful swollen calf. Ultrasound examination confirms that she has a deep venous thrombosis. How should this be managed up to the time of delivery? A : Tubigrip stocking; avoid anticoagulation. B : Initiate and then continue treatment with warfarin until delivery. C : Initiate and then continue treatment with heparin until delivery. D : Initiate treatment with heparin; load with and convert to warfarin, continued until delivery. E : Initiate and then continue treatment with heparin and warfarin until delivery. Comment : During a critical window of 6-9 weeks of pregnancy warfarin causes (warfarin embryopathy) manifest, most obviously as hypoplasia of the nose and limbs. After this period warfarin is associated with neurological damage – mental retardation, microcephaly, optic atrophy and blindness – that may be haemorrhagic in origin and hence reduced by tight anticoagulant control. Warfarin is also associated with increased risk of abruptio placentae in later pregnancy and increased bleeding in mother and fetus if used at term. Most physicians would manage this woman with low molecular weight heparin throughout her pregnancy, although this is not without problems, in particular of maternal osteoporosis. Those that would use warfarin would not continue its use to term but switch to heparin. C :

Page 351: Masterclass Book Part 2

MOHAMMED IS-HAG 350

32. In a healthy volunteer study, the diuretic response to intravenous doses of loop diuretics A, B and C were compared. A 1mg dose of diuretic A produced a similar diuresis to 40 mg of diuretic B and 50mcg of diuretic C. It was found that maximal doses of A produced a similar diuresis to maximal doses of B, but the response obtained with maximal doses of C was considerably lower than that of A or B. Which of the following statements is correct in relation to the action of A, B and C: A : Drugs A and B are of similar potency. B : Drug C is more potent than drug A but of lower efficacy. C : Drug C is of lower potency than drugs A and B but of greater efficacy. D : Drug A is of greater efficacy than drugs B and C. E : No conclusion can be drawn regarding the relative efficacy and potency of the three drugs. Comment : The potency of a drug relates to the amount of drug needed to produce a given effect. An equivalent diuresis to that seen with drugs A and B is produced with a considerably smaller dose of C. It is therefore more potent. Efficacy relates to the maximal response that can be produced by the drug when taken to high levels. Despite increasing the dose, drug C is unable to produce an equivalent maximal diuresis to that obtained with drugs A or B, it is thus of lower efficacy. The potency of a drug is not often of importance but can become significant if the drug has low solubility and has to be packaged or delivered in such a way that space is limited. Examples might include metered dose aerosols or depot injections. B : 33. A 74-year-old woman presents with breathlessness. She is a small woman (55 kg) with a chest infection. She is not very unwell, but is in atrial fibrillation at a rate of 170/min. Her electrolytes are normal (K 4.2 mmol/l). As well as treating her pneumonia, you decide to digitalize by prescribing: A : Digoxin 0.25 mg orally once daily B : Digoxin 1.0 mg orally over 24 hours in divided doses C : Digoxin 1.0 mg intravenously over 20 min D : Digoxin 0.125 mg orally once daily E : Digoxin 0.25 mg orally three times daily for one week, then twice daily for one week, then once

daily thereafter. Comment : The options for treatment of atrial fibrillation are: 1. DC cardioversion if the patient is compromised haemodynamically or has ischaemic cardiac pain 2. Digoxin - 1.0-1.5 mg orally in divided doses over 24 hours, but can be given intavenously in emergency (0.25-0.5 mg over 10-20 min, repeated after four to eight hours to total intravenous loading dose of 0.5-1.0 mg) 3. 'Medical cardioversion' with amiodarone or flecainide In this clinical context it is likely that the atrial fibrillation (if new) will revert to sinus rhythm as the woman recovers from her pneumonia and most physicians would digitalize in preference to the other options described. B : 34. A 38-year-old man presents with acute renal failure, with serum creatinine 988 micromol/l. A house physician performs arterial blood gas analysis (breathing air) and finds pH 7.12, pO2 12.8 kPa, pCO2 3.2 kPa, BE -12 mmol/l. He asks you what a base excess of –12 means. You reply: A : It means that the serum bicarbonate concentration is 12 mmol/l. B : It means that the serum bicarbonate concentration is 12 mmol/l below normal. C : It means that the pH is 0.12 units below the lower limit of the normal range for the machine being used. D : An algorithm is used to predict what pH would arise in normal blood in the presence of the pCO2 actually measured, the base excess being the amount of acid that would have to be added or removed to obtain the pH actually measured. E : An algorithm is used to predict what pH would arise in normal blood in the presence of the pCO2 actually measured, the base excess being the amount of base that would have to be added or removed to obtain the pH actually measured. Comment : The base excess is a figure calculated by many blood gas machines to aid interpretation of data. The principles of the calculation are as follows: predict the pH that would arise in normal blood in the presence of the pCO2 actually measured; then calculate the amount of acid or base that would have to be added to the blood to change the calculated pH into the pH as actually measured. This value is the base deficit or excess, in mmol/l, which quantifies the metabolic component of acid-base disturbance. Renal failure causes metabolic acidosis with compensatory respiratory alkalosis. In this man the predicted pH (on the basis of measured pCO2 3.2 kPa) would be alkalotic, and acid would have to be added to the blood to change this to the acidotic value actually measured (pH 7.12). This is expressed as a NEGATIVE base excess. E :

Page 352: Masterclass Book Part 2

MOHAMMED IS-HAG 351

35. A man is brought to the accident and emergency department by ambulance. He is unconscious (GCS 5) with pin-point pupils and a slow respiratory rate. Immediate specific treatment should be: A : Naloxone (0.4 mg) intravenously, repeated if no effect B : N-acetyl cysteine (150 mg/kg over 15 min) intravenously, then 50 mg/kg over 4 hours, then 100

mg/kg over 16 hours C : Dextrose (50 ml of 50% solution) intravenously, repeated if no effect D : Naloxone (4 mg) intravenously, repeated if no effect E : Insert stomach tube (after securing airway) and give activated charcoal. Comment : The working diagnosis must be opioid overdose, the treatment for which is intravenous naloxone (0.4 mg), repeated up to a total dose of 2 mg depending on clinical response. The half-life of naloxone is shorter than that of opioids, hence if this man wakes up it can be anticipated that he will 're-narcose'. A naloxone infusion may be necessary. A : 36. A 56-year-old man is admitted to hospital with crushing central chest pain. He is known to have had a myocardial infarction (MI) 3 months ago. Despite treatment with simvastatin in addition to other usual cardiac medication, his ECG shows that he has had a second MI. Which of the following statements is LEAST accurate? A : The overall effect of a drug depends on the net effect on polymorphisms within its metabolic

pathways and genetic variation in target cells. B : There is an interaction between apo E genotype and lipid response to statin therapy. C : When a genetic variation affects more than 1% of the population it is termed a polymorphism. D : Lack of concordance with treatment is a possible explanation. E : Data from RCTs show that all MI patients should receive a statin. Comment : A, B and C are correct statements about pharmacogenetics, and D is also true. E looks sensible, but remember that RCTs show what happens in a group of patients in comparison with a group of controls. In any trial showing the survival advantage on a drug, some individuals will have a greater than average response, some less than average and some will be harmed. At present we cannot easily distinguish between these groups, so all patients are given a statin. E : 37. A 17-year-old male fails to breathe spontaneously after an operation. Talking to his family, his sister has previously had similar problems. Which of the following drugs could have caused this problem? A : Thiopentone B : Atracurium C : Suxamethonium D : Cisatracurium E : Halothane. Comment : Suxamethonium is a depolarising neuromuscular blocking agent that is metabolised by plasma pseudocholinesterases. Approximately one in 2500 individuals have deficiency of this enzyme, resulting in prolonged neuromuscular blockade if given suxamethonium. Management of these patients is by prolonged ventilation until the action of the drug wears off. Relatives of affected patients should be screened. C : 38. A patient is taking morphine sulphate continuous (MST) 240 mg b.d. for carcinoma of the prostate with painful bone metastases. What dose of oramorph for breakthrough pain would you prescribe? A : 40 mg B : 10 mg C : 100 mg D : 80 mg E : 60 mg Comment : The breakthrough dose of short acting morphine should be 1/6th of the total 24-hour dose. There should be no time limit on the prescription. D : 39. A 45-year-old man is taking long-term theophylline for asthma. One evening, he is admitted to the Accident and Emergency department with convulsions. You suspect theophylline toxicity. Which one of the following statements is true? A : His convulsions should not be treated until a theophylline level is available. B : Theophylline toxicity may have been precipitated by the concomitant prescription of phenytoin. C : Theophylline toxicity may have been precipitated by the concomitant prescription of

erythromycin. D : Theophylline toxicity only occurs in the elderly. E : Theophylline is an example of a drug with a wide therapeutic range (therapeutic index). Comment : Convulsions should be treated immediately in the usual way, without waiting for confirmation of the theophylline level.Theophylline is metabolised by the CYP450 enzymes in the liver. Erythromycin inhibits CYP450 enzymes and increases the half-life of theophylline and hence plasma

Page 353: Masterclass Book Part 2

MOHAMMED IS-HAG 352

theophylline concentrations, which may lead to toxicity. By contrast, phenytoin induces CYP450 enzymes, which will decrease the half-life of theophylline and may lead to inadequate therapeutic levels.Theophylline toxicity is more likely in the elderly due to age-related reduction in the rate of its metabolism, but it can occur at any age.Theophylline is an example of a drug with aNARROW therapeutic range. It is recommended that plasma theophylline levels be maintained between 10 and 20 mg/l. C : 40. In a patient with normal renal function, a dose of 40mg of furosemide (frusemide) will initiate a significant diuresis. By contrast, in a patient with advanced chronic renal failure, a dose of 250mg furosemide may be needed to have the same effect. Why is this? A : Absorption of furosemide from the gut is reduced. B : Renal blood flow is reduced. C : Tubular secretion of furosemide is reduced. D : The NaK2Cl co-transporter in the renal tubule is downregulated. E : Renal metabolism of furosemide is increased. Comment : Furosemide is secreted into the renal tubule by the same mechanism with which weak organic acids are excreted and works from within the tubular lumen by inhibiting the action of the NaK2Cl co-transporter in the medullary thick ascending limb. Organic acids accumulate in renal failure and compete for tubular secretion with furosemide. This competition can be overcome by using a larger dose of the drug. C : 41. Drug metabolism is often affected by renal function. Serum creatinine is often used as a measure of renal function, but creatinine concentration is affected by factors other than the creatinine clearance, the most commonly used clinical method of determining glomerular filtration rate (GFR). The Cockcroft and Gault formula makes allowance for some of these factors to provide an estimate of creatinine clearance from the serum creatinine concentration. Which factors, apart from serum creatinine concentration itself, are taken into account when calculating creatinine clearance (and therefore GFR) by the Cockcroft and Gault formula? A : Sex, age, body weight B : Sex, body weight, height C : Age, body weight, height D : Sex, age, body weight, height E : Sex, age, body mass index Comment : The formula for determining a male’s creatinine clearance is (1.1 x [150-age] x body weight in Kg) / (72 x serum creatinine concentration in micromol/l). The formula for determining a female’s creatinine clearance is (0.9 x [150-age] x body weight in Kg) / (72 x serum creatinine concentration in micromol/l). A : 42. A 35-year-old man presents with pneumonia requiring antibiotic treatment, but is noted to suffer from acute intermittent porphyia. Which of the following antibiotics would be the best treatment choice? A : Cephradine B : Amoxicillin C : Erythromycin D : Cefotaxime E : Flucloxacillin. Comment : Prescribing in patients with acute porphyrias can be difficult, since drug classes or individual drugs can precipitate acute attacks. The antibiotic classes of cephalosporins and sulphonamides precipitate acute attacks. Additionally, small changes in chemical structures of drugs can change the porphogenicity of drugs within a class (amoxicillin is safe, but flucloxacillin is not). Further information is available from the Welsh Medicines Information Centre in Cardiff. B : 43. A patient with chronic Hepatitis C is referred to your clinic for anitiviral treatment. However, interferon-alfa and ribavirin may not be suitable for those patients who: A : have moderate-severe disease B : were previously intravenous drug users C : are heavy alcohol drinkers D : are haemophiliacs and have not had a liver biopsy E : have relapsed following initial response to interferon-alfa monotherapy. Comment : Combination therapy with interferon-alfa and ribavirin is generally recommended for those with moderate-sever disease (histological diagnosis of significant scarring and/or significant necrotic inflammation). While NICE guidance suggests that problems with drug interactions, safety, and compliance may arise in existing intravenous drug users, those who have given up the habit should not be excluded from therapy. However, treatment is not generally recommended in those patients who consume large quantities of alcohol, given the increased risk of liver damage. In cases where a liver biopsy carries a high risk (e.g. haemophilia), treatment can be initiated without histological confirmation.Both treatment-naïve patients and those who have relapsed following initial response to interferon-alfa should be considered for 6 months of combination therapy. C :

Page 354: Masterclass Book Part 2

MOHAMMED IS-HAG 353

44. A patient presents with acute dystonia and oculogyric crisis after being treated with metoclopramide. Which statement is true with regards to this adverse drug reaction? A : It occurs only after long-term use of metoclopramide. B : It is most common in middle-aged men. C : It can persist for several days after withdrawal. D : It does not occur with prochlorperazine. E : It is best treated with procyclidine. Comment : Dystonic reactions are well-recognized with dopamine receptor antagonists. They occur shortly after starting therapy, particularly in girls and young women as well as the elderly. The problem usually subsides within 24 hours following cessation of treatment and can be treated with procyclidine 5-10 mg i.m. (antimuscarinic). E : 45. A 24-year-old Type I diabetic is currently on a basal-bolus regime, comprising twice a day basal Isophane insulin, complemented by short-acting insulin at meal times. He has recently heard about insulin glargine, and wondered if it would be suitable for him. Which statement concerning insulin glargine is true? A : It is formulated by adding zinc suspension to insulin B : It is rapid-acting and should be injected just before meals C : It is particularly useful for patients troubled by hypoglycaemic episodes D : It needs to be mixed thoroughly before injecting E : It has little effect on fasting blood glucose. Comment : Insulin glargine is a long-acting insulin analogue, produced by modifying the chemical structure of insulin. This gives it a smooth, prolonged absorption profile with no peaks. As such, it is a long-acting agent, suitable for providing a basal level of insulin which attempts to mimic the normal physiological state. Its smooth profile reduces the risk of hypoglycaemia, and when given at night, provides good control of the fasting blood glucose. Unlike crystalline suspensions, insulin glargine does not need to be mixed thoroughly prior to injection, thus making it easier to use. C : 46. A 28-year-old man with asthma presents with an acute attack. He is very breathless and cannot complete sentences. Which of the following is the best immediate management? A : Nebulised salbutamol (5 mg) driven with air B : Organise chest radiograph to exclude pneumothorax C : Nebulised salbutamol (5 mg) driven with high flow oxygen via reservoir bag D : Nebulised salbutamol (50 mg) driven with 35% oxygen E : Nebulised salbutamol (5 mg) driven with 35% oxygen. Comment : Beta2-agonists are pulmonary vasodilators as well as bronchodilators. Their administration can rapidly worsen the V/Q mismatch which is the cause of hypoxia in asthma. They can therefore cause reduction in arterial oxygen tension unless supplemental oxygen is given, and this man should be given the highest inspired oxygen concentration that can be obtained.It would also be reasonable to mix ipatropium bromide (500 microg) with the salbutamol given in option C. C : 47. A 40-year-old woman has developed haemolytic anaemia secondary to drug therapy. Which of the following drugs is NOT a well-recognised cause of haemolytic anaemia? A : Phenoxymethylpenicillin B : Mefenamic acid C : Methyldopa D : Ranitidine E : Rifampicin. Comment : Several drugs cause haemolytic anaemia, by a variety of mechanisms. Penicillin binds covalently to the red cell membranes; rifampicin causes immune complex association with red cell membranes leading to complement activation; methyldopa and mefenamic acid may induce the formation of autoantibodies against components of red cells. Ranitidine does not commonly cause haemolytic anaemia. D : 48. An 87-year-old woman is admitted after being found wandering in her nightie. She says she is looking for her cat. On examination, she smells strongly of alcohol, but is otherwise well. Her medications include paroxetine for depression, lorazepam for anxiety (she is not sure how many she is taking), cimetidine for longstanding peptic ulcer disease and thyroxine 50mcg daily. Which medication can be stopped immediately? A : Paroxetine B : Lorazepam C : Alcohol D : Cimetidine E : Thyroxine. Comment : This patient has an acute confusional state, and all causes of this should be excluded, especially infection.

Page 355: Masterclass Book Part 2

MOHAMMED IS-HAG 354

It is important to remember that many psychiatric drugs should not be stopped precipitously and these include selective serotonin reuptake inhibitors (SSRIs) such as paroxetine and benzodiazepines (such as lorazepam). If the dose of these medications needs to be changed, this must be done very slowly, otherwise they can produce an acute withdrawal state with worsening confusion and agitation. The same applies to alcohol. Her family can bring in her usual tipple, so that the nurses can monitor intake. Otherwise, prescribe chlordiazepoxide in reducing doses.It is important to check her thyroid-stimulating hormone (TSH) level to ensure she is on the correct dose of thyroxine, but this is unlikely to be the cause of the confusion. Cimetidine has been shown to cause confusion in older people.Does she still need ulcer healing treatment? If so, consider a proton pump inhibitor (PPI). D : 49. A 48-year-old woman with a renal transplant is established on ciclosporin, azathioprine and prednisolone to prevent transplant rejection, and enalapril and bendrofluazide for hypertension. After a 14-day course of ketoconazole for oesophageal candidiasis her creatinine is found to have increased from 100 mmol/L to 400 mmol/L.Her deterioration in renal function is most likely attributable to: A : hypertension poorly controlled on enalapril and bendrofluazide B : nephrotoxic effects of ketoconazole C : ciclosporin toxicity due to inhibition of ciclosporin metabolism by ketoconazole D : transplant rejection due to induction of ciclosporin metabolism by ketoconazole E : effect of enalapril on background of stenosis of artery supplying renal transplant. Comment : Ketoconazole inhibits liver enzymes that metabolise cyclosporin, increasing plasma cyclosporin concentration. Cyclosporin is nephrotoxic; ketoconazole may be hepatotoxic. A deterioration may also be seen in renal function if the patient has severe hypertension, which could be caused by cyclosporin and/or prednisolone therapy, or is started on enalapril on a background of renal artery stenosis. However in this case the close relationship between the timing of the course of ketoconazole and the increase in creatinine makes option C the most likely. C : 50. A 38-year-old woman with a previous history of essential hypertension presents in the sixth week of pregnancy requesting advice about drug therapy for hypertension in pregnancy. Which one of the following statements is true? A : Blood pressure usually rises during the 2nd trimester of pregnancy, hence additional drug

therapy may be required. B : Pre-eclampsia is no more common in women with pre-exisiting essential hypertension than

previously normotensive women. C : Methyldopa is contra-indicated during pregnancy. D : Beta blockers may restrict foetal growth if given during pregnancy. E : Anti-hypertensive therapy should always be avoided in early pregnancy. Comment : Blood pressure usually falls during the second trimester of pregnancy before rising again in the third trimester. Women with pre-existing hypertension have a slightly higher risk of developing pre-eclampsia than those who are normotensive.Methyldopa, labetalol and (increasingly) nifedipine are usually the antihypertensive treatments of choice in pregnancy. Methyldopa is often poorly tolerated due to side-effects. ACE inhibitors and angiotensin receptor antagonists are contra-indicated as they may cause renal agenesis and foetal loss. Beta blockers may restrict foetal growth but are sometimes used. In some cases anti-hypertensive therapy is necessary in early pregnancy, with the risks and benefits of any drug therapy carefully considered and discussed with the patient. In some cases antihypertensive therapy is necessary in early pregnancy, with the risks and benefits of any drug therapy carefully considered and discussed with the patient. D : 51. A 5-year-old girl presents having ingested some of her mother's ferrous sulphate tablets. Which of the following is true? A : Whole bowel irrigation is appropriate in all cases. B : Gastrointestinal toxicity occurs 24 hours post ingestion. C : Serial iron concentrations should be measured after desferrioxamine has been given. D : Activated charcoal should be given if presenting less than 1 hour after ingestion. E : Desferrioxamine can be given i.m. Comment : Iron tablets are radio-opaque on abdominal radiographs until they have dissolved. In patients where tablets are seen in the stomach and the small intestine then either gastric lavage or whole bowel irrigation should be considered respectively. Iron is not adsorbed by activated charcoal. Following ingestion toxicity can be divided into phases: · Phase 1 - Gastrointestinal toxicity within 30 minutes to hours · Phase 2 - Latent asymptomatic phase · Phase 3 - Lethargy, neurological toxicity, renal failure, pulmonary oedema, hepatic failure and DIC can occur after 12 to 48 hours · Phase 4 - Gastrointestinal strictures occur 2 to 5 weeks after ingestion.Indications for desferrioxamine include a serum iron concentration of greater than 90micromol/L or significant hypotension, metabolic acidosis and neurological features. Desferrioxamine should be given i.v. but in exceptional circumstances can be given i.m. Following administration, standard biochemical measures of iron concentration are not useful as these will measure both free and chelated iron. E :

Page 356: Masterclass Book Part 2

MOHAMMED IS-HAG 355

52. A 39-year-old lady who who has a past history of treated hypertension is in her 3rd trimester of pregnancy and requires on-going anti-hypertensive treatment. Which anti-hypertensive would you definitely not prescribe? A : Hydralazine B : Labetalol C : Lisinopril D : Methyl-dopa E : Nifedipine. Comment : Evidence underpinning the choice of anti-hypertensive therapy in pregnancy is inadequate to make firm recommendations. There are no reports of serious effects with methyl-dopa following long and extensive use. Calcium antagonists, labetalol and hydralazine are commonly used, particularly for resistant hypertension in the third trimester. However, angiotensin-converting enzyme (ACE)-inhibitors should be avoided because they may cause oligohydramnios, renal failure and intra-uterine death. C : 53. A 21-year-old university student complains of difficulty sleeping. She is in the middle of sitting her final exams and would like some medication for a few days to help her sleep. However, she is concerned about potential 'hang-over' effects and would prefer a drug which doesn't cause daytime drowsiness. Which agent would you prescribe? A : Diazepam B : Midazolam C : Promethazine D : Loprazolam E : Clomethiazole. Comment : Diazepam has a long half-life, principally because of its active metabolites. Midazolam is short-acting but is only used intravenously. Promethazine is an antihistamine with a 12-hour half-life and may cause daytime sedation. Clomethiazole is less safe in overdose, has dependence potential and is only licensed for sedation in the elderly. Loprazolam is short-acting (half-life 6–12 hours) and would be a reasonable choice. D : 54. A 38-year-old asthmatic woman presents with an acute attack. Her arterial blood gases breathing air are as follows: pH 7.36, pO2 9.8 kPa, pCO2 5.2 kPa. These mean: A : the attack is not severe B : she should be given supplemental oxygen, but is unlikely to need a high FiO2 to achieve

normoxia C : cardiorespiratory arrest could be imminent D : her respiratory effort may be failing because she is getting tired E : she could have had a pneumothorax. Comment : A normal or elevated pCO2 in an asthmatic indicates failing respiratory effort, and although this woman's oxygen saturation is not severely depressed she is in danger of decompensation and - aside from high flow oxygen, nebulised salbutamol and ipatropium, and steroids - it would be prudent to inform the ICU of her existence. The gases are not bad enough, however, to suggest that cardiorespiratory arrest is imminent.Pneumothorax must be excluded in any asthmatic, but the presence or absence of pneumothorax can never be inferred from arterial blood gas analysis. D : 55. A 79-year-old lady presents to A&E with confusion, headache and tinnitus. Her GP has recently started her on an analgesic and you are worried she may have taken too much. Which of the following would most likely explain her symptoms? A : Paracetamol B : Aspirin C : Voltarol D : Co-codamol E : Codeine phosphate. Comment : Aspirin in excess causes symptoms of nausea, vomiting, headache, confusion and tinnitus or hearing difficulties. Whilst the co-codamol and codeine phosphate could cause confusion, they would not cause the tinnitus. All analgesics taken for a prolonged period of time can lead to an analgesic-induced headache. B : 56. A 73-year-old gentleman with dementia attends clinic with his wife. She has heard about memantine and wonders if it would be suitable for her husband. Which of the following are true of memantine? A : It has no interaction with amantadine. B : It is licensed for patients with all types of dementia. C : It inhibits renal excretion of ranitidine. D : It enhances the effects of barbiturates. E : It is an acteylcholinesterase inhibitor. Comment : Memantine is the first licensed NMDA receptor antagonist for the management of moderate to severe Alzheimer's disease. There is some published evidence that memantine has small

Page 357: Masterclass Book Part 2

MOHAMMED IS-HAG 356

benefits in reducing deterioration in patients with Alzheimer's disease, but little evidence for use in other types of dementia. Several drug interactions are known: · NMDA antagonists “N-Methyl-D-aspartate” (e.g. ketamine, amantadine) can precipitate psychosis · Dopamine agonists - effects enhanced · Barbiturates and neuroleptics - effects reduced · Drugs excreted by cationic transporters in the kidney (e.g. ranitidine, quinine, nicotine) - excretion reduced leading to higher plasma concentrations C : 57. A 16-year-old girl has been referred from her GP. She states that she has taken thirty paracetamol tablets (500mg) over the last 12 hours. She is currently seeing the psychiatrists for outpatient management of anorexia. Which of the following is the most appropriate immediate management? A : Perform gastric lavage and check paracetamol levels four hours after the last tablet was taken B : Give activated charcoal and check paracetamol levels four hours after the last tablet was taken C : Give activated charcoal and check paracetamol levels immediately D : Give activated charcoal and start n-acetyl cysteine E : Wait for the results of paracetamol levels before instituting treatment. Comment : Paracetamol levels are very difficult to interpret if the patient has taken the tablets over a period of time. This girl is at high risk because of the alleged number she has taken and the coexistant anorexia. Gastric lavage is only ever indicated if the patient has taken a large number in the hour or so before presentation. Charcoal will help to prevent absorption. It would be safest to start n-actyl cysteine and check her clotting over the next 24 hours. D : 58. A middle-aged man is brought by ambulance to the Medical Admissions Unit. He was fitting when picked up and is still having a grand mal convulsion. The most appropriate treatment is: A : Lorazepam 2 mg intravenously B : Fosphenytoin 15 mg/kg body weight phenytoin equivalent, intravenously at a rate of 100-150 mg phenytoin equivalent /min C : Phenytoin 15 mg/kg body weight, intravenously at a rate of 50mg/min D : Diazepam 10 mg intravenously E : Phenobarbitone 10 mg/kg body weight, intravenously at a rate of 100 mg/min. Comment : All of these are recognized treatments for status epilepticus. First-line treatment should be with intravenous benzodiazepine, with lorazepam preferred to diazepam because of its longer duration of action. Fosphenytoin is the preferred second-line treatment (phenytoin if this is not available). Phenobarbitone is one of several agents that can be used as third-line treatment, but seek specialist advice if first and second-line treatments are ineffective. A : 59. A 27-year-old woman develops difficulty breathing and her lips and tongue swell about five minutes after starting to eat a curry. She is brought to the accident and emergency department by ambulance. She is cyanosed and wheezing. Aside from high flow oxygen via a reservoir bag, which of the following treatments would be your top priority? A : Hydrocortisone 200 mg intravenously B : Chlorpheniramine 10 mg intravenously C : Epinephrine (adrenaline) - 0.5 ml of 1/1000 solution intravenously D : Epinephrine (adrenaline) - 0.5 ml of 1/1000 solution intramuscularly E : Salbutamol 5 mg nebulized. The history clearly suggests anaphylaxis and treatment with intramuscular epinephrine (0.5 ml of 1/1000) is required.In extremis, epinephrine can be given intravenously, but at reduced dosage: make a 1/10,000 solution (by diluting 1 ml of 1/1000 to 10 ml with 0.9% saline) and give this at 1 ml/min (0.1 mg/min) until a response has been obtained (or a total of 0.5 mg - 5 ml - has been given). D : 60. A 23-year-old male has a diagnosis of methaemoglobinaemia. Which of the following statements is false? A : Pulse oximetry is a not a good guide to his oxygen saturation. B : Ascorbic acid reduces the methaemoglobinaemia. C : Oral activated charcoal should be given for dapsone poisoning. D : Methylene blue is required only if there are signs of toxicity. E : Methylene blue is a cause of methaemoglobinaemia. Comment : Methaemoglobinaemia occurs when haemoglobin is oxidised and is unable to carry oxygen. Drug classes which cause this condition include: · antibiotics (dapsone, sulphonamides and trimethoprim) · nitrites and nitrates · local anaesthetics (lignocaine and prilocaine). Pulse oximeters measure both oxyHb and metHb, therefore giving false reassurance in patients with high levels of metHb, whose measured oxygen saturation does not accurately represent ability to carry oxygen to the tissues. Treatment of patients with less than 30% metHb is removal from the exposure, oxygenation and ascorbic acid (directly reduces the metHb). Methylene blue is used in those with severe poisoning or MetHb levels greater than 30%; excessive doses of methylene blue can themselves cause ethaemoglobinaemia. Dapsone is adsorbed by activated charcoal. D :

Page 358: Masterclass Book Part 2

MOHAMMED IS-HAG 357

61. You see a 48-year-old Afro-Caribbean man in the outpatient clinic with uncomplicated essential hypertension. His blood pressure today is 154/102mmHg despite optimization of non-pharmacological therapy. Which one of the following drugs would you use as the first-line agent in this patient? A : Atenolol 50mg od B : Nifedipine 10mg tds C : Amlodipine 5mg od D : Ramipril 2.5mg od E : Enalapril 5mg bd. Comment : Non-pharmacological therapy should always be optimized prior to commencement of medication, whenever possible. Hypertension is particularly common in Afro-Caribbeans and associated with particularly higher risk of complications. Therefore effective long-term treatment, with a low threshold for multiple therapy where necessary, is particularly important. Studies indicate that drugs such as ACE (angiotensin-converting enzyme) inhibitors and Beta-receptor antagonists are less effective in Afro-Caribbeans. The reason appears to be related to the finding that the renin-angiotensin-aldosterone (RAA) system is commonly suppressed in the majority of Afro-Caribbeans. As such, drugs that suppress the RAA system are less likely to be effective. Calcium-channel blockers (CCBs) and diuretics appear to be more effective in this subgroup. However, diuretics may not be suitable in this case as they are commonly associated with impotence. Short-acting CCBs do not provide prolonged BP control, can cause reflex tachycardia and may be associated with higher mortality. Therefore, long-acting CCB should be the first-line drug of choice. Ideally, a once-daily agent with that provides a smooth 24-hour BP control (e.g. Nifedipine LA 30mg od or Amlodipine 5 mg od) to improve compliance would be preferable. C : 62. A 72-year-old man is admitted to the coronary care unit with an acute myocardial infarction. He suffers a cardiac arrest. Basic life support is being given as you arrive. The ECG monitor reveals ventricular fibrillation (VF). The first defibrillation attempt should be made at: A : 200 J B : 400 J C : 360 J D : 20 J E : 100 J. Comment : The European Resuscitation Council guidelines for adult Advanced Life Support suggest that - after Basic Life Support and a precordial thump (if appropriate) - VF/VT should be treated with up to three defibrillation shocks, the first two at 200 J and the third at 360 J. A : 63. A 48-year-old man of African origin presents with blood pressure 200/110mmHg. Urinalysis is negative and fundoscopy shows AV nipping. Which of the following treatment options is most appropriate? A : Oral atenolol 50mg od if hypertension confirmed over 1–2 weeks B : Oral enalapril 10mg bd if hypertension confirmed over 1–2 weeks C : Urgent admission for control of accelerated hypertension D : Oral nifedipine capsules 20mg tds if hypertension confirmed over 1–2 weeks E : Oral nifedipine LA 30mg od if hypertension confirmed over 1–2 weeks Comment : The diagnosis of accelerated hypertension requires the finding of fundal haemorrhages and exudates, with or without papilloedema, as manifestations of fibrinoid necrosis. This patient has very high blood pressure but no evidence of accelerated hypertension. BTS guidelines suggest that the finding of blood pressure 200–219/110–119mmHg should be confirmed with repeat measurements after 1–2 weeks, then treated if still elevated. Black patients have low renin hypertension, therefore ACE inhibitors and beta-blockers as single agents do not lower blood pressure in this group. Calcium channel blockers and diuretics are effective agents. Nifedipine should be used as a long-acting preparation, not a short acting one. E : 64. An 18-year-old woman is brought to the accident and emergency department by a friend, who says that she has been ill for 24 hours with 'flu-like symptoms and headache. She is unwell, drowsy and has a purpuric rash on her arms. Your immediate action is to: A : order a CT scan of her brain B : perform a lumbar puncture C : take blood cultures and await result D : give acyclovir 10 mg/kg intravenously E : give cefotaxime 2 g intravenously. Comment : The working diagnosis must be meningococcal meningitis and the woman must be given appropriate an appropriate antibiotic (e.g. cefotaxime 2 g) intravenously without delay.In the very old or immunocompromised it would be appropriate to add ampicillin 2 g six-hourly to cover Listeria, and acyclovir (10 mg/kg eight-hourly, with dose reduction in renal failure) should also be given if herpes simplex encephalitis is a possibility. E :

Page 359: Masterclass Book Part 2

MOHAMMED IS-HAG 358

65. A 50-year-old man with metastatic colorectal cancer complains of regurgitation of food and a feeling that it sticks retrosternally. Chest radiograph and upper gastrointestinal endoscopy are normal. Which antiemetic is most likely to be of benefit? A : Cyclizine B : Haloperidol C : Prochlorperazine D : Metoclopramide E : Ondanestron. Comment : It sounds as though there may be a neuro muscular cause for his symptoms. Another possibility is external compression, but the endoscopy did not show this. Metoclopramide is both an antidopaminergic and gastrokinetic agent. It may improve oesophageal motility. D : 66. A 29-year-old man with a history of epilepsy has been well controlled on carbamazepine and clonazepam for the last 5 years. He now wishes to consider withdrawing from or reducing his medication. Which of the following statements are correct? A : There is about a 60% chance of experiencing a relapse in the first year during withdrawing from

anti-epilepsy treatment. B : Both anti-epileptics can be safely withdrawn simultaneously. C : The dose of carbamazepine can be reduced safely by 10% every 2-4 weeks. D : He can be advised that he can continue driving during withdrawal from anti-epilepsy treatment as

long as she remains free from seizures. E : It is likely that he will subsequently require higher doses to regain control with the current therapy, if discontinuation fails. Comment : Studies suggest that about 25% of patients who have decided to stop their anti-epilepsy treatment relapse within a year of starting to taper down their medication. The likelihood of seizure is greatest during withdrawal and in the subsequent 6 months. The DVLA recommends that patients should not drive during this period. Doses of drugs such as carbamazepine, lamotrigine, phenytoin, sodium valproate and vigabatrin should be reduced by about 10% every 2-4 weeks. Barbiturates, benzodiazepines and ethosuximide should be tapered more slowly by reducing dosage by about 10% every 4-8 weeks. Only one drug should be withdrawn at one time, with a period of 1 month between completing withdrawal of one drug and beginning withdrawal of the next. There is no evidence to support the belief that patients become resistant to their original therapy following discontinuation. C : 67. A 43-year-old woman with pulmonary hypertension attends clinic and asks to be prescribed bosentan. Which of the following is true? A : It is licensed for use in all patients with pulmonary hypertension. B : It inhibits the effects of endothelin-2 (ET-2). C : It binds to both endothelin A (ET-A) and endothelin B (ET-B) receptors. D : It is excreted primarily unchanged in the urine. E : It is not effective in patients with scleroderma. Comment : Bosentan is an antagonist of endothelin-1 binding to ET-A and ET-B receptors. It is licensed for the treatment of 'pulmonary arterial hypertension (PAH) to improve exercise capacity and symptoms in patients with grade III functional status'. It has been shown to be effective in patients with primary PAH and in those with PAH secondary to scleroderma. It is excreted in bile following metabolism by the cytochrome P450 enzymes and this is a potential source of interaction with drugs metabolised by the same isoenzyme (e.g. glibenclamide, ritonavir, ketoconazole, ciclosporin and itraconazole). C : 68. A 72-year-old Caucasian woman in your outpatient clinic has uncomplicated essential hypertension. Her blood pressure is 162/102mmHg despite optimization of non-pharmacological therapy. Which one of the following treatments would you choose as the first-line agent for her? A : Atenolol 50 od B : Bendrofluazide 2.5mg od C : Bendrofluazide 5mg od D : Enalapril 5mg od E : Ramipril 2.5mg od. Comment : Hypertension is particularly common in those aged above 60 not least because of the steady rise in systolic blood pressure with age. These patients are at a high absolute risk of cardiovascular complications. Furthermore, anti-hypertensive treatment may also reduce incidence of heart failure and possibly dementia. Non-pharmacological therapy for hypertension should always be optimised prior to commencement of medication, whenever possible. Low-dose diuretics are accepted as the first-line treatment for hypertension in the elderly and appear to confer greater benefit than Beta-adrenergic receptor antagonists in this subgroup. 1. Treatment of isolated systolic hypertension in the elderly with the long-acting calcium channel blocker, nitrendipine, has been shown to reduce stroke and cardiovascular outcome. Therefore, calcium channel blockers may be suitable when diurectics are not tolerated, ineffective or contra-indicated.2 B :

Page 360: Masterclass Book Part 2

MOHAMMED IS-HAG 359

69. A 65-year-old gentleman attending the cardiology clinic complains of swelling and tenderness of his breasts. You diagnose probable gynaecomastia. Which of the following drugs is most likely to be the cause? A : Simvastatin B : Amiodarone C : Digoxin D : Aspirin E : Ramipril. Comment : Digoxin is the most likely cause of his gynaecomastia. This side effect is more common with longer-term use and may be unilateral or bilateral. Important differential diagnoses to consider include male breast cancer, liver disease, testicular tumours and hyperthyroidism. Other drugs that can cause gynaecomastia include oestrogens, spironolactone, cimetidine, verapamil and nifedipine. The gynaecomastia usually improves on stopping the drug or reducing the dose. C : 70. A 53-year-old woman presents with a digoxin overdose. Which of the following statements is false? A : Tachyarrhythmias do not occur. B : Peak effects can be delayed after ingestion by 6-12 hours. C : Digoxin can precipitate severe hyperkalaemia (>6.5mmol/l). D : DC cardioversion can precipitate intractable ventricular fibrillation (VF) or asystole. E : Activated charcoal reduces absorption. Comment : Digoxin is slowly absorbed, hence peak effects can be delayed by up to 12 hours after overdose. Oral activated charcoal may reduce absorption, although giving multiple doses is controversial. Nausea and vomiting occur early after poisoning; other features include confusion, headache and visual disturbances. Any brady or tachyarrhthymia can occur. Treatment is with correction of hyperkalaemia and atropine for bradyarrhythmias. Digoxin-specific antibodies are useful in i) those with hyperkalaemia resistant to treatment, ii) bradyarrhythmias with hypotension non-responsive to atropine and iii) tachyarrhythmias with hypotension. Anti-arrhythmic drugs and DC cardioversion should be avoided as they can precipitate intractable asystole or VF. A : 71. You have decided to start a syringe driver on a dying patient whose symptoms were previously well controlled on oxycodone SR (OxyContin) 80mg bd. What dose of diamorphine should you chose for your 24-hour syringe driver? A : 30mg B : 60mg C : 90mg D : 110mg E : 130mg. Comment : 160mg oxycodone over 24 hours is equivalent to 320mg morphine which is equivalent to approx 106mg diamorphine D : 72. A 28-year-old male presents following an overdose and anticholinergic syndrome is suspected. Which one of the following is true? A : Tricyclic antidepressants are not a cause B : Bradycardia is common C : Physostigmine is the treatment of choice D : Mydriasis occurs E : Urinary incontinence is common. Comment : Anticholinergic syndrome occurs following overdose with drugs that have prominent anticholinergic activity including tricyclic antidepressants, antihistamines and atropine. Features include dry, warm, flushed skin, urinary retention, tachycardia, mydriasis (dilated pupils) and agitation. Although physostigmine, a reversible inhibitor of acteylcholinesterase, is effective in treating symptoms, there is a significant risk of cardiac toxicity (bradycardia, AV conduction defects and asystole). Treatment therefore consists of withdrawal of the precipitating drug and supportive care. D : 73. A frail 58-year-old lady with advanced breast cancer is admitted with abdominal pain and constipation secondary to opioids. Which of the following is the laxative of choice? A : Sodium Picosulphate B : Fybogel C : Senna D : Codanthramer E : Lactulose. Comment : Bulk forming drugs such as fybogel have little to offer in opioid-induced constipation. Senna can cause abdominal cramps. Codanthramer can cause skin burns in faecal incontinence. This patient is frail and may not be able to clean herself well and is therefore at risk of this most unpleasant complication. Lactulose can cause bowel distension and increased abdominal cramps. A :

Page 361: Masterclass Book Part 2

MOHAMMED IS-HAG 360

74. In relation to the use of inliximab in inducing remission in inflammatory bowel disease, which of the following statements is correct? A : Infliximab is a monoclonal antibody that inhibits interleukin. B : Infliximab is licensed for severe or fistulating ulcerative colitis. C : Infliximab increases the risk of infection. D : Acute infusion-related reactions are very rare (<0.05) with infliximab. E : Development of antibodies to infliximab is almost universal (>80%). Comment : Infliximab is a monoclonal antibody that inhibits tumour necrosis factor. It is licensed for severe or fistulating Crohn's disease refractory to corticosteroid or immunosuppressant therapy, but not for ulcerative colitis. Acute infusion-related reactions are common (1-10%) but serious reactions such anaphylactic reactions are uncommon (0.1-1%). Antibodies to infliximab develop in about 24% of patients taking other immunosupressants and in about 37% of those not on such therapy. Infliximab is associated with an increased likelihood of infection and especially tuberculosis (0.1-1% of patients). Therefore, patients should be evaluated for active and inactive tuberculosis prior to treatment; and monitored closely for infections. C : 75. A 24-year-old man with chronic renal failure, for which he receives haemodialysis three times per week, decides to go on holiday. One week later, having missed two dialysis sessions, he presents feeling unwell and breathless. Examination reveals pulmonary oedema. His ECG shows no P waves, broad QRS complexes and peaked T waves. What is the first treatment you should give him? A : 10 U actrapid with 50 ml of 50% glucose intravenously B : 10 ml of 10% calcium gluconate intravenously C : 10 mg salbutamol by nebuliser D : 100 mg lignocaine intravenously E : Transfer to the dialysis unit for haemodialysis. Comment : All of the interventions listed, with the exception of intravenous lignocaine, are recognised treatments for hyperkalaemia, but this man is at clear risk of cardiac arrest and only calcium gluconate acts instantly to 'stabilise' the heart (although it does not alter the serum potassium level).ECG monitoring will show that the ECG assumes a more normal morphology after calcium gluconate is given. Appropriate treatment then would be to give insulin / dextrose or salbutamol whilst arranging urgent transfer to the dialysis unit. B : 76. It has been decided that a patient should be changed from MST 100mg bd to the equivalent transdermal fentanyl dose. Choose the correct dose from those shown below A : 125ug/hr B : 25ug/hr C : 100ug/hr D : 50ug/hr E : 75ug/hr. Comment : Fentanyl Conversion Table Oral 24 hour morphine dose Fentanyl ug/hr Less than 135 mg 25 50 75 100 125 150 175 200 225 250 275 1035-1124 300 D :

Page 362: Masterclass Book Part 2

MOHAMMED IS-HAG 361

1. Four new drugs are tested for radical cure of vivax malaria. All the new drugs are more expensive and have more severe side effects than primaquine, so you will only change if there is at least a 25% increase in cure rate. Which drug do you recommend having seen these results? (For each one primaquine is the standard treatment against which the new drug is compared.) A : Drug A. Relative Risk of relapse 1.63 (95% confidence interval 1.2-2.3) B : Drug B. Relative Risk of relapse 0.67 (95% confidence interval 0.35-1.2) C : Drug C. Relative Risk of relapse 0.87, p <0.0001 D : Drug D. Relative Risk of relapse 0.7, p 0.01 E : Stick to primaquine. Comment : Drug A is associated with a statistically significant increase in relapse. For drug B the 95% confidence intervals cross 1, so the result is not statistically significant. Drug C is more effective than primaquine, and this is highly statistically significant, but it is less than 25% better. Drug D is more than 25% better and this is statistically significant. D : 2. A placebo-controlled study randomised 10 000 patients undergoing surgery for hip fracture to 160 mg aspirin / day, started preoperatively and continued for 35 days. About 1.5% of the patients allocated aspirin had DVT or pulmonary embolism (PE), compared with about 2.5% allocated placebo. Which one of the following statements about this trial is true? A : Aspirin produced a 1.5% absolute risk reduction in DVT/PE. B : Aspirin produced a 40% absolute risk reduction in DVT/PE. C : Aspirin produced a 1% proportional risk reduction in DVT/PE. D : Aspirin produced a 40% proportional risk reduction in DVT/PE. E : The Number Needed to Treat (NNT) to prevent one DVT/PE is 100/1.5 = 67. Comment : In this study aspirin reduces the risk of DVT/PE from 2.5% to 1.5%: this is an absolute risk reduction of 1% and a proportional (or relative) risk reduction of 1/2.5 = 40%. The NNT to prevent one DVT/PE is 1/absolute risk reduction = 1/0.01 = 100. D : 3. A 95% confidence interval means: A : 95% of the data fall within the confidence interval. B : there is a 95% chance that two groups are different C : that p=0.05 D : there is a 95% chance that the true value falls within the confidence interval E : there is a 95% chance that the finding is clinically significant. Comment : The 95% confidence intervals (95% CI) around a value are the range within which there is a 95% chance that the true value lies. Similarly, the 95% CIs around a difference are the range in which there is a 95% chance that the true difference lies. If the means of two groups have overlapping 95% CIs, then the two groups are not statistically significantly different. If the 95% CI of the difference between two groups overlaps zero, then the difference between the two groups in not statistically significant. Statistical and clinical significance should not be confused. A very large study can generate very narrow 95% CIs (or very small p values) for very small differences, which may be of no clinical significance at all. By contrast, a small study may fail to show a statistically significant effect even if the effect is both large and clinically important. D : 4. To compare two groups of categorical data, e.g. dead / alive by drug / placebo, the correct test is: A : Student’s t-test B : Analysis of variance (ANOVA) C : Wilcoxon rank-sum D : chi-squared E : p value Comment : Chi-squared tests (and variants thereof) are widely used to compare percentages or proportions of categorical data. From the chi-squared statistic a p value is read off a statistical table to give the degree of significance. Traditionally a p value of less than 0.05, indicating a less than 5% probability that a result has arisen by chance, is taken (arbitrarily) as indicating that chance alone is not responsible for the difference between groups. Normally distributed data can be compared with a Student’s t-test (with correction for multiple comparisons when appropriate). Skewed continuous data can be compared with a Wilcoxon rank-sum test or a Mann-Whitney U-test. D :

Page 363: Masterclass Book Part 2

MOHAMMED IS-HAG 362

5. Concerning the statistical power of studies, which one of the following statements is FALSE? A : International journals do not publish studies that are underpowered. B : A power calculation must always be performed before conducting randomized clinical trials.

C : A type II error occurs if it is claimed two treatments are the same when the study is not large

enough to detect equivalence. D : A type I error is where the null hypothesis is falsely rejected. E : The smaller the difference you want to detect, the larger a study must be. Comment : It is only ethical to conduct a clinical trial if it is capable of detecting a meaningful difference between two treatments to guide future practice. If a trial is underpowered it cannot detect a statistically significant difference. It is therefore mandatory to do a proper power calculation before exposing patients to a clinical trial.The rather daunting formal definition of a type I error means that a study falsely (but not deliberately) appears to find a difference between two groups which has actually arisen by chance alone. The conventional cut-off of p <0.05 will arise by chance alone one time in twenty. As many thousands of studies are published every month, type I errors are not rare.A type II error is formally where the null hypothesis is falsely accepted. To claim two treatments are ‘equivalent’ requires huge numbers, and most studies are underpowered (too small) to reliably rule out a small difference between one treatment and another.Most published studies are small enough that type I and type II errors are a real possibility, and examples are published in good journals every week. A : 6. In the field of statistics, a factorial trial means that: A : two or more treatments are tested sequentially B : two or more treatments are tested simultaneously C : the patients in the trial are stratified into two or more groups D : each patient receives one active drug and one placebo treatment E : the trial takes more than one factor into account. Comment : Most trials evaluate just one treatment. This does not have to be so: factorial trials test two or more treatments simultaneously. A famous example of a factorial trial was the ISIS-2 study in which comparison was made between placebo and each of two drugs, streptokinase and aspirin. Patients with suspected myocardial infarction were randomised to receive IV streptokinase alone, aspirin alone, both active drugs, or double placebo. The trial showed that each of the drugs produced about a 25% reduction in mortality, also that their effects were additive. B : 7. The sensitivity of a test is defined as follows: A : The number of true negatives detected by the test divided by the number of all true negatives in

the population tested. B : The number of true positives detected by the test divided by the number of all true positives in

the population tested. C : The number of true positives detected by the test divided by the total number of true positives in

the population tested. D : The number of true negatives detected by the test divided by the total number of true negatives

in the population tested. E : The number of true negatives detected by the test divided by the total number of true positives

in the population tested. Comment : The sensitivity of a test is as defined by answer ‘B’. A test that is 95% sensitive will detect 95% of all cases in the population tested (and miss 5% of cases). Sensitivity and specificity (defined by answer ‘A’) are absolute properties of a test, but do not necessarily indicate whether or not it is useful in clinical practice. This depends at least as much on the likelihood of the patient having the condition for which they are being tested: if the pre-test probability is low, then most positive tests will be false positives, even if the test has, in absolute terms, a high specificity. B : 8. A manuscript is submitted to a medical journal regarding a randomised trial in which a new treatment for Clostridium difficile diarrhoea is compared with an established treatment. A reviewer states that they are concerned that there might be type 2 statistical error. What does this mean? A : That the method of statistical analysis used is inappropriate B : That the study has shown a difference between the treatments that is statistically significant but

which is unlikely to be clinically significant C : That the study claims to find a difference that does not really exist, ie. the result is a statistical

fluke D : That the data is skewed (not normally distributed) and analysis should have used non-parametric

rather than parametric statistical techniques E : That the study claims that there is no difference between the treatments, when in reality the trial

was just too small to detect a difference.

Page 364: Masterclass Book Part 2

MOHAMMED IS-HAG 363

The null hypothesis is always that there is no difference between groups under study.A type 1 error occurs when ‘the null hypothesis is falsely rejected’. In practice this means that the study claims to find a difference that does not really exist, i.e. the result is just a statistical fluke.A type 2 error occurs when ‘the null hypothesis is falsely accepted’. This means that it is claimed that there is no difference between two groups, when in reality the study is simply too small to detect a difference. This type of error can be avoided by making explicit power calculations before embarking on any study. This will answer the question ‘if I am studying an outcome that occurs in (say) 20% of a conventionally treated group and want to show a (say) halving in the rate of this outcome, then how many patients do I need to study?’ E : 9. For continuous data, which one of the following statements is true? A : A positive s test (skew test) result means that the data is not normally distributed. B : Chi2 is the best test for comparing skewed continuous data. C : The Wilcoxon rank-sum test is best used with normally distributed data. D : It is always wrong to use the mean to describe skewed data. E : The interquartile range is two standard deviations wide. Comment : Looking at a set of data plotted out on a graph is a good way of determining whether or not it is skewed. There is no such thing as a ‘skew test’.There are few absolutes in statistics, but one is that skewed data should never be described by the mean, as it will lead to misleading distortions.Chi2 is for categoriacal data.It is possible to use Wilcoxon rank-sum to test the difference between two sets of normally distributed data. However, the Student t-test is more powerful, and so should be used in preference where the data are normally distributed. The rank-sum test is reserved for skewed data, where the t-test cannot be used.There is no relationship between standard deviation and interquartile range. Interquartile range is a good way of sumarising skewed data where the standard deviation (based on the mean) is not appropriate. D : 10. A type 1 statistical error in a clinical trial means that: A : patients were not allocated into groups with an appropriate randomisation method B : the null hypothesis is falsely accepted C : the null hypothesis is falsely rejected D : the statistical analysis was incomplete or incorrect E : the statement of the hypothesis to be tested was incomplete or flawed. Comment : A type 1 error is formally defined as being where the null hypothesis (which is that there is no difference between the groups) was falsely rejected. In practice this means that the study claims to find a difference that does not really exist, i.e. the result is a statistical fluke.The conventional cut-off for significance is P=0.05, or a 1-in-20 chance. Hence if 20 trials were conducted, you would expect to get one that was ‘positive’ by chance alone.What is a type 2 error? If you don’t know: read the reference below. C : 11. The number needed to treat (NNT) is: A : 1 divided by absolute risk reduction B : 1 divided by relative risk reduction C : 1 divided by the p value D : 100 divided by relative risk reduction E : The same as the absolute risk reduction. The clinical significance of a reported reduction in absolute risk, relative risk or odds ratio is not always obvious. The concept of the number needed to treat (NNT) was devised to make this clearer.If 9.4% of patients given aspirin after myocardial infarction die, compared with 11.8% of those not given aspirin, then the absolute risk reduction produced by aspirin is 11.8 – 9.4 = 2.4%, the relative risk reduction when taking aspirin is 9.4/11.8 = 0.8 (80%), and the NNT is 1 divided by 0.024 = 42, meaning that 42 patients with myocardial infarction must be treated with aspirin to prevent one death. A : 12. If data are skewed, then they should be summarized in the form: A : mean and standard deviation B : median and range C : mean and range D : median and standard deviation E : mean and 95% confidence intervals. Comment : Skewed data should always be summarized using the median and range. Standard deviation is based on the mean, which is not appropriate for skewed data. How do you decide if data are skewed? Plot them out and look at them, or find the median and calculate the mean: if these are more than slightly different, then the data is skewed. B : 13. Regarding case-control studies, which one of the following statements is FALSE? A : They are good for investigating rare diseases. B : They may be uninterpretable if controls are selected poorly. C : They are good for identifying rare causes of disease. D : They can examine multiple risk-factors for a single disease. E : They compare exposures of interest in cases and controls.

Page 365: Masterclass Book Part 2

MOHAMMED IS-HAG 364

Comment : Case-control studies compare exposures of interest in cases and controls. Two of their great strengths is that they can be used with rare diseases (because cases are pre-selected), and can examine multiple risk-factors (exposures).They are not good at identifying rare exposures. If the question is whether or not a rare exposure causes a disease then the appropriate design is a cohort study, where one group with the particular exposure of interest is compared with a control group without that exposure.The greatest difficulty in designing case-control studies is selection of an appropriate control group, and poor control selection often makes otherwise well-conducted studies uninterpretable. C : 14. In a study of patients with myocardial infarction, the death rate of those given aspirin is 8%, compared with 10% in those not given aspirin. This means that: A : the relative risk of death after myocardial infarction is 1.25 in those given aspirin B : the relative risk reduction produced by aspirin is 2% C : the number needed to treat with aspirin to prevent one death is 8 D : the number needed to treat with aspirin to prevent one death is 10 E : the absolute risk reduction produced by aspirin is 2%. Comment : Absolute risk reduction (or increase) = (Risk in group 1) minus (Risk in group 2), which is 2% in this example.Relative risk is the difference of outcome in one group compared to another = (Risk in group 1) divided by (Risk in group 2). In this case aspirin reduced relative risk by 20%.The Number Needed to Treat = 1 divided by (Absolute Risk Reduction), which is 1/0.02 or 50 in this example. E : 15. The specificity of a test is defined as follows: A : The number of true negatives detected by the test divided by the number of all true negatives in

the population tested. B : The number of true positives detected by the test divided by the number of all true positives in

the population tested. C : The number of true positives detected by the test divided by the total number of true positives in

the population tested. D : The number of true negatives detected by the test divided by the total number of true negatives

in the population tested. E : The number of true negatives detected by the test divided by the total number of true positives

in the population tested. Comment : The specificity of a test is as defined by answer ‘A’. A 75% specificity means that 75% of all true negatives will test negative, or conversely that 25% of true negatives will test positive. Sensitivity (defined by answer ‘B’) and specificity are absolute properties of a test, but do not necessarily indicate whether or not it is useful in clinical practice. This depends at least as much on the likelihood of the patient having the condition for which they are being tested: if the pre-test probability is low, then most positive tests will be false positives, even if the test has, in absolute terms, a high specificity. A : 16. The statistical reviewer of a paper states that they are concerned that the findings are biased. In statistical terms, ‘bias’ means: A : There is a flaw in study design that leads to a built-in likelihood that the wrong result may be

obtained. B : There is a flaw in statistical analysis leading to a likelihood that the wrong result may be

obtained. C : There is reason to believe that the authors wanted to obtain the result that the study showed. D : Both study design and statistical analysis are flawed, leading to a likelihood that the wrong result

may be obtained. E : The study is not of sufficient statistical power to exclude the missing of a significant effect. Comment : Bias means a flaw in study design that leads to a built-in likelihood that the wrong result may be obtained. It cannot be controlled for at the analysis stage.It can be extremely difficult to design studies without potential bias, particularly when there are complex interactions between exposures under study. Techniques such as restriction and stratification are commonly used to reduce potential for bias (see references). A : 17. The following is a description of a test. “The new test has a sensitivity of 99% and a specificity of 83%. In those under 65 years old it has a positive predictive value of 48%”. A test with these characteristics would NOT be appropriate in which one of the following situations? A : Screening anonymously donated blood for HIV before transfusion. B : As a preliminary part of a medical assessment looking for heart disease in potential army

recruits. C : A screening test for head lice in children. D : Rapid screening for HIV in a same-day-result STI clinic. E : As an indication for sigmoidoscopy and barium enema for lower gastrointestinal malignancy in

patients with chronic diarrhoea. Comment : Interpreting the sensitivity and specificity of a test depends on what you are using it for.The poor specificity of this test means that it would be inappropriate to use it as reason for telling

Page 366: Masterclass Book Part 2

MOHAMMED IS-HAG 365

people they have HIV infection; in the at-risk population over 50% diagnosed positive will not have the disease. By contrast, the balance of risk is different in screening blood for HIV, where the risk of missing a positive case far outweighs the risk of discarding some blood units unnecessarily, and similarly in looking for bowel cancer.The test would also potentially be appropriate in screening for head lice where the disease is not serious but you do not want to miss cases and the treatment is simple and safe. Predictive value depends as much on prevalence of a condition as the sensitivity. The positive predictive value in young patients means most army recruits with a positive test would be false-positive, with more being false positive than true positive, but since very few are likely to have heart disease a preliminary screening test with a positive predictive value of 48% would be a reasonable test to use. D : 18. Concerning cohort studies, which one of the following statements is true? A : They can only be used to compare two groups with one another. B : They are particularly useful with rare outcomes. C : Cohort studies are retrospective. D : They are better than other study designs for measuring prevalence of a disease in a population. E : They are better than other study types for measuring the incidence of a disease in a population. Comment : With a cohort study you start with two (or more) groups with different exposures. This could be exposure to an occupational hazard, or to different drugs or different infections. You then follow them over time to see whether, and when, they develop an outcome (in medicine usually a disease, complication or death). Cohort studies are therefore very good for investigating the effects of rare exposures, as you set the exposure. If the outcome is rare it is unlikely enough cases will occur in the follow-up time to draw any conclusions, so they are not good for rare outcomes. Unlike other study designs cohort studies follow individuals over time, so are particularly good for measuring incidence of a disease. They are not the best study design for measuring prevalence of a disease in a population (cross-sectional studies are very well designed for this). E : 19. A committee is deciding whether a trial of a new treatment against an established treatment of a malignancy is ethical. Which one of the following would mean that the trial was ethically justifiable? A : The new treatment is thought to be better than the established treatment. B : The established treatment is not a very effective treatment of the malignancy. C : It is not certain which of the two treatments is best. D : The established treatment is effective, but has a very high incidence of significant side effects. E : The malignancy is very aggressive, with a median survival of 6 months with the established

treatment. Comment : Whether or not a clinical trial is ethical is governed by the ‘uncertainty principle’, the fundamental criterion being that both patient and doctor should be substantially uncertain about the appropriateness of each of the trial treatments for that particular patient. If there are strong preferences for one treatment or another (by either the patient or the doctor), then that patient is ineligible: but if both parties are substantially uncertain, then randomisation is appropriate. C : 20. A researcher is trying to design a study to find out the cause (or causes) of a rare disease, about which very little is known. What study design is most likely to be appropriate? A : Geographical B : Cross-sectional C : Cohort D : Intervention E : Case control. Comment : Geographical studies, also called ecological studies, are good at generating hypotheses, but not very helpful in testing them.Cross-sectional studies, also called prevalence studies, look at the number of cases of a disease at a particular point in time. They are not useful for investigating rare diseases or exposures.In cohort studies, one group with an exposure of interest is selected and compared over time with another cohort without that exposure. If the control group is well selected, then cohort studies are good for examining the effects of rare exposures, but they are not suited to investigating the cause(s) of a rare disease.An intervention study cannot be used to look for the cause(s) of a disease. E : 21. The practice of evidence-based medicine (EBM) begins with the formulation of an answerable question. Which one of the following elements is NOT required for a question to be answerable? A : A particular patient or problem. B : An intervention. C : A comparison. D : An outcome measure. E : At least one relevant randomised controlled trial.

Page 367: Masterclass Book Part 2

MOHAMMED IS-HAG 366

Comment : The first step of EBM involves converting the need for information into an answerable question. To be answerable a question must be focused and should include each of the following four elements: 1. A patient or a problem, defined by specific characteristics that are likely to influence the applicability of the evidence 2. An intervention: this might be a diagnostic test, a therapeutic intervention, or information concerning prognosis 3. A comparison: in questions about diagnosis this might be with a well-established test; for treatment, it might be with placebo or an alternative treatment 4. Outcome measures: it is vital to identify outcome measures that are clinically important, rather than those that are easily measured, eg. angina, re-infarction and death are more important than thallium scan measurements. EBM requires explicit use of the best available evidence and its applicability is not restricted to those areas where there is randomised controlled trial evidence. E : 22. Regarding crossover trial design, which one of the following statements is true? A : It can be used to compare treatments for an acute infection. B : It is a good method for comparing analgesics in arthritis. C : It cannot be double-blinded. D : It cannot be randomised. E : Tends to need more patients than are required with other trial designs to get adequate statistical

power. Comment : The principle of a crossover design is that a patient has one drug or treatment, then a washout period, and then another drug, and the effect is compared between the two in a single individual. For this reason it is a good study design for treatment of chronic conditions, but not appropriate for acute conditions. It is just as easy (or difficult) to randomize and double-blind as for other study designs. Because each person is acting as their own control, it is usually possible to use smaller numbers to get the same power. B : 23. In the field of statistics, which of the following statements regarding a forest plot is true? A : The area of the squares is proportional to the number of events in each study. B : The lengths of the horizontal lines emerging from the squares represent standard deviations. C : The lengths of the horizontal lines emerging from the squares represent two standard

deviations. D : The area of the squares is proportional to the magnitude of the treatment effect in each study. E : It is a method of combining data when individual studies are of insufficient power to show an

effect. Comment : Forest plots are most commonly used in meta-analyses as a concise and elegant way of presenting information from many individual trials, allowing a convenient visual comparison of the separate trial results together with a synthesis of the data. The horizontal lines emerging from the squares represent confidence intervals. Larger studies have narrower confidence intervals; hence the largest squares are typically associated with the smallest horizontal lines. A : 24. Regarding the description and comparison of two groups of data, which one of the following statements is true? A : Categorical data should be described as percentages and compared using a Student’s t-test. B : Normally distributed continuous data should be described as median and range and compared

using a Chi-squared test. C : Skewed continuous data should be described as median and range and compared using a

Wilcoxon rank-sum test. D : Normally distributed continuous data should be described as mean and standard deviation and

compared using a Chi-squared test. E : Skewed continuous data should be described as mean and standard deviation and compared

using a Student’s t-test. Comment : Categorical variables are not continuous, e.g. drug / placebo, dead / alive. They should be described as percentages or proportions and compared with a Chi-squared test. Normally distributed continuous data should be described as mean and standard deviation and compared with a Student’s t-test.Skewed continuous data should be described as median and range and compared using a test such as the Wilcoxon rank-sum test or the Mann-Whitney U-test. C :